Printable Version of Topic
Click here to view this topic in its original format
თბილისის ფორუმი > ჯანმრთელობა და მედიცინა > შეკითხვები ინტერნისტებს(თერაპევტებს)


Posted by: vano_t 11 Aug 2007, 12:43
პატივცემულო საზოგადოება!
კაი ხალხის ინიციატივით მინდა გავხსნა ეს თემა სადაც პასუხები იქნება გაცემული იუზერთა შეკითხვებზე შინაგანი მედიცინის სხვადსხვა სფეროში. ყველა სფეროს ჩამოთვლას ვერ მოვახერხებთ მარა შევეცდებით პასუხი გავცეთ კითხვებს შემდეგ სფეროებში: ნევროლოგია, ყელ-ყურ-ცხვირი, ოფთალმოლოგია, გასტროენტეროლოგია, კარდიოლოგია, პულმონოლოგია(ანუ ფილტვების დაავადებები), ინფექციური დაავადებები, ოფისის ტრამვატოლოგია, ენდოკრინოლოგია, კანის დაავადებები, ალრგიული და იმუნოლოგიური დაავადებები, ოფისის მცირე ქირურგიული დაავადებები, ნეფროლოგია, ონკოლოგია, ჰემატოლოგია, რევმატოლოგია და ცოტ-ცოტა ყველაფერი.
მოკლედ რომ ვთქვათ ინტერნისტს უხდება შეხება ყველა ამ დარგთან და ინტერნისტის ვალია, რომ პაციენტი, ასე ვთქვათ, დააკვალიანოს. რა თქმა უნდა ვერცერი ინტერნისიტი ვერ მოგცემთ ამომწურავ პასუხს(ისეთ პასუხს რაც სპეციალისტის კომპეტენციაა), თუმცა ყველა ინტერნისტს ამ დარგებში არსებულ დაავადებებზე რაღაც გზამკვლევი შეუძლია რომ მოგაწოდოთ. ასე რომ თუ დახმარება დაგჭირდეთ, თქვენთან ვართ.

Posted by: donvaso 11 Aug 2007, 13:26
QUOTE
ოფისის ტრამვატოლოგია, ოფისის მცირე ქირურგიული დაავადებები 

ეს პირადად ჩემთვის რაღაც ახალია.....
არა რა, ძალიან დიდ პროგრესს განიცდის მედიცინა ისევე როგორც მთელი მსოფლიო.....

Posted by: vano_t 11 Aug 2007, 14:31
QUOTE (donvaso @ 11 Aug 2007, 13:26 )
QUOTE
ოფისის ტრამვატოლოგია, ოფისის მცირე ქირურგიული დაავადებები 

ეს პირადად ჩემთვის რაღაც ახალია.....
არა რა, ძალიან დიდ პროგრესს განიცდის მედიცინა ისევე როგორც მთელი მსოფლიო.....

პირდაპირი თარგმანი გავაკეთე ინგლისურიდან და შეიძლება ვერ გამოვიდა კარგად. ისე რასაც ვგულისხმობ არის ეს. ინტერნისტს ოფისში შეუძლია ბევრი ტრამვატოლოგიური პათოლოგიების მკურნალობა (გიფსის დადების ჩათვლით). მაგალითად მეტარსალური ძვლების მოტეხილობა(თუ displaced არ არის მოტეხილობა. ქართულად შეიძლება ცთომილი იყოს, თუ როგორ?), სხივის მოტეხილობა, ფალანგების მოტეხილობა და ასე შემდეგ. ისეთი მოტეხილობების მკურნალობაზე მაქვს საუბარი როცა ტრამვატოლოგთან გაგზავნა აუცილებელი არ არის და ოფისში შეიძლება გიფსი დაადო, ან არტაშანი დაადო(მთლად გიფსი რომ არ არის, ნახევრად ღია როა ხო არტაშანი ქვია?) და კონსერვატიული მკურნალობა დაუნიშნო. მხრის წინა ამოვარდნილობას არ ჭირდება წესით ორთოპედი თუ ინტერნისტს გაკეთებული აქვს ერთხელ ეგ პროცედურა. ან ბავშვებში იდაყვის ამოვარდნილობა ოფისში შეიძლება გაკეთდეს.

რაც შეეხება მცირე ქირურგიულ პათოლოგიებს რაც ოფისში შეიძლება გაკეთდეს და ნამკურნალები იქნას არის: მცირე აბსცესების დრენირება, უცხო სხეულების ამოღება(კანიდან, ყურიდან, ცხვირიდან, თვალიდან), კანზე მცირე ზომი საეჭვო კვანძების თუ წარმონაქნების მოცილება, ჩაბრუნებული ფრჩხილის მოცილება, ცისტების დრენირება, სახსრების ასპირაცია და სტეროიდების ინექცია. ესენი ყველაფერი ინტერნისტის ოფისში შეიძლება გაკეთდეს.

ერთი საოცარი შემთხვევა ვნახე რამოდენიმე კვირის წინ. 6 წლის ბავშვი მოდის ჩივილებით რომ ყურში რაღაც ფრთხიალებსო. უფრო სწორად ბებიამ, როცა შვილიშვილს პარკში ასეირნებდა, დაინხახა რომ ბავშვს მარჯვენა ყურში პეპელა შუფრინდა smile.gif ხოდა საწყალ ბავშვს ტიმპანური მემბრანის წინ პეპელა უფორთხიალებდა და რომ გამოვიღე თუ გამოვიყვანე პეპელა, ეგ შობელძაღლი 2 საახალწლოდ მოსუქებული ბუზის ტოლი იყო.

Posted by: tms 11 Aug 2007, 14:34
vano_t
QUOTE
ინტერნისტს ოფისში შეუძლია ბევრი ტრამვატოლოგიური პათოლოგიების მკურნალობა (გიფსის დადების ჩათვლით). მაგალითად მეტარსალური ძვლების მოტეხილობა(თუ displaced არ არის მოტეხილობა. ქართულად შეიძლება ცთომილი იყოს, თუ როგორ?), სხივის მოტეხილობა, ფალანგების მოტეხილობა და ასე შემდეგ. ისეთი მოტეხილობების მკურნალობაზე მაქვს საუბარი როცა ტრამვატოლოგთან გაგზავნა აუცილებელი არ არის და ოფისში შეიძლება გიფსი დაადო, ან არტაშანი დაადო(მთლად გიფსი რომ არ არის, ნახევრად ღია როა ხო არტაშანი ქვია?) და კონსერვატიული მკურნალობა დაუნიშნო. მხრის წინა ამოვარდნილობას არ ჭირდება წესით ორთოპედი თუ ინტერნისტს გაკეთებული აქვს ერთხელ ეგ პროცედურა. ან ბავშვებში იდაყვის ამოვარდნილობა ოფისში შეიძლება გაკეთდეს.

ვის ოფისში, ექიმის ოფისში თუ პაციენტის ოფისში? smile.gif

Posted by: lexomd 11 Aug 2007, 14:36
QUOTE
თუ displaced არ არის მოტეხილობა. ქართულად შეიძლება ცთომილ

ცდომით ან ცდომის გარეშე
QUOTE
კანზე მცირე ზომი საეჭვო კვანძების თუ წარმონაქნების მოცილება

ამას რითი აცილებთ?
QUOTE
გამოვიღე თუ გამოვიყვანე პეპელა

ცოცხალ მწერებს მშვენივრად კლავს სპირტის ჩაწვეთება...
* * *
QUOTE
ოფისის ტრამვატოლოგია, ოფისის მცირე ქირურგიული დაავადებები

ამბულატორიული ქირურგია და ტრავმატოლოგია ალბათ...
QUOTE
(მთლად გიფსი რომ არ არის, ნახევრად ღია როა ხო არტაშანი ქვია?

QUOTE
გიფსი

თაბაშირის ნახვევი....

Posted by: donvaso 11 Aug 2007, 14:51
QUOTE
ერთი საოცარი შემთხვევა ვნახე რამოდენიმე კვირის წინ. 6 წლის ბავშვი მოდის ჩივილებით რომ ყურში რაღაც ფრთხიალებსო. უფრო სწორად ბებიამ, როცა შვილიშვილს პარკში ასეირნებდა, დაინხახა რომ ბავშვს მარჯვენა ყურში პეპელა შუფრინდა  ხოდა საწყალ ბავშვს ტიმპანური მემბრანის წინ პეპელა უფორთხიალებდა და რომ გამოვიღე თუ გამოვიყვანე პეპელა, ეგ შობელძაღლი 2 საახალწლოდ მოსუქებული ბუზის ტოლი იყო. 


არადა ამბობენ, პეპელა ერთი დღე ცოცხლობსო..... biggrin.gif biggrin.gif wink.gif

Posted by: vano_t 11 Aug 2007, 15:02
lexomd
პირველ რიგში დიდი მადლობა ინფორმაციისათვის. ტერმინების ცოდნა მჭირდება ქართულად და თუ რაიმე წყაროს მიმითითებ მთლად კაი იქნება.

QUOTE
QUOTE
კანზე მცირე ზომი საეჭვო კვანძების თუ წარმონაქნების მოცილება

ამას რითი აცილებთ?

ჩვეულებრივ სკალპელით, საღი ქსოვილის ფარგლებში ამოკვეთა უნდა წარმონაქმნს. მერე შეგიძლია დახურო ჭრილობა (დღესდღეობით რამოდენიმე საშუალება არსებობს დახურვის: გაკერვა, steristrip-ანუ თხელი ზონრებია რომლებიც ჭრილობის ორივე მხარეს შეგიძლია მიამაგრო ჭრილობის კიდეებს ერთმანეთს რომ დაუახლოვებ მერე, dermabond ან ნებისმიერი სხვა სახის გაწოვადი წებო რომლითაც კანის კიდეები შეგიძლია მიაწებო) ან დატოვო მეორადად შეხორცებისათვის, თუ დახურვა რაიმე მიზეზის გამოშეუძლებელია.

QUOTE
QUOTE
ოფისის ტრამვატოლოგია, ოფისის მცირე ქირურგიული დაავადებები

ამბულატორიული ქირურგია და ტრავმატოლოგია

ალბათ ეგ უფრო სწორია. მაგრამ არის ბევრი ამბულატორიული პროცედურა (საკმაოდ რთულებიც) რომლესაც ქირურგი ან ტრამვატოლოგი ოფისში გააკეთებს, თუმცა ინტერნისტებს არ შეუძლიათ. ოჯახის ექიმებს შეუძლიათ ბევრი პროცედურის გაკეთება. მე ვიცნობ ერთს რომელიც აკეთებს აპენდექტომიებს, თიაქრებს, ამპუტაციებს, საკეისროს და თითქმის ყოველგვარი სკოპიებს.

Posted by: lexomd 11 Aug 2007, 15:33
QUOTE
ტიმპანური მემბრანის

დაფის აპკი ქვია...

Posted by: anakatrin 11 Aug 2007, 21:03
vano_t
QUOTE
ნევროლოგია, ყელ-ყურ-ცხვირი, ოფთალმოლოგია, გასტროენტეროლოგია, კარდიოლოგია, პულმონოლოგია(ანუ ფილტვების დაავადებები), ინფექციური დაავადებები, ოფისის ტრამვატოლოგია, კანის დაავადებები, ალრგიული და იმუნოლოგიური დაავადებები, ოფისის მცირე ქირურგიული დაავადებები და ცოტ-ცოტა ყველაფერი

მთელს ფორუმზე ენდოკრინოლოგს ვერ მივაკვლიე biggrin.gif თუკი ინტერნისტის დაქვემდებარებაში რაცაა ყველაფერზე გასცემთ პასუხებს ენდოკრინოლოგიამ რა დააშავა? biggrin.gif ყველას ალბათ ის სფერო აინტერესებს რა პრობლემაც თვითონ აქვს,დაამატეთ რა ენდოკრინოლოგიაც თუა შესაძლებელი smile.gif

Posted by: Tami28 11 Aug 2007, 22:03
vano_t

საქართველოს ჭკვიანი თერაპევტები ძალიან აკლია. ან ისეთები ჰყავს რომ დიაგნოზს ვერ გისვავენ სანამ კრიტიკულ ზღვარზე არ მიხვალ. ვიცი არაკორექტულია სახელის და გვარების მითითება, თუმცა ჩემი პირადი გამოცდილება მაძლევს ამის უფლებას.

გირგი გვილია - თერაპევტი, რომელიც ზის ინტერკლინიკაში და ითვლება ჭკვიან სპეციალისტად. დედაჩემს აღმოჩნდა სიმსივნე, თუმცა სამი თვე მასთან სიარულის მიუხედავად დიაგნოზი ვერ დაუსვა. და საქმე იქამდე მივიდა რომ ექსკოპიაზე აღმოჩენილი მეტასტაზების შემდეგ მიხვდა.


Posted by: vano_t 12 Aug 2007, 04:56
QUOTE (anakatrin @ 11 Aug 2007, 21:03 )
vano_t
QUOTE
ნევროლოგია, ყელ-ყურ-ცხვირი, ოფთალმოლოგია, გასტროენტეროლოგია, კარდიოლოგია, პულმონოლოგია(ანუ ფილტვების დაავადებები), ინფექციური დაავადებები, ოფისის ტრამვატოლოგია, კანის დაავადებები, ალრგიული და იმუნოლოგიური დაავადებები, ოფისის მცირე ქირურგიული დაავადებები და ცოტ-ცოტა ყველაფერი

მთელს ფორუმზე ენდოკრინოლოგს ვერ მივაკვლიე biggrin.gif თუკი ინტერნისტის დაქვემდებარებაში რაცაა ყველაფერზე გასცემთ პასუხებს ენდოკრინოლოგიამ რა დააშავა? biggrin.gif ყველას ალბათ ის სფერო აინტერესებს რა პრობლემაც თვითონ აქვს,დაამატეთ რა ენდოკრინოლოგიაც თუა შესაძლებელი smile.gif

ენდოკრინოლოგიაც მიმატებულია smile.gif

Posted by: anakatrin 13 Aug 2007, 00:34
vano_t
QUOTE
ენდოკრინოლოგიაც მიმატებულია

ძალიან კარგი,მიხარია,იმდია დასმულ კითხვებზეც მივიღებთ პასუხებს smile.gif

Posted by: iFFii 13 Aug 2007, 12:15
რამდენიმე თვეა სურდო მაქ,
ანუ ქრონიკულია ხო? და კიდე ქრონიკული ტონზილიტი + სიცხე.(37-37,5)

სახლის პირობებში რისი გაკეთება შემიძლია?რამეთუ ექიმთან ვერ მივდივარ ჯერhelp.gif

Posted by: vano_t 13 Aug 2007, 12:59
QUOTE (iFFii @ 13 Aug 2007, 12:15 )
რამდენიმე თვეა სურდო მაქ,
ანუ ქრონიკულია ხო? და კიდე ქრონიკული ტონზილიტი + სიცხე.(37-37,5)

სახლის პირობებში რისი გაკეთება შემიძლია?რამეთუ ექიმთან ვერ მივდივარ ჯერhelp.gif

შენ ალბათ ამბობ რომ რამოდენიმე თვეა გამონადენი გაქვს ცხვირიდან. მასეთი ქრონიკული გამონადენი ქრონიკული რინიტია ან ქრონიკული სინუსიტი და არა ვირუსული ინფექცია. თავის მხრივ ქრონიკული რინიტი/სინუსიტი სხვადასხვა მიზეზების გამო შეიძლება იყოს: ცხვირის ძგიდის გამრუდება, ადენოიდები, ალერგიული, არასპეციფიური. ერთ რამეს მინიმუმ ვაკეთებ მასეთ სემტხვევაში: სინუსები კომპიტერულ ტომოგრაფიას. სამკურნალოდ, იმის მიხედვით თუ რა იწვევს პრობლემას, სხვადასხვა რამე გამოიყენება. ალერგიისათვის მაგალიტად ცხვირში შესასხმელი სტეროიდები წავა; ადენოიდებისათვის და გამრუდებული ძგიდიდსათვის ქირურგია და ასე შემდეგ.

37 და 37,5 სიცხე არ არის.

ტონზილიტს შენით ვერ დასვავ. ვიღაცამ უნდა ჩახედოს ყელში და დაინახოს გაწითლებულ/გადიდებულ და შესაძლოა ჩირქოვანი ტონზილები.

Posted by: iFFii 13 Aug 2007, 13:11
vano_t
QUOTE
37 და 37,5 სიცხე არ არის.

და ეს ნორმაა?რამდენიმე თვეა ასეთი T მაქ, ყოველთვის.
QUOTE
ტონზილიტს შენით ვერ დასვავ. ვიღაცამ უნდა ჩახედოს ყელში და დაინახოს გაწითლებულ/გადიდებულ და შესაძლოა ჩირქოვანი ტონზილები.

ჩამხედეს უკვეsmile.gif მაქვს.
QUOTE
სინუსები კომპიტერულ ტომოგრაფიას

სად შეიძლება გაკეთება? და რა ღირს?



Posted by: vano_t 14 Aug 2007, 00:22
iFFii
37 და 37,5 ნორმაალური სიცხეა ხშირ შემთხვევაში. და მასეთ სიცხეს არავინ იკვლევს.

QUOTE
ჩამხედეს უკვეsmile.gif  მაქვს.

რამდენჯერ ჩაგხედეს? ერთხელ ჩახედვით ეგ დიაგნოზი არ ისმება. მერე, ტონზილებია ჩირქოვანი თუ ქრონიკული ფარინგიტი გაქვს, რაც თავის მხრივ ხშირია თუ ქრონიკული პოსტანზალური წვეთა (postnasal drip) გაქვს. ეგ არის ცხვირის გამონადენის ყელში გადადენა ქრონიკული სინუსიტების და რინიტების დროს, რაც ბევრ სიმპტომს შეიძლება იწვევდეს ყელის ტკივილების ჩათვლით.

QUOTE
სად შეიძლება გაკეთება? და რა ღირს?

ყველგან სადაც კომპიტერული ტომოგრაფია გააჩნიათ.

Posted by: liluu 14 Aug 2007, 11:29
vano_t
QUOTE
ალერგიისათვის მაგალიტად ცხვირში შესასხმელი სტეროიდები წავა;


მაინტერესებს რომელ გაიდლაინში არის ნაჩვენები რომ ალერგიული რინიტის დროს სასტარტო მედიკამენტია "ცხვირში შესასხმელი სტეროიდი"


rolleyes.gif rolleyes.gif

Posted by: vano_t 14 Aug 2007, 21:26
liluu
QUOTE
QUOTE
ალერგიისათვის მაგალიტად ცხვირში შესასხმელი სტეროიდები წავა;


მაინტერესებს რომელ გაიდლაინში არის ნაჩვენები რომ ალერგიული რინიტის დროს სასტარტო მედიკამენტია "ცხვირში შესასხმელი სტეროიდი"


rolleyes.gif rolleyes.gif

1) არავითარი გაიდლაინი არ გჭირდება ავადმყოფის მკურნაოლობის დროს. გაიდლაინი მხოლოდ საშუალება გაძლევს მოქმედების ფარგლები დააწესო.

2) 1-დან გამომდინარე, ნებისმიერი წამლის(მოცემული დიაგნოზის ფარგლებში) დაწყება შეიძლება ამა-თუ-იმ პათოლოგიის დროს თუ ექიმს გარკვეული მოსაზრება გააჩნია პაციენტთან არსებულ გარემოებებთან დაკავშირებით და არცერთი გაიდლაიანი არ არის სრულყოფილი ინფორმაციის შემცველი ასეთი გარემოებების თაობაზე.

3) მე არ მითქვამს გაიდლაინის მიხედვით თქო.

4) შემიძლია, საკუთარ პრაქტიკაზე დაყრდნობით, ვთქვა რომ ცხვირში შესასხმელი სტეროიდები ყველაზე ეფექტური საშუალებაა ალერგიული რინიტის სამკურნალოდ.

5) American Family Physician-ს ვებსაიტიდან:
QUOTE
Antihistamines vs. Nasal Corticosteroids. The majority of studies favor the use of intranasal corticosteroids over sedating or nonsedating antihistamines for relief of symptoms of nasal allergy. These results are true for seasonal and perennial allergic rhinitis.


თუ უფრო დაწვრილებით გინდა ინფორმაცია ალერგიული და არაალერგიული რინიტის მკურნალობის თაობაზე, იხილე ქვემდებარე ვებსაიტი, რომელიც ამერიკის ოჯახის ექიმთა ასოციაციის ვებსაიტია:
http://www.aafp.org/afp/20021201/practice.html

Posted by: liluu 15 Aug 2007, 20:17
vano_t

რა თქმა უნდა ინტრანაზალური კორტიკოსტეროიდი გამოიყენება ალერგიული რინიტის დროს, მაგრამ რამდენადაც მე ვიცი და სადაც წამიკითხავს სასტარტო მედიკამენტი არის მეორე თაობის ანტიჰისტამინური პრეპარატები (ცეტირიზინი, ფექსოფენადენი, კლარიტინი)

Posted by: vano_t 16 Aug 2007, 00:14
liluu
QUOTE
რა თქმა უნდა ინტრანაზალური კორტიკოსტეროიდი გამოიყენება ალერგიული რინიტის დროს, მაგრამ რამდენადაც მე ვიცი და სადაც წამიკითხავს სასტარტო მედიკამენტი არის მეორე თაობის ანტიჰისტამინური პრეპარატები (ცეტირიზინი, ფექსოფენადენი, კლარიტინი)

ისევ და ისევ, კლინიკურმა სიტუაციამ უნდა გიკარნახოს უფრო თუ რა უნდა გამოიყენო ვიდრე გარკვეული ავტორის მოსაზრებამ.

აი მაგალითად, როცა ავადმყოფს მარტო ალერგიული რინიტის სიმპტომები აქვს, რატომ გამოიყენებ სისტემურ პრეპარატს (სისტემურობის გამო სისტემური გვერდითი ეფექტების შანსი იზრდება) ვიდრე ადგილობრივს? თანაც თუ იცი რო მცემულ ლოკალური პრეპარატი უფრო ეფექტურია სტატისტიკურად.

ან მოდის ავადმყოფი და გეუბნება რომ მისმა ახლობლებმა ცადეს რინოკორტი და ისე მოუხდათ რომ ავადმყოფი აღფრთოვანებულია რინოკორტის მოსინჯვით. რას დაუნიშნავ? რინოკორტს, რომელიც ადგილობრივი სტეროიდია, თუ ანტიჰიტამინს, რომელის დავუშვათ გაიდლაინით არის პირველადი წამალი?

რა თქმა უნდა, არის სიტუაციები სადაც ანტიჰისტამინური ტაბლეტების გამოყენება უფრო მიზანშეწონილად ჩაითველბა. მაგალითად თუ პაციენტს ალერგიული რინიტთან ერთად ალერგიული კონინქტივიტიც გააჩნია, პლუს ცოტა ალერგიული ბრონქოსპაზმი.

ისე აშშ-ს გაიდლაინის მიხედვით, ცხვირში შესასხმელი სტეროიდებს ენიჭება პირველადი მნიშვნელობა:
QUOTE
Nasal corticosteroids. Nasal corticosteroids are considered the most potent medications available for treating allergic rhinitis [A]. They control itching, sneezing, rhinorrhea, and stuffiness in most patients, but do not alleviate ocular symptoms. They have a relatively good safety profile, but long-term perennial use, as well as prolonged use in children, may be problematic.
Oral antihistamines. Oral antihistamines prevent and relieve itching, sneezing, and rhinorrhea, but tend to be less effective for nasal congestion [A]. If an initial trial with a first-generation (OTC) antihistamine is unsuccessful or poorly tolerated, a second-generation antihistamine may be substituted. Second generation antihistamines are less sedating, but are expensive.


ეს ციტატა ნაციონალური გაიდლაინების საიტიდან არის: www.guidelines.gov რომელიც არაკომერციული ვებსაიტია. თვითონ ალერგიულ რინიტზე რეკომენდაციები ქვემდებარე ვებსაიტზე შეიძლება იხილო:
http://www.guidelines.gov/summary/summary.aspx?doc_id=3373&nbr=002599&string=allergic+AND+rhinitis

Posted by: liluu 16 Aug 2007, 17:46
vano_t
QUOTE
აი მაგალითად, როცა ავადმყოფს მარტო ალერგიული რინიტის სიმპტომები აქვს, რატომ გამოიყენებ სისტემურ პრეპარატს (სისტემურობის გამო სისტემური გვერდითი ეფექტების შანსი იზრდება) ვიდრე ადგილობრივს? თანაც თუ იცი რო მცემულ ლოკალური პრეპარატი უფრო ეფექტურია სტატისტიკურად.


ხომ არსებობს ადგილობრივი ანტიჰისტამინური პრეპარატიც? მაგალითად აზელასტინი. მართალია რინოკორტი ადგილობრივია, მარგრამ მაინც სტეროიდია. და მსუბუქ შემთხვევებში როცა შესაძლოა ანტიჰისტამინურიც ეფექტური იყოს, რატომ დავნიშნოთ სტეროიდი თუნდაც ადგილობრივი?!

ხოლო ის საკითხი რომ ალერგიული რინიტების დროს და განსაკუთრებით ქრონიკული ფორმის შემთხვევაში კორტიკოსტეროიდი გამოიყენება, სადავო არ არის.

QUOTE
ისევ და ისევ, კლინიკურმა სიტუაციამ უნდა გიკარნახოს უფრო თუ რა უნდა გამოიყენო ვიდრე გარკვეული ავტორის მოსაზრებამ.



yes.gif yes.gif

Posted by: bato-bato 16 Aug 2007, 19:17
ცოტა ყური მტკივა, თან ხვალ ზღვაზე მივდივარ და იქ რო ამტკივდეს რა ჩავიწვეთო?
მგონი ანთება არ უნდა იყოს
ისე არ მტკივა
მაგ ყურზე რო ვწვები ან რო ჩამოვქაჩავ, მაშინ მტკივა და იქნებ დამეხმაროთ

Posted by: ტანკე 16 Aug 2007, 19:20
თავი მტკივა ბლინ,უკვე 2 კვირა იქნება ყოველ დღე მტკივა sad.gif კომპიუტეერული ტომოგრაფიის გაკეთება რა ღირს ჩვენთან არავინ იცით? ის მამშვიდებს,რომ დღის მანძილზე იკრეფს მოშს და დილის ტკივილები არ მაქვს მარტო,არც ღებინების შეგრძნება მაქვს.

Posted by: vano_t 16 Aug 2007, 20:37
QUOTE (bato-bato @ 16 Aug 2007, 19:17 )
ცოტა ყური მტკივა, თან ხვალ ზღვაზე მივდივარ და იქ რო ამტკივდეს რა ჩავიწვეთო?
მგონი ანთება არ უნდა იყოს
ისე არ მტკივა
მაგ ყურზე რო ვწვები ან რო ჩამოვქაჩავ, მაშინ მტკივა და იქნებ დამეხმაროთ

შეიძლება გარე ყურის ანთება გაქვს. ოტოსკოპით უნდა ჩახედოს ექიმმა და დიაგნოზიც დასმული იქნება. ჩაწვეთება შეიძლება ანტიბიოტიკის წვეთების (ან ანტიბიოტიკს+კორტიკოსტეროიდი კომბინაცია), მარა თუ მთლად სიფრთხილის ფარგლებში გინდა იმოქმედო, მაშინ ექიმს ჯობია ეჩვენო. მიზეზი ის არის რომ თუ დაფის აპკის(ყურის ბარაბანს რომ ეძახიან) პერფორაცია(ანუ ყურის ბარაბნი გახვრეტილია) იქნა ნანახი, მაშინ წვეთები არ არის რეკომენდირებული, რადგან მასე ინფექცია შეიძლება შუა ყუში შეიტანო. მაგ შემთხვევაში დასალევ ანტიბიოტიკებს უნიშნავენ.
* * *
QUOTE (ტანკე @ 16 Aug 2007, 19:20 )
თავი მტკივა ბლინ,უკვე 2 კვირა იქნება ყოველ დღე მტკივა sad.gif კომპიუტეერული ტომოგრაფიის გაკეთება რა ღირს ჩვენთან არავინ იცით? ის მამშვიდებს,რომ დღის მანძილზე იკრეფს მოშს და დილის ტკივილები არ მაქვს მარტო,არც ღებინების შეგრძნება მაქვს.

კარტერამ, რაც მთავარია, ორსულობა გამორიცხა smile.gif

ისე თუ სხვა რაიმე ნევროლოგიური სიმპტომები/ნიშნები არ გაქვს, მაშინ შეიძლება გადადო კომპიტერული ტომოგრაფია. მაგრამ თუ გაქვს ცვლილება მხედველობაში, სმენაში, წონასწორობის დარღვევა, პარესთეზია/დიზესთეზია, ცალმხრივი სისუსტე, ასიმეტრიული რეფლექსები(ერთი სიტყვით რაიმე რაც მიუთითებს თავის ტვინის დაზიანებაზე) მაშინ კი უნდა კომპიტერული ტომოგრაფია.

ყველაზე ხშირი მიზეზები თავის ტკივილის: ძილის დარღვევა, შიმშილი, სტრესი, მიგრენი, კლასტერი თავის ტკივილები და ტენზიური თავისტკივილები.

გარდა იმისა რომ არ ღებინება არის პროგნოზისათვის + ფაქტორი, სხვა დადებიტ ფაქტორებს მიეკუთვნება: თუ არ გაღვიძებს თავის ტკივილი, თუ იხსნება თავის ტკივილი აცეტამინოფენით, იბუპროფენით(და საერთოდ სუსტი ტკივილგამაყუჩებლებით)

Posted by: ტანკე 16 Aug 2007, 21:35
კარტერა
ვორნი +20%
--------------------------

Posted by: ტანკე 16 Aug 2007, 21:54
vano_t
QUOTE
ისე თუ სხვა რაიმე ნევროლოგიური სიმპტომები/ნიშნები არ გაქვს, მაშინ შეიძლება გადადო კომპიტერული ტომოგრაფია. მაგრამ თუ გაქვს ცვლილება მხედველობაში, სმენაში, წონასწორობის დარღვევა, პარესთეზია/დიზესთეზია, ცალმხრივი სისუსტე, ასიმეტრიული რეფლექსები(ერთი სიტყვით რაიმე რაც მიუთითებს თავის ტვინის დაზიანებაზე) მაშინ კი უნდა კომპიტერული ტომოგრაფია.

არცერთი ეგ ნისანი არ მაქვს,გარდა მხედველობის დაქვეითებისა.თვალების პრობლემა წლებია მაწუხებს და არ ვუკავშირებ ამას.თუმცა ბოლო დროს გამიღრმავდა პრობლემა.ნუ double vision არ მაქვს რაც მთავარია smile.gif
QUOTE
ყველაზე ხშირი მიზეზები თავის ტკივილის: ძილის დარღვევა, შიმშილი, სტრესი, მიგრენი, კლასტერი თავის ტკივილები და ტენზიური თავისტკივილები.

anxiety disorder ვფიქრობ user.gif


QUOTE
გარდა იმისა რომ არ ღებინება არის პროგნოზისათვის + ფაქტორი, სხვა დადებიტ ფაქტორებს მიეკუთვნება: თუ არ გაღვიძებს თავის ტკივილი, თუ იხსნება თავის ტკივილი აცეტამინოფენით, იბუპროფენით(და საერთოდ სუსტი ტკივილგამაყუჩებლებით)

არა არ მაღვიძებს თავის ტკივილი ძილის დროს,brain tumor sucks smile.gif

ისე ვერ ვიტყვი რომ ძაან მტკივა,ძაან ყრუ ტკივილები მაქვს,ხან კეფაში,ხან შუბლის არეში,ხან თვალბუდის მიდამოში,ხან საფეთქელ-ქვედა ყბის სახსრის მიდამოში,ხან თხემის არეში,ხან საფეთქლის არეში,კისერში...მოკლედ ყველგან.დენივით გამივლის,ხან ზეწოლის შეგრძნება მაქვს,ხან დაჭიმვის.5 წამი გრძელდება,მერე 0.5-1-2 წუთი აღარ არის,მერე ისევ და ა.შ. დილით რომ ვიღვიძებ არ მაქვს ტკივილი როგორც წესი,1 საათის მერე მეწყება და საღამოსკენ უფრო იმატებს.წამლებს არ ვსვამ.ხანდახან თვალებზე ზეწოლას ვგრძნობ,წქნევა არ მაქვს და გლაუკომას გამოვრიცხავ smile.gifმადლობა პასუხისთვის

Posted by: თეოგოგო 16 Aug 2007, 21:54
ტანკე
QUOTE
კომპიუტეერული ტომოგრაფიის გაკეთება რა ღირს ჩვენთან არავინ იცით? 



230 ლარი ღირს ვაკეში (მეცხრესთან).

ასე ღირდა, ყოველ შემთხვევაში...
დაზღვევა არ გაქვს? rolleyes.gif

Posted by: ტანკე 16 Aug 2007, 22:02
თეოგოგო
გასაგებია,მადლობა smile.gif დაზღვევა არ მაქვს

Posted by: vano_t 16 Aug 2007, 23:01
QUOTE (liluu @ 16 Aug 2007, 17:46 )
ხომ არსებობს ადგილობრივი ანტიჰისტამინური პრეპარატიც? მაგალითად აზელასტინი. მართალია რინოკორტი ადგილობრივია, მარგრამ მაინც სტეროიდია. და მსუბუქ შემთხვევებში როცა შესაძლოა ანტიჰისტამინურიც ეფექტური იყოს, რატომ დავნიშნოთ სტეროიდი თუნდაც ადგილობრივი?!

მე ის ავღნიშნე რომ სტატისტიკურად უფრო ეფექტურია და თანაც ადგილობრივ კორტიკოსტეროიდებს სისტემური გართულებები თითქმის არ გააჩნია თუ სწორად დაუნუშნავ ავადმყოფს.
და რა თქმა უნდა ადგილობრივი ანტიჰისტამინურებითაც შეიძლება მკურნალობის დაწყება. არავინ არ დავობს მაგას. მოკლედ ექიმის არჩევანზეა დამოკიდებული.

Posted by: bato-bato 16 Aug 2007, 23:36
vano_t
ექიმს ვაჩვენოოო?
აუ ხვალ ზღვაზე მივდივარ და sad.gif
ცოტა შემეშინდა ხო იცი sad.gif

Posted by: vano_t 16 Aug 2007, 23:42
QUOTE (bato-bato @ 16 Aug 2007, 23:36 )
vano_t
ექიმს ვაჩვენოოო?
აუ ხვალ ზღვაზე მივდივარ და sad.gif
ცოტა შემეშინდა ხო იცი sad.gif

საშიში არაფერია. გარეთა ყურის ინფექცია(და შუა ყურისაც) ადვილი სამკურნალოა და გართულებები იშვიათად აქვს ჯანმრთელ ადამიანში.

Posted by: lizi11 17 Aug 2007, 13:14
სად კეთდება პაპილომა ვირუსზე ანალიზი და შესამამისად, მკურნალობის დანიშვნა?

თუ არის შესაძლებელი, რომ პაპილომა ვირუსმა გამოიწვიოს თვალის მინისებრი სხეულის ანთება (უვეიტი) და თვალში პრეციპიტატების გაჩენა?
* * *
ენდოკრინოლოგთან თუ არის შესაძლებელი შეკითხვა?

Posted by: vano_t 18 Aug 2007, 08:26
lizi11
QUOTE
სად კეთდება პაპილომა ვირუსზე ანალიზი და შესამამისად, მკურნალობის დანიშვნა?

თუ არის შესაძლებელი, რომ პაპილომა ვირუსმა გამოიწვიოს თვალის  მინისებრი სხეულის ანთება (უვეიტი) და თვალში პრეციპიტატების გაჩენა?
* * *
ენდოკრინოლოგთან თუ არის შესაძლებელი შეკითხვა?

პაპილომა ვირუსის დიაგნოზი კლინკურად ისმება. გამოვლინება ძირითადად არის კანის და ლორწოვანის სიმსივნეები: მეჭეჭი და კონდილომები.

ყველაზე მნიშვნელოვანი ქალებისთვის ის არის რომ საშვილოსნოს ყელის კიბოს გამომწვევად ითვლება რამოდენიმე შტამი. მარა ყელის კიბოს რისკის გამოსავლენად ხმარობენ ე.წ. პაპინიკოლაუს ტესტს რომელიც გინეკოლოგის ოფისში, ან ნორმალურ ქვეყანაში, თერაპევტის ოფისში კეთდება. მარტივი ტესტია ეგ. საშვილოსნოს ყელიდან იღებენ ნაცხს და უფზავნიან გამოცდილ ციტოპათოლოგს, რომელსაც მაგ სიმსივნის რისკის დანახვა შეუძლია უჯრედებზე დაკვირვებით. სისხლის ტესტი არსებობს მაგ ვირუსის დნმ-ს დასადგენად მარა თითქმის არავითარი კლინკიური ღირებულება არ აქვს გარდა იმ შემთხვევისა როცა ციტოლოგიური გამოკვლევის რეზულტატი საორჭოფოა.

მკურნალობა კიდევ რისი გაინტერესებს? მეჭეჭების? კონდილომების? თუ საშვილოსნოს ყელის წინა-კიბოსმაგვარი მდგომარეობის სამკურნალოდ?

და რა ენდოკრინოლოგიური კითხვა გაქვს?

Posted by: lizi11 19 Aug 2007, 17:34
vano_t
პირველ რიგში, გმადლობ გამოხმაურებისთვის.

QUOTE
მკურნალობა კიდევ რისი გაინტერესებს? მეჭეჭების? კონდილომების? თუ საშვილოსნოს ყელის წინა-კიბოსმაგვარი მდგომარეობის სამკურნალოდ?

და რა ენდოკრინოლოგიური კითხვა გაქვს?


ე.ი. საშვილოსნოს ყელზე პაპილომების არსებობა ნიშნავს პაპილომა-ვირუსის არსებობას?

და სისხლის ანალიზი აღარ არის საჭირო?

მე მაინტერესებს პაპილომა ვირუსი თუ იწვევს ყელზე ჯირკვლების გადიდებას, რომელიც ხელით ისინჯება, ასევე, ჩიყვის სიმპტომებს. ჩიყვის ჰორმონებზე გაკეთებული მაქვს ანალიზი და ჩიყვი გამოირიცხა. ასევე თვალის დაავადებას.

საშვილოსნოს ყელის წინა-კიბოსმაგვარი მდგომარეობის სამკურნალოდ?
როცა გაკეთებულია კოლპოსკოპია და ტესტი და არ არის დაფიქსირებული კიბო, მაგრამ ყელზე არის პაპილომები და ის სიმპტომები, რომელიც ზემოთ აღვნიშნე, მხოლოდ პაპილომა ვირუსზეა სამკურნალო?

Posted by: vano_t 20 Aug 2007, 03:16
lizi11
QUOTE
მე მაინტერესებს პაპილომა ვირუსი თუ იწვევს ყელზე ჯირკვლების გადიდებას, რომელიც ხელით ისინჯება, ასევე, ჩიყვის სიმპტომებს. ჩიყვის ჰორმონებზე გაკეთებული მაქვს ანალიზი და ჩიყვი გამოირიცხა. ასევე თვალის დაავადებას.

ყელზე ლიმფური კვანძების გადიდებას ბევრი ვირუსი იწვევს, მაგრამ პაპილომა ვირუსით არ გამიგია მასეთი რამ(თუმცა შესაძლებელია). არც აქვს რაიმე მნიშვნელობა მაგას. ლიმფური კვანძები თუ დიდი ხანია გადიდებული, იმას ბიოფსია უნდა ისედაც. თუ მხოლოდ რამოდენიმე დღე ან კვირა გაქვს გადიდებული, მაშინ არაფერი გამლკვლევა არ ჭირდება მარტივი გასინჯვის და გამოკითხვის გარდა(მაგალითად კარიესები თუ გაქვს, ეგ ნორმალური მოვლენაა. ან ზემო სასუნთქი გზების ინფექციები, რასაც გაციებას ვეძახით, იწვევს მაგას. ვირუსული ფარინგიტები და ტონსილიტებიც იწვევენ მაგს. ეს ყველაფერი მარტივი გამოკვლევით-პირის ღრუს დათვალიერებით დგინდება). ჩიყვს ჰორმონების შემოწმებით ვერ დაადგენ. ჰორმონების შემოწმებით დაადგენ ფარისებური ჯირკვლის მომატებულ ან დაკლებულ ფუნქციას. ჩიყვი ნიშნავს გადიდებულ ფარისებურ ჯირკვალს და აქედან გამომდინარე მხოლოდ გასინჯვით ან ულტრაბგერით დგინდება. თვითონ ჩიყვი შეიძლება ფარისებური ჯირკვლის ფუნქციის მომატებით, დაკლებით ან ნორმულობით მიმდინარეობდეს. თუ ჩიყვი დადგინდა, შემდეგი ეტაპი(ჰორმონების გამოკვლევის შემდეგ)

QUOTE
ე.ი. საშვილოსნოს ყელზე პაპილომების არსებობა ნიშნავს პაპილომა-ვირუსის არსებობას?
არა, რადგანაც პაპილომას ხშირად ეძახიან პატარა წარმონაქმნებს რომელსაც სპეციფიური ფორმა გააჩნია. თუ ამ მხრივ ხმარობენ მაგ სიტყვას, მაშინ ის არ ნიშნავს პაპილომა ვირუსით გამოწვეულ ინფექციას.

QUOTE
და სისხლის ანალიზი აღარ არის საჭირო?
თუ მაინც და მაინც დიაგნოზის დასმა გინდა გამონაყარზე, ბიოფსიის გარეშე და მიკროსკოპში ჩახედვის გარეშე მაინ ვერ დასვამ მაგის დიაგნოზს, პაპილომის ვირუსზე ტესტი დადებითიც რომ იყოს.

QUOTE
საშვილოსნოს ყელის წინა-კიბოსმაგვარი მდგომარეობის სამკურნალოდ?
როცა გაკეთებულია კოლპოსკოპია და ტესტი და არ არის დაფიქსირებული კიბო, მაგრამ ყელზე არის პაპილომები და ის სიმპტომები, რომელიც ზემოთ აღვნიშნე, მხოლოდ პაპილომა ვირუსზეა სამკურნალო?
კითხვა მაქვს: ყელი შენ სხვადასხვა კონტექსტში იხმარე. ორიევეგან საშვილოსნოს ყელს გულისხმობ? ბოლო ციტატის კონტექსტში აშკარაა რომ საშვილოსნოს ყელზე ლაპარაკობ და პასუხსაც ამის მიხედვით გაგცემ. როცა კიბო გამორიცხულია, მაშინ მკურნალობის არჩევანი სხვა რამეებზეა დამოკიდებული, კერძოდ დიაგნოზზე და სიმპტომებზე. მაგალითად, თუ წარმონაქმნი მტკივნეულია, ან ხშირი სისხდენა ახასიათებს, ან ჩასახვას უშლის ხელს(და ბევრი რამ შეიძლება იყოს სიმპტომი თუ ნიშანი) მაშინ მკურნალობა ალბათ საჭიროა. თუ უსიმპტომოა და არაფერს უშლის ხელს(და როგორც ვთქვით კიბო გამორიცხულია-კიბოსთან ერთად წარმონაქმნის "გაკიბოიანების" რისკიც უნდა იყოს დაბალი) მაშინ შეიძლება უმკურნალო დატოვო.

რაც შეეხება კანის და ლორწოვანის წარმონაქმნების მკურნალობას რამოდენიმე ზოგადი მეთოდი არსებობს: მაღალი ტემპერატურით მოწვა, დაბალი ტემპერატურით მოყინვა, ლაზერით მკურნალობა, ქირურგიული მოცილება და ადგილობრივი კრემები(იმიქვიმოიდი და 5-FU მაგალითად)


Posted by: lizi11 20 Aug 2007, 13:54
vano_t
QUOTE
ჩიყვს ჰორმონების შემოწმებით ვერ დაადგენ. ჰორმონების შემოწმებით დაადგენ ფარისებური ჯირკვლის მომატებულ ან დაკლებულ ფუნქციას. ჩიყვი ნიშნავს გადიდებულ ფარისებურ ჯირკვალს და აქედან გამომდინარე მხოლოდ გასინჯვით ან ულტრაბგერით დგინდება. თვითონ ჩიყვი შეიძლება ფარისებური ჯირკვლის ფუნქციის მომატებით, დაკლებით ან ნორმულობით მიმდინარეობდეს. თუ ჩიყვი დადგინდა, შემდეგი ეტაპი(ჰორმონების გამოკვლევის შემდეგ)

მე გავიკეთე ჩიყვის ჰორმონებზე ჩაჩავაში ანალიზი და იქ მითხრეს, მთავარია ჰორმონები იყოს ნორმაში და როცა ის ნორმაშია, ჩიყვის არსებობა გამორიცხულიაო, ასევე მითხრა ენდოკრინოლოგმა ჟორდანიაში და ასევე მითხრეს არამიანცის საავადმყოფოში 7 წლის წინ - გავიკეთე ეხოსკოპია, ფარისებრი ჯირკვალი გადიდებულია მაგრამ ჩიყვი არ არის ჩამოყალიბებულიო. ამ დროს, ყოველთვის მაქვს შეგრძნება, თითქოს ყელში რაღაც მეჩხირება.
* * *
ასევე, ოფთალმოლოგმა მითხრა, თვალში უვეიტი და პრეციპიტატები შეიძლება ჩიყვისგან მოდიოდესო.
რადგან ჩიყვი გამოირიცხა,
გინეკოლოგმა - ალბათ, პაპილომა ვირუსმა გამოიწვია თვალის დაზიანებაო

Posted by: bassbomber 20 Aug 2007, 15:23
იქნებ ვინმემ იცოდეს რა დაავადებაა "ბორენიე'? ძალიან მჭირდება.წინასწარ გიხდით მადლობას.

Posted by: vano_t 20 Aug 2007, 19:40
lizi11
QUOTE
მე გავიკეთე ჩიყვის ჰორმონებზე ჩაჩავაში ანალიზი და იქ მითხრეს, მთავარია ჰორმონები იყოს ნორმაში და როცა ის ნორმაშია, ჩიყვის არსებობა გამორიცხულიაო, ასევე მითხრა ენდოკრინოლოგმა ჟორდანიაში და ასევე მითხრეს არამიანცის საავადმყოფოში 7 წლის წინ - გავიკეთე ეხოსკოპია, ფარისებრი ჯირკვალი გადიდებულია მაგრამ ჩიყვი არ არის ჩამოყალიბებულიო. ამ დროს, ყოველთვის მაქვს შეგრძნება, თითქოს ყელში რაღაც მეჩხირება.
არ არის მართალი. ჩიყვს თავისი განსაზღვრება აქვს როგორც გადიდებული ფარისებური ჯირკვალი და ჯირკვლის ფუნქციისაგან დამოუკიდებელია. კარგია რა თქმა უნდა როცა ჯირკლის ფინქცია ნორმალურია.

QUOTE
ასევე, ოფთალმოლოგმა მითხრა, თვალში უვეიტი და პრეციპიტატები შეიძლება ჩიყვისგან მოდიოდესო.
რადგან ჩიყვი გამოირიცხა,
გინეკოლოგმა - ალბათ, პაპილომა ვირუსმა გამოიწვია თვალის დაზიანებაო
რაც შეეხება უვეიტს, ყველაზე ხშირი მიზეზები არის ავტოიმუნური პროცესებით მიმდინარე დაავადებები და როცა უვეიტის დიაგნოზი ისმება რევამტოლოგმა(თუ გააჩნია მაგდენი ინფორმაცია) უნდა გამორიცხოს გარკვეული დაავადებები. ნაკლებ შემთხვევაში არაავტოიმუნური დაავედებებიც იწვევენ უვეიტს. პაპილომა ვირუსისაგან გამოწვეული უვეიტი არ გამიგია. ინტერნეტიც დავძებნე რაიმე იშვიათი შემთხვევა თუ ყოფილა, მაგრამ ვერ ვნახე. ზოგიერთ მიზეზზს ჩამოვთვლი უვეიტისას, მაგრამ ეს არ არის სრული სია: ბეჩეტის დაავადება, მაწყლულებელი კოლიტი, ფსორიაზი, მაანკილოზებელი სპონდილიტი და ასე შემდეგ. უვეიტის დიაგნოზის დასმა ნიშნავს მეორადი მიზეზების გამოკვლევას.

Posted by: lizi11 21 Aug 2007, 13:12
vano_t
QUOTE
რაც შეეხება უვეიტს, ყველაზე ხშირი მიზეზები არის ავტოიმუნური პროცესებით მიმდინარე დაავადებები და როცა უვეიტის დიაგნოზი ისმება რევამტოლოგმა(თუ გააჩნია მაგდენი ინფორმაცია) უნდა გამორიცხოს გარკვეული დაავადებები.

რევმატოლოგმაც გამორიცხა გარკვეული დაავადებები.

ე.ი. დარჩა დანარჩენი

QUOTE
ზოგიერთ მიზეზზს ჩამოვთვლი უვეიტისას, მაგრამ ეს არ არის სრული სია: ბეჩეტის დაავადება, მაწყლულებელი კოლიტი, ფსორიაზი, მაანკილოზებელი სპონდილიტი და ასე შემდეგ.


რა სიმპტომები აქვს მაწყლულებელ კოლიტს? თუ გამოიხატება ტკივილით?

რა არის ბეჩეტის დაავადება და მაანკილოზებელი სპონდილიტი?

QUOTE
ინტერნეტიც დავძებნე რაიმე იშვიათი შემთხვევა თუ ყოფილა

თითქმის ერთი წელია, სხვადასხვა ექიმთან დავდივარ და ამდენი ინფორმაცია ვერ მომაწოდეს, კიდევ ერთხელ გმადლობ. smile.gif
* * *
QUOTE
არ არის მართალი. ჩიყვს თავისი განსაზღვრება აქვს როგორც გადიდებული ფარისებური ჯირკვალი და ჯირკვლის ფუნქციისაგან დამოუკიდებელია. კარგია რა თქმა უნდა როცა ჯირკლის ფინქცია ნორმალურია


და როცა არის ასეთი შემთხვევა, ანუ - გადიდებილი ფარისებრი ჯირკვალი და ნორმალური ჯირკვლის ფუნქცია - მოქმედებს თუა არა უარყოფითად და არის თუ არა სამკურნალო?

Posted by: mtvareuli 21 Aug 2007, 19:27
vano_t
QUOTE
ნევროლოგია, ყელ-ყურ-ცხვირი, ოფთალმოლოგია, გასტროენტეროლოგია, კარდიოლოგია, პულმონოლოგია(ანუ ფილტვების დაავადებები), ინფექციური დაავადებები, ოფისის ტრამვატოლოგია,ენდოკრინოლოგია,კანის დაავადებები, ალრგიული და იმუნოლოგიური დაავადებები, ოფისის მცირე ქირურგიული დაავადებები და ცოტ-ცოტა ყველაფერი


ამ ცოტ-ცოტაში ეს სფეროები იგულისხმება?

ნეფროლოგია, ონკოლოგია, ჰემატოლოგია, რევმატოლოგია

თუ გამოჩნდება ამ ჩამოთვლილებზე კითხვები, პასუხებს გასცემთ?


Posted by: vano_t 21 Aug 2007, 19:40
QUOTE (mtvareuli @ 21 Aug 2007, 19:27 )
vano_t
QUOTE
ნევროლოგია, ყელ-ყურ-ცხვირი, ოფთალმოლოგია, გასტროენტეროლოგია, კარდიოლოგია, პულმონოლოგია(ანუ ფილტვების დაავადებები), ინფექციური დაავადებები, ოფისის ტრამვატოლოგია,ენდოკრინოლოგია,კანის დაავადებები, ალრგიული და იმუნოლოგიური დაავადებები, ოფისის მცირე ქირურგიული დაავადებები და ცოტ-ცოტა ყველაფერი


ამ ცოტ-ცოტაში ეს სფეროები იგულისხმება?

ნეფროლოგია, ონკოლოგია, ჰემატოლოგია, რევმატოლოგია

თუ გამოჩნდება ამ ჩამოთვლილებზე კითხვები, პასუხებს გასცემთ?

გავცემ ინტერნისტის კომპეტენციის ფარგლებში

Posted by: mtvareuli 21 Aug 2007, 21:18
vano_t
QUOTE
გავცემ ინტერნისტის კომპეტენციის ფარგლებში 


ოკ, ჩავამატებ თქვენის ნებართვით პირველ პოსტში


Posted by: BadbadGirl 22 Aug 2007, 20:37
ცალკე თემის გახსნა არ მიმაჩნია მიზანშეწონილად და თუ შეგიძლიათ აქ მიპასუხეთ: რა იცით პრეპარატ "პარკონი"-ს შესახებ?!

Posted by: vano_t 23 Aug 2007, 00:51
lizi11
QUOTE
რევმატოლოგმაც გამორიცხა გარკვეული დაავადებები.

ე.ი. დარჩა დანარჩენი

არც მასე ადვილია საქმე. მედიცინაში ყოველთვის არის ე.წ. პირველადი (იდიოპათრური) დაავადებები. ანუ დაავადებები რომლის გამომწვევი მიზეზებია არ იციან და თვლიან რომ ცალკე მექანიზმია აქვს ასეთ დაავდებებს. ასევეა უვეიტიც. არის უვეიტის ფორმები როცა მიზეზები უცნობია. მასეთ ფორმას როგორც ავღნიშნე პირველადი ან იდიოპათური შეიძლება ეწოდოს.

QUOTE
რა სიმპტომები აქვს მაწყლულებელ კოლიტს? თუ გამოიხატება ტკივილით
მუცლის თკვივილები, სისხლიანი განავალი, ჭინთვები, წონაში დაკლება, მადის დაქვეითება, გულისრევა, ღებინება-ნებისმიერი კომბინაციით(ანუ შეიძლება ერთი სიმპტომი აქედან, ან ყველა ერთად, ან სხვა კომბინაცია). ეს სიმპტომები თავის მხრივ მუცლის ღრუს ორგანოების ბევრ დაავადებას შეიძლება ახასიათებდეს. აქედა გამომდინარე ეს სიმპტომებია არასპეციფიური და საბოლოო დიაგნოზი დამოკიდებულია კოლონოსკოპიაზე და ბიოფსიაზე.

QUOTE
რა არის ბეჩეტის დაავადება და მაანკილოზებელი სპონდილიტი?
არვინ იცის რა არის ბეჩეტის დაავადება. ერთი ის იციან რომ წყლელები ახასიათებს პირის ღრუს და სასქესო ორგანოების. გარდა ამისა სხვა სიმპტომებიც (სისტემური სახის, ანუ რაც სხვადასხვა ორგანოთა სისტემებს მოიცავს) შეიძლება ახასიათბდეს: მაგალითად ართრიტი ან უვეიტი.
არც მაანკილოზებელი სპონდილიტი არ იციან რა არის(ანუ არ იციან რა იწვევს, როგორ ხდება დაავადება). ისე ეგ დაავადება ძირითადად აზიანებს ხერხემალს. როგორც ფიქრობენ, ავტოიუმუნური უნდა იყოს, ანუ იმუნური სისტემა ებრძვის საკუთარ ორგანიზმს იმის გამო რომ საკუთარი მოლეკულები უცხო გონია. ამ დაავადებასაც ახასითებს სისტემური სიმპტეომები, მათ შორის როგორც ვთქვით უვეიტი.

QUOTE
და როცა არის ასეთი შემთხვევა, ანუ - გადიდებილი ფარისებრი ჯირკვალი და ნორმალური ჯირკვლის ფუნქცია - მოქმედებს თუა არა უარყოფითად და არის თუ არა სამკურნალო?
გადიდებულ ფარისებურ ჯირკვალს თავისთავად არაფერი არ უნდა, თუ რა თქმა უნდა იმდენად გადიდდა რომ მეზობელ სტრუქტურებს აწვება და სიმპტომებს იწვევს(თეორიულად ვამბობ, პრაქტიკაში არ შემხვედრია). როცა ფარისებურის ფუნქცია ნორმალურია და ულტრაბგერა რაიმე კვანძს არ აჩვენებს, მაშინ არაფერს არ უნდა იწვევდეს ეგ.

ერთი რამ უნდა დაიმახსოვრო. მედიცინა (ისევე როგორც მეცნიერებათა უმრავლსობა) "დაახლოებითი" მეცნიერებაა და არასდროს ეცოდინები სრული პასუხი არაფერზე, შენ კითხვებზე პასუხების ჩათლით.
* * *
BadbadGirl
QUOTE
ცალკე თემის გახსნა არ მიმაჩნია მიზანშეწონილად და თუ შეგიძლიათ აქ მიპასუხეთ: რა იცით პრეპარატ "პარკონი"-ს შესახებ?!

პარკონი დავგუგლე როგორც parcon და მაგ წამალს ქვია გენერიული სახელი კარბიდოფა. თავისთავად კარბიდოფას არ ხმარობენ არაფრის სამკურნალოდ(ყოველ შემთხვევისათვის ასეა აშშ-ში დღესდღეობით). ისე მაგ წამალს ხმარობენ მეორე წამალთან, ლევოდოფა-სთან, ერთად. თვით ლევოდოფა იხმარება პარკინსონის დაავადების დროს. როგორც მეცნიერები ფიქრობენ პარკინსონიზმის დროს ტვინის გარკვეულ უბნებში არის დოფამინის(ე.წ. ნეიროტრანსმიტერია) ნაკლებობა. დოფამინს პირდაპირ ვერ დაუნიშნავ(ძალიან ხანმოკე ეფექტი აქვს რადგანაც სწრაფად იშლება ორგანიზმიში და მხოლოდ წვეთოვანის სახით შეიძლება იხმარო დაბალი წნევების დროს). ლევოდოფა კიდევ, თავის მხრივ დიდ ხანს რჩება ორგანზმიში, ტვინში შედის სისხლის მიმოქცევის საშუალებით და ტვინშივე გარდაიქმნება დოფამინად. კარბიდოფა ტვინში არ შედის, ამიტომ დოფამინის რაოდენობას ვერ გაზრდის ტვინში. მაგას ლევოდოფას იმიტომ უმატებენ, რომ ლევოდოფა სანამ ტვინში მოხდებოდეს მანამდე ორგანიზმიში გარდაქმნას განიცდის სხვადასხვა ფერმენტებით, და კარბიდოფა კი ამ პერიფერულ გარდაქმნას აბლოკირებს და შესაბამისად ზრდის ლევოდოფას ათვისებას.

Posted by: lizi11 23 Aug 2007, 12:57
vano_t
QUOTE
არც მაანკილოზებელი სპონდილიტი არ იციან რა არის(ანუ არ იციან რა იწვევს, როგორ ხდება დაავადება). ისე ეგ დაავადება ძირითადად აზიანებს ხერხემალს. როგორც ფიქრობენ, ავტოიუმუნური უნდა იყოს, ანუ იმუნური სისტემა ებრძვის საკუთარ ორგანიზმს იმის გამო რომ საკუთარი მოლეკულები უცხო გონია. ამ დაავადებასაც ახასითებს სისტემური სიმპტეომები, მათ შორის როგორც ვთქვით უვეიტი.


და რა სიმპტომებით გამოიხატება ხერხემლის დაზიანება?

Posted by: mtvareuli 23 Aug 2007, 15:33
lizi11
QUOTE
და რა სიმპტომებით გამოიხატება ხერხემლის დაზიანება?


ანუ მაანკილოზებელ სპონდილიტს რა სიმპტომები აქვს? smile.gif

დაავადების დასაწყისში თავს იჩენს არცთუ ძლიერი ტკივილი წელის არეში, რომელიც თანდათან ძლიერდება და ხერხემლის სხვა ნაწილებზეც ვრცელდება. ტკივილი მყარია, მედიკამენტების ზემოქმედება მას მხოლოდ მცირე ხნით ასუსტებს. დროთა განმავლობაში მას მოჰყვება ხერხემლის მოძრაობის შეზღუდვა, რომელსაც პაციენტი ვერც კი ამჩნევს და მხოლოდ გასინჯვისას ვლინდება. არის შემთხვევები, როცა ტკივილი უმნიშვნელოა, ან სრულიად არ შეინიშნება, დაავადების ერთადერთი გამოვლინება კი ხერხემლის მოძრაობის შეზღუდვაა. ჩვეულებრივ, ის ქვემოდან ზემოთ ვრცელდება და ყველაზე ბოლოს ხერხემლის კისრის ნაწილის მოძრაობა იზღუდება, თუმცა არ არის გამორიცხული ეს არე დაავადების პირველ წლებშივე დაზიანდეს.

გულმკერდის მალებისა და ნეკნების დამაკავშირებელ სახსართა დაზიანების გამო იზღუდება გულმკერდის მოძრაობა, რაც შეიძლება სუნთქვის დარღვევისა და ფილტვების ქრონიკული დაავადებების მიზეზად იქცეს.

ზოგიერთს, გარდა ხერხემლის დაზიანებისა, აღენიშნება მხრის, მენჯ-ბარძაყის, საფეთქელ-ქვედაყბის სახსრების ტკივილი და მოძრაობის შეზღუდვა, უფრო იშვიათად-ზემო და ქვემო კიდურთა სახსრების ტკივილი და შეშუპება, ტკივილი გულმკერდში. ეს მოვლენები მეტწილად ხანმოკლეა, იშვიათად-მყარი და მძიმე. სხვა დაავადებებისგან განსხვავებით, მაანკილოზებელი სპონდილიტის დროს, ართრიტს იშვიათად მოსდევს სახსრების დაშლა, მაგრამ მოძრაობის შეზღუდვა გარდაუვალია.

ზოგჯერ აღინიშნება სხვა ორგანოებისა და სისტემების დაზიანებაც. უმეტესად ვლინდება ცალი თვალის შეწითლება, რაც უვეიტის ან ირიდოციკლიტის გამოვლინებაა. არ არის გამორიცხული, აღმოცენდეს გულის პათოლოგია-აორტის ნაკლოვანება ან გულის რიტმის დარღვევა, თირკმელებისა და ფილტვების დაზიანება. დაავადების სერიოზული გართულებაა თირკმლის ამილოიდოზი, რაც საბოლოოდ, ორგანოს სიმპტომებს იწვევს.

ეს რაც შეეხებოდა მაანკილოზებელი სპონდილიტის ნიშნებს.... vano_t მაპატიე რომ შემოვიჭერი თქვენ საუბარში, მაგრამ ცოტა ხნის წინ წავიკითხე ამ დაავადების შესახებ, ხოდა... biggrin.gif

Posted by: vano_t 23 Aug 2007, 23:01
QUOTE (mtvareuli @ 23 Aug 2007, 15:33 )
lizi11
QUOTE
და რა სიმპტომებით გამოიხატება ხერხემლის დაზიანება?


ანუ მაანკილოზებელ სპონდილიტს რა სიმპტომები აქვს? smile.gif

დაავადების დასაწყისში თავს იჩენს არცთუ ძლიერი ტკივილი წელის არეში, რომელიც თანდათან ძლიერდება და ხერხემლის სხვა ნაწილებზეც ვრცელდება. ტკივილი მყარია, მედიკამენტების ზემოქმედება მას მხოლოდ მცირე ხნით ასუსტებს. დროთა განმავლობაში მას მოჰყვება ხერხემლის მოძრაობის შეზღუდვა, რომელსაც პაციენტი ვერც კი ამჩნევს და მხოლოდ გასინჯვისას ვლინდება. არის შემთხვევები, როცა ტკივილი უმნიშვნელოა, ან სრულიად არ შეინიშნება, დაავადების ერთადერთი გამოვლინება კი ხერხემლის მოძრაობის შეზღუდვაა. ჩვეულებრივ, ის ქვემოდან ზემოთ ვრცელდება და ყველაზე ბოლოს ხერხემლის კისრის ნაწილის მოძრაობა იზღუდება, თუმცა არ არის გამორიცხული ეს არე დაავადების პირველ წლებშივე დაზიანდეს.

გულმკერდის მალებისა და ნეკნების დამაკავშირებელ სახსართა დაზიანების გამო იზღუდება გულმკერდის მოძრაობა, რაც შეიძლება სუნთქვის დარღვევისა და ფილტვების ქრონიკული დაავადებების მიზეზად იქცეს.

ზოგიერთს, გარდა ხერხემლის დაზიანებისა, აღენიშნება მხრის, მენჯ-ბარძაყის, საფეთქელ-ქვედაყბის სახსრების ტკივილი და მოძრაობის შეზღუდვა, უფრო იშვიათად-ზემო და ქვემო კიდურთა სახსრების ტკივილი და შეშუპება, ტკივილი გულმკერდში. ეს მოვლენები მეტწილად ხანმოკლეა, იშვიათად-მყარი და მძიმე. სხვა დაავადებებისგან განსხვავებით, მაანკილოზებელი სპონდილიტის დროს, ართრიტს იშვიათად მოსდევს სახსრების დაშლა, მაგრამ მოძრაობის შეზღუდვა გარდაუვალია.

ზოგჯერ აღინიშნება სხვა ორგანოებისა და სისტემების დაზიანებაც. უმეტესად ვლინდება ცალი თვალის შეწითლება, რაც უვეიტის ან ირიდოციკლიტის გამოვლინებაა. არ არის გამორიცხული, აღმოცენდეს გულის პათოლოგია-აორტის ნაკლოვანება ან გულის რიტმის დარღვევა, თირკმელებისა და ფილტვების დაზიანება. დაავადების სერიოზული გართულებაა თირკმლის ამილოიდოზი, რაც საბოლოოდ, ორგანოს სიმპტომებს იწვევს.

ეს რაც შეეხებოდა მაანკილოზებელი სპონდილიტის ნიშნებს.... vano_t მაპატიე რომ შემოვიჭერი თქვენ საუბარში, მაგრამ ცოტა ხნის წინ წავიკითხე ამ დაავადების შესახებ, ხოდა... biggrin.gif

არაფერი საპატიო არ გაქვს wink.gif თემასაც მასე ქვია: შეკითხვა ინტერნისტებს და არა ინტერნისტს. თანაც მშვენივრად აღწერე დაავადება და მისი მიმდინარეობ პაციენტის გასაგებ ენაზე. რაც მეტი თერაპევტი მივეხმარებით ჩვენს ხალხს, ფორუმზე იქნება თუ ოფისში, მით უფრო კარგია.
* * *
ხვალიდან შვებულებაში ვარ 10 დღე და ვისვენებ ყველაფრისაგან wink.gif თუ დრო მქონდა კომპთან დაჯდომის, კარგი იქნება. რომ დავბრუნდები, ფორუმსაც დავუბრუნდები 11 დღეში. გისურვებთ ყველას ჯანმრთელ დღეებს. არ გაცივდეთ, გრიპი არ აიკიდოთ, ტრავმა არ მიიღოთ, არ მოწიოთ, არ დალიოთ, სექსი მხოლოდ ზომიერების ფარგლებში, დუდუკი დაბალ ხმაზე.

Posted by: lizi11 24 Aug 2007, 13:10
mtvareuli
გმადლობ პასუხისთვის

QUOTE
ანუ მაანკილოზებელ სპონდილიტს რა სიმპტომები აქვს?

დაავადების დასაწყისში თავს იჩენს არცთუ ძლიერი ტკივილი წელის არეში, რომელიც თანდათან ძლიერდება და ხერხემლის სხვა ნაწილებზეც ვრცელდება. ტკივილი მყარია, მედიკამენტების ზემოქმედება მას მხოლოდ მცირე ხნით ასუსტებს. დროთა განმავლობაში მას მოჰყვება ხერხემლის მოძრაობის შეზღუდვა, რომელსაც პაციენტი ვერც კი ამჩნევს და მხოლოდ გასინჯვისას ვლინდება. არის შემთხვევები, როცა ტკივილი უმნიშვნელოა, ან სრულიად არ შეინიშნება, დაავადების ერთადერთი გამოვლინება კი ხერხემლის მოძრაობის შეზღუდვაა. ჩვეულებრივ, ის ქვემოდან ზემოთ ვრცელდება და ყველაზე ბოლოს ხერხემლის კისრის ნაწილის მოძრაობა იზღუდება, თუმცა არ არის გამორიცხული ეს არე დაავადების პირველ წლებშივე დაზიანდეს.


...ისე, ხერხემლის ტკივილი მაწუხებს, თუ მიწევს დიდხანს ჯდომა. რევმატოლოგიური გამოკვლევა ჩავიტარე და არ დადასტურდა არანაირი ანთებითი პროცესი სახსრებში, ხერხემალში.

მაანკილოზებელი სპონდილიტის გამოკვლევა სად ხდება?

Posted by: mtvareuli 24 Aug 2007, 13:49
lizi11
QUOTE
მაანკილოზებელი სპონდილიტის გამოკვლევა სად ხდება?

სად ხდება არ ვიცი, მაგრამ გეტყვი როგორ ხდება smile.gif

ექიმის მიერ ჩვეულებრივი გამოკვლევის, ავადმყოფის ისტორიის შესწავლის და მისთანების მერე, დიაგნოზის დასადასტურებლად შეიძლება საჭირო გახდეს შემდეგი კვლევები:

რენტგენოლოგიური კვლევა-ის ექიმს საშუალებას აძლევს გამოავლინოს ძვლებსა და სახსრებში არსებული ცვლილებები, რომლებიც დაავადების ადრეულ სტადიაზე ჩვეულებრივი გასინჯვით არ ვლინდება, დროულ დიაგნოსტიკასკი დიდი მნიშვნელობა აქვს. ზოგ შემთხვევაში სახსრების ანთებითი თუ სხვა ცვლილებების გამოსავლენად შესაძლოა საჭიროდ მიიჩნიონ კომპიუტერული ტომოგრაფია ან მაგნიტურ-რეზონანსული კვლევა.

სისხლის გამოკვლევა-განსაზღვრავენ ედს-ს (ერითროციტების დალექვის სიჩქარეს) და C-რეაქტიული ცილის შემცველობას. მათი მატება ანთებითი პროცესის მაჩვენებელია. ტარდება სისხლის საერთო ანალიზიც, რაც დაავადებისას ქრონიკული ანთებითი პროცესის შედეგად აღმოცენებული ანემიის გამოკვლევაში ეხმარება.

vano_t
QUOTE
შვენივრად აღწერე დაავადება და მისი მიმდინარეობ პაციენტის გასაგებ ენაზე. რაც მეტი თერაპევტი მივეხმარებით ჩვენს ხალხს, ფორუმზე იქნება თუ ოფისში, მით უფრო კარგია.

არ ვარ მე ჯერ ექიმი, მესამე კურსზე ახლა გადავედი... smile.gif მაგრამ ამ დაავადებაზე ცოტა ხნის წინ წავიკითხე და თავს უფლება მივეცი აქაც დამეწერა smile.gif
QUOTE
ხვალიდან შვებულებაში ვარ 10 დღე და ვისვენებ ყველაფრისაგან

წარმატებები wink.gif
QUOTE
არ გაცივდეთ, გრიპი არ აიკიდოთ, ტრავმა არ მიიღოთ, არ მოწიოთ, არ დალიოთ, სექსი მხოლოდ ზომიერების ფარგლებში, დუდუკი დაბალ ხმაზე.

biggrin.gif

Posted by: lexomd 25 Aug 2007, 01:06
lizi11
QUOTE
...ისე, ხერხემლის ტკივილი მაწუხებს, თუ მიწევს დიდხანს ჯდომა

არა ჰგავს ეგ სპონდილიტს. სკოლიოზი ან კუნთების სისუსტე უფროა..........

Posted by: lizi11 26 Aug 2007, 14:09
lexomd
QUOTE
...ისე, ხერხემლის ტკივილი მაწუხებს, თუ მიწევს დიდხანს ჯდომა


არა ჰგავს ეგ სპონდილიტს. სკოლიოზი ან კუნთების სისუსტე უფროა......

გაიხარე
სპონდილიტი რომ იყოს, რევმატოლოგიური გამოკვლევისას გამოჩნდებოდა. ხომ?
* * *
ე,ი. მაინტერესებს რომ დავადგინო
რა იწვევს თვალში უვეიტს და, ამავდროულად, პრეციპიტატების გაჩენას
გამოირიცხა
სპონდილიტი, კიბო, ტუპერკულიოზი, სახსრები, ჰიპოფიზის ჯირკვალი, წყლული
თუ შეიზლება ეს მოდიოდეს ღვიძლიდან? ნაღვლის ბუშტიდან? გასტრიტიდან?
* * *
lexomd
QUOTE
yava damilevia!
manqanaSi cin vmjdarvar!
vakeshi vyofilvar!
im dges qalis suratic vnaxe!
magare uar

up.gif tongue.gif

Posted by: lexomd 26 Aug 2007, 22:04
lizi11
ეგ უვეიტი თქვენ შემთხვევაში რაში გამოიხატება?

Posted by: lizi11 27 Aug 2007, 13:32
lexomd
QUOTE
ეგ უვეიტი თქვენ შემთხვევაში რაში გამოიხატება?

'ოფთალმიჯში' ლაზერული გამოკვლევის შედეგად დამისვეს დიაგნოზი -თვალის მინისებრი სხეულის ანთება ანუ უვეიტი, და პრეციპიტატები. ამის შედეგად მხედველობა 60 პროცენტამდე შემცირდა. ეს იყო ორგანიზმში შინაგანი ცვლილებით გამოწვეული და არა თვალის სისუსტით.
ჩავიტარე მკურნალობა (10 გადასხმა ოფტადექსამეტაზონი+ფიზიოლიგიური ხსნარი).
მომცა შედეგი, მაგრამ არა 100-პროცენტიანი
ვერ დადგინდა გამომწვევი მიზეზი და მაინტერესებს

Posted by: ტანკე 4 Sep 2007, 22:32
მეოთხე გვერდზე იყოს ეს თემა sad.gif(((((((((((((((((((((((((((((((((
* * *
ავპინავ მაშინ ცოტა ხნით და თუ აქტიურობა არ იქნება,მერე ჩამოვპინავ ისევ
და სხვებსაც

Posted by: makomako 6 Sep 2007, 02:14
ნაღველის ტკივილის დროს რა არის სასარგებლო და პირიქით საკვებს ვგულისხმობ იქნებ მომწეროთ

Posted by: vano_t 7 Sep 2007, 00:42
QUOTE (makomako @ 6 Sep 2007, 02:14 )
ნაღველის ტკივილის დროს რა არის სასარგებლო და პირიქით საკვებს ვგულისხმობ იქნებ მომწეროთ

ისე ზოგადად ცხიმიან საჭმელს უნდა აარიდო თავი. მაგრამ საკვები ინდივიდუალურია ყველასათვის. ეგ თვითონ ავადმყოფმა უნდა განსაზღვროს რა წყენს და რა არა.
* * *
QUOTE (lizi11 @ 27 Aug 2007, 13:32 )
'ოფთალმიჯში' ლაზერული გამოკვლევის შედეგად დამისვეს დიაგნოზი -თვალის მინისებრი სხეულის ანთება ანუ უვეიტი, და პრეციპიტატები. ამის შედეგად მხედველობა 60 პროცენტამდე შემცირდა. ეს იყო ორგანიზმში შინაგანი ცვლილებით გამოწვეული და არა თვალის სისუსტით.
ჩავიტარე მკურნალობა (10 გადასხმა ოფტადექსამეტაზონი+ფიზიოლიგიური ხსნარი).
მომცა შედეგი, მაგრამ არა 100-პროცენტიანი
ვერ დადგინდა გამომწვევი მიზეზი და მაინტერესებს

თვალის მინისებური სხეულის ანთება არ არსებობს. უვეიტი არის თვალის სისხლძარღოვანი გარსის, ევეის, ანთება. თვითონ უვეა მოთავსებულია თვალის გარე გარსსა(კონიუნქტივის) და შიგა გარსს(რეტინას ანუ ბადურას) შორის. ეს სისხლძარღვოვანი გარსი შეიცავს ფერად გარსს(ანუ ირისს), ქოროიდულ შრეს და წამწამოვან სხეულს. ყველაზე ხშრირია ირისის ანთება რასაც ირიტს უწოდებენ.

როგორც ადრე ვთქვი ბევრ შემთხვევაში პირველადი მიზეზების დადგენა შუძლებელია.

Posted by: baby-bobo 9 Sep 2007, 14:38
ჰიპერკუაგულაცია ორსულობის დროს

რა გართულებები შეიძლება გამოიწვიოს ორსულობაში.... როგორ ვუშველოთ მას....

D-დიმერები 2500... სFMK-12.... პროთრომბინი 106.... ფიბრინოგენი 450....

10 დღეში ერთხელ ვიკეთებ პლაზმაფერეზს.... ვსვავ თრომბო ასს...... ვიკეთებდი ფრაქსიპარინს....

Posted by: vano_t 10 Sep 2007, 05:26
baby-bobo
QUOTE
10 დღეში ერთხელ ვიკეთებ პლაზმაფერეზს.... ვსვავ თრომბო ასს...... ვიკეთებდი ფრაქსიპარინს....
მოდი იმ მედიცინის ფარგლებში მოგცემ პასუხს რაც მე მასწავლეს. პირველ რიგში ძალიან იშვიათად მინახია დაავადებები სადაც ავადმყოფს პლაზმაფერეზეს უკეთებენ. ჰიპერკოაგულაციური მდგომარეობების დროს მე რაც ვიცი(რაც მინახია იმის საფუძველზე ვამბობ) პლაზმაფერეზს უკეთებენ მხოლოდ HUS-ის(ჰემოლიზურ ურემიული სინდრომი) TTP-ის(თრომბოზული თრპმბოციტოპენიური პურპურა) და DIVC-ის(დისემინირებული ინტრავასკულარული კოაგულაცია) დროს. ეგ სამივე სერიოზული მდგომარეობა. რა თქმა უნდა ორსულობის დროს შეიძლება ჩამოყალიბდეს ეს სამივე მდგომარეობა, მარა მე რაც ავადმყოფები მინახია ყველა ინტენსიურ განყოფილებაში (ICU-ში) იწვნენ. ძალიან საეჭვოა რომ რომელიმე აქედან გქონდეს. არსებობს DIVC-ის ქრონიკული "ნელი" ფორმებიც მაგრამ ისინიც რაღაც სერიოზულ დაავადებებს ახლავს ხოლმე თან.

QUOTE
D-დიმერები 2500... სFMK-12.... პროთრომბინი 106.... ფიბრინოგენი 450....
ამ ტესტების გაკეთება იძლევა მეორე ეჭვს რომ არცერთი ზემოაღნიშნული დაავადებები არ უნდა გაგაჩნდეს. ერთადერთი ტესტი რომელსაც კლინიკური ღირებულება გააჩნია მაგ ტესტებიდან აშშ-ში არის D-DIMER. და ისიც მხოლოდ იშვიათად იყენებენ და მხოლოდ ერთი ჩვენება გააჩნია რაც მე ვიცი. ამ ტესტს აკეთებენ მხოლოდ ღრმა ვენების თრომბოზის გამოსარიცხად(და არა დიაგნოზის დასასმელად). D-DIMER წარმოიშობა თრომბის დაშლის დროს და თრომბოზული მდგომარეობების დროს თითქმის ყოველთვის მაღლა ადის. ამიტომ როცა ტესტს აკეთებ და უარყოფითია, ღრმა ვენების თრომბოზი თითქმის შეიძლება გამორიცხო(ასეთ ტესტებს ეწოდებათ სენსიტიური ტესტები). თუმცა დადებითი ტესტი სულაც არ ნიშნავს თრომბოზულ მდგომარეობას და ძალიან ბევრი მწვავე დაავადებების დროს შეიძლება აიწიოს მაღლა. უფრო მეტიც, დადებითი ტესტის უმრავლესობა არათრომბოზული დაავადებების შედეგია. სFMK-12 ან რაიმე მსგავსი საერთოდ არ გამიგია სიმართლე რომ გითხრა. ამიტომ არ ვიცი რა ტესტია და რას იკვლევს. ფიბრინოგენს და პროთრომბინს არავინ იკვლევს. იკვლევენ PT-ს(პროთრომბინის დროს) და aPTT-ს(აქტივირებული ნაწილობრივი თრომბოპლასტინის დროს). და საერთოდ, თრომბოზური მდგომარეობების სადიაგნოსტიკოდ ეგ ტესტები უნდა გაკეთდეს თრომბოციტების რაოდენობის დათვლასთან ერთად და თრომბოციტების და ერითროციტების სტრუქტურის გამოკვლევასთან ერთად, რადგანაც ზემოაღნიშნული დაავადებების დროს ხშირად ხდება ერითროციტების დამახინჯება რაც მიკროსკოპში შეიძლება გამოჩნდეს. კიდევ დამათებით ტესტებს აკეთებენ. მაგაზე სხვა დროს.

QUOTE
ჰიპერკუაგულაცია ორსულობის დროს

რა გართულებები შეიძლება გამოიწვიოს ორსულობაში.... როგორ ვუშველოთ მას....
მე ვფიქრობ შენს შემთხვევაში ლაპარაკი შეიძლება იყოს ღრმა ვენების თრომბოზზე, რაც მომატებული სიხშირით არის ორსულებში. ამ დიაგნოზს აკეთებენ ქვემო კიდურების ვენების ულტრაბგერითი გამოკვლევით და მკურნალობა კიდევ მოითხოვს ჰეპარინის ან დაბალმოლეკულური წონის ჰეპარინის კანქვეშ ინექციებს. ამ დაავადების გართულება არის ე.წ. ფილტვის არტერიის ემბოლიზმი. ამ დროს ქვემო კიდურის ვენებში არსებული სისხლის კოლტი შეიძლება მოწყდეს ადგილიდან და სისხლის მიმოქცევის საშუალებით ფილტვების არტერიები დააცოს, რაც შეიძლება ძალიან არასახარბიელო რამ იყოს. შენს შემთხვევაში, როგორც ჩანს, ჰეპარინის რაღაც ფორმას(ფრაქსიპარინი ისე ჟღერს რომ ჰეპარინი ან დაბალმოლეკულური ჰეპარინი უნდა იყოს) იკთებ. ამიტომ დაცული ხარ მაგ გართუებისაგან. ერთი კითხვა: მოცემულ მომენტში ორსული ხარ თუ არა? თუ ორსული არ ხარ მაშინ ე.წ. კუმადინის ანუ ვორფარინის ტაბლეტები შეიძლება სვა. ეგ გაცილებით იაფი ჯდება ნემსებზე. თანაც თუ ღრმა ვენების თრომბოზი გაქვს, მაგას მინიმუმ 6 თვე უნდა მკურნალობა.

ზემოთ აღნიშნული დაავადებების (HUS, TTP, DIVC) და ღრმა ვენების თრომბოზის გართულებებიც და მკურანლობის პრინციპებიც იმდენად განსხვავებულია რომ სწორი დიაგნოზის დასმა აბსოლიტური მოთხოვნილებაა. ამიტომ თუ ზუსტად მეტყვი რა დიაგნოზი დაგისვეს და რის საფუძველზე, უფრო კარგად დაგეხმარები. მე თუ არ მეცოდინება პასუხი, ჰემატოლოგს შევეკითხები თუ საჭიროა.

Posted by: Tornike Alashvili 21 Sep 2007, 04:03
ერთი კითხვა მაქვს ინტერნისტ ასისტენტებთან.

სრულიად ვეთანხმები იმ აზრს რომ მომავალმა ექიმმა ყველა სამედიცინო სპეციალობა უნდა ისწავლოს და შემდეგ როგორც ხდება კარგად დაივიწყოს გარდა იმისა რომელსაც ამოირჩევს.მაგრამ როგორ უნდა ამოირჩიოს პაციენტმა ექიმ-სპეციალისტებიდან რომელს მიმართოს და თან არსებობს თუ არა სახემძღვანელოები,თუ როგორ ამასთანავე პაციენტმა დაარიგოს თავისი ჩივილები ექიმის კაბინეტების მიხედვით?
მაპატიეთ დამაღამდა

user posted image

Posted by: vano_t 22 Sep 2007, 10:39
QUOTE (Tornike Alashvili @ 21 Sep 2007, 04:03 )
ერთი კითხვა მაქვს ინტერნისტ ასისტენტებთან.

სრულიად ვეთანხმები იმ აზრს რომ მომავალმა ექიმმა ყველა სამედიცინო სპეციალობა უნდა ისწავლოს და შემდეგ როგორც ხდება კარგად დაივიწყოს გარდა იმისა რომელსაც ამოირჩევს.მაგრამ როგორ უნდა ამოირჩიოს პაციენტმა ექიმ-სპეციალისტებიდან  რომელს მიმართოს და თან არსებობს თუ არა სახემძღვანელოები,თუ როგორ ამასთანავე პაციენტმა დაარიგოს თავისი ჩივილები ექიმის კაბინეტების მიხედვით?

ავადმყოფმა არ უნდა გადაწყვიტოს თუ რომელი სპეციალისტი უნდა აირჩიოს. ავადმყოფი პირველად მიდის ზოგადი პროფილის ექიმთან(ყოველ შემთხვევაში ასე უნდა იყოს ნორმალურ ქვეყანაში) და შემდეგ ზოგადი პროფილის ექიმი(ინტერნისტი იქნება თუ თერაპევტი) წყვეტს ავამყოფს სჭირდება თუ არა სპეციალისტთან გაგზავნა და თუ ჭირდება მაშინ შესაბამის სპეციალისტსაც აირჩევს ექიმი.

Posted by: baby-bobo 23 Sep 2007, 15:37
ხელის გულები მაქვს ისეთი წითელი გეგონება ჭარხალი წავისვი, ეს წინა ორსულობაშიც მქონდა მაგრამ ოდნავ და ყურადღებაც არ მიმიქცევია.... ვინმეს გქონიათ ასეთი რამ? ნეტა რისი ბრალი შეიძლება რომ იყოს???
ექიმამდეც ვერ მივდივარ ამ წოლითი რეჟიმის გამო, მინდა 7 თვეს გადავცდე და მერე იქნებ მოვინახულო ექიმი.....

Posted by: vano_t 24 Sep 2007, 01:37
baby-bobo
QUOTE
ხელის გულები მაქვს ისეთი წითელი გეგონება ჭარხალი წავისვი, ეს წინა ორსულობაშიც მქონდა მაგრამ ოდნავ და ყურადღებაც არ მიმიქცევია.... ვინმეს გქონიათ ასეთი რამ? ნეტა რისი ბრალი შეიძლება რომ იყოს???
ექიმამდეც ვერ მივდივარ ამ წოლითი რეჟიმის გამო, მინდა 7 თვეს გადავცდე და მერე იქნებ მოვინახულო ექიმი.....

მაგას ქვია ხელის გულების ერითემა (palmar erythema). ორსულობის დროს ძალიან ხშირად იცის და ნორმალურია. არაორსულსაც შეიძლება ქონდეს ნორმაში. და რა თქმა უნდა არსებობს სხვადასხვა დაავადებები,რომელსაც ახასიათებს ეგ. შენს შემტხვევაში ორსულობასთან დაკავშირებულია ალბათ და არაფერი გამკვლევა არ ჭირდება, თუ სხვა რამ არ გაწუხებს.

Posted by: Tornike Alashvili 24 Sep 2007, 15:19
QUOTE (vano_t @ 22 Sep 2007, 10:39 )
QUOTE (Tornike Alashvili @ 21 Sep 2007, 04:03 )
ერთი კითხვა მაქვს ინტერნისტ ასისტენტებთან.

სრულიად ვეთანხმები იმ აზრს რომ მომავალმა ექიმმა ყველა სამედიცინო სპეციალობა უნდა ისწავლოს და შემდეგ როგორც ხდება კარგად დაივიწყოს გარდა იმისა რომელსაც ამოირჩევს.მაგრამ როგორ უნდა ამოირჩიოს პაციენტმა ექიმ-სპეციალისტებიდან  რომელს მიმართოს და თან არსებობს თუ არა სახემძღვანელოები,თუ როგორ ამასთანავე პაციენტმა დაარიგოს თავისი ჩივილები ექიმის კაბინეტების მიხედვით?

ავადმყოფმა არ უნდა გადაწყვიტოს თუ რომელი სპეციალისტი უნდა აირჩიოს. ავადმყოფი პირველად მიდის ზოგადი პროფილის ექიმთან(ყოველ შემთხვევაში ასე უნდა იყოს ნორმალურ ქვეყანაში) და შემდეგ ზოგადი პროფილის ექიმი(ინტერნისტი იქნება თუ თერაპევტი) წყვეტს ავამყოფს სჭირდება თუ არა სპეციალისტთან გაგზავნა და თუ ჭირდება მაშინ შესაბამის სპეციალისტსაც აირჩევს ექიმი.

ნორმალურ ქვეყანაში ზოგადი პროფილის(ოჯახის)ექიმი წყვიტავს მარა თუ გაჭირდა დიაგნოსტიკა და მკურნალობა და თან რამოდენიმე ქრონიკული პრობლემა აქვს -პაციენტს უწევს თავის მტვრევა სად წავიდეს და რა ჩივილები ვისთან დაანაწილოს.
ეს ფსიქოლოგიური მომენტია და არა აქვს დღეს მედიცინის სისტემას გათვალისწინებული.

Posted by: baby-bobo 24 Sep 2007, 23:49
vano_t
QUOTE
რა თქმა უნდა არსებობს სხვადასხვა დაავადებები,რომელსაც ახასიათებს ეგ

მაგალითად????

----------------------------------------
მადლობა პასუხისთვის, დამამშვიდე ძალიან smile.gif

Posted by: vano_t 25 Sep 2007, 09:49
QUOTE (baby-bobo @ 24 Sep 2007, 23:49 )
vano_t
QUOTE
რა თქმა უნდა არსებობს სხვადასხვა დაავადებები,რომელსაც ახასიათებს ეგ

მაგალითად????

----------------------------------------
მადლობა პასუხისთვის, დამამშვიდე ძალიან smile.gif

არ გინდა მაგალითები. მერე ტყუილად ნერვიულობას დაიწყებ. დამიჯერე, თუ სხვა რაიმესაგან არის გამოწვეული შენი ხელისგულების სიწითლე, პირველ რიგში სხვა უფრო სერიოზული სიმპტომები გექნებოდა.

Posted by: Konkia 1 Oct 2007, 10:29
საიდან დავიცხო არ ვიცი... 7 ში მეჯვარე ვარ, ცინათგრძნობა მქონდა რო ცინა დგეებში დავიწყებდი "სიკვდილს"... ჯერ მარცხენა ფეხზე რაღაც ინფექცია შემეყარა, ათასნაირი პროპილაქტიკის მერე ჰა ჰა მირჩება, მერე გრიპი, ცხვირ-პირი მთლად გაციტლებული მქონდა, ახლაც არ ვარ გამოსული, მაგრამ ვებრძვი....მაგრამ მთავარი გერპესიიი.... უკვე აღარ ვიცი რა ვქნა?! დავიღალე ორი კვირაა თავს ვებრძვი...რა ჯანდაბა ვქნა ახლა??? ვიცი ანალიზები და მკურნალობა სჭირდება, მაგრამ ახლა რა ჯანდაბა ვქნა? ორ სამ დღეში რომ მოვრჩე....HELP

Posted by: baby-bobo 1 Oct 2007, 23:56
vano_t
QUOTE
არ გინდა მაგალითები. მერე ტყუილად ნერვიულობას დაიწყებ. დამიჯერე, თუ სხვა რაიმესაგან არის გამოწვეული შენი ხელისგულების სიწითლე, პირველ რიგში სხვა უფრო სერიოზული სიმპტომები გექნებოდა.

გთხოვ დაწერე რაა, მარტო ჩემს გამო არ მაინტერესებს, ჩემს დაქალსაც იგივე ჭირს პროსტა იმას უფრო წითლად აქვს....

Posted by: თეოგოგო 2 Oct 2007, 00:11
ზაფხულობით მაწუხებს სიმძიმე ქვედა კიდურებში, განსაკუთრებით გაღვიძებისას . sad.gif
და ტკივილი და უსიამოვნო შეგრძნება მაქვს ქუსლიდან ზემოთ, ასწვრივ ერთ ზოლად...
შარშან ვენესცინი და ვენენ-გელი ''გამოვუწერე'' ჩემ თავს და შეღავათი მქონდა.. წელს არ გამიმეორებია და... სულ ბჟუილის შეგრძნება და სიმძიმე მაქვს..

კი ვიცი, რომ ვენებთან დაკავშირებული პრობლემებია, მაგრამ იქნებ ცოტა კონკრეტულად გამარკვიოთ რა დაავადებასთან მაქვს საქმე?
გმადლობთ smile.gif

პ.ს. ჰო, კიდევ.. ზამთარში სულ გაყინული ფეხებით დავდივარ.. (ეხლაც კი ცივი მაქვს ტერფები არადა ოთახში არც კი გრილა sad.gif)

Posted by: vano_t 2 Oct 2007, 03:44
baby-bobo
QUOTE
QUOTE
არ გინდა მაგალითები. მერე ტყუილად ნერვიულობას დაიწყებ. დამიჯერე, თუ სხვა რაიმესაგან არის გამოწვეული შენი ხელისგულების სიწითლე, პირველ რიგში სხვა უფრო სერიოზული სიმპტომები გექნებოდა.

გთხოვ დაწერე რაა, მარტო ჩემს გამო არ მაინტერესებს, ჩემს დაქალსაც იგივე ჭირს პროსტა იმას უფრო წითლად აქვს....

ყველაზე ხშირად, თუ ორსულობას არ ჩავთვლით, იდიოპათიურია ანუ არ იციან მიზეზები. სხვა მიზეზებს განეკუთვენება დერმატიტის ფორმები (მაგალითად ეგზემა), ფსორიაზი, ღვიძლის ციროზი, ე.წ. შემაერთებელი ქსოვილის დაავადებები(რევმატოიდული ართრიტი, წითელი მგლურა, სარკოიდოზი), პოლიციტემია(ანუ სისხლის წითელი უჯრედების მომატება ძვლის ტვინის დაავადების გამო), თირეოტოქსიკოზი(ანუ ფარისებურის მომატებული ფუნქცია) და კიდევ რამოდენიმე იშივათი სიმსივნეები.
* * *
თეოგოგო
QUOTE
ზაფხულობით მაწუხებს სიმძიმე ქვედა კიდურებში, განსაკუთრებით გაღვიძებისას .  sad.gif
და ტკივილი და უსიამოვნო შეგრძნება მაქვს ქუსლიდან ზემოთ, ასწვრივ ერთ ზოლად...
შარშან ვენესცინი და ვენენ-გელი ''გამოვუწერე'' ჩემ თავს და შეღავათი მქონდა.. წელს არ გამიმეორებია და... სულ ბჟუილის შეგრძნება და სიმძიმე მაქვს..

კი ვიცი, რომ ვენებთან დაკავშირებული პრობლემებია, მაგრამ იქნებ ცოტა კონკრეტულად გამარკვიოთ რა დაავადებასთან მაქვს საქმე?
  გმადლობთ smile.gif

პ.ს. ჰო, კიდევ.. ზამთარში სულ გაყინული ფეხებით დავდივარ.. (ეხლაც კი ცივი მაქვს ტერფები არადა ოთახში არც კი გრილა sad.gif)

ასაკი? სქესი ნიკიდან უნდა მივხვდე წესით. სიგარეტს ეწევი? ტკვილები სად იწყება: ქუსლის შიდა მხრიდან თუ გარეთა მხრიდან? სადამდე ადის? ორივე ქუსლს მოიცავს თუ ერთს? ტრავმა ხომ არ მიგიგღია როდისმე? დატვირთვისას ტკივილი ძლიერდება თუ არა? არის თუ არა ტკივილი უფრო ძლიერი პირველი ნაბიჯების გადადგმისას? გაქვს თუ არა ქვემო კიდურების შეშუპება? გაქვს თუ არა ქვემო კიდურებზე კანის ცვლილება? (ან სიწითლე, ან ვენების ვარიკოზული გაგანიერება და ასე შემდეგ) გაქვს თუ არა ზურგის ტკივლი? რაიმე წამალს თუ სვამ? რა პროფესიის ხარ? პროფესია კი არ მაინტერესებს, არამედ შენი დღე თუ მოიცავს დიდი რაოდენობით სირბილს ან დიდი ხნით ფეხზე დგომას ან სიარულს(მაგალითად სპორცმენები ხშირად დარბიან). ჯერ ამ კითხვებით შემოვიფარგლები. პასუხები შეგიძლია PM-ში მოიწერო თუ აქ არ გინდა.

ზოგადად ქვემო კიდურების ტკვილი ბევრმა რამემ იცის. მაგ ადგილას ყველაზე სავარაუდოა აქილევსის მყესების ანთება, ფეხის გულის ფასციი ანთება (პლანტარული ფასციიტი), მეტატარსალური ძვლების ფარული მოტეხილობა, ტარსლური ტუნელის სინდრომი(tibial nerv-ზე ზეწოლა). სხვა რამეებმაც შეიძლება მოგცეს მაქ ტკივლი: ქვემო კიდურების ღრმა ვენების თრომბოზი, არტერიული უკმარისობა, ვარიკოზული ვენები. იტოგში უფრო მეტი ინფორმაციაა საჭირო.

სანამ გაარკვევდე რა არის, შეგიძლია ანთების საწინააღმდეგო ტკივილგამაყუჩებლები დალიო. მაგალითად: იბუპროფენი, ნაპროქსენი, ვოლტარენი და ასე შემდეგ.

Posted by: GT.ge 2 Oct 2007, 10:47
თავის ტკივილის შემთხვევაში, ვის უნდა მიაკითხოს პაციენტმა?

Posted by: tms 2 Oct 2007, 14:34
GT.ge
QUOTE
თავის ტკივილის შემთხვევაში, ვის უნდა მიაკითხოს პაციენტმა?

ნევროლოგს
smile.gif

Posted by: თეოგოგო 3 Oct 2007, 01:52
ასაკი - 27 წლის
სქესი - მდედრობითი
პროფესია - სტომატოლოგი

ტკივილები ქუსლის გარეთ ვრცელდება (უფრო სწორედ ქუსლიდან ზემოთ და არა თვითონ ქუსლი).
ტრავმა სერიოზული არაფერი მიმიღია..
შეშუპება არ მაქვს, მხოლოდ, რამოდენიმე ადგილას წვრილი კაპილარული ქსელი გაჩნდა ამ ბოლო დროს.. sad.gif (კოჭთან შევამჩნიე ახლახანს).
ზურგის ტკივილი მაქვს.. sad.gif
სტომატოლოგი კი ვარ , მაგრამ დამჯდარიც ვმუშაობ და ფეხზე მდგომიც (ანუ სულ ფეხზე არ ვდგავარ)
და ეს უსიამოვნო შეგრძნებაც მატო ფეხზე დგომის დროს არ მაქვს..
ეუთიროქსს (L-თიროქსინი)ვსვამ მხოლოდ.. სხვას არაფერს.
შეიძლება, რომ ზოგადად მქონდეს ვენებთან დაკავშირებული პრობლემები და მხოლოდ წვივებზე გამოვლინდეს ამ შემთხვევაში?
ამას იმიტომ ვკითხულობ, რომ ხანდახან ხელის მტევნებზე ვენები ''მებერება''. ხან კი ნეკი და არათითი მიბუჟდება სუსტად.. sad.gif
პროთრომბინზე გავიკეთო ანალიზი თუ კუაგულოგრამა აჯობებს ამ შემთხვევაში?

რაღაცით ხომ უნდა დავიწყო ანალიზების კეთება , ეგრევე ვენების გამტარიანობაზე ხომ არ გავიკეთებ ტესტს?



ააუ, რა სერიოზული ავადმყოფი ვყოფილვააარ...ეს რამდენი დამიწერია vis.gif

Posted by: vano_t 5 Oct 2007, 12:56
თეოგოგო
QUOTE
პროთრომბინზე გავიკეთო ანალიზი თუ კუაგულოგრამა აჯობებს ამ შემთხვევაში?

რაღაცით ხომ უნდა დავიწყო ანალიზების კეთება , ეგრევე ვენების გამტარიანობაზე ხომ არ გავიკეთებ ტესტს?
პროთრომბინი არ ვიცი რა არის. პროთრომბინის დრო კი ვიცი. და საერთოდ სისხლის ეგ მახასიათებელი არავითარ დიაგნოზს არ მოგცემს ვენების დაავადებაზე. კოაგულოგრამა(ზუსტად არ ვიცი რას გულისხმობ მაგაში) წამროდგენას გაძლევს სისხლის შემადედებელი ფაქტორების რაოდენობასა და თვისებებზე.
თუ ვენების პრობლემებზეა ლაპარაკი მაშინ ულტრაბგერა უნდა მაგას.

QUOTE
შეიძლება, რომ ზოგადად მქონდეს ვენებთან დაკავშირებული პრობლემები და მხოლოდ წვივებზე გამოვლინდეს ამ შემთხვევაში?
შეიძლება

რაც შენ აღწერე სავარაუდოა ვარიკოზები იყოს. ვარიკოზების დადგენა ადვილია დათვალიერებით. წესით და რიგით კაი თერაპევტს თუ ნახავ უნდა დაგაკვალიანოს.

Posted by: iFFii 5 Oct 2007, 20:22
ააააააააააააააააააააააააააააააა

ფიზიკური დატვირთვისას მარჯვნივ მტკივა რაღაც...ღვიძლია მგონი...
ბრმა ნაწლავი არ მაქს..
ჰოდა რა მჭირს ნეტა?

Posted by: თეოგოგო 6 Oct 2007, 03:22
vano_t
QUOTE
რაც შენ აღწერე სავარაუდოა ვარიკოზები იყოს. ვარიკოზების დადგენა ადვილია დათვალიერებით.

არა, არანაირი ვარიკოზები... (ხელის მტევნებს გულისხმობ ალბათ, ხომ?) უბრალოდ ვენები მკვეთრდება და მოხაზულობა მემჩნევახოლმე ხანდახან.

არც ფეხზე მაქვს ვარიკოზის მსგავსი რამ. (ღმერთმა დამიფაროს smile.gif)

უბრალოდ სიმძიმე მაქვს ფეხებში და ზოლზე მტკივა ქუსლიდან ზემოთ, წვივის შიდა მხარეს.


პროთრომბინი და კუაგულოგრამა კი იმიტომ ვიკითხე, რომ სისხლი, ხომ არ მაქვს ''ზედმეტად სქელი'' და ამის გამო შეგუბება, ხომ არ ხდება ვენებშითქო (სიმძიმე ფეხებში).
QUOTE
კოაგულოგრამა წამროდგენას გაძლევს სისხლის შემადედებელი ფაქტორების რაოდენობასა და თვისებებზე.

yes.gif ვიცი



smile.gif

Posted by: vano_t 6 Oct 2007, 14:54
თეოგოგო
QUOTE
არა, არანაირი ვარიკოზები... (ხელის მტევნებს გულისხმობ ალბათ, ხომ?) უბრალოდ ვენები მკვეთრდება და მოხაზულობა მემჩნევახოლმე ხანდახან.
ხელის მტევნებს არ ვგულსხმობ. ხელის მტევნებზე არ მინახია ვარიკოზები. ქვემო კიდურებს ვგულსხმობდი, იმიტომ რომ რაღაც ახსენე კაპილარული ქსლები გაჩნდაო. ისე ვარიკოზები ძაან გაფართოებული ვენებია. სიმძიმის შეგრძნებაც იცის.

QUOTE
პროთრომბინი  და კუაგულოგრამა კი იმიტომ ვიკითხე, რომ სისხლი, ხომ არ მაქვს ''ზედმეტად სქელი'' და ამის გამო შეგუბება, ხომ არ ხდება ვენებშითქო (სიმძიმე ფეხებში).
პროთრომბინი და კოაგულოგრამა(ან მაგის შესაბამისი ტესტები დასავლურ მედიცინაში) არ ადგენს სისხლის "ზედმეტ სისქელეს", არამედ "სითხელეს" ადგენს. ვენებში სისხლი შეგუბება როცა ხდება(ანუ სტაზი რაც არის ვირხოვის ტრიადის ერთ-ერთი შემადგენელი) მაშინ იზრდება თრომბოზების შანსი. მაგის სადიაგნოსტიკოდ კიდევ ისევ ულტრაბგერა გამოიყენება. ადრე ვენოგრაფიას ხმარობდენ, ძალიან ინვაზიური ტესტია. დღეს მაგ ტესტ აღარ აკეთებენ განვითარებულ ქვეყნებში. აი თუ მაგის დიაგნოზს დაადგენენ ახალგაზრდა ასაკში და პაციენტს რაიმე რისკ ფაქტორი არ გააჩნია (დავუშვათ მაგალითად ახალგაზრდა ქალი რომელიც არ იღებს ჩასახვის საწინააღმდეგო წამლებს) მაგ შემთხვევაში იკვლევენ გაძლიერებული შედედების მიზეზებს სპეციალური ტესტებით, როგორიცაა პროტეინი C ან S, ანტითრომბინი, ლეიდენის ტიპის მე-5 ფაქტორი და ასე შემდეგ.

Posted by: თეოგოგო 6 Oct 2007, 22:43
vano_t
QUOTE
ახსენე კაპილარული ქსლები გაჩნდაო

კი, გაჩნდა, მაგრამ მხოლოდ ძაფისებური, ძალიან წვრილი ხაზები გაჩნდა და არანაირი ვარიკოზები.
შეიძლება, რომ ღრმა ვენებზე გაჩნდეს ვარიკოზები ისე, რომ არ ჩანდეს? (ალბათ უკვე თავი მოგაბეზრე). drug.gif

QUOTE
პროთრომბინი და კოაგულოგრამა(ან მაგის შესაბამისი ტესტები დასავლურ მედიცინაში) არ ადგენს სისხლის "ზედმეტ სისქელეს", არამედ "სითხელეს" ადგენს. ვენებში სისხლი შეგუბება როცა ხდება(ანუ სტაზი რაც არის ვირხოვის ტრიადის ერთ-ერთი შემადგენელი) მაშინ იზრდება თრომბოზების შანსი.


სითხელე-სისქელის დადგენა ერთი და იგივე არ გამოდის? spy.gif


QUOTE
(დავუშვათ მაგალითად ახალგაზრდა ქალი რომელიც არ იღებს ჩასახვის საწინააღმდეგო წამლებს)

ეს რა შუაშია ვერ მივხვდი... ups.gif


უღრმესი მადლობა გულისხმიერებისათვის smile.gif

Posted by: Solveig 6 Oct 2007, 23:04
თეოგოგო
QUOTE
ეს რა შუაშია ვერ მივხვდი...

რა შუაშია და ეგ საშუალებები თრომბოზების რისკს ზრდის, ამიტომ ვისაც სისხლძარღვებზე პრობლემა აქვს, არ ურჩევენ მიღებას..

Posted by: BadbadGirl 6 Oct 2007, 23:35
ხალხებო აი ასეთი სიმპტომები მაქვს:
1. ცხვირიდან მომდის სითხე (დღეს დილიდან)
2.ვაპცხიკებ ხშირ ხშირად (დღეს დილიდან)
3.ყელი მტკივა , არა ღრმად არამედ ენასთან ახლოს , (ხახასთან ) (ასე 4 დღეა მტკივა)

რა ვუშველო თავს (სიცხის საზომი არ გვაქვს და არ ვიცი მაქვს თუ არა სიცხე, )

Posted by: თეოგოგო 7 Oct 2007, 00:24
Solveig
გმადლობ! 2kiss.gif


* * *
BadbadGirl
პირველ რიგში ლიმონიან-თაფლიანი ჩაი დალიე.
სოდიანი სავლები გამოივლე.


მერე ექიმთან წადი smile.gif





Posted by: BadbadGirl 7 Oct 2007, 21:51
თეოგოგო
QUOTE
ირველ რიგში ლიმონიან-თაფლიანი ჩაი დალიე. სოდიანი სავლები გამოივლე.

გავაკეთე
ქოლდრექსაც ვსვავ, მალიანიან ჩაისაც და არაფერი არ მშველის
კიდე როცა ვწევარ ასე ცუდად არ ვგრძნობ თავს , როცა ფეხზე ვარ გამოჭედილი მაქვს ყურებიც და ცხვირიც.

კოშმარ.

Posted by: vano_t 8 Oct 2007, 08:20
BadbadGirl
QUOTE
QUOTE
ირველ რიგში ლიმონიან-თაფლიანი ჩაი დალიე. სოდიანი სავლები გამოივლე.

გავაკეთე
ქოლდრექსაც ვსვავ, მალიანიან ჩაისაც და არაფერი არ მშველის
კიდე როცა ვწევარ ასე ცუდად არ ვგრძნობ თავს , როცა ფეხზე ვარ გამოჭედილი მაქვს ყურებიც და ცხვირიც.

კოშმარ.

შენ უბრალოდ ზემო სასუნთქი გზების ინფექცია გაქვს, მეტი არაფერი. მაგის მკურნალის არის დრო. მთავარია გაუწყლოება არ დაგემართოს, ანუ უნდა სვა დიდი რაოდენობით სითხე(სულ ერთია ას დალევ, წყალს თუ წვენს თუმცა წყალი ჯობია ყველაფერს). სიმპტომატური მკურნალობისათვის შეიძლეა მიიღო ტიალენოლი(ტკივილის ან სიცხის დასაწევად), ცხვირში შეგუბების მომხსნელი(რაც იშოვება იქ სადაც ცხოვრობ. მე სადაც ვიმყოფები იქ იხმარება სუდაფედი, აფრინი, ნეოსინეფრინი. საქართველოში ვიცი რომ გალაზოლინი იხმარებოდა) ან კიდევ ხალხრური საშუალებები.

მაგას გართულებებიც ახლავს, მაგრამ არც ისე ხშირად. შეიძლება შუა ყურის ანთება განვითარდეს ან "მოსიარულე პნევმონია" ან სინუსიტი. ამათი დადგენა ადვილია. შენ მაგის ნიშნები არ გაქვს. ანტიბიოტიკები საჭიროა გართულების დროს ან ზოგიერთ ავადმყოფში ვისაც სასუნთქი გზების პრობლემები აქვთ(მაგალითად ასთმა ან ქრონიკული ობსტრუქციული ფილტვების დაავადება, ემფიზემის და ქრონიკული ბრონქიტის ჩათვლით) ან ვისაც იმუნური სიტემა აქვს დაქვეითებული(მგალითად მოხუცებს, სიმსივნიან ავადმყოფებს, ის ვინც სტეროიდებს ღებულობს გამუდმებით).

შენ კონკრეტულ შემთხვევაში ბევრი არადერი გინდა გარდა ზემოდ აღწერილი მკურნალობისა.

Posted by: BadbadGirl 8 Oct 2007, 12:47
vano_t
QUOTE
შენ უბრალოდ ზემო სასუნთქი გზების ინფექცია გაქვს, მეტი არაფერი. მაგის მკურნალის არის დრო. მთავარია გაუწყლოება არ დაგემართოს, ანუ უნდა სვა დიდი რაოდენობით სითხე(სულ ერთია ას დალევ, წყალს თუ წვენს თუმცა წყალი ჯობია ყველაფერს). სიმპტომატური მკურნალობისათვის შეიძლეა მიიღო ტიალენოლი(ტკივილის ან სიცხის დასაწევად), ცხვირში შეგუბების მომხსნელი(რაც იშოვება იქ სადაც ცხოვრობ. მე სადაც ვიმყოფები იქ იხმარება სუდაფედი, აფრინი, ნეოსინეფრინი. საქართველოში ვიცი რომ გალაზოლინი იხმარებოდა) ან კიდევ ხალხრური საშუალებები.

უდიდესი მადლობა, ძალიან დიდი რაოდენობით ვსვავ ჩაის , ყველანაირს მწვანე,გვირილა და შავი . ასევე დილა საღამოს ვსვავ ნეო ციტრანს. ჯერ არ უშველია მაგრამ იმედია მალე გამივლის smile.gif

Posted by: niniko_j 10 Oct 2007, 12:53
ძალიან დიდად მადლობელი დაგრჩებით თუ დამეხმარებით :-)
თვალი მითამაშებს რამდენიმე დღეა, უკვე ძალიან შემაწუხებელი გახდა. ყოველდღე, დღის გარკვეულ პერიოდში, ვერ ვიტყვი რომ ზუსტად ერთიდაიგივე დროს ან ვერც იმას ვიტყვი რომ რამეს უკავშირდებოდეს (მაგ. კომპუტერს) მეწყება, მერე ისევ ჩერდება, მერე ისევ მეწყება და ა.შ. იქნებ მითხრათ რისი ბრალია და როგორ შეიძლება ვუშველო. რადგანაც საქართველოში არ ვარ ექიმთან მისვლისგან თავს ვიკავებ :-(
წინასწარ მადლობა

Posted by: vano_t 11 Oct 2007, 13:07
niniko_j
QUOTE
ძალიან დიდად მადლობელი დაგრჩებით თუ დამეხმარებით :-)
თვალი მითამაშებს რამდენიმე დღეა, უკვე ძალიან შემაწუხებელი გახდა. ყოველდღე, დღის გარკვეულ პერიოდში, ვერ ვიტყვი რომ ზუსტად ერთიდაიგივე დროს ან ვერც იმას ვიტყვი რომ რამეს უკავშირდებოდეს (მაგ. კომპუტერს) მეწყება, მერე ისევ ჩერდება, მერე ისევ მეწყება და ა.შ. იქნებ მითხრათ რისი ბრალია და როგორ შეიძლება ვუშველო. რადგანაც საქართველოში არ ვარ ექიმთან მისვლისგან თავს ვიკავებ :-(
წინასწარ მადლობა

არაფრის ბრალი არ არის. ფასციკულაცია ქვია მაგას. შენს შემთხვევაში კეთილტვისებიანი ფასციკულაციაა. როცა სერიუზული დაავედებების ბრალია, მაშინ სხვა სიმპტომებით გამოვლინდება. ბევრ ჯანმრთელ ადამიანს აქვს ეგ.

Posted by: niniko_j 11 Oct 2007, 16:28
vano_t
დიდი მადლობა. არაკეთილთვისებიანისას რა სიმპტომებია? რომელ დაავადებებს ახლავს?

Posted by: vano_t 13 Oct 2007, 09:40
niniko_j
QUOTE
დიდი მადლობა. არაკეთილთვისებიანისას რა სიმპტომებია? რომელ დაავადებებს ახლავს?

უამრავ დაავადებას შეიძლება ახლდეს. 'არაკეთილთვისებიანი' უბრალოდ იმიტომ ქვია რომ რაღაც დაავადებას ახლავს და არა იმიტომ რომ სერიოზული დაავადების თანმხლებია. უფრო სწორად არაკეთილთვისებიან ფასციკულაცია(მასეთი სახელით) არ არსებობს. კეთილთვისებიანს უწოდებენ ისეთ ფორმას რომელიც 'მოვა და წავა'. თითქის ყველა ჯანმრთელ ადამიანს განუცდია მასეთი რამ. ამიტომ, მარტო დაკვირვებაა საჭირო. თუ გადავა რამოდენიმე დღეში, არაფერი გჭირდება. თუ დიდ ხანს გაგრძელდება, მერე ნევროლოგს უნდა მიაკითხო, ან თერაპევტს(ან რაც არის მაქ ზოგადი პროფილის სპეციალისტი).

Posted by: ALEF 13 Oct 2007, 14:09
საყლაპავი მილის თიაქარი...
რას ნიშნავს,რით იკურნება?....როგორი პროგნოზი აქვსკონსერვატული მკურნალობის ან ოპერაციის შემდეგ?
ჩემს ახლობელს აქვს....ოპერაციას არ გირჩევო ექიმმა უთხრა...მაგრამ ეს კონსერვატული მკურნალობაც დიდად არ შველის...საჭმელს ძლივს ჭამს...უფრო სცორად ძლივს ყლაპავს..რამოდენიმეჯერ კინაღამ დაიხრჩო(ვერ გადაყლაპა)...მოკლედ ექიმების აზრი მაინტერესებს....დიდი მადლობა წინასწარ...smile.gif
* * *
..............ნუთუ მართლა არავინ არაფერი იცის ?...sad.gif

Posted by: kuchu 14 Oct 2007, 16:28
სამედიცინოზე მაქვს დასაწერი რეფერატი კრიპტორქიზმზე, ქართულ ინტერნეტ სივრცეში (რუსულის მსგავსად) არსებობს მზა რეფერატები ქართულად? biggrin.gif

Posted by: vano_t 17 Oct 2007, 10:03
ALEF
QUOTE
საყლაპავი მილის თიაქარი...
რას ნიშნავს,რით იკურნება?....როგორი პროგნოზი აქვსკონსერვატული მკურნალობის ან ოპერაციის შემდეგ?
ჩემს ახლობელს აქვს....ოპერაციას არ გირჩევო ექიმმა უთხრა...მაგრამ ეს კონსერვატული მკურნალობაც დიდად არ შველის...საჭმელს ძლივს ჭამს...უფრო სცორად ძლივს ყლაპავს..რამოდენიმეჯერ კინაღამ დაიხრჩო(ვერ გადაყლაპა)...მოკლედ ექიმების აზრი მაინტერესებს....დიდი მადლობა წინასწარ...smile.gif
* * *
..............ნუთუ მართლა არავინ არაფერი იცის ?...sad.gif

საყლაპავი მილი ძირითადად მდებარებოს დაიფრაგმის მაღლა(მუცლის და გულმკერდის გამყოფი კუნთია დიაფრაგმა). როგორც კი შედის საყლაპვი მუცლის ღრუში, მაშინვე უკავშირდება კუჭს. ზოგჯერ ხდება რომ კუჭის ზემო ნაწილია (ფუნდუსი) საყლაპავის დიაფრაგმის ხვრელით მაღლა ადის გულმკერდის ღრუში და მაგას ეძახიან საყლაპავის თიაქარს. საყლაპავის თიაქარი ისე არ არის დაავადება თავსთავად. საყლაპავის თაიქარი უბრალოდ უფრო ხშირად უკავშირდება ე.წ. გასტროეზოფაგალურ რეფლუქსს(რომლის დროსაც კუჭის მჟავე წვენი საყლაპავში ადის).

და ყლაპვის გაძნელების მიზეზად ნამდვილად არ ითვლება საყლაპავის თიაქარი. ყლაპვის გაძნელება შეიძლება მოხდეს დავუშვათ იმის გამო რომ ვისაც დიდი ხნის და მძიმე რეფლუქსი აქვს, იმას შეიძლება განუვითარდეს საყლაპავის შევიწროება. ასე რომ არასწორია იმის თქმა რომ საყლაპავის თიაქრას როგორ უნდა უმკურანლოო. შენს ახლობელს თუ ყლაპვა უჭირს, მაშინ ენდოსკოპიით ყლაპვის გაძნელების მიზეზი უნდა დადგინდეს და შესაბამისად მკურანლობაც დაენიშნოს.

Posted by: BadbadGirl 18 Oct 2007, 02:08
აი ასეთი შეკითხვა მაქვს:
ჩემ ქმარს დასიებული აქვს თითი, გვერძე ფრჩხილსა და კანს შორის არის რაღაცნაირად გამწვანებული. ვფიქრობ რომ ჩირქი უნდა იყოს. ძალიან სტკივა და არ ვიცი რა მოვუხერხო.
მირჩიეთ რამე პლიზ.
წინასწარ მადლობა დახმარებისატვის.

Posted by: vano_t 18 Oct 2007, 20:14
QUOTE (BadbadGirl @ 18 Oct 2007, 02:08 )
აი ასეთი შეკითხვა მაქვს:
ჩემ ქმარს დასიებული აქვს თითი, გვერძე ფრჩხილსა და კანს შორის არის რაღაცნაირად გამწვანებული. ვფიქრობ რომ ჩირქი უნდა იყოს. ძალიან სტკივა და არ ვიცი რა მოვუხერხო.
მირჩიეთ რამე პლიზ.
წინასწარ მადლობა დახმარებისატვის.

ალბათ პანარიციუმი აქვს, მარტივი ინფექციაა მაგრამ ძალიან მტკივნეული შეიძლება იყოს. თუ ექიმთან მიიყვან პატარა განაკვეთს გაუკეთებს და დრენირება მოხდება ჩირქის. შეიძლება ანტიბიოტიკიც დაუნიშნოს. ტკივილიც ეგერევე გაუვლის ჩირქის დრენირების მერე.

Posted by: BadbadGirl 18 Oct 2007, 22:03
vano_t
QUOTE
ალბათ პანარიციუმი აქვს, მარტივი ინფექციაა მაგრამ ძალიან მტკივნეული შეიძლება იყოს. თუ ექიმთან მიიყვან პატარა განაკვეთს გაუკეთებს და დრენირება მოხდება ჩირქის. შეიძლება ანტიბიოტიკიც დაუნიშნოს. ტკივილიც ეგერევე გაუვლის ჩირქის დრენირების მერე.  შ

მადლობა .
დღეს ვიყავით აფთიაქში და იქ გვითხრეს alsolsprit (kutan lösning 10mg/ml) საფენები დაიდეთო. იქაც მასე გვითხრეს , თუ ორ დღეში არ ჩაცხრა მაშინ ექიმთან წადიოთო sad.gif

Posted by: avicena 22 Oct 2007, 17:27
ვანო ძალიან კარგი იდეაა ეს ფორუმი, ერთი შეკითხვა მაქვს. ხომ არ იცი მე 3 სტადიის რაბდომიოსარკომის დროს მკურნალობის მეთოდები, ქიმიოთერაპიული საშუალებები, მათი გვერდითი ეფექტები, მათი თავიდან აცილება, პროგნოზი?
წინასწარ დიდი მადლობა

Posted by: vano_t 23 Oct 2007, 06:00
avicena
QUOTE
ვანო ძალიან კარგი იდეაა ეს ფორუმი, ერთი შეკითხვა მაქვს. ხომ არ იცი მე 3 სტადიის რაბდომიოსარკომის დროს მკურნალობის მეთოდები, ქიმიოთერაპიული საშუალებები, მათი გვერდითი ეფექტები, მათი თავიდან აცილება, პროგნოზი?
წინასწარ დიდი მადლობა

მაგის აღწერას ტომეულები არ ეყოფა. ისე ნებისმიერი სიმსივნის, სარკომების ჩათვლით, მკურნალობის 3 ფორმა არსებობს: ქირურგია, რადიოთერაპია(ანუ დასხივება) და ქიმიოთერაპია. ქიმიას ძალიან ბევრი გართულებები აქვს და აქვს გენერიული გართულებები, რომელიც საერთოა უმრავლესი ქიმიოთერაპიული საშუალებებისათვის (რაც ძირითადად მათი მოქმედების მექანიზმიდან გამომდინარეობს, კერძოდ მიტოზის დათრგუნვა) და მოცემული პრეპარატისათვის დამახასიათებელი გართულებები. გართულებებების რიცხვს მიეკუთვნება და არ შემოიფარგლება ამით:
ძვლის ტვინის დათრგუნვა და შესაბამისად ანემიების, თრომბოციტოპენიის და ლეიკოპენიის ჩამოყალიბება;
ცნს-ის და პნს-ის დაავადებები;
გასტორინტესტინალური სისტემის დაავადებები;
თირკმლის და შარდის გამომყოფი ორგანოების დაავადებები;
ძვლის და კუნთების დაავადებები და ასე შემდეგ.

Posted by: kuchu 23 Oct 2007, 13:35
რუსულიდან ვთარგმნი რაღაც სამედიცინო თემას და რუსულ ქართული ონლაინ ლექსიკონი არსებობს ნეტში? :O

Posted by: avicena 23 Oct 2007, 14:10
რუსულ ქართული ლექსიკონის შესახებ არ ვიცი გარდა ერთისა. ალტავისტას სეარჩ ენგინინის ფიშ ტრანსლატორი და ვინდოუსის ტეზაურუსის ტრანსლატორი.
* * *
ჩემს მეგობარს ჯერ გამოაყარა მკერდზე, შემდეგ შარდვისას ტკივილები დაეწყო, ორი კვირის შემდეგ ეს გაუმჯობესდა მაგრამ სმენა დააკლდა და ლაპარაკისას ხანდახან ქოშინი ეწყება და სუსტად გრძნობს თავს. ვალიდოლი და ვიტამინი ბ6 გამოუწერეს და სასწაულად მოუხდა. ახლა ხელები უსივდება საღამოობით და კბილის არეში ტკივილები აწუხებს. ხომ არ შეიძლება ეს წამლების გვერდითი ეფექტი იყოს?

Posted by: kuchu 23 Oct 2007, 14:19
avicena
ვერაფერივერგავიგე biggrin.gif

подвздошно-поховой нерв - რომელია? :O და тяжа რას ნიშნავს? rolleyes.gif

Posted by: avicena 24 Oct 2007, 02:13
კუჩუ ალტავისტა როგორც იაჰუ და გუგლი ხომ იცი იქ არის ფიში ასეთი იკონა და რომ დაკლიკავ ტრანსლატიონს გაძლევს სხვადასხვა ენებზე ასევე ვინდოუს უორდს აქვს ტეზაურუს ანუ თარგმნის ფუნქცია. დანრჩენი ხუმრობა იყო. პახავოი ნერვი რო პახაობ ის არის(: ანუ შორისის ნერვი, პოვზდოშნის ვერ ვიხსენებ.

Posted by: Pathetikos 24 Oct 2007, 11:50
განყოფილების წესებს რომ გავეცანი, ახალი თემის გახსნაზე ამაკანკალა, არადა არ ვიცოდი, სად მეკითხა, ხოდა ბარემ აქ იყოს. biggrin.gif

ნიმელისის ერთი პაკეტი წყლის რა რაოდენობაში და რა ტემპერატურაში უნდა გაიხსნას?

Posted by: perikardi 24 Oct 2007, 18:43
ხომ არ იცით საიტი სადაც იქნება მასალა ტრავამტოლოგიური პაციენტების წინა საოპერაციო მართვაზე თერაპიული კუთხით?

Posted by: vano_t 24 Oct 2007, 20:57
perikardi
QUOTE
ხომ არ იცით საიტი სადაც იქნება მასალა ტრავამტოლოგიური პაციენტების წინა საოპერაციო მართვაზე თერაპიული  კუთხით?

საიტს ინგლისურად მოგინახავ თუ გინდა, მაგრამ დიდი განსხვავება არ არის პაციენტის წინასაოპერაციო გამოკვლევაში ტრამვატოლოგიური და არატრამვატოლოგიური ოპერაციების წინ. უბრალოდ ტრამვატოლოგიური ოპერაციები შეიძლება დაბალი ან საშუალო სიმძიმის რისკის ოპერაციებად განისაზღვროს. ყველა ავადმყოფის წინასაოპერაციო შეფასება ძირითადად ერთნაირად ხდება მიუხედავად ოპერაციისა. შეფასების არსი არის პერიოპერაციული კარდიოვასკულარული გართულებების (რაშიც ძირითადად ინსულტები და ინფარქტები შედის) რისკის დადგენა. და თუ ფილტვზე ტარდება ოპერაცია (რედუქციული ტიპის) ან ავადმყოფს მნიშვნელოვანი ფილტვების დაავადება აქვს, მაშინ ამ მხრივაც უნდა შეაფასო ავადმყოფი რომ დაადგინო თუ რა შანსი აქვს ავადმყოფს ოპერაციის შემდეგ რესპირატორიდან მოხსნა/არმოხსნის. მე თვითონ რამოდენიმე რეფერალი მომდის ყოველ თვე ავადმყოფების წინასაოპერაციო შეფასებისა და მართვის და (ყველაზე ხშირად ტრამვატოლოგიური და ოფთალმოლოგიური ოპერაციებისათვის) თუ რაიმე სხვა კითხვებიც გაქვს შევეცდები გიპასუხო თუ შემიძლია. მანამდე მოკლედ გეტყვი რაზე ამახვილებ ყურადღებას:
1) ანამნეზში ძირითადად ეკითხები მკერდის ტკივილებზე, სუნთქვის უკმარისობაზე მოსვენების და დატვირთვის დროს, ქვემო კიდურების შეშუპებაზე, ორთოპნეაზე და პაროქსიზმულ დისპნეებზე ღამით. კიდევ ხანგამოშვებითი კოჭლობის სიმპტომები თუ აქვთ და საერთოდ რაც პერიფერული და ცენტრალური სისხლძარღვების დაზიანების სიმპტომებს იძლევა.
2) წარსული დაავადებებიდან გაინტერესებს დიაბეტი(რადგანაც ძალიან დიდი რისკი აქვთ ინფარქტების და ინსულტების), ინსულტი, TIA(transient ischemic attacks) ანუ გარდამავალი იშემიური შეტევები რაც ფაქტიურად 24-საათიანი ინსულტების ფომრაა, მიოკარდიუმის ინფარქტი, პერიფერული სისხლძარღვების დაავადება.
3) იმისდამიხედვით თუ რა სახის ოპერაცია კეთდება, რა სიმპტომები აქვს ავადმყოფს და რა წარსულის დაავადებები, ავადმყოფს შეიძლება დასჭირდეს ან ეკგ ან სტრეს ტესტი.
4) თუ ფილტვების პრობლემები აქვს და ოპერაცია ზოგადი ნარკოზით მიდის(ანუ ავადმყოფის ინტუბაცია არის საჭირო), მაშინ უნდა გაუკეთო PFT(pulmonary function tests) და დაადგინო რამოდენიმე მაჩვენებელი, განსაკუთრებით სასიცოცხლო ტევადობა.
5) რა თქმა უნდა რაიმე მწვავე დაავადებაც უნდა გამორიცხო, მაგალითად საშარდე გზები ინფექცია, ზემო სასუნთქი გზები ინფექცია და ასე შემდეგ. თუ რაიმე მწვავე დაავადება აქვს ავადმყოფს და ოპერაცია გადაუდებელი არ არის, მაშინ ოპერაციამდე ყველა ასეთი პრობლემა უნდა მოაგვარო. ოპერაციამდე უნდა მოაგვარო წნევის კონტროლიც, თუ არაკონტროლირებადი წნევა აქვს ავადმყოფს. ოპერაციამდე უნდა მოაგვარო დიაბეტის კონტროლიც, თუ კარგად არ არის ნაკონტროლები დიაბეტი.

http://www.guideline.gov/summary/summary.aspx?doc_id=9004&nbr=004880&string=preoperative+AND+assessment არის ამერიკული გაიდლაინი. თუ ინგლისურად კითხულობ, გამოგადგება ეგ სტატია. საერთოდ, თუ რაიმეს ამერიკული გაიდლაინი გაინტერესებს, www.guideline.gov-ზე ნახე. ეს საიტი ოფოციალური საიტია სტანდარტულად მიღებული გაიდლაინების.

Posted by: თეოგოგო 24 Oct 2007, 22:33
Pathetikos
QUOTE
ნიმელისის ერთი პაკეტი წყლის რა რაოდენობაში და რა ტემპერატურაში უნდა გაიხსნას?

ერთი ყავის ჭიქა ანადუღარი და ოთახის ტემპერატურამდე გაციებული წყალი სრულიად საკმარისია. smile.gif







QUOTE
განყოფილების წესებს რომ გავეცანი, ახალი თემის გახსნაზე ამაკანკალა, არადა არ ვიცოდი, სად მეკითხა, ხოდა ბარემ აქ იყოს.

keh.gif biggrin.gif

Posted by: ჩოხა 29 Oct 2007, 13:02
რა შეიძლება იყოს ღამით, ძილში ძლიერი ოფლიანობის გამომწვევი მიზეზი და რომელ ექიმს შეიძლება ამ საკითხზე მივმართოთ ხომ არავინ იცის? sleep.gif

Posted by: vano_t 29 Oct 2007, 17:49
choxa79
QUOTE
რა შეიძლება იყოს ღამით, ძილში ძლიერი ოფლიანობის გამომწვევი მიზეზი და რომელ ექიმს შეიძლება ამ საკითხზე მივმართოთ ხომ არავინ იცის?  sleep.gif

ძალიან ბევრი რამის ბრალი შეიძლება იყოს. თუ მთელი ცხოვრება მასე გაქვს, მაშინ არაფერი გამოკვლევა არ ჭირდება. არიან ადამიანები რომელიც ოფლიანობენ ნორმაზე მეტად, თუმცა ნორმაა მათთვის. თუ ამ ბოლო დროს დაგეწყო ეგ (რამოდენიმე კვირის ან თვის წინ) მაშინ გამოკვლევა ჭირდება. მაგრამ რადგანაც ოფლიანობა ძალიან არასპეციფიური ნიშანია (ანუ ბევრი სხვადასხვა დაავადების თანხმლები შეიძლება იყოს), ამიტომ უნდა ნახო თერაპევტი თავიდან და თერაპევტმა უნდა გააკეთოს დაწვრილებით გამოკითხვა სხვა სიმპტომების და დაწვრილებით უნდა გაგსინჯოს. შემდეგ გაკეთდება ძირითადი ლაბორატორიები და იმის მიხედვით თუ რისკენ მიუთითებს ეს ყველაფერი, შეიძლება სხვადასხვა სპეციფიური ტესტი გაგიკეთონ.

მოკლე სია ამ დაავადებების შეიძლება ასე გამოიყურებოდეს:
1) ქრონიკული ინფექციები(გასაკუთრებით ტუბერკულოზი საქართველოში);
2) დიაბეტი;
3) ალკოჰოლიზმი ან სხვა რაიმე ნარკოტიკის გამოყენება;
4) ფარისებური ჯირკვლის მომატებული ფუნქცია;
5) პანიკის შეტევებეი და საერთოდ ნევროზი;
6) ორსულობა;
7) ძილის ობსტრუქციული აპნეა (ანუ ძილში ხვრინვები და სუნთქვის გაჩერებები-ჭარბი წონის ადამიანებს აქვთ ხშირად);
8) ქალებში მენოპაუზე(კლიმაქსს რომ ეძახიან) და ორსულობა;
9) ლიმფომები, ლეიკემიები და სხვა სიმსივნეები;
10) ბევრმა სხვადასხვა წამალმა შეიძლება გამოიწვიოს ეგ;
და ასე შემდეგ. იტოგში ზოგადი სახის სპეციალისტი გჭირდება თუ ეგ პრობლემა ახალდაწყებულია და თუ მთელი ცხოვრება ეგრე გაქვს, მაშინ არაფერი უნდა.

Posted by: avicena 29 Oct 2007, 20:39
გულის დაავადებებიდან ოფლიანობას ენდოკარდიტი იწვევს ანუ გულის სარქველების ინფექცია

Posted by: vano_t 30 Oct 2007, 09:08
QUOTE (avicena @ 29 Oct 2007, 20:39 )
გულის დაავადებებიდან ოფლიანობას ენდოკარდიტი იწვევს ანუ გულის სარქველების ინფექცია

იწვევს ნამდვილად. პრინცმეტალის ანგინასაც შეუძლია გამოიწვიოს ეგ, ან ნებისმიერი სტენოკარდიების შეტევებს ღამით. თუმცა ხშირი დაავადებები უფრო ხშირია ვიდრე არახშირი დაავადებები. ანუ თრაქუნის ხმაზე ცხნებზე იფიქრე და არა ზებრაზეო wink.gif საბოლოო ჯამში ქრონიკული ინფექციებიდან, ტუბერკულოზი უფრო სავარაუდოა საქართველოში, თუმცა ინტრავენური ნარკოტიკების მოხმარება და შპრიცების განაწილება ენდოკარდიტის დიდ რისკაც იწვევს, პლიუს ამას ჰეპატიტების და შიდსის რისკიც მნიშვნელოვანია, რომელთაგან ორივე შეიძლება იყოს მიზეზი ღამის (და საერთოდ) ოფლიანობის.

Posted by: perikardi 30 Oct 2007, 18:49
vano_t
გაიხარე, მეც ვიპოვე ბევრი მასალა, მილერის ანესთეზიოლოგია მოვიპოვე და იქიდან ვკითხულობ რაგაცეებსsmile.gif

Posted by: avicena 2 Nov 2007, 03:54
ჩემო ვანო თუ დავიდან დასმულ შეკითხვას ყურადღებით წაიკითხავ " გულის დაავადებებიდან რამ შეძლება სიცხეები მოგცესო " და არა ზოგადად რამ შეიძლება სიცხეები მოგცესო. თქარათქურს ზებრებიც მოგცემს, ხარებიც, ძროხებიც და საერთოდ ბევრი მსხვილფეხა რქოსანი საქონელი, მაგრამ საუბედუროდ მე არ მაქვს ჩემო ვანო ამ საინტერესო ცხოველებთან ურთიეთობის ასეთი ნატიფი გამოცდილება რომ თქარუნის მიხედვიით ზოოლოგიური სახეობა ამოვიცნო(: ისე დიდი ხანია ამ პროფილითაც რაც დაინტერესდი?
* * *
მე ასე მგონია რომ შენ ცოტა ხანში ყველაფერს მუსიკოსის და არა ექიმის ყურით დაუწყებს სმენას, რატომაც არა, ზოგიერთი პნევმონიიანი ავადმყოფის მოსმენა ბევრი
თნამედროვე კომპოზიტორის მუსიკის მოსმენაზე ბევრად უფრო შედეგიანი შეიძლება აღმოჩნდეს უტილიტარიანისტული თვალსაზრისითsmile.gif

Posted by: BadbadGirl 24 Nov 2007, 00:39
მოკლეთ აი ასეთი რამე მჭირს. უკვე რამოდენიმე ტვეა , დილაობით საშIნელუ გულის რევის შეგრძნება მაქვს და თავბრუ მესხმის.
ორსულად არ ვარ, არაფერი არ მტკივა . მიზეზი არ ვიცი რააა + ექომთან არ მინდა წასვლა.

იქნებ აქ მირჩიოთ რა ვქნა??
წინასწარ მადლობა smile.gif

Posted by: vano_t 24 Nov 2007, 10:07
QUOTE (BadbadGirl @ 24 Nov 2007, 00:39 )
მოკლეთ აი ასეთი რამე მჭირს. უკვე რამოდენიმე ტვეა , დილაობით საშIნელუ გულის რევის შეგრძნება მაქვს და თავბრუ მესხმის.
ორსულად არ ვარ, არაფერი არ მტკივა . მიზეზი არ ვიცი რააა + ექომთან არ მინდა წასვლა.

იქნებ აქ მირჩიოთ რა ვქნა??
წინასწარ მადლობა smile.gif

ექიმთან მისვლა აუცილებელია იმიტომ კი არა, რომ რაიმე სერიოზული უნდა იყოს მაინცდამაინც, არამედ იმიტომ რომ შენი სიმპტომები იმდნენად არასპეციფიურია, რომ თითქმის ყველა ორგანოთა სისტემის დაავადებებს შეიძლება ახლდეს. არათუ საჭიროა ექიმის ნახვა, არამედ ექიმმა (პირველად თერაპევტის მსგავსი პროფილის ექიმი უნდა ნახო) პირველ ვიზიტზე დაქვრილებით უნდა გამოგკითხოს ყველაფერი და დაწვრილებით უნდა გაგსინჯოს. ამასთანავე, პირველ ვიზიტზე რამოდენიმე ლაბორატორიული ტესტიც დაგჭირდება. ასე რომ, აუცილებლად ნახე ექიმი და ინტერნეტიდან მაგაზე რჩევას ვერავინ მოგცემს.

Posted by: katrina 3 Dec 2007, 20:04
2 რამ, მაინტერესებს:
1.საიდან შეიძლება მოხვდეს ჯორჯალში ჰაერი????
2.ფილტვის ატელექტაზმა რამდენად სცრაფად შეიძლება გამოიცვიოს სიკვდილი

წინასწარ მადლობა smile.gif

Posted by: vano_t 4 Dec 2007, 01:28
QUOTE (katrina @ 3 Dec 2007, 20:04 )
2 რამ, მაინტერესებს:
1.საიდან შეიძლება მოხვდეს ჯორჯალში ჰაერი????
2.ფილტვის ატელექტაზმა რამდენად სცრაფად შეიძლება გამოიცვიოს სიკვდილი

წინასწარ მადლობა smile.gif

1. ჯორჯალის ლათინურ შესატყვისს თუ მეტყვი, შევეცდები დაგეხმარო. ის მახსოვს რომ ჯორჯალი პერიტონეუმის რომელიღაც ნაწილს ქვია. თან ისიც მითხარი როგორ დაადგინეს ჯორჯალში ჰაერი.

2. ატელექტაზი სიკვდილს არ იწვევს. ატელქტაზი ძალიან ხშირი მოვლენაა, ძალიან ხშირი. ატელექტაზი ფილტვების ქვედა წილებში წოლის დროსაც შეიძლება ჩამოყალიბდეს. ატელექტაზი უბრალოდ ნიშნავს ალვეოლების "ჩაფუშვას" რაც ძირითადად იმის გამო ხდება რომ ჰაერის მოძრაობა წყდება სასუნთქ გზებზე ზეწოლის გამო. ზეწოლის(და შესაბამისად ობსტრუქციის) დისტალურ უბნებში ჰაერის გაწოვა ხდება და ფილტვი "იფუშება". ატელექტაზმა პრობლემა რომ შექმნას, ძირითად ბრონქებს უნდა დააწვეს და მთელი ფილტვის ან ფილტვის დიდი ნაწილის ჩაჩუტვა უნდა გამოიწვიოს. არც ამ შემთხვევაშია მოსალოდნელი სიკვდილი, თუ ადამიანს მეორე ფილტვი ნორმასთან ახლოს უმუშავებს. ატელექტაზის ღირებულება ის არის, რომ ატელექტაზი გეუბნება ფილტვებში რაღაც პროცესი მიმდინარეობს, რომელიც იწვევს სასუნთქი გზების ობსტრუქციას.

Posted by: mtvareuli 4 Dec 2007, 01:31
vano_t
QUOTE
ჯორჯალის ლათინურ შესატყვისს თუ მეტყვი

mesenterium smile.gif

Posted by: vano_t 4 Dec 2007, 04:54
mtvareuli
QUOTE
mesenterium smile.gif

დიდი მადლობა smile.gif

კატრინა
თუ გაიხსენებ, ანატომიურად ჯორჯალი გარს აკრავს ნაწლავებს თითქმის ყველა მხრიდან და ერთ პატარა ადგილას ჯორჯლის ფურცლები ერთმანეთს ხვდება და ქმნის მაგ წარმონაქმნს. აქედან გამომდინარე, ნაწლავის პათოლოგიამ, რომელმაც შეიძლება გამოიწვიოს ნაწლავის პერფორაცია, შეიძლება მოგცეს ნაწლავური ჰაერის გაჟონვა ჯორჯალში. ნებისმიერი სახის ნაწლავურმა დაავადებამ შეიძლება მოგცეს პერფორაცია ნაწლავისა, თუ დაავადება მძიმედ მიმდინარეობს. გარდა ამისა გამჭოლმა ტრამვებმაც, ნაწლავის დაზიანების გარეშე ან ნაწლავის დაზიანებით, შეიძლება მოგცეს ატმოსფერული (ან ნაწლავური ჰაერის, თუ ნაწლავიც დაზიანდა ტრამვის დროს) შესვლა ჭრილობიდან ჯორჯალში. თეორიულად ფილტვის(ან ბრონქის ან საყლაპავის და კუჭის) ჰაერმაც შეიძლება მოგცეს ჰაერის მოხვედრა ჯორჯალში. პრაქტიკაში რამდენად ხდება ეგ არ ვიცი. თეორიულად, ასევე, პერიტონიუმის ჰაერის (პნევმოპერიტონიუმი) შეწოვაც შეიძლება მოხდეს ჯორჯალში. პნევმოპერიტონიუმის მიზეზი კი უბრალოდ ჩვეულებრივი ან ლაპაროსკოპიული ქირურგიული ჩარევა შეიძლება იყოს ყოველგვარი გართულებების გარეშე.

ბოლო ჯამში, ავადმყოფის მდგომარეობაზეა ალბათ დამოკიდებული ეს ჰაერი სერიოზული მიზეზებით მოხვდა იქ თუ არა. ავადმყოფის ისტორიაც ბევრ ინფორმაცია მოგცემს მაგაზე. თუ მაგალითად ავადმყოფმა ტრავმა მიიღო, მაშინ სერიოზულ ქირურგიულ დაზიანებაზე იფიქრებ; ქირურგიული ჩარევის შემდეგ განვითარებული ჰაერი, შეიძლება მიუთითოს ნაწლავის შემთხვევით პერფორაციაზე ან უბრალოდ ქირურგიის დროს პერიტონეუმში შესული ატმოსფერული ჰაერის შეწოვაზე ჯორჯალში. ნუ ავადმყოფი თუ სერიოზულად ავად არის (მაგალითად სეპტიურია), მაშინ რა თქმა უნდა პერფორაციაზე და სერიოზულ დაავადებებზე უნდა იფიქრო და ასე შემდეგ.

Posted by: DATATUTASHXIA 4 Dec 2007, 14:38
ვინმეს შეუძლია მითხრას ხერხემლის გამრუდება რამდენად საშიში პრობლემა?
ან იკურნება თუ არა
ამ ბოლო დროს ძალიან შემაწუხა...
გამრუდება არის S ის ფორმის, ანუ მხრებში მოხრილი არა ვარ, უბრალოდ ძალიან ხშირად მაქვს ტკივილები ხერხემლის არეში და თუ ძალიან გამიძლიერდა ტკივილი სუნთქვაც მიძნელდება...
იმის დრო არა მაქვს რომ სამუშო საათებში, კვირის განმავლობაში მასაჟებზე ვიარო, ან ექიმტან მაინც მივიდე და იქნებ ვარჯიში ან ფიტნესი, ან რამე ამდაგვარი თუ შემიმსუბუქებს დროებით ტკივილებს...
რა ასაკში შეიძლება ამ დაავადების შველა, ანუ თუ შეიძლება რომ საერთოდ გასწორდეს ხერხემალი (23 წლის ვარ)


კიდევ ერთი რამ მაინტერესებს...
შეიძლება დაღლილობამ გამოიწვიოს თავისტკივილი? გამუდმებით (დაახოებით) 15 წლის ასაკიდან მტკივა ძალიან ხშირად მხოლოდ ერთი მხარე თავის, მარჯვენა მხარე და ტკივილის დროს მარჯვენა თვალიში მხედველიბა მიქვეითდება ოდნავ. ანუ რაღაც ბინდის მაგვარი მიჩნდება და ცრემლი მომიდს.
შეიძლება ეს უბრალოდ გადაღლლილობის ბრალი იყოს? თუ მივიდე ექიმთან. ტკივილი გამაყიჩებელზე რეაგირებს, მაგრამ იშვიათად საერთოდ არ ყუჩდება არანაირი გამაყუცენლით სანამ თავისით არ გაივლის.

ხოდა თუ ძალიან რთული შეკითხვები არ დავსვი იქნებ მიპასუხოს ვინმემ smile.gif

Posted by: vano_t 4 Dec 2007, 23:45
QUOTE (DATATUTASHXIA @ 4 Dec 2007, 14:38 )
ვინმეს შეუძლია მითხრას ხერხემლის გამრუდება რამდენად საშიში პრობლემა?
ან იკურნება თუ არა
ამ ბოლო დროს ძალიან შემაწუხა...
გამრუდება არის S ის ფორმის, ანუ მხრებში მოხრილი არა ვარ, უბრალოდ ძალიან ხშირად მაქვს ტკივილები ხერხემლის არეში და თუ ძალიან გამიძლიერდა ტკივილი სუნთქვაც მიძნელდება...
იმის დრო არა მაქვს რომ სამუშო საათებში, კვირის განმავლობაში მასაჟებზე ვიარო, ან ექიმტან მაინც მივიდე და იქნებ ვარჯიში ან ფიტნესი, ან რამე ამდაგვარი თუ შემიმსუბუქებს დროებით ტკივილებს...
რა ასაკში შეიძლება ამ დაავადების შველა, ანუ თუ შეიძლება რომ საერთოდ გასწორდეს ხერხემალი (23 წლის ვარ)


კიდევ ერთი რამ მაინტერესებს...
შეიძლება დაღლილობამ გამოიწვიოს თავისტკივილი? გამუდმებით (დაახოებით) 15 წლის ასაკიდან მტკივა ძალიან ხშირად მხოლოდ ერთი მხარე თავის, მარჯვენა მხარე და ტკივილის დროს მარჯვენა თვალიში მხედველიბა მიქვეითდება ოდნავ. ანუ რაღაც ბინდის მაგვარი მიჩნდება და ცრემლი მომიდს.
შეიძლება ეს უბრალოდ გადაღლლილობის ბრალი იყოს? თუ მივიდე ექიმთან. ტკივილი გამაყიჩებელზე რეაგირებს, მაგრამ იშვიათად საერთოდ არ ყუჩდება არანაირი გამაყუცენლით სანამ თავისით არ გაივლის.

ხოდა თუ ძალიან რთული შეკითხვები არ დავსვი იქნებ მიპასუხოს ვინმემ smile.gif

ხერხემლი გამრუდების რამოდენიმე სახე არსებობს. შენ რასაც ამბობ სქოლიოზს გავს(არის კიდევ ე.წ. გადაჭარბებული კიფოზი და ლორდოზი). როგორც წესი ხერხემლის გამრუდებამ არ უნდა მოგცეს სუნთქვის პრობლემა თუ ძალიან გამრდუებული არ არის. გამრუდების ხარისხს ადგენს რენტგენოლოგი ან ორთოპედი ქირურგი. და მაგის მკურნალობაც მთლიანად მაგათი საქმე.

რაც შეეხება თავის ტკივილებს, რასაც შენ აღწერ გავს კლასტერულ ტკივილს. თუმცა შესაძლებელია მიგრენიც იყოს. მკურნალობის რამოდენიმე ფორმა არსებობს. კლასეტრული ტკივილი შეიძლება შეამცირო ჟანგბადით, ხანმოკლე სტეროიდული საშუალებებით, ანთების საწინააღმდეგო არასტეროიდული საშუალებებით და სხვა წამლებით. როცა თავის ტკივილი ამდენი წელია გაქვს, მაშინ რაიმე სერიოზულის შანსი ძაან ცოტაა. ექიმთან უნდა მიხვიდე და დაგინიშნავს წამლებს. თერაპევტიც გამოგადგება(თუ ესმის საკითხი) და ნევროპათოლოგიც.
ისე ტკივილები გამუდმებით გაქვს თუ რაღაც პერიოდების განმავლობაში და რამდენი ხანი გრძელდება ტკივილები? ხმაური ან სინათლე თუ გიძლიერებს ტკივილს? რა ხსნის ტკივილის?

Posted by: DATATUTASHXIA 5 Dec 2007, 04:54
vano_t
QUOTE
ისე ტკივილები გამუდმებით გაქვს თუ რაღაც პერიოდების განმავლობაში და რამდენი ხანი გრძელდება ტკივილები? ხმაური ან სინათლე თუ გიძლიერებს ტკივილს? რა ხსნის ტკივილის?

კარგი დასვენება მიხსნის ტკივილს
ანუ თუ დილიდან საღამომდე კომპთან არ ვზივარ და + ნორმალურადაც ვიკვებები არანაირი თავის ტკივილი არ მაწუხებს smile.gif

QUOTE
ხერხემლი გამრუდების რამოდენიმე სახე არსებობს. შენ რასაც ამბობ სქოლიოზს გავს(არის კიდევ ე.წ. გადაჭარბებული კიფოზი და ლორდოზი). როგორც წესი ხერხემლის გამრუდებამ არ უნდა მოგცეს სუნთქვის პრობლემა თუ ძალიან გამრდუებული არ არის. გამრუდების ხარისხს ადგენს რენტგენოლოგი ან ორთოპედი ქირურგი. და მაგის მკურნალობაც მთლიანად მაგათი საქმე.

დიდი მადლობა პასუხისთვის

Posted by: vano_t 6 Dec 2007, 03:47
QUOTE (DATATUTASHXIA @ 5 Dec 2007, 04:54 )
კარგი დასვენება მიხსნის ტკივილს
ანუ თუ დილიდან საღამომდე კომპთან არ ვზივარ და + ნორმალურადაც ვიკვებები არანაირი თავის ტკივილი არ მაწუხებს smile.gif

ეგრე მხოლოდ ჯანმრთელ ადამიანებს ემართებათ. თუ მარტო სტრესი გიშვები მაგას და იცი ზუსტად რა იწვევს სტრესს, მაშინ უნდა გამორიცხო სტრესის გამომწვევი გარემოება. შეამცირე კომპთან ჯდომა smile.gif

Posted by: Tami28 7 Dec 2007, 11:26
vano_t

გამარჯობათ, გთხოვთ მირჩიოთ რა სახის გამოკვლევები უნდა ჩავიტარო წელიწადში ორჯერ, რომ მუდმივად ვაკონტროლო საკუთარი ჯანმრთელობა. რა სახის გამოკვლევები არის აუცილებელი? (სისხლის ანალიზი, ულტრასონოგრაფია და ა.შ) იქნებ კონკრეტულად მითხრათ ამის შესახებ.

მადლობა წინასწარ.

Posted by: vano_t 8 Dec 2007, 11:40
Tami28
QUOTE
გამარჯობათ, გთხოვთ მირჩიოთ რა სახის გამოკვლევები უნდა ჩავიტარო წელიწადში ორჯერ, რომ მუდმივად ვაკონტროლო საკუთარი ჯანმრთელობა. რა სახის გამოკვლევები არის აუცილებელი? (სისხლის ანალიზი, ულტრასონოგრაფია და ა.შ) იქნებ კონკრეტულად მითხრათ ამის შესახებ.

მადლობა წინასწარ.

რა სახის ლაბორატორიები უნდა გაიკეთო დამოკიდებულია სქესზე, ასაკზე, რაიმე დაავადების არსებობაზე, გარკვეული წამლების მიღებაზე და ოჯახურ ისტორიაზე(ანუ გენეტიკურ დაავადებებზე, რომელიც განსაკუთრებული სიხშირით ხდება ადამიანში).

თუ ჯანმრთელია ადამიანი, მხოლოდ ე.წ. სკრინინგ ტესტები უკეთდება და მეტი არაფერი, ისიც წელიწადში ერთხელ ან უფო იშვიათად. როგორც წესი, ასეთ ტესტებს მიეკუთვნება სისხლში შაქრის შემცველობის გაზომვა წელიწადში ერთხელ (შაქრიანი დიაბეტის ადრეული გამოვლინებისათვის), ქოლესტერინის შემცველობა სისხლში (ახალგაზრდებში, რომელსაც ჰიპერლიპიდემიის-ანუ მაღალი ცხიმების-რაიმე განსაკუთრებული გენეტიკური დაავადება თუ არის ნათესავებში, მაშინ წელიწადში ერთხელ) 3 წელიწადში ერთხელ. ახლაგაზრდა ქალებში აკეთებენ პაპინოკოლაუს ტესტს (ეს სისხლის ტესტი არ არის, არამედ გინეკოლოგიური გასინჯვისას ქალებს უღებენ "ნაცხს" და ეს ტესტი ტარდება საშვილოსნოს ყელის კიბოს ადრეული სტადიის გამოსავლენად) წელიწადში ერთხელ. 40 (ან 50) წელს გადაცილებულ ადამიანებში აკეთებენ კოლონოსკოპიას(მსხვილი ნაწლავის კიბოს ადრეულ გამოსავლენად) და 40 (ან 50) წელს გადაცილებულ მამაკაცებში აკეთებენ პროსტატის მანუალურ გამოკვლევას და სისხლის ტესტს PSA-ს(პროსტატის კიბოს ადრეული სტადიების გამოსავლენად).

Posted by: la cioccolata calda 11 Dec 2007, 22:42
ისა ჰაი გამარჯობათ!
ჰეპატიტის აცრები სად ჩავიტარო და რა ღირს? smile.gif

Posted by: vano_t 12 Dec 2007, 06:02
QUOTE (la cioccolata calda @ 11 Dec 2007, 22:42 )
ისა ჰაი გამარჯობათ!
ჰეპატიტის აცრები სად ჩავიტარო და რა ღირს? smile.gif

კაი გამარჯობა შენი.

საქართველოში რა აცრები სად კეთდები ვერ გეტყვი, არ ვიცი. სხვამ უნდა დაგეხმაროს მაგ ამბავში.

Posted by: Cor-toni 12 Dec 2007, 20:44
QUOTE
ჰეპატიტის აცრები სად ჩავიტარო


ინფექციურში, პავლოვზე.

Posted by: Tami28 13 Dec 2007, 12:40
vano_t

გამარჯობა,

რა ინფორმაცია გაქვთ NOVOGEN-ის შესახებ? მაინტერესებს რამდენად ეფექტურია ავთვისებიანი ნაწალვური სიმსივნის (ღვიძლის მეტასტაზით) დროს . როგორ ხდება მისი გამოყენება და ხელს ხომ არ უშლის ქიმიოთერაპიის მკურნალობას.

მადლობთ

Posted by: vano_t 16 Dec 2007, 10:09
QUOTE (Tami28 @ 13 Dec 2007, 12:40 )
vano_t

გამარჯობა,

რა ინფორმაცია გაქვთ NOVOGEN-ის შესახებ? მაინტერესებს რამდენად ეფექტურია ავთვისებიანი ნაწალვური სიმსივნის (ღვიძლის მეტასტაზით) დროს . როგორ ხდება მისი გამოყენება და ხელს ხომ არ უშლის ქიმიოთერაპიის მკურნალობას.

მადლობთ

მაგ წამალი არ გამიგია. დავგუგლე რომ გამეგო რა წამალი იყო, მარა როგორც ჩანს ნოვოგენი ფირმის სახელია, რომელიც ძირითადად OTC (Over The Counter) წამლებს უშვებს. თუ წამალი მართლა არსებობს, მაშინ გენერიული სახელი მითხარი ან შემადგენლობა და შევეცდები გიპასუხო კიხვაზე.

Posted by: Tami28 18 Dec 2007, 12:41
vano_t

გამარჯობა,

ქიმიოთერაპიის გამო პაციენტს საგრძნობლად დაეწია ლეიკოციტებისა და ნეიტროფილების რაოდენობა. (ლეიკოციტებია 2,7; ნეიტროფილები 56 % (segments 55%, banas 1%)) თუ შეიძლება მომწეროთ რა საშუალებებით შეიძლება მათი მოპმატება. მაინტერესებს ასევე ბუნებრივი საშუალებებით ( კვება, პროდუქტები და ა.შ)
მადლობთ.

Posted by: vano_t 18 Dec 2007, 20:10
QUOTE (Tami28 @ 18 Dec 2007, 12:41 )
vano_t

გამარჯობა,

ქიმიოთერაპიის გამო პაციენტს საგრძნობლად დაეწია ლეიკოციტებისა და ნეიტროფილების რაოდენობა. (ლეიკოციტებია 2,7; ნეიტროფილები 56 % (segments 55%, banas 1%)) თუ შეიძლება მომწეროთ რა საშუალებებით შეიძლება მათი მოპმატება. მაინტერესებს ასევე ბუნებრივი საშუალებებით ( კვება, პროდუქტები და ა.შ)
მადლობთ.

2.7 მაინც საკმარისი რაოდენობის ლეიკოციტია, რომ სპონტანური ინფექცია არ განვითარდეს. მაგ დროს ალბათ შეიძლება თვალყურიც ადევნო ლეიკოციტებს(თუმცა ეგ ონკოლოგის გადასაწყვეტია) ყოველდღიურად და თუ ლეიკოციტების ვარდნა გრძელდება მერე გაუკეთო ე.წ. გრანულოციტების კოლონიის მასტიმულირებელი ფაქტორები(G-CSF), რომელთა რიცხვსაც მიეკუთვნება მაგალითად ნეულასტა და ნეუპოგენი(neulasta, neupogen). თუმცა ამ წამლების ფირმული დასახელება შეიძლებ განსხვავებული იყოს სხვადასხვა ქვეყნებში. ზემო სახელებით გამოდის ეგ წამლები აშშ-ში.

განსაკუთრებული ბუნებრივი საშუალებები არ ვიცი მაგისათვის. თუმცა კვება (მომატებული რაოდენობით ალბათ პროტეინების, ცხიმების, ნახშირწყლების და ვიტამინების მიღება) აუცილებელი კომპონენტია ძვლის ტვინის ნორმალური ფუნქციონირებისათვის.

Posted by: Mkvdari_Mgeli 21 Dec 2007, 13:41
დენს უერტყმეიენებეჰ...
აღარ შემეძლეაჰ რკენეს საგანს მოუკედებ თუ არა ხელს ნაპერწკლებს უყრეჰ...
მასწაულედ რა უნდა უქნაჰ....
ემ დღეს მობელურს უტუმბაუდეჰ და ხელშეჰ აუეღეჰ დენეს ტალღაჰ უეგძენეჰ და მტუმბაუეჰ აფეთქდაჰ...

help

Posted by: mtvareuli 21 Dec 2007, 21:07
Mkvdari_Mgeli
აქ ნორმალურად რომ წერო არ შეიძლება? smile.gif შეიძლება ვერ გაიგონ რას ეკითხები

Posted by: shanyva 21 Dec 2007, 21:10
mtvareuli
დენს ვარტყამო, აი კლავიატურაც ალბათ დენდარტყმული აქვს და ვერ წერს კარგად.... biggrin.gif

Posted by: vano_t 21 Dec 2007, 23:34
QUOTE (Mkvdari_Mgeli @ 21 Dec 2007, 13:41 )
დენს უერტყმეიენებეჰ...
აღარ შემეძლეაჰ რკენეს საგანს მოუკედებ თუ არა ხელს ნაპერწკლებს უყრეჰ...
მასწაულედ რა უნდა უქნაჰ....
ემ დღეს მობელურს უტუმბაუდეჰ და ხელშეჰ აუეღეჰ დენეს ტალღაჰ უეგძენეჰ და მტუმბაუეჰ აფეთქდაჰ...

help

ეგ მედ პრობლ არ არ. ეგ ელექტრომანტიორ პრობლ არ. smile.gif

სტატიკური ელექტროობა გაქვს აკრეფილი ბლომად. მაგას რამდენიმე რჩევა უშველის. არ ატარო სინთეზური ტანსაცმელი და მატყლის ტანსაცმელი(ეგენი ელექტრონებს ან გასცემენ ან იძენენ და კარგად ინახავენ სტატიკურ ელქტროობას). აარიდე რეზინის ძირიან ფეხსაცმელებს თავი. სახლში, თუ გაქვს საშუალება ოთახის დამატენიანებელი მოწყობილობა იხმარე, რომ ტენიანობა გაზარდო(ღია წყლით სავსე ჭურჭელიც შეგიძლია სახლში დადო). სპეციალური სამაჯურებიც იყიდება სტატიკური ელექტროობისათვის(კომპიტერების მაღაზიაში შეიძლება ქონდეთ)

და ყველაზე მნიშვნელოვანი თხოვნა. თვალები კარგად არ მიჭრის და ცოტა გასარკვევად დამიწერე თორემ ცრემლი წამომივიდა ბევრი და დიდ ხანს ვერ გავარჩიე რისი თქმა გინდოდა. შეგვიბრალე ქართველები smile.gif

Posted by: Bivrili 27 Dec 2007, 13:45
დამეხმარეთ რა ექიმებო : რას ნიშნავს ლიმფა ?

და კიდევ სად შეიძლება გაკეთდეს სისხლის უტყუარი ანალიზი ყველა მახასიეთებლებით რაც შედის იმ ანალიზში?................

Posted by: dato82 27 Dec 2007, 13:51
გამარჯობათ,ფილტვის ჰამენ-რიჩის სინდრომზე შეგიძლიათ მითხრათ რამე?მაგალითად რა სიმპტომები აქვს,დიაგნოსტირება როგორ ხდება,მკურნალობის მეთოდები და ა.შ. რეფერატი მაქვს დასაწერი და ლიტერატურაც თუ იცით რამე,სადაც ამ სინდრომზე რამეს ვიპოვი,მიმითითეთ რა? წინასწარ მადლობა.

Posted by: vano_t 28 Dec 2007, 00:19
dato82
QUOTE
გამარჯობათ,ფილტვის ჰამენ-რიჩის სინდრომზე შეგიძლიათ მითხრათ რამე?მაგალითად რა სიმპტომები აქვს,დიაგნოსტირება როგორ ხდება,მკურნალობის მეთოდები და ა.შ. რეფერატი მაქვს დასაწერი და ლიტერატურაც თუ იცით რამე,სადაც ამ სინდრომზე რამეს ვიპოვი,მიმითითეთ რა? წინასწარ მადლობა.

http://wrongdiagnosis.com/medical/hamman_rich_syndrome.htm
რეფერატის დაწერაში ვერ დაგეხმარები. მაგ საიტზე ბევრი წყაროა კიდევ.

Bivrili
QUOTE
დამეხმარეთ რა ექიმებო : რას ნიშნავს ლიმფა ?

და კიდევ სად შეიძლება გაკეთდეს სისხლის უტყუარი ანალიზი ყველა მახასიეთებლებით რაც შედის იმ ანალიზში?................
ლიმფურ სისტემაში მოცირკულირე სითხეს ქვია ლიმფა.

სად კეთდება ანალიზები "კარგად", ეგ არ ვიცი.

Posted by: kuchu 29 Dec 2007, 22:34
ერთ დღეში იდაყვზე კანი გამიღიზიანდა , თითქოს დაფუფქულიაო, მექავება და მტკივა, რასთან შეიძლება იყოს დაკავშირებული? : ( ჰერპესთან რაიმე კავშირი შეიძლება რომ ჰქონდეს?

Posted by: dato82 7 Jan 2008, 16:44
vano_t
დიდი მადლობა,ჰამენ რიჩის სინდრომზე ვიპოვე რაც მინდოდა..შობა-ახალ წელს გილოცავ!

Posted by: Natosha 7 Jan 2008, 23:05
ასეთი შეკითხვა მაქვს, რეინოს სინდრომზე ვწერ რეფერატს, გადავათვალიერე სხვა და სხვა საიტებზე დადებული მასალა. ერთი რამე ვერ გავარკვიე ბოლომდე რეინოს სინდრომი და რეინოს ფენომენი ერთი და იგივეა? თუ არა რა განსხვავებებია.
ზოგი სტატიიდან შეიძლება იფიქრო რომ ზოგადი მოვლენა არის რეინოს ფენომენი, პირველადი რეინო არის რეინოს დაავადება(raynaud's disease), ხოლო მეორეული (ანუ დაავადებას რომ ახლავს) რეინოს სინდრომი. ზოგ სტატიაში პირიქით გამოდის, ხოლო ზოგში შეიძლება იფიქრო რომ ერთი და იგივეა. დავიბენი user.gif

მაგალითები:

"Raynaud phenomenon (secondary Raynaud) should be distinguished from Raynaud disease (primary Raynaud). They are distinct disorders that share a similar name. Raynaud disease is the occurrence of the vasospasm alone, with no association with another illness. Raynaud phenomenon is usually used in the context of vasospasm associated with another illness, most commonly an autoimmune disease. Other terms used for this distinction are primary Raynaud (disease) and secondary Raynaud (phenomenon). "
დასკვნა ფენომენი არის მეორეული, დაავადება პირველადი.(სინდრომი საერთოდ არ არის ნახსენები. სტატია emedicine.com-იდან)

"Рейно синдром (М. Raynaud, франц. врач, 1834—1881) — свойственная некоторым болезням особая форма ангиодистонии, характеризующаяся приступообразной локальной ишемией кистей (обычно пальцев), иногда стоп, реже других участков тела."
აქედან გამოდის რომ სინდრომი არის მეორეული რეინო(რომლადაც ზევით მოიხსენიებოდა რეინოს ფენომენი). სტატია რუსული მედ.საიტიდან.

"Raynaud’s syndrome (RS) can be a debilitating condition that causes periods of severely restricted blood flow to the fingers and toes (and sometimes to other parts of the body such as the nose or ears). In the worst case scenario, this can result in amputation of the damaged digits."
აქ: რეინოს სინდრომი არის მდგომარეობა, რომლის დროსაც .......
შეიძლება იყოს პირველადი და მეორეული. საიტი http://www.lef.org/protocols/heart_circulatory/raynauds_syndrome_01.htm (რამდენად სანდო წყაროა არ ვიცი).

წიკიპედია:
"Raynaud's phenomenon (RAY-noz), in medicine, is a vasospastic disorder causing discoloration of the fingers, toes, and occasionally other extremities, named for French physician Maurice Raynaud (1834 - 1881). The cause of the phenomenon is unknown, but emotional stress and cold are classically triggers, and the discoloration follows a characteristic pattern in time: white, blue and red. It comprises both Raynaud's disease (primary Raynaud's), where the phenomenon is idiopathic, and Raynaud's syndrome (secondary Raynaud's), where it is secondary to something else."
ფენომენი-მოვლენა. დაავადება- პირველადი რეინო, სინდრომი - მეორეული რეინო.


ვეღარ გავიგე მოკლედ sad.gif




Posted by: donvaso 7 Jan 2008, 23:17
სხვადასხვაა ......... wink.gif
რეინოს სინდრომი(გულმკერდის გამოსვლის სინდრომი) მოიცავს
რეინოს:
-ავადმყოფობა
-განგრენა
-ფენომენი(მეორადი).
უხშირესად გვხდება ქალებში(ჩემი ნაცნობი ორევე ქალია), დაზიანება სიმეტრიულია და წარმოადგენს ანგიოტროფონეკროზს კაპილარების დაზიანებით


http://forum.ge/?f=43&showtopic=33681970&st=60

Posted by: Natosha 7 Jan 2008, 23:21
donvaso
წაკითხული მაქვს ეგ თემა smile.gif

QUOTE
-განგრენა

ეგ ცალკე არის გამოყოფილი? spy.gif მე ვიცი, როგორც ერთ-ერთი შედეგი..

Posted by: donvaso 7 Jan 2008, 23:27
Natosha
რავი, ამ თემაზე ვცადე რაღაცეების გაგება, მაგრამ ბევრი ვერაფერი გავიგე ქართველი ექიმების ხელში....wink.gif

P.S. თუ რამე გამოგივა და იმედია გამოგივა, თუ პრობლემა არ იქნება წამაკითხე შენი თემა..... smile.gif

Posted by: Natosha 7 Jan 2008, 23:29
donvaso
QUOTE
თუ პრობლემა არ იქნება წამაკითხე შენი თემა.....

biggrin.gif თუ დროის ფაქტორის გამო, ვალდებულების მოხდის მიზნით დაწერილი თემა არ გამომივა წაგაკითხებ smile.gif

Posted by: vano_t 8 Jan 2008, 02:20
QUOTE (Natosha @ 7 Jan 2008, 23:05 )
ასეთი შეკითხვა მაქვს, რეინოს სინდრომზე ვწერ რეფერატს, გადავათვალიერე სხვა და სხვა საიტებზე დადებული მასალა. ერთი რამე ვერ გავარკვიე ბოლომდე რეინოს სინდრომი და რეინოს ფენომენი ერთი და იგივეა? თუ არა რა განსხვავებებია.
ზოგი სტატიიდან შეიძლება იფიქრო რომ ზოგადი მოვლენა არის რეინოს ფენომენი, პირველადი რეინო არის რეინოს დაავადება(raynaud's disease), ხოლო მეორეული (ანუ დაავადებას რომ ახლავს) რეინოს სინდრომი. ზოგ სტატიაში პირიქით გამოდის, ხოლო ზოგში შეიძლება იფიქრო რომ ერთი და იგივეა. დავიბენი user.gif

მაგალითები:

"Raynaud phenomenon (secondary Raynaud) should be distinguished from Raynaud disease (primary Raynaud). They are distinct disorders that share a similar name. Raynaud disease is the occurrence of the vasospasm alone, with no association with another illness. Raynaud phenomenon is usually used in the context of vasospasm associated with another illness, most commonly an autoimmune disease. Other terms used for this distinction are primary Raynaud (disease) and secondary Raynaud (phenomenon). "
დასკვნა ფენომენი არის მეორეული, დაავადება პირველადი.(სინდრომი საერთოდ არ არის ნახსენები. სტატია emedicine.com-იდან)

"Рейно синдром (М. Raynaud, франц. врач, 1834—1881) — свойственная некоторым болезням особая форма ангиодистонии, характеризующаяся приступообразной локальной ишемией кистей (обычно пальцев), иногда стоп, реже других участков тела."
აქედან გამოდის რომ სინდრომი არის მეორეული რეინო(რომლადაც ზევით მოიხსენიებოდა რეინოს ფენომენი). სტატია რუსული მედ.საიტიდან.

"Raynaud’s syndrome (RS) can be a debilitating condition that causes periods of severely restricted blood flow to the fingers and toes (and sometimes to other parts of the body such as the nose or ears). In the worst case scenario, this can result in amputation of the damaged digits."
აქ: რეინოს სინდრომი არის მდგომარეობა, რომლის დროსაც .......
შეიძლება იყოს პირველადი და მეორეული. საიტი http://www.lef.org/protocols/heart_circulatory/raynauds_syndrome_01.htm (რამდენად სანდო წყაროა არ ვიცი).

წიკიპედია:
"Raynaud's phenomenon (RAY-noz), in medicine, is a vasospastic disorder causing discoloration of the fingers, toes, and occasionally other extremities, named for French physician Maurice Raynaud (1834 - 1881). The cause of the phenomenon is unknown, but emotional stress and cold are classically triggers, and the discoloration follows a characteristic pattern in time: white, blue and red. It comprises both Raynaud's disease (primary Raynaud's), where the phenomenon is idiopathic, and Raynaud's syndrome (secondary Raynaud's), where it is secondary to something else."
ფენომენი-მოვლენა. დაავადება- პირველადი რეინო, სინდრომი - მეორეული რეინო.


ვეღარ გავიგე მოკლედ sad.gif

კლინიკურად რეინოს სინდრომი და დაავადება ერთნაირია: ანუ ორივეს ახლავს სიცივეში ხელის თითების სპეციფიური ცვლილებები(გათეთრება, გალურჯება, გაწითლება და ტკივილი). ერთი განსხვავება ის არის, რომ რეინოს სინდრომი მეორადი დაავადებაა, ანუ რაიმე სხვა დაავადების გართულებაა(უმეტესწილად რევმატოლოგიური დაავადებების, როგორიც არის დავუშვათ ლუპუსი). რეინოს დაავადებას მაშინ არქმევ რეინოს სინდრომს, როცა გამომწვევ მიზეზს(ანუ სხვა დაავადებას, მაგალითად ლუპუსს) ვერ ნახულობ.

ანუ მასეთი სიმპტომებით როცა მოდის ავადმყოფი, ჯერ ცდილობ მოძებნო აქვს თუ არა ავადმყოფს სხვა დაავადება, რომელსაც ეგ სიმპტომები შეუძლია გამოიწვიოს. თუ გამომწვევ დაავადებას ნახავ, მაშინ მაგ სიმპტომებს დაარქმევ რეინოს სინდრომს. თუ ვერ ნახულობ სავარაუდო გამომწვევ დაავადებას, მაშინ დაარქმევ სახელს რეინოს დაავადებას.

რეინოს სინდრომი და გულმკერდის გამოსვლის სინდრომი (thoracic outlet syndrome) სხვადასხვა რამ არის. გულმკერდის გამოსვლის სინდრომი შეიძლება გაძლევდეს რეინოს მსგავსს სიმტომებს, მაგრამ ეგ ერთეული გამოწვეულია სისხლძარღვების ან ნერვების ან ორივეს ერთად ზეწოლით, როცა ეს წარმონაქმნები ექცევა ლავიწის ძვალსა და პირველ ნეკნს შორის.

Posted by: ტანკე 8 Jan 2008, 02:36
მოკლედ საშინელ სტრესში ვარ,დავპანიკდი.
ახლა რა ხდება...რამდენიმე თვე იქნება,რაც მარცხენა თვალის პერიფერიული მხედველობა შემეცვალა.არ ვიცი ზუსტად როგორ,მაგრამ ვგრძნობ ისე აღარ არის,როგორც ადრე იყო.მარცხენა თვალის ოდნავი ლატერალური გადანაცვლებაც საკმარისია და საკუთარ ყვრიმალს და თვალბუდის კედლებს ვხედავ მკაფიოდ,შავად ჩანს მტელი უბანი...ადრე ასე არ იყო.არ ვიცი რა ხდება პერიფერიული მხედველობა შემეცვალა,თუ უფრო მუქად ვხედავ და ამიტომ ვეფეთები მაგ თვალით მაგ შავ უბანს.ნერვებზე ძაან მოქმედებს.მარჯვენა თვალი ძაან უნდა გადავწიო ლატერალურად,რომ იგივე უბანი დავინახო და უფრო ღია ფერებში ვხედავ(როგორც ნორმა იყოს ჩემთვის),მარცხენაში კი მუქად ვხედავ და თითქოს წამოსულია ეგ პერიფერიული უბანი ცემტრისკენ და ოდნავი მიტრიალება თვალის და ვხედავ შავად sad.gif ზოგჯერ ნერვები მაგრა ამიყვას ხოლმე მაგის გამო,მგონი ვბრმავდები.
ვცდილობდი არ მიმექცია აქამდე ყურადღრება,მაგრამ ბოლო დღეებია უკვე იგივე მხარეს-მარცხნივ ყურში მესმის საკუთარი გულისცემა sad.gif pulsatile tinnitus მაკლდა ახლა ყველა სიკეთესთან ერთად და ამან საერთოდ მანდრაჟში ჩამაგდო,ნერვები მაქვს უკვე ძაან ცუდ დღეში.ვშიშობ ანევრიზმა არ მქონდეს sad.gif კიდევ იდიოპატიურ ინტრაკრანიალურ ჰიპერტენზიაზე ვფიქრობ,მაგრამ უფრო ანევრიზმა მგონია.შეიძლება პანიკის გამო).
მარცხენა ყურში ამ გულისცმა ძირითადად ღამე დაწოლისას ვგრძნობ,როცა სიჩუმეა და ვერტიკალურიდან ჰორიზონტალრ მდგომარეობაში ვწვები.არაა მკაფიო,ჩუმად ისმის,ხრაშა-ხრუშის ხმებით,მაგრამ მაინც მოქმედებს ნერვებზე sad.gif
მოკლედ უნდა გადავიღო თავზე ტომოგრამა? მჭირდება ტომოგრაფია? სახლშიც არ ვეუბნები ჩემებს,არ მინდა დავაფეთო ამ ახალ წლებზე და მოკლედ ცუდ დღეში ვარ,უკვე აღარ ვიცი რა ვქნა sad.gif

სხვა სიმპტომები რავი,თავის ტკივილი არ მაქვს,ადრე მქონდა 2 კვირიანი თავის ტკივილი და ახლა აღარ მაწუხებს,ბავშვონიდან მაქვს ჰიპერტენზია და pulsatile tinnitus-ს მაგას დავაბრალებდი რომ არა მარცხენა თვალში პერიფერიული მხედველობის შეცვლა.მხედველობა ამ ბოლო დროს დამიქვეითდა ორივე თვალში და თვალების ტკივილი,წვაც მაწუხებს.any suggestions? ვიცი ექიმთან ვარ მისასვლელი ხო sad.gif
რის ექიმთან მივიდე ისიც არ ვიცი,ნევროპათოლოგთან,ანგიოლოგთან,ოკულისთან,თერაპევტთან თუ სად...

Posted by: Natosha 8 Jan 2008, 03:34
vano_t
smile.gif დავიბენი. ანუ, შენი ახსნა კი გასაგებია ყველაფერი. უბრალოდ, რა მაინტერესებს იცი?
აი შენ ხარ ასისტენტი, მიეცი შენს სტუდენტს თემა "რეინოს სინდრომი", რომელზეც გჭირდება რეფერატი.
ამ რეფერატში შენ რა გინდა, რომ სტუდენტმა დაგიწეროს რეინოს სინდრომის და თანმხლები დაავადებების შესახებ, თუ ამ ყველაფერს რეინოს დაავადებაც მიუმატოს?
ჩემი პრობლემა იმაშია, რომ ვერ გავარკვიე თემა რა მიმართულებით განვიხილო biggrin.gif

Posted by: vano_t 8 Jan 2008, 10:41
QUOTE (Natosha @ 8 Jan 2008, 03:34 )
vano_t
smile.gif დავიბენი. ანუ, შენი ახსნა კი გასაგებია ყველაფერი. უბრალოდ, რა მაინტერესებს იცი?
აი შენ ხარ ასისტენტი, მიეცი შენს სტუდენტს თემა "რეინოს სინდრომი", რომელზეც გჭირდება რეფერატი.
ამ რეფერატში შენ რა გინდა, რომ სტუდენტმა დაგიწეროს რეინოს სინდრომის და თანმხლები დაავადებების შესახებ, თუ ამ ყველაფერს რეინოს დაავადებაც მიუმატოს?
ჩემი პრობლემა იმაშია, რომ ვერ გავარკვიე თემა რა მიმართულებით განვიხილო biggrin.gif

გააჩნია რომელი კურსის სტუდენტი ხარ. თუ ბოლო კურსებზე ხარ, მაშინ ყურადღება უნდა გაამახვილო რეინოს სინდრომზე, მერე გამომწვევ მიზეზებზე და დიფერენციალურ დიაგნოზზე. პირველადიც (ანუ რეინოს დაავადება) და მეორადიც(ანუ როგორც სხვა დაავადების თანმხლები) უნდა ახსენო თემაში.

დასაწყისი კურსის სტუდენტმა ალბათ ყურადღება უნდა გაამახვილოს დაავადების პათოფიზიოლოგიაზე.

Posted by: donvaso 8 Jan 2008, 11:36
vano_t

QUOTE
რეინოს სინდრომი და გულმკერდის გამოსვლის სინდრომი (thoracic outlet syndrome) სხვადასხვა რამ არის. გულმკერდის გამოსვლის სინდრომი შეიძლება გაძლევდეს რეინოს მსგავსს სიმტომებს, მაგრამ ეგ ერთეული გამოწვეულია სისხლძარღვების ან ნერვების ან ორივეს ერთად ზეწოლით, როცა ეს წარმონაქმნები ექცევა ლავიწის ძვალსა და პირველ ნეკნს შორის


რავი, ჩემს დას დიაგნოზი რომ დაუსვეს ასე დაუწერეს : რეინოს სინდრომი-გულმკერდის ორმხრივი გამოსვლის სინდრომი.
და იცით რა მაინტერესებს, ჩემს დას კისრის მე7 მალის განივი მორჩი იწვევს სწორედ რომ სისხლძარღვის კომპრესიას, თითოჯერ მოაჭრეს ორივე მხარეს რამდენიმე წლის უკან(4-5) და ახლა ეუბნებიან ისევ გგაიზარდა და ისევ უნდა მოვჭრათო. არ არის რამე ისეთი მეთოდი სადმე სადაც ამ ამბავს ცოტა უფრო მარტივად მოაგვარებენ, ან თუნდაც ერთი ოპერაცით ...........
p.s. ცოტა გაუგებრად თუ დავწერე მერე დავწერ უფრო გასაგებად.... wink.gif

Posted by: SOULFUL 8 Jan 2008, 12:50
Natosha

Classified as either the primary form (Raynaud's disease, idiopathic Raynaud's phenomenon, primary Raynaud's phenomenon) when no underlying cause can be found or the secondary form (secondary Raynaud's phenomenon, Raynaud's phenomenon) which is always associated with an underlying disease or some aspect of the patient's lifestyle

http://www.mdconsult.com/das/pdxmd/body/85261490-10/660452091?type=med&eid=9-u1.0-_1_mt_1014243

Posted by: dato82 8 Jan 2008, 15:45
Natosha
აუ არ გინდა რეფერატი ჰამენ-რიჩის სინდრომზე დაწერო?...და მერე გადამაწერიოო wink.gif

Posted by: Natosha 8 Jan 2008, 19:51
ყველას მადლობა რჩევებისთვის smile.gif

dato82
QUOTE
აუ არ გინდა რეფერატი ჰამენ-რიჩის სინდრომზე დაწერო?...და მერე გადამაწერიოო

რამდენს გადამიხდი, გააჩნია tongue.gif

Posted by: dato82 8 Jan 2008, 20:01
Natosha
რამდენ ქულასაც გაქაჩავს რეფერატი,მეც იმის და მიხედვით გადავიხდი tongue.gif

Posted by: vano_t 11 Jan 2008, 08:40
donvaso
QUOTE
რავი, ჩემს დას  დიაგნოზი რომ დაუსვეს ასე დაუწერეს : რეინოს სინდრომი-გულმკერდის ორმხრივი გამოსვლის სინდრომი.
და იცით რა მაინტერესებს, ჩემს დას კისრის მე7 მალის განივი მორჩი იწვევს სწორედ რომ სისხლძარღვის კომპრესიას, თითოჯერ მოაჭრეს ორივე მხარეს რამდენიმე წლის უკან(4-5) და ახლა ეუბნებიან ისევ გგაიზარდა და ისევ უნდა მოვჭრათო. არ არის რამე ისეთი მეთოდი სადმე სადაც ამ ამბავს ცოტა უფრო მარტივად მოაგვარებენ, ან თუნდაც ერთი ოპერაცით ...........
p.s. ცოტა გაუგებრად თუ დავწერე მერე დავწერ უფრო გასაგებად.... wink.gif

რეინო და გულმკერდის გამოსვლის სინდრომი(გგს) სხვადასხვა რამეა დასავლეთის მედიცინაში. შეიძლება საქართველოში გგს-ის აღსანიშნავად რეინოც იხმარება.

რაც შეეხება მეორე აბზაცს, ყველაფერი გასაგებად დაწერე, მაგრამ თვითონ ექიმების დასკვნაა გაუგებარი. რას ქვია კისრის მე-7 მალის განივი მორჩი იწვევს სისხლძარღვზე კომპრესიას? ერთი სისხლძარღვი რომელიც კისრის მალების განივ მორჩებში (მის ხვრელებში) გადის, არის ვერტებრალური არტერია, რომელიც ზურგის ტვინს კვებავს. გგს-ის დროს ლავიწქვეშა (მე მგონი ასე ქვია subclavian artery-ის არა?) არტერიის ზეწოლა შეიძლება მოხდეს ლავიწის ძვალსა და პირველ ნეკნს შორის. თანაც ამოჭრის შემდეგ ძვლის ზრდა არ გამიგია (ნუ ოსტეოფიტების ჩამოყალიბება გასაგებია).

უბრალოდ ამიხსენი მეგობრულად რომელ არტერიას აწვებოდა კისრის მე-7 მალის განივი მორჩი, რა სიმპტომებს იწვევდა და როგორ დასვეს ზეწოლის დიაგნოზი. მე თუ ვერ გავიგე რა ხდება,ნეიროქირურგს შევეკითხები და გავიგებ რაშია საქმე.

Posted by: donvaso 11 Jan 2008, 20:11
vano_t
ახლა ვნახე ეს და აგერ.....
ჩემი დის

მოკლე ანამნეზი:
ავადმყოფი კლინიკაში შემოსვლისას უჩიოდა ორივე ზედა კიდურის გაბუჟებას, ტკივილს, ფუნქციის დაქვეითებას, ოფლიანობას.
კლინიკო-ლაბორატორიულად და ინსტრუმენტალური გამოკვლევებით დაუდგინდა დიაგნოზი: ორმხრივი გულმკერდის გამოსვლის სინდრომი, რეინოს ფენომენი. ავ. ზოგადი ენდოტრაქეალური ანესთეზიით გაუკეთდა მარცხენამხრივი სკალენოტომია, მკერდის მცირე კუნთის გადაკვეთა, C VII გადიდებული განივი მორჩის ამოკვეთა. ოპერაციის შემდგომი პერიოდი მიმდინარეობდა გართულების გარეშე. ავადმყოფის ჩივილებიდან აღშანიშნავია მარცხენა მტევნის I-II თითის გაბუჟება.
ავადმყოფი ბინაზე გაეწერა ამბულატორიული მკურნალობის ქვეშ. მიეცა შესაბამისი დანიშნულება, უგრძელდება მკურნალობა.
18.IX.2000წ.


მოკლე ანამნეზი:
ავადმყოფს აწუხებდა სიმსივნური წარმონაქმნი კისრის წინა ზედაპირზე, მარჯვენა ხელის პერიოდული დაბუჟება, ძალის დაქვეითებაზე.
პაციენტი 2000 წლის სექტემბერში მკურნალობდა ჩვენს კლინიკაში ორმხრივი გულმკერდის გამოსვლის სინდრომის გამო. ჩაუტარდა მარცხენამხრივი სკალენოტომია, მკერდის მცირე კუნთის გადაკვეთა, C VII განიცი მორჩის რეზექცია. ამა წლის 11 მარტს ენდოტრაქეალური ანესთეზიით ავ. კვანძოვანი ჩიყვის და მარჯვენამხრივი გულმკერდის გამოსვლის სინდრომის გამო ჩაუტარდა სიმულატანტური(?) ოპერაცია - ფარისებრი ჯირკვლის ორივე წილის რეზექცია, მარჯვენამხრივი სკალენოტომია, მარჯვენა მკერდის მცირე კუნთის გადაკვეთა, არტერიონევროლიზი.
ოპერაციის შემდგომი პერიოდი მიმდინარეობს დამაკმაყოფილებლად. ავ. უგრძელდება ამბულატორიული მკურნალობა.
19 მარტი 2004წ.


კისრის მალების რენტგენოგრაფია
პირდაპირ პროექციაში VII მალის მარჯვენა გვერდით მორჩთან ისახება ჩაკირული უბანი. გვერდით პროექციაში ფიზიოლოგიური ლორდოზი. C6-7 მალთაშუა სივრცე მცირედ შევიწროებული.
17.05.2006წ


ბევრი რამ ვერ ამოვიკითხე მაგრამ მაინც დავწერ:
ორივე ზემო კიდურების არტერიების სანათური თავისუფალი, ნაკადი მაგისტრალური ტიპის,
პროვოკაციული ტესტი ლავიწქვეშა არტერიებზე ორმხრივი ??????(დადებით???)?.(ძალიან გაუგებარი). მარჯვენამხრიდან 90(გრადუსი)-ზე - სრული მოჭყლეტა. მარცხმხრ. 90(გრადული)-ზე - V -300სმ/წმ, და შეესაბამება "რაღაცნაირ"ხარისხს >50%, 80(გრადუსი) - V - 210სმ/წმ .>/(ნაკლები ან ტოლი) 50%.
კიდევ რაღაც წერია მაგრამ ვერაფერი გავარკვიე.....

ხო, კიდევ აქვს: მარცხენა პარკუჭის დამატებითი ქორდა.


ახლა ეუბნებიან ქირურგები რომ ისევ უნდა გაიკეთოო ოპერაცია....... wink.gif smile.gif sad.gif

Posted by: vano_t 11 Jan 2008, 22:56
donvaso
არავითარ შემთხვევაში არ დააშვებინო მე-2 ქირურგია. ახლა სამასხურში მივდივარ და შემდეგ დაწვრილებით მოგწერ ყველაფერს PM-ში.

Posted by: donvaso 11 Jan 2008, 23:38
vano_t
ო.კ.......................... wink.gif wink.gif

Posted by: Tami28 15 Jan 2008, 17:49
vano_t

იქნებ რამე მირჩიოთ.

ორი თვის წინ გავიკეთე მარცხენა მაჯაზე ოპერაცია. მქონდა ჰიგრომა ( GANGLION). საკმაოდ დიდი. დრეს ვიყავი ექოსკოპიაზე და მემგონი რეციდივია, ანუ აღმოჩნდა სამი პატარა ჩანარი. მითხრეს, რომ ეს არის მაჯის მყესის პათოლოგია და ეს იწვევს კისტური წარმონაქმნის გაჩენას მაჯაში. ხომ არ იცით როგორ შეიძლება ამის მკურნალობა? ესე დაუსრულებლად ოპერაციები ძალიან ძნელია.
მადლობთ.

Posted by: vano_t 15 Jan 2008, 21:21
QUOTE (Tami28 @ 15 Jan 2008, 17:49 )
vano_t

იქნებ რამე მირჩიოთ.

ორი თვის წინ გავიკეთე მარცხენა მაჯაზე ოპერაცია. მქონდა ჰიგრომა ( GANGLION). საკმაოდ დიდი. დრეს ვიყავი ექოსკოპიაზე და მემგონი რეციდივია, ანუ აღმოჩნდა სამი პატარა ჩანარი. მითხრეს, რომ ეს არის მაჯის მყესის პათოლოგია და ეს იწვევს კისტური წარმონაქმნის გაჩენას მაჯაში. ხომ არ იცით როგორ შეიძლება ამის მკურნალობა? ესე დაუსრულებლად ოპერაციები ძალიან ძნელია.
მადლობთ.

ჰიგრომა და განგლიონური კისტა სხვადასხვა რამეა. ჰიგრომის კისტა ვითარდება თავთან ახლოს და შიცავს ცერებროსპინალურ (ანუ თავზურგტვინის) სითხეს. განგლიონის კისტა ვითარდება მაჯაზე და სინოვიალურ (ანუ სახსრის) სითხეს შეიცავს. შენ გქონდა განგლიონი. განგლიონი ხშირად გამეორებადია. ქირურგიის შემდეგ გამეორების სიხშირე 20-30 %-იც შეიძლება იყოს. ინტერნისტის ოფისში მარტივი პროცედურა კეთდება ჩვეულებრივ. კერძოდ, ნემსი შეყავთ განგლიონში და შიგთავსის ამოღებას (ანუ ასპირაციას) აკეთებენ და ხშირად სტეროიდსაც(ჰორმონალურ პრეპარატს) ასხამენ განგლიონის ღრუში. ასპირაციის შემდეგ განგლიონის გამეორების სიხშირე უფრო დიდია. ასპირაციას ის უპირატესობა აქვს, რომ მარტივი მეთოდია და სწრაფი, ამასთანავე უმტკივნეულო. ბოლო ბოლო განმეორებითი ასპირაციების შემდეგ როგორც წესი ქრება განგლიონი. განმეორებითი ქირურგიაც შეიძლება გაიკეთო თუ გინდა.

Posted by: ტანკე 16 Jan 2008, 00:59
vano_t
ვიყავი გამოკვლევაზე smile.gif
http://forum.ge/?f=43&showtopic=33810612

Posted by: vano_t 16 Jan 2008, 05:12
QUOTE (ტანკე @ 16 Jan 2008, 00:59 )
vano_t
ვიყავი გამოკვლევაზე smile.gif
http://forum.ge/?f=43&showtopic=33810612

მომილოცავს. ყველაფერი კარგათ არის smile.gif

სიცოცხლეს მოუკელიო ვინც გითხრას, იმას არ დაუჯერო. დღეში 24 საათი უნდა ცოცხლობდე და კვირაში 7 დღე, ანუ როგორც აშშ-ელები ამბობენ 24/7 smile.gif

Posted by: Bivrili 16 Jan 2008, 17:55
vano_t

მადლობა , გაიხარე................
QUOTE
ლიმფურ სისტემაში მოცირკულირე სითხეს ქვია ლიმფა.

და ამ სითხის (ანუ ლიმფის) ანალიზის გაკეთება შეიძლება?..................რაღაც სისულელეს თუ ვკითხულობ მომიტევეთ...................

Posted by: vano_t 17 Jan 2008, 01:40
Bivrili
QUOTE
და ამ სითხის (ანუ ლიმფის) ანალიზის  გაკეთება შეიძლება?..................რაღაც სისულელეს თუ ვკითხულობ მომიტევეთ...................

ამ სითხის ანალიზი თეორიულად (და პრაქტიკულადაც) შეიძლება, მაგრამ არავინ აკეთებს მის ანალიზს, რადგან არავითარი სადიაგნოსტიკო ღირებულება არ გააჩნია. გარდა ამისა, ამ სითხის აღება ფაქტიურად შეუძლებელია ლიმფური სადინარების სანათურის მცირე კალიბრის გამო. თვითონ ლიმფრური სითხე შეიძლება სხვადასხვა ღრუში აღმოჩნდეს (მაგალითად ფილტვების გარშემო არსებულ სივრცეში, რომელსაც პლევრის ღრუ ქვუ) და ასეთ რამის დადგენას დიაგნოსტიკური მნიშვნელობა გააჩნია. მაგალითისათვის, თუ პლევრის რღუში ლიმფური სითხე იქნა ნანახი, მაშინ ის მიუთითებს იმაზე, რომ ყველაზე დიდი ლიმფური სადინარი(გულმკერდის სადინარი) პლევრის ღრუსთან არის პათოლოგიურად დაკავაშირებული(ეს შეიძლება მოხდეს მაგალითისათვის გულმკერდის ტრამვის დროს).

Posted by: BadbadGirl 19 Jan 2008, 22:04
ისეთი სიცხე და გრიპი მაქვს თვალებიდან ცრემლები მცვივა .
რა ვქნაააააააააააააააა??/
ლიმონიანი ჩაით გავიჭყიპე , წამლებსაც ვსვავ და არაფერი მშველის sad.gif

Posted by: vano_t 20 Jan 2008, 07:21
QUOTE (BadbadGirl @ 19 Jan 2008, 22:04 )
ისეთი სიცხე და გრიპი მაქვს თვალებიდან ცრემლები მცვივა .
რა ვქნაააააააააააააააა??/
ლიმონიანი ჩაით გავიჭყიპე , წამლებსაც ვსვავ და არაფერი მშველის sad.gif

უნდა დაელოდო სანამ თავისით გაივლის. ზემო სასუნთქი გზების ინფექციები(ზსგი) ვირუსული ინფექციებია. მაგათ რაიმე წამალი არ კურნავს. ბევრი სითხე უნდა მიიღო, და სიმპტომატური მკურნალობისათვის უნდა მიიღო ურეცეპტო წამლები (ტილენოლი ან რაიმე სხვა სიცხისათვის, ხველისათვის, ყელის ტკივილსათვის და სურდოსათვისაც ადგილობრივ აფთიაქებში უნდა იკითხო რა გაიცემა რეცეპტის გარეშე).

უმრავლეს შემთხვევაში ზსგი გართულებების გარეშე მიდის. გართულებების შემთხვევაში უნდა მიაკითხო ექიმს. გართულებების სიმპტომებია სუნთქვის უკმარისობა, სინუსების (განსაკუთრებით ცალმხრივი) და ყურის ტკივილი.

Posted by: BadbadGirl 20 Jan 2008, 16:18
vano_t
მადლობა პასუხისათვის.
გართულებების სიმპტომები აერ მაქვს საბედნიეროდ.

Posted by: Lyechka 20 Jan 2008, 23:06
სიცივის დროს სისხლი რომ მომდის ცხვირიდან საშისი ხომ არაა?
ძაან მაგრად არა მაგრამ გული მიმდისავით smile.gif ხოდა ჯერ ვერ მივხვდი რატომ მერე დავაფიქსირე და ცოტა დავმშვიდდისავით
რამე უნდა ვქნა თუ პროსტო ისე გავაგრძელო დაყურადღება არ მივაქციო და გაივლის?

Posted by: vano_t 21 Jan 2008, 00:59
Lyechka
QUOTE
სიცივის დროს სისხლი რომ მომდის ცხვირიდან საშისი ხომ არაა?
ძაან მაგრად არა მაგრამ გული მიმდისავით smile.gif ხოდა ჯერ ვერ მივხვდი რატომ მერე დავაფიქსირე და ცოტა დავმშვიდდისავით
რამე უნდა ვქნა თუ პროსტო ისე გავაგრძელო დაყურადღება არ მივაქციო და გაივლის?

სიცივის დროს სისხლდენა ცხვირიდან ძალიან ხშირია. სიცივეში ცხვირის ლორწოვანი გარსი ადვილად შრება და ზედაპირული სისხლძარღვები ადვილად ზიანდება. პირველ რიგში ექიმმა (თერაპევტმა ან ყელყურცხვირის ექიმმა) უნდა დაათვალიეროს ცხვირის ლორწოვანი და დარწმუნდეს რომ სისხლდენა ხდება ცხვირის ძგიდის წინა ნაწილიდან, ე.წ. კისელბახის წნულიდან. სისხლდენათა 95 პროცენტზე მეტი აქედან ხდება. დიაგნოზის დადგენის შემდეგ გამოგიწერენ წამლებს. ამისათვის ცხვირში ხშირად უნდა იწვეთო უბრალო მარილხსნარი (ფიზიოლოგიური ხსნარიც წავა), რომელიც აფთიაქში უნდა იშოვებოდეს. 1-2 წვეთი ყოველ 2 საათში მშრალი და ცივი ამინდების დროს ლორწოვანს დაატენიანებს და სისხლდენებს შეამცირებს. სისხლდენის დროს შეგიძლია ჩაიწვეთო დეკონჯესტანტები, ცხვირში შეშუპების მომხსნელები (საქართველოში მასეთ წამალს მიეკუთვნება გალაზოლინი დ ნაფტიზინი). ასევე, სისლდენის დროს თუ ცხვირზე მოიჭერ ხელს 20 წუთის განმავლობაში (ორივე ნესტო ცერა და საჩვენებელი თითებით უნდა მიაჭირო ცხვირის ძგიდეს) ხშირ შემთხვევაში სისხლდენა შეწყდება. თუ ესარ შველის სისხლდენის შეჩერაბას თავიდან, მაშინ ექიმთთან უნდა მიხვიდე და სხვადასხვა სახის მანიპულაცია შეიძლება გააკეთოს ექიმმა.

Posted by: Lyechka 21 Jan 2008, 12:50
vano_t მადლობა დიდი დიდი
მაშინ ნაფტიზინს და მოჭერას ვიზამ tongue.gif
ექიმთან ვერ წავალ sad.gif

Posted by: *hedgehog* 22 Jan 2008, 11:02
წინასწარ გიხდით ბოდიშს, შეიძლება გაურკვევლად ვიკითხო, მაგრამ ძალიან მაინტერესებს როგორი ტკივილია საშიში მუცლის მიდამოებში smile.gif

ანუ, 2-3 დღეა რაღაც მტკივა მუცელში, ვერ გავიგე თირკმელაა თუ რა არი. სადღაც მანდ მტკივა, მაგრამ მარჯვენა მხარეს უფრო მეტად მტკივა და თან დილას შევამჩნიე, რომ წელისკენაც გადავიდა ეს ტკივილი. მუდმივად არ მტკივა, მაგრამ ხშირად მახსენებს თავს და მოძრაობას მიშლის.

არ გამივლის თავისით? ჩემი ფეხის გადამკიდეს ექიმების პანიკურად მეშინია და ვერც კი ვურეკავ sad.gif

მადლობა

Posted by: vano_t 22 Jan 2008, 20:16
*hedgehog*
QUOTE
წინასწარ გიხდით ბოდიშს, შეიძლება გაურკვევლად ვიკითხო, მაგრამ ძალიან მაინტერესებს როგორი ტკივილია საშიში მუცლის მიდამოებში smile.gif

ანუ, 2-3 დღეა რაღაც მტკივა მუცელში, ვერ გავიგე თირკმელაა თუ რა არი. სადღაც მანდ მტკივა, მაგრამ მარჯვენა მხარეს უფრო მეტად მტკივა და თან დილას შევამჩნიე, რომ წელისკენაც გადავიდა ეს ტკივილი. მუდმივად არ მტკივა, მაგრამ ხშირად მახსენებს თავს და მოძრაობას მიშლის.

არ გამივლის თავისით? ჩემი ფეხის გადამკიდეს ექიმების პანიკურად მეშინია და ვერც კი ვურეკავ sad.gif

მადლობა

მუცლის ტკივილების უმრავლესობა სტატისტიკურად არ არის საშიში. ე.წ. მწვავე მუცელი (საშიში ტკივილები რომლებიც მოითხოვენ ოპერაციულ ჩარევას) უფრო ხშირად მწვავედ იწყება, ანუ უცბად და ადამიანს აწუხებს საკმარისად, რომ ექიმს 24-48 საათის განმავლობაში მიაკითხოს. შეიძლება კიდევ ბევრი სიმპტომების ჩამოთვლა, რომელიც სტატისტიკურად ზრდის მწვავე მუცლის შანს, მაგრამ ამ ქვეყანაზე არაფერი არის აბსოლუტური. მუცლის ტკივილის სახლში (და თვითონ პაციენტის მიერ) დიაგნოზი შეუძლებელია. მუცლის ღღუშუ ყველაზე დიდი რაოდენობით ორგანო არსებობს და ნებისმიერი მათგანის დაავადება შეიძლება იწვევდეს შენს სიმტომებს. მე გირჩევდი, რომ ექიმი ნახო. ისე ღებინება თუ არ გაქვს, სისლდენა თუ არ გაქვს (სისლხლიანი ღებინება მაგალითად ან სისხლიანი განავალი), მაღალი სიცხეები არ გაქვს, სისხლში ლეიკოციტები მომატებული არ არის, მაშინ გაცილებით ნაკლებია შანსი რომ მწვავე მუცელი იყოს. მუცლის დადუდგენელი ტკივილების დროს ყველა ვარიანტში არის ექიმის ნახვა აუცილებელი და ჩემი რჩევაც ეგ იქნება.

Posted by: *hedgehog* 23 Jan 2008, 10:34
vano_t
დიდი მადლობა. გადავრჩი მგონი. რამდენიმე კვირის წინ ბრმა ნაწლავმა შემომიტია და შემეშინდა კიდევ ეგ ხომ არ არისო. დედამ მაშინ მითხრა თუ არ მიხვალ გაგისკდება და მოკვდებიო. ხოდა შემეშინდა..... შენ ნამდვილი ექიმი ხარ კი არადა ექიმი მეგობარი ხარ smile.gif

Posted by: SPC 26 Jan 2008, 06:19
აუ ხალხო ვკვდები მიშველეთ
მახველებს საშინლად და არ ჩერდება ლამისაა გავგიჟდე, ანუ წუთში სამჯერ-ოთხჯერ და ძალიან დამღლელია, კუნთები დამეღალა უკვე ყელის ( თუ აქვს კუნთები smile.gif ამდენი ხველებისგან. გუშინ მთელი ღამე არ დამაძინა, ეხლაც დილის 6 საათია და ავდექი უკვე,
სიტუაცია ასეთი იყო, გუშინწინ სახლში რომ მოვედი ცუდად ვიყავი და სიცხე რომ გავიზომე 38.3 მქონდა, დავლიე ნიმესილი 2 და მეორე დღეს სიცხემ დამიწია, საქმე მქონდა აუცილებელი და ქუჩაში გავედუი, იქ ძალიან ცივი ჰაერი იყო და სუნთქვისას საშინლად მეტკინა ბრონქები თუ სასუნთი გზები, აი ამის მერე დამეწყო ეს საშინელი მშრალი ხველება და არ მეგონა თუ ასე შეიძლება გაეწამებინა ადამიანი..
ეს ბრონქიტია? როგორ უნდა მოვაშთო მალე რაცაა?


პს. ხო კიდევ, თითქოს ზემოთ მჩხაპნისო რაღაც, ანუ ეს ხელება მაინცდამაინც ბრონქებიდან არ მოდის, თითქოს ალეგიული რარაც , აი ზემოთ მჩხაპნის

Posted by: LOLITA_24 26 Jan 2008, 12:19
ჩემს დაქალს დაახლოებით 6 წლის წინ ქონდა ნერვის ანთება და ამან სმენა საგრძნობლად დაუქვეითა, დადიოდა რაღაც პროცედურებზე, მაგრამ სმენამ ძალინ ცოტა პროცენტით მოიმატა. შემდეგ საერთოდ დაანება თავი. შეიძლება სმენის აღდგენა??? ან არის რაიმე ახალი მიღწევა სამედიცინო სფეროში ამ კუთხით??? წინასწარ მადლობა

Posted by: vano_t 28 Jan 2008, 02:59
SPC
ალბათ უკვე გამოკეთდი კიდეც. ზემო სასუნთქი გზების ინფექციებმა იცის ხველა. ეს ინფექციები უმეტესად ვირუსულია და თავისით გადის. ხველისათვის ხველები საწინააღდეგო წამალი შეგიძლია მიიღო. თუ ფილტვების რაიმე ქრონიკული დაავადება გაგაჩნია(ასთმა ან ფილტვების ქრონიკული ობსტრუქციული დაავადება) მაშინ ექიმი უნდა ნახო აუცილებლად. ასევე, თუ სუნთქვის უკმარიოსბა გაქვს დატვირთვის ან მოსვენების დროს, ექიმის ნახვა აუცილებელია რათა ფიტლვების ანთება გამოირიცხოს.

LOLITA_24
QUOTE
ჩემს დაქალს დაახლოებით 6 წლის წინ ქონდა ნერვის ანთება და ამან სმენა საგრძნობლად დაუქვეითა, დადიოდა რაღაც პროცედურებზე, მაგრამ სმენამ ძალინ ცოტა პროცენტით მოიმატა. შემდეგ საერთოდ დაანება თავი. შეიძლება სმენის აღდგენა??? ან არის რაიმე ახალი მიღწევა სამედიცინო სფეროში ამ კუთხით??? წინასწარ მადლობა
რამდენი წლისაა შენი დაქალი?
საერთო სმენის დაქვეითებას ბევრი რამ იწვევს. სმენის დაქვეითებისას 2 ძირითადი მექანიზმი უნდა განსხვავდეს ერთმანეთისაგან: ცონდუქციური ანუ გამტარი(როცა დაზიანებულია ის მექანიზმი რაც უზრუნველყოფს ჰაერის რხევების გადაცემას სმენის მიმღებს აპრატთან) და სენსორულ-ნეირონული(როცა დაქვეითებულია სმენის ის ნაწილი, რაც ჰაერის რხევებს გარდაქმნის ელექტრულ იმპულსებში და შემდეგ ამ იმპულსების ანალიზს აკეთებს). სმენის დაქვეითების ყველაზე ხშირი მიზეზია გარეთა სასმენი არხის დახშობა "გოგირდით". იმედია ეს გამოირიცხა, რადგანაც ამის მკურნალობა მარტივია.

საბოლოოდ სმენის დაქვეითების მიზეზების დადგენა და მკურნალობა ხდება ყელყურცხვირის ექიმის მიერ.

Posted by: LOLITA_24 28 Jan 2008, 11:27
25 წლისაა, ნერვის ანთების შედეგად დაქვეითებული სმენა აღდგენას ექვემდებარება???? ძალინ განიცდის ამ ყველაფერს და კომპლექსდება user.gif , მინდა დავეხმარო. იქნებ მირჩიოთ კარგი სპეციალისტი ამ სფეროში

Posted by: baby-bobo 31 Jan 2008, 00:40
vano_t
ერთი ასეთი შეკითხვა მაქვს და თუ შეგიძლია დამაკვალიანე....

62წლის ქალს აწუხებს წნევები, დღის განმავლობაში ხან მაღალი აქვს ხან დაბალი... ეხვევა ტავბრუ, მოკლედ მგონი ასაკობრივი პრობლემები აქვს....
მირჩიეთ თავის გამოსაკვლევად, მიზეზის დასადგენად და მდგომარეობის მოსაწესრიგებლად რის ექიმთან უნდა მივიდეს თავდაპირველად???

Posted by: zeppelina 2 Feb 2008, 00:06
დედაჩემს აქვს ორმხრივი ბრონქოექტაზიული დაავადება.ფილტვის სკლეროზი. არის 70წლის.ანტიბიოტიკების მიმართ ალერგიულია. 6 თვის წინ სუმეტროლიმზე ჰქონდა ანაფილაქსური შოკი,გაუსივდა და გაუშავდა ცხვირი,ხელის მტევნები,ყურები,მთელ სხეულზე ჰქონდა გამონაყარი.მანამდე უფრო იოლ ფორმებში 5-ჯერ მისცა ალერგია სხვადასხვა ანტიბიოტიკებზე.მდგომარეობიდან ძირითადად გამოიყვანა პრედნიზოლონის გადასხმებმა. ამჟამად თავს ძალიან სუსტად გრძნობს, ადვილად იღლება, ახველებს.თვითონ ფიქრობს რომ ყველაზე მეტად აწუხებს გული, განიცდის ჰაერის უკმარისობას. ბრონქოექტაზის დიაგნოზი დაუსვეს ტომოგრაფიია გადაღების შემდეგ. ძალიან გთხოვთ, თუ შეგიძლიათ მირჩიეთ როგორ მოვიქცეთ, ფილტვებს უნდა ვუმკურნალოთ თუ გულს, თუ ორივეს. ტომოგრაფიის გადაღებამდე დიაგნოზი ჰქონდა ფილტვისმიერი გული და ფილტვის სკლეროზი. ამჟამად ღებულობს ენგისტოლს, ტარტეფედრილს და იმუვიტს. რა შეიძლება გულისთვის რომ მიიგოს? რამდენად საშიშია ეს დაავადება? ძალიან გთხოვთ მიპასუხოთ.

Posted by: vano_t 3 Feb 2008, 01:21
LOLITA_24
QUOTE
25  წლისაა,  ნერვის ანთების შედეგად დაქვეითებული სმენა აღდგენას ექვემდებარება???? ძალინ განიცდის ამ ყველაფერს და კომპლექსდება user.gif , მინდა დავეხმარო. იქნებ მირჩიოთ კარგი სპეციალისტი ამ სფეროში
სამწუხაროდ არავის ვიცნობ საქართველოში და ვერ დაგეხმარები. აქ, ფორუმზე სხვას ეცოდინება ალბათ და მათ კითხე. PM-ში თუ მომწერ იფრო დაწვრილებით რა ჭირს, შემიძლია რჩევა მოგცე.

baby-bobo
QUOTE
ერთი ასეთი შეკითხვა მაქვს და თუ შეგიძლია დამაკვალიანე....

62წლის ქალს აწუხებს წნევები, დღის განმავლობაში ხან მაღალი აქვს ხან დაბალი... ეხვევა ტავბრუ, მოკლედ მგონი ასაკობრივი პრობლემები აქვს....
მირჩიეთ თავის გამოსაკვლევად, მიზეზის დასადგენად და მდგომარეობის მოსაწესრიგებლად რის ექიმთან უნდა მივიდეს თავდაპირველად???
ლაბილური წნევა არც თუ ისე იშვიათი რამ არის და წნევის წამლებით უნდა დარეგულირდეს წნევა. თავბრუსხვევაც ხშირია წინწასულ ასაკში. უმეტეს შემთხვევაში მიზეზს ვერ ნახავ, თუმცა ნერვული სისტემის და გულის გამოკვლევა აუცილებელია გარკვეულ ლაბორატორიულ ტესტებთან ერთად. თავბრუსხვევის 2 ფორმა არსებობს: 1) არის ვერტიგო, როცა ადამიანი გრძნობს თითქოს სივრცე მოძრაობს მის გარშემო(ან სივრცეში მოძრაობს) და 2) რომელიც არის გულისწასვლის შეგრძნება (აი, მაგალითად სისხლის დანახვაზე რომ ემართება ზოგიერთს). ამ ორს შორის განსხვავება უნდა მოხდეს აუცილებლად, რადგანაც ამათი მიზეზები და მკურნალობა განსხვავებულია. პირველის მიზეზი უფრო ხშირად წონასწორობის აპარატის დაავადებებია. მეორეს უფრო ხშირად მიზეზს ვერ უპოვი, თუმცა გულის სხვადასხვა დაავადებები და სისხლნაკლებობა უნდა გამოირიცხოს.

zeppelina
QUOTE
დედაჩემს აქვს ორმხრივი ბრონქოექტაზიული დაავადება.ფილტვის სკლეროზი. არის 70წლის.ანტიბიოტიკების მიმართ ალერგიულია. 6 თვის წინ სუმეტროლიმზე ჰქონდა ანაფილაქსური შოკი,გაუსივდა და გაუშავდა ცხვირი,ხელის მტევნები,ყურები,მთელ სხეულზე ჰქონდა გამონაყარი.მანამდე უფრო იოლ ფორმებში 5-ჯერ მისცა ალერგია სხვადასხვა ანტიბიოტიკებზე.მდგომარეობიდან ძირითადად გამოიყვანა პრედნიზოლონის გადასხმებმა. ამჟამად თავს ძალიან სუსტად გრძნობს, ადვილად იღლება, ახველებს.თვითონ ფიქრობს რომ ყველაზე მეტად აწუხებს გული, განიცდის ჰაერის უკმარისობას. ბრონქოექტაზის დიაგნოზი დაუსვეს ტომოგრაფიია გადაღების შემდეგ. ძალიან გთხოვთ, თუ შეგიძლიათ მირჩიეთ როგორ მოვიქცეთ, ფილტვებს უნდა ვუმკურნალოთ თუ გულს, თუ ორივეს. ტომოგრაფიის გადაღებამდე დიაგნოზი ჰქონდა ფილტვისმიერი გული და ფილტვის სკლეროზი. ამჟამად ღებულობს ენგისტოლს, ტარტეფედრილს და იმუვიტს. რა შეიძლება გულისთვის რომ მიიგოს? რამდენად საშიშია ეს დაავადება? ძალიან გთხოვთ მიპასუხოთ.

1) ბრონქიექტაზიული დაავადების დიაგნოზი მართალია კომპიუტერული ტომოგრაფიის საშუალებით ისმება, ოღონ მაღალი რეზოლუციის კტ არის საჭირო, რადგანაც სტანდარტული კტ დიდ ჭრილებს აკეთებს და ბრონქიექტაზებს მათი მცირე ზომის გამო ვერ დაინახავს. ავადმყოფის ყველაზე ხშირი სიმპტომია ხველა ნახველით და ხშირი პნევმონიები. ბრონქიექტაზიური დაავადების დიაგნოზის დასმის შემდეგ უნდა დადგინდეს გამომწვევი მიზეზები: მაგალითად უნდა გამოირიცხოს ცისტური ფიბროზი (მაგრამ ხანშიშესული ავადმყოფის შემთხვევაში არ არის საჭირო, რადგანაც ამ დაავადების სიმპტომები გამოვლინდები ადრეულ ასაკში), უნდა გამოირიცხოს ალერგიული ბრონქოპულმონალური ასპერგილიოზი, ალფა-1 ანტიტრიპსინის დეფიციტი და სხვადასხვა იმუნოლოგიური პათოლოგიები(ამისათვის უნდა გაისინჯოს ალფა-1 ანტიტრიფსინის დონე და იმუნოგლობულინების რაოდენობრივი ანალიზი); საქართველოში ტუბერკულოზის გამორიცხვაც აუცილებელია. გარდა ამისა ავადმყოფს უნდა ჩაუტარდეს ფილტვების ფუნქციონალური გამოკვლევა (PFT-pulmonary function test-რაც ძალიან კარგ მონაცემებს იძლევა ფილტვების ფუქნციაზე); ავადმყოფს უნდა ჩაუტარდეს აგრეთვე არტეიული სისხლის გაზების ანალიზი ან პულს-ოქსიმეტრია, რათა დადგინდეს ჰიპოქსიის არსებობა/არარსებობა და მისი ხარისხი; ექოკარდიოგრამა და ელექტროკარდიოგრამა გეტყვის ავადმყოფს აქვს თუ არა გულის პათოლოგიები, ან ფილტვისმიერი გული (ეკგ-ზე მაგალითად მაღალი P კბილების არსებობა და QRS ღერძის მარჯვნივ გადახრა, რაც მიუთითებს გულის მარჯვენა მხარეს და შესაბამისად ფიტვების ცირკულაციაში მაღალ წნევას. ექოკარდიოგრამაზე დაინახავ არის თუ არა მარჯვენა გული გადიდებული და ჰიპერტროფიული, აგრეთვე შეაფასებ წნევას ფილტვების არტერიაში). ეს ყველაფერი საჭიროა რათა დადგინდეს დაავადების არსებობა და პროგნოზი.
სამკურნალოდ, ბევრი არაფერია მედიცინაში. შემდეგ რამ უნდა დაიცვა: მეორეული მიზეზები უნდა გამოირიცხოს და უნდა უმკურნალო მეორეულ მიზეზებს, რიც შემთვევაშიც დაავადების მკურნალებადი შეიძლება იყოს (მაგალითად ტუბერკულოზი); თუ მწეველია, წევა უნდა შეწყვიტოს და ყველამ მის გარშემო უნდა აარიდოს წევას თავი; ინფექციებს უნდა უმკურნალო ანტიბიოტიკებით; თუ ჰიპოქსიურია, მაშინ ჟანგბადი ჭირდება უმეტესად(ჰიპოქსია როცა ნელა ვითარდება ადვილად ეგუება ადამიანი, მაგრამ პრობლემა იმაშია რომ ქრონიკული ჰიპოქსია იწვევს ფიტლვების არტერიების ვაზოსპაზმს და ფილტვისმიერი გულის ჩამოყალიბებას საბოლოო ჯამში, რაც ამა ავადმყოფების სიკვდილიანობის ძირითადი მიზეზია); ყოველ წელს აუცილებლად უნდა აიცრას გრიპზე (ეს ეხება ყველა ავადმყოფს ფილტვების ქრონიკული დაავადებებით, რადგანაც გრიპი ძალიან რთულ ფორმებში მიმდინარეობს ამ ავადმყოფებში); უნდა დადგინდეს აცრილია თუ არა ავადმყოფი ყივანახველაზე, წითელაზე და წითურაზე და თუ არ არის აცრილი, მაშინ უნდა გაუკეთდეს აცრა (რა თქმა უნდა აცრის უკუჩვენება თუ არ არსებობს); ბრონქოდილატატორების გამოყენება შეიძლება იყოს საჭირო, უფრო ხშირად ნებულიზებულ ფორმებში; ფილტვების ჰიგიენა არის საჭირო სხვადასხვა წმალებით, რომელნიც ნახველის მოცილებას აადვილებს და ბოლოს, ფილტვის ნაწილის ქირურგიულმა ამოკვეთამ შეიძლება კარგი შედეგი მოგცეს თუ ბრონქიექტაზიები ლოკალურია, ან ლოკალურად უფროა გამოხატული.

ამის გადაწყვეტა ინტერნეტიდან შეუძლებელია, ამიტომ შენს ადგილას ვნახავდი კარგ პულმონოლოგს, კარდიოლოგს და შესაძლებელია ქირურგსაც და დეტალურად გამოვაკვლევინებდი დედას. თუ რაიმე კონკრეტული კითხვა გექნება, მზად ვარ დაგეხმარო.

2) ფილტვების სკლეროზი დიაგნოზი არ არის. არსებობს ფილტვების სხვადასხვა დაავადებები რომლების ფიტლვის "შემაერთებელ" ქსოვილს აზიანებს, ე.წ. intersticial lung disease (IDL). ოღონ ამ დაავადების დიაგნოზი ბრონქოსკოპიით და ბიოფსიით ხდება (ან open lung bipsy-თ, ანუ ფილტვების ღია ბიოფსიით). მკურნალობა მოიცავს სტეროიდებს და იმუნოსუპრესანტებს და ხდება პულმონოლოგის მიერ.

ის წამლები, რაც ჩამოთვალე არ ვიცი რომელია. თუ გენერიულ სახელებს მეტვყის (არაფირმისეულ დასახელებას), კარგი იქნება.

Posted by: BadbadGirl 4 Feb 2008, 01:42
vano_t
შეგიძლიათ მითხრათ მეხსიერების გასაუმჯობესებლად რა ბუნებრივი / ნატურალური საშუალებები შემიძლია მივიღო ? ??
წინასწარ მადლობა smile.gif

Posted by: *hedgehog* 4 Feb 2008, 14:42
აქაც ვიკითხავ რა user.gif

ექიმებოა რამე მირჩიეთ


დღეს დილიდან მტკივა თავი კეფის მხარეს მარჯვნივ და მერე მოდის ყურთან და შუბლამდე მტკივდება. უკვე ძააააააალიან მტკივა. საერთოდ კეფასთან მაგ ადგილას ხშირად მტკივა, მაგრამ აი დღეს სულ სხვაა. დილიდან მტკივა. რა წამალი დავლიო? მიშველეთ რა


და ექიმიც მირჩიეთ ვის გავესინჯო

Posted by: zeppelina 4 Feb 2008, 21:47
vano_t ძალიან დიდი მადლობა გამოხმაურებისთვის. თქვენმა პასუხმა ცოტა შემაშინა. თქვენ ფიქრობთ რომ დიაგნოზი შეიძლება არასწორად იყოს დასმული? ბოლოს ვიყავით კვალიფიციურ პულმონოლოგთან და ის თვლის რომ დიაგნოზი ეჭვს გარეშეა, ასევე თქვა, ახალგაზრდა რომ იყოს ოპერაციას გავაკეთებდითო, ან ცალ ფილტვზე მაინც რომ ქონდესო.

რაც შეეხება მეორად დაავადებას, ტუბერკულოზი გამორიცხეს ნახველის ჩათესვით, თანაც დედა უკვე ავადაა 1998 წლიდან და ალბათ ტუბერკულოზი რომ ქონოდა მკურნალობის გარეშე ამდენ ხანს ვერ იცოცხლებდა. უმეტესად დაზიანებული აქვს ფილტვის ქვედა შრეები.

კარდიოგრამა გადავიღეთ ერთი წლის წინ. დიაგნოზი დაუსვეს ტაქიკარდიული იშემია. იმუნოგლობულინზე ანალიზი გავაკეთეთ. Ig E ქონდა 300-ზე მეტი. ალერგოლოგმა დაუნიშნა: მეთიპრედი, ერიუსი. გიგზავნით იმ წამლების ანოტაციებს, რომლებსა ახლა ღებულობს. ესენი ჰომეოპათიური პრეპარატებია. Энгистол / Engystol ®
таблетки подъязычные
Регистрационное удостоверение П-8-242 №007989 от 26.11.1996г.
Состав:1 таблетка содержит: Vincetoxicum hirundinaria D6 75,0 мг, Vincetoxicum hirundinaria D10 75,0 мг, Vincetoxicum hirundinaria D30 75,0 мг, Sulfur D4 37,5 мг, Sulfur D10,37,5 мг.
Показания: иммуномодулирующее средство при вялотекущих хронических заболеваниях внутренних органов, вирусных инфекциях, сепсисе, состояниях интоксикации.
Противопоказания: нет.
Побочное действие: не выявлено.
Способ применения и дозы: обычно рассасывать под языком по 1 таблетке 3 раза в день.
Форма выпуска: упаковка содержит 50 или 250 таблеток.

Тартефедрель / Tartephedreel ®
капли для внутреннего применения
Регистрационное удостоверение П-8-242 №009412 от 23.04.1997г.
Состав: 100 г содержат: Tartarus stibiatus D4, Hepatica nobilis D3, Atropa belladonna D4, Natrium sulfuricum D4, Arsenum jodatum D6, Quebracho D5 по 10 г, Naphthalinum D6, Betonica D2, Anisum stellatum D3, Lobelia inflata D4, Ipecacuanha D4, Blatta orientalis D6, Medorrhinum-Nosode D8, Ephedra distachya D3 по 5 г, этанола около 35 объемных %.
Показания: воспалительные и аллергические заболевания верхних дыхательных путей.
Противопоказания: не применять при заболеваниях щитовидной железы без назначения врача.
Побочное действие: не выявлено.
Способ применения и дозы: обычно 3 раза в день по 10 капель, растворенных в 100 мл воды; в острых случаях - по 10 капель каждые 15 минут в течение 2 часов.
Форма выпуска: флакон-капельница содержит 30 мл раствора

და კიდევ, თუ შეგიძლიათ იქნებ მიპასუხოთ, უტყუარია თუ არა სისხლის მიხედვით გაკეთებული ანალიზი ანტიბიოტიკების მიმართ ალერგიულობის დასადგენად(სინჯის აღებაზე ექიმი თავს იკავებს, შიშობს შოკი არა გამოიწვიოს)
რომელი პრეპარატის მიღებაა მიზანშეწონილი ასეთი ავადმყოფისათვის გულის აჩქარებებისა და მაღალი წნევის დროს.
წინასწარ გიხდით მადლობას.

Posted by: SOULFUL 6 Feb 2008, 02:08
Natosha
QUOTE
ასეთი შეკითხვა მაქვს, რეინოს სინდრომზე ვწერ რეფერატს



რატომღაც მეც რეინოს ფენომენი მომცეს დასაწერად biggrin.gif ჩვენი დისკუსია კარგად გამომადგა MDconsult- იდან ამომწურავი მასალა ამოვიღე, თუ კიდევ გჭირდება პრეზენტაციაც გავაკეთე უკვე smile.gif

Posted by: *hedgehog* 6 Feb 2008, 12:09
SOULFUL
QUOTE
მეც რეინოს ფენომენი მომცეს დასაწერად

gigi.gif აგერ ვარ მაგ დიაგნოზის ადამიანი, თუ დაგჭირდი gigi.gif ისე, მერე თემა გადამაკითხე რა. რა შუაში ვარ მაგ დაავადებასთან ცივი ხელების გარდა მაინტერესებს smile.gif

Posted by: SOULFUL 6 Feb 2008, 18:01
*hedgehog*

QUOTE
აგერ ვარ მაგ დიაგნოზის ადამიანი, თუ დაგჭირდი


ხელებზე ფოტო გადაიღე და გამომიგზავნე, პრეზენტაციაში ჩავსვავ , ბოლოში მივაწერ "საკუთარი მასალა" biggrin.gif ზეგ მაქვს მოხსენება, ერთადერთი პაციენტი ვნახე მაგ სინდრომით, ისიც კარებში, გარბოდა biggrin.gif

Posted by: თეოგოგო 6 Feb 2008, 23:27
SOULFUL
QUOTE
აგერ ვარ მაგ დიაგნოზის ადამიანი, თუ დაგჭირდი ისე, მერე თემა გადამაკითხე რა. რა შუაში ვარ მაგ დაავადებასთან ცივი ხელების გარდა მაინტერესებს

+1




user.gif

Posted by: shtori 7 Feb 2008, 00:29
ერთი კითხვა მაქვს და იქნებ დამეხმაროთ
1 წუთითაც რომ გაუნძრევლად ვიდგე, ან ვიჯდე, მაშინვე ფეხები მიჭრელდება. მხოლოდ აქტიური მოძრაობის დროს და ჰორიზონტალურ მდგომარეობაში მიქრება ეს სიჭრელე. ხელებიც, როცა ქვევით მაქვს დაშვებული, მიჭრელდება.
რისი ბრალია? როგორ შეიძლება მკურნალობა?
დიდი მადლობა წინასწარ

Posted by: vano_t 7 Feb 2008, 07:44
BadbadGirl
QUOTE
შეგიძლიათ მითხრათ მეხსიერების გასაუმჯობესებლად რა ბუნებრივი / ნატურალური საშუალებები შემიძლია მივიღო ?  ??
მეხსიერების პრობლემა ორგვარია: 1) თვითონ ინფორმაციის "ჩაწერა" ტვინში დაზიანებულია; 2) ყურადღება არის დაქვეითებული;

ყურადღების გასაუმჯობესებლად ყველაზე კარგი საშუალებაა ვარჯიშები: მედიტაცია ყველაზე კარგია. რაც შეეხება "ჩაწერის" პრობლემას, ბევრს ვერაფერს გეტყვი. ნატურალურ საშუალებებს არ ვიცნობ.

*hedgehog*
QUOTE
დღეს დილიდან მტკივა თავი კეფის მხარეს მარჯვნივ და მერე მოდის ყურთან და შუბლამდე მტკივდება. უკვე ძააააააალიან მტკივა. საერთოდ კეფასთან მაგ ადგილას ხშირად მტკივა, მაგრამ აი დღეს სულ სხვაა. დილიდან მტკივა. რა წამალი დავლიო? მიშველეთ რა
პასუხი დამიგვიანდა ალბათ. უკვე წესით აღარ უნდა გტკიოდეს.

shtori
QUOTE
ერთი კითხვა მაქვს და იქნებ დამეხმაროთ
1 წუთითაც რომ გაუნძრევლად ვიდგე, ან ვიჯდე, მაშინვე ფეხები მიჭრელდება. მხოლოდ აქტიური მოძრაობის დროს და ჰორიზონტალურ მდგომარეობაში მიქრება ეს სიჭრელე. ხელებიც, როცა ქვევით მაქვს დაშვებული, მიჭრელდება.
რისი ბრალია? როგორ შეიძლება მკურნალობა?
დიდი მადლობა წინასწარ
მაგას, შენ რასაც აღწერ, ლივედო (livedo reticularis) ქვია. ლივედოს მკურნალობა არ არსებობს. თუ რაიმე გამომწვევი აქვს, მაშინ ის გამომწვევი მიზეზი უნდა დადგინდეს და იმას უნდა უმკურნალო. თუ სხვა მხრივ ყველანაირად ჯანმრთელი ხარ, მაშინ გამომწვევი ალბათ არ აქვს.

zeppelina
QUOTE
ძალიან დიდი მადლობა გამოხმაურებისთვის. თქვენმა პასუხმა ცოტა შემაშინა. თქვენ ფიქრობთ რომ დიაგნოზი შეიძლება არასწორად იყოს დასმული? ბოლოს ვიყავით კვალიფიციურ პულმონოლოგთან და ის თვლის რომ დიაგნოზი ეჭვს გარეშეა, ასევე თქვა, ახალგაზრდა რომ იყოს ოპერაციას გავაკეთებდითო, ან ცალ ფილტვზე მაინც რომ ქონდესო.
დიაგნოზი შეიძლება სწორად არის დასმული. მე სხვა რამ ვთქვი: ბევრი სხვა დაავადება უნდა გამოირიცხოს თქო. მიუხედავად იმისა, რომ დაიგნოზი საკმაოდ ნათელია, მაინც უნდა ჩაუტარდეს სხვა კველევები, განსაკუთრებით გულის გამოკვლევა(ექოკარდიოგრაფია).

რაც შეეხება შენს ჩამოთვლილ წამლებს, როგორც ჩანს ნატურალური საშუალებებია და ბევრს ვერაფერს ვიტყვი რადგანაც არ გამიგია ეგ წამლები. ნატურალურ საშუალებებს, სამწუხაროდ, კარგად არ ვიცნობ.

QUOTE
და კიდევ, თუ შეგიძლიათ იქნებ მიპასუხოთ, უტყუარია თუ არა სისხლის მიხედვით გაკეთებული ანალიზი ანტიბიოტიკების მიმართ ალერგიულობის დასადგენად(სინჯის აღებაზე ექიმი თავს იკავებს, შიშობს შოკი არა გამოიწვიოს)
არ არსებობს ტესტი დედამიწაზე რომელიც უტყუარი იქნება. ყველა ტესტს აქვს ე.წ. ცრუ-დადებითი და ცრუ-უარყოფითი პასუხები. ანუ, ტესტის შეიძლება დადებითი იყოს და დაავადებას (ამ შემთხვევაშI ალერგიას) მიუთითებდეს, მაგრამ ადამიანს არ ქონდეს დაავადება ან პირიქით; ტესტი შეიძლება უარყოფითი იყოს (ანუ დაავდებას გამორიცხავდეს) და ამ დროს ადამიანს დაავადება ქონდეს. თანაც, მე პრაქტიკაში არ შემხვედრია ანტიბიოტიკების მიმართ ალერგიის ტესტებით დადგენა. ანტიბიოტიკების ბევრი ჯგუფი არსებობს: პენიცილინები, ცეფალოსპორინები, მაკროლიდები და ასე შემდეგ. ამიტომ ექიმს ყოველთვის აქვს დიდი არჩევანი სხვადასხვა ანტიბიოტიკების გამოსაყენებლად. თუ ავადმყოფს ალერგია ქონდა, დავუშვათ, პენიცილინზე (განსაკუთრებით ანაფილაქსიური რეაქცია), მაშინ არ დაუნიშნავ პენიცილინის და ცეფალოსპორინების ჯგუფს, მაგრამ გაქვს არჩევანი: ამინოგლიკოზიდები (გენტამიცინი და მისი მსგავსნი), ტეტრაციკლინების ჯგუფი, ქვინოლონები (ლევოფლოქსაცინი და ასე შემდეგ) და ასე შემდეგ.

QUOTE
რომელი პრეპარატის მიღებაა მიზანშეწონილი ასეთი ავადმყოფისათვის გულის აჩქარებებისა და მაღალი წნევის დროს.
წინასწარ გიხდით მადლობას.
წინა პოსტში დავწერე ფაქტიურად რა საშუალებები უნდა ბრონქიექტაზიან ავადმყოფს.

რაც შეეხება ტაქიკარდიულ ისქემიას (ანუ იშემიას), მასეთი რამ არ გამიგია.

Posted by: *hedgehog* 7 Feb 2008, 10:17
vano_t
QUOTE
პასუხი დამიგვიანდა ალბათ. უკვე წესით აღარ უნდა გტკიოდეს.

კი, თითქმის აღარ მტკივა. აქაიქ მახსენებს თავს. მაგრამ რა დამრჩა იცი მაგის მერე? მარჯვენა ყურში თითოქს რაღაც მაცვება შიგნიდან და კეტავს. ექიმთან ვაპირებ 2შაბთას წასვლას, ვნახოთ რას მეტყვის


SOULFUL
შეიძლება მქონდეს ხელების სურათი, მაგრამ სიცივეს არ ასხივებს gigi.gif დანარჩენი ჩვეულებრივი ხელები მაქვს gigi.gif

Posted by: baby-bobo 7 Feb 2008, 12:02
vano_t
რა მაინტერესებს....

სახლში არის 23- 24 გრადუსი ტემპერატურა, ანუ არ ცივა.... მაგრამ საღამოობით მეწყება კანკალი სიცივისგან, თითქოს გათბობა მიჭირს, მეყინება ხელ-ფეხი, რისი ბრალი შეიძლება იყოს???

Posted by: Internet_Contraband 7 Feb 2008, 15:22
მოკლედ, ზაფხულში უცნაური რამეები დამეწყო, ჰოდა თუ ექიმთან ვარ მისასვლელი მითხარით, თორემ ისე მეზარება:

ხელზე, თითზე, ფეხზე და ა.შ. ზაფხულშიც, შემოდგომაზეც და ახლა ზამთარშიც დრო და დრო მებერება ხოლმე მოხრის ადგილებში ძარღვი და ცხადია - მტკივა. რაღაცნაირი მჩხვლეტავი ტკივილი მაქვს ხოლმე, თითქოს ხიჭვი შემერჭოო.

ეს ადგილი მუქდება ხოლმე (ლურჯად) და აშკარად იბერება.

მოკლედ, რას იტყვით?

გმადლობთ

Posted by: თეოგოგო 7 Feb 2008, 20:57
Internet_Contraband
ანგიოლოგს ეჩვენე. smile.gif

Posted by: vano_t 11 Feb 2008, 09:17
baby-bobo
QUOTE
სახლში არის 23- 24 გრადუსი ტემპერატურა, ანუ არ ცივა.... მაგრამ საღამოობით მეწყება კანკალი სიცივისგან, თითქოს გათბობა მიჭირს, მეყინება ხელ-ფეხი, რისი ბრალი შეიძლება იყოს???
შეიძლება არაფრის ბრალია. ისე ფარისებური ჯირკვლის ფუნქციას შევამოწმებდი, ყოველ შემთხვევისათვის. ხანდახან ფარისებური ჯირკვლის ფუნქციის დაქვეითებამ იცის სიცივისადმი (ან შედარებით დაბალი ტემპერატურისადმი) ტოლერანტობის დაქვეითება.

Internet_Contraband
QUOTE
ხელზე, თითზე, ფეხზე და ა.შ. ზაფხულშიც, შემოდგომაზეც და ახლა ზამთარშიც დრო და დრო მებერება ხოლმე მოხრის ადგილებში ძარღვი და ცხადია - მტკივა. რაღაცნაირი მჩხვლეტავი ტკივილი მაქვს ხოლმე, თითქოს ხიჭვი შემერჭოო.

ეს ადგილი მუქდება ხოლმე (ლურჯად) და აშკარად იბერება.

მოკლედ, რას იტყვით?
ასე ძალიან ძნელია თქვა რა არის. შეხედვა უნდა მაგას. თუ სურათს დადებ, შეიძლება დაგეხმარო. შეიძლება სახსარია; შეიძლება სისხლძარღვია; შეიძლება მყესებია; შეიძლება კანია; შეიძლება ნომრაა; შეიძლება დაავადებაა. მოკლედ, ნახვა უნდა.

Posted by: ggggg_gggggg 15 Feb 2008, 17:47
ერთი კითხვა მაქვს და დიდად მადლობელი დაგრჩებით თუ დამეხმარებით!
გრიპის დროს ვღებულობდი ასპირინს C ვიტამინით და ასევე C ვიტამინს, აშკარად ზედმეტი რაოდენობის მიღება მომივიდა და როგორც C ვიტამინს სჩვევია სისხლის გათხევადება მოხდა, შესაბამისად ცხვირიდან მომდის მალმალე სისხლი. ერთი დღე ანუ გუშინ არ მივაქციე ყურადღება, მაგრამ დღესაც გაგრძელდა და საკმაოდ მიჭირს შეჩერება, ვიფიქრე ჩემს ექიმს მივმართავ თქო და დავაგვიანე, ვინაიდან გერმანიაში ვარ ჩემი ექიმის გარდა ვის შეიძლება მივმართო არ ვიცი.
თუ არის რამე წამალი, ან მაზი, ან რამე სხვა საშუალება რომლითაც შეიძლება შველა?
წინასწარ დიდ მადლობას გიხდით!

Posted by: თეოგოგო 16 Feb 2008, 00:51
ggggg_gggggg
სანამ პასუხს გაგცემდნენ.. ეს პოსტი ნახეთ აბა..
http://forum.ge/?showtopic=33759979&view=findpost&p=8221931

თუ მსგავსი პრობლემააა, იქნებ გამოგადგეთ... smile.gif

Posted by: vano_t 16 Feb 2008, 02:21
თეოგოგო
QUOTE
ggggg_gggggg
სანამ პასუხს გაგცემდნენ.. ეს პოსტი ნახეთ აბა..
http://forum.ge/?showtopic=33759979&view=findpost&p=8221931

თუ მსგავსი პრობლემააა, იქნებ გამოგადგეთ... smile.gif

დიდი მადლობა smile.gif ნამდვილად იგივე პრობლემა ჩანს და კაკუნს გადავრჩი wink.gif

კითხვის ავტორს ვუპასუხებ, რომ ც ვიტამინი მნიშვნელოვან გავლენას არ ახდენს სისხლის კოაგულაციაზე, მითუმეტეს დღიური დოზები რომელსაც ვიტამინების კომპლექსი შეიცავს.
გაციების და გრიპის წამალი შეიძლება შეიცავდეს ასპირინს. ამ უკანასკნელს კი შეუძლია თრომბოციტების აგრეგაციის (ანუ ერთმანეთთან მიწებების) დარღვევა და სისხლის "გათხელება". ამიტომ, ასპირინს არ ხმარობენ იმ ადამიანებში რომელთაც სხვადასხვა ორგანოში (ან ორგანოდან) სისხლდენის რისკი აქვთ.

Posted by: ggggg_gggggg 18 Feb 2008, 15:01
თეოგოგო
vano_t
დიდი მადლობა გამოხმაურებისათვის და რჩევებისათვის.

Posted by: teona_g 19 Feb 2008, 16:17
გამარჯობა!

თუ შეგიძლიათ რომ კონსულტაცია გამიწიოთ. მაქვს გრიპი, რომელიც დამეწყო წინა კვირას და მეორე კვორაა გრძელდება. თავიდან მხოლოდ სიხცეები და მერე დამეწყო ხველება. ამჟამად დილით მაქვს 37.4 და საღამოს მიწევს 38 მდე. მეწყება კანკალი. ღამით ვოფლიანობ. მაქვს სველი ხველა.
მაინტერესებს არის თუ არა შესაძლებელი გრიპი ასე ხანგრძლივად მიმდინარეობდეს ან გართულება ხომ არ მომცა ფილტვებზე.

Posted by: Bivrili 19 Feb 2008, 20:42
აუ მიშველეთ რა.................
მგონი ფეხზე მაქვს ნერვის ანთება................დაახლებით 5 წლის წინ კუნთში გავიკეთე რაღაც წამალი, რომელიც სულაც არ მჭირდებოდა და ისევ და ისევ ჩემი ახირების გამო გავიკეთე ...............ჰოდა , ამის მერე მეორე დღეს საშინლად დალურჯდა ნანემსარი.................. sad.gif ..........მერე რაღაც საფენები დავიდე და ცოტა გამიარა................თუმცა მალევე დაიწყო ტკივილი და მგონი უკვე ქრონიკულშია გადასული................
სიმპტომები: ფეხის გასივების შეგრძნება მაქვს, როცა ფეხი იხრება...............დაბუჟება და რაღაც ნემსისებური ჩხვლეტა უკვე ჩვეულებრივ მოვლენად იქცა...............ტკივილი უკვე მთელი ფეხის სიგრძეზე მაქვს.....................რაიმე საშუალება თუ არსებობს რომ გადავიღო არამარტო ძვალი ( რომელიც მგონი არ მჭირდება) არამედ რბილი ქსოვილიც და დავინახო რა მჭირს................ისე მე რას გავიგებ ..................ვის მივმართო?.................

cry.gif .............

აჰა გამახსენდა პრეპარატი "დონა" გამიკეთეს თურმე.................მერე კი მითხრეს რად გინდოდაო................

Posted by: vano_t 19 Feb 2008, 21:08
teona_g
QUOTE
გამარჯობა!

თუ შეგიძლიათ რომ კონსულტაცია გამიწიოთ. მაქვს გრიპი, რომელიც დამეწყო წინა კვირას და მეორე კვორაა გრძელდება. თავიდან მხოლოდ სიხცეები და მერე დამეწყო ხველება. ამჟამად დილით მაქვს 37.4 და საღამოს მიწევს 38 მდე. მეწყება კანკალი. ღამით ვოფლიანობ. მაქვს სველი ხველა.
მაინტერესებს არის თუ არა შესაძლებელი გრიპი ასე ხანგრძლივად მიმდინარეობდეს ან გართულება ხომ არ მომცა ფილტვებზე.

ვირუსული ინფექცია კი შეიძლება გაგრძელდეს 2 კვირამდე, მაგრამ მაღალი სიცხეები და სველი ხველის არსებობა მოითხოვს რომ პნევმონია გამოირცხოს. ამიტომ, მინიმუმ გილმკერდის რენტგენი და ლეიკოციტების დათვლა უნდა გაგიკეთოს ექიმმა. ასევე ფილტვების აუსკულტაცია და პულს-ოქსიმეტრია უნდა გაკეთდეს. პნევმონია რომ გამოირიცხება, შემდეგ შეიძლება თქვა რომ ვირუსია მხოლოდ.
* * *
Bivrili
QUOTE
აუ მიშველეთ რა.................
მგონი ფეხზე მაქვს ნერვის ანთება................დაახლებით 5 წლის წინ კუნთში გავიკეთე რაღაც წამალი, რომელიც სულაც არ მჭირდებოდა და ისევ და ისევ ჩემი ახირების გამო გავიკეთე ...............ჰოდა , ამის მერე მეორე დღეს საშინლად დალურჯდა ნანემსარი.................. sad.gif ..........მერე რაღაც საფენები დავიდე და ცოტა გამიარა................თუმცა მალევე დაიწყო ტკივილი და მგონი უკვე ქრონიკულშია გადასული................
სიმპტომები: ფეხის გასივების შეგრძნება მაქვს, როცა ფეხი იხრება...............დაბუჟება და რაღაც ნემსისებური ჩხვლეტა უკვე ჩვეულებრივ მოვლენად იქცა...............ტკივილი უკვე მთელი ფეხის სიგრძეზე მაქვს.....................რაიმე საშუალება თუ არსებობს რომ გადავიღო არამარტო ძვალი ( რომელიც მგონი არ მჭირდება)  არამედ რბილი ქსოვილიც და დავინახო რა მჭირს................ისე მე რას გავიგებ ..................ვის მივმართო?.................

ექიმი უნდა ნახო აუცილებლად. ბერი რამე შეიძლება იყოს ეგ, მაგრამ უფრო გავს "ნერვის" ტკივლს.

ალბათ ზურგის ბირთვულ მაგნიტური რეზონანსი დაგჭირდება წელის მალების. შეიძლებ ექიმმა ქვემო კიდურების დოპლერული ულტრასონოგრაფიაც გააკეთს.

მოკლედ ექიმი გინდა თუ ტკივილები მასე გაწუხებს. ალბათ ნევროპათოლოგი, თუმცა წესით თერაპევტმა უნდა გაგსინჯოს თავიდან და დიაგნოზიც დასვას.

გისურვებ გამოჯანმრთელებას.

Posted by: Lyechka 20 Feb 2008, 14:56
დამეხმარეთ რა
ეს იყიდება თბილისში?
გეხვეწებით სასწრაფოდ მჭირდებაააა

user posted image

პლიზზზზზზზზზზზზ

ეს სისხლის გლუკოზის შესამოწმებელი ჩხირებია

Posted by: *hedgehog* 20 Feb 2008, 15:07
Bivrili
ეხო გადაიღე, მანდ ჩანს რბილი ქსოვილები. ფეხი რა ადგილას გტკივა? ან ეგ ნემსი სად და რატო გაგიკეთეს? მეც მაქ ფეხზე პრობლემა. ის რაც შენ დაწერე მართლა გავს ჩემი ექიმების ახსნილ ნერვის ანთებას. ნევროპათოლოგთან რომ მიხვიდე, ან თუნდაც თერაპევტთან დაგინიშნავს მკურნალობას, 3 კვირა უნდა სვა წამლები. და როგორც ექიმები იტყვიან, არცერთი დაავადება არაა სახუმარო და დროზე უნდა მიხედო. მითხარი მერე რა იქნება კაი? იქნებ მეც როდისმე მეშველოს

Posted by: Bivrili 20 Feb 2008, 22:08
vano_t
QUOTE
ალბათ ზურგის ბირთვულ მაგნიტური რეზონანსი დაგჭირდება წელის მალების. შეიძლებ ექიმმა ქვემო კიდურების დოპლერული ულტრასონოგრაფიაც გააკეთს.


ასეთ რამეს აკეთებენ ნეტა ჩვენთან?..................თუ ჩვეულებრივი პროცედურაა და უბრალოდ მე ვერ გავიგე ტერმინებიდან გამომდინარე,......................... sad.gif


*hedgehog*
QUOTE
ეხო გადაიღე, მანდ ჩანს რბილი ქსოვილები. ფეხი რა ადგილას გტკივა? ან ეგ ნემსი სად და რატო გაგიკეთეს?

ჩვეულებრივი ეხო?..................მთელი ფეხი მტკივა , მთელს სიგრძეზე....................გაყინვის, გასივების და დაბუჟების შეგრძნება მაქვს როცა ფეხს ვხრი........................დედაჩემმა გამიკეთ ნემსი, და რატომ ნუ მკითხავ.................ჩემივე ახირების გამო, სისხლის ანალიზი გავიკეთე და 1 ჯვარი გაქვსო, ანუ ასე ამიხსნეს რევმატიული ხარო და თავდაცვის მიზნით ექიმმა "დონა " გამომიწერა.............რომ გამიკეთეს მეორე ნემსის მერე საშინლად დამილურჯდა კუნთი და მეტკინა , რამდენიმე დღე ასე იყო მერე გამიარა, მაგრამ მალევე დაიწყო ტკივლი.................ეს დაახლოებით 5 წლის წინ იყო..................მერე პერიოდულად მტკივა, ახლა კი ძალიან გამითამამდა...............

QUOTE
ნევროპათოლოგთან რომ მიხვიდე, ან თუნდაც თერაპევტთან დაგინიშნავს მკურნალობას

წარმოდგენა არ მაქვს ვისთან მივიდე ან ეხო სად კეთდება?..................ანუ იმას ვგულისხმობ ყველა ხომ ვერ მიხვდება ფეხის პრობლემას ეხოზე , ჰოდა ნეტა რომელ სპეციალისტს ეკუთვნის ამ ეხოს გადაღება?...................

QUOTE
და როგორც ექიმები იტყვიან

ყველაფერი არ უნდა დაუჯერო მაგათო მაფრთხილებდნენ...................დაუჯერე შედეგი სახეზეა................
არ დაუჯერო და მტკივა................ cry.gif ....................

QUOTE
მითხარი მერე რა იქნება კაი? იქნებ მეც როდისმე მეშველოს

აუცილებლად გაგაგებინებ თუ ვინმე ვიპოვე კარგი სპეციალისტი.................რას მეტყვიან მეც მაინტერესებს...............

Posted by: vano_t 21 Feb 2008, 00:59
Bivrili
QUOTE
ასეთ რამეს აკეთებენ ნეტა ჩვენთან?..................თუ ჩვეულებრივი პროცედურაა და უბრალოდ მე ვერ გავიგე ტერმინებიდან გამომდინარე,......................... sad.gif
ბირთვულ-მაგნიტური რეზონანსი გამოკვლევის მეთოდია, კომპიუტერული ტომოგრაფიის მსგავსი. საქართველოში რომ აკეთებენ, ნამდვილად ვიცი. თუმცა ექიმთან გჭიდება მისვლა.

Posted by: *hedgehog* 21 Feb 2008, 10:21
Bivrili
ეხოს ეხოსკოპისტი გადიგიღებს და ექიმთან გაგატანს რაღაც ჩანაწერს და მერე ექიმი გაიგებს ანთება არის თუ არა. მე ტატიშვილში გავიკეთე. ნევროპათოლოგი არ ვიცი, მე ტრავმატოლოგთან დავდივარ (ან დავდიოდი). მე სხვა რაღაც მჭირს. შენ რასაც ამბობ მართლა ანთებას გავს. თერაპევტთან მიდი, ნუ გეშინია მალე მოგარჩენენ 2kiss.gif

Posted by: თეოგოგო 21 Feb 2008, 13:35
http://forum.ge/?showtopic=33755915&view=findpost&p=8588622

იქნებ ინტერნისტებს მაინც დავეკვალიანებინე რამენაირად? user.gif
ეხლა პლუს ამას გრიპიც მაქვს .sad.gif

Posted by: vano_t 22 Feb 2008, 06:35
თეოგოგო
QUOTE
http://forum.ge/?showtopic=33755915&view=findpost&p=8588622

იქნებ ინტერნისტებს მაინც დავეკვალიანებინე რამენაირად? user.gif
ეხლა პლუს ამას გრიპიც მაქვს .sad.gif

სადღაც ბაჩუკი-მ დასვა წელის ტკივილზე კითხვა და იქ არის უფრო დეტალურად წელის ტკივილი განხილული. თუ ნახავ, აუცილებლად გამოგადგება. ბაჩუკის დაუ-PM-ე და შეიძლება ახსოვს თემის სათაური.

აქ იმას გეტყვი, რომ წელის ტკივილი არის ძალიან ხშირი ჩივილი ახალგაზრდებშიც და მოხუცებშიც. წელის ტკივილის დიაგნოზისას მთავარია "სერიოზული" დაავადებები გამოყო ნაკლებ სერიოზულისაგან. სერიოზულებში შედის სიმსივნური და ინფექციური სახის ტკივილები. აგრეთვე ის ტკივილები, რომლებიც განპირობებულია ზურგის ტვინზე ან კაუდა ექვინაზე ზეწოლით(მაგალითად დისკის კომპრესიული თიაქარის დროს). ყველა შემთხვევაში ტკივილის სადიაგნოსტიკოდ საუკეტესო საშუალება არის ბირთვულ მაგნიტური რეზონანსი.

შენ რა ტკივილსაც აღწერ, არ არის სპეციფიური რომელიმე დაავადებისათვის. არამედ უფრო იმაზე მიუთითებს, რომ ნერვზე ან ფესვებზე ზეწოლის გამო არის ტკივილები. ასეთი ტკივილი შეიძლება საჯდომი ნერვზე ზეწოლამ ან გაღიზიანებამაც მოგცეს; შეიძლება მალთაშუახვრების რადიუსის შემცირებამ და შესაბამისად ნერვების ფესვებზე ზეწოლა; შეიძლება ზურგის ტვინში ნერვებზე ზეწოლამ (მაგალითად, ისევ დისკის თიაქარის დროს; ან სპინალურმა სტენოზმა).

რაც შეეხება სახსრების ტკივილს, შეიძლება დაკავშირებული იყოს ერთი და იმავე პროცესთან ან შეიძლება ცალკე პათოლოგია იყოს. სახსრების ტკივილშიც რამოდენიმე სახის ტკივილი ერთმანეთისაგან უნდა განასხვავოს ექიმმა: ოსტეოართრიტი vs. ანთებითი დაავადებები. შენი ლაბორატორიების მიხედვით ანთებითი დაავადება ნაკლებ სავარაუდოა. თუ მარტო ქვემო კიდურების სახსრების ტკივილები გაქვს, შეიძლება უბრალოდ გადაცემული ტკივილია (ზურგიდან) და სხვა არაფერი.

მოკლედ MRI-ის გაკეტება უნდა და ბევრ რამეს გამორიცხავს ან დიაგნოზს დასვამს.

Posted by: თეოგოგო 23 Feb 2008, 12:55
vano_t
მადლობთ პასუხისთვის.. მეც დაახლოებით მასეთი დიაგნოზი დავუსვი ჩემს თავსაც, მაგრამ არაკომპეტენტურობის გამო თავს ვიკავებდი, მსგავსი დასკვნისგან.. sad.gif
მომიწევს მაგნიტოს გაკეთება. ტკივილები არ იკლებს... მოძრაობაში იმატებს, პირიქით..
თიაქარს გამოვრიცხავო, კი თქვა ექიმმა, მაგრამ, რავიცი.. მოკლედ, მადლობთ კიდევ ერთხელ და.. მივაკითხავ სხვა ექიმსuser.gif

Posted by: vano_t 23 Feb 2008, 13:27
თეოგოგო
QUOTE
მეც დაახლოებით მასეთი დიაგნოზი დავუსვი ჩემს თავსაც, მაგრამ არაკომპეტენტურობის გამო თავს ვიკავებდი, მსგავსი დასკვნისგან.. sad.gif
მომიწევს მაგნიტოს გაკეთება. ტკივილები არ იკლებს... მოძრაობაში იმატებს, პირიქით..
თიაქარს გამოვრიცხავო, კი თქვა ექიმმა, მაგრამ, რავიცი..

სანამ MRI-ს არ გააკეთებენ, ვერაფერს ვერ გამორიცხავ. ეს არ ნიშნავს, რომ MRI აუცილებლად უნდა გაკეთდეს. მაგ მეთოდის აბსოლიტური ჩვენება იქნებოდა თუ, მაგალითად, ავადმყოფს ზურგის ტვინზე კომპრესიის ან კაუდა ექვინას ნიშნები აქვს (მაგალითად ქვემო კიდურების სისუსტე, ან შეუკავებლობა). თუმცა დიაგნოზისათვის, ყველაზე სენსიტიური და სპეციფიური ტესტია.

http://www.emedicine.com/emerg/topic50.htm არის კარგად განხილული ზურგის ტკივილის პრაქტიკული ასპექტები. გამოგადგება თუ წაიკითხავ. თუ კონკრეტული კითხვა დაგებადება და დახმარება გინდა, დაგეხმარები.

QUOTE
Nerve root syndromes

Classic nerve root syndrome is characterized by radicular pain arising from nerve root impingement due to herniated disks. A similar syndrome can also be produced by inflammation and irritation, which may be an explanation of why patients whose presentation may be consistent with this diagnosis respond to conservative therapies.

Impingement pain tends to be sharp, well localized, and can be associated with paresthesia, whereas irritation tends to be dull, poorly localized, and without paresthesia. Impingement is associated with a positive straight leg raising sign (ie, shooting pain down contralateral leg with leg raising), while irritation is not. Neurologic deficits and pain radiation below the knee are rarely seen in irritation alone and are most commonly found with impingement.

The cause of impingement syndrome is most commonly from herniated disks, but it may also be from spinal stenosis, spinal degeneration, and cauda equina syndrome.


http://www.spineuniverse.com/displayarticle.php/article1469.html კარგი ინფორმაციაა.

Posted by: სექსტეტი 23 Feb 2008, 19:58
კეთილო ხალხო, ხომ არ იცით, რისგან შეიძლება იყოს გამოწვეული უმიზეზო სიცხე (სისულელე ვთქვი ახლა, მაგრამ უმიზეზოში ვგულისხმობ, რომ არც გაციებული ვარ, არც გრიპი მაქვს და ა.შ.). პერიოდულად, კვირაში ერთხელ ან ორჯერ, სიცხეს მაძლევს, და დამწევებით ჩვეულებრივად რეგულირდება _ იშვიათად რომ ორი-სამი დღით გაგრძელდეს.

რა შეიძლება იყოს მიზეზი, და ეშველება თუ არა ამას რამე? user.gif

Posted by: vano_t 24 Feb 2008, 08:43
ს ე ქ ს ტ ე ტ ი
QUOTE
კეთილო ხალხო, ხომ არ იცით, რისგან შეიძლება იყოს გამოწვეული უმიზეზო სიცხე (სისულელე ვთქვი ახლა, მაგრამ უმიზეზოში ვგულისხმობ, რომ არც გაციებული ვარ, არც გრიპი მაქვს და ა.შ.). პერიოდულად, კვირაში ერთხელ ან ორჯერ, სიცხეს მაძლევს, და დამწევებით ჩვეულებრივად რეგულირდება _ იშვიათად რომ ორი-სამი დღით გაგრძელდეს.

რა შეიძლება იყოს მიზეზი, და ეშველება თუ არა ამას რამე? user.gif

სიცხის მიზეზი უამრავია. მიზეზების ჩამოთვლა ძალიან შორს წაგვიყვანს.

მთავრია შენ რას ეძახი სიცხეს? რატომ იზმოვა სიცხეს? გაწუხებს თუ არა სხვა რამ, ანუ სხვა ჩივილები გაქვს თუ არა?

Posted by: სექსტეტი 24 Feb 2008, 11:07
vano_t

არა, სხვა არაფერი მაწუხებს (მგონი), სიცხეს კი იმიტომ ვიზომავ, რომ თავი მტკივა, სუსტად ვარ და მამცივნებს. სხვა შემთხვევაში, როცა თავის ტკივილს გულისრევა ერთვის (თან ამ დროს სხვანაირდა მტკივა თავი), წნევას ვიზომავ და დაბალი მაქვს ხოლმე. მოკლედ, სხვა "ჩივილი" რაც შეიძლება იყოს, მხოლოდ დაბალი წნევაა _ ესეც კონკრეტული მიზეზების გამო, როცა, მაგალითან, 3 ღამეს მიყოლებით ვათენებ, საჭმელს არ ვჭამ და ა.შ. აი სიცხეს კი ვერაფერი გავუგე sad.gif

Posted by: vano_t 24 Feb 2008, 11:55
ს ე ქ ს ტ ე ტ ი
QUOTE
არა, სხვა არაფერი მაწუხებს (მგონი), სიცხეს კი იმიტომ ვიზომავ, რომ თავი მტკივა, სუსტად ვარ და მამცივნებს. სხვა შემთხვევაში, როცა თავის ტკივილს გულისრევა ერთვის (თან ამ დროს სხვანაირდა მტკივა თავი), წნევას ვიზომავ და დაბალი მაქვს ხოლმე. მოკლედ, სხვა "ჩივილი" რაც შეიძლება იყოს, მხოლოდ დაბალი წნევაა _ ესეც კონკრეტული მიზეზების გამო, როცა, მაგალითან, 3 ღამეს მიყოლებით ვათენებ, საჭმელს არ ვჭამ და ა.შ. აი სიცხეს კი ვერაფერი გავუგე sad.gif

თავის ტკივილი და გულისრევები თუ გაქვს, მაშინ აუცილებლად უნდა ეჩვენო ექიმს, იმის გამო კი არა, რომ რაიმე სერიოზული შეიძლება იყოს (ნაწილობრივ ამის გამოც), არამედ იმიტომ რომ თავის ტკივილის დიაგნოზი დასვას ექიმმა. თავის ტკივილი როგორია, შეგიძლია აღწერო? ან თუ ახლავს სინათლის შიში (უფრო სწორად სინათლეზე გაღიზიანება) ან ხმაურზე გაღიზიანება? სად არის ტკივილები(თავის რომელ არეში)? ძილის დროს თუ გაღვიძებს? რაიმე თუ გიხნის ტკივილს? რაც შიძლება დეტალურად აღწერე ყველაფერი.

და სიცხეები რამხლეა გაქვს? ისე თუ გაგიზომია სიცხე, სხვა დროს როცა თავის ტკივილი არ გაქვს და რამდენია მაგ დროს სიცხე?

Posted by: სექსტეტი 24 Feb 2008, 12:03
vano_t

PM 2kiss.gif

...............................................

Posted by: kobalsky 28 Feb 2008, 19:12
ლიდოკაინი ვენაში არ კეთდება და თუ გაკეთდა რა პრობლემაა?
ლიდოკინის სტანდარტული დოზა კუნთში რამდენია?
დოზა თუ მეტი აღმოჩნდა მაშინ რა შედეგს გამოიწვევს?

Posted by: vano_t 28 Feb 2008, 22:49
kobalsky
QUOTE
ლიდოკაინი ვენაში არ კეთდება და თუ გაკეთდა რა პრობლემაა?
ლიდოკინის სტანდარტული დოზა კუნთში რამდენია?
დოზა თუ მეტი აღმოჩნდა მაშინ რა შედეგს გამოიწვევს?

ინტრავენურად ლიდოკაინი კი კეთდება, ოღონდ ერთი ჩვენება აქვს მარტო; კერძოდ ვენტრიკულარული არითმია. ამ ჩვენების დროს ლიდოკაინი კეთდება ერთჯერად დოზას პლიუს ე.წ. maintenance დოზა (ანუ წვეთოვანის პონტი). ზუსტი დოზა არ მახსოვს. კუნთში ლიდოკაინი ვენტრიკულარული არითმიების დროს არავითარ შემთხვევაში კეთდება. ისე თუ ვინმე ადგილობრივ ანესთეზიას აკეთებს (მაგალითად კუნთის ბიოფსიისას), მაშინ შეიძლება ლიდოკაინი გასაყუჩებლად გაუკეთო იმ არეში, სადაც ოპერაციას აკეთებ.

ლიდოკაინი არის ანტიარითმიული საშუალება (ადგილობრივი ანესთეტიკიც არის, რა თქმა უნდა). როგორც ყველა ანტიარითმიულ საშუალებას, ლიდოკაინსაც შეუძლია გამოიწვიოს არითმიები გართულების სახით, ანუ პროარითმიული ეფექტი გააჩნია. ამიტომ ვენტრიკულური არითმიების დროს ლიდოკაინს ხმარობენ მხოლოდ ინტენსიურ განყოფილებებში გამოცდილი ექიმები.

Posted by: Ultra Cosmopolitan 29 Feb 2008, 01:16
vano_t

შეგიძლიათ მიპასუხოთ რა პრე სიმპტომები ახასიათებს ინსულტს?

Posted by: vano_t 29 Feb 2008, 02:35
Ultra Cosmopolitan
QUOTE
შეგიძლიათ მიპასუხოთ რა პრე სიმპტომები ახასიათებს ინსულტს?

ინსულტის პრესიმპტომები არ გამიგია ნამდვილად. არ არსებობს მასეთი სიმპტომები რომ გითხრას ამ ადამიანს დღეს ან ხვალ ინსულტი დაემართება.

Posted by: MyLifeOnCut 29 Feb 2008, 08:32
დღეს დამეწყო მარჯვენა მხარეს ყურის ზემოთ თავში shooting pains რამოდენიმე წამიანი.. ხომ არ იცით რითია ეს გამოწვეული ან რას უნდა ნიშნავდეს? sad.gif






........

Posted by: kobalsky 29 Feb 2008, 13:44
vano_t
მადლობა smile.gif                                                    

Posted by: vano_t 29 Feb 2008, 20:47
MyLifeOnCut
QUOTE
დღეს დამეწყო მარჯვენა მხარეს ყურის ზემოთ თავში shooting pains რამოდენიმე წამიანი.. ხომ არ იცით რითია ეს გამოწვეული ან რას უნდა ნიშნავდეს? sad.gif

არა, თავის ტკივილს უამრავი დაავადება იწვევს და ასე ადვილი არ არის თქმა რისი ბრალია. ექიმის ნახვა მოგიხდება. ერთ რამეს გეტყვი, თუ ტკივილი გაივლის წამლით და აღარ განმეორდება (ან იშვიათად განმეორდება) და არ გაქვს კისრის გაშეშება, გულისრევა, სიცხე, ცალმხრივი სისუსტე კიდურებში, მაშინ შეგიძლია არ ნახო ექიმი. თუ თავის ტკივილი მოკლე ხანს გრძელდება და არანარკოტიკული ტკივილის წამლები აყუჩებენ, შერიოზული დაავადება არ იქნება.

Posted by: mtvareuli 1 Mar 2008, 14:41
განყოფილებაში არჩევნების დაწყებასთან დაკავშირებით დროებით უნდა ჩამოვპინო აპინული თემები..... აქტუალური თემებია და პირველ გვერდს მაინც არ გასცდება, ასე რომ, არამგონია ვერ იპოვოთ ან რამე სხვა პრობლემები შეგექმნათ ამასთან დაკავშირებით. არჩევნების დასრულებისთანავე თემები კვლავ აიპინება.

Posted by: Guardian 2 Mar 2008, 00:27
მინდა დიდი მადლობა გადავუხადო vano_t-ს იმ დახმარებისათვის, რომელიც მან გამიწია პირადად მე ამ თემის გახსნით.
კიდევ უფრო და ბევრად უფრო დიდი მადლობა მას იმისთვის, რომ უაღრესად კვალიფიციურ და ამომწურავ პასუხებს სცემდა იუზერთა შეკითხვებს.
ვანო, მაგარი კაცი ხარ! up.gif

Posted by: SOULFUL 2 Mar 2008, 01:59
Guardian

QUOTE
ვანო, მაგარი კაცი ხარ!



შენც მაგარი კაცი იქნები, თუ არ დაივიწყებ ამ განყოფილებას, ორჯერ წავიკითხე შენი ნიკი, მეგონა მეჩვენებოდა biggrin.gif როგორ ხარ?

Posted by: MyLifeOnCut 3 Mar 2008, 06:36
vano_t
მადლობა ხო იმ დღის მერე მეტი არ განმეორებულა smile.gif

Posted by: Sly 4 Mar 2008, 17:56
აქ თემა გავხსენი და დამიხურეს და აქ ვიკითხავ smile.gif
ანუ უკვე 1 წელია მაწუხებს მაგრამ რა ვერ ვხვდები, ცხვირიდან პირში(იმ ადგილიდან სურათზე რომ აღვნიშნე) გადამდის რაღაც ლორწოს მაგვარი, ცხვირიდან არ მომდის და იქნებ ვინმემ იცოდეთ რა შეიძლება იყოს და რის ექიმს უნდა მივმართო მადლობა წინასწარ smile.gif
http://123.ge/

Posted by: vano_t 4 Mar 2008, 21:47
Sly
QUOTE
აქ თემა გავხსენი და დამიხურეს და აქ ვიკითხავ smile.gif
ანუ უკვე 1 წელია მაწუხებს მაგრამ რა ვერ ვხვდები, ცხვირიდან პირში(იმ ადგილიდან სურათზე რომ აღვნიშნე) გადამდის რაღაც ლორწოს მაგვარი, ცხვირიდან არ მომდის და იქნებ ვინმემ იცოდეთ რა შეიძლება იყოს და რის ექიმს უნდა მივმართო მადლობა წინასწარ smile.gif

მაგას ინგლისურად ეწოდება post-nasal drip (ანუ ცხვირის უკან წვეთა-ალბათ ქართულად ასე შეიძლება ჟღერდეს). ძალიან ხშირი პრობლემაა ეგ საერთოდ. ცხვირის, ცხვირხახის და პირხახის ლორწოვანი გარსი დაფარულია სპეციალური უჯრედებით, რომელსაც წამწამოვანი ეპითელიუმი ქვია. ამ უჯრედებს მიკროსკოპული "წამწამები" აქვთ. ამ უჯრედების ფუნქციაა, რომ მცირე რაოდენობით ტენი გამოყონ და ეს ტენი შემდეგ მოაცილონ წამწამების საშუალებით მაგ გარემოს. როცა გამოყოფილი ტენი ძალიან სქელდება (განსაკუთრებით სიმშრალის გამო), ამ გასქელებული ლორწოს მოცილება ვეღარ ხდება და ეს ლორწო აღიზიანებს ხახის უკანა კედელს. რამოდენიმე რჩევას შეიძლება მოგცემ. თუ სიგარეტს ეწევი, თავი დაანებე და ნელ-ნელა ყველაფერი გადავა. ცხელი სითხეები (ცხელი ჩაი განსაკუთრებით-ისეთი ცხელი არა ოღონდ პირი რომ დაგწვას) ლორწოვანის დატენიანებასაც უწყობს ხელს და მიკროსკოპული "წამწამების" კარგად მუშაობასაც. თუ ინფექცია გაქვს, მაშინ ანტიბიოტიკი დაჭირდება ცოტა ხანი. კარგი შედეგი აქვს ცხვირის ღრუს ირიგაციას ფიზიოლოგიური ხსნარით. ფიზიოლოგიური ხსნარი შენ თვითონაც შეგიძლია მოამზადო თუ გამოხდილ წყალს ჩვეულებრივ მარილს მიამატებით, ისე რომ 0.9 % -იანი ხსნარი მიიღო(9 გ. მარილი 1 ლ. წყალზე). ფიზიოლოგიური ხსნარით ირიგაცია ან რაიმე ხელსაწყოს მეშვეობით შეიძლება, ან შეგიძლია ხსნარი მუჭში დააგროვო და შემდეგ ეს ხსნარი რითმულად "შეისუნთქო და ამოისუნთქო".

აი, რამოდენიმე ლინკი ინგლისურად:
http://www.ent-consult.com/faq.html#irrigation
http://www.ent-consult.com/pnd.html
http://ezinearticles.com/?Post-Nasal-Drip-Remedy&id=778439

Posted by: LULA_QABABI 4 Mar 2008, 22:12
QUOTE (vano_t @ 4 Mar 2008, 12:47 )
კარგი შედეგი აქვს ცხვირის ღრუს ირიგაციას ფიზიოლოგიური ხსნარით. ფიზიოლოგიური ხსნარი შენ თვითონაც შეგიძლია მოამზადო თუ გამოხდილ წყალს ჩვეულებრივ მარილს მიამატებით, ისე რომ 0.9 % -იანი ხსნარი მიიღო(9 გ. მარილი 1 ლ. წყალზე). ფიზიოლოგიური ხსნარით ირიგაცია ან რაიმე ხელსაწყოს მეშვეობით შეიძლება, ან შეგიძლია ხსნარი მუჭში დააგროვო და შემდეგ ეს ხსნარი რითმულად "შეისუნთქო და ამოისუნთქო".

აი, რამოდენიმე ლინკი ინგლისურად:
http://www.ent-consult.com/faq.html#irrigation
http://www.ent-consult.com/pnd.html
http://ezinearticles.com/?Post-Nasal-Drip-Remedy&id=778439

ვანოს გაუმარჯოს! ხო ხარ მაგრა.

ირიგაციაზე ერთს დავაზუსტებ, მნიშვნელოვანი მომენტია მონაცვლეობით შესუნთქვა ერთ ნესტოში და გამორეცხვა მეორედან (არ შეგეშინდეთ, ერთმანეთთან დაკავშირებულები არიან). ნოტიო-ზაციის (რავარია) გარდა ხდება ალერგენების გამორეცხვა. განსაკუთრებუთ თუ ალერგიული რინიტი გაწუხებთ

Posted by: Sly 4 Mar 2008, 22:36
სპ და მაგის გამო შეიძლება ლიმფური კვანძი მქონდეს გადიდებული? გადიდებული მაქ და ექიმთან ვაპირებ მისვლას და რის ექიმთან უნდა მივიდე რა ქვია ცხვირის ექიმს? და კარგ ექიმს ხომ ვერ მირჩევთ?

Posted by: vano_t 5 Mar 2008, 02:19
LULA_QABABI
QUOTE
ვანოს გაუმარჯოს! ხო ხარ მაგრა.

  ირიგაციაზე ერთს დავაზუსტებ, მნიშვნელოვანი მომენტია მონაცვლეობით შესუნთქვა ერთ ნესტოში და გამორეცხვა მეორედან (არ შეგეშინდეთ, ერთმანეთთან დაკავშირებულები არიან).  ნოტიო-ზაციის (რავარია) გარდა ხდება ალერგენების გამორეცხვა. განსაკუთრებუთ თუ ალერგიული რინიტი გაწუხებთ

ვა, გაუმარჯოს ლადოს. რავა ხარ თუ იცი? ნოტიო-ზაცია კაია wink.gif ირიგაციაზე ნამდვილად მასე უნდა: ნესტოების მონაცვლეობით.

Sly
QUOTE
მაგის გამო შეიძლება ლიმფური კვანძი მქონდეს გადიდებული? გადიდებული მაქ და ექიმთან ვაპირებ მისვლას და რის ექიმთან უნდა მივიდე რა ქვია ცხვირის ექიმს? და კარგ ექიმს ხომ ვერ მირჩევთ?
შესაძლებელია მაგის გმო გქონდეს გადიდებული, მაგრამ მაინც ეჩვენე ექიმს. წესით ნორმალურმა თერაპევტმა უნდა დაგიდგინოს დიაგნოზი და სცადოს საწყისი მკურნალობა. თუ არა და, ყველყურცხვირის ექიმს ეჩვენე.

Posted by: Sly 5 Mar 2008, 08:20
ოკ მადლობა wink.gif

Posted by: MALI 5 Mar 2008, 10:54
vano_t
რა პრინციპით კეთდება ბიცილინ-5?
რომ სტაციონარებშიც კი ან ძლივს, ან საერთოდ ვერ მიკეთებენ? (მაინც მირჩიეს ყოველთვიური ინექცია 2 წლის განმავლობაში)

Posted by: LULA_QABABI 5 Mar 2008, 11:13
QUOTE (MALI @ 5 Mar 2008, 01:54 )
vano_t
რა პრინციპით კეთდება ბიცილინ-5?
რომ სტაციონარებშიც კი ან ძლივს, ან საერთოდ ვერ მიკეთებენ? (მაინც მირჩიეს ყოველთვიური ინექცია 2 წლის განმავლობაში)

ვანოს თანხმობით მე გაგცემ პასუხს, რომლის სახალხო გარჩევას არ გირჩევ wink.gif

Posted by: lizi11 5 Mar 2008, 15:00
vano_t
რას ნიშნავს ნაწლავების ატონია?

Posted by: MALI 5 Mar 2008, 15:26
LULA_QABABI
რატომ ? რეციდული წითელი ქარის შემთხვევაში დამინიშნეს უბრალოდ ვერ მიკეთებენ კარგად. სტაციონარში ექთნები თვითონაც წვალობენ და მეც მაწვალებენ. იყინება ხშირად.
მე მაინტერესებს არ არსებობს რამე ისეთი პრინციპი რომ წამალი არ გაიყინოს?! ზეთოვანი არ არის...

Posted by: vano_t 5 Mar 2008, 19:38
lizi11
QUOTE
vano_t
რას ნიშნავს ნაწლავების ატონია?

არ ვიცი რა არის ნაწლავის ატონია. შეკრულობა, ილეუსი, ნაწლავების ობსტრუქცია გამიგია. ნაწლავების დაავადებები გამიგია, რომლის დროსაც ნაწლავის პერისტალტიკა არის დაქვეითებული (მაგალითად ჰირშპრუნგის დაავადება). ატონია, როგორც ასეთი არ გამიგია. შეიძლება ტერმინი არსებობს, მაგრამ თუ ეგ ტერმინი რამეს მიუთითებს, ალბათ იმას რომ ნაწლავების გლუვი კუნთები (smooth muscles) არ იკუმშება სათანადოდ. მასეთი დიაგნოზი, როგორც ასეთი, ალბათ არ არსებობს.

MALI
QUOTE
რა პრინციპით კეთდება ბიცილინ-5?
რომ სტაციონარებშიც კი ან ძლივს, ან საერთოდ ვერ მიკეთებენ? (მაინც მირჩიეს ყოველთვიური ინექცია 2 წლის განმავლობაში)
LULA_QABABI-ზე უკეთესად მაგ კითხვაზე(და საერთოდ ინფექციური დაავადებების) პასუხს ვერავინ გაგცემთ. ამიტომ მის რჩევებს უფრო გავითვალისწინებდი შენს ადგილას.

მაგას რა ჯობია ლადო თუ შემოგვეხმარები smile.gif ველი კიდევ უფრო მეტ აქტიურობას შენგან.

Posted by: MALI 5 Mar 2008, 20:47
vano_t
მე LULA_QABABI- ის რჩევა ვერსად წავიკითხე
უკაცრავად, მე თქვენი ხუმრობა ვერ გავიგე......

Posted by: LULA_QABABI 5 Mar 2008, 21:06
QUOTE (MALI @ 5 Mar 2008, 11:47 )
vano_t
მე LULA_QABABI- ის რჩევა ვერსად წავიკითხე
უკაცრავად, მე თქვენი ხუმრობა ვერ გავიგე......

არ მიხუმრია, ბიცილინს უმეტეს შემთხვევებში (აბსოლუტური უმრავლესობა) სიფილისის დროს ნიშნავენ. და არ გირჩიე ამ დაავადების სახალხოდ გარჩევა. აღმოჩნდა რომ "რეციდივულ წითელ ქართან" გვაქვს საქმე.
დანარჩენი პმ-ში მოგწერე.

ვანო, მადლობა კეთილი სიტყვებისთვის wink.gif

Posted by: MALI 5 Mar 2008, 21:15
LULA_QABABI
მადლობთ, ნამდვილად რჩევა გინდოდა smile.gif
მე vano_t ს იმიტომ მივმართე, რომ ადრე წითელ ქართან დაკავშირებით დმეხმარა
ჩემთვის მნიშვნელობა არ აქვს, რომელი მირჩევთ მირჩევდით
დიდი მადლობა ორივეს... smile.gif

Posted by: Cor-toni 5 Mar 2008, 22:45
QUOTE
ატონია, როგორც ასეთი არ გამიგია.


spy.gif

ატონია არის ჩონჩხის კუნთებისა და შინაგან ორგანოებში (კუჭში, ნაწლავებში, საშვილოსნოში) ბუნებრივი დაჭიმულობის, ელასტიკურობისა და შეკუმშვის უნარის შესუსტება.

LULA_QABABI

ბიცილინი რევმატიზმის სამკურნალოდაც ინიშნება smile.gif

Posted by: MyLifeOnCut 6 Mar 2008, 08:25
vano_t
შეკითხვა მაქვს, დედაჩემს ავაარიის შემდეგ კისერი ტკივა მარჯვენა მხარეს და მოკლედ ეხლა ექიმა უთხრა epidermal injection ს გავაკეთებთო... და მაინტერესებს რამდენად უსაფრთხოა? არის რაიმე რაც უნდა ვიცოდე წინასწარ.. მადლობა smile.gif

Posted by: LULA_QABABI 6 Mar 2008, 09:53
QUOTE (cor-toni @ 5 Mar 2008, 13:45 )
QUOTE

LULA_QABABI

ბიცილინი რევმატიზმის სამკურნალოდაც ინიშნება smile.gif



ერთ რამეს დავაზუსტებ, რასაც არ მივაქციე ყურადღება; მე ვიცი ერთი სახის ბიცილინი რომელიც აქ (აშშ) იხმარება - იგივე Benzathin Pencillin G.
საქართველოში თუ სხვა ფორმებია ბიცილინის (1, 2...5?) მაგას ვერ მოგახსენებთ. მაშინ ჯობია generic სახელით მითხრათ.
Benzathin PCN G-ს ხმარება თეორიულად შეიძლება მწვავე A ჯგუფის სტრეპტოკოკული (და სხვა პენიცილინ-სენსიტიური) ინფექციების დროს, თუმცა ძლიერი პერორალური ანტიბიოტიკების არჩევანის პირობებში მთლად ჰუმანური არ უნდა იყოს დუნდულოთა ჩხვლეტა ამ უკანასკნელთათვის, მითუმეტეს თუ მკურნალობა ამბოლატორიულადაც შეიძლება.

ალბათ გინდოდა გეთქვა რევმატული ცხელება ვინაიდან "რევმატიზმი" უფრო ხალხური ტერმინია და ხშირად რევმატოიდული ართრიტის მისამართითაც იხმარება.

Posted by: MALI 6 Mar 2008, 14:08
QUOTE (LULA_QABABI @ 6 Mar 2008, 08:53 )
QUOTE (cor-toni @ 5 Mar 2008, 13:45 )
QUOTE

LULA_QABABI

ბიცილინი რევმატიზმის სამკურნალოდაც ინიშნება smile.gif



ერთ რამეს დავაზუსტებ, რასაც არ მივაქციე ყურადღება; მე ვიცი ერთი სახის ბიცილინი რომელიც აქ (აშშ) იხმარება - იგივე Benzathin Pencillin G.
საქართველოში თუ სხვა ფორმებია ბიცილინის (1, 2...5?) მაგას ვერ მოგახსენებთ. მაშინ ჯობია generic სახელით მითხრათ.
Benzathin PCN G-ს ხმარება თეორიულად შეიძლება მწვავე A ჯგუფის სტრეპტოკოკული (და სხვა პენიცილინ-სენსიტიური) ინფექციების დროს, თუმცა ძლიერი პერორალური ანტიბიოტიკების არჩევანის პირობებში მთლად ჰუმანური არ უნდა იყოს დუნდულოთა ჩხვლეტა ამ უკანასკნელთათვის, მითუმეტეს თუ მკურნალობა ამბოლატორიულადაც შეიძლება.

ალბათ გინდოდა გეთქვა რევმატული ცხელება ვინაიდან "რევმატიზმი" უფრო ხალხური ტერმინია და ხშირად რევმატოიდული ართრიტის მისამართითაც იხმარება.

LULA-QABABI
ა.შშ ხარ?
მე ნოემბერში მოვდივარ საცხოვრებლად აშშ- ში
აბა გამოდის ბიცილინ - 5 ს მანდ სხვა დასახელება აქვს?
მედიკოსი არ ვარ და ზუსტი დასახელებები არ ვიცი
srt. anhaymoliticus - აი ეს სტრეპტოკოკია და რომელი ჯგუფის არ ვიცი..

Posted by: Cor-toni 6 Mar 2008, 16:01
LULA_QABABI

QUOTE
ალბათ გინდოდა გეთქვა რევმატული ცხელება ვინაიდან "რევმატიზმი" უფრო ხალხური ტერმინია და ხშირად რევმატოიდული ართრიტის მისამართითაც იხმარება.


yes.gif რევმატოიდული ცხელება მინდოდა მეთქვა biggrin.gif


Posted by: vano_t 6 Mar 2008, 20:12
MyLifeOnCut
QUOTE
vano_t
შეკითხვა მაქვს, დედაჩემს ავაარიის შემდეგ კისერი ტკივა მარჯვენა მხარეს და მოკლედ ეხლა ექიმა უთხრა epidermal injection ს გავაკეთებთო... და მაინტერესებს რამდენად უსაფრთხოა? არის რაიმე რაც უნდა ვიცოდე წინასწარ.. მადლობა smile.gif

epidermal injection არ ქვია მაგას. მაგის სახელია ეპიდურული ინექცია (epidural injection) smile.gif კისრის არეში უფრო სახიფათოა ვიდრე წელის არეში სადაც ზურგის ტვინი უკვე დამთავრებულია და მხოლოდ ნერვებია დარჩენილი. გამოცდილი ექიმის ხელში გართულებების რისკი მცირეა. თუმცა, ეპიდურულ ინექციამდე სხვა რამეების შეიძლება გაკეთდეს. პირველ რიგში დიაგნოზი უნდა დაისვას, რატომ ტკივა კისერი (ლისთეზი განუვითარდა, მალის მოტეხილობაა, აქსისის მოტრხილობაა, ნერვებზე ზეწოლაა თუ რა?). ავარიის შემდეგ დიაგნოზს განსაკუთრებული მნიშვნელობა აქვს. თუ გარკვეული ტიპის დაზიანებები გამოირიცხა, მაშინ ფიზიკური თერაპია შეიძლება მოისინჯოს პირველი ან ეპიდურული ინექცია. მე (თუ გარკვეული რამეები გამოირიცხა) პირველად ფიზიკურ თერაპიას დავუნიშნავდი.

Posted by: MyLifeOnCut 6 Mar 2008, 21:24
vano_t
მოტეხილობა არ აქვს, თერაპიაც ჩაუტარდა არ უშველა და ტკვილთან ერთად ეხლა ის აქვს რომ გატიშვის მომენტები ეწყება ხოლმე.. ნემსებიც ცადეს (acupuncture) da is aqvs disc herniation მოკლედ მაგიტო არ ვიცი ეს უშველის მაინც ? ექიმა 1 % ნაკლებიაო რაიმე გართულება მოხვდესო. მადლობა smile.gif

Posted by: vano_t 7 Mar 2008, 04:00
MyLifeOnCut
QUOTE

მოტეხილობა არ აქვს, თერაპიაც ჩაუტარდა არ უშველა და ტკვილთან ერთად ეხლა ის აქვს რომ გატიშვის მომენტები ეწყება ხოლმე.. ნემსებიც ცადეს (acupuncture)  da is aqvs  disc herniation მოკლედ მაგიტო არ ვიცი ეს უშველის მაინც ? ექიმა 1 % ნაკლებიაო რაიმე გართულება მოხვდესო. მადლობა smile.gif

ეპიდურული ინექციები ხშირად შველის ტკივილებს, მაგრამ შეიძლება არც უშველოს ან რამოდენიმე ინექცია დაჭირდეს. მაგ დროს სტეროიდებს უკეთებენ ზურგის ტვინის გარშემო სივრცეში (epidural space). გართულებების გარეშე არცერთი პროცედურა არ არსებობს. ყველაფერს აქვს გართულება და ძალიან კარგი ექიმის ხელშიც. მთავარია ექიმმა მოგცეს ინფორმაცია გართულებების თაობაზე და რამდენად ხშირად ხდება. ამის შემდეგ ავადმყოფის გადასაწყვეტია რა უნდა იმოქმედოს. საზღვარგარეთ (აშშ-ში მაგალითად) მასეთი პროცედურების ჩატარების წინ ავადმყოფი აწერს ხელს ე.წ. informed consent-ს. რაც იმას ნიშნავს, რომ ავადმყოფს აუხსნა ექიმმა პროცედურის არსი, მისი ჩვენება, გართულებები და მათი სიხშირე, ალტერნატიული მკურნალობის მეთოდები.

Posted by: MyLifeOnCut 7 Mar 2008, 04:13
vano_t
ხოო მესმის ყველაფერი... მადლობა და მაინც რა რჩევას მომცემდი რო ჯობია გაუკეთონ?




..........

Posted by: vano_t 7 Mar 2008, 04:20
MyLifeOnCut
QUOTE
ხოო მესმის ყველაფერი... მადლობა და მაინც რა რჩევას მომცემდი რო ჯობია გაუკეთონ?

რასაც გირჩევს ექიმი ის. მე უბრალოდ ზოგადი ინფორმაცია მოგაწოდე. ყველაფერი მკურნალი ექიმის და ავადმყოფის გადასაწყვეტია. თუ კონკრეტული კითხვა გაქვს, შევეცდები გაგცე პასუხი. მაგრამ მკურნალობის არჩევა პაციენტის გადასაწყვეტია. ერთი რამე შემიძლია გითხრა: მასეთი პროცედურას ბევრი პაციენტი იკეთებს და უმეტესობა კმაყოფილია. თუმცა არიან უკმაყოფილოებიც ან იმის გამო, რომ არ უშველა პროცედურამ ან რაღაც გართულება დაემართა.

Posted by: MyLifeOnCut 7 Mar 2008, 04:25
vano_t


მადლობა wink.gif მერე დავწერ შედეგს smile.gif



....................

Posted by: vano_t 7 Mar 2008, 04:29
MyLifeOnCut
QUOTE
მადლობა wink.gif მერე დავწერ შედეგს smile.gif

აბა გისურვებთ წარმატებებს smile.gif

Posted by: LULA_QABABI 7 Mar 2008, 08:26
QUOTE (MALI @ 6 Mar 2008, 05:08 )

srt. anhaymoliticus - აი ეს სტრეპტოკოკია და რომელი ჯგუფის არ ვიცი..

ეგეთი რამ ბუნებაში არ არსებობს.
ალბათ გულისხმობ Group A beta-haemolytic strep, იგივე S. pyogenes

Posted by: MALI 7 Mar 2008, 10:04
............. რა ვიცი, რა ვიცი
ანალიზის პასუხში მხოლოდ ეს წერია. sad.gif

Posted by: lizi11 7 Mar 2008, 15:41
cor-toni
გმადლობ. smile.gif
კიდევ მაქვს კითხვა - რა დაავადებები შეიძლება გამოიწვიოს თირკმელებში მარილების ჭარბმა რაოდენობამ? რა სიმპტომებით ხასიათდება?
- რა გარეგნული ნიშნებით ხასიათდება ღვიძლის ფუნქციების მოშლა?

და კიდევ, ჩემი მოკრძალებული აზრით, კარგი იქნება ზოგადად განიხილებოდეს ცალკეული დაავადებები და მათი სიმპტომები. უფრო საინტერესო გახდებოდა განყოფილება. wink.gif

Posted by: vano_t 7 Mar 2008, 21:28
lizi11
QUOTE
კიდევ მაქვს კითხვა - რა დაავადებები შეიძლება გამოიწვიოს თირკმელებში მარილების ჭარბმა რაოდენობამ?
"თირკმელში მარილები" არის სახალხო ტერმინი. მედიცინაში მასეთ ტერმინს არ ხმარობენ. არსებობს თირკმლის კენჭოვანი დაავადება, როცა თირკმელში და საშარდე გზებში კენჭების წარმოქმნა ხდება. კენჭები ძალიან სხვადსხვა შედგენილობისაა და უამრავ დაავადებას ახლავს თან (მაგალითად გარკვეული ტიპის ინფექციებს, ჰიპერურიკემიით მიმდინარე მდგომარეობებს, ჰიპეროქსალატემიებს და ასე შემდეგ). თვითონ კენჭი თირკმელში რაიმე დაავადებას არ გამოიწვევს. თუ კენჭმა საშარდე გზების სანათური დაახშო ინფექცია შეიძლება განვითარდეს ობსტრუქციის პროქსიმალურად.

QUOTE
რა სიმპტომებით ხასიათდება?
რას გულისხმობ: რა სიმპტომებით ხასიათდება თირკმლის კენჭოვანი დაავადება და მისი გართულება?

QUOTE
რა გარეგნული ნიშნებით ხასიათდება ღვიძლის ფუნქციების მოშლა?
ძალიან ზოგადი კითხვაა და თუ შეიძლება დააზუსტე რას გულისხმობ ღვიძლის ფუნქციის მოშლაში?

Posted by: Cor-toni 8 Mar 2008, 13:03
lizi11

თირკმლების კენჭოვან დაავადებას გულისხმობ ალბათ. თუ კენჭმა შარდსაწვეთის სანათური დაახშო, შარდის შეგუბება ხდება თირკმელში (ჰიდრონეფროზი).
ღვიძლის რომელი ფუნქციის მოშლას გულისხმობ?

Posted by: lizi11 9 Mar 2008, 15:41
vano_t
QUOTE
თირკმელში მარილები" არის სახალხო ტერმინი. მედიცინაში მასეთ ტერმინს არ ხმარობენ.

მაგალითად, შარდის ანალიზში პაციენტს დაუწერეს - მარილის კრისტალები დიდი რაოდენობით.
და თუ შეიძლება ამან გამოიწვიოს ორგანიზმის და სახის შეშუპება?

cor-toni
QUOTE
ღვიძლის რომელი ფუნქციის მოშლას გულისხმობ?

თუ არის შესაძლებელი ღვიძლის ფუნქციები ჩამომიყალიბო ზოგადად?
მე მაინტერესებს რომელი ფუნქციის მოშლის შემთხვევაში ჩდება სახეზე ყავისფერი ლაქები, შეშუპება, პერიოდულად თვალის ქუთუთოების შეწითლება და შეშუპება მთელ სახეზე და ტანზე.....


Posted by: vano_t 9 Mar 2008, 17:28
lizi11
QUOTE
მაგალითად, შარდის ანალიზში პაციენტს დაუწერეს - მარილის კრისტალები დიდი რაოდენობით.
და თუ შეიძლება ამან გამოიწვიოს ორგანიზმის და სახის შეშუპება?
კრისტალების ნახვა შარდში თავისთავად არაფერს ამბობს. დიდი რაოდენობით კრისტალები შეიძლება გაუწყლოებისას შეგხვდეს მაგალითად. ამ დროს თირკმელი შეეცდება სითხის შეკავებას და შარდიც შესაბამისად კონცენტრირებული იქნება.

როცა ადამიანს შეშუპება ემართება, რამდენიმე ორგანოს ფუნქციას ამოწმებ. შეშუპებისას ხდება არტერიული სისხლიდან გასულ სითხესა და უკან ვენური სისხლში შემოსულ სითხეს შორის ბალანსის დარღვევა. ნორმაში ამ გასვლა-შემოსვლას შორის ბალანსი არსებობს. როცა ვენებში ზოგადად წნევის მომატება იწყება ან ე.წ. კოლოიდურ-ონკოზური წნევის დაქვეითება იწყება (ამ წნევას განაპირობებს სისხლში ძირითადად ალბუმინების დონე) მაშინ ხდება პლაზმის მომატებული ფილტრაცია ქსოვილებში და ვითარდება შეშუპება. ამიტომ ყველა ზემოთდასახელებული მიზეზები უნდა იქნეს გამოკვლეული. წნევის მომატება ვენებში ხშირად ხდება როცა გულის მარჯვენა მხარე ვერ მუშაობს კარგად და ვერ "ქაჩავს" საკმარისი რაოდენობით სისხლს, რომელიც მასში შემოდის. აგრეთვე, როცა ორგანიზმში სითხის რაოდენობა ზოგადად იზრდება იმის გამო, რომ თირკმელი საკმარისი რაოდენობით სითხეს არ გამოყოფს. ალბუმინების წარმოშობა ღვიძლში ხდება ზოგადად, თუმცა დაბალი დონე შეიძლება მოგცეს აგრეთვე ე.წ. malnutrition-მა (კახექსიამ).

ყველაფერ ამის გამო, შეშუპების მიზეზის დასადგენად ჯერ ერთი ექიმმა დაწვრილებითი ანამნეზი უნდა შეკრიბოს და სრული ფიზიკური გამოკვლევა გაუკეთოს ავადმყოფს და მეორეც, მიმიმუმ შემდეგი ტესტები უნდა გაუკეთოს ავადმყოფს: 1) ექოკარდიოგრაფია, რაც დაადგენს მარჯვენა და მარცენა პარკუჭების შეკუმშვის და მოდუნების ფუნქციებს, სარქველების ფუქციებს და ფილტვის მიმოქცევაში წნევის რაოდენობრივ სიდიდესაც მოგცემს დაახლოებით (მაგალითად ფილტვის არტერიაში წნევის მომატებმამ შეიძლება მოგცეს შეშუპება, რადგანაც ეს მომატებული წნევა საბოლოოდ ვენებს გადაეცემა); 2)უნდა გაიზომოს კრეატინინი, რომელიც კარგი ტესტია თირკმლების ფუნქციის დასადგენად; 3) გაიზომოს ალბუმინი. როცა ექიმი დაადგენს, რომ ესადაეს ორგანო არის მიზეზი შეშუპების განვითარებისა, ამის შემდეგ დიაგნოზი უნდა დასვას თუ რა იწვევს იმ ორგანოს დაავადებას. მოკლედ ასეა ეს ყველაფერი.

შეშუპება სხვა რამეებმაც შეიძლება გამოიწვიოს. მაგალითად თუ შეშუპება ლოკალურია (დავუშვათ მარტო 1 ქვემო კიდურში), ადგილობრივი სისხლძარღვოვანი პრობლემა შეიძლება იყოს ეგ (ვენების უკმარისობა და ღრმა ვენების თრომბოზი). გენერალიზებული შეშუპება აგრეთვე შეიძლება მოგცეს ალერგიამ, თუმცა ამის გამოცნობა მარტივიც არის იმის გამო, რომ ავადმყოფს კანის ქავილიც ექნება და ურტიკარიული გამონაყარი.

Posted by: ancho 10 Mar 2008, 01:08
ერთი რამე მინდა გკითხოთ. თუ შარდის საერთო ანალიზში ყველაფერი ნორმაშია, შეიძლება თირკმელების დაავადება გამოირიცხოს?

Posted by: vano_t 10 Mar 2008, 07:05
ancho
QUOTE
ერთი რამე მინდა გკითხოთ. თუ შარდის საერთო ანალიზში ყველაფერი ნორმაშია, შეიძლება თირკმელების დაავადება გამოირიცხოს?

ნორმალური შარდის ანალიზი არ გამორიცხავს თირკმელების დაავადებას. უფრო მეტიც, შარდის ანალიზს შეზღუდული შესაძლებლობები აქვს და დაავადებათა მხოლოდ მცირე კატეგორიის დასადგენად იყენებენ. რა თქმა უნდა, როცა თირკმლის დაავადებაზე ფიქრობ, შარდის ანალიზი ყველა შემთხვევაში კეთდება და ნორმალურ ანალიზსაც თავისებური სადიაგნოსტიკო ღირებულება აქვს. გარდა ამისა, არანორმალური შარდის ანალიზი ყოველთვის არ მიუთითებს თირკმლების დაავადებას ან რაიმეს დაავადებას საერთოდ.

შარდის ანალიზთან ერთად, თირკმლის დაავადებების დიაგნოსტიკაში დიდი მნიშვნელობა აქვს კრეატინინის და ელექტროლიტების დადგენას. ასევე დიდი მნიშვნელობა აქვს თირკმლის გამოსახულების ანალიზს (ულტრაბგერა იქნება, კომპიტერული ტომოგრაფია თუ ბირთვულ-მაგნიტური რეზონანსი).

Posted by: lizi11 10 Mar 2008, 13:46
vano_t
QUOTE
2)უნდა გაიზომოს კრეატინინი, რომელიც კარგი ტესტია თირკმლების ფუნქციის დასადგენად; 3) გაიზომოს ალბუმინი.

სისხლიდან ხდება კრეატინინის და ალბუმინის გაზომვა?

Posted by: vano_t 10 Mar 2008, 16:55
lizi11
QUOTE
QUOTE
2)უნდა გაიზომოს კრეატინინი, რომელიც კარგი ტესტია თირკმლების ფუნქციის დასადგენად; 3) გაიზომოს ალბუმინი.

სისხლიდან ხდება კრეატინინის და ალბუმინის გაზომვა?

ორივეს გაზომვა სისხლში ხდება. ალბუმინთან ერთად გააზომინე ამინოტრანსფერაზები და ბილირუბინი. ასევე კრეატინინთან ერთად გააზომინე შარდოვანა და ელექტროლიტები. ამ სახით თირკმლის და ღვიძლის მუშაობაზე უფრო სრულყოფილი წარმოდგენა ხდება.

Posted by: maomao 11 Mar 2008, 01:19
არ ვიცი სად დავცერო,3 მარტს ექიმების დაუდევრობით, ჟვანიას სახელობის ბავშვთა საავადმყოფოში დაიღუპა 5 წლის გოგონა,სწორედ იმის გამო რომ მათ არ აუღეს სისხლის ანალიზი და დროულად ვერ მოხერხდა სეფსისის დადგენა,რის შედეგადაც ბავშვი გარდაიცვალა,ჩემი აზრით მათ პასუხი უნდა აგონ.

Posted by: Natosha 11 Mar 2008, 01:39
maomao
ეგ განცხადება აქეთ: http://forum.ge/?f=43&showtopic=33584978&st=0

Posted by: Camel 11 Mar 2008, 10:23
რა სინდრომები შეიძლება ჰქონდეს ჩიყვს?

Posted by: keen 11 Mar 2008, 11:04
vano_t
სისხლის საერთო ანალიზის შემთხვევაში, სისხლის გამოდინების გაძნელება რისი მიზეზით შეიძლება იყოს? baby.gif

Posted by: lizi11 11 Mar 2008, 14:19
vano_t
QUOTE
ორივეს გაზომვა სისხლში ხდება. ალბუმინთან ერთად გააზომინე ამინოტრანსფერაზები და ბილირუბინი. ასევე კრეატინინთან ერთად გააზომინე შარდოვანა და ელექტროლიტები. ამ სახით თირკმლის და ღვიძლის მუშაობაზე უფრო სრულყოფილი წარმოდგენა ხდება.

ეს ყველაფერი სისხლის საერთო ანალიზში ჩანს? თუ ცალკე კეთდება?

Posted by: Sly 11 Mar 2008, 16:17
QUOTE (vano_t @ 4 Mar 2008, 21:47 )
Sly
QUOTE
აქ თემა გავხსენი და დამიხურეს და აქ ვიკითხავ smile.gif
ანუ უკვე 1 წელია მაწუხებს მაგრამ რა ვერ ვხვდები, ცხვირიდან პირში(იმ ადგილიდან სურათზე რომ აღვნიშნე) გადამდის რაღაც ლორწოს მაგვარი, ცხვირიდან არ მომდის და იქნებ ვინმემ იცოდეთ რა შეიძლება იყოს და რის ექიმს უნდა მივმართო მადლობა წინასწარ smile.gif

მაგას ინგლისურად ეწოდება post-nasal drip (ანუ ცხვირის უკან წვეთა-ალბათ ქართულად ასე შეიძლება ჟღერდეს). ძალიან ხშირი პრობლემაა ეგ საერთოდ. ცხვირის, ცხვირხახის და პირხახის ლორწოვანი გარსი დაფარულია სპეციალური უჯრედებით, რომელსაც წამწამოვანი ეპითელიუმი ქვია. ამ უჯრედებს მიკროსკოპული "წამწამები" აქვთ. ამ უჯრედების ფუნქციაა, რომ მცირე რაოდენობით ტენი გამოყონ და ეს ტენი შემდეგ მოაცილონ წამწამების საშუალებით მაგ გარემოს. როცა გამოყოფილი ტენი ძალიან სქელდება (განსაკუთრებით სიმშრალის გამო), ამ გასქელებული ლორწოს მოცილება ვეღარ ხდება და ეს ლორწო აღიზიანებს ხახის უკანა კედელს. რამოდენიმე რჩევას შეიძლება მოგცემ. თუ სიგარეტს ეწევი, თავი დაანებე და ნელ-ნელა ყველაფერი გადავა. ცხელი სითხეები (ცხელი ჩაი განსაკუთრებით-ისეთი ცხელი არა ოღონდ პირი რომ დაგწვას) ლორწოვანის დატენიანებასაც უწყობს ხელს და მიკროსკოპული "წამწამების" კარგად მუშაობასაც. თუ ინფექცია გაქვს, მაშინ ანტიბიოტიკი დაჭირდება ცოტა ხანი. კარგი შედეგი აქვს ცხვირის ღრუს ირიგაციას ფიზიოლოგიური ხსნარით. ფიზიოლოგიური ხსნარი შენ თვითონაც შეგიძლია მოამზადო თუ გამოხდილ წყალს ჩვეულებრივ მარილს მიამატებით, ისე რომ 0.9 % -იანი ხსნარი მიიღო(9 გ. მარილი 1 ლ. წყალზე). ფიზიოლოგიური ხსნარით ირიგაცია ან რაიმე ხელსაწყოს მეშვეობით შეიძლება, ან შეგიძლია ხსნარი მუჭში დააგროვო და შემდეგ ეს ხსნარი რითმულად "შეისუნთქო და ამოისუნთქო".

აი, რამოდენიმე ლინკი ინგლისურად:
http://www.ent-consult.com/faq.html#irrigation
http://www.ent-consult.com/pnd.html
http://ezinearticles.com/?Post-Nasal-Drip-Remedy&id=778439

დღეს ექიმთან ვიყავი ცხვირის ოტორინგოლოგი თუ რაქვია არვიცი ზუსტად და რო ჩამხედა მითხრა ძგიდი გაქ ოდნავ გადიდებულიო და რაც შეეხება ლორწოსო ყელის და ცხვირის ნაცხის ანალიზე გამიშვა ბაქტერიოფაგში და ძგიდის ოდნავ გამრუდებაზე ოპერაცია საჭიროა? და ლიმფურ კვანზეც მითხრა არ გინდა ექიმთან მისვლაო არადა 1 წელზე მეტია გადიდებული მაქ იქნება ერთი 3-4 სმ და რას მირჩევთ? სპ წინასწარ 2kiss.gif

Posted by: vano_t 11 Mar 2008, 18:16
Camel
QUOTE
რა სინდრომები შეიძლება ჰქონდეს  ჩიყვს?


ჩიყვს შეიძლება საერთოდ არ ქონდეს სიმპტომები, ან ფარისებური ჯირკვლის მომატებული ფუნქციის სიმპტომები ქონდეს (წონაში კლება, გაძლიერებული ოფლიანობა, მომატებული მადა, გულის "ფრიალი", სიცხეებისადმი ტოლერანტობის დაქვეითება და ასე შემდეგ), ან ქონედეს ფარისებური ჯირკვლის დაკლებული ფუნქციის სიმპტომები (დაკლებული მადა, სისუსტე, წონის მატება, მოქმედებების შენელება, რეფლექსების დაქვეითება, სიცივისადმი ტოლერანტობის დაქვეითება და ასე შემდეგ), ან ქონდეს კვანძის მიერ მეზობელ ორგანოებზე ზეწოლის სიმპტომები(ხმის ჩახლეჩა, სუნთქვის ან ყლაპვის გაძნელება).

keen
QUOTE
სისხლის საერთო ანალიზის შემთხვევაში, სისხლის გამოდინების გაძნელება რისი მიზეზით შეიძლება იყოს?
რას გულისხმობ სისხლის გამოდინების გაძნელებაში? ყველას თავისებურად ესმის ეგ გაძნელება. გარდა ამისა სისხლის "გამოდენის" სისწრაფე ბევრ რამეზეა დამოკიდებული: სისხლის ვისკოზურ თვისებებზე, სისხლის ასაღებ ჭურჭელში წნევის სიდიდეზე (წნევა სისლძარღში არსებულ წნევაზე რამდენად მეტია ან ნაკლები), ნემსის კალიბრზე, კანულირებული სისხლძარღვის კალიბრზე და ასე შემდეგ.

lizi11
QUOTE
ეს ყველაფერი სისხლის საერთო ანალიზში ჩანს? თუ ცალკე კეთდება?
არ ვიცი სისხლის საერთო ანალიზს რას ეძახიან. თუ სისხლის უჯრედების დათვლას ეძახიან სისხლის საერთო ანალიზს, მაშინ არ გამოჩნდება. კრეატინინი, შარდოვანა და ელექტროლიტები სისხლის ქიმიურ ანალიზში შევა. ამინოტრანსფერაზე, ალბუმინი და ბილირუბინი შევა ღვიძლის ფუნქციურ ტესტებში (liver function tests, or LFT).

Sly
QUOTE
დღეს ექიმთან ვიყავი ცხვირის ოტორინგოლოგი თუ რაქვია არვიცი ზუსტად და რო ჩამხედა მითხრა ძგიდი გაქ ოდნავ გადიდებულიო და რაც შეეხება ლორწოსო ყელის და ცხვირის ნაცხის ანალიზე გამიშვა ბაქტერიოფაგში და ძგიდის ოდნავ გამრუდებაზე ოპერაცია საჭიროა?
ოპერაციის საჭიროებას ისე ოტორინოლარინგოლოგი გადაწყვეტს სიმპტომების მიხედვით და ანატომიური დარღვევების ხარისხის მიხედვით. არ მგონია მცირე გამრუდებამ რაიმე მნიშვნელოვანი პრობლემა გამოიწვიოს. ცხვირის ძგიდის მცირე გამრუდება ბევრს აქვს. თუ ექიმი დარწმუნებულია რომ მცირე გამრუდებით არის პრობლემა გამოწვეული და გამრუდების გამოსწორებით პრობლემაც გამოსწორდება, მაშინ ეგ სხვა საქმეა.

QUOTE
და ლიმფურ კვანზეც მითხრა არ გინდა ექიმთან მისვლაო არადა 1 წელზე მეტია გადიდებული მაქ იქნება ერთი 3-4 სმ და რას მირჩევთ?
მოდი ასე ვთქვათ. პირველ რიგში უნდა დადგინდეს რა არის გადიდებული. პატიენტმა შეიძლება ყველაფერს ლიმფური კვანძი დაარქვას და სინამდვილეში რაიმე ნორმალური სტრუქტურა იყოს. ისე თუ ნამდვილად ლიმფური კვანძია გადიდებული მაგ ზომაზე, მაშინ აუცილებლად ჭირდება გამოკვლევა და განსაკუთრებით ბიოფსია და მიკროსკოპული გამოკვლევა. სად გაქვს ეგ "კვანძი"? ვინ მოსინჯა პირველად, შენ თუ ექიმმა? მტკივნეულია თუ არა? მოძრავია თუ შეზრდილია კანქვეშა ქსოვილთან? მაგარია თუ რბილი და საერთოდ რა კონსისტენციისაა? სხვაგან თუ არის კვანძები კიდევ? ოფლიანობ თუ არა? ძლიერი სისუსტეები გაქვს თუ არა? და ასე შემდეგ. ამ კითხვებს ყველას აქვს მნიშვნელობა დიაგნოზის დასადგენად და დაზუსტებით რაიმეს თქმა ძნელია თუ ეს ყველაფერი არ იქნა გამოკითხული.

Posted by: Sly 12 Mar 2008, 08:09
ტკივილით არ მტკივა, არც ოფლიანობა და სისუსტე მაწუხებს,მოძრაობითაც მოძრაობს, რბილი არ არი მაგარია და ხელით რომ ვსინჯავ თითქოს ორი კვანზი არი ერთმანეთზე მიწებებულიო, და სხვა ჯირკვალი მემგონი უკან თავზეც მაქვს ოღონდ ზაან დიდი ხაია ბავშობიდან ის არ მოძრაობს. და ადრე 1 წლის უკან ვიყვი თერაპევტან და რავი იმან რომ გამსინჯა არაფერი უთქვია, ამ ცხვირის ექიმაც ნახა და რავი არ გინდა მისვალო ესე მითხრა და ისე რის ექიმთან უნდა მივიდე?

Posted by: Camel 12 Mar 2008, 09:21
vano_t

კვანძოვანი ჩიყვი რას ნიშნავს? შეიძლება ჰქონდეს სიმპტომი სიმხურვალე? და რითი უნდა იმკურნალო?

Posted by: keen 12 Mar 2008, 09:46
vano_t
QUOTE
რას გულისხმობ სისხლის გამოდინების გაძნელებაში? ყველას თავისებურად ესმის ეგ გაძნელება. გარდა ამისა სისხლის "გამოდენის" სისწრაფე ბევრ რამეზეა დამოკიდებული: სისხლის ვისკოზურ თვისებებზე, სისხლის ასაღებ ჭურჭელში წნევის სიდიდეზე (წნევა სისლძარღში არსებულ წნევაზე რამდენად მეტია ან ნაკლები), ნემსის კალიბრზე, კანულირებული სისხლძარღვის კალიბრზე და ასე შემდეგ.


დიდი მადლობა, მაგრამ... არ ვიცი, სისხლის ანალიზის აღებისას, სხვადასხვა დროს და სხვადასხვა სამედიცინო დაწესებულებებში, ამ პროცედურის ჩატარება გაუჭირდათ ლაბორანტებს. პირველ შემთხვევაში საჭირო რაოდენობის სისხლი ვერც აიღეს და რისი პათოლოგია შეიძლება იყოს ეს თუ არის საერთოდ ასეთი? baby.gif

Posted by: vano_t 12 Mar 2008, 18:06
Sly
QUOTE
ტკივილით არ მტკივა, არც ოფლიანობა და სისუსტე მაწუხებს,მოძრაობითაც მოძრაობს, რბილი არ არი მაგარია  და ხელით რომ ვსინჯავ თითქოს ორი კვანზი არი ერთმანეთზე მიწებებულიო, და სხვა ჯირკვალი მემგონი უკან თავზეც მაქვს ოღონდ ზაან დიდი ხაია ბავშობიდან ის არ მოძრაობს. და ადრე 1 წლის უკან ვიყვი თერაპევტან და რავი იმან რომ გამსინჯა არაფერი უთქვია, ამ ცხვირის ექიმაც ნახა და რავი არ გინდა მისვალო ესე მითხრა და ისე რის ექიმთან უნდა მივიდე?
მასე ვერ გეტყვი რა არის. თუ დიდი ზომისაა (2სმ-ზე დიდი) და დანამდვილებით გითხრეს რომ ლიმფური კვანძია, მაშინ აუცილებლად უნდა ბიოფსია. შეიძლება ლიმფური კვანძი არ არის და უბრალოდ ლიპომაა ან sebaceous cyst (ქართულად არ ვიცი რა ქვია). რაც არ უნდა იყოს, ექიმმა სახელი უნდა დაარქვას. თუ ექიმმა გითხრა არ გინდა მისვლაო, ალბათ ნორმალური სტრუქტურაა. რა ადგილას არის კვანძი? და ექიმმა რა გითხრა რა არისო? PM-ში შეგიძლია მომწერო თუ გინდა.

Camel
QUOTE
კვანძოვანი ჩიყვი რას ნიშნავს? შეიძლება ჰქონდეს სიმპტომი სიმხურვალე? და რითი უნდა იმკურნალო?
ჩიყვი უბრალოდ ნიშნავს გადიდებულ ფარისებურ ჯირკვალს და კვანძოვანი მიუთითებს კვანძის არსებობას, რომელიც ან ხელით ისინჯება ან ულტრაბგერით ნახეს. თავისთავად ჩიყვმა სიმხურვალე არ იცის, მარა თუ ფარისებური ჯირკვლის ფუნქვია მომატებულია, მაშინ შეიძლება მოგცეს სიმხურვალის შეგრძნება. ჩიყვის ნახვისას ავადმყოფს უნდა ჩაუტარდეს TSH-ის (და თუ საჭირო დარჩა FT4-ს და FT3-ს) ანალიზი. ამით დადგინდება ფუქცია მომატებულია თუ არა? როგორ დადგინდა კვანძოვანი ჩიყვი? გაუკეთდა თუ არა ავადმყოფს სხვა გამოკვლევა?

keen
QUOTE
სისხლის ანალიზის აღებისას, სხვადასხვა დროს და სხვადასხვა სამედიცინო დაწესებულებებში, ამ პროცედურის ჩატარება გაუჭირდათ ლაბორანტებს. პირველ შემთხვევაში საჭირო რაოდენობის სისხლი ვერც აიღეს და რისი პათოლოგია შეიძლება იყოს ეს თუ არის საერთოდ ასეთი?
მასეთი რამ არ გამიგია. ვენიდან სისხლის აღებისას ერთადერთი პრობლემა რაც შეიძლება შეგექმნას არის ის, რომ ან ვენაში ვერ ხვდები ან ხვდები, მაგრამ ვენის კედლები მყიფეა და ადვილად სკდება. მე თვითონ ბევრჯერ ამიღია ვენიდან სისხლი და თუ ვერ ამიღია ზემოთ მითითებულის გამო. სხვა რაიმე პრობლემა არა მგონია. თუ ხელის ვენებიდან ჭირს აღება, მაშინ ფეხის ვენებზე უნდა ცადონ, ან ყელის ვენებზე (ყელის ვენიდან ექთანი არ იღებს სისხლს, მხოლოდ გამოცდილი ექიმი და ისიც უმეტეს შემთხვევაში იმიტომ რომ ავადმყოფს ჭირდება სითხის გადასხმა).

Posted by: keen 12 Mar 2008, 18:11
vano_t
ვენიდან არა... თითიდან და დამაინტერესა, რამე პათოლოგია ხომ არ შეიძლება იყოს თქო baby.gif

Posted by: vano_t 12 Mar 2008, 18:19
keen
QUOTE
ვენიდან არა... თითიდან  და დამაინტერესა, რამე პათოლოგია ხომ არ შეიძლება იყოს თქო baby.gif

არა მგონია რაიმე პათოლოგია იყოს. ყოველ შემთხვევაში არც გამიგია მასეთი ნიშანი სადმე რომ იყოს აღწერილი. სადაც ვმუშაობ იმ სავადმყოფოს ლაბარატორიის უფროსს ვკითხე ახლა და მითხრა რომ ხშირად ქონია მასეთი შემთხვევა და ეგ არაფერს ნიშნავსო. ეგ კანის თავისებურება იქნება, ან შეიძლება თითები არ იყო საკმაო გამთბარი, ან მჩხვლეტავი ნემსი არ ჩადის საკმაო ღრმად და ასე შემდეგ. რაც არ უნდა იყოს, არასდროს არ გამოუგზავნიათ ავადმყოფი მაგის გამო რომ სისხლს ვერ ვიღებთ და გამოიკვლიეთო. მაგაზე არ ინერვიულო smile.gif

Posted by: keen 12 Mar 2008, 18:19
vano_t
დიდი მადლობა, გაიხარე! smile.gif

Posted by: Camel 13 Mar 2008, 09:55
QUOTE
ჩიყვი უბრალოდ ნიშნავს გადიდებულ ფარისებურ ჯირკვალს და კვანძოვანი მიუთითებს კვანძის არსებობას, რომელიც ან ხელით ისინჯება ან ულტრაბგერით ნახეს. თავისთავად ჩიყვმა სიმხურვალე არ იცის, მარა თუ ფარისებური ჯირკვლის ფუნქვია მომატებულია, მაშინ შეიძლება მოგცეს სიმხურვალის შეგრძნება. ჩიყვის ნახვისას ავადმყოფს უნდა ჩაუტარდეს TSH-ის (და თუ საჭირო დარჩა FT4-ს და FT3-ს) ანალიზი. ამით დადგინდება ფუქცია მომატებულია თუ არა? როგორ დადგინდა კვანძოვანი ჩიყვი? გაუკეთდა თუ არა ავადმყოფს სხვა გამოკვლევა?



ეხოზე გამოჩნდა კვანძები. ეგ ანტისხეულებზე გამოკვლევა ჩატარდა , რომელმაც ბევრი არაფერი აჩვენა , რომლის შემდეგაც ექიმმა დანიშნა ეიმოლოკი,გრანდაქსინი და იოდბალანსი. რა წამლებია რას უშველის?

Posted by: ancho 13 Mar 2008, 13:52
vano_t
დიდი მადლობა smile.gif
შეიძლება პმ-ში გკითხო რაღაც თირკმელებთან დაკავშირებით?

Posted by: vano_t 13 Mar 2008, 18:42
Camel
QUOTE
ეხოზე გამოჩნდა კვანძები. ეგ ანტისხეულებზე გამოკვლევა ჩატარდა , რომელმაც ბევრი არაფერი აჩვენა , რომლის შემდეგაც ექიმმა დანიშნა ეიმოლოკი,გრანდაქსინი და იოდბალანსი. რა წამლებია რას უშველის?

ანტისხეულები არ არის ის რაც ჩამოვწერე. ეგ ჰორმონებია და ფარისებურის გამააქტრიურებელი ნივთიერება. ანტისხეულებზე ამოწმებენ მხოლოდ მაშინ, როცა ფარისებური ჯირკვლის ფუნქცია არის არანორმალური, ანუ როცა TSH მაღალია ან დაბალი. თუ TSH გაგიკეთეს, შეგიძლია თქვა რამდენი იყო? გარდა ამისა, როცა კვანძს ნახულობს ექიმი ულტრაბგერაზე, კვანძის ბიოფსია შეიძლება იყოს საჭირო, რასაც ენდოკრინოლოგი გადაწყვეტს.

არ ვიცი ეგენი რა წამლებია. თუ მეტყვი გენერიულ დასახელებას ("საერთაშორისო" სახელს რომ ეძახიან) მაშინ შეიძლება ვიცოდე რა წამლებია.

თუმცა იდობალანსი ალბათ იოდია. გრანდაქსინი (თუ Grandaxin იწერება) არის ბენზოდიაზეპინი და დამაწყნარებელია. ერთადერთი რაც თავში მომდის იმის თაობაზე თუ რატომ შეიძლება ეგ დანიშნულიყო არის ის, რომ მომატებული ფუნქცია გქონდა ფარისებურის და პანიკური შეტევები გემართებოდა ან სხვანაირად იწერება და სხვა წამალი.



ancho
QUOTE
შეიძლება პმ-ში გკითხო რაღაც თირკმელებთან დაკავშირებით?
შეიძლება.

Posted by: lizi11 14 Mar 2008, 13:51
vano_t
QUOTE
არ ვიცი სისხლის საერთო ანალიზს რას ეძახიან. თუ სისხლის უჯრედების დათვლას ეძახიან სისხლის საერთო ანალიზს, მაშინ არ გამოჩნდება. კრეატინინი, შარდოვანა და ელექტროლიტები სისხლის ქიმიურ ანალიზში შევა. ამინოტრანსფერაზე, ალბუმინი და ბილირუბინი შევა ღვიძლის ფუნქციურ ტესტებში (liver function tests, or LFT).

გმადლობ.სასარგებლო ინფორმაციაა. ძიებაში ვარ და იმედია, გამომადგება. ჩვენი თერაპევტები რაცხა მოიკოჭლებენ, სამწუხაროდ... sad.gif

Posted by: FUA 14 Mar 2008, 15:55
დზნ ბევრს მახველებს და ნახველის გარეშე მშრალი ხველი ხველა და რა მჭირს?

Posted by: Bivrili 14 Mar 2008, 21:55
vano_t


და ისა რა უნდა გკითხოთ , საქართველოში არ აპირებთ ჩამოსვლას ?......................







და კიდევ ერთი შეკითხვა: რამდენად საშიშია ნერვის ანთება დ ა რა შეიძლება გამოიწვიოს მან თუ არ იქნა მკურნალობა დროულად ჩატარებული.....................?

Posted by: lastochka 16 Mar 2008, 18:03
ყელის კატარზე ვერ მეტყვით რამეს? სიმპტომები და მკურნალობამაინტერესებს cry.gif cry.gif cry.gif ექიმმა მითხრა ადრე და რაც დამინიშნა (ვაკის ლეჩში ვიყავი საკმაოდ ცნობილთან) და გაზგასული ბორჯომი და კვერცხის ცილა გამოივლე და ნისტატინი მიაყოლე მერე ენაზეო და რიავი{გული მერევა მაგაზე cry.gif

Posted by: vano_t 16 Mar 2008, 23:23
lastochka
QUOTE
ყელის კატარზე ვერ მეტყვით რამეს? სიმპტომები და მკურნალობამაინტერესებს  cry.gif  cry.gif  cry.gif  ექიმმა მითხრა ადრე და რაც დამინიშნა (ვაკის ლეჩში ვიყავი საკმაოდ ცნობილთან) და გაზგასული ბორჯომი და კვერცხის ცილა გამოივლე და ნისტატინი მიაყოლე მერე ენაზეო და რიავი{გული მერევა მაგაზე cry.gif
დააკონკრეტე თუ შეიძლება რას გულისხმობ. კატარს უბრალოდ ეძახიან ლორწოვანის ანთების სიმპტომებს. უფრო ხშირად ეს ვირუსული დაავადებების დროს ხდება და მალევე გადადის. თუ ზემო სასუნთქი გზების ვირუსული ინფექცია გაქვს, მაშინ სიმპტომატური მკურნალობა გინდა და რაც გშველის (არ აქვს მნიშვნელობა ბორჯომი იქნება, კვერცხის ცილა თუ ასპირინი თუ ტაილენოლი) ის შეგიძლია იხმარო. თუ ქრონიკული სიმპტომები გაქვს, მაშინ დიაგნოზია საჭირო.

Bivrili
QUOTE
და კიდევ ერთი შეკითხვა: რამდენად საშიშია ნერვის ანთება დ ა რა შეიძლება გამოიწვიოს მან თუ არ იქნა მკურნალობა დროულად ჩატარებული.....................?
ანთება არის ძალიან არასპეციფიური ტერმინი და საერთოდ მიუთითებს ქსოვილის რეაქციას ამა-თუ-იმ გამღიზიანებელზე. ანთება მიმდინარეობს ნებისმიერი გამღიზიანებლის მიმართ: ფიზიკური იქნება(სითბო, სიცივე), ქიმიური (სხვადასხვა ქიმიური ნივთიერებების მოქმედების შედეგად), რადიაქტიული დასხვივების, ელექტრული დენის შედეგად და ასე შემდეგ. ეს ეხება ნერვსაც (ნერვულ ქსოვილს). მედიცინაში კიდევ ხმარობენ სპეციფიურ დიაგნოზებს. ამიტომ, კითხვას " რამდენად საშიშია ნერვის ანთება დ ა რა შეიძლება გამოიწვიოს მან თუ არ იქნა მკურნალობა დროულად ჩატარებული" არ აქვს აზრი იმდენად, რამდენადაც ნერვული ქსოვილის თითქმის ყოველგვარი დაზიანება ანთებით წავა. სპეციფიური დიაგნოზი მითხარი თუ შეიძლება და შევეცდები გიპასუხო კითხვაზე.
* * *
FUA
QUOTE
დზნ ბევრს მახველებს და ნახველის გარეშე მშრალი ხველი ხველა და რა მჭირს?

რადმენი ხანი გახველებს და სხვა რამ თუ გაწუხებს?

Posted by: menak 21 Mar 2008, 11:52
იცით რა, 1-2 კვირაა ყრუდ მტკივა სახსარი კოჭთან და რაღაცნაირი სისუსტის შეგრძნება მაქვს. სამსახურში სულ ვზივარ, დავაკვირგი და აღმოვაჩინე რომ ძირითადად ამ პეხს ვეყრდნობი და სულ ვატრიალებ აქეთ იქეთ სკამს, სახსარი ხომ არ გადამეღლებოდა თუ რა ჯანდაბაა? user.gif ისე რევმატიულიც ვარ cry.gif

Posted by: Reference Frame 25 Mar 2008, 13:00
ე.ი. 3 დღეა მაქვს ტკივილები შუბლის, თვალის ზემოთ მარჯვენა მხარეს...
ვარ გაციებული მაგრამ პრინციპში სურდო მომეხსნა 1-2 დღის წინ... სიცხე თამაშობს ხან არმაქ ხან 37.4 ხან 38-მდე... ზემოთ აღარ...
ვითომ რაღაც ახალი ვირუსიაო და 4-5 დღე უნდაო რო მორჩეო რავი...

მე ვფიქრობ გაიმორიტი ხოარმაქთქო მარა არც ვიცი ზუსტად რაარი ეგ და რა სიმპტომებიაქვს...

Posted by: lizi11 25 Mar 2008, 15:25
vano_t
შეიძლება პმ-ში მოგმართო?

Posted by: Anna_ 25 Mar 2008, 17:46
ექიმებო, მითხარით ვის უნდა მივმართო, ან რისი ბრალი შეიძლება იყოს როცა ფეხის ფეხზე გადადებისას და სიარულის დროს საშინლად მეჭიმება და მტკივდება ფეხის წინა ზედაპირი მუხლიდან ტერფამდე. ვარ 21 წლის. მირჩიეთ რამე...
* * *
კიდევ მაინტერესებს სად შეიძლება ლიპომის მოშორება თავიდან და რა ჯდება ეს სიამოვნება smile.gif

Posted by: vano_t 25 Mar 2008, 23:47
Reference Frame
QUOTE
ე.ი. 3 დღეა მაქვს ტკივილები შუბლის, თვალის ზემოთ მარჯვენა მხარეს...
ვარ გაციებული მაგრამ პრინციპში სურდო მომეხსნა 1-2 დღის წინ... სიცხე თამაშობს ხან არმაქ ხან 37.4 ხან 38-მდე... ზემოთ აღარ...
ვითომ რაღაც ახალი ვირუსიაო და 4-5 დღე უნდაო რო მორჩეო რავი...

მე ვფიქრობ გაიმორიტი ხოარმაქთქო მარა არც ვიცი ზუსტად რაარი ეგ და რა სიმპტომებიაქვს...
შენ, მე მგონი, მართალს ფიქრობ. შეიძლება ფრონტალური სინუსების ინფექცია გქონდეს (ჰაიმორიტიც სინუსების ინფექციაა, მაგრამ ზუსტად არ ვიცი რომლის: ფრონტალურის თუ მაქსილარულის). ამისათვის ექიმი უნდა ნახო და თუ ინფექციის ნიშნები დაგიდგინდა (უკვე გაქვს თუმცა ზოგიერთი ნიშანი: მაგალითად ცალმხრივი ტკივილი შუბლის სინუსების ზემოთ და სიცხე), მაშინ ანტიბიოტიკი დაგჭირდება.

lizi11
რა თქმა უნდა შეიძლება, რომ PM-ში მომწერო. და ვისაც უნდა რომ PM-ში მომწეროს, ნუ მოგერიდებათ.

Anna_
QUOTE
ექიმებო, მითხარით ვის უნდა მივმართო, ან რისი ბრალი შეიძლება იყოს როცა ფეხის ფეხზე გადადებისას და სიარულის დროს საშინლად მეჭიმება და მტკივდება ფეხის წინა ზედაპირი მუხლიდან ტერფამდე. ვარ 21 წლის. მირჩიეთ რამე...
მაგ სიმპტომები ხშირად იცის ნერვის "მოჭყლეტამ" (ე.წ. common peroneal nerve compression), როცა ფეხს ფეხზე გადადებ. ეგ არის ნეიროპათიის სახე. თავიდან ექიმი არ გჭირდება. შეგიძლია თავის აარიდო ფეხის გადაჯვარედინებას და შესაძლებელია ნერვის ნორმალური ფუნქციონირება აღდგეს. მე მასე ვიზმადი შენს ადგილას. თუ ამის შემდეგ 4-6 კვირაში სიმპტომებმა არ გაგიარა, მაშინ შეიძლება ნევროპათოლოგს მიაკითხო.

menak
QUOTE
იცით რა, 1-2 კვირაა ყრუდ მტკივა სახსარი კოჭთან და რაღაცნაირი სისუსტის შეგრძნება მაქვს. სამსახურში სულ ვზივარ, დავაკვირგი და აღმოვაჩინე რომ ძირითადად ამ პეხს ვეყრდნობი და სულ ვატრიალებ აქეთ იქეთ სკამს, სახსარი ხომ არ გადამეღლებოდა თუ რა ჯანდაბაა? user.gif ისე რევმატიულიც ვარ
თუ ტრამვა არ მიგიღია, სახსარი არ არის გასიებული, შეწითლებული და ცხელი, მაშინ არ მგონია რაიმე სერიოზული იყოს. შეგიძლია უბრალოდ რაიმე ვარჯიშები გააკეთო სახსრისათვის და სპეციალური (ესლასტიური) შესახვევები იხმარო სახსრის გარშემო (სახსრის დასაფიქსირებლად) და ნახო გაგივლის თუ არა ტკივილი. თუ მასე არ გაგიარა, შეგიძლია მიხვიდე ექიმთან და ან რენგენტს გადაგიღებენ ან მაგნიტურ რეზონანს.

Posted by: Tami28 26 Mar 2008, 10:42
vano_t

გამარჯობა.

ეს კითხვა ალბათ სჯობს ონკოლოგს დავუსვა თუმცა თქვენი აზრიც მაინტერესებს. შესაძლებელი არის თუ არა და რამდენად დადებითი შედეგის მომტანია ღვიძლის ტრანსპლანტაცია იმ პაციანტისთვის ვისაც მრავლად აქვს ღვიძლში მეტასტაზი? (პირველდი არის ნაწლავური სიმსივნე). თუ შეგიძლიათ რამე ინფორმაცია მომაწოდოთ ან მირჩიოთ.

Posted by: lizi11 26 Mar 2008, 13:04
vano_t
QUOTE
lizi11
რა თქმა უნდა შეიძლება, რომ PM-ში მომწერო. და ვისაც უნდა რომ PM-ში მომწეროს, ნუ მოგერიდებათ.

გმადლობ.

Posted by: Cor-toni 26 Mar 2008, 23:22
vano_t

QUOTE
ჰაიმორიტიც სინუსების ინფექციაა, მაგრამ ზუსტად არ ვიცი რომლის: ფრონტალურის თუ მაქსილარულის


ჰაიმორიტი არის ზედა ყბის წიაღის ლორწოვანის ანთება, ფრონტიტი_შუბლის წიაღის ლორწოვანის ანთება. სინუსიტი კი_ჰაიმორიტი და ფრონტიტი ერთად wink.gif

Reference Frame

თავი დახრისას ტკივილი ძლიერდება და ზეწოლის შეგრძნება გაქვს?
რენტგენოგრამით ხდება დიაგნოსტირება yes.gif


Posted by: vano_t 27 Mar 2008, 01:14
Tami28
QUOTE
ეს კითხვა ალბათ სჯობს ონკოლოგს დავუსვა თუმცა თქვენი აზრიც მაინტერესებს. შესაძლებელი არის თუ არა და რამდენად დადებითი შედეგის მომტანია ღვიძლის ტრანსპლანტაცია იმ პაციანტისთვის ვისაც მრავლად აქვს ღვიძლში მეტასტაზი? (პირველდი არის ნაწლავური სიმსივნე). თუ შეგიძლიათ რამე ინფორმაცია მომაწოდოთ ან მირჩიოთ.

არ აქვს მასეთ შემთხვევაში აზრი ღვიძლის ტრანსპლანტაციას. ნაწლავის სიმსივნე შორეული მეტასტაზით არის მე-4 სტადიის სიმსივნე და სტატისტიკურად ცუდი გამსავალი აქვს. ღვიძლში მეტასტაზები არ არის ცუდი გამოსავალის მიზეზი.

ერთეული ღვიძლის მეტასტაზის დროს შეიძლება მეტასტაზის ამოკვეთა გაააკეთონ თუ პირველადი სიმსივნის ამკვეთაც არის შესაძლებელი. მაგრამ მრავლობით მეტასტაზების დროს პალიატიურ მკურნალობას უნიშნავენ.

cor-toni
QUOTE
ჰაიმორიტი არის ზედა ყბის წიაღის ლორწოვანის ანთება, ფრონტიტი_შუბლის წიაღის ლორწოვანის ანთება. სინუსიტი კი_ჰაიმორიტი და ფრონტიტი ერთად
ალბათ დამოკიდებულია იმაზე თუ რომელ ქვეყანაში მოღვაწეობ. ჰაიმორიტის და ფრონტიტის დიაგნოზს, მაგალითად, ბრიტანეტში და აშშ-ში არ ხმარობენ. ხმარობენ სინუსიტის დაიგნოზს და უთითებენ ადგილს (მაგალითად, frontal sinusitis, maxillary sinusitis, sphenoid sinusitis). ასე რომ, სინუსიტი არის სინუსის ანთება და უნდა მიუთითო სად არის ანთება. თუ ორივეგან არის ანთება, უნდა მიუთითო ფრონტალური და მაქსილარული სინუსიტი. ვიმეორებ ისევ, შეიძლება გააჩნია ქვეყანას. თუმცა ან საქართველოში რატომ უნდა ერქვას სინუსიტი მაინცდამაინც ფრონტალური და მაქსილარული სინუსის (და არა ცალკეული სინუსის ანთებას, ან სფენოიდალური და ფრონტალური სინუსის ერთდორულ ანთებას) ერთდროულ ანთებას, სავსებით გაუგებარია ჩემთვის.

QUOTE
Reference Frame

თავი დახრისას ტკივილი ძლიერდება და ზეწოლის შეგრძნება გაქვს?
რენტგენოგრამით ხდება დიაგნოსტირება
რენტგენოგრამა სრულებით არ არის საჭირო დიაგნოზისათვის. რენტგენზე გამოჩნდება მხოლოდ სითხე არის თუ არ არის სინუსში გარკვეული რაოდენობით. თუ მაგალითად კომპიუტერულ ტომოგრაფიას გაუკეთებ ავადმყოფს შესაძლო სინუსიტის დიაგნოზით (კლინიკურად), შეიძლება რადიოლოგმა ასეთი დასკვნა მოგცეს: სინუსების ლორწოვანი გარსი შესქელებულია/შეშუპებულია. ასეთი რამ რენტგენზე ბევრ შემთხვევაში არ გამოჩნდება, მიუხედავად იმისა, რომ ანთების მიმანიშნებელია. ეს ყველაფერი ითარგმნება ასე: რენტგენის სენსიტიურობა (ანუ უნარი "ჩართოს"-include- მოცემული დაავადება ტესტის ფარგლებში) მწვავე სინუსიტების დიაგნოზსში არის დაბალი. http://dmfr.birjournals.org/cgi/content/abstract/32/1/60. ასე რომ, კლინიკურად უნდა გადაწყვიტო მაინც ეგ საკითხი(ანუ ინფექციური სინუსიტის არსებობა/არარსებობა).

რაც შეეხება თავის დახრისას ტკივილის მომატებაზე, ეგ მართლა არის ხშირი სიმპტომი. მაგრამ ისევ და ისევ, ბევრ შემთხვევაში არ აღინიშნება. და რაც მთავარია, არავითარ დიფერენციაციას არ აკეთებს ვირუსულ იფექციით გამოწვეულ სინუსიტსა და ბაქტერიული ინფექციით გამოწვეულ სინუსიტს შორის. არადა, ზუსტად ეს არის მთავარი საკითხი მაგ დროს: საჭირო არის თუ არა ანტიბიოტიკი. საერთოდ, ამისათვის არის გარკვეული გაიდლაინები, რომელიც გაძლევს საშუალებას სტატისტიკურად იმსჯელო ბაქტერიული სინუსიტის არსებობაზე.

Posted by: Reference Frame 27 Mar 2008, 15:12
vano_t
QUOTE
შენ, მე მგონი, მართალს ფიქრობ. შეიძლება ფრონტალური სინუსების ინფექცია გქონდეს (ჰაიმორიტიც სინუსების ინფექციაა, მაგრამ ზუსტად არ ვიცი რომლის: ფრონტალურის თუ მაქსილარულის). ამისათვის ექიმი უნდა ნახო და თუ ინფექციის ნიშნები დაგიდგინდა (უკვე გაქვს თუმცა ზოგიერთი ნიშანი: მაგალითად ცალმხრივი ტკივილი შუბლის სინუსების ზემოთ და სიცხე), მაშინ ანტიბიოტიკი დაგჭირდება.

cor-toni
QUOTE
თავი დახრისას ტკივილი ძლიერდება და ზეწოლის შეგრძნება გაქვს? რენტგენოგრამით ხდება დიაგნოსტირება yes.gif

yes.gif
თავის დახრისას ძლიერდება...
დღეს ვიყავი ექიმთან გადავიღე სურათი და მარჯვენამხრივი ფრონტიტი მაქვს...
დაჩრდილვები არის... ჩირქი არისო...
და ანტიბიოტიკები დამინიშნა და კიდე გამოვლებები მარილწყლის 10%-იანი ხსნარით...
მემგონი მეშველება...

Posted by: Cor-toni 28 Mar 2008, 13:55
Reference Frame

QUOTE
მემგონი მეშველება...


yes.gif

vano_t

QUOTE
რენტგენოგრამა სრულებით არ არის საჭირო დიაგნოზისათვის. რენტგენზე გამოჩნდება მხოლოდ სითხე არის თუ არ არის სინუსში გარკვეული რაოდენობით. თუ მაგალითად კომპიუტერულ ტომოგრაფიას გაუკეთებ ავადმყოფს შესაძლო სინუსიტის დიაგნოზით (კლინიკურად)


კომპიუტერული ტომოგრაფია ჰაიმორიტზე? spy.gif საქართველოში კომპიუტერულ ტომოგრაფიას ხშირად აუცილებლობის დროსაც ვერ იკეთებენ, ფასის გამო.
Reference Frame-ს საკმაო სიმპტომი ჰქონდა ჰაიმორიტზე ეჭვის მისატანად: ცალმხრივი ტკივილი ლოკალურად, ტემპერატურა, სურდო, ტკივილის გაძლიერება თავის დახრისას და რენტგენოგრამა დიაგნოზის დასაზუსტებლად იყო საჭირო და CT უაზრო ზედმეტი ხარჯი იქნებოდა ჩემი აზრით.

Posted by: vano_t 28 Mar 2008, 22:06
cor-toni
QUOTE

QUOTE
კომპიუტერული ტომოგრაფია ჰაიმორიტზე? spy.gif საქართველოში კომპიუტერულ ტომოგრაფიას ხშირად აუცილებლობის დროსაც ვერ იკეთებენ, ფასის გამო.
Reference Frame-ს საკმაო სიმპტომი ჰქონდა ჰაიმორიტზე ეჭვის მისატანად: ცალმხრივი ტკივილი ლოკალურად, ტემპერატურა, სურდო, ტკივილის გაძლიერება თავის დახრისას და რენტგენოგრამა დიაგნოზის დასაზუსტებლად იყო საჭირო და CT უაზრო ზედმეტი ხარჯი იქნებოდა ჩემი აზრით.
მართალი ხარ, კომპიუტერული ტომოგრაფიის გაკეთება სინუსების ინფექციების დროს არ ხდება(განსაკუთრებული შემთხვების გარდა). PM ნახე თუ შეიძლება.

Posted by: MALI 4 Apr 2008, 14:51
მირჩიეთ კარგი თანამედროვე ინფექციონისტი რა?!
დავიღალე ექიმებთან სიარულით sad.gif (პრობლემები ისევ და ისევ წითელი ქარის შესახებ)

Posted by: vano_t 4 Apr 2008, 20:47
MALI
QUOTE
მირჩიეთ კარგი თანამედროვე ინფექციონისტი რა?!
დავიღალე  ექიმებთან სიარულით sad.gif  (პრობლემები ისევ და ისევ წითელი ქარის შესახებ)

ადრე, როგორც მახსოვს, მაგაზე ვისაუბრეთ. ინფექციონისტი მარტო ვერაფერს გიშველის. განმეორებითი ინფექციების მიზეზი უნდა დადგინდეს (ქვედა კიდურების ვენების უკმარისობა იქნება თუ რაიემე იმუნური დაავადებები და სხვა) და იმას უნდა ემკურნალოს აუცილებლად.

Posted by: MALI 4 Apr 2008, 22:02
vano_t
ზუსტად გახსოვს, ასეა, მაგრამ თუ განმეორებითი მიზეზი ქრონიკულია და იმას არაფერი ეშველება, რაღაც გამოსავალი უნდა იყოს, რათა ეს ინფექცია არ განმეორდეს.
და აი, მე ისევ ძიებაში ვარ
ანგიოლოგი-ინფექციონისტი
ამიტომ ვეძებ სხვა ექიმს, იქნებ დამხმაროს.
გმადლობ გამოხმაურებისთვის
პ.ს იმუნურ დაავადებაში რა იგულისხმება?

Posted by: virusa 5 Apr 2008, 13:44
დაახლოებით ერთი თვის წინ მეგობარს დაუბუჟდა ფეხის ცერა თითები დაბუჟება რამოდენიმე დღეში სხვა თითებზე შემდეგ კი ტერფსა და მთელ ფეხზე გავრცელდა, ნევროპათალოგთან ვიზიტმა შედეგი არ გამოიღო, ყველა რეფლექსი ნორმაში გაქვს და რაზე გიმკურნალოო . სისხლის ანალიზები ყველაფერი ნორმაშია არავითარი გადახრა. ფეხები კი ისევ გაშეშებული აქვს მუხლში ძალა დაკარგული, ამ ბოლო დღეებში კი ამბობს რომ დაბუჯება წელზეც ამოვიდა და და მარცხენა მხარე მთლიანად შEგრძნება დაკარგული მაქვსო. იქნებ ვინმეს ქონია მსგავსი სიმპტომები სად და ვისთან იმკურნალა დაწერეთ თუ შეიძლება.
* * *
QUOTE
მირჩიეთ კარგი თანამედროვე ინფექციონისტი რა?!
დავიღალე  ექიმებთან სიარულით  (პრობლემები ისევ და ისევ წითელი ქარის შესახებ)

რამოდენიმე წლის წინ ნათესავი ქალი ლოგინად იყო ჩავარდნილი წითელი ქარის გამო, შემდეგ ვიღაცამ თუშმელიშვილის მალამო ურჩია, საუკეთესო შედეგი ქონდა, როგორც ვიცი იმ ქალმა ჯერ სოკოზე შემდეგ ქუსლის ნახეთქებზე უმკურნალა .მხოლოდ ამის შემდეგ დაუწყო წითელ ქარზე მკურნალობა. მისი მისამართია თამარაშვილის 21 ბინა16 ტელ 397742. ელისო.

Posted by: MALI 5 Apr 2008, 14:04
virusa
დიდი მადლობა
დავრეკავ აუცილებლად და ავუხსნი ჩემს პრობლემას..

Posted by: vano_t 6 Apr 2008, 10:57
MALI
QUOTE
იმუნურ დაავადებაში რა იგულისხმება?
იმუნურ დაავადებებში იგულისხმება იმუნიტეტის დამაზიანებელი დაავადებები (მაგალითად ლიმფოციტების ან ლეიკოციტების დაავადებები). ბევრნაირი შეიძლება იყოს ასეთი დაავადება. მაგრარ, როგორც წესი ესენი უფრო ხშირად სხვადასხვა ადგილას იძლევიან ინფექციებს.

virusa
PM ნახე.

Posted by: MALI 7 Apr 2008, 09:55
vano_t
,აჰა გასაგებია,

virusa
წარმოუდგენლად მიმაჩნია ცივილიზაციის დროს, მედიცინა უძლური იყოს ამ შემთხვევაში sad.gif
......და ექიმბაშებთან უხდებათ ადამიანებს წასვლა

Posted by: tatusha25 7 Apr 2008, 22:49
QUOTE
vano_t

მაქვს ბრონქიტი ალერგიული დიდი ხანია.ფილტვების ანთების შემდეგ დამრჩა. ვმკურნალობდი მაგრამ ვერ მიმკურნალეს ბოლომდე.ტუბ დისპანსერშიც გავესინჯე მაგრამ...მაქვს სპაზმები .რეაქცია მკვეთრ დუხის სუნებზე და სარეცხ ფხვნილზე .იკნებ მირჩიოთ რამე sad.gif

Posted by: texasuri jleta benzoxerxit 8 Apr 2008, 02:17
QUOTE (ტანკე @ 16 Aug 2007, 21:54 )
vano_t
QUOTE
ისე თუ სხვა რაიმე ნევროლოგიური სიმპტომები/ნიშნები არ გაქვს, მაშინ შეიძლება გადადო კომპიტერული ტომოგრაფია. მაგრამ თუ გაქვს ცვლილება მხედველობაში, სმენაში, წონასწორობის დარღვევა, პარესთეზია/დიზესთეზია, ცალმხრივი სისუსტე, ასიმეტრიული რეფლექსები(ერთი სიტყვით რაიმე რაც მიუთითებს თავის ტვინის დაზიანებაზე) მაშინ კი უნდა კომპიტერული ტომოგრაფია.

არცერთი ეგ ნისანი არ მაქვს,გარდა მხედველობის დაქვეითებისა.თვალების პრობლემა წლებია მაწუხებს და არ ვუკავშირებ ამას.თუმცა ბოლო დროს გამიღრმავდა პრობლემა.ნუ double vision არ მაქვს რაც მთავარია smile.gif
QUOTE
ყველაზე ხშირი მიზეზები თავის ტკივილის: ძილის დარღვევა, შიმშილი, სტრესი, მიგრენი, კლასტერი თავის ტკივილები და ტენზიური თავისტკივილები.

anxiety disorder ვფიქრობ user.gif


QUOTE
გარდა იმისა რომ არ ღებინება არის პროგნოზისათვის + ფაქტორი, სხვა დადებიტ ფაქტორებს მიეკუთვნება: თუ არ გაღვიძებს თავის ტკივილი, თუ იხსნება თავის ტკივილი აცეტამინოფენით, იბუპროფენით(და საერთოდ სუსტი ტკივილგამაყუჩებლებით)

არა არ მაღვიძებს თავის ტკივილი ძილის დროს,brain tumor sucks smile.gif

ისე ვერ ვიტყვი რომ ძაან მტკივა,ძაან ყრუ ტკივილები მაქვს,ხან კეფაში,ხან შუბლის არეში,ხან თვალბუდის მიდამოში,ხან საფეთქელ-ქვედა ყბის სახსრის მიდამოში,ხან თხემის არეში,ხან საფეთქლის არეში,კისერში...მოკლედ ყველგან.დენივით გამივლის,ხან ზეწოლის შეგრძნება მაქვს,ხან დაჭიმვის.5 წამი გრძელდება,მერე 0.5-1-2 წუთი აღარ არის,მერე ისევ და ა.შ. დილით რომ ვიღვიძებ არ მაქვს ტკივილი როგორც წესი,1 საათის მერე მეწყება და საღამოსკენ უფრო იმატებს.წამლებს არ ვსვამ.ხანდახან თვალებზე ზეწოლას ვგრძნობ,წქნევა არ მაქვს და გლაუკომას გამოვრიცხავ smile.gifმადლობა პასუხისთვის

მასკირებულმა(ლავრირებულმა) დეპრესიამაც იცის ეგეთი სიმპტომები.იფიქრე ამ მიმართულებით.

სომატიზირებული დეპრესიის დროს საკუთრივ დეპრესიული აფექტი შეიძლება არც იყოს გამოხატული,ან უმნიშვნელოდ,რაც ინტერპრეტირდება,როგორც "რეაქცია" სომატურ პათოლოგიაზე

Posted by: vano_t 10 Apr 2008, 06:33
tatusha25
QUOTE
მაქვს ბრონქიტი ალერგიული დიდი ხანია.ფილტვების ანთების შემდეგ დამრჩა. ვმკურნალობდი მაგრამ ვერ მიმკურნალეს ბოლომდე.ტუბ დისპანსერშიც გავესინჯე მაგრამ...მაქვს სპაზმები .რეაქცია მკვეთრ დუხის სუნებზე და სარეცხ ფხვნილზე .იკნებ მირჩიოთ რამე sad.gif

დანიშნულება და გამოკვლევა რა თქმა უნდა შენმა ექიმმა უნდა გააკეთოს. მე შემიძლია მოგცე კონკრეტული რჩევები. მაგალითად, მასეთი სიმპომების დროს ავადმყოფს უნდა გაუკეთდეს რამოდენიმე ტესტი, მათ შორის გულმკერდის რენტგენოგრაფია, ფილტვის ფუნქციური ტესტები (ალერგენით პროვოკაციის ან/და ალბუტეროლით) და შესაძლებელია კანის ტესტები ალერგიაზე.

თუ ძალიან არ გაწუხებს სიმპტომები და მარტო გარკვეული ტიპის ალერგენთან კონტაქტის დროს ჩნდება, მაშინ უბრალოდ შეგიძლია აარიდო თავის ალერგენს. თუ ალერგენთან კონტაქტი ხშირად გიწევს (დავუშვათ პროფესიის გამო ან სხვა მიზეზების გამო), მაშინ შეგიძლია კონტაქტის წინ ალბუტეროლის ინჰალტორი მიიღო ექიმის დანიშნულებით; თუ სხვა ალერგიული სიმპტომები გაქვს სუნთქვის გაძნელებასთან ერთად (მაგალითად გამონაყარი), მაშინ შეიძლება წამალიც დაგჭირდეს (მაგალითად კრომოლინის ნათესავები). მოკლედ, ბევრი რამის გათვალისწინება დაჭირდება ექიმს რომ საუკეთესო გადაწყვეტილება მიიღოს.

თუ ალერგიის მომრჩენ წამალზე მეკითხები რაიმეს, მაშინ ეგეთი წამალი არ არსებობს. თუმცა თვითონ ალერგიამ შეიძლება დროის განმავლობაში უკუგანვითარება განიცადოს.

Posted by: intriga55 10 Apr 2008, 16:42
texasuri jleta benzoxerxit
QUOTE
მასკირებულმა(ლავრირებულმა) დეპრესიამაც იცის ეგეთი სიმპტომები.იფიქრე ამ მიმართულებით.

სომატიზირებული დეპრესიის დროს საკუთრივ დეპრესიული აფექტი შეიძლება არც იყოს გამოხატული,ან უმნიშვნელოდ,რაც ინტერპრეტირდება,როგორც "რეაქცია" სომატურ პათოლოგიაზე


რა ოპერატიული ხარ? biggrin.gif

დეპრესიის გამო მიგვატოვე ტანკე? wink.gif




givi.gif


Posted by: tikope 10 Apr 2008, 20:36
ხალხო , გამარჯობათ ჩემ ქმარს ძირმაგარა აქვს მუცელზე და იქნებ მირჩიოთ რამე რითი ვუშველო , დამეხმარეთ რა ან ვის მივმართო პორუმზე, მადლობთ
* * *
vano_t
იქნებ დამეხმაროთ. ძირმაგარა აქვს ჩემს მეუღლეს და სტკივა , რითი შეიძლება ვუმკურნალო? თან მუცელზე აქვს,

Posted by: texasuri jleta benzoxerxit 10 Apr 2008, 23:52
QUOTE (vano_t @ 25 Sep 2007, 09:49 )
QUOTE (baby-bobo @ 24 Sep 2007, 23:49 )
vano_t
QUOTE
რა თქმა უნდა არსებობს სხვადასხვა დაავადებები,რომელსაც ახასიათებს ეგ

მაგალითად????

----------------------------------------
მადლობა პასუხისთვის, დამამშვიდე ძალიან smile.gif

არ გინდა მაგალითები. მერე ტყუილად ნერვიულობას დაიწყებ. დამიჯერე, თუ სხვა რაიმესაგან არის გამოწვეული შენი ხელისგულების სიწითლე, პირველ რიგში სხვა უფრო სერიოზული სიმპტომები გექნებოდა.

"მაგალითები არ გინდა" მომეწონა.თან ამ კონკრეტულ,პალმარული ერითემის შემთხვევაში lol.gif

მე შენგან განსხვავებით "ბოროტი" ექიმი ვარ,ვგიჟდები ავადმყოფების შეშინებაზე(ოღონდ ისე,იატროგენია რომ არ გამოვიწვიო).
ამ შემთხვევაშიც აუცილებლად ვიტყოდი პალმარული ერითემის უხშირეს საზარელ მიზეზს lol.gif

Posted by: LULA_QABABI 11 Apr 2008, 07:22
QUOTE (tikope @ 10 Apr 2008, 11:36 )
ხალხო , გამარჯობათ ჩემ ქმარს ძირმაგარა აქვს მუცელზე და იქნებ მირჩიოთ რამე რითი ვუშველო , დამეხმარეთ რა ან ვის მივმართო პორუმზე, მადლობთ
* * *
vano_t
იქნებ დამეხმაროთ. ძირმაგარა აქვს ჩემს მეუღლეს და სტკივა , რითი შეიძლება ვუმკურნალო? თან მუცელზე აქვს,

თუ ძირმაგარა უკვე "მომწიფებულია" ანუ აბსცესის ჩამოყალიბდა საჭიროა ქირურგიული ჩარევა. ანუ მცირე განაკვეთი რათა ჩირქი გამოიდევნოს. მოკლედ, ყველა შემთხვევაში უნდა ენახოს ქირურგს, სასურველია კარგს. ანტიბიოტიკის რჩევა ცოტა გამიჭირდება, ვინაიდან არ ვიცი სტაფილოკოკის მეთიცილინ-რეზიზტენტულობის ეპიდ მდგომარეობა საქართველოში. მესმის რომ "ჩინურად" ვლაპარაკობ მაგრამ გამგები გაიგებს. მოკლედ, უნდა მიხვიდე ექიმთან/ქირურგთან.


Posted by: texasuri jleta benzoxerxit 11 Apr 2008, 13:55
QUOTE (LULA_QABABI @ 11 Apr 2008, 07:22 )
QUOTE (tikope @ 10 Apr 2008, 11:36 )
ხალხო , გამარჯობათ ჩემ ქმარს ძირმაგარა აქვს მუცელზე და იქნებ მირჩიოთ რამე რითი ვუშველო , დამეხმარეთ რა ან ვის მივმართო პორუმზე, მადლობთ
* * *
vano_t
იქნებ დამეხმაროთ. ძირმაგარა აქვს ჩემს მეუღლეს და სტკივა , რითი შეიძლება ვუმკურნალო? თან მუცელზე აქვს,

თუ ძირმაგარა უკვე "მომწიფებულია" ანუ აბსცესის ჩამოყალიბდა საჭიროა ქირურგიული ჩარევა. ანუ მცირე განაკვეთი რათა ჩირქი გამოიდევნოს. მოკლედ, ყველა შემთხვევაში უნდა ენახოს ქირურგს, სასურველია კარგს. ანტიბიოტიკის რჩევა ცოტა გამიჭირდება, ვინაიდან არ ვიცი სტაფილოკოკის მეთიცილინ-რეზიზტენტულობის ეპიდ მდგომარეობა საქართველოში. მესმის რომ "ჩინურად" ვლაპარაკობ მაგრამ გამგები გაიგებს. მოკლედ, უნდა მიხვიდე ექიმთან/ქირურგთან.

შენ რომელი ქვეყნიდან პოსტავ? სად გვყავს ჩვენ ასეთი სვეტსკი (მეტიცილინ-რეზისტენტული)სტაფილოკოკები. lol.gif ალაგ-ალაგ თუ მოიპოვებიან ჰოსპიტალური შტამების სახით.
სამაგიეროდ ჩვენთან აქტუალურია E.coli(0:157),Enterococcus faecalis,Shigella dysenteriae,Entamoeba hystolytica etLeishmania donovani lol.gif lol.gif lol.gif ხო მაგარია?
* * *
QUOTE (ტანკე @ 8 Jan 2008, 02:36 )
მოკლედ საშინელ სტრესში ვარ,დავპანიკდი.
ახლა რა ხდება...რამდენიმე თვე იქნება,რაც მარცხენა თვალის პერიფერიული მხედველობა შემეცვალა.არ ვიცი ზუსტად როგორ,მაგრამ ვგრძნობ ისე აღარ არის,როგორც ადრე იყო.მარცხენა თვალის ოდნავი ლატერალური გადანაცვლებაც საკმარისია და საკუთარ ყვრიმალს და თვალბუდის კედლებს ვხედავ მკაფიოდ,შავად ჩანს მტელი უბანი...ადრე ასე არ იყო.არ ვიცი რა ხდება პერიფერიული მხედველობა შემეცვალა,თუ უფრო მუქად ვხედავ და ამიტომ ვეფეთები მაგ თვალით მაგ შავ უბანს.ნერვებზე ძაან მოქმედებს.მარჯვენა თვალი ძაან უნდა გადავწიო ლატერალურად,რომ იგივე უბანი დავინახო და უფრო ღია ფერებში ვხედავ(როგორც ნორმა იყოს ჩემთვის),მარცხენაში კი  მუქად ვხედავ და თითქოს წამოსულია ეგ პერიფერიული უბანი ცემტრისკენ და ოდნავი მიტრიალება თვალის და ვხედავ შავად sad.gif ზოგჯერ ნერვები მაგრა ამიყვას ხოლმე მაგის გამო,მგონი ვბრმავდები.
ვცდილობდი არ მიმექცია აქამდე ყურადღრება,მაგრამ ბოლო დღეებია უკვე იგივე მხარეს-მარცხნივ ყურში მესმის საკუთარი გულისცემა sad.gif  pulsatile tinnitus მაკლდა ახლა ყველა სიკეთესთან ერთად და ამან საერთოდ მანდრაჟში ჩამაგდო,ნერვები მაქვს უკვე ძაან ცუდ დღეში.ვშიშობ ანევრიზმა არ მქონდეს sad.gif  კიდევ იდიოპატიურ ინტრაკრანიალურ ჰიპერტენზიაზე ვფიქრობ,მაგრამ უფრო ანევრიზმა მგონია.შეიძლება პანიკის გამო).
მარცხენა ყურში ამ გულისცმა ძირითადად ღამე დაწოლისას ვგრძნობ,როცა სიჩუმეა და ვერტიკალურიდან ჰორიზონტალრ მდგომარეობაში ვწვები.არაა მკაფიო,ჩუმად ისმის,ხრაშა-ხრუშის ხმებით,მაგრამ მაინც მოქმედებს ნერვებზე sad.gif
მოკლედ უნდა გადავიღო თავზე ტომოგრამა? მჭირდება ტომოგრაფია? სახლშიც არ ვეუბნები ჩემებს,არ მინდა დავაფეთო ამ ახალ წლებზე და მოკლედ ცუდ დღეში ვარ,უკვე აღარ ვიცი რა ვქნა sad.gif

სხვა სიმპტომები რავი,თავის ტკივილი არ მაქვს,ადრე მქონდა 2 კვირიანი თავის ტკივილი და ახლა აღარ მაწუხებს,ბავშვონიდან მაქვს ჰიპერტენზია და pulsatile tinnitus-ს მაგას დავაბრალებდი რომ არა მარცხენა თვალში პერიფერიული მხედველობის შეცვლა.მხედველობა ამ ბოლო დროს დამიქვეითდა ორივე თვალში და თვალების ტკივილი,წვაც მაწუხებს.any suggestions? ვიცი ექიმთან ვარ მისასვლელი ხო sad.gif
რის ექიმთან მივიდე ისიც არ ვიცი,ნევროპათოლოგთან,ანგიოლოგთან,ოკულისთან,თერაპევტთან თუ სად...

მაგას ჩემო ტანკე იპოქონდრიულ-სენესტოპათიური სინდრომი და/ან მასკირებული დეპრესია ქვია.
ანაფრანილი უნდა მაგას, ან სეროტონინის უკუმიტაცების სელექტიური ინჰიბიტორები(SSRI-პროზაკი,ზოლოფტი,ფევარინი,რექსეტინი,ციპრალექსი...)

Posted by: tikope 11 Apr 2008, 21:33
LULA_QABABI
გმადლობთ . ხვალე გვეწვევა ქირურგი და გულის ვიმაგრებ. იმხელა რამეა რომ დათო ექიმსაც კი გაუკვირდა . მაგარი ექიმია
* * *
texasuri jleta benzoxerxit
სხვათა შორის ცოტა გასაგები ენით რომ იღადაოთ, შეიძლება მეც გავიგო თქვენი მედიცინურები, და მეც გავიცინო ჰა ჰა გავიგიმო,

Posted by: intriga55 11 Apr 2008, 23:13
texasuri jleta benzoxerxit

შენ სიცხე ხო არ გაქვს? spy.gif
ტანკეზე ნუ ნერვიულობ, კარგად არის. გამოკვლევები გაიკეთა და დაწერა კიდეც აქ. შენ ალბათ პირველი გვერდიდან დაიწყე კითხვა და ვიდრე ბოლოში ჩახვალ, ზაფხული მოვა givi.gif

tikope

QUOTE
სხვათა შორის ცოტა გასაგები ენით რომ იღადაოთ, შეიძლება მეც გავიგო თქვენი მედიცინურები, და მეც გავიცინო ჰა ჰა გავიგიმო,


QUOTE
texasuri jleta benzoxerxit
რო წაიკითხავ, არ გაგეცნება?
biggrin.gif biggrin.gif biggrin.gif biggrin.gif



Posted by: texasuri jleta benzoxerxit 12 Apr 2008, 00:22
QUOTE (intriga55 @ 11 Apr 2008, 23:13 )
texasuri jleta benzoxerxit

შენ სიცხე ხო არ გაქვს? spy.gif
ტანკეზე ნუ ნერვიულობ, კარგად არის. გამოკვლევები გაიკეთა და დაწერა კიდეც აქ. შენ ალბათ პირველი გვერდიდან დაიწყე კითხვა და ვიდრე ბოლოში ჩახვალ, ზაფხული მოვა givi.gif

tikope

QUOTE
სხვათა შორის ცოტა გასაგები ენით რომ იღადაოთ, შეიძლება მეც გავიგო თქვენი მედიცინურები, და მეც გავიცინო ჰა ჰა გავიგიმო,


QUOTE
texasuri jleta benzoxerxit
რო წაიკითხავ, არ გაგეცნება?
biggrin.gif biggrin.gif biggrin.gif biggrin.gif

სიცხე არ მაქვს,ტანკეს ბედი,რომ არ მოგატყუო მკიდია,მაგრამ მიხარია უკეთ თუ არის(ისე ყველა კარგად თუ იქნა დავრჩებით ექიმები უმუშევარი)
* * *
ეს სამედიცინო ტოპიკია და თავს უფლებას ვაძლევ "მედიცინურად" ბიღადავო.ჩვეულებრივ მოკვდავთავის სხვანაირად ვხუმრობ smile.gif
* * *
ჩემი ნიკის მიზანიც ეგაა,რომ წაიკითხავ გაგეცინოს.
რაღაც შენი გადახერხვა მომინდა მენჯ-ბარძაყის სახსრის დონეზე lol.gif lol.gif lol.gif
* * *
QUOTE (LULA_QABABI @ 6 Mar 2008, 09:53 )
QUOTE (cor-toni @ 5 Mar 2008, 13:45 )
QUOTE

LULA_QABABI

ბიცილინი რევმატიზმის სამკურნალოდაც ინიშნება smile.gif



ერთ რამეს დავაზუსტებ, რასაც არ მივაქციე ყურადღება; მე ვიცი ერთი სახის ბიცილინი რომელიც აქ (აშშ) იხმარება - იგივე Benzathin Pencillin G.
საქართველოში თუ სხვა ფორმებია ბიცილინის (1, 2...5?) მაგას ვერ მოგახსენებთ. მაშინ ჯობია generic სახელით მითხრათ.
Benzathin PCN G-ს ხმარება თეორიულად შეიძლება მწვავე A ჯგუფის სტრეპტოკოკული (და სხვა პენიცილინ-სენსიტიური) ინფექციების დროს, თუმცა ძლიერი პერორალური ანტიბიოტიკების არჩევანის პირობებში მთლად ჰუმანური არ უნდა იყოს დუნდულოთა ჩხვლეტა ამ უკანასკნელთათვის, მითუმეტეს თუ მკურნალობა ამბოლატორიულადაც შეიძლება.

ალბათ გინდოდა გეთქვა რევმატული ცხელება ვინაიდან "რევმატიზმი" უფრო ხალხური ტერმინია და ხშირად რევმატოიდული ართრიტის მისამართითაც იხმარება.

არა რევმატიზმი არ არის ხალხური ტერმინი,ის დამოუკიდებელი ნოზოლოგიაა,აქვს თავისი კრიტერიუმები(დიდი და მცირე)მაგsad.gifმფრინავი პოლი)ართრიტი,კარდიტი,ქორეა,ბეჭდისებრი ერითემა,კანქვეშა კვანძები,დადებითი რეაქცია ანტისტრეპტოკოკულ ანტისხეულებზე,მაღალი რაოდენობრივი და თვისობრივი C რეაქტიული ცილა და ა.შ.

რევმატული ცხელება,რევმატული პოლიმიალგია სხვა არის.
ეტყობა თქვენთან განსხვავებული კლასიფიკაციაა
* * *
QUOTE (virusa @ 5 Apr 2008, 13:44 )
დაახლოებით ერთი თვის წინ მეგობარს დაუბუჟდა ფეხის ცერა თითები დაბუჟება რამოდენიმე დღეში სხვა თითებზე შემდეგ კი ტერფსა და მთელ ფეხზე გავრცელდა, ნევროპათალოგთან ვიზიტმა შედეგი არ გამოიღო, ყველა რეფლექსი ნორმაში გაქვს და რაზე გიმკურნალოო . სისხლის ანალიზები ყველაფერი ნორმაშია არავითარი გადახრა. ფეხები კი ისევ გაშეშებული აქვს მუხლში ძალა დაკარგული, ამ ბოლო დღეებში კი ამბობს რომ დაბუჯება წელზეც ამოვიდა და და მარცხენა მხარე მთლიანად შEგრძნება დაკარგული მაქვსო. იქნებ ვინმეს ქონია მსგავსი სიმპტომები სად და ვისთან იმკურნალა დაწერეთ თუ შეიძლება.
* * *
QUOTE
მირჩიეთ კარგი თანამედროვე ინფექციონისტი რა?!
დავიღალე  ექიმებთან სიარულით   (პრობლემები ისევ და ისევ წითელი ქარის შესახებ)

რამოდენიმე წლის წინ ნათესავი ქალი ლოგინად იყო ჩავარდნილი წითელი ქარის გამო, შემდეგ ვიღაცამ თუშმელიშვილის მალამო ურჩია, საუკეთესო შედეგი ქონდა, როგორც ვიცი იმ ქალმა ჯერ სოკოზე შემდეგ ქუსლის ნახეთქებზე უმკურნალა .მხოლოდ ამის შემდეგ დაუწყო წითელ ქარზე მკურნალობა. მისი მისამართია თამარაშვილის 21 ბინა16 ტელ 397742. ელისო.

კიდევ ერთხელ ეჩვენოს (უფრო)კვალიფიციურ ნევროლოგს.
ეს სიმპტომები შეიძლება ისტერია იყოს(ანუ ფუნქციური,ფსიქიკური),განსაკუთრებით თუ დაბუჟების არეალი არ შეესაბამება კონკრეტული ნერვის მიერ ინერვირებულ უბანს

Posted by: intriga55 12 Apr 2008, 11:01
texasuri jleta benzoxerxit

QUOTE
ჩემი ნიკის მიზანიც ეგაა,რომ წაიკითხავ გაგეცინოს.
რაღაც შენი გადახერხვა მომინდა მენჯ-ბარძაყის სახსრის დონეზე   


Gეი ხარ? lol.gif lol.gif lol.gif lol.gif

QUOTE
ეს სამედიცინო ტოპიკია და თავს უფლებას ვაძლევ "მედიცინურად" ბიღადავო.ჩვეულებრივ მოკვდავთავის სხვანაირად ვხუმრობ 


დარწმუნებული ვარ ექიმი არ ხარ. სტუდენტი ხარ და "ცოდნას" ამარიაჟებ yes.gif

QUOTE
ტანკეს ბედი,რომ არ მოგატყუო მკიდია,მაგრამ მიხარია უკეთ თუ არის(ისე ყველა კარგად თუ იქნა დავრჩებით ექიმები უმუშევარი)


rolleyes.gif

Posted by: texasuri jleta benzoxerxit 12 Apr 2008, 13:46
QUOTE (intriga55 @ 12 Apr 2008, 11:01 )
texasuri jleta benzoxerxit

QUOTE
ჩემი ნიკის მიზანიც ეგაა,რომ წაიკითხავ გაგეცინოს.
რაღაც შენი გადახერხვა მომინდა მენჯ-ბარძაყის სახსრის დონეზე   


Gეი ხარ? lol.gif lol.gif lol.gif lol.gif

QUOTE
ეს სამედიცინო ტოპიკია და თავს უფლებას ვაძლევ "მედიცინურად" ბიღადავო.ჩვეულებრივ მოკვდავთავის სხვანაირად ვხუმრობ 


დარწმუნებული ვარ ექიმი არ ხარ. სტუდენტი ხარ და "ცოდნას" ამარიაჟებ yes.gif

QUOTE
ტანკეს ბედი,რომ არ მოგატყუო მკიდია,მაგრამ მიხარია უკეთ თუ არის(ისე ყველა კარგად თუ იქნა დავრჩებით ექიმები უმუშევარი)


rolleyes.gif

გეი არ ვარ,თუმცა არაფერი მაქვს მათი საწინააღმდეგო.

ნეტა სტუდენტი ვიყო.

ვერ ღადაობ საერთოდ,იქნებ შენი "მე"-ს სხვა მხარეები გაგევითარებინა და წინ წანოგეწია,ცოტა საინტერესო რომ გახდე.ეს ისე "სამედიცინო-ფსიქოლოგიურ-სოციოლოგიური" რჩევა.

Posted by: tikope 12 Apr 2008, 17:35
თქვენ აქ იღადავეთ რამდენიც გინდათ, მეკი დაგიწერთ რომ დგეს გავუკეთეთ ოპერაცია და კარგადა ვართ, ახლა ძმაკაცებშია და იმათ ქეიფს უკურებს მარტო. სხვა რა გზა აქვს მარხვაზეცაა და არც შეიძლებაLULA_QABABI
დიდი მადლობა ,

Posted by: shtori 13 Apr 2008, 01:31
იქნებ დამეხმაროთ. რისი ბრალი შეიძლებ იყოს: თავის ტკივილი, დაბალი სიცხეები, ცხვირით ვერ სუნთქავს და გამონადენი აქვს, სახე შესიებული აქვს, ყურის ქვევითაც შესიებული აქვს, სისუსტე. კიდევ ხერხემალი უოფლიანდება და ტკივა. დაახლოებით 15 წლის წინ ჰქონდა ქაიმორიტი. შეიძლება რომ ეხლაც იგივე იყოს ტუ სხვა რამეა? თავის ტვინზე ხომ არ შეიძლება იყოს რამე, თუ ეს ხეხემლის ტკივილი რას შეიძლება გამოეწვია? თუ შეგიძლიათ მირჩიეთ რამე. რა შეიძლება რომ იყოს? და სად აჯობებს მისვლა, ვისთან?
დიდი მადლობა ყველას დახმარებისთვის

Posted by: intriga55 13 Apr 2008, 10:58
texasuri jleta benzoxerxit

QUOTE
ვერ ღადაობ საერთოდ,


აუF, მაგარი საღადაო ადგილი ნახე რა! რა მაგარი "ექიმი" ხარ?! vis.gif

QUOTE
იქნებ შენი "მე"-ს სხვა მხარეები გაგევითარებინა და წინ წანოგეწია,ცოტა საინტერესო რომ გახდე.ეს ისე "სამედიცინო-ფსიქოლოგიურ-სოციოლოგიური" რჩევა.


მადლობა რჩევისთვის gigi.gif


Posted by: texasuri jleta benzoxerxit 14 Apr 2008, 00:46
QUOTE (intriga55 @ 13 Apr 2008, 10:58 )
texasuri jleta benzoxerxit

QUOTE
ვერ ღადაობ საერთოდ,


აუF, მაგარი საღადაო ადგილი ნახე რა! რა მაგარი "ექიმი" ხარ?! vis.gif

QUOTE
იქნებ შენი "მე"-ს სხვა მხარეები გაგევითარებინა და წინ წანოგეწია,ცოტა საინტერესო რომ გახდე.ეს ისე "სამედიცინო-ფსიქოლოგიურ-სოციოლოგიური" რჩევა.


მადლობა რჩევისთვის gigi.gif

ექიმი კარგი ვარ ნამდვილად.შევწყვიტოთ ამ ტოპიკში ჩეთაობა,კარგი?
* * *
QUOTE (shtori @ 13 Apr 2008, 01:31 )
იქნებ დამეხმაროთ. რისი ბრალი შეიძლებ იყოს: თავის ტკივილი, დაბალი სიცხეები, ცხვირით ვერ სუნთქავს და გამონადენი აქვს, სახე შესიებული აქვს, ყურის ქვევითაც შესიებული აქვს, სისუსტე. კიდევ ხერხემალი უოფლიანდება და ტკივა. დაახლოებით 15 წლის წინ ჰქონდა ქაიმორიტი. შეიძლება რომ ეხლაც იგივე იყოს ტუ სხვა რამეა? თავის ტვინზე ხომ არ შეიძლება იყოს რამე, თუ ეს ხეხემლის ტკივილი რას შეიძლება გამოეწვია? თუ შეგიძლიათ მირჩიეთ რამე. რა შეიძლება რომ იყოს? და სად აჯობებს მისვლა, ვისთან?
დიდი მადლობა ყველას დახმარებისთვის

რაღა თქმა უნდა ჰაიმორიტის და/ან ფრონტიტის გამწვავებაა.ყბა-სახის ქირურგთან,ან ოტო-რინო-ლარინგოლოგთან.

ხერხემლის ოფლიანობა ცოტა გაუგებარია,ალბათ ზურგი იგულისხმე.ოფლიაობა ცოგადინფექციური პროცესის კლასიკური თანამგზავრია.ტკივილი ხერხემალშ-ოდტეოქონდროზი ან სხვა რამე.რენტგენია საჭირო.ასევე სისხლის საერთო.შარდის საერთოი და ტავის ქალის რენტგენოგრაფია,რამე უფრო სერიოზული დაავადება რომ არ გამოგეპაროთ.
ჰაიმორიტმა ფრონტიტმანატომიური თავისებურების გამო თავისუფლად შეიძლება გართულება მოგცეს მენინგიტის,შუა
ყურის ანთების სახით.

Posted by: tatusha25 14 Apr 2008, 14:42
QUOTE
vano_t

მადლობთ!ყველანაირი გამოკვლევა გავეთებული მაქვს.ეხლა უნდა გავაკეთო დათესვა გამოკვლევა მეორად ფლორაზე.იმედია მეშველება(((((

Posted by: ტანკე 15 Apr 2008, 21:48
QUOTE
ტანკეს ბედი,რომ არ მოგატყუო მკიდია,

გაიხარე

Posted by: texasuri jleta benzoxerxit 15 Apr 2008, 21:56
QUOTE (ტანკე @ 15 Apr 2008, 21:48 )
QUOTE
ტანკეს ბედი,რომ არ მოგატყუო მკიდია,

გაიხარე

არა "კეთილად" მკიდია.პასიურ-კეთილგანწყობილ-ინდიფერენტულად.ისე კარგი "რჩევები" დაგიწერე.დროსა და სივრცეში ვერ დავემთხვიეთ ერთმანეთს ოღონდ yes.gif

Posted by: gogra_gogra 16 Apr 2008, 03:18
მაინტერესებს კანზე ლაქები გამიჩნდა და ამის თაობაზე აქ უნდა ვიკითხო? (არა ეგ მეც ვიცი რომ ექიმთან მოსვლა გაცილებით სასურველია smile.gif )

Posted by: mtvareuli 16 Apr 2008, 03:53
gogra_gogra
QUOTE
მაინტერესებს კანზე ლაქები გამიჩნდა და ამის თაობაზე აქ უნდა ვიკითხო?

შეგიძლია აქაც იკითხო --> http://forum.ge/?f=43&showtopic=33686668
QUOTE
მეც ვიცი რომ ექიმთან მოსვლა გაცილებით სასურველია smile.gif )

რათქმაუნდა

მაგრამ თუ აქაც გინდა რამე სავარაუდო პასუხის მიღება, ცოტა უფრო კონკრეტულად მაინც თქვი wink.gif

რა ადგილას გაგიჩნდა ეგ ლაქები
რამდენი ხანია
რა ზომისაა, რა ფორმის, რამდენი
რა ფერის
როგორი კონტურები აქვს
რას უკავშირებ გაჩენას
ხო არ გტკივა, გექავება, გექერცლება
და ა.შ.

Posted by: ikebana 16 Apr 2008, 20:07
ნევროლოგიაში გამცემთ პასუხს?

აი თავის ტვინის კტ. კვლევის დასკვნა:
თავის ტვინის კტ. ჭრილებზე პერივენტრიკულური თეთრი ნივთიერების სიმკვრივე ორმხრივ ნორმასთან შედარებით დაქვეითებულია.
პარკუჭოვანი სისტემა დეფორმაციის გარეშეა.
სუბარაქნოიდული სივრცეები თავის ტვინის კონვექსზე ორივე მხარეს (მეტად თხემ-კეფის მიდამოში) შევიწროებულია.
თავის ტვინის შუამდებარე სტრუქტურათა გადახრა არ ვლინდება. ქალას ძვლოვანი სისტემა, რეტრობულბარული სივრცეები, წიაღები ხილული პათოლოგიის გარეშეა.
თურქული კეხის ფორმა და ზომა უცვლელი. ინტრასერალურად მოცულობით წარმონაქმნზე მიმანიშნებელი კტ.ნიშნები არ ფიქსირდება.


რას ნიშნავს ეს? ნორმიდან რა გადახრაა?

Posted by: *hedgehog* 17 Apr 2008, 13:10
მეგობრებო, თქვენი დახმარება მჭირდება.

მინდა მიმასწავლოთ კარგ თერაპევტთან, რომელიც ცოტა უცნაურ სიმპტომებში გაერკვევა და უბრალოდ არ მოგიცილებს ადამიანს თავიდან, დასკვნით: იგონებ, არაფერი არ გტკივა. ვიცი, რომ ცოტა არაეთიკურ რამეს ვითხოვ ექიმების მხრიდან, მაგრამ თქვენც ხომ იცით, რომ არსებობენ კარგი და ცუდი ექიმები. ძალიან გთხოვთ, პმ-ში მაინც მომწეროთ ექიმების კოორდინატები. წინასწარ დიდი მადლობა

Posted by: barca65 17 Apr 2008, 17:09
მემგონი ჩემი კითხვა უფრო ნეიროქირურგს ეხება, მაგრამ მსგავსი თემა ვერ ვნახე. იქნებ მაინც მიპასუხოს ვნმემ, ვინც ამაში ერკვევა:

შეიძლება თუ არა კუნთში (საჯდომზე) ნემსის გაკეთებამ გამოიწვიოს ფეხის პარალიზება? (ნაწილობრივ ან მთლიანად) თუ კი, რატომ და რითი შეიძლება მერე ფეხის კუნთების ფუნქციის აღდგენა?

Posted by: vano_t 18 Apr 2008, 23:54
ikebana
QUOTE
ნევროლოგიაში გამცემთ პასუხს?

აი თავის ტვინის კტ. კვლევის დასკვნა:
თავის ტვინის კტ. ჭრილებზე პერივენტრიკულური თეთრი ნივთიერების სიმკვრივე ორმხრივ ნორმასთან შედარებით დაქვეითებულია.
პარკუჭოვანი სისტემა დეფორმაციის გარეშეა.
სუბარაქნოიდული სივრცეები თავის ტვინის კონვექსზე ორივე მხარეს (მეტად თხემ-კეფის მიდამოში) შევიწროებულია.
თავის ტვინის შუამდებარე სტრუქტურათა გადახრა არ ვლინდება. ქალას ძვლოვანი სისტემა, რეტრობულბარული სივრცეები, წიაღები ხილული პათოლოგიის გარეშეა.
თურქული კეხის ფორმა და ზომა უცვლელი. ინტრასერალურად მოცულობით წარმონაქმნზე მიმანიშნებელი კტ.ნიშნები არ ფიქსირდება.


რას ნიშნავს ეს? ნორმიდან რა გადახრაა?
უპირველეს ყოვლისა რისთვის გაუკეთდა ადამიანს კომპიტერული ტომოგრაფია (რა სიმპტომები ქონდა?) და ასაკი ავადმყოფის.

რაც შეეხება კტ-ს მსგავს დასკვნებს, ბევრ დასკვნა მინახია რადიოგოის გაკეთებული, რომელიც periventricular white matter changes (chronic vascular disease, multiple small infarcts) მიუთითებს და სიმართლე რომ გითხრა, არ მინახია ავადმყოფი 70 წლის ზევით, რომელსაც მსგავს დასკვნას არ უწერენ. ასე რომ, არაფერს არ ნიშნავს საერთოდ. კტ არც არის ტესტი, რომელიც თეთრი ნივთიერების "დაავადებების" დიაგნოზისათვის იხმარება. ამისათვის ბმრ იხმარება. ასე რომ, არ ვინერვიულებდი მაგ დასკვნაზე. რისი დანახვაც კტ-ს შეუძლია კარგად (მაგალითად სიმსივნის ან ქალასშიდა წნევის მომატების ნიშნების, ან სისხლჩაქცევების) არ არის მაგ ტესტზე.

barca65
QUOTE
მემგონი ჩემი კითხვა უფრო ნეიროქირურგს ეხება, მაგრამ მსგავსი თემა ვერ ვნახე. იქნებ მაინც მიპასუხოს ვნმემ, ვინც ამაში ერკვევა:

შეიძლება თუ არა კუნთში (საჯდომზე) ნემსის გაკეთებამ გამოიწვიოს ფეხის პარალიზება? (ნაწილობრივ ან მთლიანად) თუ კი, რატომ და რითი შეიძლება მერე ფეხის კუნთების ფუნქციის აღდგენა?
შეიძლება. მაგ ადგილას გადის საჯდომი ნერვი, რომელიც ქვედა კიდურების ინერვაცია აკეთებს. საჯდომი ნერვის დაზიანება შეიძლება მოგცეს ნერვის დაზიანებამ ნებისმიერი ფიზიკური გამღიზიანებლით, ნემსის ჩათვლით.

ნერვის რეგენერაცია ან საერთოდ არ ხდება, ან ძნელად. თუ ნერვი მართლაც დაზიანდა, უნდა დადგინდეს დაზიანების ტიპი მაგალითად. აქსონის დაზიანება და ნერვის გარსის (ე.წ. შვანის გარსის ან მიელინის) დაზინაება სხვადასხვა შედეგებს მოგცემენ და პროგნოზიც სხვადასხვა იქნება. ისე დაზინებული ნერვის მთავარი მკურნალია დრო. ფიზიკური თერაპიაც იხმარება მის სამკურნალოდ.

Posted by: ikebana 19 Apr 2008, 00:10
vano_t
დიდი მადლობა პასუხისთვის.

ასაკია 30. კტ-ს მიზანი იყო ჰიპოფიზის ადენომის დადგენა.
დასკვნაში კი წერია ინტრაკრანიალური ჰიპერტენზია.

ეს რამეს ცვლის?

Posted by: vano_t 19 Apr 2008, 02:30
ikebana
QUOTE
ასაკია 30. კტ-ს მიზანი იყო ჰიპოფიზის ადენომის დადგენა.
დასკვნაში კი წერია ინტრაკრანიალური ჰიპერტენზია.

ეს რამეს ცვლის?

არა მგონია რამე შეცვალოს. ინტრაკრანიალური ჰიპერტენზია ან იმის გამო ვითარდება, რომ რაღაც მასა არის თავის ტვინში (რაც მაგ კტ-ზე არ აღინიშნება), ან იდიოპათიურია (ფსევდოტუმორ ცერებრისაც ეძახიან ამას). ასეთ ავადმყოფის სიმტომები არის თავისტკივილი (თითქმის ყველას აქვს ეგ), მხედველობის ცვლილება. რაც მთავარია, ოპტიკური დისკის შეშუპება აქვს თითქმის ყველას და რეტინის დათვარიელება არის საკმარისი ამისათვის. მომატებული წნევის დამტკიცება კი ცერებროსპინალური სითხის (ცსს) პუნქციით ხდება, რის დროსაც ადგენენ "გახსნის წნევას" (opening pressure). თუ ავადმყოფს არავითარი სიმპტომები არ აქვს ინტრაკრანიალური ჰიპერტენზიის და ბადურაც ნორმალურია, მაშინ კტ-ს მონაცემს ალბათ მნიშვნელობა არ აქვს. უფრო მეტიც, მე როგორც ვიცი კტ-ს იმისთვის კი არ აკეთებენ რომ დაადგინონ ინტრაკრანიალური ჰიპერტენზია, არამედ რომ გამორიცხონ მასის არსებობა სიმპტომატურ პაციენტებში (ანუ იმ პაციენტებში, რომელთაც აქვთ სიმტოტომები, ოპტიკური დისკის შეშუპება აქვთ და ცსს-ის წნევა მომატებული აქვთ-ანუ მომატებული ქალასშიგა წნევის სიმპტომები და ნიშნები აქვთ). სანდო ნევროლოგისაგან შეიძლება უფრო მეტი ინფორმაცია მიიღო.

ჰიპოფიზის ადენომის გამოკვლევას კიდევ ბმრ გაცილებით ჯობია როგორც მე ვიცი. კტ-მ მაგ არეში ბევრი არტეფაქტი შეიძლება მოგცეს ძვლოვანი სტრუქტურების სიჭარბის გამო.

http://www.ajnr.org/cgi/reprint/17/10/1953.pdf არის აღწერილი ორი კეისი, empty sella-სი (ჰიპოფიზის ატროფია) მსგავს პაციენტებში, თუმცა მაინც არაფერს ცვლის ეს.

ასე რომ, თუ ავადმყოფს ჩამოთვლილი რამეები არ აწუხებს და თვალის ფსკერი ნორმალურია, მაშინ არფერია სანერვიულო მაგ კტ-თი.

Posted by: ikebana 19 Apr 2008, 11:41
vano_t
კი, ბადურა ნორმალურია. თავის ტკივილები იშვიათად, რაც ძირითადად დამალ წნევას უკავშირდება.
დიდი მადლობა ამომწურავი პასუხისთვის smile.gif

Posted by: texasuri jleta benzoxerxit 19 Apr 2008, 13:27
გადარეული ვარ ვანოს პროფესიონალიზმით,მართლა!

Posted by: lala 19 Apr 2008, 16:58
ვარ 21 წლის
ძალიან ცუდი მახსოვრობა მაქვს
და რაღაც გათიშვები მაქვს ხოლმე. მავიწყდება ყველაფერი, მაგალლითად 5 წუთის წინ სად წავიღე რაღაც საგანი.....
რა პრეპარატი შეიძლება რო მივიღო
უკვე ძაან მაწუხებს
გთხოვთ დამეხმაროთ
ან რისი ბრალი შეიძლება იყოს?

Posted by: tamtuna 19 Apr 2008, 20:22
მადის დაკარგვა რას ნიშნავს ? რაღაც ჭამა აღარ მინდება....

Posted by: vano_t 19 Apr 2008, 21:00
lala
QUOTE
ვარ 21 წლის
ძალიან ცუდი მახსოვრობა მაქვს
და რაღაც გათიშვები მაქვს ხოლმე. მავიწყდება ყველაფერი, მაგალლითად 5 წუთის წინ სად წავიღე რაღაც საგანი.....
რა პრეპარატი შეიძლება რო მივიღო
უკვე ძაან მაწუხებს
გთხოვთ დამეხმაროთ
ან რისი ბრალი შეიძლება იყოს?
შეიძლება არფერიც არ იყოს, მარტივი სტრესის გარდა. რამდენი ხანი გაქვს ეგ პრობლემა და სხვა რაიმე ნიშნები თუ გაქვს? PM-ში შეგიძლია მომწერო თუ გინდა.

ნევროლოგის გამოკვლევა დაჭირდება მაგას. შეიძლება რაიმე ფსიქიატრიულიც იყოს (მაგალითად დეპრესია). თუმცა ყველა შემთხვევაში უნდა ინახოს რაიმე ორგანული დაავადება ხომ არ გაქვს. თავიდან ისევ ჯობია კარგად გაგსინჯოს და გამოგიკვლიოს თერაპევტმა, რომ გამორიცხოს სხვადასხვა შინაგანი დაავადებები (ფაქტიურად ალბათ ყველაფერს შეუძლია ეგ მოგცეს). ასევე უნდა დაგინდეს ფორმალური ტესტებით მახსოვრობა არ გივარგა თუ კონცენტრაცია. ახალგაზრდაში უფრო ხშირად პრობლემა ზუსტად კონცეტრაციის დაქვეითება არის (მაგალითი შეიძლება იყოს ყურადღების დეფიტის დაავადება ან მსგავი ყურადღების დეფიტის და ჰიპერაქტიურობის დაავადება). რაიმე რომ დაიმახსოვორ, ჯერ ყურადღებით უნდა დააკვირდე. არ არის აუცილებელი, რომ ეს ინფორმაცია ტვინში მიდიოდეს და იმის აღწარმოებას ვერ ახერხებდე. ტვინამდე თუ არ მივიდა, ვერაფერს გაიხსენებ. თუ თერაპევტმა/ნევროლოგმა გამორიცხა ორგანული დაავადება და პრობლემა გრძელდება და/ან სხვა სიმპტომებიც ჩნდება, მაშინ შეიძლება ფსიქიატრის/ფსიქოთერაპევტის/ფსიქოლოგის ნახვა დაგჭირდეს.

ყველა შემთხვევაში გირჩევდი ნახო თერაპევტი (ან ნევროლოგი) თავიდან. თუ ძალიან მოკლე ხნის პრობლემაა და სხვა არაფერი გაწუხებს, შეიძლება დაიცადო კიდევაც და დაიწყო ყურადღების გამახვილებით. თუ დეპრესიის სიმპტომები გაქვს (ადრე გაღვიძება, უხასიათობა, მელანქოლია, ინტერესის დაკარგვა არსებული ჰობების მიმართ, სუიციდის იდეები-განსაკუთრებით ეს- და ასე შემდეგ) მაშინ ეგრევე ფსიქიატრი უნდა ნახო.

tamtuna
მადის დაკარგვა შეიძლება მოგცეს ნებისმიერმ დაავადებამ რაც კი აღწერილია მედიცინაში. ასევე შეიძლება იყოს ფიზიოლოგიურიც. შეიძლება მოგცეს წამლებმაც. ამიტომ რაიმეს ასე თქმა შეუძლებელია. ეგ ერთ-ერთი ყველაზე არასპეციფიური სიმპტომი.

Posted by: tamtuna 19 Apr 2008, 21:26
vano_t
წამლებს არ ვსვავ, რავი და რაიმე პიზიკურად განსხვავებას ვერ ვგრძნობ

Posted by: natashkaa19 20 Apr 2008, 00:19
გამარჯობათ,
ყურიდან გამჭირვალე სითხე რომ გამოდის,საკმაოდ ბევრი, ისე რომ ბამბა ნამიანდება,რისი ბრალი შეიძლება იყოს? იქნებ ვინმე დამეხმაროს

Posted by: texasuri jleta benzoxerxit 20 Apr 2008, 00:50
ნატაშკა,აუცილებლად მიმართე ოტორინოლარინგოლოგს(ყელ-ყურ-სხვირის სპეციალისტს).ეს შეიძლება შედარებით უწყინარი გარეთა სასმენი მილის ანტება იყოს,ან დაზიანება სულაც შუა ყურში იყოს,დაფის აპკი სანახავია...ტკივილი არ გაქვს ყურის არეში?ქავილი?

უბრალოდ ვერ ვხვდები,ყურიდან თუ გადმოგდის სითხე რატომ არ მიდიხარ შესაბამის სპეციალისტთან?...შუა და შიგნითა ყურის ტვინის გარსებთან და თავის ტვინთან მჭიდრო ანატომიურ-ფიზიოლოგიური კავშირის გამო,ყურის პათოლოგია შეიძლება ამ სტრუქტურებზეც გადავიდეს!!!

Posted by: natashkaa19 20 Apr 2008, 21:08
texasuri jleta benzoxerxit
მადლობა პასუხისთვის.ტკივილი არ მქონდა არავითარი, სადღაც ნახევარი საათი გაგრძელდა და მაშინ მოგმართეთ და მერე თავისით შეწყდა,მაგრამ ექიმთან მაინც მივალ,უცნაური რამ იყო rolleyes.gif

Posted by: Tami28 21 Apr 2008, 12:35
vano_t

დედაჩემი ონკოლოგიური პაციენტია და იკეთებს ქიმიოთრპიას უკვე 9 თვეა. ბოლო პერიოდში დაეწყო ნეიტროფილებისა და ლეიკოციტების ვარდნა. ორჯერ გაკეთდა ნეუპოგენი. გთხოვთ მირჩიოთ კიდე რა საშუალებებით შეიძლება ამის გამოსწორება და/ან შეიძლება თუ არა ერთჯერადად ნეუპოგენის გაკეთება. გთხოვთ მირჩიოთ რამე. მადლობთ.

Posted by: vano_t 21 Apr 2008, 21:01
Tami28
QUOTE
დედაჩემი ონკოლოგიური პაციენტია და იკეთებს ქიმიოთრპიას უკვე 9 თვეა. ბოლო პერიოდში დაეწყო ნეიტროფილებისა და ლეიკოციტების ვარდნა. ორჯერ გაკეთდა ნეუპოგენი. გთხოვთ მირჩიოთ კიდე რა საშუალებებით შეიძლება ამის გამოსწორება და/ან შეიძლება თუ არა ერთჯერადად ნეუპოგენის გაკეთება. გთხოვთ მირჩიოთ რამე. მადლობთ.

ეგ ონკოლოგის გადასაწყვეტია. ნეუპოგენი მიეკუთვნება გრანულოციტების კოლონიის მასიმულირებელ ფაქტორს (G-CSF). ნეუპოგენი ფირმული დასახელებაა. რა თქმა უნდა არსებობს სხვა ფირმული წამლებიც, მაგალითად ნეულასტა.

თუმცა, მე არ ვიცი რაიმე განსხვავება თუ არის ეფექტის რაოდენობას შორის. ისე ლეიკოციტების რაოდენობა სპონტანურადაც უბრუნდება ნორმას ქიმიოთერაპიის შემდეგ. უბრალოდ, როცა აბსოლიტური ლეიკოპენია გააჩნია ავადმყოფს, ანტიბიოტიკებით რპოფილაქტიკა შეიძლება იყოს საჭირო; ავადმყოფმა უნდა აარიდოს კონტაქტი სხვებთან, რათა ინფექციური დაავადებების შეძენის რისკი შეამციროს. რამოდენიმე კვირაში მაინც უნდა აიწიოს ნეიტროფილები, მკურანლობის გარეშეც. მკურნალობა უბრალოდ აჩქარებს მაგ პროცესს. ბოლო ბოლო G-CSF ორგანიზმში წარმოქმნილი ნაერთია.

natashkaa19
QUOTE
გამარჯობათ,
ყურიდან გამჭირვალე სითხე რომ გამოდის,საკმაოდ ბევრი, ისე რომ ბამბა ნამიანდება,რისი ბრალი შეიძლება იყოს? იქნებ ვინმე დამეხმაროს
თავის ქალას ტრამვა ხომ არ მიგიღია ამ დღეებში (მაგალითად დაეცი და თავი დაარტყი?). მაგ შემთხვევაში ქალას ტრამვა უნდა გამოირიცხოს აუცილებლად. ყელყურის ექიმი ჩაგხედავს ყურში და შესაძლო მიზეზსაც დაადგენს. იმედია უკვე ექიმთან მიხვედი.

Posted by: baby-bobo 21 Apr 2008, 23:52
lala
QUOTE
ვარ 21 წლის
ძალიან ცუდი მახსოვრობა მაქვს
და რაღაც გათიშვები მაქვს ხოლმე. მავიწყდება ყველაფერი, მაგალლითად 5 წუთის წინ სად წავიღე რაღაც საგანი.....
რა პრეპარატი შეიძლება რო მივიღო
უკვე ძაან მაწუხებს
გთხოვთ დამეხმაროთ
ან რისი ბრალი შეიძლება იყოს?

მეც შენს დღეში ვარ და რა უნდა ვქნა გავიგე უკვე vano_t -ს პასუხიდან...

აქამდე მშობიარობის სტრესს ვაბრალებდი და თურმე სხვა მიზეზებიც შესაძლებელია არსებობდეს.... sad.gif

Posted by: Jacomo 1 May 2008, 20:28
ხალხო, მაინტერესებს, დიკლოფენაკის პარალელურად შეიძლება თუ არა ფურამაგის, ერიტრომიცინისა და ნისტატინის მიღება.

Posted by: Miss_Witch 1 May 2008, 22:00
ექიმებო შეკიტხვა მაქვს თქვენთან! smile.gif
მართალია რომ თუ უარყოფითი სისხლის ჯგუფი გაქვს ეს ცუდია შენი ჯანმრთელობისთვის? რომ უარყოფითი სისხლის მქონეებს უჭირთ მშბიარობა და სიკვდილის რისკი მაღალია?
ანუ ცუდია უარყოფითი სისხლის ჯგუფი?

Posted by: mtvareuli 1 May 2008, 22:57
Miss_Witch

არა, შენი ჯანმრთელობისთვის არაა ცუდი...

მშობიარობას რაც შეეხება, მართალია რეზუს-დადებითიან ნაყოFთან რეზუს-კომფლიქტი შეიძლება მოხდეს, მაგრამ ექიმების მეთვალყურეობის ქვეშ პრობლემას აღარ წარმოადგენს smile.gif

ერთადერთი ცუდი ისაა, რომ თუ სისხლის გადასხმა დაგჭირდა, მხოლოდ რეზუს-უარყოFითიანის სისხლი შეიძლება გადაგისხან, რაც უფრო იშვიათია და შესაბამისად შეიძლება დონორის პოვნა გაჭირდეს.


Posted by: texasuri jleta benzoxerxit 1 May 2008, 23:51
QUOTE (Jacomo @ 1 May 2008, 20:28 )
ხალხო, მაინტერესებს, დიკლოფენაკის პარალელურად შეიძლება თუ არა ფურამაგის, ერიტრომიცინისა და ნისტატინის მიღება.

კი შეიძლება,თუმცა ერთმანეტზე შეიძლება ისე იმოქმედოს,რომ მათი ნახევარგამოყოფის პერიოდი გახანგრძლივდეს ოდნავ.ეს მედიკამენტები მცირე ხნით გექნება დანიშნული და ამირტომ სვი თამამად

Posted by: tikope 2 May 2008, 16:55
vano_t
ხომ ვერ მირჩევთ რამე ბუნებრივ საშუალებას სისიხლის გასაწმენდად, რაიმე ბალახეული თუ არსებობს ან სხვა რამ?

Posted by: vano_t 3 May 2008, 10:26
Miss_Witch
QUOTE
მართალია რომ თუ უარყოფითი სისხლის ჯგუფი გაქვს ეს ცუდია შენი ჯანმრთელობისთვის? რომ უარყოფითი სისხლის მქონეებს უჭირთ მშბიარობა და სიკვდილის რისკი მაღალია?
ანუ ცუდია უარყოფითი სისხლის ჯგუფი?

მთვარეულმა ყველაფერი კარგად აღნიშნა. ერთს დავამატებდი ნაყოფის ამბავში, თუ რეზუს უარყოფით დედას რეზუს დადებითი ნაყოფი გააჩნია, მაშინ ნაყოფს შეიძლება განუვითარდეს მძიმე დაავადება (ჰიდროპს ფეტალის ეძახიან), თუმცა დღესდღეობით არსებობს სპეციალური ანტისხეულიბი რომელსაც რეზუს უარყოფით დედას უკეთებენ, რათა ეს პრობლემა თავიდან აიცილონ. ამას ყველა გინეკელოგი აკეთებს ახლა მემგონი.

tikope
QUOTE
ხომ ვერ მირჩევთ რამე ბუნებრივ საშუალებას სისიხლის გასაწმენდად, რაიმე ბალახეული თუ არსებობს ან სხვა რამ?
მსგავსი რამ (სისხლის გაწმენდას ვგულისხმობ) არ არსებობს მედიცინაში smile.gif ეგ უფრო ხალხური ტერმინია, მაგრამ არც მაგ ხალხურის შესაბამისი რამ არსებობს. სისხლს ჩვეულებრივ "წმინდავს" მშვენივრად ნორმალურად მომუშავე თირკმელი (ძირითადად და ნაწილობრივ ღვიძლი). რაიმე ტოქსინის გაძლიერებული გამოყოფისათვის (კლიერენსს ეძახიან მედიცინაში ამას) წყლის მეტი არაფერია საჭირო. ბევრი წყალი! ეს არის საუკეთესო "გამწმენდი" სისხლისა. თუმცა უზომოდ ბევრი წყლის მიღებაც არ შეიძლება. ძალიან ბევრი წყლის მიღებამ ძლიერი ჰიპონატრემია (ნატრიუმის რაოდენობის დაქვეითება) და მასთან დაკავშირებული სერიოზული პრობლემები შეიძლება მოგცეს. 3-4 ლიტრი სითხე დღეში ნორმალურია.

Jacomo
QUOTE
ხალხო, მაინტერესებს, დიკლოფენაკის პარალელურად შეიძლება თუ არა ფურამაგის, ერიტრომიცინისა და ნისტატინის მიღება.
texasuri jleta benzoxerxit-მა სრული პასუხი მოგცა ფაქტიურად. ერთს დავამატებდი მათთვის ვისაც აინტერესებს წამალთა ურთიერთქმედების გამოკვლევა. დღესდღეობით იმდენი წამალია მედიცინაში, რომ ფაქტიურად შეუძლებელია ამ ურთიერთქმედებების ცოდნა და არაპრაქტიკულიც არის. მე ამისათვის ვიყენებ epocrates (www.epocrates.com ვებსაიტიდან). ან ვებსაიტზე შეგიძლია გაწევრიანდე, ან ჩამოტვირთო და PDA-ზე დააინსტალირო იპოკრატეს პროგრამა. ჩამოტვირთვა და დაინსტალირება უფასოა. ვებსაიტიდან მე მგონი ფულის გადახდა უნდა. გარდა ამისა, შეგიძლიათ დაგუგლოთ drug interaction checker და რამოდენიმე ვებსაიტს ნახავთ სადაც შეგიძლიათ შეამოწმოთ. http://www.medscape.com/druginfo/druginterchecker?src=google&cm_mmc=google-_-News%2C+Journal%2C+Conferences+%26+Reference_Drug+Interaction+Checker-_-drug+interaction-_-Broad%7C-%7C100000000000000001105 სანდოა, ოღონდ გაწევრიანება უნდა ჯერ.

საერთოდ მაკროლიდებთან (ერიტრომიცინი, აზიტრომიცინი და ა.შ.) ფრთხილად უნდა იხმარო ანტიჰისტამინები (ალეგრა, ზირტეკი და ა.შ.), განსაკუთრებით მათში ვისაც გულის პრობლემები აქვთ. ამ კომბინაციამ შეიძლება მოგცეს QT ინტერვალის გახანგრძლივება და torsades de pointes (აბუ ტორსადების, რომელიც პოლიმორფული ვენტრიკულური არითმიაა და ძალიან საშიში არითმიაა) რისკი გაგიზარდოს. ამ კომბინაციას კი ჭირდება სიფრთხილე.

Posted by: tikope 3 May 2008, 17:36
vano_t
QUOTE
მცა უზომოდ ბევრი წყლის მიღებაც არ შეიძლება. ძალიან ბევრი წყლის მიღებამ ძლიერი ჰიპონატრემია (ნატრიუმის რაოდენობის დაქვეითება) და მასთან დაკავშირებული სერიოზული პრობლემები შეიძლება მოგცეს. 3-4 ლიტრი სითხე დღეში ნორმალურია.

შეგიძლიათ განმიმარტოთ უფრო ზუსტად და ჩემთვის გასაგებად, რა მოვლენები ახასიათEბს გარეგნულს ვგულისხმობ,გმადლობთ

Posted by: texasuri jleta benzoxerxit 3 May 2008, 21:27
Q-T ინტევალის გახანგრძლივების უნარი აქვთ ასევე ბოლო თაობის ფთორქინოლონებს.პეფლოქსაცინი,მოქსიფლოქსაცინი...კომბინაციის გარეშეც.

Posted by: Pathetikos 3 May 2008, 21:47
ბრმა ნაწლავის თემაშიც დავსვი ეს შეკითხვა და აქაც გავიმეორებ:


ოთხშაბათიდან მოყოლებული განსაკუთრებით საღამოობით თითქოს ძალა მერთმევა და ძარღვი მეჭიმებასავით მარჯვენა ფეხში. მუცლის მარჯვენა მხარეს,ქვევით, ტკივილი არ მაქვს, მაგრამ თითქმის გამუდმებით მაქვს რაღაც დაძაბულობის შეგრძნება, ხანდახან წვასავით (თითქოს შიგნით სიცივეა). ეს ყველაფერი განსაკუთრბით ჯდომისას მაქვს.
ბრმა ნაწლავი ამოჭრილი არ მაქვს.
ვიცი, რომ მარჯვენა ფეხის "გაკავება" ერთერთი სიმპტომია ბრმა ნაწლავის, მაგრამ რადგან არ ვიცი ზუსტად სადაა ბრმა ნაწლავი, მგონია, რომ შეიძლება ნაღველიც გამიღიზიანდა (ადრეც მქონია, ჩემი ღორობიდან გამომდინარე) და ეს წუხილი უფრო ნაღველისაა, ვიდრე ბრმა ნაწლავის.
პირველ რიგში, მინდა აღვნიშნო, რომ დეგენერატი ვარ, გუშინ რომ არ მივედი ექიმთან. ეხლა ორშაბამთამდე უნდა ვიჯდე და ვიყო ასე, თუ სასწრაფო არ დამჭირდა. biggrin.gif

მარა თითქოს იმაზე უფრო ქვევით მტკივა, ვიდრე ბრმა ნაწლავი და ნაღველია. ვვარაუდობ, რომ შეიძლება ძარღვიც იყოს გაციებული.

Posted by: baby-bobo 3 May 2008, 22:07
vano_t
QUOTE
მაშინ ნაყოფს შეიძლება განუვითარდეს მძიმე დაავადება (ჰიდროპს ფეტალის ეძახიან), თუმცა დღესდღეობით არსებობს სპეციალური ანტისხეულიბი რომელსაც რეზუს უარყოფით დედას უკეთებენ, რათა ეს პრობლემა თავიდან აიცილონ. ამას ყველა გინეკელოგი აკეთებს ახლა მემგონი.

აი მე არ ჩამითარეს ეგ მკურნალობა და კიდევაც იწვალა ცემმა პატარამ...
საბედნიეროდ ყველაფერი კარგად დამთავრდა...
QUOTE
მძიმე დაავადება (ჰიდროპს ფეტალის ეძახიან)

ეს შეგიძლია მოკლედ განმიმარტო რას ნიშნავს???
მისი კლინიკური ნიშნები და პროფილაქტიკა?
მადლობა....

Posted by: ikebana 3 May 2008, 22:31
აქ ვიკითხავ.

ვიონ ითვლება თბილისში კარგ ოპერატორად ღვიძლზე?

ექინოკოკის შემთხვევაა.

Posted by: texasuri jleta benzoxerxit 3 May 2008, 23:55
QUOTE (Pathetikos @ 3 May 2008, 21:47 )
ბრმა ნაწლავის თემაშიც დავსვი ეს შეკითხვა და აქაც გავიმეორებ:


ოთხშაბათიდან მოყოლებული განსაკუთრებით საღამოობით თითქოს ძალა მერთმევა და ძარღვი მეჭიმებასავით მარჯვენა ფეხში. მუცლის მარჯვენა მხარეს,ქვევით, ტკივილი არ მაქვს, მაგრამ თითქმის გამუდმებით მაქვს რაღაც დაძაბულობის შეგრძნება, ხანდახან წვასავით (თითქოს შიგნით სიცივეა). ეს ყველაფერი განსაკუთრბით ჯდომისას მაქვს.
ბრმა ნაწლავი ამოჭრილი არ მაქვს.
ვიცი, რომ მარჯვენა ფეხის "გაკავება" ერთერთი სიმპტომია ბრმა ნაწლავის, მაგრამ რადგან არ ვიცი ზუსტად სადაა ბრმა ნაწლავი, მგონია, რომ შეიძლება ნაღველიც გამიღიზიანდა (ადრეც მქონია, ჩემი ღორობიდან გამომდინარე) და ეს წუხილი უფრო ნაღველისაა, ვიდრე ბრმა ნაწლავის.
პირველ რიგში, მინდა აღვნიშნო, რომ დეგენერატი ვარ, გუშინ რომ არ მივედი ექიმთან. ეხლა ორშაბამთამდე უნდა ვიჯდე და ვიყო ასე, თუ სასწრაფო არ დამჭირდა. biggrin.gif

მარა თითქოს იმაზე უფრო ქვევით მტკივა, ვიდრე ბრმა ნაწლავი და ნაღველია. ვვარაუდობ, რომ შეიძლება ძარღვიც იყოს გაციებული.

რა თქმა უნდა თუ მიაღწიე ოთხშაბათამდე...უმძიმესი დაავადების სიმპტომებს აღწერ.არც კი ვიცი,რა გითხრა.ფაქტიურად გათავებული ამბავია gigi.gif gigi.gif gigi.gif
ვხუმრობ რა თქმა უნდა.ბრმა ნაწლავის(ვოობშე-ტო ჭიაყელა ნაწლავის)ანთებას ცოტა სხვა სიმპტომები აქვს.ტემპერატურა,ტკივილი ჯერ ეპიგასტრიუმში(კუჭის არეში) ან დიფუზურად და მეორე-მესამე დღეს ტკივილი ლოკალიზდება ბრმა ნაწლავის საპროექციო არეში(თუ ის ტიპიურად მდებარეობს.ატიპიური მდებარეობის შემთხვევაში უფრო ძნელია დიაგნოსტიკა),ხშირია გულისრევა/ღებინება და რაც მთავარია მუცლიც ფარის(პერიტონეუმის)გაღიზიანების ნიშნები,რაც გასინჯვით ვლინდება.თუმცა შეიძლება იყოს წაშლილი ფორმები და ეს სიმპტომები საერთოდაც არ იყოს.

გირჩევ ეჩვენო ქირირგს.
მე კი უფრო ნევრალგია,ნევრიტი მგონია.საჯდომი ნერვის ანთება.("იშიაზი"),ამას კი ნევრო(პათო)ლოგი ჭირდება.გირჩევდი ჯანო ვალიშვილს ან დათო ბათიაშვილს რესპუბლიკურ საავადმყოფოში.

Posted by: Pathetikos 4 May 2008, 10:32
texasuri jleta benzoxerxit
მადლობა. biggrin.gif


ოჯახში გუშინ დავადგინეთ, რომ ის ადგილი, სადაც მაწუხებს, ბრმა ნაწლავზე და ნაღველზე ბევრად ქვევითაა. biggrin.gif
მხედველობაში ვიღებ რა, რომ წინა კვირაში "ჩუმად" შემცივდა და მეც ტიტლიკანა დავბროდი, ნერვის გაციება-ანთება დავადგინეთ (რომლის არ ვიცი biggrin.gif), ხოდა ვგლიჯე ნიმესილი და დღეს დილით ჩიტივით გავიღვიძე. გავაგრძელებ დღეს ნიმესილის სმას და ხვალ უკვე ექიმთან წავალ.


დიდი მადლობა. ბაროკოს მადლი არ მოგკლებოდეს. biggrin.gif

Posted by: ikebana 5 May 2008, 00:10
QUOTE
აქ ვიკითხავ.

ვიონ ითვლება თბილისში კარგ ოპერატორად ღვიძლზე?

ექინოკოკის შემთხვევაა.



აზრზე არაა არავინ?

Posted by: vano_t 5 May 2008, 08:26
tikope
QUOTE
QUOTE
მცა უზომოდ ბევრი წყლის მიღებაც არ შეიძლება. ძალიან ბევრი წყლის მიღებამ ძლიერი ჰიპონატრემია (ნატრიუმის რაოდენობის დაქვეითება) და მასთან დაკავშირებული სერიოზული პრობლემები შეიძლება მოგცეს. 3-4 ლიტრი სითხე დღეში ნორმალურია.

შეგიძლიათ განმიმარტოთ უფრო ზუსტად და ჩემთვის გასაგებად, რა მოვლენები ახასიათEბს გარეგნულს ვგულისხმობ,გმადლობთ
რა მოვლენები ახასიათებს რას? დიდი რაოდენობით წყლის მიღებას?

baby-bobo
QUOTE
ეს შეგიძლია მოკლედ განმიმარტო რას ნიშნავს???
მისი კლინიკური ნიშნები და პროფილაქტიკა?
მადლობა....
ჰიდროპსი (hysdrops fetalis) მოკლედ არის ნაყოფის დაავადება და გამოწვეულია იმით, რომ დედის ორგანიზმში წარმოქმნილი ანტისხეულები იწვევენ ნაყოფის სისხლის "დაშლას", რასაც მოყვება ნაყოფის შეშუპება და დიდი რაოდენობით სითხის დაგროვება ნაყოფის სხვადასხვა ორგანოებში. უმეტეს შემთხვევაში ნაყოფის სიკვდილით მთავრდება. ჰიდროპსი ნაყოფის სხვადასხვა დაავადებებმა შეიძლება გამოიწვიოს. რეზუს შეუთავსებლობა უბრალოდ ერთერთი მიზეზია. მისი დადგენა არის გინეკოლოგის საქმე და დედის სისხლის ანალიზითა და ნაყოფის ულტრაბგერული გამოკვლევებით ხდება. პროფილაქტიკა იმით ხდება, რომ დედას უნიშნავენ სპეციალურ ანტისხეულებს, რომელიც საბოლოოდ ბლოკავს დედის მიერ "ნაყოფის სისხლის დამშლელი" ანტისხეულეიბის წარმოშობას.

Posted by: baby-bobo 5 May 2008, 14:58
არ ვიცი სად ვიკითხო და აქ დავწერ იქნებ დამეხმაროთ....

14 წლის გოგოს უკბინა ზაზუნამ... საჭიროა აცრა ცოფზე???
როგორ მოვიქცეთ???

Posted by: texasuri jleta benzoxerxit 5 May 2008, 15:33
QUOTE (baby-bobo @ 5 May 2008, 14:58 )
არ ვიცი სად ვიკითხო და აქ დავწერ იქნებ დამეხმაროთ....

14 წლის გოგოს უკბინა ზაზუნამ... საჭიროა აცრა ცოფზე???
როგორ მოვიქცეთ???

სახლის ზაზუნამ?

სასწრაფოდ რაბიოლოგთან!!!(ინფექციურში მიდით,მიმღებში)ის გადაწყვეტს შემდგომ ტაქტიკას!

მღრღნელები ცოფის ვირუსის ხშირი გადამტანები არიან.სასწრაფოდ!

Posted by: baby-bobo 5 May 2008, 15:49
texasuri jleta benzoxerxit
QUOTE
სახლის ზაზუნამ

ხო სახლის ზაზუნამ sad.gif
QUOTE
სასწრაფოდ რაბიოლოგთან!!!(ინფექციურში მიდით,მიმღებში)ის გადაწყვეტს შემდგომ ტაქტიკას!

მღრღნელები ცოფის ვირუსის ხშირი გადამტანები არიან.სასწრაფოდ!

sad.gif

Posted by: tikope 5 May 2008, 20:37
vano_t
QUOTE
რა მოვლენები ახასიათებს რას? დიდი რაოდენობით წყლის მიღებას?

დიახ ეს მაინტერესებს, რა გარეგნული გამოხატულება აქვს?

Posted by: vano_t 5 May 2008, 23:44
tikope
QUOTE
QUOTE
რა მოვლენები ახასიათებს რას? დიდი რაოდენობით წყლის მიღებას?

დიახ ეს მაინტერესებს, რა გარეგნული გამოხატულება აქვს?

ბევრი რამ შეიძლება ახასიათებდეს: მაგალითად სისუსტე, დაღლილობა, კუნთების ციმციმი, მომატებული ძილიანობა, უმადობა და ასე შემდეგ. ამით დიაგნოზს ვერ დასვამ. დიაგნოზის დასასმელად უბრალოდ სისხლის შემოჭმება არის საჭირო და შარდის შემოწმება. სისხლში დაკლებული იქნება ნატრიუმი ძალიან და ოსმოლარობაც (ერთერთი მახასიათებელია სისხლისა, რაც განისაზღვრება სისხლში ნაწილაკლების-მოლეკულების, იონების-საერთო რაოდენობით) დაბალი იქნება. ასევე, შარდის ოსმოლარობა იქნება დაბალი. როცა ეს ლაბორატორიული მონაცემები გაქვს, მაშინ იცი რომ ავადმყოფს აქვს წყლისაგან "მოწამლვა", ანუ პირველადი პოლიდიპსია.

Posted by: tikope 6 May 2008, 10:44
vano_t
დიდი მადლობა
smile.gif smile.gif

Posted by: intriga55 6 May 2008, 13:53
tikope
vano_t
ჰეჰ! ვერც კითხვისგან გავიგე რამე და ვერც პასუხისგან vis.gif

თუ ცხელა და თან მარილიანი ვჭამე და მერე ბევრი წყალი ვსვი, ანალიზებზე უნდა გავიქცე? baby.gif


Posted by: keen 7 May 2008, 16:28
შავი წერტილები ფრჩხილებში, სავარაუდოდ - სისხლჩაქცევები, რით შეიძლება იყოს გამოწვეული? baby.gif

Posted by: vano_t 7 May 2008, 18:30
keen
QUOTE
შავი წერტილები ფრჩხილებში, სავარაუდოდ - სისხლჩაქცევები, რით შეიძლება იყოს გამოწვეული?  baby.gif

თვითონ ფრჩხილში სისხლჩაქცევა შეუძლებელია. ფრჩხილებს არ აქვს სისხლმომარაგება. ფრჩხილი რა ქსოვილზეც ძევს, იქ შეიძლება მოხდეს სისხლჩაქცევა და ყველაზე ხშირად ეგ არის ტრამვის მიზეზი. ბაქტერიულ ენდოკარდიტსაც შეუძლია მოგცეს ეგ, მაგრამ ბაქტერიული ენდოკარდიტი სერიოზულად მიმდინარეობს და ექიმს არ შეეშლება.

გარდა ამისა, შეიძლება სისხლჩაქცევა არ იყოს და უბრალოდ ფრჩხილის პიგმენტაცია იყოს. სურათი დადე და უფრო ადვილად დაგეხმარები.

QUOTE
თუ ცხელა და თან მარილიანი ვჭამე და მერე ბევრი წყალი ვსვი, ანალიზებზე უნდა გავიქცე?
თუ ცხელა, მარილს უნდა აარიდო თავი (განსაკუთრებით დამარილებულ ვობლას). თუ მარილს არ აარიდე თავი, მაშინ ბევრი წყლის დალევა მოგიწევს. ექიმზე არ ინერვიულო მაგ დროს.

Posted by: sexi-bicola 12 May 2008, 19:05
მოკლედ 2 წელია ვიღებ სპლატს შლაკებისგან გასაწმენდად
შარშან გაწყდა და მაშინ ჭორები წავიდა რო აკრძალესო
ეხლა ისევ გამოჩნდა და ისევ ვიყიდე
იქნებ მიტხრათ რა ხდება?

Posted by: ivereanu 13 May 2008, 22:51
ხალხო ვინმე ვერ ამიხსნით რა არის ეზოფაგოგასტროდუოდენოფიბროსკოპია ? help.gif

Posted by: texasuri jleta benzoxerxit 14 May 2008, 00:33
QUOTE (ivereanu @ 13 May 2008, 22:51 )
ხალხო ვინმე ვერ ამიხსნით რა არის ეზოფაგოგასტროდუოდენოფიბროსკოპია ? help.gif

აგიხსნი:"ზონდის გადაყლაპვა"
ასეთია მისი "ხალხური" სახელი.გულისხმობს საყლაპავ-კუჭ-თორმეტგოჯას ენდოსკოპიურ გამოკვლევას.

Posted by: ivereanu 14 May 2008, 00:43
texasuri jleta benzoxerxit
ეგრეც ვიცოდი მადლობააა

Posted by: tikope 14 May 2008, 23:44
იქნებ ნივთიერებათა ცვლაზე დაწეროთ, და ასევე თუ დარღვეულია, რა საშუალებით იკურნება?გმადლობთ

Posted by: baby-bobo 16 May 2008, 16:07
ხშირად მეკვანძება ფეხი.
ჭიანჭველებივით დამდის რაღაც და რამდენიმე წუთი ფეხს ვერ ვადგავ.
რისი ბრალი შეიძლება იყოს, ვის მივმართო?

Posted by: Cor-toni 16 May 2008, 17:37
baby-bobo
QUOTE
ხშირად მეკვანძება ფეხი.
ჭიანჭველებივით დამდის რაღაც და რამდენიმე წუთი ფეხს ვერ ვადგავ.
რისი ბრალი შეიძლება იყოს, ვის მივმართო?

პანანგინი (ან ასპარკამი) დალიე 1-2 აბი 3-ჯერ დღეში. თუმცა მანამდე უმჯობესია კალიუმის დონე შეამოწმო სისხლში. ბევრი სითხე დაკარგე ბოლო დღეებში (ღებინება, ხშირი შარდვა, ფაღარათი)?


Posted by: baby-bobo 16 May 2008, 17:40
cor-toni
QUOTE
თუმცა მანამდე უმჯობესია კალიუმის დონე შეამოწმო სისხლში. ბევრი სითხე დაკარგე ბოლო დღეებში (ღებინება, ხშირი შარდვა, ფაღარათი)?

არაა, არაფერი მაფდაგვარი.

მადლობა დიდი...

Posted by: hirondelle 18 May 2008, 10:16
ეგზემა გამიჩნდა თავზე, რა ჯანდაბაა ხომ ვერ ამიხსნით? საშიშია ნეტა?

Posted by: Cor-toni 18 May 2008, 18:00
hirondelle
რა იცი, რომ ეგზემაა? ნებისმიერ შემთხვევაში დერმატოლოგს უნდა მიმართო.

Posted by: hirondelle 18 May 2008, 20:20
cor-toni
მივმართე და რაღაც მაზი დამინიშნა, მაგრამ აქაური (ბელგია) ექიმები სიტყვაძუნწები არიან და ნორმალურად არც აუხსნია რა არის ეს. რამდენად საშიშია ნეტა? ეთადერთი რაც მითხრა ეგზემაა და გადამდები არ არისო....

Posted by: baby-bobo 18 May 2008, 21:27
აუ ფეხები მაწუხებს მაგრად. მტკივა და სიმძიმის შეგრძნება მაქვს.
ორსულობაში მქონდა ჰიპერკუაგულაცია.
მშობიარობის მერე ანალიზი არ გადამიმოწმებია.

ვის მივმართო? ანგიოლოგს ალბათ ხო....
ფრაქსიპარინს ვიკეთებდი ორსულობაში და თრომბოასპირინს ვსვავდი....

ვინ არის ამ საკითხში კარგი ექიმი? კოორდინატები?

Posted by: Cor-toni 18 May 2008, 23:12
baby-bobo
ფეხების ტკივილი სხვადასხვა მიზეზის გამო შეიძლება იყოს. რამდენიმე დღის წინ შენ თქვი ღამით ფეხები მეკვანძებაო. ამ დროს კალიუმის დეფიციტია უფრო ხშირად მიზეზი და სწორედ ამიტომ გირჩიე პანანგინი და პარალელურად_კალიუმის დონის განსაზღვრა სისხლში.

თუ ტკივილი და სიმძიმის შეგრძნება გაქვს, შეიძლება ვენური უკმარისობის გამო იყოს. ამ დროს ფეხების ზევით აწევა შვებას გაძლევს? და დატვირთვის/ბევრი სიარულის შემდეგ გტკივა? დეტრალექსსაც ხომ არ სვამდი ადრე?

თუ "დაბუჟების" შეგრძნება გაქვს, შეიძლება ქვემო კიდურის ნერვის კომპრესიის გამო იყოს. ორივე ფეხზე გაქვს ტკივილი თუ ერთზე? ან კიდევ ჰიპერკოაგულაციის გამო. მითუმეტეს, ადრეც გქონია ეს (ისე ასპირინი ორსულობის დროს არ ინიშნება, თუნდაც თრომბო ან კარდიოასპირინი).

ამიტომ, ძალიან კარგი გადაწყვეტილებაა ანგიოლოგს მიმართო! yes.gif smile.gif


Posted by: texasuri jleta benzoxerxit 18 May 2008, 23:34
QUOTE (hirondelle @ 18 May 2008, 20:20 )
cor-toni
მივმართე და რაღაც მაზი დამინიშნა, მაგრამ აქაური (ბელგია) ექიმები სიტყვაძუნწები არიან და ნორმალურად არც აუხსნია რა არის ეს. რამდენად საშიშია ნეტა? ეთადერთი რაც მითხრა ეგზემაა და გადამდები არ არისო....

საშიში რა არ მოგკლაცვს,მაგრამ სასიამოვნო არ არის.
ადგილობრივი მკურნალობის გარდა აუცილებლად ნერვული სისტემა უნდა დაიმშვიდო.ემოციური,მდრძნობიარე,მშფოთვარე-დეპრესიული შტრიხები უნდა გქონდეს პიროვნულ სტრუქტურაში.

Posted by: baby-bobo 18 May 2008, 23:44
cor-toni
QUOTE
ფეხების ტკივილი სხვადასხვა მიზეზის გამო შეიძლება იყოს. რამდენიმე დღის წინ შენ თქვი ღამით ფეხები მეკვანძებაო. ამ დროს კალიუმის დეფიციტია უფრო ხშირად მიზეზი და სწორედ ამიტომ გირჩიე პანანგინი და პარალელურად_კალიუმის დონის განსაზღვრა სისხლში.

ეს ფეხის კვანძა 3-4 დღე მქონდა, ეხლა გადამიარა. ფეხები მანამდეც მაწუხებდა....
სიარული არაფერშუაშია, დილით უკვე ფეხები დამძიმებული მაქვს და სულ დაზელვის სურვილი მაქვს.
მთელი ორსულობა ასე ვიყავი. ორივე ფეხი ერთნაირად მაწუხებს sad.gif
QUOTE
ან კიდევ ჰიპერკოაგულაციის გამო

მე უფრო ამას ვვარაუდობ, მოკლედ ექიმთან უნდა მივიდე....

ხომ ვერ მირჩევდით ვინმეს?


QUOTE
(ისე ასპირინი ორსულობის დროს არ ინიშნება, თუნდაც თრომბო ან კარდიოასპირინი).

იცი დღეს უკვე ამას აქტიურად ნიშნავენ, რამდენიმე კიდევ ვიცი ფორუმზე ვინც ამას სვავდა ორსულობის დროს....
მეც წავიკითხე ანოტაციაში რომ არ შეიძლება მაგრამ კონკრეტულად მესამე ტრიმესტრში, მერე არც დამილევია.
რავიცი აბა ექიმმა დალიო და ვსვი.

მადლობა დიდი.

Posted by: hirondelle 18 May 2008, 23:46
texasuri jleta benzoxerxit
დიდი მადლობა..... ისე ამ ბოლო დროს ცოტა დეპრესიული კი გავხდი. ნოსტალგია და გაუცხოების გრძნობა მაწუხებს.....

Posted by: Cor-toni 19 May 2008, 00:08
baby-bobo
QUOTE


QUOTE
(ისე ასპირინი ორსულობის დროს არ ინიშნება, თუნდაც თრომბო ან კარდიოასპირინი).


იცი დღეს უკვე ამას აქტიურად ნიშნავენ, რამდენიმე კიდევ ვიცი ფორუმზე ვინც ამას სვავდა ორსულობის დროს....
მეც წავიკითხე ანოტაციაში რომ არ შეიძლება მაგრამ კონკრეტულად მესამე ტრიმესტრში, მერე არც დამილევია.
რავიცი აბა ექიმმა დალიო და ვსვი.


http://forum.ge/?f=43&showtopic=33843750

ეს "დღევანდელია" და არა უკვე მოძველებული ინფო და, შესაბამისად, ასპირინის დანიშვნა ორსულებში სიახლე არ არის. ამასთან, ასპირინი არ ინიშნება არა მარტო მესამე ტრიმესტრში.

QUOTE
მადლობა დიდი.

არაფერს smile.gif მერე დაწერე როგორ იქნები smile.gif


Posted by: texasuri jleta benzoxerxit 19 May 2008, 00:17
QUOTE (hirondelle @ 18 May 2008, 23:46 )
texasuri jleta benzoxerxit
დიდი მადლობა..... ისე ამ ბოლო დროს ცოტა დეპრესიული კი გავხდი. ნოსტალგია და გაუცხოების გრძნობა მაწუხებს.....

დერმატოლოგ ნინო ცისკარიშვილთან მიდი რა.ნინო და უდიდეს ჩვენ ფსიქიატრ- შოთა გამყრელიძე ახალი მიმდინარეობის-ფსიქოდერმატოლოგიის " ფუძემდებლები არიან საქართველოში.დერმატოლოგიური პრობლემები ხშირ შემთხვევაში აფექტური აშლილობების ნიღბად,მის ექვივალენტად განიხილება.ჯერ დეპრესიული გახდი და მერე გაჩნდა ეგზემა ხო?ან ერთდროულად?
ნინო ცისკარიშვილი კონსტიტუციის ქუჩაზე,კან.ვენ.-ის კლინიკაშია. აბა წარმატებები!

Posted by: vano_t 20 May 2008, 04:15
baby-bobo
ქვემო კიდურები ღრმა ვენების თრომბოზის დაიგნოზს ჭირდება მხოლოდ ულტრაბგერა. ულტრაბგერა საკმარისია იმისათვის, რომ დაიგნოზი დასვა ან გამორიცხო. არავითარი ანგიოლოგი არ ჭირდება დიაგნოზისათვის. ნებისმიერ თერაპევტს შეუძლია მაგ ტესტის შეკვეთა. ხოლო ტესტს კითხულობს რადიოლოგი.

QUOTE
ეს ფეხის კვანძა 3-4 დღე მქონდა, ეხლა გადამიარა. ფეხები მანამდეც მაწუხებდა....
სიარული არაფერშუაშია, დილით უკვე ფეხები დამძიმებული მაქვს და სულ დაზელვის სურვილი მაქვს.
მთელი ორსულობა ასე ვიყავი. ორივე ფეხი ერთნაირად მაწუხებს sad.gif
როგორც კორტონმა თქვა, ბევრი მიზეზი შეიძლება იყოს. ულტრაბგერის გაკეთება ალბათ კარგი იქნება. გარდა ამისა, უნდა შემოწმდეს სისიხლი სხვადასხვა ანალიზზე და დეტალურად უნდა გამოგკითხოს და გამოგიკვლიოს ექიმმა. თვითონ ორსულობის დროს ხშირია მასეთი სიმპტომები. მე მაგალითად გავაკეთებდი სისიხლის საერთო ანალიზს, ერითროციტების დალექვსი სიჩქარეს, რევმატოიდულ ფაქტორს, ელექტროლიტებს, შარდოვანას, კრეატინინს და ღვიძლის ენზიმებს და TSH-ს. თუ ყველაფერი ნორმაში იქნება, მაშინ აღარფერს გავაკეთებდი (გარდა მკურნალობის დანიშვნისა-ტკივიგამაყუჩებლები და ვარჯიშები) და მოვიცდიდი რამოდენიმე თვეს. თუ კიდევ გაგრძელდებოდა, მერე შეიძლება ბევრ რამეზე იფიქრო და სხვადასხვა ტესტი გააკეთო.

კორ-ტონი იმედია არ მიწყენს ამას, ცოტა შესწორება უნდა ასპირინის ამბებს. სხვადასხვა მოსაზრებაა ამის თაობაზე. თუმცა რუტინულად ასპირინის სმას არ ურჩევენ არცერთ შმეთხვევაში. ზოგ შემთხვევაში შეიძლება დანიშნოს ექიმმა ასპირინი (მაგალითად პრე-ეკლამპსიის რისკის შესამცირებლად იმ ავადმყოფებში ვისაც დიდი რისკი აქვთ, ან ანტიფოსფოლიპიდური ანტისხეულების სინდრომის დროს ჰეპარინთან ერთად) თუმცა. ყველა შემთხვევაში ექიმის მეთვალყურეობით ხდება ეს.

Posted by: baby-bobo 20 May 2008, 11:21
vano_t
დიდი მადლობა.
გთხოვ ნახო PM.

Posted by: Cor-toni 20 May 2008, 15:12
vano_t
QUOTE
ქვემო კიდურები ღრმა ვენების თრომბოზის დაიგნოზს ჭირდება მხოლოდ ულტრაბგერა. ულტრაბგერა საკმარისია იმისათვის, რომ დაიგნოზი დასვა ან გამორიცხო. არავითარი ანგიოლოგი არ ჭირდება დიაგნოზისათვის.

მე ვიგულისხმე ვენური უკმარისობა და არა ღრმა ვენების თრომბოზი. ქვემო კიდურის ვენებისა და მისი სარქველების მდგომარეობის განსაზღვრა. ამიტომ არის საჭირო ანგიოლოგი.
QUOTE
კორ-ტონი იმედია არ მიწყენს ამას, ცოტა შესწორება უნდა ასპირინის ამბებს. სხვადასხვა მოსაზრებაა ამის თაობაზე. თუმცა რუტინულად ასპირინის სმას არ ურჩევენ არცერთ შმეთხვევაში. ზოგ შემთხვევაში შეიძლება დანიშნოს ექიმმა ასპირინი (მაგალითად პრე-ეკლამპსიის რისკის შესამცირებლად იმ ავადმყოფებში ვისაც დიდი რისკი აქვთ, ან ანტიფოსფოლიპიდური ანტისხეულების სინდრომის დროს ჰეპარინთან ერთად) თუმცა. ყველა შემთხვევაში ექიმის მეთვალყურეობით ხდება ეს.

რა უნდა მეწყინოს? spy.gif სწორედ ეგ ვთქვი მეც! ალბათ არ წაგიკითხავს ეს:
http://forum.ge/?f=43&showtopic=33843750
ანუ იგივე წერია მანდაც სამ ენაზე: ქართულად, რუსულად, ინგლისურად yes.gif smile.gif

QUOTE
ზოგ შემთხვევაში შეიძლება

არ ნიშნავს რომ სწორია ეს:
QUOTE
დღეს უკვე ამას აქტიურად ნიშნავენ, რამდენიმე კიდევ ვიცი ფორუმზე ვინც ამას სვავდა ორსულობის დროს....

smile.gif


Posted by: Bivrili 20 May 2008, 15:50
ესე იგი დავიტანჯე რა ...................
მუსიკის მოსმენა მინდა არადა ვერ ვახერხებ.................
ყურთსასმენების გამოყენების შემდეგ (თუნდაც 10 წუთით) საშინელი თავის ტკივილი მეწყება..............
ტკივილი იწყება თავის ზედა მხარეს და კისრის მიდამოში და მერე მთელს თავზე ვრცელდება......................
თუ გამაყუჩებელი არ დავლიე არ მივლის.......................


ისე თავის ტკივილები ადრეც მქონდა და ვიმკურნალე (მითხრეს რომ თავის შიგა ქალის წნევა იყო მომატებული) , მაგრამ რამდენად ეფექტური იყო არ ვიცი...........................


რამე სერიოზული იქნება ვითომ? .............რატომ ვერ უძლებს თავი სულ მცირე ხნის დატვირთვასაც კი?..................თუ ყურის ბრალი შეიძლება იყოს?......................

Posted by: vano_t 20 May 2008, 21:27
cor-toni
QUOTE
არ ნიშნავს რომ სწორია ეს:
QUOTE
დღეს უკვე ამას აქტიურად ნიშნავენ, რამდენიმე კიდევ ვიცი ფორუმზე ვინც ამას სვავდა ორსულობის დროს....
ნამდვილად. სწორი კი არა, არასწორია ასე ასპირინის ყველა ავადმყოფში დანიშვნა.

QUOTE
მე ვიგულისხმე ვენური უკმარისობა და არა ღრმა ვენების თრომბოზი. ქვემო კიდურის ვენებისა და მისი სარქველების მდგომარეობის განსაზღვრა. ამიტომ არის საჭირო ანგიოლოგი.
მართალი ხარ. ვენური უკმარისობაც იწვევს მსგავს სიმპტომებს, ოღონდ მაგის დიაგნოზიც უნდა მოხდეს თერაპევტის მიერ წესით (ულტრაბგერა მოგცემს მაგის საშუალებას) და წესით თერაპევტმა უნდა მიიტანოს მაგაზეც ეჭვი. შესაბამისად, თერაპევტმა უნდა დანიშნოს ტესტიც (ულტრაბგერა, თუ ვენოგრამა არის საჭირო, მაშინ ალბათ ანგიოლოგმა დანიშნოს ტესტი ჯობია). მკურანალობა რა თქმა უნდა ანგიოლოგის მიერ უნდა იქნას ჩატარებული.

ისიც მესმის რომ დღევანდელი მიდგომა მართლა მასეა საქართველოში, როგორც შენ აღნიშნე. ავადმყოფი პირდაპირ მიდის სპეციალისტთან უმეტეს შემთხვევაში, თუმცა არასწორია მაგ და უნდა შეიცვალოს მიდგომა wink.gif მერე პაციენტსიც კმაყოფილი იქნება და თერაპევტი ექიმიც (რადგანაც გაღრმავებულ ცოდნის შეძენა დაჭირდება და ფართო პროფილის დარჩება). შეიძლება დღეს უტოპიაა, მაგრამ მსგავსი განათლება და მომზადება თერაპევტებისა დღეს უნდა დაიწყოს.

Posted by: Cor-toni 20 May 2008, 22:02
vano_t
QUOTE
cor-toni
QUOTE

არ ნიშნავს რომ სწორია ეს:
QUOTE
დღეს უკვე ამას აქტიურად ნიშნავენ, რამდენიმე კიდევ ვიცი ფორუმზე ვინც ამას სვავდა ორსულობის დროს....

ნამდვილად. სწორი კი არა, არასწორია ასე ასპირინის ყველა ავადმყოფში დანიშვნა.

biggrin.gif მე რომ ვამბობ თეთრზე თეთრია-თქო, შენ მპასუხობ, თეთრი კი არა თეთრიაო lol.gif
QUOTE
ისიც მესმის რომ დღევანდელი მიდგომა მართლა მასეა საქართველოში, როგორც შენ აღნიშნე. ავადმყოფი პირდაპირ მიდის სპეციალისტთან უმეტეს შემთხვევაში, თუმცა არასწორია მაგ და უნდა შეიცვალოს მიდგომა  wink.gif  მერე პაციენტსიც კმაყოფილი იქნება და თერაპევტი ექიმიც (რადგანაც გაღრმავებულ ცოდნის შეძენა დაჭირდება და ფართო პროფილის დარჩება). შეიძლება დღეს უტოპიაა, მაგრამ მსგავსი განათლება და მომზადება თერაპევტებისა დღეს უნდა დაიწყოს.

ალბათ მართალი ხარ, თერაპევტო wink.gif


Posted by: vano_t 21 May 2008, 01:34
cor-toni
QUOTE
biggrin.gif  მე რომ ვამბობ თეთრზე თეთრია-თქო, შენ მპასუხობ, თეთრი კი არა თეთრიაო  lol.gif
არა, მე გიპასუხე არამარტო თეთრია მეთქი, არამედ "იმენნა" თეთრია თქო biggrin.gif იტოგში მართალი ხარი და მაგას გავუსვი ხაზი, რომ არამარტო მართალი ხარ, არამედ მართალი ხარ თქო wink.gif

Posted by: JUVILI[PPL] 22 May 2008, 12:20
ჩემს პრობლემას მოგახსენებთ და იქნებ მიშველოთ რამე aba.gif მოკლედ ძალიან ბევრი საჭმელი მწყენს და ეგრევე გულისრევა მემართება, მაგრამ პრობლემა იმაშია რომ ერთიდაიგივე საჭმელი ერთხელ ტუ შემერგება სხვა დროს მწყენს:მაგ: ტომატიანი ან კვერცხი, პირდაპირ რომ გითხრათ დავიგალე ამდენი გულისრევებით, სიგარეტს ვეწევი, თუ ამას რამე მნიშვნელობა აქვს და ღამე გვიან ვიძინებ დაახლოებით 3-4 საათზე, ველი გამოხმაურებას

Posted by: texasuri jleta benzoxerxit 22 May 2008, 14:05
QUOTE (JUVILI[PPL] @ 22 May 2008, 12:20 )
ჩემს პრობლემას მოგახსენებთ და იქნებ მიშველოთ რამე aba.gif მოკლედ ძალიან ბევრი საჭმელი მწყენს და ეგრევე გულისრევა მემართება, მაგრამ პრობლემა იმაშია რომ ერთიდაიგივე საჭმელი ერთხელ ტუ შემერგება სხვა დროს მწყენს:მაგ: ტომატიანი ან კვერცხი, პირდაპირ რომ გითხრათ დავიგალე ამდენი გულისრევებით, სიგარეტს ვეწევი, თუ ამას რამე მნიშვნელობა აქვს და ღამე გვიან ვიძინებ დაახლოებით 3-4 საათზე, ველი გამოხმაურებას

აუუუ,რამხელა თემაა.ენთუზიაზმი არ მყოფნის,მეზარება.
ერთი კითხვა,ბავშვობიდან მასე ხარ?
თუ რაღაც მომენტიდან დაიწტო,ხომ არ უკაშირდება ეს მომენტი დროში რაიმე სტრესს,(ერთჯერად ძლიერს,ან სუსტს მაგრამ გახანგრძლივებულს),რაიმე ინფექციას,ოპერაციულ ჩარევას.
ძალიან ზოგადად დაიწყე ,ასე არ შეიძლება,სქესი,ასაკი,ადრე გადატანილი დაავადებები,ჩივილების აღმოცენების ზუსტი დრო და მისი დინამიკა...ასე უნდა.ჩივილების კავშირი საკვების ხასიათთან როგორც ვხედავ არ გაქვს?შეიძლება რაღაც კარგად "შეგერგოს" და სხვა დროს გულისრევა გამოიწვიოს...ასეა?
გამოკვლევები თუ ჩაგიტარებია?(ეს ინფორმაცია თავად უნდა დადო ხოლმე,ასე ერთი ჩივილით გამოკითხვის დაწყება ვირტუალურ რეჟიმში ცოტა ძნელია.)
და მთელი ეს პერიოდია ხომ არ გაქვს გუნებ-განწყობის დაქვეითება,უმიზეზო ნერვიულობა-შფოთვა,ამ სიმპტომების გაუარესება დილაობით ან პირიქით საღამოს,ერთსა და იმავე დროს?შიში და ნერვიულობა,რომ რაღაც "სერიოზული" გჭირს,ხომ არ აკვირდები და "აყურადებ"შენ სხეულებრივ შეგრძნებაბს?
ინტერესების სფერო?ხომ არ დაქვეითდა?კვლავინდებურად გსიამოვნებს ის სიტუაციები და საქმიანობა,რაც ადრე გაინტერესებდა და გსიამოვნებდა?
და ბოლოს რა მკურნალობა გაქვს ჩატარებული.
ერთი კითხვა-თქო და მთელი ანამნეზი შევკრიბე."მადა ჭამაში მოდისო"

Posted by: JUVILI[PPL] 22 May 2008, 19:26
გეთანხმებით მართლა ზოგადად დავწერე, ვარ 25 წლის,სქესი-მამრობითი, ადრე ბავშვობაში სიმაღლიდან ცჰამოვვარდი ტვინის სჰერყევა მკონდა, მაგრამ იმის მერე დიდი დრო გავიდა და არც გულისრევები მქონია შესაბამისად ბავშვობიდან არ ვუჩივი,ოპერაციულ ჩარევას გამოვრიცხავ, ინფექციისაც რა მოგახსენოთ, მაგრამ დაახლოებით 5 წელია რაც კომპიუტერი მაქვს და დიდ დროს ვატარებდი კომპიუტერთან, ღამე ცუდად მეძინა, რაღაც პრობლემები კი მქონდა პირადულში და ალბათ ნერვიულობის ბრალია, საკმაოდ ნერვიული და აგრესიულიც ვარ, ალბათ სამი წელი იქნება რაც გულისრევები მაწუხებს, მკურნალობა არაფერი ჩამიტარებია და ექიმთანაც რომ მივიდე ვიცი უკვე ბევრი ორგანო მექნება ჩათრეული თირკმელიო ნაღველაო და ასე შემდეგ მაგრამ ალბატ სხვა გზა არ მაქვს... დიდი მადლობა

Posted by: nu_nuka 27 May 2008, 00:09
ნაღვლის ბუშტის ლაპარასკოპია გავიკეთე. ამის შემდეგ ერთი თვე ხდება მარტო მოხარშულ კარტოფილს, საქონლის ხორცს, თევზს , ყველს, მაწონს და ხაჭოს ვჭამ, ორი დღის პურს. სამი დGეა ერთი ვაშლის ჭამა დავიწყე. თან ბავშვი მყავს ბუნებრივ კვებაზე. ქირურგმა მითხრა მარტო ერთი თვე იდიეტეფო მაგრამ მე მაინც მეშინია რამის დამატება. ხომ არ იცით რა შეიძლება ვჭამო და რა არა.

Posted by: vano_t 27 May 2008, 02:43
QUOTE (nu_nuka @ 27 May 2008, 00:09 )
ნაღვლის ბუშტის ლაპარასკოპია გავიკეთე. ამის შემდეგ ერთი თვე ხდება მარტო მოხარშულ კარტოფილს, საქონლის ხორცს, თევზს , ყველს, მაწონს და ხაჭოს ვჭამ, ორი დღის პურს. სამი დGეა ერთი ვაშლის ჭამა დავიწყე. თან ბავშვი მყავს ბუნებრივ კვებაზე. ქირურგმა მითხრა მარტო ერთი თვე იდიეტეფო მაგრამ მე მაინც მეშინია რამის დამატება. ხომ არ იცით რა შეიძლება ვჭამო და რა არა.

ყველანაირი ხილისა და ბოსტნეულის ჭამა შეგიძლია და შეგეძლო თავიდანვე. თავი უნდა აარიდო უბრალოდ ცხიმიან საკვებს. ხორცს (საქონელი თუ ღორი) დასავლეთში არ ურჩევენ. კვერცხს უნდა აარიდო თავი. პური შეგიძლია ჭამო ასევ 1 დღის და 23 საათის smile.gif შენ რასაც ჭამ და რასაც არ ჭამ, ნაღველკენჭოვან დაავადებას უფრო ანვითარებს, ვიდრე ხელს უშლიდეს. რა ვქნა, არ მინდოდა დამეწერა, მარა ხანდახან საქართველოში რაღაც საოცარი მედიცინით სარგებლობენ (ყოველგვარი ახსნის და რაციონალური მარცვალის გარეშე). ხილი და ბოსტნეული ყველაფერს უხდება. ერთი ლობიოს უნდა აარიდო ცოტა თავი.

ეს ჩემი რჩევაა, მთლად ნუ დამიჯერებ, შეიძლება ვცდებოდე.

Posted by: tikope 27 May 2008, 07:17
vano_t
ხომ არ იცით რისი ბრალია თუ ადამიანი დგის განმავლობაში მარტო 1-2 ჭიქა წყლით თუ კმაყოფილდება?

Posted by: nu_nuka 27 May 2008, 09:33
vano_t
დიდი მადლობა. ბოსტნეული ალბათ უფრო მოხარშულ მდგომარეობაში არა?

Posted by: vano_t 28 May 2008, 10:43
tikope
QUOTE
ხომ არ იცით რისი ბრალია თუ ადამიანი დგის განმავლობაში მარტო 1-2 ჭიქა წყლით თუ კმაყოფილდება?
1-2 ჭიქა წყლის მიღებით ადამიანი როგორც წესი დიდხანს ვერ გაძლებს. საქმე იმაშია სითხეს რამდენს იღებ საერთოდ? ამაში შედის ყველანაირი სითხე: წყალი, რზე, წვენები, სუპი და ა.შ.

nu_nuka
QUOTE
ბოსტნეული ალბათ უფრო მოხარშულ მდგომარეობაში არა?
გინდა მოხარშული გინდა უმი. უმი საერთოდ ყოველთვის უკეთესია თუ კუჭნაწლავის სისტემა იტანს (ზოგს ემართება შებერილობები და თავს არიდებს). ყოველ შემთხვევაში უმი (ან მოხარშული) ბოსტნეული ნაღველ-კენჭოვან დაავადებაზე ცუდად არ უნდა მოქმედებდეს.

Posted by: magdalina 30 May 2008, 20:52
ჩემ გოგოს დღეს თითი გადაექერცლა სეტკაზე ეზოში(ისე ქონდა ჩაბღაუჭებული,რომ ძლივს გავაშვებინე),
საშიში ხომ არ არის? ჟანგიანივით იყო ის სეტკა,ხოდა ძაან შევშინდიsad.gif ბავშვი 9 თვისაა,
ძაან გთხოვთ მიპასუხეთ რა ...

Posted by: vano_t 31 May 2008, 00:03
magdalina
QUOTE
ჩემ გოგოს დღეს თითი გადაექერცლა სეტკაზე ეზოში(ისე ქონდა ჩაბღაუჭებული,რომ ძლივს გავაშვებინე),
საშიში ხომ არ არის? ჟანგიანივით იყო ის სეტკა,ხოდა ძაან შევშინდიsad.gif  ბავშვი  9 თვისაა,
ძაან გთხოვთ მიპასუხეთ რა ...

საშიში არ არის ნამდვილად. უბრალოდ საპნიანი წყლით მოურეცხე ეგ ადგილი დღეში ორჯერ და რაიმე ანტიბიოტიკის შემცველი კრემი წაუსვი 2-3 დღე (მაგალითად ნეოსპორინი თუ მაქეთ იშოვება, ან სხვა კრემი). მეტი არაფერი უნდა. ისე დააკვირდი: თუ სიწითლე დაეწყება და გავრცელება დაიწყო, ან ჭრილობასთან კანი ძალიან გაცხელდა, ან ჩირქი წამოუვიდა, ან ძალინ შეუშუპდა ეგ ადგილი, ან სიცხე აუწია, მაშინ ექიმთან მიიყვანე.

P.S. საპონი რბილი უნდა იყოს, რომ არ გაუღიზიანოს კანი (მაგალითად ბავშვთა საპონი ჯონსონის მიერ).

Posted by: nu_nuka 31 May 2008, 07:24
magdalina
vano_t
QUOTE
უბრალოდ საპნიანი წყლით მოურეცხე ეგ

როგორც მე ვიცი სარეცხი საპონის გამოყენება კარგია რადგან ერთგვარი ანტისეპტიკია.

Posted by: MALI 31 May 2008, 19:30
გამარჯობა
მაინტერესებს რა ქვია დაავადებას როდესაც ადამიანს ქრონიკულად ცხვირიდან აქვს გამონადენი, ყოველგავრი ტკივილების გარეშე, თუ ბერი რამის გამი შეიძლება ეს მოხდეს???

მადლობთ

Posted by: Cor-toni 31 May 2008, 19:38
MALI
QUOTE
მაინტერესებს რა ქვია დაავადებას როდესაც ადამიანს ქრონიკულად ცხვირიდან აქვს გამონადენი, ყოველგავრი ტკივილების გარეშე, თუ ბერი რამის გამი შეიძლება ეს მოხდეს???

ალბათ ქრონიკულ სურდოს_ალერგიულ რინიტს გულისხმობ. გამონადენი აღინიშნება მთელი წლის განმავლობაში. ანუ სეზონური არ არის.



Posted by: MALI 31 May 2008, 20:09
quote]cor-toni[/quote]
არა სეზონური არ არის წლების განმავლობაში გრძელდება....
შეიძლება ეს იყოს ინფეციის მატარებელი როგორ უნდა გავიგო?თ?? რაიმე ანალიზია საჭირო?

Posted by: Cor-toni 31 May 2008, 20:17
MALI
ყელ-ყურ-ცხვირის სპეციალისტს_ოტორინოლარინგოლოგს უნდა მიმართო.

Posted by: MALI 31 May 2008, 20:25
cor-toni
კარგით გავითვალისწინებ
მადლობთ smile.gif

Posted by: gt500 1 Jun 2008, 14:37
გამარჯობათ, ერთი პრობლემა მაქ იქნებ დამეხმაროთ დაახლოებით 6 დღის უკან დავლიე და მეორე დილას რო გავიღვიძე ყელი მტკიოდა საშინლად მაგრამ ისე არ მტკიოდა გაციებული რო ხარ და გეწვის გეგონება დაბეჯილი გაქო ნერწყვის გადაყლაპა მიჭირდა. ნასვამზე მახსოვს ნაყინი ვჭამე რამდენიმე და მაგას ვაბრალებდი თან ძგიდე მაქ გამრუდებული და გამე პირი ღია მაქ რო მძინავს. ეს დღეებია დღეში 2-3 გვირილის ნაყენს ვივლებ და ვსვავ და ეხლა შედარებაა არაა თითქმის გამიარა მაგრამ შიგნით უკან ყელში ცოტა კიდე მტკივა ჩამის დროს და ისეც ხანდახან და რისი ბრალი ხომ არ იცით , გლანდების ამბავში მეშინია და რამე საშიში ხო არა, ისე გლანდები საერთოდ არ ვიცი სად მაქ არასდროს არ შეუწუხებია და შეიძლება ესე გამციებოდა ? იქნებ დამეხმაროთ რამე მირჩიოთ. წინასწარ დიდი მადლობა

Posted by: vano_t 1 Jun 2008, 21:34
gt500
QUOTE
გამარჯობათ, ერთი პრობლემა მაქ იქნებ დამეხმაროთ დაახლოებით 6 დღის უკან დავლიე და მეორე დილას რო გავიღვიძე ყელი მტკიოდა საშინლად მაგრამ ისე არ მტკიოდა გაციებული რო ხარ და გეწვის გეგონება დაბეჯილი გაქო ნერწყვის გადაყლაპა მიჭირდა. ნასვამზე მახსოვს ნაყინი ვჭამე რამდენიმე და მაგას ვაბრალებდი თან ძგიდე მაქ გამრუდებული და გამე პირი ღია მაქ რო მძინავს. ეს დღეებია დღეში 2-3 გვირილის ნაყენს ვივლებ და ვსვავ და ეხლა შედარებაა არაა თითქმის გამიარა მაგრამ შიგნით უკან ყელში ცოტა კიდე მტკივა ჩამის დროს და ისეც ხანდახან და რისი ბრალი ხომ არ იცით , გლანდების ამბავში მეშინია და რამე საშიში ხო არა, ისე გლანდები საერთოდ არ ვიცი სად მაქ არასდროს არ შეუწუხებია და შეიძლება ესე გამციებოდა ? იქნებ დამეხმაროთ რამე მირჩიოთ. წინასწარ დიდი მადლობა

თუ მდგომარეობა გამოკეთდა, მაშინ არაფერი გაქვს სანერვიულო. გაივლის მთლიანად, კიდევ დააცადე ერთი ორი დღე. დალევის შემდეგ შეიძლება ყელი გეტკინოს (განსაკუთრებით თუ სიგარეტსაც ბევრს მოწევ, თუმცა არ არის სიგარეტის მოწევა აუცილებელი). ალკოჰოლიც და ნიკოტინიც აღიზიანებს ლორწოვან გარსებს. გარდა ამისა, დალევის დროს ყელის სიმშრალე ადვილად ხდება. ამ ყველაფერს ჭირდება უბრალოდ დასვენება ყელის. მარტივი სავლებიც შეიძლება გამოივლო: მაგალითად მარილიანი ან სოდიანი წყალი. გვირილის ნაყენიც კარგია.

მოკლედ კარგად იქნები.

Posted by: gt500 1 Jun 2008, 22:52
ძალიან დიდი მადლობა დახმარებისთვის !!

Posted by: margusha 4 Jun 2008, 03:44
vano_t

ღამით სახსრები მტკივა ხოლმე ყრუთ, განსაკუთრებით მუხლები და რა არის, ართრიტია ვითომ?
ზოგჯერ მარტო სახსრები კი არა ფეხების ძვლებიც მღრღნის, ეს ბავშვობიდან მომყვება.

Posted by: vano_t 4 Jun 2008, 07:15
margusha
QUOTE
ღამით სახსრები მტკივა ხოლმე ყრუთ, განსაკუთრებით მუხლები და რა არის, ართრიტია ვითომ?
ზოგჯერ მარტო სახსრები კი არა ფეხების ძვლებიც მღრღნის, ეს ბავშვობიდან მომყვება.
არსებობს ართირტების რამოდენიმე ფორმა. ამიტომ ასე ძენლი სათქმელია ართრიტია თუ არა. უფრო მეტი ინფორმაცია ჭირდება მაგ ყველაფერს და გამოკვლევაც. გარდა ამისა, მსგავსი სიმპტომები შეიძლება მოგცეს ე.წ. მოუსვენარი ფეხის სინდრომმა (restless leg syndrome). თუ უფრო მეტ ინფორმაცია მომცემ, უფრო სავარაუდო დიაგნოზს გეტყვი. ქვემოჩამოთვლილი სიმპტომებიდან თუ გაქვს რომელიმე?

1) სისუსტე
2) წონაში დაკლება
3) მადის პრობლემები
4) სიცხეები

5) ტკივილები მარტო ღამით გაქვს (განსაკუთრებით ძილის წინ) თუ დღის განმავლობაშიც?

6) ქვემო კიდურების გარდა სხვა სახსრების პრობლემები თუ გაქვს?

7) რაიმე გამოკვლევა თუ ჩაგიტარებია ადრე, განსაკუთრებით რომელიმე სახსრის რენტგენი, სისხლის საერთო ანალიზი, ერითროციტების დალექვის სიჩქარე და რევმატოიდული ფაქტორი? თუ ჩაგიტარებია რა პასუხები მოგცეს?

8) რაიმე წამალი თუ გშველის?

9) ტკივილის ხასიათი როგორია? "ჭიანჭველების კიდურებში მოძრაობის" შეგრძნება ხომ არ გაქვს? და კიდურის მოძრაობა თუ ამცირებს სიმპტომებს?

Posted by: margusha 4 Jun 2008, 10:52
vano_t

1 არანაირი სისუსტე
2 თავისით რომ იკლებდეს რაღა მიჭირსbiggrin.gif
3 გაქანებული ბულემია მაქვს
4 სიცხეები არ მაქვს
5 დღისითაც
6 თუ სახსრები ამტკივდა ყველა სახსარი მტკივა, მაგრამ უფრო გამოკვეთილად მუხლები
7 არ ჩამიტარებია, სისხლის საერთო ორი წლის წინ ორსულობისას ვიღებდი ხოლმე და წესრიგში იყო ყველაფერი
8 წამალი არ დამილევია სახსრების გამო, არ მაქვს ისეთი ტკივილები გაყუჩება რომ ჭირდებოდეს
9 არა ჭიანჭველები არ მაწუხებენ, აი როგორ გითხრა ყრუ ტკივილია, უფრო სწორედ ყველა სახსარს ვგრძნობ რომ მაქვსbiggrin.gif
იმოძრავე არ იმოძრავე ერთი ლხინია. მოხუცი ქალივით თუ დავჯექი და მუხლები დავიზილე ერთი 20 წამი ვითომ მიყუჩდება. თითების სახსრები რომ მტკივა, თითქოს ძალა მეცლება ხელებიდან, რაღაც მსუბუქი გაბუჟების შეგრძნება მაქვს. დაზელვა საერთოდ ძალიან მსიამოვნებს სახსრების ტკივილის დროს.




Posted by: vano_t 4 Jun 2008, 11:20
margusha
QUOTE
1 არანაირი სისუსტე
2 თავისით რომ იკლებდეს რაღა მიჭირსbiggrin.gif
3 გაქანებული ბულემია მაქვს
4 სიცხეები არ მაქვს
5 დღისითაც
6 თუ სახსრები ამტკივდა ყველა სახსარი მტკივა, მაგრამ უფრო გამოკვეთილად მუხლები
7 არ ჩამიტარებია, სისხლის საერთო ორი წლის წინ ორსულობისას ვიღებდი ხოლმე და წესრიგში იყო ყველაფერი
8 წამალი არ დამილევია სახსრების გამო, არ მაქვს ისეთი ტკივილები გაყუჩება რომ ჭირდებოდეს
9 არა ჭიანჭველები არ მაწუხებენ, აი როგორ გითხრა ყრუ ტკივილია,  უფრო სწორედ ყველა სახსარს ვგრძნობ რომ მაქვსbiggrin.gif
იმოძრავე არ იმოძრავე ერთი ლხინია. მოხუცი ქალივით თუ დავჯექი და მუხლები დავიზილე ერთი 20 წამი ვითომ მიყუჩდება. თითების სახსრები რომ მტკივა, თითქოს ძალა მეცლება ხელებიდან, რაღაც მსუბუქი გაბუჟების შეგრძნება მაქვს. დაზელვა საერთოდ ძალიან მსიამოვნებს სახსრების ტკივილის დროს.

ე.ი. მუხლის სახსრის რენტგენი, სისხლის საერთო ანალიზი, ერითროციტების დალექვის სიჩქარე, ანტი-ბირვული ანტისხეულები და რევმატოიდული ფაქტორი რომ გააკეთებინო, მაშინ შეიძლება დიაგნოზთან ცოტა ახლოს მისვლა. თუ კარგ რევმატოლოგს იცნობ, მაშინ რევმატოლოგი ნახე.

ჩამოთვლილი ტესტებიდან თუ ყველაფერი ნორმაში იქნა, მაშინ ანთებითი ართრიტი ნაკლებ სავარაუდოა. შეიძლება იყოს ფიბრომიალგია-ეს ერთერთი ახალი ნოზოლოგიაა. მოკლედ რომ ვთქვათ, ნიშნავს "არ ვიცით რა არის."

რაც შეეხება ტკივილს, ანთების საწინააღმდეგო საშუალებები შეიძლება დალიო (სანამ დიაგნოზს დაგიდგენენ) თუ რაიმე უკუჩვენება არ გაქვს. ამ წამლებში შედის იბუპროფენი მაგალითად.

Posted by: margusha 4 Jun 2008, 11:44
vano_t

დიდი მადლობაsmile.gif

მაგრამ მაგ გამოკვლევებს ალბათ არმაგედონის წინ თუ ჩავიტარებ, ეგ კი არა თერმომეტრის გამოღება მეზარება ხოლმე რომ ვნახო სიცხე რამდენი მაქვს. : D

Posted by: texasuri jleta benzoxerxit 4 Jun 2008, 12:03
QUOTE (vano_t @ 4 Jun 2008, 11:20 )
margusha
QUOTE
1 არანაირი სისუსტე
2 თავისით რომ იკლებდეს რაღა მიჭირსbiggrin.gif
3 გაქანებული ბულემია მაქვს
4 სიცხეები არ მაქვს
5 დღისითაც
6 თუ სახსრები ამტკივდა ყველა სახსარი მტკივა, მაგრამ უფრო გამოკვეთილად მუხლები
7 არ ჩამიტარებია, სისხლის საერთო ორი წლის წინ ორსულობისას ვიღებდი ხოლმე და წესრიგში იყო ყველაფერი
8 წამალი არ დამილევია სახსრების გამო, არ მაქვს ისეთი ტკივილები გაყუჩება რომ ჭირდებოდეს
9 არა ჭიანჭველები არ მაწუხებენ, აი როგორ გითხრა ყრუ ტკივილია,  უფრო სწორედ ყველა სახსარს ვგრძნობ რომ მაქვსbiggrin.gif
იმოძრავე არ იმოძრავე ერთი ლხინია. მოხუცი ქალივით თუ დავჯექი და მუხლები დავიზილე ერთი 20 წამი ვითომ მიყუჩდება. თითების სახსრები რომ მტკივა, თითქოს ძალა მეცლება ხელებიდან, რაღაც მსუბუქი გაბუჟების შეგრძნება მაქვს. დაზელვა საერთოდ ძალიან მსიამოვნებს სახსრების ტკივილის დროს.

ე.ი. მუხლის სახსრის რენტგენი, სისხლის საერთო ანალიზი, ერითროციტების დალექვის სიჩქარე, ანტი-ბირვული ანტისხეულები და რევმატოიდული ფაქტორი რომ გააკეთებინო, მაშინ შეიძლება დიაგნოზთან ცოტა ახლოს მისვლა. თუ კარგ რევმატოლოგს იცნობ, მაშინ რევმატოლოგი ნახე.

ჩამოთვლილი ტესტებიდან თუ ყველაფერი ნორმაში იქნა, მაშინ ანთებითი ართრიტი ნაკლებ სავარაუდოა. შეიძლება იყოს ფიბრომიალგია-ეს ერთერთი ახალი ნოზოლოგიაა. მოკლედ რომ ვთქვათ, ნიშნავს "არ ვიცით რა არის."

რაც შეეხება ტკივილს, ანთების საწინააღმდეგო საშუალებები შეიძლება დალიო (სანამ დიაგნოზს დაგიდგენენ) თუ რაიმე უკუჩვენება არ გაქვს. ამ წამლებში შედის იბუპროფენი მაგალითად.

არადა უნდა ჩაიტარო.ს სუსტავამი შუტკი პლოხი.მინიმუმი დაგიწერა ვანომ.ერთს დავამატებდი:ოტო-რინო-ლარინგოლოგის კონსულტაცია,თუ გლანდები ადგილზე გაქვს რა თქმა უნდა.
ისე,როგორი შთაბეჭდილებაც შემექმნა შენზე ვირტუალურად,არ არის გამორიცხული,ეგ სიმპტომები ფუნქციური,ფსიქოგენური იყო(სომატოფორმული შლილობის ან მასკირებული დეპრესსიის სახით).ყრუ ხასიათი,დიფუზურობა ჩივილების,დღე და ღამე ერთნაირად+ დაბუჯების გრძნობა უფრო აქეთკენ მიუთითებს,მაგრამ ორგანული დაავადება უნდა გამიურუცხოს ჯერ

Posted by: margusha 4 Jun 2008, 13:11
texasuri jleta benzoxerxit

როგორ გგონია ჩემი ხალისიანი ბავშვობის გადამკიდე გლანდები ადგილზე მექნება? biggrin.gif გლანდებიც ამოჭრილი მაქვს, პოლიპებიც და ადენოიდებიც და ჭია (ბრმა) ნაწლავიც.
სომატომორფული აშლილობა არა მგონია მქონდეს, ადრე ვდგები ადრე ვიძინებ, გვიან ვდგები გვიან ვიძინებ, უძილობის პრობლემა არ მაქვს, უბრალოდ მე თვითონ ვირღვევ ხოლმე რეჟიმს. ის ამიტრიპტილინზე რომ გითხარი, რის გამო ვსვავდი ისიც ხომ გითხარი biggrin.gif არც დეპრესიული ვარ, ყოველშემთხვევაში ჯერ არაფერს ჩავუგდივარ დეპრესიაში. უბრალოდ ემოციური ვარ და აფექტური. ეს სახსრებიც კიდე უფრო მაშინ მტკივდება თუ ფეხზე შემცივდება, სულ ქრონიკულად არ მტკივა

Posted by: lgogokhia 5 Jun 2008, 04:08
ტონზილექტომია გავიკეთე 2 წლის წინ. სადაზღვევო კომპანია მეკითხება მქონდა თუ არა რაიმე ქირურგიული ჩარევა ბოლო 10 წლის მანძილზე. მე ვპასუხობ უარს, მაგრამ მაგზავნიან საკონტროლო კვლევებზე. შესაძლებელია თუ არა რაიმე მეთოდებით დადგინდეს ტონზილექტომია 2 წლის წინ გავიკეთე თუ 10-ის? უბრალოდ სადაზღვევო პრემია დიდი სხვაობით მატულობს თუ ტონზილექტომიაზე ჩავწერე... smile.gif)
* * *
პ.ს. უბრალოდ ძალიან პარანოიდულად ვარ განწყობილი მსგავს ტყუილებზე, სიტუაციის გამო...

Posted by: vano_t 5 Jun 2008, 04:27
lgogokhia
QUOTE
შესაძლებელია თუ არა რაიმე მეთოდებით დადგინდეს ტონზილექტომია 2 წლის წინ გავიკეთე თუ 10-ის?
გამოკვლევის მეთოდებით არა. კარტოთეკაში თუ ცაიხედებიან. დოკუმენტებით შეიძლება მხოლოდ მაგის დადგენა და კიდევ ავადმყოფის გადმოცემით. ავადმყოფს კიდევ არ აწყობს smile.gif ე.ი. დარჩატ მხოლოდ საბუთების გადამოწმება. იმასაც თუ გავითვალისწინებთ, რომ თუ ქირურგმა უფრო ნაკლები აიღო და ქირურგია არ გააფორმა, მაშინ დოკუმენტიც შეიძლება არ არსებობდეს smile.gif

Posted by: lgogokhia 5 Jun 2008, 06:59
ოპერაცია საქართველოში გავიკეთე, ეს ამბები კი მის ფარგლებს გარეთ ხდება, ამიტომ მაგის შანსი არაა smile.gif მადლობა...

Posted by: kartveli gogo 10 Jun 2008, 20:59
ძალიან გთხოვთ ამიხსნათ... 0,25 დიგოქსინის ნახევარს იგებდა ავადმყოფი,ეხლა გვაქვს 0,00025იანი აბები რა დოზა უნდა მივცე?მკურნალ ექიმს ვერ ვუკავშირდები.........

Posted by: vano_t 10 Jun 2008, 21:10
kartveli gogo
QUOTE
ძალიან გთხოვთ ამიხსნათ... 0,25 დიგოქსინის ნახევარს იგებდა ავადმყოფი,ეხლა გვაქვს 0,00025იანი აბები რა დოზა უნდა მივცე?მკურნალ ექიმს ვერ ვუკავშირდები.........

არ არსებობს 0,00025 დოზა. დიგოქსინი ორნაირად შეიძლება გამოხატო: ან მიკორგრამებში, მაგ 250 მკგ, ან მილიგრამებში რაც იქნება 0,25 მგ (250 მიკროგრამის შესაბამისი). 0,00025 რომ მიიღო, მაშინ გრამებში უნდა გამოხატო დიგოქსინი. გრამებში გამოხატული არ მინახია. ამიტომ, თუ მეტყვი რა სიდიდე უწერია 0,00025-ს (გრამი, მილიგრამი თუ რაც არის), მაშინ გეტყვი რა დოზა უნდა მიიღოს.

თუ გრამებშია 0,00025 გამოხატული და 0,25 იყო მილიგრამებში გამოხატული, მაშინ ეგ ორი დოზა ერთმანეთის ტოლფასია.

Posted by: kartveli gogo 10 Jun 2008, 21:14
vano_t
QUOTE
თუ გრამებშია 0,00025 გამოხატული და 0,25 იყო მილიგრამებში გამოხატული, მაშინ ეგ ორი დოზა ერთმანეთის ტოლფასია.


დიდ მადლობა форма выпуска 0,25 (0,00025). ეხლა აღმოვაჩინე თქვენი მითითებით. 2kiss.gif

Posted by: margusha 13 Jun 2008, 23:14
vano_t
ამაზე შეგიძლია რომ კომენტარი გააკეთო?
http://forum.ge/?showtopic=33735606&view=findpost&p=10034933

Posted by: ivereanu 14 Jun 2008, 00:57
ე.ი. ასეთი კითხვა მაქვს biggrin.gif

რომ დავიდვარ განუწყვეტლივ მიტკაცუნებს ფეხი... ეს ასე ერთი წლის წინ შევნიშნე... ახლა უფრო იმატა და სადღაც ყოველ ნაბიჯის გადადგამზე თუ არა სამის გადადგამაზე მიანც ტკაცუნობს..სადღაც კოჭთან... დისკომფორტს ასე თუ ისე მიქმნის.. რისი ბრალი შეიძლება რომ იყოს და არის რამე საშველი? biggrin.gif

Posted by: vano_t 14 Jun 2008, 05:22
ivereanu
QUOTE
ე.ი. ასეთი კითხვა მაქვს biggrin.gif

რომ დავიდვარ განუწყვეტლივ მიტკაცუნებს ფეხი... ეს ასე ერთი წლის წინ შევნიშნე... ახლა უფრო იმატა და სადღაც ყოველ ნაბიჯის გადადგამზე თუ არა სამის გადადგამაზე მიანც ტკაცუნობს..სადღაც კოჭთან... დისკომფორტს ასე თუ ისე მიქმნის.. რისი ბრალი შეიძლება რომ იყოს და არის რამე საშველი? biggrin.gif
"ტკაცუნი" თავისთავად არაფერია. ყველაზე კარგი იქნება თუ კარგ ორთოპედთან მიხვალ და დაგაკვალიანებს.

margusha
QUOTE
ამაზე შეგიძლია რომ კომენტარი გააკეთო?
http://forum.ge/?showtopic=33735606&view=findpost&p=10034933

გილი შემიწუხდა იმხელა პოსტია biggrin.gif კონკრეტულად რა გაინტერესებს მაქედან?

Posted by: margusha 14 Jun 2008, 10:53
vano_t

სისხლის გადასხმა არ გადასხმის "საინტერესო" თეორია აქვსbiggrin.gif

Posted by: mainc 14 Jun 2008, 11:01
ჰოდა
აი რა მაინტერესებს
დონორთა დღეს
დამაინტერესა
ბოტკინგადატანილი (თუნდაც ბავშვობაში) ადამიანის სისხლი გამოუსადეგარია
?

eek.gif


რომ ჩააბაროს აზრი არ აქვს
?

მმმ


Posted by: miqstura 14 Jun 2008, 15:07
mainc
გააჩნია, ბოტკინს რას ეძახი. თუ გადატანილი A ჰეპატიტია, არაა პრობლემა, B ჰეპატიტიანის კი (თუნდა გადატანილი და სრულიად მორჩენილი) სისხლის გადასხმა არ შეიძლება

Posted by: nu_nuka 14 Jun 2008, 18:56
ნანემსარის გამაგრების შემთხვევაში რა შეიძლება გავაკეთო?

Posted by: shtori 14 Jun 2008, 20:04
მაინტერესებს კალციუმის ნაკლებობა რას იწვევს და როგორ უნდა ვიმკურნალო? რა პრეპარატებიტ? კვერცხის ნაჭუჭიაო კარგიო და რომ მივიღო მიშველის? და კიდევ კალციუმის დიდ შემცველობა რამე უარყოფიტს ხომ არ გამოიწვევს? ნაკლებობა ვიცი ძველებს შლის და 1000 უბედურებაა და მეტობა?

Posted by: vano_t 15 Jun 2008, 09:53
nu_nuka
QUOTE
ნანემსარის გამაგრების შემთხვევაში რა შეიძლება გავაკეთო?
თუ არ გტკივა, არაფერი უნდა. თავისით გაიწოვება ნელა.

shtori
QUOTE
მაინტერესებს კალციუმის ნაკლებობა რას იწვევს და როგორ უნდა ვიმკურნალო? რა პრეპარატებიტ? კვერცხის ნაჭუჭიაო კარგიო და რომ მივიღო მიშველის? და კიდევ კალციუმის დიდ შემცველობა რამე უარყოფიტს ხომ არ გამოიწვევს? ნაკლებობა ვიცი ძველებს შლის და 1000 უბედურებაა და მეტობა?
კალციუმი ძვლებისათვის არის საჭირო. ამიტომ კალციუმის ნაკლებობამ შეიძლება მოგცეს ძვლის განლევა (რასაც ოსტეოპოროზი ქვია). კიდევ კალციუმი აუცილებელია ნერვების ნორმალური მუშაობისათვის. ამიტომ კალციუმის ნაკლებობამ კუნთების კრუნჩხოვანი შეკუმშვები შეიძლება მოგცეს. კალციუმი რძეში და რძის პროდუქტებშია ბევრი. კვერცხის ნაჭუჭში ალბათ არის კალციუმი, მარა არ მგონია რაღაც საკვებ პროდუქტს რომ წარმოადგენდეს. თანაც, მაგ კალციუმის შეწოვა როგორ უნდა მოხდეს კუჭში, არ ვიცი. მე რძეს, ხაჭოსს, ყველს და ბოსტნეულს გირჩევ. ბოსტნეულიდან ისპანახი შეიცავს ბევრ კალციუმს.

კალციუმის მეტობამაც ბევრი რამე შეიძლება გამოიწვიოს. ბევრი კალციუმი შეიძლება სხვადასხვა ქსოვილებში ჩალაგდეს მარილების სახით და მათი დაზიანება შეიძლება გამოიწვიოს. მაგალითად თირკმელში კენჭები შეიძლება განვითარდეს. შეიძლება ასევე მოგცეს სისუსტე, შარდივის გახშირება და ე.წ. უშაქრო დიაბეიტი, გულისრევა, ღებინება. კიდევ გააჩნია რამდენად მაღალია კალციუმი ორგანიზმში.

Posted by: shtori 15 Jun 2008, 14:30
vano_t
მადლობა დამხმარებისთვის smile.gif
და შესაძებელია ორგანიზმში კალციუმის შემცველობის გაკონტროლება? გამოკვლევების გარეშე?

Posted by: iFFii 15 Jun 2008, 17:00
ამბოლოდროს
ვენები მებერება ჰელებზე
და საშინელ სიმძიმეს ვგრძნობ..
და შესახედავადაც საშინელებაა..
: s

ძაან დავსუსტდი და მაგი ბრალიაო..
დარვიცი..

Posted by: vano_t 16 Jun 2008, 09:07
shtori
QUOTE
და შესაძებელია ორგანიზმში კალციუმის შემცველობის გაკონტროლება? გამოკვლევების გარეშე?
კითხვა მთალდ ვერ გავიგე. შევეცდები პასუხი გაგცე. კალციუმის გაკონტროლებაში რას გულისხმობ? ჩვეულებრივ, როგორც წესი, თუ კალციუმს შეამოწმებ სისხლში, ნახავ მომატებულ კალციუმს და ამას დიაგნოზი ჭირდება, ანუ მიზეზის დადგენა. თუმცა, თუ კალციუმის შემცველობა ნორმალურია სისხლში, ეს არ ნიშნავს, რომ ორგანიზმი საკმაო რაოდენობით კალციუმს შეიცავს. ხანშიშესულ ქალებს ხშირად ემართებათ ოსტეოპოროზი, მიუხედავად იმისა, რომ სისხლში კალციუმი ნორმალურია. აქ მთავარი ის არის, რომ ძვლიდან ხდება დიდი რაოდენობით კალციუმის გაწოვა და სისხლში შესაბამისად ნორმაშია კალციუმი.

კალციუმი შეიძლება გაიზომოს მარტივი ტესტით.

shtori
QUOTE
ამბოლოდროს
ვენები მებერება ჰელებზე
და საშინელ სიმძიმეს ვგრძნობ..
და შესახედავადაც საშინელებაა..
: s

ძაან დავსუსტდი და მაგი ბრალიაო..
დარვიცი..
ვენების დაბერვა სისუტეს არ მოგცემს. შეიძლება ერთიდაიგივე რამ იწვევდეს პრობლემას, მაგრამ არა ვენების დაბერვა-სისუსტეს.

ვენების დაბერვა თავისთავად არაფერია. უფრო სწორად გასინჯვით და დანახვით უნდა ანახოს ექიმმა, რომ გაიგოს რაზეა ლაპარაკი. სხვანაირად ძნელია იმის გარკვევა, რასთან გვაქვს საქმე. წონაში დაკლებას კი, გამოკვლევა უნდა. ეგ იმნდენად არასპეციფიური სიმპტომია, რომ ასობით დაავადებებმა შეიძლება მოგცეს ეგ. ამიტომ დეტალური გამოკითხვა და გამოკვლევაა ექმიმის მიერ საჭირო.

Posted by: nu_nuka 17 Jun 2008, 14:46
vano_t
QUOTE
თუ არ გტკივა, არაფერი უნდა. თავისით გაიწოვება ნელა.

ხელის დადების შემთხვევაში მტკივა თან თვენახევრის წინ გამიკეთეს

Posted by: vano_t 17 Jun 2008, 20:02
nu_nuka
QUOTE
ხელის დადების შემთხვევაში მტკივა თან თვენახევრის წინ გამიკეთეს

მაშინ ექიმი უნდა ნახო, რომ აბსცესი გამორიცხოს. ჩვეულებრივ, ნემსის გაკეთების შემდეგ ტკივილი მაგდენ ხანს არ გრძელდება და თუ შემაგრებულია ეგ ადგილი, ჩირქგროვა უნდა გამოირიცხოს. ამიტომ, მე გირჩევდი ექიმთან მისვლას.

Posted by: nu_nuka 17 Jun 2008, 22:37
vano_t
ზუსტად მაგისი მეშინია

Posted by: texasuri jleta benzoxerxit 18 Jun 2008, 02:45
QUOTE (nu_nuka @ 17 Jun 2008, 22:37 )
vano_t
ზუსტად მაგისი მეშინია

მაგის უმკურნალებლობის გეშინოდეს.ოღონდ მართლა.თან ძალიან

Posted by: nu_nuka 18 Jun 2008, 11:44
სავანდყოფოს უკვე ვეღარ ვიტან წარმოიდგინე მარტის ბავშვი გავაჩინე, ნაღვლის ბუშტი ამოვიჭერი, ქმარს სამსახურში რელსიან კარებში ფეხი მოყვა და თითები ჩაემსხვრა ჯერ კიდევ გიფსში აქვს, სამი დღის წინ პატარას საცრემლე სადინრები გაუწმინდეს და ეხლა კიდე ეს weep.gif weep.gif weep.gif

Posted by: annak 18 Jun 2008, 11:45
nu_nuka 2kiss.gif
რაც მალე მიხვალ ექიმთან მით უკეთესი yes.gif თორემ ტირილი მერე ნახე. yes.gif

Posted by: Avoie 18 Jun 2008, 12:43
მოკლედ ამ კვირის პარასკევს და შაბათს ვერ გამოვცხადდები სამსახურში, იმიტომ რომ სხვა სამსახურში ვაპირებ გადასვლას და იქ უნდა მიიღონ გადაწყვეტილება ჩემს აყვანასთან დაკავშირებით.

აქ ვერ ვთხოვ ოფროსებს გამანთავისუფლეთთქო ისედაც ბევრს ვბოდიალობ ცემს ჭკუაზე და მოკლედ პარასკევ დილით რომ დავრეკო და ვთქვა სადარბაზოს კიბეებზე დავეცი და ფეზო კოჭში ვიღრძეთქო რამდენად დამაჯერებელი იკნება, მერე ორსაბათს კოჭლობით ხომ აღარ მომიწევს სიარული?

აბა რჩევებიიიიიიიიიიიიიი პლზზზზზზზზზზზზ

help.gif

Posted by: texasuri jleta benzoxerxit 18 Jun 2008, 13:08
არ იქნება დამაჯერებელი."ტყუილს მოკლე ფეხები აქვსო..."
ჯობია კვებითი ტოქსიკოინფექცია მოიმიზეზო.

Posted by: @Ani@ 24 Jun 2008, 09:42
არ ვიცი სად უნდა ვიკითხო და ანემია რას ნიშნავს ზუსტად ხომ ვერ მეტყვიით? sad.gif

Posted by: Cor-toni 24 Jun 2008, 12:11
@Ani@
QUOTE
ანემია რას ნიშნავს ზუსტად ხომ ვერ მეტყვიით?
სისხლნაკლებობას. ზუსტად რა გაინტერესებს?

Posted by: cremli-ani 24 Jun 2008, 16:04
QUOTE
@Ani@

QUOTE
ანემია რას ნიშნავს ზუსტად ხომ ვერ მეტყვიით?

სისხლნაკლებობას. ზუსტად რა გაინტერესებს?


გემოგლობინის ნაკლებობას ხომ?

Posted by: texasuri jleta benzoxerxit 24 Jun 2008, 17:20
QUOTE (cremli-ani @ 24 Jun 2008, 16:04 )
QUOTE
@Ani@

QUOTE
ანემია რას ნიშნავს ზუსტად ხომ ვერ მეტყვიით?

სისხლნაკლებობას. ზუსტად რა გაინტერესებს?


გემოგლობინის ნაკლებობას ხომ?

ჰემოგლობინის,ერითროციტების ან ორივეს ერთად ნაკლებობა

Posted by: ტანკე 25 Jun 2008, 22:05
გაუმარჯოს ხალხს
ერთი კითხვა მაქვს - MRI ან MRT-თი მთლიანად ორგანიზმის სკანირება ხდება,თუ მარტო იმ ორგანოსი,რომელსაც მიუთითებ? ანუ თუ მთელი ორგანიზმის სკანირება გინდა,ეს უფრო ბევრი ჯდება? თუ თავზე იკეთებ MRT-ს,მაშინ მარტო თავს ასკანირებენ? და თუ კონკრეტულად თავის ტვინის სისხლძარღვებზე ხდება დაკვირვება(პაციენტის ექიმთან მისვლის მიზეზი),იქ თავის ტვინის ყველა ანომალიასაც გამოავლენენ,თუ ასეთი რამ არსებობს? მაგალითად,სიმსივნე,ალცჰეიმერი და სხვ.
წინასწარ დიდი მადლობა

Posted by: vano_t 25 Jun 2008, 22:09
QUOTE (ტანკე @ 25 Jun 2008, 22:05 )
გაუმარჯოს ხალხს
ერთი კითხვა მაქვს - MRI ან MRT-თი მთლიანად ორგანიზმის სკანირება ხდება,თუ მარტო იმ ორგანოსი,რომელსაც მიუთითებ? ანუ თუ მთელი ორგანიზმის სკანირება გინდა,ეს უფრო ბევრი ჯდება? თუ თავზე იკეთებ MRT-ს,მაშინ მარტო თავს ასკანირებენ? და თუ კონკრეტულად თავის ტვინის სისხლძარღვებზე ხდება დაკვირვება(პაციენტის ექიმთან მისვლის მიზეზი),იქ თავის ტვინის ყველა ანომალიასაც გამოავლენენ,თუ ასეთი რამ არსებობს? მაგალითად,სიმსივნე,ალცჰეიმერი და სხვ.
წინასწარ დიდი მადლობა

მარტო სხეულის ნაწილების. ტექნიკურად შესაძლებელია მთელი სხეულის სკანირება, მაგრამ როცა რაიმე პათოლოგიას ეძებ, ასევე სკანირების გარკვეულ მეთოდებს იყენებ და ამას (მაგალითად T2-weighted). თანაც უფრო ბევრი დაჯდება გაცილებით.

Posted by: ტანკე 25 Jun 2008, 22:12
vano_t
გაიხარე და ეს?
QUOTE
და თუ კონკრეტულად თავის ტვინის სისხლძარღვებზე ხდება დაკვირვება(პაციენტის ექიმთან მისვლის მიზეზი),იქ თავის ტვინის ყველა ანომალიასაც გამოავლენენ,თუ ასეთი რამ არსებობს? მაგალითად,სიმსივნე,ალცჰეიმერი და სხვ.


Posted by: Cor-toni 25 Jun 2008, 22:23
ტანკე
QUOTE
თუ თავზე იკეთებ MRT-ს,მაშინ მარტო თავს ასკანირებენ? და თუ კონკრეტულად თავის ტვინის სისხლძარღვებზე ხდება დაკვირვება(პაციენტის ექიმთან მისვლის მიზეზი),იქ თავის ტვინის ყველა ანომალიასაც გამოავლენენ,თუ ასეთი რამ არსებობს? მაგალითად,სიმსივნე,ალცჰეიმერი და სხვ.

კი, მხოლოდ თავს ასკანირებენ და ამ დროს არა მარტო სისხლძარღვების მდგომარეობა დგინდება_სიმსივნეც და ალცჰეიმერიც (ქერქის მდგომარეობის მიხედვით).



როგორ ხარ?




Posted by: ტანკე 25 Jun 2008, 22:28
cor-toni
მე ანევრიზმაზე გავესინჯე და MRT გავიკეთე.არ აღმომაჩნდა.ჰიპოფიზის თანდაყოლილი დეფექტი(განვითარების დეფექტი) მითხრეს და ეგ არაფერიაო.ხოდა თუ მარტო სისხლძარღვებს ამოწმებდნენ,მაშინ მაგას არ მეტყოდნენ,არა?

ანუ,თუ ექიმს ვეტყვი,რომ თავის ტვინის სისხლძარღვების გამოკვლევა მინდა და MRt გამიკეთე-მეთქი,მაშინ მთლიანად თავს დაასკანირებენ და არა მარტო სისხლძარღვოვან ანომალიებს,არამედ თავის ტვინის ყველანაირ პრობლემასაც გამოავლენენ?

QUOTE
როგორ ხარ?

გმადლობთ,კარგად smile.gif შეძლებისდაგვარად smile.gif ვცდილობ

Posted by: Cor-toni 25 Jun 2008, 22:32
ტანკე
QUOTE
მე ანევრიზმაზე გავესინჯე და MRT გავიკეთე.არ აღმომაჩნდა.

ვიცი. მახსოვს smile.gif მივხვდი, რომ ამასთან დაკავშირებით იკითხე smile.gif
QUOTE
ანუ,თუ ექიმს ვეტყვი,რომ თავის ტვინის სისხლძარღვების გამოკვლევა მინდა და MRt გამიკეთე-მეთქი,მაშინ მთლიანად თავს დაასკანირებენ და არა მარტო სისხლძარღვოვან ანომალიებს,არამედ თავის ტვინის ყველანაირ პრობლემასაც გამოავლენენ?

კი, რათქმაუნდა.

Posted by: ტანკე 25 Jun 2008, 22:40
cor-toni
ხო,ჩავხედე ახლა ჩემი თავის ტვინის სურათებს და ყველაფერი ჩანს smile.gif
უბრალოდ,მე ვიფიქრე,რომ რახან სისხლძარღვების გამოკვლევა მინდოდა,მარტო სისხლძარღვებს გამოიკვლევდნენ და მთლიანად ტვინის გამოკვლევა უფრო ძვირი დაჯდებოდა smile.gif თუმცა,სურათებს ჩავხედე და ... ყველაფერი უნდა ჩანდეს


ესე იგი,თუ რამეა,შევეშვა თავის ტვინს,ხო? იქ არაფერი არაა ისეთი?

პ.ს. თირკმელზედა ჯირკვლის ჰიპერფუნქცია მაქვს(3-ჯერ მომატებული და მაგიტომ მაქვს ჰიპერტენზია sad.gif )
მერე მაგასაც მივხედავ.მთავარია, თავის ტვინი წესრიგშია biggrin.gif

Posted by: Cor-toni 25 Jun 2008, 22:48
ტანკე
QUOTE
ესე იგი,თუ რამეა,შევეშვა თავის ტვინს,ხო? იქ არაფერი არაა ისეთი?

პ.ს. თირკმელზედა ჯირკვლის ჰიპერფუნქცია მაქვს(3-ჯერ მომატებული და მაგიტომ მაქვს ჰიპერტენზია  )
მერე მაგასაც მივხედავ.მთავარია, თავის ტვინი წესრიგშია

თავის ტვინზე რაიმე სერიოზული რომ ყოფილიყო, გეტყოდნენ მაშინ.

თირკმელზედა ჯირკვლის ჰიპერფუნქცია და ჰიპერტენზია აუცილებლად მისახედია და "მერე"-სთვის არ გადადო, ენდოკრინოლოგს მიმართე yes.gif
ამასაც შეიძლება ჰქონდეს მნიშვნელობა:
QUOTE
ჰიპოფიზის თანდაყოლილი დეფექტი(განვითარების დეფექტი)



Posted by: vano_t 26 Jun 2008, 00:02
ტანკე
QUOTE
და თუ კონკრეტულად თავის ტვინის სისხლძარღვებზე ხდება დაკვირვება(პაციენტის ექიმთან მისვლის მიზეზი),იქ თავის ტვინის ყველა ანომალიასაც გამოავლენენ,თუ ასეთი რამ არსებობს? მაგალითად,სიმსივნე,ალცჰეიმერი და სხვ.
მაგნიტურ რეზონანსური ანგიგოგრაფია სპეციალური ტესტია, რომელიც მარტო სისხლძარღვებს უყურებს და ტვინის ქსოვილი არ ჩანს ზედ კარგად. ამას სპეციალური ტექნიკით ახდენენ. ამიტომ, თუ MRA (ანუ magnetic resonance angiography) გაგიკეთეს, ამაზე ვერ დაინახავენ ტვინის ქსოვილს. ტვინის ქსოვილის სკანირებისათვის "ჩვეულებრივი" მაგნეტური რეზონანსი ჭირდება.

http://www.nazarethimaging.com/images/mra_brain_axial.jpg მოცემულია MRA. http://path.upmc.edu/cases/case59/images/gross1.jpg კიდევ მოცემულია ტვინის MRI. აქ ნათლად ჩანს განსხვვავებები. მოკლედ, ანგიოგრამაზე ტვინის ქსოვილის სუბსტრაქცია ხდება სპეციალური მეთოდების გამოყენებით.

რაც შეეხება ალცჰეიმერს, ეს არცერთ გამოსახულებით გამოკვლევებზე (MRI, CT, SPECT თუ სხვა) არ ჩანს. ალცჰეიმერი არის კლინიკური დიაგნოზი.

MRI შეიძლება დაგეხმაროს მაგალითად იმაში, რომ მრავლობითი ინფარქტები დემენციის მიზეზად (multi-infarct dementia) ჩათვალო, თუმცა აქაც გარკვეული კლინიკური სურათი უნდა გქონდეს.

Posted by: ტანკე 26 Jun 2008, 00:36
vano_t
mrt გავიეკთე smile.gif)))
QUOTE
რაც შეეხება ალცჰეიმერს, ეს არცერთ გამოსახულებით გამოკვლევებზე (MRI, CT, SPECT თუ სხვა) არ ჩანს

ნამდვილად? მეორე ფაზა,მგონი,ჩანს.ტვინის ატროფია ხდება...



Posted by: texasuri jleta benzoxerxit 26 Jun 2008, 01:18
ტანკე
ამ კაცს აქვს იპოქონდრიულ-სენესტოპათიური სინდრომი(წინა პორსტებიც გამახსენდა),რომლის უკანაც მასკირებული დეპრესია შეიძლება იმალებოდეს.
თირკმელზედას რაც შეეხება,კუშინგის სინდრომია სასწრაფოდ გამოსარიცხი:წონაში მოიმატე,ხშირად გშივდება და ხშირად შარდავ/სვამ სითხეებს?თუ მოიმატე რა ადგილებში(როგორ ჩალაგდა ცხიმი)სახე დაგიმრგვალდა?ლოყები წითელი?კუნღებში სისუსტე?სტრიები მუცლის წინა გვერდით ზადალირებზე და ბარძაყების ლატერალურ ზედაპირებზე,დუნდულებზე?კუშინგი თვითონ იწვევს დეპრესიას.თუმცა ზომიერი ჰიპერკორტიციზმი(რაც დექსამეტაზონით ნონ-სუპრესიის ტესტით დგინდება) შეიძლება დეპრესიამაც მოგცეს.ჰიპერტენზიას კი ორივე იწვევს....

Posted by: ტანკე 26 Jun 2008, 01:25
texasuri jleta benzoxerxit
QUOTE
ამ კაცს აქვს იპოქონდრიულ-სენესტოპათიური სინდრომი

ინგლისურად რა ჰქვია მაგ სინდრომს?
QUOTE
წინა პორსტებიც გამახსენდა

ხო,ჩემი ჯანმრთელობა რომ მოშნად გკიდია,სწორედ ეგ ვარ
QUOTE
რომლის უკანაც მასკირებული დეპრესია შეიძლება იმალებოდეს.

შეიძლება.
QUOTE
კუშინგის სინდრომია სასწრაფოდ გამოსარიცხი:წონაში მოიმატე,ხშირად გშივდება და ხშირად შარდავ/სვამ სითხეებს?

ხშირად მშივდება,კი.ხშირად ვიღებდი სითხეებს,ახლა იმდენად არა.შარდვა ნორმალური მაქვს.
QUOTE
თუ მოიმატე რა ადგილებში(როგორ ჩალაგდა ცხიმი)

ღიპი,ლოყები და ღაბაბი ძირითადად.
QUOTE
ლოყები წითელი?

ლოყებგაჟღაჟა არასოდეს ვყოფილვარ,პირიქითაც

QUOTE
სახე დაგიმრგვალდა?

მრგვალი სახე არასოდეს მქონია,თუმცა გამხდარზე ცოტა სხვანაირი სახე მქონდა,შეიძლება არც კი.ნაკვთები შემეცვალა
QUOTE
კუნღებში სისუსტე?

არ ვიცი.ხელს ვერავის ვუწევ,მაგას თუ სისუსტე ჰქვია და ისე დისკომფორტს არ განვიცდი

QUOTE
სტრიები მუცლის წინა გვერდით ზადალირებზე და ბარძაყების ლატერალურ ზედაპირებზე,დუნდულებზე?

არა

Posted by: vano_t 26 Jun 2008, 01:41
ტანკე
QUOTE
mrt გავიეკთე smile.gif)))
QUOTE
რაც შეეხება ალცჰეიმერს, ეს არცერთ გამოსახულებით გამოკვლევებზე (MRI, CT, SPECT თუ სხვა) არ ჩანს

ნამდვილად? მეორე ფაზა,მგონი,ჩანს.ტვინის ატროფია ხდება...
mrt უბრალოდ მაგნიტურ რეზონანსურ ტომოგრაფიას ნიშნავს. ალბათ ეგ MRI არის ანგიოგრაფიის გარეშე. MRT სისხლძარღვების პათოლოგიებს ვერ დაინიახავს კარგად.

ტვინის ატროფია (უფრო სწორად ქერქის ატროფია) საერთოდ არ არის სპეციფიური ალცჰეიმერისათვის. ქერქის ატროფია მოხუცებულ ასაკში (რაც ალცჰეიმერის ასაკს შეესაბამება) ისედაც ხშირია ყოველგვარი დემენციის გარეშე. ანუ, ავადმყოფს შეიძლება ქერქის ატროფია ქონდეს (თანაც ძლიერი) მაგრამ მენტალური ფუნქციები (მახსოვრობის და აზროვნების ჩათვლით) ნორმაში იყოს. ასევე, ბევრ ალცჰეიმერიან ავადმყოფს საერთოდ შეიძლება არ ქონდეს ატროფია ტვინის სკანირებისას გამოხატული. ქერქის ატროფია, მოკლედ, არ არის სპეციფიური ალცჰეიმერისათვის. ალცჰეიმერის დაავადების დიაგნოზი თავიდან ბოლომდე კლინიკური ნიშნების მიხედვით ისმება. არ არსებობს მისი დამადასტურებელი რაიმე ტესტი. არსებობს კვლევები ისეთ ტესტებზე რომლებიც ტვინის მეტაბოლიზმს სწავლობენ (მაგალითად პოზიტრონ ემისიული ტომოგრაფია) მარა არც ეს ტესტებია სპეციფიური. ასე რომ, თუ ვინმემ იტყვის, ტვინის სკანირებაზე დაყდნობით, ავადმყოფს ალცჰეიმერი ჭირსო, არ იქნება მართალი.

Posted by: ტანკე 26 Jun 2008, 01:44
vano_t
QUOTE
mrt უბრალოდ მაგნიტურ რეზონანსურ ტომოგრაფიას ნიშნავს. ალბათ ეგ MRI არის ანგიოგრაფიის გარეშე. MRT სისხლძარღვების პათოლოგიებს ვერ დაინიახავს კარგად.

სწორედ მაგნიტურ რეზონანსური ტომოგრაფია გავიკეთე yes.gif
thx ინფოსთვის

Posted by: texasuri jleta benzoxerxit 26 Jun 2008, 03:45
ტანკე
ვიცი,რომ ის ხარ,ვისზედაც დავწერე მკიდია-თქო biggrin.gif ეს ფიგურალურად იყო ნათქვამი...მე არავის ჯანმრთელობა არ მკიდია,პირიქით,რაც მეტი პრობლემები ჯანმრთელობასთან დაკავშირებით,მიტ მეტად მიხარია. fig.gif gigi.gif
ინგლისურად მასევეა-senestopathy. hypochondry(თუ არ ვცდები)
კუშინგი აუცილებლად გამოსარიცხია(უნდა ინახოს კორტიზოლი სისხლში,17-ოქსიკეტოსტეროიდები შარდში,სავარაუდოდ გაკეთდეს დექსამეტაზონით ნონსუპრესიის ცდა,გაკეთდეს თირკმელზედა ჯირკვლების ექოსკოპია,CT(ჰიპოფიზი გამოკვლეული გაქვს),კალიუმი სისხლში,გლუკოზა ჭამის წინ და ჭამის შემდეგ(დატვირთვის ტესტი ტოლერანტობაზე),ინსულინი ჭამის წინ და პოსტპრანდიალური(ჭამიდან 2 საათში),ლიპიდური ცვლა...)არ ხარ საქართველოში?
რაც შეეხება იპოქონდრიულ-სენესტოპათიურ სინდრომს ეს 100%სახეზეა.იმალება თუ არა ამის უკან მასკირებული დეპრესია,ეს საკითხი ღიად რჩება.ალბათ კი.თან ორივე ზემოხსენებული პათოლოგიისას იხმარება ანტიდეპრესანტები-სეროტონინერგული-ანაფრანილი,პროზაკი,ზოლოფტი,პაქსილი(რექსეტინი),ციპრალექსი,ფევარინი.
მათ შორის ლიდერობს ანაფრანილი.
ენდოკრინოლოგს მიაკითხე და პარალელურად ან ოდნავ მერე ფსიქიატრს(ექიმი ხარ,როგორც მახსოვს და ფსიქიატრის ხსენებაზე მიხვდები ალბათ,რომ 'გიჟად"არ გთვლი.ნევროზებიც ფსიქიატრის წასაყვანია,ნევროლოგები რომ დაეპატრონენ.იპოქონდრია კი ძირითადად ნევროზული რეგისტრისაა,თუმცა შეიძლება იყოს ფსიქოზური givi.gif ).რაციონალური ანტიდეპრეიული მკურნალობით 1-2 თვეში ვეღარ იცნობ თავს.დეპრესიის ისეთი კლასიკური ნიღაბიც,როგორიცაა არტერიული ჰიპერტენზია,მოწესრიგდება(თუ მისი მიზეზი კუშინგი არაა)
უდაჩი!

Posted by: ტანკე 26 Jun 2008, 03:57
texasuri jleta benzoxerxit
QUOTE
არ ხარ საქართველოში?

საქართველოში ვარ.ჰიპოფიზი სრულ წესრიგშია.

კუშინგის ბევრი სიმპტომი არ ემთხვევა და რავი...
QUOTE
რაც შეეხება იპოქონდრიულ-სენესტოპათიურ სინდრომს ეს 100%სახეზეა

ამის შესახებ არაფერი მსმენია და რა სიმპტომები ახასიათებს?
QUOTE
ენდოკრინოლოგს მიაკითხე და პარალელურად ან ოდნავ მერე ფსიქიატრს(ექიმი ხარ,როგორც მახსოვს და ფსიქიატრის ხსენებაზე მიხვდები ალბათ,რომ 'გიჟად"არ გთვლი.

ნევროპათოლოგთანაც ვარ ნამყოფი და ისეთი არაფერიო.
ფსიქიატრთან მისვლას არ ვაპირებ...საკUტარ ტავთან არასოდეს ვარ გულახდილიდ ა სხვასთან ხომ მითუმეტეს biggrin.gif everybody lies © Dr. Gregory House M.D. biggrin.gif

QUOTE
რაციონალური ანტიდეპრეიული მკურნალობით 1-2 თვეში ვეღარ იცნობ თავს

adre vsvamdi da mombezrda.sisulelea.წამლებს უარი გამოვუცხადე
QUOTE
დეპრესიის ისეთი კლასიკური ნიღაბიც,როგორიცაა არტერიული ჰიპერტენზია,მოწესრიგდება

არა.ჰიპერტენზია და ტაქიკარდია ბავშვობიდან მაქვს,ნევროზი-მოგიანებით
QUOTE
უდაჩი!

გაიხარე

Posted by: texasuri jleta benzoxerxit 26 Jun 2008, 13:19
ტანკე
ფარისებრი ჯირკვალი გამოიკვლიე?
წამლებს(მით უმეტეს ანტიდეპრესანტებს)უარი თუ გამოუცხადე შენი აჯობებს... biggrin.gif ფსიქიატრთან შეიძლება იმაზე მეტი გულახდილობა არ დაგჭირდეს რასაც ფორუმზე ავლენ საქვეყნოდ....
არ არის აუცილებელი კუშინგი კლასიკური სურატით იყოს წარმოდგენილი
იპოქონდრია:შიში და/ან დარწმუნება,რომ გაქვს რაიმე სერიოზული დაავადება.ინფექციური სნეულებეის შიში,სიმსივნის.ანევრიზმის და ა.შ...(ცოტა პოპულარულ ენაზე ვწერ).იპოქონდრია ეს არის ისტერიის ერთ-ერთი სახე.ისტერია მსუბუქ ფორმაში ეს არის პიროვნების აქცენტუაცია,მძიმე ფორმაში-ფსიქოპათია.ისტერიული შტრიხებია:ყურადღების ცენტრში ყოფნის მოჭარბებული სურვილი,ფსიქიკური ინფანტილურობა,ტენდენცია საკუთარი თავის,მით უმეტეს (ჯანმრთელობის)პრობლემების თეატრალური,დემონსტრატიული წარმოჩენისაკენ.ტიპიური ისტერიული თავდაცვითი რეაქცია სტრესის/პრობლემის დროს,ეს არის "გაქცევა ავადმყოფობაში",რაც პასუხისმგებლობის თავიდან აცილების და ყურადღების ცენტრში დარჩენის კარგი საშუალებაა.ისტერიას ნებისმიერი დაავადების იმიტირება შეუძლია,მაგ. სიბრმავის,ეპილეფსიური კრუნჩხვის,თუმცა ასეთი ფორმით დღეს იშვიათად გვხვდება.უფრო ხშირია სენესტოპათიები:სხვა და სხვა პათოლოგიურიშეგრძნებები ორგანოების და ორგანოტა სისტემის მხრივ:თავის ტკივილი,გულისრევა,თავბრუ,უცნაური შეგრძნება თავში,მუჩელში,გულის არეში,კიდურებში,ქაიკარდია,გამოვარდნები,ჰიპერ-ან ჰიპოტენზია,კუჭ-ნაწლავის სისტემის აშლილობები,სხვა და სხვა სახის ტკივილები.ავადმყოფი ახდენს ამ შეგრძნებების იპოქონდრიულ ინტერპრეტაციას,დარბის სხვადასხვა პროფილის ექიმებთან,იტარებს უამრავ(უძვირეს გამოკვლევას),აღშფოთებულია,როდესაც არწმუნებენ,რომ ის ჯანმრთელია.ან პირიქით დროებით წყნარდება,მაგრამ შემდეგ თავს იჩენს სხვა სიმპტომები და ა.შ.

იპოქონდრია ძირითადად ობსესიურ-კომპულსიური (აკვიატების ნევროზის)სპექტრის აშლილობაა,მაგრამ ხშირად ის ბოდვით რეგისტრს აღწევს და წომელიმე სერიოზული,ფსიქოზური რეგისტრის დაავადაბის სტრუქტურაში შედის,რაც როგორც მგონია,შენ არ გემუქრება.
ასეა თუ ისე იპოქონდრაზე ტრადიციულ ანტიფსიქიზურ(ნეიროლეფსიურ)მედიკანტებზე ბევრად უფრო ეფექტურად მოქმედებს სეროტონინერგული ანტიდეპრესანტები(იხ.ჩემი წინა პოსტი),განსაკუთრებით ანაფრანილი.
მე არა მარტო ექიმის სიბრტყიდან გელაპარაკები.მეც მქონდა სომატიზირებული დეპრესია,სენესტო-იპოქონდრია,არტერიული ჰიპერტენზია.ვიცი ეს რა არის,როგორ წამლავს ცხოვრებას.რაციონალური თვითმკურნალობის შედეგად(ანტიდეპრესანტით-მე,როგორც ექიმს,მაქვს ამის უფლება),ფანტასტიურად ვარ,შფოთვა რა არის,საერთოდ აღარ მახსოვს,მკიდია,რაც გინდა დამემარატოს აღარ მეშინია,ნუ დეპრესიის და წნევის კვალიც კი აღარ არის.
ამხელა ეპისტოლეს მერე კიდევ გგონია,რომ მკიდია შენი ჯანმრთელობა?
biggrin.gif biggrin.gif biggrin.gif biggrin.gif

Posted by: nomatter77 26 Jun 2008, 17:56
texasuri jleta benzoxerxit,
შთამბეჭდავი პოსტია; მეორეს მხრივ, კაცმა ჩათვალოს იპოქონდრია მჭირსო და დაავადებაზე თვალები დახუჭოს .. ალბათ ისევ გამოკვლევებით თუ იმსჯელებ და გამორიცხვის მეთოდით დაადგენ იპოქონდრიას.

თუ შეიძლება მეც გკითხავთ ჩემი სიმპტომების შესახებ..
ბავშვობიდან მქონდა კუშის მოქმედების პრობლემა, გარდატეხის ასაკიდან დამეწყო პერიოდული (ალბათ უფრო სეზონური) სუნთქვის უკმარისობა, რასაც თან ერთოდა გულის არეში დისკომფორტი და იშვიათად ამოვარდნები.. კარდიოგრამები და სხვა გამოკვლევები არაფერ პრობლემაზე არ მიუთითებდნენ (ეს პრობლემა ახლაც თან მდევს); 17 წლისას დამეწყო თმის ცვენა (+ დეპრესია), რაც 20 წლის ასაკში შეწყდა. შემდეგ (22 წლისას) წყლული (ხანგრძლივმა მკურნალობამ უშველა), დაახლოებით 2 თვეში ერთხელ დამეწყო ტკივილი, რომელიც იწყებოდა ქვედა მუცლის არიდან, იზრდებოდა ძლიერ ტკივილში კუჭის მიდამოებში და მთავრდებოდა ღებინებით. მთლიანად გრძელდებოდა 5-6 საათი - არც თუ ისე დიდი ხნის წინათ ამომაჭრეს ბრმა ნაწლავი (შეიძლება რამე კავშირში იყო); გარდა ამისა მუცლის მოძრაობისას მაქვს უსიამოვნო რეაქცია (თითქოს ტკაცუნის), რაც ზეპირად დააკავშირეს მუცლის ღრუში ლიმფურ კვანძებს (მეზენტერიტი?).
წლის დასაწყისში დამისვეს ჩირქოვანი ტონზილიტის დიაგნოზი (და დააკავშირეს სუნთქვის პრობლემებს).

მაქვს ჩაბარებული უამრავი ანალიზი... (მათ შორის HIV, Hepatitis B/C, ფარისებრი/ღვიძლის ფუნქციები, სისხლი, შარდი, ეტც.) ყველაფერი ნორმაშია..
მეზენტერიტზე რა გამოკვლევები შეიძლება ჩავიტარო..? ცოტა არ იყოს დავიღალე ამ პრობლემებით + იპოქონდრია მგონი მეც მჭირს...
ჰელპ

Posted by: ტანკე 26 Jun 2008, 22:23
texasuri jleta benzoxerxit
QUOTE
ფარისებრი ჯირკვალი გამოიკვლიე?

სრულ წესრიგშია smile.gif
QUOTE
წამლებს(მით უმეტეს ანტიდეპრესანტებს)უარი თუ გამოუცხადე შენი აჯობებს..

არ შველის : D
QUOTE
იპოქონდრია:შიში და/ან დარწმუნება,რომ გაქვს რაიმე სერიოზული დაავადება.

სერიოზულ დაავადებაზე არ ვფიქრობ.მე მაწუხებს ის,რომ ვერ გაუგია ვერავის,თუ რა მჭირს.ბავშვობიდან რომ მჭირს რაღაც და მთლად ჯანმრთელი არ ვარ,ეს ფაქტია smile.gif მივხედავ მაგ თირკმელზედა ჯირკვალს და,იმედია,პრობლემების ნახევარი მაინც მოგვარდება smile.gif ვიცი ყველა დაავადების თუ არა,უმეტესი დაავადებების კლინიკური ნიშნები და გაციებაზე არასოდეს ვამბობ,რომ რაღაც საშინელებაა biggrin.gif
QUOTE
ინფექციური სნეულებეის შიში,სიმსივნის.ანევრიზმის და ა.შ...

ინფექციური დაავადებებიდან ერთადერთი აივ-ის მეშინია(პლიუს ც ჰეპატიტი)-ვირუსული დაავადებებიც შევიყვანე მანდ.სხვა-არა.სიმსივნეები-არ ვიცი,თუ არაფერი მაწუხებს და კლინიკაც არ ემთხვევა,მაშინ არა.ანევრიზმის არასოდეს მეშინოდა.აი,საკუთარ გულისცემას რომ ვუსმენდი მარცხენა ყურში,მაშინ გადავწყვიტე გავსინჯულიყავი.თან ბავშვობიდან მაღალ წნევას ვატარებ და იყო იმის შანში,რომ ანევრიზმა დამმართნოდა.
QUOTE
იპოქონდრია ეს არის ისტერიის ერთ-ერთი სახე.ისტერია მსუბუქ ფორმაში ეს არის პიროვნების აქცენტუაცია,მძიმე ფორმაში-ფსიქოპათია.

იპოქონდრია ვიცი,რაც არის : ) ისტერია...არ ვიცი,არა მგონია.

.
QUOTE
ისტერიული შტრიხებია:ყურადღების ცენტრში ყოფნის მოჭარბებული სურვილი,

პირიქით,მიყვარს "უკან" და "სიბნელეში" ყოფნა smile.gif თავს ასე უფრო კომფორტულად ვგრძნობ.
QUOTE
ფსიქიკური ინფანტილურობა

აქ რა იგულისხმება?
QUOTE
ტენდენცია საკუთარი თავის,მით უმეტეს (ჯანმრთელობის)პრობლემების თეატრალური,დემონსტრატიული წარმოჩენისაკენ.

პირიქით,ოჯახის წევრებისგან ვმალავ,თუ რამეზე მაქვს ეჭვი.არ მიყვარს ზედმეტი ფოფინი და შემაწუხებელი ყურადღება.ფორუმზე ვწერ,რასაც ვწერ,სხვებთან არ ვსაუბრობ ამ საკითხებზე smile.gif
QUOTE
ტიპიური ისტერიული თავდაცვითი რეაქცია სტრესის/პრობლემის დროს,ეს არის "გაქცევა ავადმყოფობაში",რაც პასუხისმგებლობის თავიდან აცილების და ყურადღების ცენტრში დარჩენის კარგი საშუალებაა.

მძულს ეგეთი ადამიანები.არასდროს არაფერს გავურბივარ.პრობლემების მოგვარება ჩემი ჰობია და ამით ძალიან ვამაყობ smile.gif
QUOTE
თავის ტკივილი,გულისრევა,თავბრუ,უცნაური შეგრძნება თავში

არა
QUOTE
მუცელში

ბოლო რამდენიმე დღეა მაწუხებს,მაგრამ არანაირად არ ვუკავშირებ ამას მაგ ყველაფერს
QUOTE
გულის არეში,კიდურებში

არა
QUOTE
კუჭ-ნაწლავის სისტემის აშლილობები

ეს მაქვს.მრავლობით-ეროზიული გასტროენტერიტი მაქვს.(უფრო სწორად-გასტროდუოდენიტი)

QUOTE
დარბის სხვადასხვა პროფილის ექიმებთან,იტარებს უამრავ(უძვირეს გამოკვლევას),

ნწუ.
ექიმებთან სიარული არ მიყვარს.სულ ორჯერ ვარ ნამყოფი ბოლო წლებში- ზონდის გადაყლაპვაზე და მეორედ ანევრიზმაზე.ზონდის გადაყლაპვაზე ჩემი ფულით მივედი,სახლში არც მითქვამს და ანევრიზმაზე ოჯახის დახმარება დამჭირდა ფინანსურად,ამიტომ ვთქვი სახლში.(3 თვე ვითმინე და მერე ვთქვი).ხოდა რომ არაფერი აღმომაჩნდა და წნევების მიზეზები ვერ გავიგეთ,მერე ჯაჭვური რეაქციით მატარეს სხვადასხვა გამოკვლევებზე(ჩვენი ახლობლის(ექიმის) რეკომენდაციით).
QUOTE
აღშფოთებულია,როდესაც არწმუნებენ,რომ ის ჯანმრთელია.

ნეტავი მითხრან.იმას მეუბნებიან,რომ სპორტული ცხოვრება დაივიწყე,არ ხარ ჯანმრთელიო.
QUOTE
ან პირიქით დროებით წყნარდება,მაგრამ შემდეგ თავს იჩენს სხვა სიმპტომები და ა.შ.

ეს უფრო სწორია ჩემს შემთხვევაში.
QUOTE
შფოთვა რა არის,საერთოდ აღარ მახსოვს

ძილი მაქვს ძალიან მაგარი : ) ნუ,თავიდან 3 საათი ვერ ვიძინებ ხოლმე დაწოლისას(სულ ასე ვიყავი,რაც თავი მახსოვს),მაგრამ თუ დამეძინა,ტკბილად მძინავს 14-16 საათი smile.gif
QUOTE
ამხელა ეპისტოლეს მერე კიდევ გგონია,რომ მკიდია შენი ჯანმრთელობა?

biggrin.gif
უღრმესი მადლობა სუპერ პოსტისთვის smile.gif გაიხარე

* * *
ცხოვრებაში ერთხელ მტკიოდა თავი 2-3 კვირით ი ვსიო

Posted by: Guardian 26 Jun 2008, 22:47
ტანკე
QUOTE
ექიმებთან სიარული არ მიყვარს.სულ ორჯერ ვარ ნამყოფი ბოლო წლებში- ზონდის გადაყლაპვაზე და მეორედ ანევრიზმაზე.

აივ ინფექციაზე კვლევა დაგავიწყდა.

ცოლი გინდა შენ, ცოლი - და თანაც - ASAP! biggrin.gif

Posted by: ტანკე 26 Jun 2008, 23:38
Guardian
QUOTE
ექიმებთან სიარული არ მიყვარს.სულ ორჯერ ვარ ნამყოფი ბოლო წლებში- ზონდის გადაყლაპვაზე და მეორედ ანევრიზმაზე.

ლოლ,ხო ეგ ახლა გამახსენდა biggrin.gif
ეგ სპონტანურად მოხდა biggrin.gif
აივ-ის კვირეული იყო და უფასო გამოკვლევებს ატარებდნენ ეგ ერთი კვირა.ერთს გავყევი და თან მეც შემომეკეთა შემთხვევით ეგ ანალიზი biggrin.gif
QUOTE
ცოლი გინდა შენ, ცოლი - და თანაც - ASAP! biggrin.gif

საბოლოოდ მღუპავ? biggrin.gif დეპრესიაში მაგდებ? biggrin.gif ცოლიღა მაკლია ყველა სიკეთესთან ერთად vis.gif

Posted by: Guardian 27 Jun 2008, 07:49
ტანკე
QUOTE
დეპრესიაში მაგდებ?

პირიქით - დეპრესიიდან გამოსვლის საუკეთესო გზას გეუბნები.
QUOTE
ცოლიღა მაკლია ყველა სიკეთესთან ერთად

ცოლი არის ის, რაც შენ ეხლა ჰაერივით გჭირდება. biggrin.gif

* * *
http://youtube.com/watch?v=PaZWQHzfGXs

Posted by: tamtuna 27 Jun 2008, 21:27
სად დავპოსტო არ ვიცი მარა ტვინის შერყევის სიმპმტომები იქნებ მითხარათ user.gif

Posted by: vano_t 28 Jun 2008, 00:36
QUOTE (tamtuna @ 27 Jun 2008, 21:27 )
სად დავპოსტო არ ვიცი მარა ტვინის შერყევის სიმპმტომები იქნებ მითხარათ user.gif

ღებინება, გულისრევა, თავისტკივილი, ტრამვამდე ან ტრამვის შემდეგ მომხდარის დავიწყება, დიზორიენტაცია, გულყრა, აგზნება, ლეთარგია და ა.შ.

მოკლედ ბევრი რამ შეიძლება ახასიათებდეს.

Posted by: lala 29 Jun 2008, 23:02
იცით რა მაინტერესებს?
ჭარბთმიანობის გამომწვევი მიზეზები. აბსოლუტურად ყველა მიზეზი და შესაბამისად ის ექიმები ვისთანაც უნდა მივიდე ამის გამოსაკვლევად. გინეკოლოგი ენდოკრინოლოგი და აშ
ჩამომითვალეთ რა

Posted by: vano_t 30 Jun 2008, 06:03
QUOTE (lala @ 29 Jun 2008, 23:02 )
იცით რა მაინტერესებს?
ჭარბთმიანობის გამომწვევი მიზეზები. აბსოლუტურად ყველა მიზეზი და შესაბამისად ის ექიმები ვისთანაც უნდა მივიდე ამის გამოსაკვლევად. გინეკოლოგი ენდოკრინოლოგი და აშ
ჩამომითვალეთ რა

მიზეზები ბევრია. ყველაზე ხშირი, თუმცა, ჰორმონულებია, განსაკუთრებით საკვერცხეების პოლიკისტოზი. ენდოკრინოლოგთან მიდი და ენდოკრინოლოგი დაგაკვალიანებს.

Posted by: lala 30 Jun 2008, 20:06
vano_t
ეგ გამოკვლეული მაქვს და არაფერი არ აღმომაჩნდა
ჰორმონებზეც მაქვ რამდენიმე ანალიზი აღებული ეგეც ნორმაშია
ამიტომ დამაინტერეა კიდევ სხვა მიზეზებმა. თუ იცით კიდევ სხვა მიზეზებიც დამისახელეთ რა

Posted by: vano_t 1 Jul 2008, 09:06
QUOTE (lala @ 30 Jun 2008, 20:06 )
vano_t
ეგ გამოკვლეული მაქვს და არაფერი არ აღმომაჩნდა
ჰორმონებზეც მაქვ რამდენიმე ანალიზი აღებული ეგეც ნორმაშია
ამიტომ დამაინტერეა კიდევ სხვა მიზეზებმა. თუ იცით კიდევ სხვა მიზეზებიც დამისახელეთ რა

კიდევ არის გენეტიკური და კიდევ არის ე.წ. იდიოპათური, რაც ნიშნავს, რომ მიზეზი არ არის ცნობილი.

Posted by: shtori 3 Jul 2008, 23:43
ტუჩის კუთხესთან პერიოდულად მცირე ზომის ნახეთქი მიჩნდება. რისი ბრალი შეიძლება იყოს?
* * *
კიდე კბილებზე ბზარები გამიჩნდა, ძვლები მტკივა: წვივის წინა მხარე(სიარულის დროს) და გულ-მკერდის შუა ადგილი ხელის დაჭერისას; ხელის თითების სასხარი, ნეორე ხელის წემოქმედებით რომ მოვხარო, ზოგს რომ უტკაცუნებს, ჩემი შხრიალის ხმას გამოსცემს(თითქოს ძვლები ერთმანეთზე იხეხებაო); ტერფი მტიკვდება ხოლმე კიდე თითქოს თაღი იბერება, მრუდდება თუ რავიი და ცერა თითს მიკავებს, ვეღარ ვშლი ხოლმე და ესე 2 წუთში გაივლის; და კიდე თმები მცვივა. არის ამათ შორის რამე კავშირი. თუ სხვადასახვა რამის ბრალია? მჭირს რამე? თუ მჭირს მაშინ რა არის და რა მიშველის? rolleyes.gif

თუ ძალიან უაზროდ (უაზრობა)არ დავწერე და რამე გაიგეთ და დამეხმარებით მადლობა smile.gif

Posted by: vano_t 5 Jul 2008, 00:14
shtori
QUOTE
ტუჩის კუთხესთან პერიოდულად მცირე ზომის ნახეთქი მიჩნდება. რისი ბრალი შეიძლება იყოს?
* * *
კიდე კბილებზე ბზარები გამიჩნდა, ძვლები მტკივა: წვივის წინა მხარე(სიარულის დროს) და გულ-მკერდის შუა ადგილი ხელის დაჭერისას; ხელის თითების სასხარი, ნეორე ხელის წემოქმედებით რომ მოვხარო, ზოგს რომ უტკაცუნებს, ჩემი შხრიალის ხმას გამოსცემს(თითქოს ძვლები ერთმანეთზე იხეხებაო); ტერფი მტიკვდება ხოლმე კიდე თითქოს თაღი იბერება, მრუდდება თუ რავიი და ცერა თითს მიკავებს, ვეღარ ვშლი ხოლმე და ესე 2 წუთში გაივლის; და კიდე თმები მცვივა. არის ამათ შორის რამე კავშირი. თუ სხვადასახვა რამის ბრალია?  მჭირს რამე? თუ მჭირს მაშინ რა არის და რა მიშველის?  rolleyes.gif

თუ ძალიან უაზროდ (უაზრობა)არ დავწერე და რამე გაიგეთ და დამეხმარებით მადლობა smile.gif

ტუჩებთან რომ გემართება მაგას angular cheilitis ქვია. დერმატიტის (კანის ანთების) ფორმაა ეგ და სხვადასხვა რაიმეს შეუძლია გამოიწვიოს, მათ შორის რკინის და ვიტამინების უკმარისობამ და ხანდახან სოკოვან ინფეზციასაც შეუძლია ეგ მოგცეს.

დანარჩენი სიმპტომები არასპეციფიურია (და შეიძლება ერთმანეთთანაც არის დაკავშირებული) და მასე ადვილად დიაგნოზს ვერ დასვამ. ყველაზე კაი რჩევა იქნება, ვინმე კარგი თერაპევტი ნახო და კარგად გამოგიკვლიოს.

Posted by: *hedgehog* 7 Jul 2008, 12:23
იქნებ მითხრათ სახსრის სკლეროზი რა არის და ინტერნეტში როგორ მოვძებნო, ან შესაბამისი ლინკები დამიდეთ sad.gif

Posted by: vano_t 7 Jul 2008, 19:31
QUOTE (*hedgehog* @ 7 Jul 2008, 12:23 )
იქნებ მითხრათ სახსრის სკლეროზი რა არის და ინტერნეტში როგორ მოვძებნო, ან შესაბამისი ლინკები დამიდეთ sad.gif

სახსრის სკლეროზი არ არსებობს. სკლეროზი საერთოდ ნიშნავს გამაგრებას, განსაკუთრებით ისეთ ქსოვილში, სადაც შემაერთებელი ქსოვილის ელემენტებია. სახსრის გარშემო რენტგენოლოგმა შეიძლება აღწეროს ძვლების სკლროზი, რაც უბრალოდ მიუთითებს ახალი ძვლის "არადალაგებულ" წარმომქნას.

Posted by: *hedgehog* 8 Jul 2008, 10:18
vano_t
QUOTE
სახსრის გარშემო რენტგენოლოგმა შეიძლება აღწეროს ძვლების სკლროზი, რაც უბრალოდ მიუთითებს ახალი ძვლის "არადალაგებულ" წარმომქნას

და ეს არადალაგებული წარმოქმნა როგორ შეიძლება, რომ დალაგდეს? არის შანსი, რომს სხვა სახსრებზეც განვითარდეს იგივე? ამის შესახებ ინფორმაცია სად შეიძლება წავიკითხო. წინასწარ დიდი მადლობა

Posted by: vano_t 9 Jul 2008, 00:52
*hedgehog*
QUOTE
და ეს არადალაგებული წარმოქმნა როგორ შეიძლება, რომ დალაგდეს? არის შანსი, რომს სხვა სახსრებზეც განვითარდეს იგივე? ამის შესახებ ინფორმაცია სად შეიძლება წავიკითხო. წინასწარ დიდი მადლობა

"არადალაგებულში" ვიგულისხმე არაორგანზებული. არაფერია ეს. ნორმალური პროცესია, რომელიც მიმდინარეობს სახსარში, როცა სახსრის "გაცვეთა" (wear and tear) ხდება. ოსტეოართრიტის მახასიათებელია. არაფერზე გადადება არ ხდება. ყველაზე კარგი საშუალება სახსრებისათვის არის ცურვა და ვარჯიში. რენტგენოლოგიურ მონაცემებს ყურადღებას ნუ მიაქცევ, მაინც ძნელი იქნება მაგის ახსნა. თუ ავადმყოფს ტკივილები აქვს, ფიზიკური თერაპია ყველაზე კარგი საშუალებაა, განსაკუთრებით ცურვა.

ოსტეოართრიტზე http://www.mayoclinic.com/health/osteoarthritis/DS00019 შეგიძლია წაიკითხო პაციენტისათვის გასაგებ ენაზე (ოღონდ ინგლისური საიტია).

Posted by: *hedgehog* 9 Jul 2008, 10:36
vano_t
QUOTE
თუ ავადმყოფს ტკივილები აქვს, ფიზიკური თერაპია ყველაზე კარგი საშუალებაა, განსაკუთრებით ცურვა

მე მჭირს ეს რაღაც და 1 წელია მტკივა უკვე, რაგაც წამლები დამინიშნეს ახლა და მერე ბლოკადა უნდა გავაკეთოთ, თუ არ გიშველისო. ყველაზე მეტად შენ დამამშვიდე, თორე ყველა რაღაც ისეთ სახელს იღებდა მეგონა კიბო მჭირდა. ვეცდები ცურვა ვისწავლო smile.gif და ვარჯიშებს რაც შეეხება, რაიმე განსაკუთრებული ვარჯიშები უნდა ვაკეთო?

Posted by: vano_t 9 Jul 2008, 19:41
*hedgehog*
QUOTE
მე მჭირს ეს რაღაც და 1 წელია მტკივა უკვე, რაგაც წამლები დამინიშნეს ახლა და მერე ბლოკადა უნდა გავაკეთოთ, თუ არ გიშველისო. ყველაზე მეტად შენ დამამშვიდე, თორე ყველა რაღაც ისეთ სახელს იღებდა მეგონა კიბო მჭირდა. ვეცდები ცურვა ვისწავლო smile.gif და ვარჯიშებს რაც შეეხება, რაიმე განსაკუთრებული ვარჯიშები უნდა ვაკეთო?

განსაკუთრებული არაფერი. იმ სახსრის შესაბამისი ვარჯიში, რომელი სახს(ა)რ(ებ)იც გტკივა. რომელი სახსარი ან სახსრები გაწუხებს მითხარი და მოგიძებნი შესაბამის ვარჯიშებს.

და სხვა რამეები (სისხლის ანალიზი, გასინჯვა და ა.შ.) თუ არის ნორმაში?

Posted by: *hedgehog* 9 Jul 2008, 21:52
vano_t
QUOTE
სისხლის ანალიზი

თუ სწორად მიკეთებენ ნორმაშია smile.gif რაღაც ვეღარ ვენდობი იმ კლინიკას სადაც ანალიზი გავიკეთე ბოლოს

გასინჯვაში რას გულისხმობ? ხელის დაჯერით, მოძრაობაში მტკივა, კოჭ-წვივის სახსარია მარჯვნივ და მარცხენა მხარეს მხარში დამეწყო იგივე ტკივილები გაზაფხულიდან. მხარზე ჯერ არაფერი გადამიღია, მაგრამ ფეხზე ტომოგრაფია გავიკეთე და მხოლოდ იქ გამოჩნდა რაღაც წაწვეტებული და ეს არისო "სკლეროზის" ნიშანი. როგორც ექიმმა ამიხსნა, იოგია გაღიზიანებული და ის უშლის სახსარს მოძრაობას. რაც უფრო ცხელა უფრო მტკივდება.

მხარს რაც შეეხება ჯერ დაკვირვების ქვეშ არის.

QUOTE
შესაბამის ვარჯიშებს

ხვალიდან დავიწყებ ვარჯიშებს და ვეცდები ცურვაზეც დავიწყო სიარული.

ძალიან დიდი მადლობა გამოხმაურებისთვის

Posted by: Vika 11 Jul 2008, 00:59
მეგობრებო, თბილისში კარგი გასტროენტეროლოგი როგორ ვიპოვო? მერსი.

Posted by: baby_boo 11 Jul 2008, 11:19
დიდი ხანია მტკივა ფეხები,საშინლად მიშუპდება და არანორმალურად მტკივა დილიდანვე . დიდხანს სიარული და დგომა საერთოდ არ შემიძლია მივედი ანგიოლოგთან ძალიან ყურადღებიანი და კარგი კაცი აღმოჩნდა,დამიწერა დანიშნულება : ქვედა კიდურების გრ.ვენ.უგბ.კომპენსაც სტადია( იქ ვერ გავარჩიე რა უწერია ბოდიში )მძიმე ფეხის სინდრომი,ხერხემლის ოსტეოქონდროზი.დამინიშნა დეტრალექსი 2 თვე,კარდიონაგნილი 2თვე,ასკორუტინი 1 თვე და ლიოტონი-1000. მითხრა გამეკეთებინა კოაგულოგრამა და კისრის მალების რენტგენოგრაფია .

შედეგი ასეთია:

კოაგულოგრამა:

1.sisxlis shededebis dro lit da uait -7,25

2. INR- 1,1

3.fibriogenis koncentracia mgr %shi- 325

4.B-FIBRINOGENI unda iyos - da maqvs +

5.etanolis sinji unda iyos - da maqvs +

6.a-PTT-28

7.TT trombinis dro -12

8.protrombini-98%


და რენტგენი კისრის :


ხერხემლის სვეტის კისრის ნაწილშI ფიზიოლოგიური ლორდოზი გასწორებულია,მიდრეკილი კიფოზისაკენ.მალების სხეულებსა და მათ მორჩებში დესტრუქციული და ტრავმული ხასიათის ცვლილებები არ აღინიშნება .მალების სხეულების წინა ზედაპირები დეფორმირებულია,მეტად c4-c5 მალთაშუა ნაპრალები თანაბარია.ჩამკეტი ფირფიტები დეფორმულია სკლეროზული ცვლილებების გარეშე .სასახსრე .წვეტიანი და განივი მორჩები წაწვეტიანებულია .c7 მალის განივი მორჩები ნორმალური ზომისაა.დამატებითი ნეკნი არ ფიქსირდება.


ეს შედეგები რომ მივუტანე მითხრა ყველაფერი კარგადაა და ნევროპათოლოგთან მიდიო,ამხელა ცვლილებები რომ არის მეთქი და ეგ ნორმალურიაო ,რავი აბა იქ მინუსები რომ უნდა იყოს ? ??? და რაღაც წვეტები და რამეები . თან ეხლა ფილმ პათოლოგიას ვუყურებ და ცოტა დავიზმენდი )))) გთხოვთ არ წამიშალოთ უცებ რაა, გადახედეთ არ დაგეზაროთ .ძააან მიყვარხართ და გენდობით ფორუმელებს ( ქლესა ღიმილ გადაკრული სახე) და აბა თქვენი იმედი მაქვს


უკაცრავად კოპიპეისტისათვის smile.gif)))

Posted by: ბაჩუკი 11 Jul 2008, 13:09
Vika
რესპუბლიკურში კოტე მაისაია, ძაან მაგარი ექიმია.

Posted by: Vika 11 Jul 2008, 16:50
ბაჩუკი
დიდი მადლობა.

Posted by: texasuri jleta benzoxerxit 11 Jul 2008, 16:58
ბაჩუკი
+1 გადასარევია კოტე

Posted by: vano_t 12 Jul 2008, 00:08
baby_boo
QUOTE
დიდი ხანია მტკივა ფეხები,საშინლად მიშუპდება და არანორმალურად მტკივა დილიდანვე . დიდხანს სიარული და დგომა საერთოდ არ შემიძლია მივედი ანგიოლოგთან ძალიან ყურადღებიანი და კარგი კაცი აღმოჩნდა,დამიწერა დანიშნულება : ქვედა კიდურების გრ.ვენ.უგბ.კომპენსაც სტადია( იქ ვერ გავარჩიე რა უწერია ბოდიში )მძიმე ფეხის სინდრომი,ხერხემლის ოსტეოქონდროზი.დამინიშნა დეტრალექსი 2 თვე,კარდიონაგნილი 2თვე,ასკორუტინი 1 თვე და ლიოტონი-1000. მითხრა გამეკეთებინა კოაგულოგრამა და კისრის მალების რენტგენოგრაფია .

შედეგი ასეთია:

კოაგულოგრამა:

1.sisxlis shededebis dro lit da uait -7,25

2. INR- 1,1

3.fibriogenis koncentracia mgr %shi- 325

4.B-FIBRINOGENI unda iyos - da maqvs +

5.etanolis sinji unda iyos - da maqvs +

6.a-PTT-28

7.TT trombinis dro -12

8.protrombini-98%


და რენტგენი კისრის :


ხერხემლის სვეტის კისრის ნაწილშI ფიზიოლოგიური ლორდოზი გასწორებულია,მიდრეკილი კიფოზისაკენ.მალების სხეულებსა და მათ მორჩებში დესტრუქციული და ტრავმული ხასიათის ცვლილებები არ აღინიშნება .მალების სხეულების წინა ზედაპირები დეფორმირებულია,მეტად c4-c5 მალთაშუა ნაპრალები თანაბარია.ჩამკეტი ფირფიტები დეფორმულია სკლეროზული ცვლილებების გარეშე .სასახსრე .წვეტიანი და განივი მორჩები წაწვეტიანებულია .c7 მალის განივი მორჩები ნორმალური ზომისაა.დამატებითი ნეკნი არ ფიქსირდება.


ეს შედეგები რომ მივუტანე მითხრა ყველაფერი კარგადაა და ნევროპათოლოგთან მიდიო,ამხელა ცვლილებები რომ არის მეთქი და ეგ ნორმალურიაო ,რავი აბა იქ მინუსები რომ უნდა იყოს ? ??? და რაღაც წვეტები და რამეები . თან ეხლა ფილმ პათოლოგიას ვუყურებ და ცოტა დავიზმენდი )))) გთხოვთ არ წამიშალოთ უცებ რაა, გადახედეთ არ დაგეზაროთ .ძააან მიყვარხართ და გენდობით ფორუმელებს ( ქლესა ღიმილ გადაკრული სახე) და აბა თქვენი იმედი მაქვს 


უკაცრავად კოპიპეისტისათვის smile.gif)))

რამდენი წლის ხარ? ფეხები რა ადგილას გტკივა? (ტერფი, კოჭი, წვივი, მუხლი, ბარძაყი თუ რა ადგილი?) სად გაქვს შეშუპებები? ვარჯიში გიძლიერებს თუ გიუარესებს ტკივილს? კისრის რენტგენი რატომ გადაიღე? კისერიც გტკივა თუ ხელების ტკივილი ან რაიმე სიმპტომები გაქვს ხელებში?

სხვა ლაბორატორიები თუ გაქვს რაიმე? (მაგალითად ერითროციტების დალექვის სიჩქარე, სისხლის საერთო ანალიზი, ანტიბირთვული ანტისხეულები და რევმატოიდული ფაქტორი?)

ნევროპათოლოგი მართალს ამბობს. ეგ რენტგენი ნორმალურია.

Posted by: baby_boo 12 Jul 2008, 21:48
vano_t


26 წლის ვარ.მტკივა ტერფებიდან მუხლამდე.კოჭები შემიშუპდა,ვარჯიში არ ვიცი რას მიშვება ,იმიტომ რომ მეჯავრება ვარჯიში და არ ვვარჯიშობ ,სიარულის შემდეგ იმდენად მტკივა ,რომ არ ვიცი,დაჟე დილით რომ ვიღვიძებ ხანდახან მაშინაც შეშუპებული მაქვს და მტკივა.კისერი არ მტკივა ,წელი მტკივა საშინლად ხელებზე კი უბრალოდ ვენები მეტყობა უფრო. სხვა ანალიზები არა მაქვს

Posted by: gvelis_wiwila 13 Jul 2008, 17:38
ნევროზი მაქვს.

ძალიან მაწუხებს. yes.gif '

რა ვქნა? spy.gif

ვინმე ექიმი მითხარით, რომ კარგად ამიხსნანს რა მჭირს, წამლები დამინიშნოს და რაც შეიძლება იაფი გამომართვას ამაში smile.gif

Posted by: vano_t 16 Jul 2008, 23:29
gvelis_wiwila
QUOTE
ნევროზი მაქვს.

ძალიან მაწუხებს.  yes.gif '

რა ვქნა?  spy.gif

ვინმე ექიმი მითხარით, რომ კარგად ამიხსნანს რა მჭირს, წამლები დამინიშნოს და რაც შეიძლება იაფი გამომართვას ამაში smile.gif

წესით ჯერ ექიმთან უნდა მიხვიდე, რომ დიაგნოზი დაგისვას და მერე მკურნალობა დაიწყოს შესაბამისად. ნევროზი რომც გქონდეს, რამოდენიმე ფორმაა მაგის (ხვადასხვა სპეციფიური ფობიები, გენერალიზებული "შეშფოთება" და ა.შ.) და ამას ექიმ-ფსიქიატრი (ან ფსიქოლოგი) გაგრკვევს. მკურნალობაც შეიძლება იყოს სხვადასხვა წამლებით არ არაწამლებით.

ტეხასური ჟლეტა არის მაგის სპეციალისტი. უფრო ბევრს გეტყვის.

Posted by: Nensi_1988 16 Jul 2008, 23:57
არ ვიცი რამდენად სწორად ავირჩიე ეს თემა მარა იქნებ მიშველოთ ვინმემ.

მზეზე დავიწვი სასტიკად.... გადავაჭარბე "ზაგრის" მიღება... საშინლად მიხურს, მაწონი წავისვი მარა რო ჩამოვიბანე მაინც მიხურს.... როგორ მოვიქცე რომ სიმხურვალემ გამიაროს და რა ვქნა რომ კანი არ ამძვრეს? ძალიან თეთრი ვარ და ადვილად ვიწვები

Posted by: texasuri jleta benzoxerxit 17 Jul 2008, 02:24
Nensi_1988
კანი აუცილებლად აგძვრება ეს მინიმუმ.მზის დაკვრას თუ გადაურჩი ხომ კარგი...და ჰიპერინსოლაციისგან შენი კანის(მით უმეტეს ხალის)უჯრედებმა მუტაცია თუ არ განიცადეს კიდევ უფრო კარგი biggrin.gif
მიიღე სითხე დიდი რაოდენობით,კარგია ერთი-ორი დღე პანანგინს ან ასპარკამს(კალიუმის პრეპარატებია)თუ მიიღებ,1-2 ტა. 3ჯერ ჭ.შ.
ტაქიკარდიის შემთხვევაში ანაპრილინი 0,01გ 1-2ჯერ დღეში.
თუ დაგეწყო თავის ყკივილი,თავბრუსხვევა,გულისრევა,ტემპერატურის მატება სასწრაფოდ მიმართე თერაპევტს და/ან ნევროლოგს

Posted by: tikope 17 Jul 2008, 08:24
vano_t
და ასევე ფორუმის სხვა ექიმობო
იქნებ მითხრათ ჭინჭრის ციების შესახებ რამე , და რა მეთოდებით იკურნება?მადლობთ

Posted by: Nensi_1988 17 Jul 2008, 11:21
texasuri jleta benzoxerxit


მგონი გადავრჩი, დღეს აღარც სიცხე მაქვს და აღარც გუშინდელივით მიხურს კანი. მხოლოდ ხელის დადებით ვგრძნობ ოდნავ მხურვალებას. ანუ მზის დაკვრას გადავრჩი biggrin.gif სიწითლეც ჩაცხრა, მხოლოდ შევარდისფერებულია კანი. გულისრევა და თავბრუსხვევა არ მაქვს საბედნიეროდ.

კანის აძრობა გარდაუვალია ხომ? weep.gif weep.gif weep.gif

Posted by: MebenZinis dzma 17 Jul 2008, 19:45
vano_t
სხვა ექიმ-იუზერებო, ან პირიქით...
თუ არ შეწუხდებით, რა პროცესმა და რომელ ორგანოტა სისტემაში (თუ არსებობს ასეთი ზოგადი მიდგომა), შეიძლება გამოიწვიოს დაბალი ტემპერატურა?

Posted by: vano_t 17 Jul 2008, 21:04
tikope
QUOTE
იქნებ მითხრათ ჭინჭრის ციების შესახებ რამე , და რა მეთოდებით იკურნება?მადლობთ

ზუსტად არ მახსოვს რას ეძახიან ჭინჭრის ციებას, მარა, რომელიღაც ალერგიულ დმგომარეობას ეძახიან. შენ რა სიმპტომები გჭირს?

ზოგადად, ნებისმიერი I ტიპის ალერგიული რეაქცია გამოწვეულია ანტიგენის (უცხო ნივთიერების) ურთიერთქმედებით ანტისხეულთან (ორგანიზმში წარმოქმნილ მოლეკულებთან, რომელიც ებრძვის ანტიგენებს). ამ ურთეირთქმედებისას ბევრი უჯრედის აქტივაცია ხდება და ისეთი ნაერთები გამოიყოფა ადგილობრივად და სისხლში, რომელიც იწვევს ალერგიული რეაქციების სიმპტომებს: ცემინება, კანზე განონაყარი, ქავილი და ა.შ. (ბევრნაირი სიმპტომი შეიძლება ქონდეს ალერგიას). განკურნებით მსგავსი რამეები არ იკურნება. შეიძლება სპონტანურად მოხდეს განკურნება, მარა წამალი არ არსებობს. წამლები მხოლოდ ამცირებენ სიმპტომებს. რამოდენიმე მეთოდი არსებობს ალერგიასთან საბრძოლველად: 1) ალერგენისაგან თავის არიდება, თუ შესაძლებელია; 2) თუ ალერგენთან კონტაქტი გიწევს, მაშინ ანტიალერგიული წამლები უნდა დაენიშნოს ავადმყოფს; 3) ძალიან მცირე რაოდენობით ალერგენის შეყვანა, რაც იწვევს ე.წ. დესენსიტიზაციას (ანუ, ალერგენისადმი მგრძნობელობის დაქვეითებას). ამ მეთოდს მარტო გამოცდილი სპეციალისტი იყენებს. ყველა ექიმს არ შეუძლია ამის გაკეთება-ძირითადად ალერგოლოგები აკეთებენ.

და ბოლოს, როგორც ყოველთვის, მკურნალობამ შეიძლება შედეგი გამოიღოს და შეიძლება არა. რადგან რამოდენიმე საშუალება არსებობს, როგორც წესი, უმეტესობა ავადმყოფებისათვის მოიძებნება წამალი, რომელიც მოცემულ ავადმყოფს შველის.

MebenZinis dzma
QUOTE
რა პროცესმა და რომელ ორგანოტა სისტემაში (თუ არსებობს ასეთი ზოგადი მიდგომა), შეიძლება გამოიწვიოს დაბალი ტემპერატურა?
ზოგადად მასეთი მდგომარეობა არ არსებობს. თბორეგულაციას ორგანიზმში ბევრი პროცესი არეგულირებს. ტემპერატურის დაწევა (ან აწევა) დამოკიდებულია სითბოს წარმოშობასა და მისი "გაცემას" შორის ბალანსზე. თუ ნაკელბი რაოდენობით სითბო წარმოიშობა, ვიდრე გაიცემა, მაშინ ორგანიზმის ტემპერატურა კლებულობს.

მაგალითად, ცივ ამინდში (ან ცივ წყალში) დიდი ხანი ყოფნისას, სითბოს წარმოშობა (კანკალით) არ არის ისე სწრაფი, როგორც მისი გაცემაა გარემოში. თუ ფარისებული ჯირკვლის ჰორმონების უკმარისობაა, იგივე ხდება. ფარისებული ჯირკვლის ჰორმონები თითქმის ყველა უჯრედზე მოქმედებნ და მათ მეტაბოლიზმს ამაღლებენ, რაც გამოიხატება სითბოს გამოყოფითაც. თუ ამის ნაკლებობაა, მაშინ ორგანიზმის ტემპერატურა შეიძლება დაეცეს. ასევე, თირმკელზედა ჯირკვალიც არეგულირებს ამ პროცესს და მისმა დაავადებებმაც შეიძლება გამოიწვიოს დაბალი ტემპერატურა. ტვინში, ჰიპოფიზი და ფიპოთალამუსი აღიქვამს ორგანიზმის ტემპერატურას და მოკლე ფარგლებში რომ არეგულიროს, ორივე მექანიზმს (სითბოს წარმოშობასაც და მის გაცემასაც) ცვლის. თუ ამ ორგანეობის დაავადებაა, მაშინ თბორეგულაცია შეიძლება დაირღვეს.

პროცესების ამბავში, ეს შეიძლება იყოს ორგანოს ტრამვა, ანთება (რომელიც პრინციპში ტრამვასაც მოსდევს), ფიზიკური თუ ქიმიური დაზიანება (მაგალითად მაღალ ან დაბალ ტემპერატურაზე ექსპოზიცია, ტუტეებით და მჟავებით დაზიანება და ა.შ.), რადიაციული გამოსხივებით დაზიანება, ინფექცია და ა.შ. ნებისმიერ ამ პროცესს შეუძლია ორგანო დააზიანოს და ამგვარად თბორეგულაციის დარღვევა მოგცეს.

ეს არის მოკლე სია. საბოლოოდ ბევრი ორგანოს პროებლემას შეუძლია ეგ გამოიწვიოს. და ხანდახან კიდევ შეიძლება ნორმალურიც იყოს მოცემული ადამიანისათვის. თუ ვიღაცას დაბადებიდან 35გრადუსი ტემპერატურა აქვს და არაფერი აწუხებს, მაშინ ეს ნორმაა მისთვის.

Posted by: Archer 18 Jul 2008, 04:20
შეკითხვა გასტროენტეროლოგთან.
ადგილი აქვს გინგივიტს, რეფლუქ-ეზოფაგიტს, ეროზიულ გასტრიტს ph-ის მომატებისკენ ტენდენციით (სავარაუდოა რეფლუქსი), თორმეთგოჯა ნაწლავის პროქსიმალური ნაწილი ანთებას, ნაღვლის სადინარის დისკინეზიას, ქრ. ქოლეცისტიტი ჰოპომოტორული ფორმა, ღვიძლის დისფუნქცია (ცილოვანი ფრაქციის კერძოდ). ეს ყველაფერი ისე მსუბუქადაა, რომ სუბიექტურ სიმპტომებს არ იწვევს.
რა სახის მკურნალობას დანიშნავდით. რა ჯგუფის პრეპარატებით? ბალახებით?

Posted by: vano_t 18 Jul 2008, 08:37
Archer
QUOTE
რა სახის მკურნალობას დანიშნავდით. რა ჯგუფის პრეპარატებით? ბალახებით?
თუ ავადმყოფს სიმპტომები არ გააჩნია, რატომ უნდა უმკურნალო? მკურნალობ ავადმყოფს და არა ლაბორატორიულ მონაცემებს, ხომ ასეა? (ყოველთვის ასე არ არის, მარა ამ შემთხვევაში ასეა ალბათ)

კითხვა მაქვს, თუ შეიძლება: 1) რამდენი წლისაა ავადმყოფი? 2) რატომ ჩაუტარდა გასტროსკოპია და ღვიძლის ულტრაბგერა (ან კომპიუტერული ტომოგრაფია) თუ ავადმყოფს სიმპტომები არ აქვს? 3) რას ნიშნავს ღვიძლის დისფუნქცია ცილოვანი ფრაქციის მომატებით? რამხელაა ღვიძლის ენზიმები (ALT, AST), ბილირუბინი (პირდაპირი და არაპირდაპირი) და ალბუმინ და გლობულინი?

რეფლუქსი საერთოდ ძალიან ხშირია და უსიმპტომო რეფლუქსს არავითარი მკურნალობა არ ჭირდება. თუ ეზოფაგიტი აქვს ავადმყოს (რაც გასტროსკოპიაზე საყლაპავის ზედაპირის პირადპირი დათვალიერებით ჩანს) შეიძლება გაუკეთო ბიოპსოა, თუ ბარეტის ეზოფაგუსია გამოხატული (ანუ, საყალპავის ენდოთელიუმის მეტაპლაზია კუჭის ენდოთელიუმში). ბარეტის ეზიფაგუსს საყლაპავის კიბოს გაზრდილი რისკი აქვს და ამის გამო ხდება ბარეტზე დაკვირვება. თუ საჭიროა, PPI (proton pump inhibitors) შეიძლება დაუნიშნო ავადმყოფს. ამაში შედის "პრაზოლები".

ქრონიკული ქოლეცისტიტი მარტო თუ ულტრაბგერით დასვი (ნაღვლის ბუშტის კედლის გასქელება), არაფერი ჭირდება ავადმყოს. გინგივიტი, თუ სიმპტომურია (სიხლდენა ღძილებიდან ან ღრძილების ტკივილი) ალბათ პირის ღრუს ჰიგიენა უნდა და მეტი არაფერი. გინგივიტი მთავარია მწვავე ინფექციური არ იყოს, რასაც ანტიბიოტიკები ჭირდება; და მთავარია წამლისმიერი არ იყოს (მაგალითად, ფენიტოინს შეუძლია გამოიწვიოს), რასაც, თუ შესაძლებელია, წამლის შეწყვეტა შეიძლება დაჭირდეს.

Posted by: MebenZinis dzma 18 Jul 2008, 15:52
vano_t
ძალიან დიდი მადლობა, მაგრამ მე დაბალ ტემპერატურაში სულ სხვა რამ ვიგულისხმე sad.gif
ანუ, მე რაც ვიგულისხმე, არასწორად გამოვხატე. საუბარია, თუ არ ვცდები, სუფებრალიტეტი ჰქვია, სხეულის ტემპერატურა 37, 2 37,4 . მითხრეს, ასეთი ტემპერატურა შეიძლება გამოიწვიოს ქრონიკულმა ჩირქოვანმა ტონზილიტმა, ქრონიკულმა ქოლეცესტიტმა, კარიესმა და ტუბერკულოზმაო... ასევე, ასეთ ტემპარატურას უჩივიან თურმე ე.წ. აქცენტუირებული პიროვნებები (ლეონგარდის მიხედვით)...
თუ არ შეწუხდებით, ამ საკითხზე შემეხმიანეთ. ბოდიშს გიხდით, დრო რომ დაგაკარგინეთ.

Posted by: lidia 18 Jul 2008, 19:37
QUOTE
ძალიან დიდი მადლობა, მაგრამ მე დაბალ ტემპერატურაში სულ სხვა რამ ვიგულისხმე 
ანუ, მე რაც ვიგულისხმე, არასწორად გამოვხატე. საუბარია, თუ არ ვცდები, სუფებრალიტეტი ჰქვია, სხეულის ტემპერატურა 37, 2 37,4 . მითხრეს, ასეთი ტემპერატურა შეიძლება გამოიწვიოს ქრონიკულმა ჩირქოვანმა ტონზილიტმა, ქრონიკულმა ქოლეცესტიტმა, კარიესმა და ტუბერკულოზმაო... ასევე, ასეთ ტემპარატურას უჩივიან თურმე ე.წ. აქცენტუირებული პიროვნებები (ლეონგარდის მიხედვით)...
თუ არ შეწუხდებით, ამ საკითხზე შემეხმიანეთ. ბოდიშს გიხდით, დრო რომ დაგაკარგინეთ.

ტეხასური ჯლეტაააააააააააააააააააააა!
სასწრაფოდ წირდები ფორუმს. ესეთ რამეებში ტეხასური ჯლეტა სუპერია.
მე შემიძლია გითხრა, რომ სუბფებრილიტეტი შჰეიზლება ქრონიკულმა ინფექციებმა მოგცეს, პირველ რიგში ტუბერკულოზი უნდა გამოირიცხოს. შეიძლება სულაც ფარისებრი ჯირკვლის პრობლემების, ამანაც იცის სუბფებრილიტეტი. პირველ რიგში სისხლის საერთო ანალიზია გასაკეთებელი, მერე ექიმთან მიდი და ის დაგაკვალიანებს.

Posted by: Archer 18 Jul 2008, 20:31
vano_t
იმიტომ უნდა მკურნალობა, რომ დაავადებების პროგრესირება შემცირდეს. თორემ ახლა არ იწვევს ჩივილებს და 10 წლის შემდეგ? ან რად უნდა 10 წელი smile.gif
24 წლის ვარ.
ბილირუბინი, ალტ და ასტ ნორმაშია. ალბუმინების და გლობულინების თანაფარდობაა დარღვეული. კერძოდ ალბუმინების და ალფა-1 გლობულინების რაოდენობაა მცირედ მომატებული, ხოლო ალფა2 მკვეთრადაა შემცირებული და გამაც. კიდევ აღინიშნება 5-10მმ-ის რამოდენიმე კალციფიკატი. ჰეპატიტები არ მქონია არანაირი. ბავშვობაში ბრონქული ასთმის გამო გადასხმებს მიკეთებდნენ და შესაძლოა მაგის ბრალი იყოს. სმითაც ლოთი არ ვარ, მაგრამ რეგულარულად ვსვამ ნორმალურად.
საერთოდ ექიმთან იმიტომ მივედი, რომ პირში უცნაური ლითონის გემო და სუნი მაქვს ხოლმე (განსაკუთრებით დილით).
სტომატოლოგმა თქვა გინგივიტი ისე მცირედაა გამოხატული, რომ ალბათ მაინც გასტროეზოფაგური რეფლუქსის ბრალი უნდა იყოს გემოვო.
ნაღვლის ბუშტში შარშან ექოსკოპიით მესამედი ამოვსებული იყო სილით. მკურნალობის შემდეგ გავწმინდე.
PPI-ის რაც შეეხება მჟავიანობა ისედაც დაბლისკენაა და სუკრალფატი დამინიშნა ექიმმა მცირე დოზებით.
რაც თავი მახსოვს ჩემი ტემპერატურა თამაშობს 36,6-სა და 37,3-მდე. 37,5-ზე უკვე ცუდად ვარ.
ბარეტის ეზოფაგიტს რაც შეეხება... იმედი მაქვს smile.gif

Posted by: Guardian 18 Jul 2008, 20:55
Normal oral temperature in 99% of the population ranges from 36.0 to 37.7° C, with a circadian variation of 1° C or more between the morning nadir and the evening peak. Mean oral temperature in healthy adults is 36.8 ± 0.4° C, with women exhibiting slightly higher values (36.9 vs. 36.7° C).

Goldman: Cecil Medicine, 23rd ed. 2007.

Posted by: Solveig 18 Jul 2008, 21:01
QUOTE
ასეთ ტემპარატურას უჩივიან თურმე ე.წ. აქცენტუირებული პიროვნებები (ლეონგარდის მიხედვით)...

ეს რაღას ნიშნავს?

Posted by: Archer 18 Jul 2008, 21:29
Solveig
როგორც ფსიქიატრიის სახელმძღვანელოდან მახსოვს ფსიქოპათებსა და ნორმალურ ადამიანებს შორის მდგარი პირები არიან.

Posted by: სხვანაირა 18 Jul 2008, 22:11
დოქტორებო, ასეთი რაღაცა მჭირს ბოლო 3-4 თვეა, ზოგჯერ წამომივლის ტკივილები თირკმელების არეში და 1-2 წუთი გრძელდება. 1-2 წუთი კი ვუძლებ, მაგრამ მეტხანს რომ გაგრძელდეს (რაც ჯერ არ მომხდარა), შეიძლება კედლებზე ავიდე. ეს მემართება სადღაც 2-3 კვირაში ერთხელ. რამე საშიში ხომ არ შეიძლება იყოს?
* * *
Archer
QUOTE
ნაღვლის ბუშტში შარშან ექოსკოპიით მესამედი ამოვსებული იყო სილით.

მშვილდოსანო, ახალგაზრდა ადამიანები რომ უჩივიან ნაღველს, სულ ერთი რამე მინდა ვკითხო და შენც გკითხავ: "სკამზე" ცხოვრობ? smile.gif

მე ვცხოვრობ და მაშინებს ქვიშა-ქვების პერსპექტივა, იმიტომ გეკითხები...

Posted by: Archer 18 Jul 2008, 22:34
სხვანაირა
რა ვიცი რამოდენიმე საათი დღეში ვზივარ კი. მაგრამ დავდივარ კიდეც.

Posted by: texasuri jleta benzoxerxit 19 Jul 2008, 01:54
Archer
PPI-დაბალი მჟავიანობის გამო არ დავნისნავდი,თუმცა არის მათ რიგებში პრეპარატები,რომლებიც PH-ის ნორმულ ან დაბალ მაჩვენებელზე არ "ირთვებიან",ასე,რომ ჰელიკობაქტერიის ერადიკაციის სქემაში მათი ჩასმა შეიზლება.სუკრალფარი და სხვა შემომგარსავი საშუალებები არაჩვეულებრივია.ქოლეცისტიტისთვის-რაფაქოლინ-ცე ან დისკინებილი+რამე ჰეპატოპროტექტორი ღვიძლისთვის.მიუხედავა დ იმისა,რომ ჰეპატიტები არ გქონია,ვირუსული ჰაპატიტების მარკერები მაინც გადაამოწმე,განსაკუთრებით-C.
+რა თქმა უნდა დიეტა.
ლითონის გემო ხშირად იცის საძილე საშუალება სომნოლმა(ზოპიკლონი)ამას ხომ არ სვამ?
MebenZinis dzma
აბსოლუტურად სწორი დიფერენციალდიაგნოზური სვლებია,კარიესი,ქრ.ქოლეცისტიტ,TBC,ქრ.ჩირქოვანი ტონზილიტი,ჰიპერთირეოზი.სულ ბოლოს შეიძლება დაისვას "ნევროზის"დიაგნოზი(როგორც ასთენო-იპოქონდრიული სინდრომი,შემავალი სომატოფორმული აშლილობის ან სომატიზირებული დეპრესიის სტრუქტურაში)lidia
მადლობა ნდობისთვის,თუმცა ვანო აქ უჩემოდაც მშვენივრად გასცემს პასუხს

Posted by: Archer 19 Jul 2008, 02:24
texasuri jleta benzoxerxit
ვირუსებზე 2-ჯერ გავიკეთე ანალიზი. ადრე ვსვამდი ხანდახან სომნოლს, მაგრამ უფრო მწარე გემო მქონდა. ალბათ ნერწყვში ხდება ექსკრეცია.

Posted by: Natuka NGN 19 Jul 2008, 02:35
Archer
QUOTE
ადრე ვსვამდი ხანდახან სომნოლს

რას ქვია სვავდი? ექიმმა გამოგიწერა თუ შენით? საშინელი პრეპარატია!!!
მე მაპატიეთ ა ერთი რცევა - ექიმის გარეშე წამლებს ნუ იღებთ ხალხო... sad.gif

Posted by: Archer 19 Jul 2008, 02:43
Natuka NGN
საშინელი რა. დოზა უნდა იცოდე და ზომა smile.gif
მე თვითონ ვარ ექიმი

Posted by: Natuka NGN 19 Jul 2008, 03:00
Archer
რა ვიცი ჩემო კარგო. ჩემმა დეიდაშვილმა მიიღო და სულ ერთი კვირის მიღების შემდგე კუჭის 6 თვანი მკურნალობა დასჭირდა. თორემ, მე პირადად არასოდეს მიმიღია.
smile.gif

Posted by: Archer 19 Jul 2008, 03:27
Natuka NGN
რა დამართა კუჭზე?

Posted by: texasuri jleta benzoxerxit 19 Jul 2008, 14:02
Natuka NGN
მასე კუჭის გაღიზიანება არასპეციფიკურად ნებისმიერ პრეპარატს შეიძლია,თვით გასტროპროტექტორებსაც. biggrin.gif კუჭს ყველაზე ხშირად აღიზიანებს აასს-ები,სტეროიდები,კალიუმის პრეპარატები,რკინის პრეპარატები,ბეტაჰისტინი(ბეტასერკი,მიკროზერი),ნოოტროპები და თავის ტვინის მიკროცირკულაციის გამაუმჯობებსაბეი პრეპარატები და ა.შ

Posted by: Natuka NGN 19 Jul 2008, 14:18
texasuri jleta benzoxerxit
არ ვიცი ჩემო კარგო, კუჭის გაღიზიანება ყველაფერს შუძლია. ისე, მაგ წამალმა, ძილის მოსაწესრიგებლად მისცეს /54 წლის ქალია/. არადა, პირში სიმწარით რომ დაეწყო, მერე საშინელ ტკივილებშI გადაეზარდა. არ გამოვრიცხავ, რაიმე აწუხებდა, მაგრამ ამან ბიძგი მისცა, ან გააღიზიანა.
ჰო, ისე ის ვიუსიდი, ძალიან მაგარი პრეპარატია მართლა, პერპარატიც არაა, საჭმლის დამატებად ითვლება. თითქმის ყველას ვურჩევ, მაგრამ ისეთი ძვირია biggrin.gif ისე, ცოდვას ხომ არ ჩავდივარ ამ წამლის არსებობაზე ჩემს საახლობლოს რომ ვაუწყებ?

Posted by: Tornike Alashvili 19 Jul 2008, 17:56
Natuka NGN მედიცინის შესწავლა დაიწყე თუ ექიმებზე ფსიქოლოგიურ გაკვირვებას აწარმოებuser posted image

Posted by: mtvareuli 19 Jul 2008, 18:30
vano_t
QUOTE
ზუსტად არ მახსოვს რას ეძახიან ჭინჭრის ციებას

ურტიკარიას yes.gif

Posted by: MebenZinis dzma 19 Jul 2008, 18:43
Solveig
აქცენტუირებულ პიროვნებებზე მეც აქ წავაწყდი ინფოს. მერე დავგუგლე და წავიკითხე, მაგრამ ჩემს შემთხვევასთან კავშირი ვერ დავძებნე. ვიფიქრე, ეგებ აქიმებმა რამე იციან-თქო...
* * *
texasuri jleta benzoxerxit

ბოლოს შეიძლება დაისვას "ნევროზის"დიაგნოზი(როგორც ასთენო-იპოქონდრიული სინდრომი,შემავალი სომატოფორმული აშლილობის ან სომატიზირებული დეპრესიის სტრუქტურაში)

ეს ცოტა პოპულარული ენითაც რომ განმარტოთ... გმადლობ, თუმცა ცოტა შემეშინდა sad.gif

Posted by: Tornike Alashvili 19 Jul 2008, 19:15
QUOTE (ტანკე @ 25 Jun 2008, 22:05 )
გაუმარჯოს ხალხს
ერთი კითხვა მაქვს - MRI ან MRT-თი მთლიანად ორგანიზმის სკანირება ხდება,თუ მარტო იმ ორგანოსი,რომელსაც მიუთითებ? ანუ თუ მთელი ორგანიზმის სკანირება გინდა,ეს უფრო ბევრი ჯდება? თუ თავზე იკეთებ MRT-ს,მაშინ მარტო თავს ასკანირებენ? და თუ კონკრეტულად თავის ტვინის სისხლძარღვებზე ხდება დაკვირვება(პაციენტის ექიმთან მისვლის მიზეზი),იქ თავის ტვინის ყველა ანომალიასაც გამოავლენენ,თუ ასეთი რამ არსებობს? მაგალითად,სიმსივნე,ალცჰეიმერი და სხვ.
წინასწარ დიდი მადლობა

არი სუსტი (ოპენი)0,5 და ძლიერი (ავანტო) მრტ –ეს თოდუას კლინიკის აპარატებში –რა თქმა უნდა ოპენი უფრო იაფია(თავის თვინი–250 ლარი) თუ სისხლძარღვები გინდათ 20–30 ლარით მეტია
ავანტა –კიდევ მეტი და თუ მეტი კეთდება –უფრო ძვირია.თავიდან ფეხებამდე თეორიულად–შეიძლება მარა პრაქტიკულად არ კეთდება
ხერხემალი –450 ლარი ღირს ,ცალკე წელის მალები ან კისრის ან გულმკერდის–230

Posted by: texasuri jleta benzoxerxit 20 Jul 2008, 01:11
Natuka NGN
ცოდავ.ეგ ჩვენი საქმეა
MebenZinis dzma
პოპულარულ ენაზე მიწერია-ნევროზი.ძალიან კარგი თუ შეგეშინდა,ნევროზი არ არის უწყინარი დაავადება

Posted by: Natuka NGN 20 Jul 2008, 01:56
texasuri jleta benzoxerxit
ბოდიში ბატონო ექიმო, არ ვიჭრები თქვენს კომპეტენციაში, მაგრამ აი ამ ეტქსტმა გადამაწყვეტინა: Все ортомолекулярные препараты, представленные на сайте ortho.ru, зарегистрированы в виде биологически активных добавок к пище. Формально для них не нужно рецепта врача, и любой человек вправе сам принять решение на их приобретение и употребление..
ეს რჩევაც არაა, უბრალოდ ვამცნობ მსგავსი საკვების დანამატის არსებობას smile.gif
ჰოო, აგერ თორნიკე კი მიცნობს და ისიც იცის როგორი წინააღმდეგი ვარ საერთოდ ყველანაირი ზედმეტი ფარმაკოლოგიური ჩარევის ადამიანის ორგანიზმზე smile.gif და მით უმეტეს ზედემტს არასოდეს ვცოდავ smile.gif
Tornike Alashvili
როგორ ხარ თოთო? დაკვირვებას? smile.gif დამზერას... არა, არც ერთი და არც მეორე, უბრალოდ შემოვიჭყიტები და თუ რაიმე ვიცი მარტივ რჩევას ვაძლევ ადამიანს smile.gif
აი, ისეთს, მაგალითად, ბერალგამინის მიღება სრულიადაც არ გინდათქო ანდა როგორ ივარჯიშონ რომ ადვილად გახდნენ და ა.შ. smile.gif

Posted by: texasuri jleta benzoxerxit 20 Jul 2008, 02:08
Natuka NGN
ჩვენ ყველა საკმარისზე მეტს ვცოდავთ.საკვები დანამატებიც ჯობია ექიმთან შეთანხმდეს.ჯობია კი არა,აუცილებელია.
ფარმაკოთერაპია,ადექვატურ დოზებში და სწორად დანიშნული,თუნდაც ბევრი წამალი იყოს ერთდროულად,ასევე გამართლებულია და ავადმყოფის გემოვნების მიუხედავად,ჩვენი საქმეა

Posted by: MebenZinis dzma 20 Jul 2008, 02:21
texasuri jleta benzoxerxit
ჰაიტ, მალადეც ჰაქიმო...

Posted by: Natuka NGN 20 Jul 2008, 14:02
texasuri jleta benzoxerxit
საკვებ დანამატებზე, მე ინეტის ინფოს ვეყრდნობი. ცხოვრებაში პირველად მომიწია მსგავსი რამის მიღება და უფრო ღრმად შევისწავლი მაგ საკითს smile.gif
აი, ცოდვებზე, კი, მართალია, ადამიანები იმაზე მეტს სცოდავენ, ვიდრე საჭიროა...
QUOTE
ფარმაკოთერაპია,ადექვატურ დოზებში და სწორად დანიშნული,თუნდაც ბევრი წამალი იყოს ერთდროულად,ასევე გამართლებულია და ავადმყოფის გემოვნების მიუხედავად,ჩვენი საქმეა

ამაშიაც გეთანხმები, სრულიად!!! მაგრამ არის შემთხვევები, როდედაც ექიმი აშკარა შეცდომას უშვებს და მე ამ შემთხვევებზე გეუბნები. უბარლოდ, ცოტა უკეთ რომ მიცნობდე, ალბათ მსგავს კომენტარს არ გააკეთებდი. ადრე მაქვს დადებული ეს ისტორია და ახლაც დავწერ, არ მეზარება. მე მსგავს შემთხვევებზე ვამბობ და არა იმ შემთხვევაზე, როცა მართლაც აუცილებლად საჭიროა წამლის მიღება და სხვა გამოსავალი არაა. ჰო, არ ვიცი სხვა თემაში დავდე და აქაც ვიტყვი, ვარ ფილოსოფიურ მეცნიერებათა დოქტორი და ჩემს დროს ფილოსოფიის ფაკულტეტზე პირველ ორ კურსზე მხოლოდ საბუნებისმეტყველო მეცნიერებებს გვასწავლიდნენ, მათ შორის ადამიანის ფიზიოლოგიას, გენეტიკას, ფიზიკას და ა.შ., რომ დაგვემტკიცებინა მატერიის პირველადობა smile.gif ანუ, მატერიალისტებად გვზრდიდნენ biggrin.gif ჰოდა, აივენგო შათირიშვილთან 5 მყავდა. თავად ბიოლოგებს უკვირდათ. ყველას ბოდიში, თვითრეკლამისათვის ოფისათვის...
აი, ახლა ის ისტორია. ჩემი ძმისშვილი 4 თვის იყო და მხოლოდ ძუძუს გეახლებოდათ. ჰოდა, ჩემმა რძალმა ერთ დღესაც კაი გემრიელი ღორის სალები გეახლათ და ბავშვსაც მუცელი აერია. მანამდე, ახალდაბადებულსაც კი არ ქონია რაიმე პრობლემა. გავიქეცით პედიატრთან. პედიატრმა დედა იაშვილის კლინიკაში გაამწესა, რძის სინჯების გასაკეთებლად, თუ ცხიმის როგორი შემცველობა იყო მასში და ბალღს რაც მახსოვს გამოუწერა: მეზიმფორტე, ლინექსი, ბეიბინოსი, ფესტალი და კიდევ რაღაც არ შემეშალოს, 8 წლის წინ იყო. აუუ, შენ უნდა გენახა ბავშვი რა დღეში ჩავარდა - აქამდე წამალი კი არა წყალიც არ ქონდა მიღებული, ვერც სხვას ევრაფერს ვაძლევდით გარდა ძუძუსი, გვიფურთხებდა. ჩემი რძალი წავიდა იაშვილში და დაუდგინეს: ცხიმის დიდი ბურთულები უფრო მეტია შენს რძეში და ამის მიცემა ბავშვისათვსი არ შაიძლებაო biggrin.gif დიეტაზე დაჯექიო, არ მიიღო არანაირი ცხიმიო და ა.შ. და + კვების რაღაც ნაწლი ხელოვნურით ჩაანაცვლეო. აუუ, აქ უნდა გენახა ჩვენი პატარას პროტესტი. სოსკა და ხელოვნიური არ გაიკარა, შიმშილით კვდებოდა, მაგრამ მაინც!!! ჰოდა, 4 დღე რომ ვუყურებდი ამათ ტანჯვას /ბავშვისა და რძლის - ორივე მშიერი იყო/, მოვკიდე ორივეს ხელი და გავიქეცი ჩემს ექიმთან. არ დავწერ მის სახელს, არ მიდნა ზედმეტი რეკლამა, თუმცა ერთადერთი ექიმია, რომლის სიტყვა ჩემთვის საბოლოო ინსტანციაა!!! ჰოდა, ჩემმა ექიმმა რომ მოისმინა ეს ყველაფერი, ასე თქვა მოდი არაფერი არ მივცეთ ბავშვს არაფერი უნდაო /კუჭის არ გამოსწორებულა ამ წამლების მიღებსი ფონზე, არ იფიქრო + პატარს ნევროზიც დაეწყო, სულ ტიროდა და შიოდა/ და დედამისს კი უთხრა, შენ ჭამე ყველაფერი, კარაქიც, ხორციც /ოღონდ არა ღორის და სალაო/ და მიიღე სამჯერ იმაზე მეტი ჩვეულებრივი წყალი, ვიდრე სვავო. ანდა, მიახლოვებით მოდი გავთვალოთ, ქაბატო დღეში რამდენ რძეს გეახლებაო. ეს ცოტა რთული აღმოჩნდა, იმიტომ რომ ძუძუზე იყო და დროც განსაზღვრული არა ქონდა. ნუ, მიახლოვებით გავთვალეთ - სადღაც 4 ლიტრა წყალი მაინც უნდა დაელია პირველ ეტაპზე /ამაში არ ითვლებოდა საკვებში შემავალი წყალი: ჩაი, სუპები და ა.შ./. გადავედით ისევ ჩევულ რეჟიმზე, გარდა ერთისა - დედა მეტ წყალს სვავდა. ჰოდა, მეორე დღეს ბავშვს კუჭი ნორმაში ქონდა, დამშვიდდა ისევ და ხასიათიც დაუბრუნდა. რომ მახსენდება მარტო წამლის მიღებაზე რა პროეტსტი ქონდა, როგორ აფურთხებდა და ტიროდა, ახლაც გული მეკუმშება. მის მერე მუცელს აღარ შეუწუხებია. კიდევ მეტს გეტყვი, ძუძუს მეტი 9 თვემდე ბავშვს არაფერი მიუღია, წყალიც კი. არ უნდოდა!!! ძუძუს კი თავი 3 წლის ასაკში ძლივს დაანებებინეს /გოგო მაინ არ იყოს biggrin.gif/ და პირველი სხვა საკვები ქათმის ხორცი გახლდათ /ამ დროისათვის უკვე 8 კბილი ქონდა/. არცერთი დამატებითი სხვა პრეპარატი არ მიუღია. უბნის პედიატრი გვეუბნებოდა - D ვიტამინი ხომ არ დავნიშნოო... მაგიც გადმოგვიფურთხა და აღარ მივეცით smile.gif
აი, მსგავს შემთხვევაზე გეუბნები და ვერ დამარწმუნებ, რომ იმ 4 თუ 5 წამლის დანიშვნა ამ შემთხვევაში საჭირო იყო, ან დედის რძის ხელოვნური საკვებით ჩანაცვლება.
ხოლო, აი, ყოველთვის ვეწინააღმდეგები, როდესაც უბრალოდ გაცივდება ადამიანი, გარბის და ამპიცილინს ყიდულობს /მარტივი ანტიბიოტიკიაო, რას ნიშნავს მარტივი არ ვიცი/, ხოლო შემდეგ ღმერთმა დაუფაროს და მართლაც დასჭირდეს ანტიბიოტიკოთერაპია, უკვე მგრძნობელობა ძალიან ცოტა პრეპარატზე აქვს!!! ანდა მუცელი ასტკივდება და სულფიდები /ბისეპტოლი/ მომარჯვებული აქვს!!! + ლევომიცეტინთან ერთად /ერთ-ერთ ჩერმს ახლობელს დაუნიშნა ექიმმა/.
ჰოო, შეიძლება აქაც ვცოდავ, მაგრამ როდესაც ვინმეს ჩემს ოჯახში მუცელი ასტკივდება, ელექტროლიტები მომარჯვებული მაქვს მხოლოდ!!! დანარჩენი პაატასთან შეთანხმებით.
ხოლო, რაც შეეხება ჩემს ბოლოდროინდელ მოწამვლას, ეს ცოტა განსხვავებული რამ იყო, არასტანდარტული!!! smile.gif და ზუსტი დიაგნოზი დამისვი ინეტით.
მგონი თავი გაგაბეზრეთ, ბოდიში ყველას. დანარჩენ ისტორიებს, სხვა თემებში დავწერ smile.gif და სხვა დროს....

Posted by: tikope 20 Jul 2008, 16:44
Natuka NGN
აი ეს მესმის,ბარავო
QUOTE
ხოლო, აი, ყოველთვის ვეწინააღმდეგები, როდესაც უბრალოდ გაცივდება ადამიანი, გარბის და ამპიცილინს ყიდულობს /მარტივი ანტიბიოტიკიაო, რას ნიშნავს მარტივი არ ვიცი/, ხოლო შემდეგ ღმერთმა დაუფაროს და მართლაც დასჭირდეს ანტიბიოტიკოთერაპია, უკვე მგრძნობელობა ძალიან ცოტა პრეპარატზე აქვს!!!

+1
და ასევე მაკვირვებს , რომ უბრალო ამპიცილინს აღარ ნიშნავენ, ეგრევე ძლიერ ანტიბიოტიკებზე გადადიან.
ბავშვს , რომელსაც არასოდეს არ ჰქონდა არანაირი ცამალი გასინჯული 6 თვემდე დაუნიშნეს ზოვირაქსი, დაახლოებიტ 5 წლის წინ, ახლა კიდევ უფრო ძლიერებია შემოსული......

Posted by: Natuka NGN 20 Jul 2008, 17:27
tikope
ზოვირაქსი ანტიბიოტიკი არაა, აციკლოვირის ბრიტანული ნაირსახეობაა და შესაბამისად ანტივირუსული პრეპარატია. ან სახელი გეშლება. და რატომ დაუნიშნეს, რომელი ვირუსის გამო? ჰერპესი ქონდა? ძალიან ქონდა ეს ჰერპესი გამოვლენილი, რომ პირდაპირ ზოვირაქსის აბებს ეცნენ? ან სად გაასინჯინე. მაგრად მაინტერესებს.
აი, მაგაზე ისეთი ისტორიები შემიძლია მოვყვე biggrin.gif უმმაგრესი... და ანტიბიოტიკოტერაპიაზეც მოვყვები, მაგრამ მგონი ამისათვის, რაღაც თემა იყო, სასაცილო სიტუაციები მედიცინასთან თუ რაღაც მსგავსი და იქ ჯობია smile.gif

Posted by: tikope 20 Jul 2008, 20:02
Natuka NGN
[quote]ზოვირაქსი ანტიბიოტიკი არაა, აციკლოვირის ბრიტანული ნაირსახეობაა და შესაბამისად ანტივირუსული პრეპარატია. ან სახელი გეშლება. და რატომ დაუნიშნეს, რომელი ვირუსის გამო? ჰერპესი ქონდა? ძალიან ქონდა ეს ჰერპესი გამოვლენილი, რომ პირდაპირ ზოვირაქსის აბებს ეცნენ? ან სად გაასინჯინე. მაგრად მაინტერესებს. აი, მაგაზე ისეთი ისტორიები შემიძლია მოვყვეუმმაგრესი... და ანტიბიოტიკოტერაპიაზეც მოვყვები, მაგრამ მგონი ამისათვის, რაღაც თემა იყო, სასაცილო სიტუაციები მედიცინასთან თუ რაღაც მსგავსი და იქ ჯობია
უი, ახლა გადავავლე ჩემს პოსტს თვალი და ზოვირაქსი კი არა და ზინატი, დიდი ბოდიში
biggrin.gif

Posted by: Natuka NGN 20 Jul 2008, 21:17
tikope
მივხვდი, რომ შეცდომა იყო. ბოდიშ რაზე მიხდი? wink.gif საქმე რაშია იცი? მთავარი ის კი არაა, მსგავს სინთეზურ, ფართოსპექტრის მეორე თაობის ანტიბიოტიკებს ასე ადვილად რომ ნიშნავენ, საქმე იმაშია, რომ ამას ყოველგვარი წინასწარი მგრძნობელობის შემოწმების გარეშე აკეთებენ. იქნებ სულ არ მაქვს ეს ოხერი მგრძნობელობა კონკრეტულ ანტიბიოტიკზე. უბრალოდ, ისევ ამ წამლის სიძლიერეს ეყრდნობიან და ან ''არტყავენ'' ან ვერა. აქ გამართლებაც შიეძლება იყოს - მკურნალობა სასწრაფოდ არის ჩასატარებელი და სად დაველოდო ამოტესვის პასუხსო. კეთილი, ბატონო, ეს გადაუდებელ შემთხვევებში, მაგრამ როდესაც 3 დღის მოცდა შეიძლება, აი ამის არგაკეთება შეცდომად /რბილად ვამბობ/ მიმაჩნია!!! და განსაკუთრებით ახლა, როდესაც ისევ შემოგვიტია ტუპერკულიოზმა კოხის ჩხირით, რომლის ახალი შტამი მეტად მდგრადია წამლების მიმართ და ეს ზედმეტი მდგრადობა, არა თქვა და სწორედაც ამ ანტიბიოტიკების ''უპრაკუნო'' დანიშვნის და მიღების შედეგია.
ასე რომ...

Posted by: Tornike Alashvili 20 Jul 2008, 22:33
tikope



ნატუკა კი მაგარია მარა არამგონია ასეთი შეცდომები აქიმებმა შეგარჩინონ–ზოვირაქსი ანტიბიოტიკი არ არის შედარებით უსაფრთხო ანტივირუსული პრეპარატია.მაგრამ იმაში რომ ბოლო თაობების ანტიბიოტიკებს –სცორედ ზინატს და სხვებს ნიშნავენ მაშინ როდესაც საერთოდ არ არის ნაჩვენები ანტიბიოტიკი –ნამდვილად მართალი ბრძანდებით.ხდება გიპერდიაგნისტიკაც –სასტიკი მკურნალობა რომ გააართლონ.მოკლედ რაც დრო გადის მკურნალობა სულ უფრო საშიში და სარისკო ხდება –განსაკუთრებით ბავშვებისთვის.ოჯახის ექიმის ინსტიტუტის შემოღება ამას სულ უფრო არასახარბიელოს გახდის–პედიატრებს არ შეუძლიათ უბრალო ბრონქიტების მკურნალობა ანტიბიოტიკების გარეშე და თერაპევტი გადაკვალიფიცირებული რას იზამს?

Posted by: tikope 21 Jul 2008, 07:49
Tornike Alashvili
QUOTE
ზოვირაქსი ანტიბიოტიკი არ არის შედარებით უსაფრთხო ანტივირუსული პრეპარატია.მაგრამ იმაში რომ ბოლო თაობების ანტიბიოტიკებს –სცორედ ზინატს

დიდი მადლობა გამოხმაურებისთვის , მაგრამ ეს უბრალოდ ჩემი შეცდომა იყო, ზინატის მაგივრად ზოვირაქსი დავწერე biggrin.gif ,ორივე მიხმარია და ვიცი რაც არის,.არავის შეცდომაში შეყვანა არ მიფიქრია.

QUOTE
.ხდება გიპერდიაგნისტიკაც –სასტიკი მკურნალობა რომ გააართლონ.მოკლედ რაც დრო გადის მკურნალობა სულ უფრო საშიში და სარისკო ხდება –განსაკუთრებით ბავშვებისთვის.ოჯახის ექიმის ინსტიტუტის შემოღება ამას სულ უფრო არასახარბიელოს გახდის–პედიატრებს არ შეუძლიათ უბრალო ბრონქიტების მკურნალობა ანტიბიოტიკების გარეშე და თერაპევტი გადაკვალიფიცირებული რას იზამს?

yes.gif yes.gif გეთანხმებით

Posted by: lolita55 22 Jul 2008, 10:54
მიშველეთ გეხვეწებით.
მეოთხე დღეა უკვე გადაბმულად მტკივა ნაწლავები ყრუდ. ზოგჯერ ძლიერადაც და თან ბუყბუყებს. გუშინ ვიყავი ექიმთან და მითხრა გაზებია და თავისით გაგივლისო, არაფერი არ გამომიწერა. მე კიდე არ მივლის, წუხელ მთელი ღამე მტკიოდა და ახლაც მტკივა, რა გავაკეთო ან რამე წამალი არსებობს რომ დავლიო? cry.gif sad.gif

Posted by: Natuka NGN 22 Jul 2008, 11:20
lolita55
გაწყინა რამემ? შეიძლება რაღაც საკვები მიიღე და აგრია. ისე, კი არის ე.წ. აქტიური ნახშირი, მაგრამ რამდენად ეფექტურია ვერ გეტყვი, არასოდეს გამომიცდია smile.gif უბრალოდ ერთგვარ დიეტაზე გადადი - თხევადი საკვები მიიღე ბევრი ზეთზე დამზადებული. ხილი ჭამე და მინერალური მიაყოლე. ეტყობა მართლაც სერიოზული არაფერი გაქვს, თორემ ექიმი რამეს დაგინიშნავდა. კვების რეჟიმი მოაწესრიგე.

Posted by: lolita55 22 Jul 2008, 11:52
QUOTE
lolita55
გაწყინა რამემ? შეიძლება რაღაც საკვები მიიღე და აგრია. ისე, კი არის ე.წ. აქტიური ნახშირი, მაგრამ რამდენად ეფექტურია ვერ გეტყვი, არასოდეს გამომიცდია  უბრალოდ ერთგვარ დიეტაზე გადადი - თხევადი საკვები მიიღე ბევრი ზეთზე დამზადებული. ხილი ჭამე და მინერალური მიაყოლე. ეტყობა მართლაც სერიოზული არაფერი გაქვს, თორემ ექიმი რამეს დაგინიშნავდა. კვების რეჟიმი მოაწესრიგე.

ხო, სერიოზული არაფერიაო მაგრამ მტკივა და ცუდად ვარ. რა ვიცი რა ვჭამე არ მახსოვს მაგრამ ამდენი დღე რატომ არ მივლის?!
მინერალური წყალი ხომ გაზიანია? ეგ რო დავლიო უფრო არ დამიგროვებს გაზებს?

Posted by: Tornike Alashvili 22 Jul 2008, 15:10
lolita55
ცოტა გაუშვი გაზი და ისე დალიე მინერალურ წყლები.და რაც მთავარია გააკეთე ექოსკოპია ღვიძლის და ნაღვლის ბუშტის –ინტერკლინიკაში ან ჯანმრთელობის სახლში

Posted by: iza 22 Jul 2008, 18:22
მაინტერესებს სისხლში ლეპტინისა და ლიპიდების დონე. რა ანალიზი უნდა გავიკეთო ზემოაღნიშნულის დასადგენად, კონკრეტულად რა ჰქვია ამ ანალიზს და სად გავიკეთო? ან რა ღირს აღნიშნული ანალიზის გაკეთება?

Posted by: lidia 22 Jul 2008, 18:41
QUOTE
მაინტერესებს სისხლში ლეპტინისა და ლიპიდების დონე. რა ანალიზი უნდა გავიკეთო ზემოაღნიშნულის დასადგენად, კონკრეტულად რა ჰქვია ამ ანალიზს და სად გავიკეთო? ან რა ღირს აღნიშნული ანალიზის გაკეთება?

ლიპიდების გამოკვლევა თი გინდა ლიპიდური სპექტრი უნდა განისაზღვროს, ანუ აქ შედის საერთო ქოლესტეროლი, ტრიგლიცერიდები, მაღალი და დაბალი სიმკვრივის ლიპოპროტეინები. ეს გამოკვლევა თბილისში ბევრ ადგილას კეთდება

Posted by: Tornike Alashvili 22 Jul 2008, 23:49
ანეგდიტი–––
–ექიმო მე სტაფილოკოკი მჭირს
–შენ სტაფილიკოკი კი არა ვირუსი გჭირს.
და უნიშნავს ანტიბიოტიკს
აი ესაა დღევანდელი მედიცინა

Posted by: vano_t 23 Jul 2008, 07:50
Natuka NGN
QUOTE
საქმე რაშია იცი? მთავარი ის კი არაა, მსგავს სინთეზურ, ფართოსპექტრის მეორე თაობის ანტიბიოტიკებს ასე ადვილად რომ ნიშნავენ, საქმე იმაშია, რომ ამას ყოველგვარი წინასწარი მგრძნობელობის შემოწმების გარეშე აკეთებენ. იქნებ სულ არ მაქვს ეს ოხერი მგრძნობელობა კონკრეტულ ანტიბიოტიკზე. უბრალოდ, ისევ ამ წამლის სიძლიერეს ეყრდნობიან და ან ''არტყავენ'' ან ვერა.  აქ გამართლებაც შიეძლება იყოს - მკურნალობა სასწრაფოდ არის ჩასატარებელი და სად დაველოდო ამოტესვის პასუხსო. კეთილი, ბატონო, ეს გადაუდებელ შემთხვევებში, მაგრამ როდესაც 3 დღის მოცდა შეიძლება, აი ამის არგაკეთება შეცდომად /რბილად ვამბობ/ მიმაჩნია!!! და განსაკუთრებით ახლა, როდესაც ისევ შემოგვიტია ტუპერკულიოზმა კოხის ჩხირით, რომლის ახალი შტამი მეტად მდგრადია წამლების მიმართ და ეს ზედმეტი მდგრადობა, არა თქვა და სწორედაც ამ ანტიბიოტიკების ''უპრაკუნო'' დანიშვნის და მიღების შედეგია.
ასე რომ...
თუ ინფექციის დიაგნოზი დაისვა, საერთოდ ანტიბიოტიკს მაშინვე იწყებენ. თუ რაიმე სითხეს (სისხლს, ჩირქს ან რაიმე ორგანოდან გამონადენს) ან ქსოვილს აგზავნი დასათესად, კარგია რა თქმა უნდა. საქართველოში არ არის ამის შესაძლებლობა-გერმანიაში ან სხვა ქვეყნებში აგზავნიან საანალიზოდ. ძალიან არაპრაქტიკულია ასე. პასუხს დიდი დრო ჭირდება. საერთოდ, ნორმალურ ქვეყნებში, ინფექციური დაავადებების სტატისტიკა დგინდება. ასეთი სტატისტიკის მიზანია სხვადასხვა ორგანოს ინფექციების ხშირი გამომწვევი დაადგინონ და ამ გამომწვევთა მგრძნობელობა შეისწავლონ. როცა ეს მონაცემები გაქვს, მაშინ შეგიძლია დაიწყო მკურნალობა და არც გააგზავნო რაიმე დასათესად. მაგალითად, თუ პნევმონიის დიაგნოზს აკეთებ და იცი, რომ გამომწვევების 95 %-ზე უმეტესობა შენს ქვეყანაში არის პნევმოკოკი, რომელიც რომელიღაც ანტიბიოტიკ(ებ)ისადმი არის მგრძნობიარე, მაშინ შეგიძლია პირდაპირ ეს ანტიბიოტიკი დაუნიშნო, თუ ავადმყოფს სერიოზული ინფექცია არ ჭირს. თუ სეპსისით შემოდის, მაშინ რაც შეიძლება მეტი სითხე (შარდი, სისხლი, ნახველი და ა.შ.) უნდა გააგზავნო დიაგნოზის გასაკეთებლად და ყველანაირი შესაძლო გამომწვევი უნდა დაფარო (გრამ ნეგატიური, გრამ პოზიტიური და ანაერობული ბაქტერიები-გამძლე შტამების ჩათვლით: მაგალითად, მეთიცილინ-რეზისტენტული ოქროსფერი სტაფილოკოკი, სანამ პასუხი მოგივა. ეს არის მოკლე პრინციპი. სინამდვილეში, ანტიბიოტიკის არჩევისას ინფექციონისტი ძალიან ბევრ ფაქტორს იღებს მხედველობაში, ავადმყოფის სპეციფიური პრობლემების ჩათვლით.

Posted by: Natuka NGN 23 Jul 2008, 13:14
vano_t
ვანო ექიმო, სანამ შენს პოსტს ვუპასუხებ, მანამდე თხოვნა მაქვს, გვერდით თემაში აინტერესებთ კომიანებს რა საკვებს აძლევენ გარდა იმ საკვებისა, რაც დაწერეს. იქნებ დააკვალიანო. ეს თხოვნა. ახლა კი პასუი.
QUOTE
საქართველოში არ არის ამის შესაძლებლობა-გერმანიაში ან სხვა ქვეყნებში აგზავნიან საანალიზოდ. ძალიან არაპრაქტიკულია ასე. პასუხს დიდი დრო ჭირდება

და ეს ვინ თვქა რომ არ არის? და ბაქტეროოფაგი რას აკეთებს ან იგივე ''ტეტსი'' სადაც მუდმივად დავდივარ?
მე ხაზგასმით აღვნიშნე ჩემს პოსტში - შემთხვევები, როდესაც გადაუდებლად არის საჭირო ანტიბიოტიკოტერაპია, უნდა დაინიშნოს მეთქი, მაგრამ არის შემთხვევები, როდესაც ამის გაკეთება წინასწარი დიაგნოსტიკის გარეშე უბრალოდ უაზრობაა. მგონი გავაბეზრე საზოგადოება, მაგრამ მაინც დადებ იმ ფაქტს, როემლიც უშუალოდ მე შემემთხვა.
გლანდები 4 წლის ასაკში ამქვს ამოჭრილი /მაშინ მოდაში იყო/, მაგრამ იქ სადაც ამოჭრილია, უცნაურია და ჩირქის საცობები საკმაოდ ინტენსიურად მიჩნდებოდა. მე-9 საავადმყოფოში დავდიოდი, რას არ მკურნალობდნენ. ერთი სიტყვით, ეს გლანდები ბოლომდე არ იყო ამოჩრილი, არღაც ანწილები დარჩენილი იყო და ისეთია არც გაზრდილი არაა, დანას ვერ მოვკიდებთო და მაშინ ლაზერის აპარატურად არ ჰქონდათ, ნუ ერთი სიტყვით, საჭირო იყო თერაპიული მედიკამენტოზური მკურნალობა.
დამინიშნეს ერითრომიცინი, მივიღე -1998 წელია. კარგად იცი, როგორი ცუდი პრეპარატია ის იგივე კუჭისათვის. არანაირი წინსვლა. ექიმს ვეხვეწები, მოდი ვნახოთ ზუსტად რა ამოითესება და რა წამალი ან ფაგი მივიღოთქო. არ გინდაო თავიდან. თავდაჯერებული იყო ჩემი პროფესორი ბ-ნი გურამი. ნუ შედეგი რომ ვერა ვნახეთ, წავალთქო, ვიძალავე და ყელის ნაცხი დავათესინე. ამოითესა ოქროსფერი სტაფილოკოკი, მგრძნობელობა ოთხ პრეპარატზე იყო, ამპიცილინი, კლაფორანი, ციპროფლოქსაცინი და გენტამიცინი. ბაქტერიოფაგში ფაგები შემომთავაზეს, თუმცა მხოლოდ პიოფაგზე მქონდა მგრძნობელობა. წავეწდი ექიმთან და მინიშნავს ამპიცილინსმხოლოდ. გურამი ექიმო, თუ მხოლდო ამპიცილინს დამინიშნავთ ნისტატინის აგრეშე ან რამე მსგავსი პრეპარატის გარეშე, შემდეგ საფუარას მკურნალობა ცალკე დაგჭირდებათ ჩემზეთქო. შენ რა იციო. ზუსტად ამ ყელზე მკურნალობისას 5 წლის წინ მქონდა ანალოგიური შემთხვევა და ამპიცილინით მკურნალობის შემდეგ, სოკოს უფრო ანგრძლივი მკურნალობა დამჭირდა. ნუ ეს გადახვევა. არ გინდაო. 10 დღიანი კურსი დამინიშნა. უკვე კამათის თავი არ მქონდა. წამოვედი და გადავწყვიტე მე თავად რა მიმეღო, ვინაიდან მგრძნბელობა ვიცოდი, ციპროფლოქსაცინი ავირჩიე, აფთიაქში მისვლისას ბაიერის ციპრობაი შევარჩიე 10 დღიანი ციპრობაის კურსი ჩავიტარე. კი, მაშინ ეს პრეპარატი 48 ლარი დამიჯდა /ამპიცილინი 5-იც არ დამიჯდებოდა/. ექიმის არჩევანი იმიტომ არ შეჩერდა ციფროფლოქსაცინზე, რომ ამ პრეპარატს სპეციფიურად თირკმლის ანთებებისას ვირჩევთო და ყელზე რა უნდა გიშველოსო. ერთი სიტყვით, მე საკუთარი არჩევანი გავაეთე და სრულიად მართალიც აღმოვჩნდი. ხელახალი ანალიზის აღებისას ეს ოქროსფერი სტაფილოკოკი გაქრა და მერე აღარც მქონია.
დედაჩემსაც ქრონიკული ცისტიტი ჰქონდა ამ სამიოდე წლის წინ და მხოლოდ შარდის დათესვის შემდეგ არჩეულმა ანტიბიოტიკმა გამოიყვანა. იმ პრეპარატებზე, რასაც ტრადიციულად უნიშანვენ ხოლმე ცისტიტის დროს /ბისეპტოლი, ფურადონინი, ფურაზოლიდონი/ საერთოდ არა ქონდა მგრძნობელობა. ანტიბიოტიკის სახელი არ მახსოვს რაზეც მგრძნობელობა აღმოაჩნდა მის სტრეპროკოკს. ასე რომ... ჰო და როდესაც სეფსისიანი ავადმყოფი შემოგდის და ხელში გაკვდება ან მენინგოკოკცემია აქვს ან უმძიმესი პნევმონია, ვინ ამბობს სასწრაფოდ გადაუდებლად პრეპარატი არ გაუკეთოთქო smile.gif
უბრალო, ბანალური რესპირატორული დაავადებისას II თაობის ანტიბიოტიკების დანიშვნა არ მესმის. ჩემს ირგვლივ ამის უამრავი შემთხვევაა და ამიტომ ვამბობ.
მართალია, დაბინძურებულია გარემო ისე აქვეითებს ჩვენი ორგანიზმის იმუნუტ სტატუსს რომა.... და ხანდახან სრულიად არასაჭირო მედიკამენტოზური ჩარევა ისედაც დარღვეულ ჩვენს ბიოფლორას კიდევ უფრო ანადგურებს. ბეღურებს გაანადგურებ რომ ბრინჯი არ შეგიჭამონ?! ჭია ორმაგად გადაუვლის!

Posted by: rooney 23 Jul 2008, 17:16
გამარჯობათ... ჩემს მეგობარს დაუსვეს ბოტკინის დიაგნოზი და იქნებ გამარკვიოთ რა ნიშნებით შეიძლება მივხდე ბოტკინი თუ მაქვს???
წინასწარ მადლობა

Posted by: lidia 23 Jul 2008, 20:09
QUOTE
გამარჯობათ... ჩემს მეგობარს დაუსვეს ბოტკინის დიაგნოზი და იქნებ გამარკვიოთ რა ნიშნებით შეიძლება მივხდე ბოტკინი თუ მაქვს???
წინასწარ მადლობა

გულისრევა ღებინება, შეხედვით მოყვითალო ელფერი, სკლერები მოყვიტალო ელფერის, ასევე თუ დააკვირდებით შარდის გამიქება - ლუდისფერი შარდი და განავლის გაღიევება. ბოტკინის დიაგნოზი რომ დასვან უნდა გამოიკვლიონ"
1. ვირუსის მარკერები, თუ რომელი ვირუსია (ა, ბ, ც, დ)
2. ღვიძლის ფუნქციური სინჯები.
ამის მერე შეიძლება ჰეპატიტზე (ბოტკინის დაავადებაზე) ლაპარაკი

Posted by: rooney 23 Jul 2008, 23:38
lidia
სპასიბა ინფოსთვის

Posted by: Lacrimosa_ 25 Jul 2008, 09:57
ჩემს ძმას ფიზიკური დატვირთვის შემდეგ ბრწკენისმაგვარი გრძნობა აქვს ძირითადად ხელებზე და წითლად ეწინწკლება. შემდეგ თავისით გაუვლის.
რისი ბრალი შეიძლება იყოს?

* * *
ერთი კითხვა მაქვს კიდევ.
ყოველ 6 თვეში ერთხელ ვიტარებ ანალიზებს. ყოველთვის ყველაფერი რიგზე მქონდა. გასულ ზამთარს კი სისხლის ანალიზმა როე 30 აჩვენა. ოჯახის ექიმთან მივედი და ნორმალურიაო მითხრა, eek.gif , საერთოდაც ყველაფერზე ეგ პასუხი აქვს, რომ ძალიან ჯანმრთელი ვარ და ყველაფერი რიგზე მაქვს. მაგრამ მაინც ჩავიტარე სრულყოფილი გამოკვლევები. არსად არაფერი არ აღმოჩნდა. მხოლოდ ყელი მტკიოდა და ამოსაჭრელ გლანდებს დააბრალეს. ვიმკურნალე ერთი კვირა და განმეორებითმა ანალიზმა როე 6 აჩვენა. როგორც ამიხსნეს, მაქვს რეაქტიული სისხლი და შესაძლებელი იყო სწორედ გლანდების და გაციებული ყელის ბრალი ყოფილიყო 30 როე.
რას ნიშნავს რეაქტიული სისხლი და ნორმალურია თუ არა 30 როე

Posted by: texasuri jleta benzoxerxit 25 Jul 2008, 12:36
Lacrimosa_
30 ედსი მთლად ნორმალური ვერ არის.მიზეზი მოგიძებნია,წრონიკულმა (ჩირქოვანმა)ტონზილიტმა,რაგორც ნებისმიერმა სხვა ანთებამ,იცის ედსის მომატება.სასხლიც სწრაფად "რეაგირებს"ანთებით პროცესზე და იზრდება ერითროციტების დალექვის სიჩქარე.
6 თვეში ერთხელ რატომ იტარებ გამოკვლევებს?შფოთვა გაქვს შენი ჯანმრთერლობის მიმართ?გეშინია დაავადებების,ინფექციების,სიმსივნის,შიდსის?ყოველდღიურად არგენიშნება აღნიშნულ საკითხთან მიმართებაში შფოთვა?

Posted by: Lacrimosa_ 25 Jul 2008, 13:06

QUOTE
6 თვეში ერთხელ რატომ იტარებ გამოკვლევებს?შფოთვა გაქვს შენი ჯანმრთერლობის მიმართ?გეშინია დაავადებების,ინფექციების,სიმსივნის,შიდსის?ყოველდღიურად არგენიშნება აღნიშნულ საკითხთან მიმართებაში შფოთვა?


დაზღვევა მაქვს და რაღაცაში ხომ უნდა გამოვიყენო rolleyes.gif
მშიშარა არა ვარ, უბრალოდ ანალიზები კარგი უნდა მქონდეს სულ

დიდი მადლობა, დავმშვიდდი, ამოვიჭრი ამ გლანდებს, ჯანდაბას.

იქნებ ჩემს ძმაზეც მითხრათ რაიმე?

QUOTE
ჩემს ძმას ფიზიკური დატვირთვის შემდეგ ბრწკენისმაგვარი გრძნობა აქვს ძირითადად ხელებზე და წითლად ეწინწკლება. შემდეგ თავისით გაუვლის.
რისი ბრალი შეიძლება იყოს?


Posted by: ikaragua 25 Jul 2008, 23:48
რა მინდა გკითხოთ. ერთ გოგუცუნას ფეხები უშუპდება კოჭებთან და წვივებთან. ტკივილით არ ტკიოდა, მარა ამას წინათ ოდნავ ასტკივდა, მერე გაუარა. სამსახური აქვს ისეთი, დიდხანს უწევს ჯდომა და მუშაობა. საგანგაშოა? რისი ბრალი შეიძლება იყოს და რა იქნება საშველი? თუ ძაან მწირი ინფორმაცია არაა, იქნებ მითხრათ?

Posted by: lidia 26 Jul 2008, 11:30
ikaragua
რამდენი წლისაა და დიდხანს დგას ფეხზე?

Posted by: texasuri jleta benzoxerxit 26 Jul 2008, 12:55
ikaragua
ძალიან მწირი ინფორმაციაა

Posted by: BadbadGirl 26 Jul 2008, 12:55
გამარჯობათ.
ღებინებისას წყალივით თხევადი ყვითელი სითხე რისი ნიშანია?? საჭმლის ერთი ნამცეციც არ იყო, უბრალოდ ყვითელი სითხე.

Thanks

Posted by: Solveig 26 Jul 2008, 14:58
BadbadGirl
თუ მწარეც იყო, ნაღვლის დენას გავს და სასწრაფოდ ექიმს უნდა მიაკიტხოს.

Posted by: BadbadGirl 26 Jul 2008, 15:01
Solveig
ოკ, ვკითხავ ახლავე.
....
არ იყოვო მწარე, მჟავე იყოვო.
რა წამალი უნდა გამოვიყენო ან რა ვქნა?

Posted by: Natuka NGN 26 Jul 2008, 18:13
ikaragua
QUOTE
ისეთი, დიდხანს უწევს ჯდომა და მუშაობა. საგანგაშოა? რისი ბრალი შეიძლება იყოს და რა იქნება საშველი? თუ ძაან მწირი ინფორმაცია არაა, იქნებ მითხრათ?

ამისი საყვარელო. ამ დროს სისხლი ცუდად მოძრაობს ფეხებში. მეც მქონდა მსგავსი. აუცილებლად უნდა იაროს და ივარჯიშოს და ჯდომისას რაიმე სიმაღლეზე დადოს ფეხი რომ სულ დაკიდებული არა ქონდეს. აუცილებლად მოზრაობაა საჭირო და სარამოს ერტი ნახევარი საათი კედელზე დალაგებული ფეხებიტ იწვეს, თუ მეტი, კიდევ უკეთესი smile.gif
-----
და თუ დილას ადგომისასაც მსგავსად აქვს შეშუპებული, მაშინ ექიმთან.

Posted by: ikaragua 26 Jul 2008, 18:50
Natuka NGN
ძალიან, ძალიან, ძალიან დიდი მადლობა 2kiss.gif

lidia

23-ის და ფეხზე დგომა ნაკლებად უწევს.

texasuri jleta benzoxerxit

კიდევ რა ინფორმაციაა საჭირო? მითხარით და ვკითხავ.
მადლობთ წინასწარ

Posted by: Natuka NGN 26 Jul 2008, 19:10
ikaragua
არაფრის რისი მადლობა. უბრალოდ ბენზოხერხი რომ გეუბნება მეტი ინფოო, მესმის რატომაც ამბობს. უნდა გამორიცხოს გული და თირკმელები. გარდა კოჭებისა უსივდება კიდევ რამე? მაგალიტად თვალები ან ხელები. მგონი არა... და მგონი სისხლძარღვებისა და უმაძრაობის ბრალია. უბრალოდ 23 წელი მაინც ახალგაზრდაა მსგავის შესივებისათვის, თუმცა არ მიკვირს - ისეტი უმოძრაო ცხოვრება უწევს უმეტესობას და ვარჯიში საერთოდ არ იცის რააა. ჩემს 19 წლის სტუდენს მესამე სართულზე ქოშინის გარეშე ასვლა არ შეუძლია, მე ვასწრებ და არც ვიღლები. უნდა იმოძრაოას აუცილებლად!!! და არ გაგიკვირდეს, წყალი სვავს საკმაო რაოდენობით. ამ მექანიზმის აღწერა უბრალოდ ახლა მეზარება, სიცხის დროს ოფლი ბევრი გამოიყოფა და თირკმელებიდან საჭირო რაოდენობის მარილის გამოყოფა არა ხდება. ასე რომ... ჯანმრთე;ლობას ვუსურვებ შენს მეგობარს.

Posted by: Tornike Alashvili 27 Jul 2008, 02:52
QUOTE (BadbadGirl @ 26 Jul 2008, 15:01 )
Solveig
ოკ, ვკითხავ ახლავე.
....
არ იყოვო მწარე, მჟავე იყოვო.
რა წამალი უნდა გამოვიყენო ან რა ვქნა?

არაფერია საშიში –რეფლუქსი ექნება დუოდენალოგასტრალური და ალბათ წინა დღეს ალკოჰოლი დალია ან რამემ აწყინა –საჭმელი ან კუჭის ძირზე დარჩა ან გადავიდა უკვე კუჭიდან –მინერალური წყალი სვას ან კუჭი ამოირეცხოს სოდიანი წყლით

Posted by: oto_terabyte 27 Jul 2008, 10:36
აუ, გუშინ 3 კეტონალი დავლიე (დილას, შუადღეს და საღამოს)და ეხლა რომ გავიღვიძე მუცელი მტკივა, თავი მტკივა და გაბრუებული მაქვს და რა ვქნა? cry.gif
კბილი მტკიოდა და... sad.gif
არადა რა სწრაფად აყუჩებს...
ეხლაც მტივა კიჭი, ორშაბათამდე ვერ მივალ ექიმთან და რა ვქნა? კიდე დავლიო? მეშინია რამე არ დამემართოს.
პ.ს. არადა რა საყვარლად აყუჩებს... user.gif ))

P.S. ლუდთან რეაქციაში ხომ არ შედის? წუხელ რომ დავლიე ლუდი დავაყოლე და... eek.gif

Posted by: Tornike Alashvili 28 Jul 2008, 00:11
oto_terabyte
ალკოჰოლთან ერთად ეთერზეთებს წარმოშობს და კუჭს საკმაოდ აღიზიანებს -მაღალ მჟავიანობასაც იწყება და ეს თავის ტკივილს იწვევს. უნდა ბორჟომი სვა ბლომათ და კისელი,გერკულესი ჭამე.თუ ჰოეოპათს იცნობ ვინმეს ნუკს ვომიკა გამოართვი კუჭზე გიშვილის და კბილზეც მოგცემს გამაყუჩებელს

Posted by: sofi-18 28 Jul 2008, 14:48
HI. მაინტერესებს რამ იცის, სახსრების, ფეხების, კუნთების ,მუდმივი ტკივილი... ტკივილი ძლიერდება მოძრაობის დროს... 22 წლის

Posted by: lidia 28 Jul 2008, 15:08
sofi-18
უამრავმა რამემ, გლანდები ამოჭრილი გაქვს?

Posted by: sofi-18 28 Jul 2008, 15:11
გლანდები არა მაქვს საერთოდ... და ოპერაციაც არ გამიკეტებია

Posted by: lidia 28 Jul 2008, 15:17
sofi-18
დარწმუნებული ხარ, რომ ყელის პრობლემები არ გაქვს?

Posted by: sofi-18 28 Jul 2008, 15:22
ჩიყვი? მქონდა 10 წლის წინ, მაგრამ აღარ მიმკურნალია მერე....

Posted by: lidia 28 Jul 2008, 15:24
sofi-18
ჩიყვი სულ სხვა საქმეა და თუ გქონდა აუცილებლად მისახედია.

Posted by: sofi-18 28 Jul 2008, 15:25
მერე მაგ პრობლემის მიზეზი ეგ არის?

Posted by: lidia 28 Jul 2008, 15:31

sofi-18

სახსრებს გულისხმობ?
რა არის იცი, სჰეიზლება ჰიპერთირეოზმა არტრიტი ს პროვოცირება მოახდინოს, სხვა მხრივ თუ ჯანმრთელი ხარ, რავი აბა C-რეაქტიული ცილა მაინც აიღე

Posted by: sofi-18 28 Jul 2008, 15:35
ჰიპერთირეოზი ჩიყვთან კავშირშია?

და რომელ ექიმს უნდა მივმართო მაინც კონსულტაციისთვის?

Posted by: lidia 28 Jul 2008, 15:38
sofi-18
ჩიყვი არის ხალხში გავრცელებული სახელი, სინამდვილეში საქმე ეხება ფარისებრ ჯირკვალს. ჩიყვის ქვეშ გულისხმობენ ჰიპერტირეოზს და ჰიპოტირეოზსაც, ამიტომ ბევრი რიმ არ ვიმკითხაოთ, ენდოკრინოლოგს მიმართე.

Posted by: sofi-18 28 Jul 2008, 15:40
მადლობა, გაიხარე ლიდია... 2kiss.gif

Posted by: nika0201 30 Jul 2008, 22:38

მუცელში ბუყბუყის შეგრძნება რისი ბრალია? მაშინ როცა ბოლო ერთი კვირა მუდმივად აღგენიშნება ასეთი რამ

Posted by: DimS 2 Aug 2008, 15:39
რამდენიმე წელია ფილტვების არეში ნეკნების უმნიშვნელო, ყრუ ტკივილი მაწუხებს, არ მივაქცევდი ყურადღებას მარა ეხლახან დამეწყო ბეჭების და კისრის არეში ხერხემლის უფრო საგრძნობი ტკივილი და მაინტერესებს შეილება ესენი ერთმანეთთან რამე კავშირში იყვნენ? ან საერთოდ ძვლის რა დაავადებები არსებობს?

Posted by: what Katy did next 4 Aug 2008, 10:20
მე მაქვს ასეთი კითხვა : ბავშობაში ვწერდი მარცხენა და მარჯვენა ხელით...მაგრამ შემდეგ მარცხვენა ხელით წერაზე ძალით გადამაჩვიეს...ხოდა ეხლა როცა ვცდილობ მარცხენა ხელით დავწერო გული მერევა და თავი მტკივდება...ასეთი პრობლემა მაქვს მანქანაშიც, საკმარისია მარცხენა მხარეს გავიხედო, ან საჭე მარცხენა ხელით დავიჭირო რომ გულისრევის შეგრძნება მემართება და თავბრუ მეხვევა..თავიდან ამას ყურადღებას არ ვაქცევდი და ეხლა გარკვეულ პრობლემებს მიქმნის..იგივე მართვის მოწმობას აღებისასsad.gif
ამ შემთხვევაში ვის უნდა მივმართო ?

Posted by: lidia 6 Aug 2008, 18:09
what Katy did next
ფსიქოლოგს

Posted by: სხვანაირა 6 Sep 2008, 21:32
ექიმებო, ასეთი რამ მაწუხებს: ბოლო 5 თვეა მაქვს მოვლითი ტკივილები თირკმელებში, პერიოდულად მემართება. თავიდან ასე, ორ კვირაში ერთხელ მომივლიდა და მაქსიმუმ 30 წამი გრძელდებოდა. რაც დრო გადის ტკივილი უფრო ინტენსიური ხდება და ხანგრძლივობაც მატულობს- 4-5 დღეში ერთხელ და გრძელდება 2-3 წუთი. უნდა აღვნიშნო, რომ ძალიან მტკივნეულია, ტკივილისგან იძულებული ვხდები, მოვიკუნტო. ეს რომ ნახევარი საათი გაგრძელდეს, ალბათ ტკივილგამაყუჩებელი დამჭირდება პირდაპირ კუნთში. დღეს უკვე ორჯერ მქონდა. რისი ბრალი შეიძლება იყოს? თირკმელებზე პრობლემა არასდროს მქონია. 17 წლის ვარ.
* * *
dim78
QUOTE
რამდენიმე წელია ფილტვების არეში ნეკნების უმნიშვნელო, ყრუ ტკივილი მაწუხებს, არ მივაქცევდი ყურადღებას მარა ეხლახან დამეწყო ბეჭების და კისრის არეში ხერხემლის უფრო საგრძნობი ტკივილი და მაინტერესებს შეილება ესენი ერთმანეთთან რამე კავშირში იყვნენ? ან საერთოდ ძვლის რა დაავადებები არსებობს?


მოხრილი ხომ არ ხარ? შეიძლება მაგის ბრალი იყოს.

Posted by: texasuri jleta benzoxerxit 7 Sep 2008, 00:54
სხვანაირა
თირკმელების ექოსკოპია რომ გაიკეთო არა?სულ მცირე თირკმელ-კენჭოვანი დაავადების გამორიცხვის მიზნით.შარდის საერთო ანალიზიც.შარდმა ფერი იცვალა?

Posted by: ikebana 7 Sep 2008, 01:11
ტეხასელო, შენც დაბრუნდი?

ველქამ ბექ smile.gif

Posted by: სხვანაირა 7 Sep 2008, 11:18
texasuri jleta benzoxerxit
QUOTE
თირკმელების ექოსკოპია რომ გაიკეთო არა?სულ მცირე თირკმელ-კენჭოვანი დაავადების გამორიცხვის მიზნით.

ვაი, მეც ვიფიქრე, კენჭი ხომ არ არის-მეთქი. sad.gif გავიკეთებ აუცილებლად.

QUOTE
შარდის საერთო ანალიზიც.შარდმა ფერი იცვალა?

არა no.gif


ახლა წავიკითხე ერთ თემაში, კენჭმა მარტო ცალი თირკმელის ტკივილი იცისო, მე ორივე მტკივა. არ მინდა კენჭი sad.gif

Posted by: natiko 9 Sep 2008, 19:34
ძალიან გთხოვთ მირჩიეთ ყველაზე უკეთესად ვინ აკეთებს ნაღვლისბუშტის ოპერაციას? საუკეთესო სპეციალისტი მინდა ვიცოდე დედას უნდა გავუკეთოთ ოპერაცია ძალიან დიდი ქვები აქვს და ახალი მეთოდებით არ მირჩიეს


Posted by: თეოგოგო 9 Sep 2008, 23:03
natiko
QUOTE
ძალიან დიდი ქვები აქვს და ახალი მეთოდებით არ მირჩიეს

ქვების ზომას არ აქვს მნიშვნელობა ლაპარასკოპიის გაკეთებისას. "დიდ ქვებს" მუცლის ღრუშივე "ამტვრევენ" და ისე გამოაქვთ ნაწილ-ნაწილ... smile.gif დეიდაჩემს გაუკეთეს მაგ ტიპის ოპერაცია და დანამდვილებით ვიცი. smile.gif

Posted by: sofi-18 11 Sep 2008, 11:38
ჰაი, გთხოვთ მითხარით

რომელ ორგანოებთანაა დაკავშირებული ჭიპი?
რამ შეიძლება გამოიწვიოს ყოველდღიური გამონადები(სუნით) ჭიპიდან?
არის 19 წლის გოგო

მადლობა...

Posted by: Nuna 12 Sep 2008, 12:14
ასეთი შეკითხვა მაქვს... ვარ ა ჰეპატიტ გადატანილი, შემიძლია თუ არა გადავუსხა სისხლი ვინმეს? მადლობა

Posted by: texasuri jleta benzoxerxit 12 Sep 2008, 17:32
Nuna
თეორიულად კი,მაგრამ როგორც ვიცი ასეთ შემთხვევაშიც არ ასხამენ

Posted by: BadbadGirl 12 Sep 2008, 18:32
დაქვეითებულ იმუნიტეტს როგორ ვებრძოლო ?? მითხარით რა ფლიზ, თორემ შევწუხდი, ერთს დაუბერავს თუ არა ეგრევე გრიპი მხვდება.

Posted by: texasuri jleta benzoxerxit 12 Sep 2008, 18:46
BadbadGirl
იმუნო რიცი,ირს19,ბრონქომუნალი,ციკლოფერონი,გოლდ-რეი...

Posted by: lidia 12 Sep 2008, 22:23
BadbadGirl
ჯანსაღი ცხოვრების წესი დაიმკვიდრე, სპორტით დაკავდი, მარო წამლები არ გიშველის

Posted by: თეოგოგო 13 Sep 2008, 02:31
sofi-18
QUOTE
ამ შეიძლება გამოიწვიოს ყოველდღიური გამონადები(სუნით) ჭიპიდან?

ბანაობის შემდეგ კარგად გამოიმშრალოს ჭიპი (ყურის ჩხირებით მაგალითად), ოღონდ ნაზად და ეგ პრობლემა არ მგონია კიდევ ჰქონდეს მომავალში.

თუ მაგან არ უშველა მგონი ენდოკრინოლოგს უნდა მიაკითხოს. კომპეტენტურებო - დამეხმარეთ. biggrin.gif

Posted by: natiko 13 Sep 2008, 11:16
გთხოვთ საიტზე ექიმები თუ არიან მირჩიეთ ყველაზე უკეთესად ვინ აკეთებს ნაღვლისბუშტის ოპერაციას? საუკეთესო სპეციალისტი მინდა ვიცოდე დედას უნდა გავუკეთოთ ოპერაცია ძალიან დიდი ქვები აქვს და ახალი მეთოდებით არ მირჩიეს



Posted by: თეოგოგო 13 Sep 2008, 14:41
natiko
http://forum.ge/?showtopic=33759979&view=findpost&p=10799652
ეს არ წაიკითხე ალბათ. user.gif
ლაპარასკოპია ნაკლებ ტრავმატულია "ჩვეულებრივ" ოპერაციასთან შედარებით, რეაბილიტაციის პერიოდიც შემოკლებული აქვს. თიაქარის განვითარების ალბათობაც .

Posted by: shtori 29 Sep 2008, 21:22
პერიოდულად მაქვს დაბალი სიცხეები (37-37,5) რამოდენიმე დღის განმავლობაში (ერთ კვირამდე). დღის განმავლობაში რამდენჯერმე ამიწვეს, ისევ ნორმაზე დგება და ესე. 1წელზე მეტია ესე ვარ. შეიძლება 2 წელზე მეტიც იყოს. არ ვაქცევდი ყურადრებას.
რამ შეიძლება გამოიწვიოს ესეთი მდგომარეობა? ნუ იქიდან გამომდინარე, რომ ამდენი ხანია სხვას არაფერს ვუჩივი შეიძლება უმიზეზოდ მქონდეს ეს სიცხე? ძალების ნაკლებობს კი გრძნობ. მალე ვიღლები.

Posted by: Romina 4 Oct 2008, 21:28
shtori
გოგო ხარ თუ ბიჭი?

Posted by: shtori 7 Oct 2008, 00:31
Romina
გოგო
-------------------------

Posted by: texasuri jleta benzoxerxit 7 Oct 2008, 01:32
shtori
მასკირებული დეპრესია,ტუბერკულოზი,ქრ.ქოლეცისტიტი,პირის ღრუს ქრონიკული ინფეციები და ქრ.ჩირქოვანი ტონზილიტი,სხვა და სხვა ლატენტური ინფექცია...
პირველ რიგში ქრ.ტონზილიტი(გლანდები)გამორიცხე და ყელ-ყურის სპეციალისტს ეჩვენე.

Posted by: Romina 7 Oct 2008, 17:48
shtori
QUOTE
გოგო

ხოდა თუ გოგო ხარ ეგ მარტივად შეიძლება აიხსნას "გოგოობით" smile.gif smile.gif
დააკვირდი ოვულაციის პერიოდს თუ ემთხვევა ტემპერატურის მომატება (ანუ მენსტრუალური ციკლის სადღაც შუაში თუა). ეგ ფიზიოლოგიურია და ყველა ქალს მასე აქვს თითქმის.

Posted by: Tami28 8 Oct 2008, 12:52
მაქვს პრობლემები ხერხემალთან დაკავშირებით. ტკივილი და დაბუჟების შეგრძნება მარცხენა ხელის და ფეხის. გთხოვთ მირჩიოთ რა გამოკვლევები უნდა ჩავიტარო და რომელ ექიმს მირჩევთ.
მადლობთ

Posted by: Solveig 8 Oct 2008, 18:29
QUOTE
დააკვირდი ოვულაციის პერიოდს თუ ემთხვევა ტემპერატურის მომატება (ანუ მენსტრუალური ციკლის სადღაც შუაში თუა). ეგ ფიზიოლოგიურია და ყველა ქალს მასე აქვს თითქმის.

მე მგონია, რომ იგულისხმა ტემპერატურის ჩვეულებრივი წესით გაზომვა. ოვულაციის დროს კი იმატებს ე. წ. ბაზალური ტემპერატურა (რომელსაც იღლიის ფოსოში არ ისინჯავენ) და ისიც არ იწევს 37'5-მდე. რამდენადაც ვიცი.

იმედი მაქვს, საშიში არაფერი იქნებ,ა მაგრამ მაინც მგონია რომ არ არის სწორი თავის დამშვიდება იმით, რომ ეს ფიზიოლოგიურია და ყველა ქალს ეგრე აქვს. ნამდვილად არაა ეგრე. მოკლედ, ექიმის კონსულტაცია აუცილებელია. სჯობს, ექიმმა დაგამშვიდოს, გამოკვლევის შემდეგ, ვიდრე უცნობებმა ფორუმზე biggrin.gif


Posted by: Romina 8 Oct 2008, 19:26
Solveig
იმის მიხვედრა რომ ბაზალურ ტემპერატურას არ გულისხმობდა ძნელი არ იყო.
ოვულაციის პერიოდში კი სხეულის (აქსილარული გაზომვით) ტემპერატურაც იმატებს მცირედ. 37-იც შეიძლება იყოს და მეტიც (განსაკუთრებით ლაბილური ნერვული სისტემის მქონე ადამიანებში). ამიტომ ვურჩიე დააკვირდეს როდის აქვს ცხელება.

2 წლის განმავლობაში სხვა არანაირი სიმპტომი სერიოზულ დაავადებაზე არ მიუთითებს. თუმცა აქაც შეიძლება იყოს იშვიათი გამონაკლისები. მაგრამ მათი ჩამოთვლით რატო უნდა შემეშინებინა როცა მარტივი ახსნაც შეიძლება არსებობდეს.

თუ ეს არ ემთხვევა ოვულაციას, მაშინ უცნობი ეტიოლოგიის ცხელების მიზეზის დადგენისთვის ინფექციონისტიდან დაწყებული ჰემატოლოგით და რევმატოლოგით დამთავრებული ყველასთან უნდა გაიაროს კონსულტაცია. რაც უკვე ურჩიეს ზემოთ დიაგნოზების სახით და პატივს ვცემ ამ აზრს.

იმის ილუზია რომ აქ ვინმე ვინმეს დიაგნოზს დაუსვავს არავის არ აქვს. იკითხა და ჩემი აზრი ვუთხარი. ერთ-ერთი "უცნობი" ექიმის აზრია უბრალოდ.

Posted by: Solveig 8 Oct 2008, 21:28
Romina

სიმართლე გითხრა, სამედიცინო განათლების მაჩვენებელი შენი არცერთი წერილი თვალში არ მომხვედრია და ამიტომ ვერ დავადგენდი, ექიმი იყავი თუ არა. ფორუმზე რჩევებს სამწუხაროდ, არაექიმებიც იძლევიან და მომიტევე, მაგრამ შენი რჩევაც უფრო არაექიმისას გავდა, ვიდრე ექიმისას...

მას არ დაუზუსტებია, მართლა 2 წელია, მეტი თუ ნაკლები. ისე კი, საინტერესოა, რატომ აქამდე არ მიმართა ექიმს? შეშინება სულაც არაა საჭირო. მე ვთვლი, რომ უკეთესია, მაშინვე ექიმთან კონსულტაცია ურჩიო. აგერ თვითონაც წერ, რომ:

QUOTE
უცნობი ეტიოლოგიის ცხელების მიზეზის დადგენისთვის ინფექციონისტიდან დაწყებული ჰემატოლოგით და რევმატოლოგით დამთავრებული ყველასთან უნდა გაიაროს კონსულტაცია.


და იმის თქმა, რომ ეს ნორმალურია და ყველა ქალს ასე აქვსო, არ არის სწორი, ჩემი აზრით. რადგან ეგ ადამიანი არაა ექიმი. ექიმთან მისვლა კი ყველას ეზარება (სხვათა შორის, მეც) და წელავს-დღეს, ხვალ, დღეს, ხვალ....ჰოდა, მსგავსი რჩევით შეიძლება, ჩათვალოს, რომ საყურადღებო არაფერია. ესაა და ეს.


QUOTE
ოვულაციის პერიოდში კი სხეულის (აქსილარული გაზომვით) ტემპერატურაც იმატებს მცირედ. 37-იც შეიძლება იყოს და მეტიც (განსაკუთრებით ლაბილური ნერვული სისტემის მქონე ადამიანებში).


ჰო, მაგრამ ეს არ ნიშნავს, რომ ყველას ეგრე ემართება. ამასთანავე, ადამიანი ამბობს, რომ ტემპერატურა დღეში რამდენჯერმე იცვლება, ხანდახან 1 კვირამდე გრძელდება (თუკი ამასაც ზუსტად დააკვირდა) და დაღლილობასაც აღნიშნავს.

იმედია, არ მიწყენ, ასე რომ გწერ.

Posted by: vano_t 8 Oct 2008, 21:40
რომინა მართალია. ამ ყველაფერს ფიზიოლოგიური ახსნა უნდა ქონდეს (განსაკუთრებით ახალგაზრდა გოგოში, რომელსაც სხვა ჩივილიები არ აქვს). ოვულაციასთან ამის დაკავშირება და ფიზიოლოგიური მიზეზის გამორიცხვა ძალიან მარტივია. ამიტომ, ჯერ ზუსტად ეს მიზეზი უნდა გამოირიცხოს. პრაქტიკაშიც იყენებენ იმ მომენტს როცა ოვულაციის დროს ტემპერატურა მაღლა ადის (მაგალითად ჩასახვის საწინააღმდეგო ბუნებრივი მეთოდია-არც ისე საიმედო, როგორც ბევრი სხვა მეთოდი, მარა სამაგიერობ ბუნებრივი wink.gif)

ერთ რამეს დავამატებდი მხოლოდ. უცნობი ეტიოლოგიის ცხელებას როცა იკვლევ იქ ტემპერატურა 38 გრადუს ცელსიუზზე მეტი უნდა იყოს. ამას თავისი მიზეზი აქვს. 37 ან 37.5 გრადუსი ძალიან ხშირი ნორმალური ტემპერატურაა და უცნობი ეტიოლოგიის ცხელების დროს აღებული ეგ რიცხვი (38-ზე ზევით) ჰაერიდან არ მოდის. ეგ სტატისტიკური ანალიზის საფუძველზეა მიღებული. მასეთ დროს (ანუ როცა სხეულის ტემპერატურა 38-ზე მეტია) იმის შანსი, რომ დაავადება არსებობს გაცილებით დიდია. პრაქტიკულად 38 გრადუსიც შეიძლება იყოს ნორმალური სიცხი და 36.5 მაღალი სიცხე იშვიათ შემთხვევებში (სიცხე, როგორც სხვა მაჩვენებელი გაუსის განაწილების მისდევს).

P.S. ბაზალური ტემპერატურა არის უბრალოდ ტემპერატურა გაზომილი "ბაზალურ" პირობებში (ანუ, როცა ადამიანის მოსვენებულ მდგომარეობაში). სად გაიზომავ ამ ტემპერატურას აზრი არ აქვს (იშვიათი გამონაკლისის გარდა).

Posted by: texasuri jleta benzoxerxit 8 Oct 2008, 23:34

38მდეც გადასარევად შეიძლება ჩაითვალოს ქრ. დაავადების სიმპტომად.37,7თუ ქვს ყოველ საღამოს,წყნარად ვიყოთ?ნუ ჩავიციკლებით გაიდლაინებზე,ეს ბიბლია არ არის.თუმცე evidence based მედიცინას არავინ უარყოფსvano_t

Posted by: Romina 9 Oct 2008, 08:07
Solveig
QUOTE
მას არ დაუზუსტებია, მართლა 2 წელია, მეტი თუ ნაკლები.

QUOTE
1წელზე მეტია ესე ვარ. შეიძლება 2 წელზე მეტიც იყოს.



QUOTE
რამოდენიმე დღის განმავლობაში (ერთ კვირამდე).

ერთი კვირა სიცხე და მეორე კვირას აღარ მიუთითებს რომ ორგანული პათოლოგია არ არის. სად მიდის სხვა დროს ის მიზეზი რომელიც იწვევს სუბფებრილიტეტს?

მე მგონი ავხსენი ჩემი აზრი და ვიმეორებ. უამრავი გამოკვლევის ჩატარებამდე ვურჩიე უბრალოდ დააკვირდეს ხომ არ ემთხვევა ეს ოვულაცუას, ეს ხო ძნელი არ არი. და თუ არ ემთხვევა მერე მიაკითხავს ექიმს, რასაც აქამდეც გააკეთებდა 2 წლის განმავლობაში სერიოზულად რო აწუხებდეს რამე (იმიტომ რომ ეტყობა არ მიეკუთვნება თავისი ჯანმრთელობისადმი ინდიფერენტულ პირებს. რახან ასე აკვირდება თავის მდგომარეობას.)

ექიმობა მარტო ის არ არის პაციენტი უამრავ გამოკვლევაზე გაუშვა და ფინანსური და ფსიქოლოგიური ზარალი მიაყენო. როცა შეიძლება ეს აიხსნას მარტივად და ამისთვის საჭიროა ერთ რამეს დააკვირდეს. მერე თავისთვავად მიმართავს ექიმს.


თუ შენ ექიმი ხარ იქნებ მითხრა შენი აზრი, რამ შეიძლება გამოიწვიოს ცხელება ახალგაზრდა გოგოში, რომელიც 1-2 წელია გრძელდება, 1 კვირა თვეში თუ როგორც არის და სხბვა არანაირისიმპტომი.


vano_t
QUOTE
უცნობი ეტიოლოგიის ცხელებას როცა იკვლევ იქ ტემპერატურა 38 გრადუს
ცელსიუზზე მეტი უნდა იყოს.

კი მართალი ხარ 100%-ით. FUO-ს დეფინიცია მასეა.

Posted by: iza 9 Oct 2008, 11:23
მაინტერესებს რა არის შემაერთქსოვილოვანი დისპლაზია??? რამდენად საშიშია, რა სიმპტომები აქვს და როგორ მკურნალობენ?
მოკლედ, ყველაფერი მაინტერესებს ამ დაავადების შესახებ

Posted by: aerztin 9 Oct 2008, 11:43
Solveig
QUOTE
შენი რჩევაც უფრო არაექიმისას გავდა, ვიდრე ექიმისას...

ვერ დაგეთანხმები. Romina-ს რჩევა სწორია და ნამდვილად ექიმის რჩევაა.
შენი პოსტები კი ჰგავს არაექიმის რჩევებს, რომელიც თვლის რომ ექიმებზე მეტი იცის. ბოდიში, მაგრამ ასეა.

ფორუმი რომ ექიმის კონსულტაციას არ ცვლის, მემგონი ეს ყველასთვის ნათელია.



Posted by: kasandra 9 Oct 2008, 16:45
ერთი კითხვა მაქვს; ხანდახან მაწუხებს ტკივილი მარჯვენა ბეჭის ქვეშ, უფრო მაშინ, როცა მცივა, ან ქარიანი ამინდია rolleyes.gif
ვითომ არაფერი, მაგრამ ისეთი საძაგელი ტკივილია user.gif ლამის ვიღაც გამვლელს ვთხოვო, მუშტი ჩამცხე ზურგშითქო.. (გამოცდილია, მიჩერებს ტკივილს) user.gif
რისი ბრალია ეს ტკივილი და რა სპეციალისტს უნდა მივმართო?

Posted by: Solveig 9 Oct 2008, 18:34
aerztin
QUOTE
შენი პოსტები კი ჰგავს არაექიმის რჩევებს, რომელიც თვლის რომ ექიმებზე მეტი იცის.

მე არასოდეს არ დამიმალავს, არავისთან, რომ ექიმი არ ვარ და თუ კარგად დააკვირდი, ყველა ჩემი "რჩევა" (თუკი შეიძლება მას ასეთი რამ ეწოდოს!) უფრო იმის თქმას გავს-ექიმს მიაკითხეთ მეთქი.

დიდ ცოდვად ითვლება ის, რომ განათლების გამო ცოტა რაღაც გამეგება ანატომიასა და ფიზიოლოგიაში და ხანდახან რაღაცას ვწერ ხოლმე? გინახავს ჩემი დაწერილი დიაგნოზი ან დანიშნულება? ბოდიშიც ხომ არ მოვიხადო და გავიძურწო აქედან?


QUOTE
ფორუმი რომ ექიმის კონსულტაციას არ ცვლის, მემგონი ეს ყველასთვის ნათელია.

ყურადღებით რომ გადახედო თემებს, მიხვდები, რომ ეგრე არ არის. ექიმთან მისვლა ყველას ეზარება, სამწუხაროდ..და თვლიან-ვიკითხავ ერთი, იქნებ ისეთი არაფერიაო.

ასევე ყურადღებით თუ გადახედავ, ვერ ნახავ ჩემს მიერ გახსნილ თემას-რა ვქნა და ა. შ. მეთქი. იმიტომ, რომ ონლაინ კონსულტაციების სასტიკი წინააღმდეგი ვარ.

თუ არ გეწყინება, შენს ბოდიშს ვერ მივიღებ .

Posted by: vano_t 9 Oct 2008, 20:01
iza
QUOTE
მაინტერესებს რა არის შემაერთქსოვილოვანი დისპლაზია??? რამდენად საშიშია, რა სიმპტომები აქვს და როგორ მკურნალობენ?
მოკლედ, ყველაფერი მაინტერესებს ამ დაავადების შესახებ

ეგ დაავადება არ არის. დისპლაზია არის პათოლოგანოატომიური ტერმინი. პათოლოგ-ანატომი ჩახედავს უბრალოდ მიკროსკოპში და იმის მიხედვით იტყვის, რომ დისპლაზიაა. დისპლაზიის დროს ე.წ. მოუმწიფებელი უჯრედები (რომელიც დასაბამს აძლევს ე.წ. სპეციალიზებულ უჯრედებს) უფრო ჭარბობს, ვიდრე სპეციალიზებული უჯრედები. რაც შეეხება შემაერთებელი ქსოვილის დისპლაზიას, ეს უფრო კრებითი სახელია უამრავი დაავადებისა, რომელიც აზიანებს სხვადასხვა შემაერთებელ ქსოვილს (მაგალითად ძვლებს, სახსრებს, მყესებს, კანის შემაერთებელ ქსოვილს, სისხლძაღვების შემაერთეელ ქსოვილს და შემაერთებელ ქსოვილს ნებისმიერ ადგილას ზოგადად). ეგ დიაგნოზს არ წარმოადგენს. ამიტომ, ექიმმა უნდა გითხრას რა დაავადებასთან არის საქმე. ამ დაავადებათა რიცხვს მიეკუთვნება ელერს დანლოსის სინდრომი, მარფანის სინდრომი და მსგავსი დაავადებები, მუკოპოლისაქარიდოზი, ოსტეოქონდროდისპლაზია, Osteogenesis Imperfecta (არასრული ოსტეოგენეზი) და ა.შ.

Posted by: aerztin 9 Oct 2008, 20:47
Tami28
ნევროლოგმა უნდა გაგსინჯოს და მერე გადაწყვეტს, გჭირდება თუ არა გამოკვლევა - სავარაუდოდ, ხერხემლის კისრის ნაწილის მაგნიტურ-რეზონანსული ტომოგრაფია.


Solveig
მე არ მითქვამს, რომ განათლება ცოდვად ითვლება და ვინმემ აქ არ უნდა პოსტოს მეთქი. რა უფლება მაქვს. უბრალოდ, როცა ექიმი კომპეტენტურ აზრს გამოთქვამს და შენ სრულიად უსაფუძვლო არგუმენტებით ედავები და თან ეუბნები, შენი რჩევები ექიმის რჩევებს არ ჰგავსო, ეს არასწორად მიმაჩნია.

Posted by: margusha 9 Oct 2008, 20:56
aerztin

მიუხედავად იმისა რომ ჩემი დაცვა არანაირად არ სჭირდება, მაინც ვიტყვი რომ შენი გაწიწმატება არის ყველანაირ საფუძველს მოკლებული.smile.gif რადგან სოლვეიგმა ბევრ ექიმზე მეტი იცის და არც იმის საზღვარი ეშლება სად მთავრდება ბიოლოგი და სად იწყება ექიმი ან პირიქით. smile.gif

Posted by: aerztin 9 Oct 2008, 21:00
margusha
კაი ბატონო, ძალიან კარგი, თუ მასეა smile.gif

Posted by: iza 10 Oct 2008, 16:42
QUOTE
ეგ დაავადება არ არის. დისპლაზია არის პათოლოგანოატომიური ტერმინი. პათოლოგ-ანატომი ჩახედავს უბრალოდ მიკროსკოპში და იმის მიხედვით იტყვის, რომ დისპლაზიაა. დისპლაზიის დროს ე.წ. მოუმწიფებელი უჯრედები (რომელიც დასაბამს აძლევს ე.წ. სპეციალიზებულ უჯრედებს) უფრო ჭარბობს, ვიდრე სპეციალიზებული უჯრედები. რაც შეეხება შემაერთებელი ქსოვილის დისპლაზიას, ეს უფრო კრებითი სახელია უამრავი დაავადებისა, რომელიც აზიანებს სხვადასხვა შემაერთებელ ქსოვილს (მაგალითად ძვლებს, სახსრებს, მყესებს, კანის შემაერთებელ ქსოვილს, სისხლძაღვების შემაერთეელ ქსოვილს და შემაერთებელ ქსოვილს ნებისმიერ ადგილას ზოგადად). ეგ დიაგნოზს არ წარმოადგენს. ამიტომ, ექიმმა უნდა გითხრას რა დაავადებასთან არის საქმე. ამ დაავადებათა რიცხვს მიეკუთვნება ელერს დანლოსის სინდრომი, მარფანის სინდრომი და მსგავსი დაავადებები, მუკოპოლისაქარიდოზი, ოსტეოქონდროდისპლაზია, Osteogenesis Imperfecta (არასრული ოსტეოგენეზი) და ა.შ.

vano_t
დიდი მადლობა გამოხმაურებისათვის ჩემო კარგო. ადრე იყო რესპუბლიკურში ესეთი პროფესორი - ზურა ტაბიძე (გარდაიცვალა სამწუხაროდ). ჰოდა უნივერსიტეტში რომ ჩავაბარე , მისაღები გამოცდების დროს გადავიღალე მაგრად, ჰემოგლობინიც დამივარდა და ფეხზე ვერ ვდგებოდი ისე სუსტად ვიყავი. ჰოდა ოჯახის ახლობელმა მიგვიყვანა ზურასთან, რომელმაც მითხრა, ძ ალიან თეთრი ხარ და ნაზი, ეს შემაერთქსოვილოვანი დისპლაზია ხშირად ემართებათ შენნაირებს, ამიტომ ხაში და ხალადეცი უნდა ჭამოო. მე ვერცერთს ვერ ვჭამ. მეზიზღება ორივე. მას შემდეგ კარგად ვარ, მაგრამ დროდადრო ჰემოგლობინი ისევ მივარდება ხოლმე, ექიმის დანიშნულებით ვიღევ ფოლიუმის მჟავას და ვიტამინებს. ეხლა მაინტერესებს, საიდან შემიძლია გავიგო, ნამდვილად მჭირს თუ არ ა შემაერთქსოვილოვანი დისპლაზია? მაშინ ზურას კონკრეტულად ამაზე ანალიზები არ აუღია. თვითონ დაასკვნა უბრალოდ. და კონკრეტულად სად უნდა მივიდე და რა ანალიზი უნდა გავიკეთო, რომ დავადგინო, მაქვს თუ არა შემაერთქსოვილოვანი დისპლაზია?

Posted by: kasandra 10 Oct 2008, 18:14
ჩემს ტკივილს რა ეშველება ძვირფასო ექიმებო?
user.gif

QUOTE
ხანდახან მაწუხებს ტკივილი მარჯვენა ბეჭის ქვეშ, უფრო მაშინ, როცა მცივა, ან ქარიანი ამინდია 
ვითომ არაფერი, მაგრამ ისეთი საძაგელი ტკივილია  ლამის ვიღაც გამვლელს ვთხოვო, მუშტი ჩამცხე ზურგშითქო.. (გამოცდილია, მიჩერებს ტკივილს) 
რისი ბრალია ეს ტკივილი და რა სპეციალისტს უნდა მივმართო?


Posted by: Romina 10 Oct 2008, 19:24
მასე ძალიან ძნელია ზუსტად ლოკალიზაციის დადგენა, სად გტკივა. თან ცოტა გაურკვეველია (ჩემთვის ყოველ შემთხვევაში) ხელის დარტყმით ტკივილის გავლა. ნევრიტიც შეიძლება იყოს, მიოზიტიც (კუნთის ანთება). ორივეს პროვოცირება შეიძლება მოახდინოს სიცივემ.

გააჩნია კიდე შენ ასაკს, სტენოკარდიის გამოვლინებაც შეიძლება იყოს სიცივეში ტკივილი (მარა რა ვიცი, ამ ხელის დარტყმით გავლას ვერაფერს ვუკავშირებ)

ჩემი აზრით რევმატოლოგს ან ნევროლოგს უნდა ეჩვენო.

Posted by: chorven:) 10 Oct 2008, 19:42
სიარულისგან ისე მტკივა ფეხები, მოწყვეტილი ვარ პირდაპირ და თან მაზოლი გამიჩნდა, რა წავისვა, რომ დაძაბულობა მომეხსნას და მაზოლსაც რა ვუყო?

Posted by: vano_t 11 Oct 2008, 09:52
iza
QUOTE
ეხლა მაინტერესებს, საიდან შემიძლია გავიგო, ნამდვილად მჭირს თუ არ ა შემაერთქსოვილოვანი დისპლაზია?
როგორც ვთქვი, ეგ ტერმინი არის კრებითი და არავინ ეძებს მაგ "დაავადებას" საერთოდ. თუ ექიმი მიიტანს ეჭვს რაიმე დაავადებაზე, მაშინ ამ დაავადებაზე გააკეთებს გამოკვლევას. მაგ დაავადებებს ყველას სხვადასხვა სიმპტომები აქვს.

ზოგადად ამ დაავადებებს შეიძლება ახასიათებდეს სახსრების, მყესების, კანის, ძვლების, სისხლძარღვების პრობლემები.

თუ კარგად ხარ, არაფრის გაგება არ გჭირდება. ჰემოგლობინის ვარდნა სისხლის პრობლემაა და ეს დაავადებები, როგორც წესი, არ იწვევენ სისხლის პრობლემებს. სახელი მიხედვით (იზა) ქალი უნდა იყო. ქალებში ყველაზე ხშირი მიზეზი ანემიისა არის მომატებული სისხლდენა პერიოდების დროს. თუ ეს არ არის მიზეზი, მაშინ ჰემატოლოგმა შეიძლება დაადგინოს რა სახის ანემია გაქვს (ანემიები ძალიან ბევრი ტიპის შეიძლება იყოს).

QUOTE
მაშინ ზურას კონკრეტულად ამაზე ანალიზები არ აუღია. თვითონ დაასკვნა უბრალოდ. და კონკრეტულად სად უნდა მივიდე და რა ანალიზი უნდა გავიკეთო, რომ დავადგინო, მაქვს თუ არა შემაერთქსოვილოვანი დისპლაზია?
როგორც ვთქვი, ეს არ არის ერთი დაავადება. არამედ სხვადასხვა დაავადებების ჯგუფია და შესაბამისად ყველას თავისი ტესტი ექნება. არავინ არ გააკეთებს მაგ ყველა ტესტს ერთდრულად. ზოგადად, მსგავსი დაავადებების დროს შეიძლება საჭირო იყოს დაზიანებული ორგანოს ბიოფისა, ან გენეტიკური ანალიზი, ან რაიმე სპეციფიური ტესტი.

Posted by: kasandra 11 Oct 2008, 13:33
Romina

ძალიან დიდი მადლობა smile.gif
ისე სტენოკარდიის გამოვლინება არა მგონია იყოს, 28 წლის ვარ და ძალიან დიდი ხანია ასე მემართება, ფაქტიურად ბავშვობიდან.

Posted by: Romina 11 Oct 2008, 13:48
kasandra
QUOTE
ძალიან დიდი მადლობა

2kiss.gif

Posted by: Tikk 11 Oct 2008, 17:00
თვალის უპეები მაქვს მუდმივად ჩაშავებული - გადაღლას და უძილობას ვაბრალებდი, მაგრამ დასვენების პერიოდშიც ასე მაქვს. რისი ბრალი შეიძლება იყოს, ვერავინ მეტყვით? user.gif

Posted by: tikope 11 Oct 2008, 20:26
ყურის ტკივილის მიზეზები ვინ იცის და რითი შეიძლება მკურნალობა?
მადლობთ

Posted by: macaco-fast65 11 Oct 2008, 20:48
tikope
ოტიპაქსიიი იყიდეეეე და ჩააწვეთე

Posted by: iza 13 Oct 2008, 10:36
vano_t
უპირველეს ყოვლისა ძალიან დიდი მადლობა რომ გამომეხმაურე. სახსრები ხანდახან მტკივა ხოლმე უმეტესწილად ბევრს რომ ვივლი ფეხით ან ძალიან რომ გადავიღლები. 10 ოქტომბერს ავიღე სისხლის ანალიზი და ჰემოგლობინი 71 მაქვს ანუ მკურნალობამ შედეგი გამოიღო (64 მქონდა). ჰემატოლოგმა ფოლიუმის მჟავა და პოლიჟენი გამომიწერა მანამდე. 10 დღე ვსვავდი ფოლიუმის მჟავას (5 მილიგრამიანს, დღეში 3-ჯერ) და ეხლა კიდე 10 დღე გამომიწერა. რკინის შრატიც ამაღებინა და ნორმაში მაქვს, მაგრამ პირველ ანალიზზე მქონა მიელოციტი 2 და როე 13; შენიშვნის სახით კი მიმიწერეს მცირე ანიზოციტოზი. ეხლა რომ ავიღე 10ში, მიელოციტი 1 მაქვს და როე 18. ჰემატოლოგმა მითხრა ნელ-ლენა პროცესი ლაგდება და ეტყობა რაიმე ინფექცია იყოვო (ისე 2-ჯერ ვიყავი იმ პერიოდში გაცივებული). ჰო, კიდე ციკლოფერონი გამომიწერა, იმუნიტეტის გასაძლიერებელი. რა ვიცი, თვითონ მეუბნება ნუ გეშინია, საშიში აქ არაფერი არ არისო. 3 თვის შემდეგ დამიბარა, შრატის რკინა უნდა გავიმეოროთ, რადგან ამ 3 თვის განმავლობაში (იმ პერიოდების მერე) არ უნდა იყოს რკინის დანაკარგიო.
ისე ჩემი პრობლემები 17 წლის ასაკიდან დაიწყო. მანამდე არაფერი არ მაწუხებდა. უნივერსიტეტში რომ ვაბარებდი, არანაორმალური ცხოვრების წესით ვცხოვრობდი, დღეღამეში მხოლოდ 2 საათი მეძინა და პირველ კურსზე ძალიან ცუდად გავხდი. ისეთი დასუსტებული ვიყავი, მანქანაში ხელით ჩამსვეს ექიმთან წასაყვანად. მაშინ დამიდგინდა ნორმოქრომული ანემია. ეხლა ჯერჯერობით ვერ ვადგენო ჩემმა ჰემატოლოგმა, 3 თვის შემდეგ ვნახოთო.
ისე გულზე ჰისის კონის მარჯვენა ფეხის სრული ბლოკადა მაქვს. შალვა მეუბნება პეტრიაშვილი ეს საერთოდ არაფერი არ არის და არანაირი პათოლოგია არ არისო, ბეტალოკ-ზოგი მაქვს გამოწერილი დღეში 50 მილიგრამი. შეიძლება ეს ბლოკადა რამე კავშირში იყოს სისხლთან???
ყველა მეუბნება არაფერი არ არის სანერვიულოო, არადა მე ნერვები მეშლება რა, ხან გული მტკივა, ხან ხშირად ვცივდები და ათასი ჯანდაბა.
ისე ეხლაც არანორმალური ცხოვრების წესი მაქვს. სამსახურში დილიდან საღამომდე ურთულესი საქმე მაქვს, სახლში გვიან მივდივარ და დისერტაციას ვუჯდები, შაბათ კვირასაც ბიბლიოთეკაში ვარ. აგვისტოშიც არ მომცეს შვებულება (ან სად უნდა წავსულიყავი). ჰოდა ექიმი მეუბნება ცოტას თუ არ დაისვენებ, ასე ვერ გაძლებო. აღარ ვიცი როგორ მოვიქცე.

Posted by: ბახარხალა 15 Oct 2008, 17:20
აუ სურდო აქ და რა ვუშველო? cry.gif



ისე მართლა გაციებული ვარ, საშინლათ მახველებს და რა წამლები მივიღო? :| აფთიაქში მეზარება ჩასვლა :|

Posted by: vano_t 18 Oct 2008, 07:11
iza
QUOTE
ისე გულზე ჰისის კონის მარჯვენა ფეხის სრული ბლოკადა მაქვს. შალვა მეუბნება პეტრიაშვილი ეს საერთოდ არაფერი არ არის და არანაირი პათოლოგია არ არისო, ბეტალოკ-ზოგი მაქვს გამოწერილი დღეში 50 მილიგრამი. შეიძლება ეს ბლოკადა რამე კავშირში იყოს სისხლთან???
მარჯვენა ფეხის ბლოკი (RBBB) ტავისთავად არ არის კავშირში სისხლთან. მაგრამ შეიძლება სისხლში მოცირკულირე ნაერთებმა გამოიწვიოს ბლოკი. ეს ნაერთები შეიძლება იყოს ტოქსინები (მაგალითად წამლები) ან შეიძლება იყოს "ნორმალური" ატომი/მოლეკულა, რომელიც ზომაზე მეტი რაოდენობით ან ნაკლები რაოდენობით არის სისხლში. მაგალითად, კალიუმის, რომელიც აუცილებელი მეტალის იონია, ნაკლებობამ შეიძლება გამოიწვიოს გულის გამტარი სისტემის (ეგ ჰისის კონა და მისი ფეხები არის გულის გამტარი სისტემა) ბლოკი. როგორც წესი ასეთი ბლოკი დროებითია და პრობლემის მოხსნისას (როცა ტოქსინი მოცილდება ორგანიზმს, ან როცა ფიზიოლოგიური ატომის/მოლეკულის ნორმალიზაცია მოხდება) ეს ბლოკიც იხსნება. თუ ბლოკი მუდმივია, მაშინ ამას დიაგნოზი ჭირდება. ანუ, უნდა გაირკვეს რა იწვევს ამ ბლოკს, თუ რა თქმა უნდა შესაძლებელია დიაგნოზის დადგენა (ყველგან და ყოველთვის არ არის ამის დადგენა შესაძლებელი). ყველა ასაკში ასეთი დიაგნოზები განსხვავებულია. 17 წლის ასაკში რა დაავადებები იწვევენ ბლოკს, ეგ არის ბავშვთა კარდიოლოგის გასარკვევი. თუმცა, თვითონ გულის გამტარი სისტემის ბლოკი თავისთავად სერიოზულ პრობლემას არ წარმოადგენს. ანუ, გულის მუშაობა ამ ბლოკის დროს არ ფერხდება იშვიათი შემთხვევების გარდა.

მეორე საკითხია (შენს კითხვასთან დაკავშირებით) არსებობს თუ არა დაავადება, რომლის დროსაც გარკვეული სახის ანემია და გულის ბლოკი ერთად გვხდება? შეიძლება არსებობს, თუმცა მე ვერ გეტყვი რომელი დაავადებებია ასეთი-არ მაქვს საკმარისი ინფორმაცია ამაზე.

და ბოლოს, ამ ორი სიმპტომის ასოციაცია (ბლოკის და ანემიის) შეიძლება სრულიად შემთხვევითი იყოს და არავითარი კავშირი არ ქონდეს ერთმანეთთან.

Posted by: Guardian 18 Oct 2008, 10:48
texasuri jleta benzoxerxit
QUOTE
38მდეც გადასარევად შეიძლება ჩაითვალოს ქრ. დაავადების სიმპტომად.37,7თუ ქვს ყოველ საღამოს,წყნარად ვიყოთ?ნუ ჩავიციკლებით გაიდლაინებზე,ეს ბიბლია არ არის.თუმცე evidence based მედიცინას არავინ უარყოფსvano_t

არ შეიძლება ჩაითვალოს - ვანო მართალია.

ერთი შესწორება მხოლოდ -
vano_t
QUOTE
უცნობი ეტიოლოგიის ცხელებას როცა იკვლევ იქ ტემპერატურა 38 გრადუს ცელსიუზზე მეტი უნდა იყოს. ამას თავისი მიზეზი აქვს. 37 ან 37.5 გრადუსი ძალიან ხშირი ნორმალური ტემპერატურაა და უცნობი ეტიოლოგიის ცხელების დროს აღებული ეგ რიცხვი (38-ზე ზევით) ჰაერიდან არ მოდის. ეგ სტატისტიკური ანალიზის საფუძველზეა მიღებული.

ეგ ციფრი 38°C-ზე ზემოთ კი არ არის, არამედ - 38.3°C-ზე ზემოთ -

According to studies of healthy individuals 18–40 years of age, the mean oral temperature is 36.8° ± 0.4°C (98.2° ± 0.7°F), with low levels at 6 A.M. and higher levels at 4–6 P.M. The maximum normal oral temperature is 37.2°C (98.9°F) at 6 A.M. and 37.7°C (99.9°F) at 4 P.M.; these values define the 99th percentile for healthy individuals. In light of these studies, an A.M. temperature of >37.2°C (>98.9°F) or a P.M. temperature of >37.7°C (>99.9°F) would define a fever. The normal daily temperature variation is typically 0.5°C (0.9°F).
Rectal temperatures are generally 0.4°C (0.7°F) higher than oral readings.

Fever of unknown origin (FUO) was defined by Petersdorf and Beeson in 1961 as (1) temperatures of >38.3°C (>101°F) on several occasions; (2) a duration of fever of >3 weeks; and (3) failure to reach a diagnosis despite 1 week of inpatient investigation. While this classification has stood for more than 30 years, Durack and Street have proposed a new system for classification of FUO: (1) classic FUO; (2) nosocomial FUO; (3) neutropenic FUO; and (4) FUO associated with HIV infection.
Classic FUO corresponds closely to the earlier definition of FUO, differing only with regard to the prior requirement for 1 week's study in the hospital. The newer definition is broader, stipulating three outpatient visits or 3 days in the hospital without elucidation of a cause or 1 week of "intelligent and invasive" ambulatory investigation. In nosocomial FUO, a temperature of 38.3°C (101°F) develops on several occasions in a hospitalized patient who is receiving acute care and in whom infection was not manifest or incubating on admission. Three days of investigation, including at least 2 days' incubation of cultures, is the minimum requirement for this diagnosis. Neutropenic FUO is defined as a temperature of 38.3°C (101°F) on several occasions in a patient whose neutrophil count is <500/L or is expected to fall to that level in 1–2 days. The diagnosis of neutropenic FUO is invoked if a specific cause is not identified after 3 days of investigation, including at least 2 days' incubation of cultures. HIV-associated FUO is defined by a temperature of 38.3°C (101°F) on several occasions over a period of >4 weeks for outpatients or >3 days for hospitalized patients with HIV infection. This diagnosis is invoked if appropriate investigation over 3 days, including 2 days' incubation of cultures, reveals no source.

Harrison's Principles of Internal Medicine, 17th Ed. 2008

Posted by: texasuri jleta benzoxerxit 19 Oct 2008, 04:42
Guardian
ისევ და ისევ ნუ მოვექცევით გაიდლაინების ტყვეობაში.ყოველდღიური 37,8,ასევე საგანგაშო სიმპტომი შეიძლება იყოს.მაგალიტად ხანშიშესულებში.მათ იშვიათად აღენიშნებათ მაღალი ცხელება.ყოველდღიური სუბფერილიტეტი კი მაგალითად Cancer-ის სიმპტომი შეიძლება იყოს.ასევე იმუნოსუპრესიულ პირებში შეიძლება ტემპერატუარმ არა მარტო 38.C-ს არ მიაღწიოს ,არამედ შეიძლება 35,C-მდე დაეცეს,რაც უცნოი ეტილოგოოს ცხელების ერთგვარ "ექვივალენტადაც შეიძლება მიჩნეულ იქნეს ზოგიერთ შემთხვევაში!!!

Posted by: tamtuna 19 Oct 2008, 16:48
კაცს თავბრუ რო ეხვევა რისი ბრალი შეიძლება იყოს უკვე რამოდენიმე დღეა?








................

Posted by: სხვანაირა 19 Oct 2008, 21:07
არ ვიცი სად ვიკითხო და აქ ვიკითხავ:

წითელასა და წითურაზე რომ დაიწყო აცრები, როგორც გავიგე, 27 წლამდე ყველა უნდა აიცრასო. ორი წლის წინ მაქვს გაკეთებული ეგ აცრა (15 წლის ასაკში) და ახლა ისევ უნდა გავიკეთო თუ აღარ მჭირდება?

Posted by: vano_t 20 Oct 2008, 07:20
tamtuna
QUOTE
კაცს თავბრუ რო ეხვევა რისი ბრალი შეიძლება იყოს უკვე რამოდენიმე დღეა?
ბევრი რამ იწვევს თავბრუსხვევას. მაგრამ ყველაზე ხშირად ითველბა ვირუსული ინფექციები. ასეთ შემთხვევაში ეს ცოტა ხანი გრძელდება და მალე გადის. თუ ძალიან დიდი ხანი გრძელდება, ან განმეორებადი სიმპტომები გაქვს სმენადობის დაქვეითებასთან ერთად, მაშინ გამოკვლევა უნდა მაგას.


სხვანაირა
QUOTE
არ ვიცი სად ვიკითხო და აქ ვიკითხავ:

წითელასა და წითურაზე რომ დაიწყო აცრები, როგორც გავიგე, 27 წლამდე ყველა უნდა აიცრასო. ორი წლის წინ მაქვს გაკეთებული ეგ აცრა (15 წლის ასაკში) და ახლა ისევ უნდა გავიკეთო თუ აღარ მჭირდება?
MMR უნდა გაკეთდეს 2 ვაქცინა სულ. ანუ, ერთი აცრა აცრების ცხრილის მიხედვით კიდევ საჭიროა.

Posted by: iza 20 Oct 2008, 10:19
vano_t
დიდი მადლობა ყურადღებისთვის ჩემო კარგო smile.gif

Posted by: tamtuna 20 Oct 2008, 18:08
vano_t
მადლობა... ექიმმა სტრესულიაო smile.gif

Posted by: DOLPHIN 21 Oct 2008, 11:51
გონების დაკარგვის შემთხვევაში (თუ რამდენკერმე განმეორდა) რის ექიმმს უნდა მიმართოს კაცმა და რა უნდა გადაიღოს? sad.gif

Posted by: texasuri jleta benzoxerxit 21 Oct 2008, 14:09
DOLPHINკარდიოლოგს და ნევროპათოლოგს.აუცილებელია ეკგ.რომ გაირკვეს კოლაფსის მიზეზი არითმია ხომ არ იყო(ანუ შეიძლება 24საათიანი ჰოლტერის მონიტირინგი დაგჭირდეს).

Posted by: DOLPHIN 21 Oct 2008, 14:21
texasuri jleta benzoxerxit
ეკგ როგორ იშიფრება და ჰოლტერის მონიტორინგი რა არის (მომიტევეთ უმეცრება)?

Posted by: Cor-toni 21 Oct 2008, 14:48
DOLPHIN
QUOTE
ეკგ როგორ იშიფრება და ჰოლტერის მონიტორინგი რა არის?

ეკგ-ს ანუ კარდიოგრამას "შიფრავს" კარდიოლოგი. ჰოლტერის მონიტორინგი არის კარდიოგრამის მსგავსი. კარდიოგრამაზე ჩაწერის მომენტში არსებული ცვლილებები ფიქსირდება და, თუ არითმია პერიოდულად ვლინდება და იმ მომენტში არ არის, ცხადია ვერც დაფიქსირდება. ჰოლტერი არის 24 საათიანი მონიტორინგი. პატარა კოლოფს დაგკიდებენ ქამრით წელზე. გამოვლენილ ცვლილებებს იმახსოვრებს ჰოლტერის აპარატი. ამასაც შემდეგ კარდიოლოგი შიფრავს.

QUOTE
მომიტევეთ უმეცრება

biggrin.gif ვერ მოგიტევებთ 2kiss.gif


Posted by: DOLPHIN 21 Oct 2008, 15:15
Cor-toni
smile.gif
და თავზე არაფრის გადაღება არ არის საჭირო? გული არც მიფიქრია, გადავწყიტე, რომ ტომოგრაფია (თუ -გრამა, არ ვიცი სახელი) იქნებოდა საჭირო...

Posted by: Cor-toni 21 Oct 2008, 15:31
DOLPHIN
QUOTE
და თავზე არაფრის გადაღება არ არის საჭირო? გული არც მიფიქრია, გადავწყიტე, რომ ტომოგრაფია (თუ -გრამა, არ ვიცი სახელი) იქნებოდა საჭირო...

ამას უკვე ნევროლოგი გადაწყვეტს. კარდიოლოგის კონსულტაციაც საჭიროა.

Posted by: DOLPHIN 21 Oct 2008, 15:34
Cor-toni
QUOTE
ამას უკვე ნევროლოგი გადაწყვეტს. კარდიოლოგის კონსულტაციაც საჭიროა.

გასაგებია. დიდი მადლობა. smile.gif
კარდიოლოგიც და ნევროპათოლოგიც ორივე კარგი სპეციალისტი რომ იყოს და აპარატურაც ქონდეთ იმავე დაწესებულებაში, რომელ კლინიკას მირჩევთ, წინ და უკან რომ არ ვირბინო?

Posted by: vano_t 22 Oct 2008, 08:24
DOLPHIN
QUOTE
გონების დაკარგვის შემთხვევაში (თუ რამდენკერმე განმეორდა) რის ექიმმს უნდა მიმართოს კაცმა და რა უნდა გადაიღოს? sad.gif

რამდენი წლისაა ავადმყოფი? რა სქესის? გონება რამდენი ხნით დაკარგა? როდის დაეწყო ეს სიმპტომები? გონებას რა დროს კარგავს? (წამოდგომისას ან წამოჯდომისას მარტო თუ შეიძლება დაწოლილ ან დამჯდარ მდგომარეობაშიც დაკარგოს?) გონების დაკარგვისას თუ აღენიშნება რაიმე კრუნჩხვის მაგვარი ან შარდის შეუკავებლობა? რაიმე მწვავე დაავადება ხომ არ ქონია ახლახან (ღებინებით, ან ოფლიანობით, ან ფაღარათით)? რაიმე წამალი ხომ არ დაუწყია ამ ბოლო დროს? რაიმე ქრონიკული დაავადება თუ გააჩნია, რისთვისაც წამლებს შეიძლება სვადეს?

როგორც უკვე გითხრეს, შეიძლება გული იყოს ან ნერვული სისტემა. ოღონდ, ავადმყოფის ასაკს გააჩნია და სიტუაციას. შეიძლება უბრალოდ სისხლანკლებობა იყოს (განსაკუთრებით ახალგაზრდა ქალებში თუ მომატებული სისხლდენები აქვთ პერიოდების დროს); შეიძლება უბრალოდ გაუწყლოება იყოს, განსაკუთრებით რაიმე მწვავე ვირუსული დაავადებების შემდეგ თუ ფაღარათი და ღებინება აქვს ავადმყოფს. ასეთ შემთხვევებში მარტო სისხლის ანალიზიც იქნება საკმარისი. სისხლის ანალიზი ისეც დაჭირდება გულიც რომ იყოს მიზეზი ან ნერვული სისტემა. ახალგაზრდებში კიდევ ხშირია ე.წ. ვაზოვაგალური გულის წასვლა, როცა რაიმე ემოციური სანახაობისას ემართებათ ეგ (მაგალითად სისხლის დანახვაზე, ოღონდ სისხლი დანახვა არ არის აუცილებელი). იმის მიხედვით თუ რა პასუხს გაცემ კითხვებს, დამოკიდებულია ვის ნახავ. წესით, თერაპევტით უნდა დაიწყო და თერაპევტმა უნდა გაარკვიოს ყველაფერი და თერაპევტმა უნდა დაგაკვალიანოს.

Posted by: DOLPHIN 22 Oct 2008, 10:29
vano_t
QUOTE
რამდენი წლისაა ავადმყოფი?

ავადმყოფი მე ვარ, 28 წლის (მდედრი). wink.gif
უკვე 5-ჯერ დავკარგე გონება, ბოლო 6 თვის განმავლობაში. პირველად ფეხმძიმობის დროს გაზაფხულზე და დანარჩენი ბოლო ერთი თვის მანძილზე. ბავშვი 4 თვის ხდება და ვაჭმევ. ანემია არასდროს მქონია, სტაბილურად 72 მაქვს ჰემოგლობინი. რა სიმპტომებიც ჩამოთვალეთ, არაფერი მახასიათებს. მე უფრო გადაღლილობას ვაბრალებ, თან პირველი შვილია და ცოტა დაძაბულიც ვარ. შეიძლება გადღლის და უძილობის ბრალი იყოს? მანამდე არასდროს წამსვლია გული და ახლა როცა გონებას ვკარგავ, თვალებში მიბნელდება და დაცემისას აზრზე მოვდივარ მაშინვე. პირველად არავის გაუგია და დანარჩენების მომსწრე ჩემი ქმარი იყო და ნევროპათოლოგთან მექაჩება. არ მინდა სხვებმაც გაიგონ, არადა წამლების დალევაც არ მხიბლავს დიდად. sad.gif ვის მივმართო (კონკრეტული კლინიკა და ექიმი მირჩიოთ ეგებ)?

Posted by: EXHALATING 22 Oct 2008, 11:33
მონოციტების მომატება სისხლში რისი მანისნებელი შეიძლება იყოს?ექიმმა ფილტვებზე მომისმინა და სუფთააო. ალბათ რამე ვირუსული გადაიტანეო და მანამდე დაახლობით 3 კვირით ადრე დაბალი სიცხეები მქონდა საღამოობით. მოწევას დავანებე თავი მაგის მერე,ეხლა ცოტა დავიწყესავით ისევ და 2 დღეა საღამოობით დაბალი სიცხე მაქვს ისევ 37.2 ხოდა ეს ყველაფერი კავშირშია ერთმანეთთან? biggrin.gif ან რომელ ექიმთან მივიდე,დავიღალე ყველა ერთმანეთთან მაგზავნის sad.gif და ვერაფერს იგებენ მაინც ,,,,
მაპატიეთ ძალიან აბდაუბდა დავწერე...........

Posted by: Cor-toni 22 Oct 2008, 12:39
DOLPHIN

PM ნახე.






....................................

Posted by: vano_t 22 Oct 2008, 21:52
QUOTE (DOLPHIN @ 22 Oct 2008, 10:29 )
QUOTE
რამდენი წლისაა ავადმყოფი?

ავადმყოფი მე ვარ, 28 წლის (მდედრი). wink.gif
უკვე 5-ჯერ დავკარგე გონება, ბოლო 6 თვის განმავლობაში. პირველად ფეხმძიმობის დროს გაზაფხულზე და დანარჩენი ბოლო ერთი თვის მანძილზე. ბავშვი 4 თვის ხდება და ვაჭმევ. ანემია არასდროს მქონია, სტაბილურად 72 მაქვს ჰემოგლობინი. რა სიმპტომებიც ჩამოთვალეთ, არაფერი მახასიათებს. მე უფრო გადაღლილობას ვაბრალებ, თან პირველი შვილია და ცოტა დაძაბულიც ვარ. შეიძლება გადღლის და უძილობის ბრალი იყოს? მანამდე არასდროს წამსვლია გული და ახლა როცა გონებას ვკარგავ, თვალებში მიბნელდება და დაცემისას აზრზე მოვდივარ მაშინვე. პირველად არავის გაუგია და დანარჩენების მომსწრე ჩემი ქმარი იყო და ნევროპათოლოგთან მექაჩება. არ მინდა სხვებმაც გაიგონ, არადა წამლების დალევაც არ მხიბლავს დიდად. sad.gif ვის მივმართო (კონკრეტული კლინიკა და ექიმი მირჩიოთ ეგებ)?

შენს ასაკში ყველაზე ხშირი მიზეზია ორთოსტატული გულის წასვლა ან ვაზოვაგალური ("ნერვული", სტრესულ სანახაობაზე რომ მიდით გული). ორთოსტატული ნიშნავს, როცა დაწოლილი მდგომარეობიდან წამოჯდები ან წამოდგები (ან დამჯდარი მდომარეობიდან წამოდგები), მაშინ ეორგანიზმი ვერ ახერხებს წნევის ფიზიოლოგიური ვარდნის სწრაფ კომპენსაციას და ხდება გულისწასვლა. როცა დაეცემა ადამიანი (ანუ ვერტიკალურ მდგომარეობაში გადადის), მაშინ სისხლის მიმოქცევა ტვინში ისევ იზრდება. ეს იქნება სავარაუდო მიზეზი. ამის შემოწმება ადვილია. ავადმყოფს უზომავენ წნევას და პულს დაწოლილ, შემდეგ წამომჯდარ და შემდეგ ფეხზედამადგარ მდომარეობაში. თუ წნევა დაეცა საკმაოდ ან პულსი გაიზარდა საკმაოდ, მაშინ ეს არის მიზეზი. ამის დადგენა ექიმის ოფისში ხდება. ექიმი უნდა ნახო. თუ ჰემოგლობინი არ იქნა შემოწმებული იმის მერე რაც სიმპტომები დაგეწყო, მაშინ აუცილებლად უნდა შეამოწმო. დანარჩენი, იმის მიხედვით რას აჩვენებს გამოკვლევა, შეიძლება სხვადასხვა სახის გამოკვლევები დაგინიშნონ.

გულის წასვლის წინ თუ გაქვს რაიმე სიმპტომები: მაგალითად გულის აჩქარების შეგრძნება, გულის ტკივილი, ქოშინი? და რამდენი ხანია რაც ეგ სიმპტომები გაქვს? დღეები, კვირეები თუ უფრო მეტი ხანი? სიმპტომებს შორის როგორ ხარ? რაიმე თუ გაწუხებს?

Posted by: DOLPHIN 23 Oct 2008, 10:24
vano_t
ჰემოგლობინს ყოველ თვე ვაკონტოლებ, რომ ბავშვს არაფერი დააკლდეს და 72 მაქვს ხოლმე.
QUOTE
გულის წასვლის წინ თუ გაქვს რაიმე სიმპტომები: მაგალითად გულის აჩქარების შეგრძნება, გულის ტკივილი, ქოშინი?

no.gif თვალებში მიბნელდება ხოლმე და ვეღარაფერს ვხედავ.
QUOTE
და რამდენი ხანია რაც ეგ სიმპტომები გაქვს? დღეები, კვირეები თუ უფრო მეტი ხანი?

QUOTE
უკვე 5-ჯერ დავკარგე გონება, ბოლო 6 თვის განმავლობაში. პირველად ფეხმძიმობის დროს გაზაფხულზე და დანარჩენი ბოლო ერთი თვის მანძილზე. ბავშვი 4 თვის ხდება და ვაჭმევ.

QUOTE
სიმპტომებს შორის როგორ ხარ? რაიმე თუ გაწუხებს?

ჩვეულებრივად და არაფერი მაწუხებს. კიდევ თუ განმეორდა, აუცილებლად ვეჩვენები სპეციალისტს. მე რატომღაც გადაღლილობას და სტრესს ვაბრალებ ამ ყველაფერს.

Posted by: vano_t 24 Oct 2008, 08:11
QUOTE (DOLPHIN @ 23 Oct 2008, 10:24 )
ჩვეულებრივად და არაფერი მაწუხებს. კიდევ თუ განმეორდა, აუცილებლად ვეჩვენები სპეციალისტს. მე რატომღაც გადაღლილობას და სტრესს ვაბრალებ ამ ყველაფერს.

აბა გისურვებ ჯანმრთელობას. ისე ექიმსაც დაენახე: კაია ხანდახან smile.gif

Posted by: DOLPHIN 24 Oct 2008, 11:12
vano_t
QUOTE
აბა გისურვებ ჯანმრთელობას.

დიდი მადლობა! smile.gif
QUOTE
ისე ექიმსაც დაენახე: კაია ხანდახან 

ხო, რა ვიცით ამ ქართველებმა, სანამ არ მოვკვდებით, მანამდე არ მივდივართ ხოლმე... wink.gif

Posted by: Goli 27 Oct 2008, 14:22
მოკლედ რაღაცნაერი ტკივილები მაქვს ხოლმე მუხლზე და ბარძაყზე, აი შიგნიდან თითქოს რაღაც გჩხვლეტსო რა, ხოდა რისი ბრალია ხომ არ იცით? ფეხის კუნთებზეც მასე მჭირს ხოლმე და ხელზეც ერთხელ მასეტ პონტში დამემართა და რა მჭირს იქნებ ამიხსნათ? იმის ბრალი ხომ არ არის ვარჯიშებზე ხშირად გაუხურებლად რომ ვარჯიშობ ხოლმე, ანუ მოთელვების მაგივრად ეგრევე დარტყმებზე რომ გადავდივარ? მოკლედ ჰელპ რა!

Posted by: Romina 27 Oct 2008, 14:30
როდის გაქ ტკივილი. დღის განმავლობაში თუ ღამით? მოძრაობისას თუ მუდმივად? ბარძაყზე რას გულისხმობ, მენჯ-ბარძაყის სახსარი გტკივა თუ ფეხი მთლიანად მუხლამდე?

Posted by: Goli 27 Oct 2008, 14:44
QUOTE (Romina @ 27 Oct 2008, 14:30 )
როდის გაქ ტკივილი. დღის განმავლობაში თუ ღამით? მოძრაობისას თუ მუდმივად? ბარძაყზე რას გულისხმობ, მენჯ-ბარძაყის სახსარი გტკივა თუ ფეხი მთლიანად მუხლამდე?

რავი ხან როგორ, ხან როგორ, ხანდახან დილით, ხანდახან ღამით, ხანდახან პორსტა რომ ვზივარ მაშინ იჩენს თავს ხოლმე, რავი უეცრად ამტკივდება ხოლმე რა, დღეს მაგალითად დილით რომ ავდექი რაღაც დროის განმავლობაში ამტკივდებოდა, მერე ისევ მომირჩებოდა.

Rectus Femoris (middle head) <--აი სურათზე ეს რომ წერია აი მაგა დგილას მარჯვენა ფეხზე, ხოდა მუხლიც რაღაცნაერად მაწუხებს ხოლმე..

Posted by: Romina 27 Oct 2008, 14:59
Goli
იცი რა, ასე ზეპირად ძალიან ძნელია მიზეზის გაგება. შეიძლება სახსარზე დიდი დატვირთვაა და პატარა ართროზული მოვლენები წამოვიდა, შეიძლება სახსრის ირგვლივ ქსოვილების ანთება იყოს (პერიართრიტი). ზოგჯერ მარტო მენჯ-ბარძაყის სახსარშია პრობლემა და მუხლიც მაგიტო ტკივათ. ჩემი აზრით რევმატოლოგთან უნდა მიხვიდე ან თუ გინდა ტრავმატოლოგთან. უფრო პირველთან.

Posted by: EXHALATING 27 Oct 2008, 15:01
ეჰჰჰჰჰჰ...... ჩემს კითხვაზე ვერავი მიპასუხა sad.gif

Posted by: Goli 27 Oct 2008, 16:25
QUOTE (Romina @ 27 Oct 2008, 14:59 )
Goli
იცი რა, ასე ზეპირად ძალიან ძნელია მიზეზის გაგება. შეიძლება სახსარზე დიდი დატვირთვაა და პატარა ართროზული მოვლენები წამოვიდა, შეიძლება სახსრის ირგვლივ ქსოვილების ანთება იყოს (პერიართრიტი). ზოგჯერ მარტო მენჯ-ბარძაყის სახსარშია პრობლემა და მუხლიც მაგიტო ტკივათ. ჩემი აზრით რევმატოლოგთან უნდა მიხვიდე ან თუ გინდა ტრავმატოლოგთან. უფრო პირველთან.

გასაგებია, მადლობ დახმარებისთვის wink.gif


Posted by: Romina 27 Oct 2008, 20:35
EXHALATING
შენი პოსტი წავიკითხე, მარა რამე კონკრეტულის დაწერა არ შემიძლია. მონოციტოზი ცალკე და ცალკე ცხელება ძალიან ბევრ რამეს ახასიათებს. სიგარეტთან მე ვერ ვაკავშირებ. ჩემი აზრით აუცილებლად უნდა მიხვიდე ექიმთან, თერაპევტთან ან ჰემატოლოგთან. სისხლის ანალიზი აუცილებლად უნდა გაიმეორო.

Posted by: EXHALATING 28 Oct 2008, 01:00
Romina
ჰოო, ვიცი ძალიან არეულად დავწერე.....უბრალოდ ვეღარ გავიგე, ხან რა გამოხტება ხან რა... ჰემატოლოგი მეც ვიფიქრე.ცხელებამ კი გადამიარა მაგრამ ეხლა კიდურები მიბუჟდება მალე... კონტრაცეპტივის უკუჩვენებას გადავაბრალე(ეწერა ანოტაციაში) .... დამღალა ავადმყოფობამ sad.gif thanx anyway

Posted by: shanyva 31 Oct 2008, 11:15
მაინტერესებს ვალერიანის აბებმა შეიძლება კუჭი გააღიზიანოს?მადლობა ყველას!

Posted by: vano_t 31 Oct 2008, 22:57
QUOTE (shanyva @ 31 Oct 2008, 11:15 )
მაინტერესებს ვალერიანის აბებმა შეიძლება კუჭი გააღიზიანოს?მადლობა ყველას!

ყველანაირმა აბმა და წამლის სხვა ფორმამ შეიძლება კუჭი გაგიღიზიანოს.

Posted by: shtori 1 Nov 2008, 23:41
გულის რევის შეგრძნება მაქვს, იმის და მიუხედავად მშიერი ვარ თუ ნაჭამი. მე-3 დგეა უკვე. კუჭის არასდროს არ შევუწუხებივარ. არც ეხლა მაწუხებს.
რა მივიღო რომ გადამიაროს?

Posted by: vano_t 2 Nov 2008, 09:48
QUOTE (shtori @ 1 Nov 2008, 23:41 )
გულის რევის შეგრძნება მაქვს, იმის და მიუხედავად მშიერი ვარ თუ ნაჭამი. მე-3 დგეა უკვე. კუჭის არასდროს არ შევუწუხებივარ. არც ეხლა მაწუხებს.
რა მივიღო რომ გადამიაროს?

თუ სხვა არაფერი გაწუხებს (განსაკთრებით მაღალი სიცხე, ღებინება, მუცლის ტკივლი) მაშინ შეგიძლია დალიო პრომეთაზინი (ფენერგანი არის ბრენდი) 12.5 მგ 1 ან 2 ტაბლეტი ყოველ 6 საათში საჭიროების მიხედვით.

Posted by: Wolfwood 2 Nov 2008, 16:46
გამარჯობათ,

ესეიგი რა ხდება, რამდენიმე დღეა ( 5-6) მაწუხებს ტკივილი მარჯვენა თირკმლის არეში, თითქოს და რაგაცა მჩხვლეტს, ძალიან მსუბუქი ტკივილია ( biggrin.gif ) თუ რამით ვარ გართული შეიძლება არც შევამჩნიო მარა როცა არაფერს ვაკეთებ საკმაოდ მაწუხებს. თუ არ გამივლის რამდნეიმე დღეში ექიმთან წავალ, რამე მოსაზრება ხომ არ გაქვთ რა უბედურება მჭირს help.gif ? ხო ისე არც ვეწევი და არც ვსვამ tongue.gif

Posted by: shtori 2 Nov 2008, 21:34
vano_t
სიცხე არ ვიცი, მუცელი კი მტკივა ცოტა.
მენჯის მიდამოში, მარჯვენა მხარეს ამტკივდა, გამიმაგრდა, გამაკავა ცოტა არ იყოს. მერე გადამიარა და გულის რევის შეგრძნება მქონდა, მერე გონი დავკარგე. ეხლა კარგად ვარ. ხან მოვლითი ტკივილი მაქვს სუსტი. გულის რევის შეგრძნება კი სულ. წამილის მღების შემდეგ ესეც გამივლის smile.gif მადლობა.

Posted by: texasuri jleta benzoxerxit 2 Nov 2008, 23:49
shtori
მოდი მკურნალობა გამოკვლევების ჩატარების მერე ჩაიტარე კარგი?და ტრანსკონტინენტალური დანიშნულება მერე შეასრულე.მინიმუმ თერაპევტს და/ან გასტროენტეროლოგს ეჩვენე,ღვიძლის და სანაღვლე გზების ექოსკოპია ჩაიტარე,და ა.შ.
ელემენტარული კუტურაა.რაღაც გაწუხებს-მიდიხარ შესაბამისი პროფილის ექიმთან.ვსიო პროსტო.ბოლეწ გორაზდო ნეპრიიატნეიე

Posted by: shtori 3 Nov 2008, 00:28
texasuri jleta benzoxerxit
ვერ მივდივარ ჯერ. ასე რომ...
უკულტუროდ ვითვლები
mo.gif

Posted by: levanjik 3 Nov 2008, 00:32
მეგობარი მყავს ინგლისურის მასწავლებელია ბევრი მუშაობა უწევს ძალიან რომ გადაიტვირთება ცუდად ხდება ორ დგიანი იცის ხოლმე ვფიქრობ თან გლიცინს ღებულობს ვფიქრობ რომ დეპრესიაშიცაა ჩავარდნილი აქვს მიზეზები. ამიტრიპტილინი ვურჩიე მიიღო და იმ დგეს ცუდად გახდა წნევამ აუწია და გაითიშა .. რამდენად შეიდძლება პრეპარატის ბრალი იყოს.?????

Posted by: texasuri jleta benzoxerxit 3 Nov 2008, 00:51
levanjik
ამიტრიპტილინი,ისიც მარალ დოზებში(თუმცა ყველაფერი ინდივიდუალურია)იწვევს,პირის სიმშრალეს,მხედველოის ოდნავ გაუარესებას,ყაბზობას,შარდვის შეკავებას,ოფლიანობას,ტაიკარდიას,და კოლაფსს(წნევი ვარდნა)ან პირიქით ჰიპერტენზიას(წნევის აწევას),ძლიერ სედაციას,მაგრამ მხოლოდ მაღალ დოზებში და ისიც ბევრ ავადმყოფს საერთოდ არ უვლინდება...
შენ ექიმი ხარ?ტუ არა რატომ მიეცი თავს უფლება და ურჩიე ამიტრიპტილინი?სესაბამისი პროფილის ექიმები არ არსებობენ???? wow.gif

Posted by: levanjik 3 Nov 2008, 22:51
არა ფარმაცევტი ვარ და ცოტა გადავამეტე ცჰემს კომპეტენციებს..... მაგრამ არა მგონია მარტო ამიტრიპტილინის ბრალი იყოს პაცინტის მდგომარეობას დიდი მნიშვნელობა აქვს ზუსტად ეგ ვერ გავთვალე

Posted by: shtori 7 Nov 2008, 22:29
კისერი მტკივა მარჯვენა მხარეს, მარჯვენა ხელის ცერა თითის სახსარი მტკივა და მარკვენა ფეხის ცერა თითის სახსარი მტკივა კიდო.
პრობლემა არაა მეტკინოს, მაგრამ მაინმც და მაინც მხოლოდ მარკვენა მხარე რატომ?

Posted by: Romina 9 Nov 2008, 19:24
shtori
შენ როგორი მრავალფეროვანი და უცნაური ჩივილები გაქ ხოლმე smile.gif მთელი დღე გტკივა? მოძრაობისას დ თუ ისე? შეშუპებული ხო არ არი თითები?

Posted by: Bala 9 Nov 2008, 21:44
ხომ ვერ მეტყვით სად არის დერმატოლოგიის ინსტიტუტი? smile.gif დერმატოლოგიის ცენტრი.

სადღაც ნინოშვილის ქუჩაზეა მგონი, ზუსტად არ ვიცი sad.gif

Posted by: shtori 10 Nov 2008, 01:05
Romina
გაციების პირველი ნიშნები ყოფილა
sad.gif

Posted by: EKIKULI 10 Nov 2008, 20:53
რამდენად შეესაბამება სიმართლეს, რომ ნაღვლისბუშტის ანთება ჰაერის უკმარისობასაც იწვევს?

Posted by: Cor-toni 10 Nov 2008, 21:15
EKIKULI
QUOTE
რამდენად შეესაბამება სიმართლეს, რომ ნაღვლისბუშტის ანთება ჰაერის უკმარისობასაც იწვევს?

ნაღვლის ბუშტის ანთება ჰაერის უკმარისობას არ იწვევს, თუმცა ეს არ ნიშნავს, რომ ორივე ერთად არ შეიძლება იყოს.

Posted by: EKIKULI 10 Nov 2008, 21:28
Cor-toni
რა იწვევს Hაერის უკმარისობას იმ შემთხვევაში თუ გულის დაავადებას გამოვრიცხავთ?

Posted by: shtori 10 Nov 2008, 23:09
აუ რამე წამალი მინდა გაციებისთვის და სიცხის დამწევი. ოღონდ ისეთი სასუნთ გზებს რომ არ მიშრობდეს.
რაღაც ჩაები დავლიე, ტაბლეტები და უარესად გავხდი ისე გამომიშრო ყელი, ცხვირი სუნთქვისას ყველაფერი მტკიოდა sad.gif ამას სიცხე და ბევრი ცხვირსახოცები მირჩევინა.

Posted by: Cor-toni 11 Nov 2008, 00:51
EKIKULI
QUOTE
რა იწვევს Hაერის უკმარისობას იმ შემთხვევაში თუ გულის დაავადებას გამოვრიცხავთ?

სასუნთქი სისტემის დაავადებები.
shtori
სითხე მიიღე მეტი. პარაცეტამოლი დალიე თუ ტემპერატურა 38-ზე მეტი გექნება.


Posted by: shtori 11 Nov 2008, 01:26
Cor-toni
QUOTE
სითხე მიიღე მეტი

გმადლობ. კარგი რჩევაა, შველის. მაგრამ დროს სჭირდება. მე კიდე რამე სწარფი მეთოდი მინდა
ვერა? sad.gif

Posted by: nu_nuka 11 Nov 2008, 21:40
shtori
ცხელი ჩაი დალიე მალინით, თან ფეხები ცხელ ქყალში გქონდეს, ამისსემდეგ რაც შეიძლება თბილად ჩაწექი ლოგინში და რამდენხანსაც გაუძლებ მთლიანად გაეხვიე თავიც საბანში ჩამალე რომ ოფლი იდინო. მერე სხვა ლოგინში გადაწექი სწრაფად. მე მშველის იმედია შენთანაც ეფექტური იქნება

Posted by: BoOoOo 14 Nov 2008, 03:27
QUOTE
ნევროლოგია, ყელ-ყურ-ცხვირი, ოფთალმოლოგია, გასტროენტეროლოგია, კარდიოლოგია, პულმონოლოგია(ანუ ფილტვების დაავადებები), ინფექციური დაავადებები, ოფისის ტრამვატოლოგია, ენდოკრინოლოგია, კანის დაავადებები, ალრგიული და იმუნოლოგიური დაავადებები, ოფისის მცირე ქირურგიული დაავადებები, ნეფროლოგია, ონკოლოგია, ჰემატოლოგია, რევმატოლოგია და ცოტ-ცოტა ყველაფერი.



თავი? mo.gif

Posted by: vano_t 15 Nov 2008, 10:11
QUOTE (BoOoOo @ 14 Nov 2008, 03:27 )
თავი? mo.gif

რა გაწუხებს თქვი და იქნება მოგცენ რჩევა.

Posted by: WuTheRinG_HeiGhts 15 Nov 2008, 11:35
vano_t

ნაღვლისბუშტისთვის, ან ნაღველმდენი რომელი ბალახეული საშუალება შეიძლება შვიძინო?


Posted by: BoOoOo 15 Nov 2008, 14:16
vano_t

დღის მონაკვეთებში ხშირად
დამეწყება ჰოლმე
და იმჰელა ჰმაზე მიწუის რაღაც
საერთოდ არაფერი აღარ მესმის ჰოლმე mo.gif

Posted by: texasuri jleta benzoxerxit 15 Nov 2008, 20:44
BoOoOo
ყელ-ყურ0ცხვირმა უნდა გნახოს,საჭირო იქნება აუდიოგრამა ალბათ და აუცილებლად ფსიქიატრათან უნდა გაიარო კონსულტაცია!

Posted by: WuTheRinG_HeiGhts 16 Nov 2008, 19:18
ნაღვლისბუშტისთვის, ან ნაღველმდენი რომელი ბალახეული საშუალება არის?

Posted by: BoOoOo 16 Nov 2008, 21:39
texasuri jleta benzoxerxit

QUOTE
ფსიქიატრათან



ვაიი mo.gif

Posted by: basa-ttt 16 Nov 2008, 21:42
QUOTE
ნაღვლისბუშტისთვის, ან ნაღველმდენი რომელი ბალახეული საშუალება არის?

საუკეთესოა
კრაზანა Hypericum perforatum - Зверобой продырявленный

კიდევ
ნეგო - бессмертник
გვირილა ромашка лекарственная
ფარსმანდუკი - тысячелистник -

Posted by: WuTheRinG_HeiGhts 17 Nov 2008, 20:40
basa-ttt
QUOTE
კრაზანა Hypericum perforatum - Зверобой продырявленный


დიდი მადლობა yes.gif

Posted by: shtori 22 Nov 2008, 00:29
როე (თუ რა ჰქვია?) რას ნიშნავს? რისი მაჩვენებელია?

Posted by: vano_t 22 Nov 2008, 00:41
shtori
QUOTE
როე (თუ რა ჰქვია?) რას ნიშნავს? რისი მაჩვენებელია?

როე რუსული შემოკლებაა "ერითროციტების დალექვის სიჩქარის". ეს არის სიდიდე რომელიც მიუთითების ანთების არსებობას ორგანიზმში. უამრავი დაავადებების დროს შეიძლება იყოს მომატებული. ამიტომ ცალკე ეს ტესტი არაფერს ეუბნება ექიმს.

Posted by: OHOI_IKA 27 Nov 2008, 00:22
არ ვიცი ამ თემაში თუ უნდა დამესვა ეს შეკითხვა მარა მაინც

მოკლედ, რამდენიმე დღეა თავის ტკივილი მაქვს, ჩვულებრივად კი არ მტკივა თავი, არამედ შუბლიდან კეფამდე, ტმიტ დაფარული ნაწილი მტკივა გარედან, თითქოს თმებზე დამქაჩეს ან "სინიაკის" პონტში რა. ხელის შეხებაზე მტკივა

რა უნდა იყოს? მითუმეტეს, რომ არც არავის ვუცემივარ და არც თმები დაუწიწკნიათ...

Posted by: rock in rose 29 Nov 2008, 15:13
მეც მაქვს ერთი შეკითხვა.
უკვე მეორე ზამთარი რაც მთელი სიცივეების განმავლობაში ვერ ვსუნთქავ და ვიყენებ უამრავ ცხირის წვეთებს, რომლებით ლორწოვანს მიშრობენ და ა.შ. წინა წელს ვიყავი ექიმთან და მითხა ფრონტიტი გავსო, რადგან თავს როცა ვხრი შუბლზე დაწოლას ვგრძნობ და გამომიწერა ანტიბიოტიკი და ცხვირის წვეთები. არ მიშველა და წელსაც ვუჩივი იგივე პრობლემებს.
ჩემი ნათესავი მისული იყო კარგ ოტორილარინგოლოგთან და იმასაც ჩემი სიმფტობები ჰქონა, ხოდა იმ კაცმა უთხრა ალერგიული რინიტი გაქვსო და სპეციალური ცხვირის წვეტები გამოუწერა, რომელიც ალერგიული სიმპტომების დროს გამოიყენება. მეც ვიწვეთებდი და მიშველა. მერე ზაფხული დადგა, გადამიარა ამ ყველაფერმა, მაგრამ ახლა ისევ შემომიტია და იმ წამლის სახელი არ მახსოვს, რომელმაც მიშველა.
თქვენ რას მირჩევთ. ინჰალაციები გავიკეთო, ვიწვეთო თუ რა ვქნა? ხო და კიდევ სიცივეზე მაქვს ალერგია, ცრემლდენის სახით. ეს რამე კავშირში ჩემს დაავადებასთან? ან საერთოდაც ფრონტიტი მემგონი არ მაქვს და მეც ალერგიული რინიტი მაქვს.

გმადლობთ ყურადღებისთვის
wink.gif wink.gif wink.gif biggrin.gif biggrin.gif wink.gif

Posted by: shtori 30 Nov 2008, 12:06
ფილტვების ანთება, ფარულად მიმდინარე, მკურნალობის გარეშე როა, გონებრივ შესაძლებლობებზე თუ მოქმედებს?

Posted by: texasuri jleta benzoxerxit 30 Nov 2008, 14:18
rock in rose
გირჩევ ოტო-რინო-ლარინგოლოგთან და ალერგოლოგთან ვიზიტს

Posted by: Annabel Lee 30 Nov 2008, 20:11
ეგება მითხრათ, რა:
თბილისში მედიკამენტი Panacur -ი იშოვება?
ან ის მაინც მითხრათ, სად გავიკითხო!

Posted by: Makaveli 1 Dec 2008, 18:12
სისხლში ნორმაზე დაბალი ლეიკოციტები რისი ნიშანი შეიძლება იყოს?

დან დართული მაღალი სიცხეებით, სავარაუდოდ ვირუსული უნდა იყოს. 5 დღეს იყო 40 სიცხეები დაახლოებით და კიდევ ორი სამი დღეა გრძელდება სიცხეები დაბალი ოღონდ

Posted by: GHG 3 Dec 2008, 17:17
მოკლედ მჭირს ესეთი რაღაც, უცებ მეწყება პირის სიმშრალე,
შეგრზნება მაქვს რომ ნერწყვს ვერ ვყლაპავ და ვიხრჩობი+თითქოს ყელი შიგნიდან მაქვს შეშუპებული, რისი ბრალი შეიძლება იყოს?

Posted by: FUA 3 Dec 2008, 22:09
დოპლერის გამოკვლევა სად კეთდება და რამდენი ჯდება?

Posted by: TETISA 4 Dec 2008, 00:53
საერთო გამოკვლევის ჩატარება მინდა. ყველაფრის ანალიზი. ხოდა როგორ იწყება ხოლმე ეს? ჯერ სისხლი ბარდება, თუ როგორ? თანმიმდევრობა მაინტერესებს

Posted by: texasuri jleta benzoxerxit 4 Dec 2008, 01:22
QUOTE (GHG @ 3 Dec 2008, 17:17 )
მოკლედ მჭირს ესეთი რაღაც, უცებ მეწყება პირის სიმშრალე,
შეგრზნება მაქვს რომ ნერწყვს ვერ ვყლაპავ და ვიხრჩობი+თითქოს ყელი შიგნიდან მაქვს შეშუპებული, რისი ბრალი შეიძლება იყოს?

ყველა ვარიანტში უნდა შემოწმდეს გლუკოზა სისხლში(შაქარი)ჭამის წინ და მერე.
შეიძლება ნევროტულიც იყოს.
რამე მედიკამენტს ხომ არ იღებ?
გახშირებული შარდვა და გაძლიერებული წყურვილი?

Posted by: yellow_yellow 4 Dec 2008, 12:09
თითქმის ყოველთვის ფეხები მაქვს გაყინული ეს საშიშია? რისი ბრალია?

Posted by: GHG 4 Dec 2008, 15:53
texasuri jleta benzoxerxit
QUOTE
რამე მედიკამენტს ხომ არ იღებ?

კი ნაღვლისბუშტის ანთება მაქვს და ვმკურნალობ.
QUOTE
გაძლიერებული წყურვილი?

როდესაც სპაზმი მაქვს პირი მიშრება საშინლად.

Posted by: texasuri jleta benzoxerxit 4 Dec 2008, 17:29
GHG
რა მედიკამენტებს?ზოგიერთი სპაზმოლიზური საშუალება იძლევა პირის სიმშრალეს.
გაძლიერებული წყურვილი და ხშირი შარდვა არა?
შაქარი გადაიმოწმე მაინც

Posted by: GHG 4 Dec 2008, 19:45
texasuri jleta benzoxerxit
QUOTE
რა მედიკამენტებს

პანტაპი
მოტილიუმი
დიუსპატალინი
ფრომოლიდი უნო
მეტრონიდაზოლი

QUOTE
გაძლიერებული წყურვილი და ხშირი შარდვა არა?

არა

Posted by: teakushki 5 Dec 2008, 21:03
ვინმე ხომ ვერ დამეხმარება პრედნიზოლონის 1მგ-ს რამდენი მგ დექსამეტაზონი შეესაბამება??

Posted by: vano_t 7 Dec 2008, 22:38
teakushki
QUOTE
ვინმე ხომ ვერ დამეხმარება  პრედნიზოლონის 1მგ-ს რამდენი მგ დექსამეტაზონი შეესაბამება??

1 მგ პრედნიზოლონი დაახლოებით უდრის 0,15 მგ დექსამეტაზონს.

http://www.globalrph.com/steroid.cgi არის სხვადასხვა სტეროიდების ექვივალენტური დოზების გამოთვლის პატარა აპლეტი. ძალიან მარტივია გამოსაყენებლად.

Posted by: Giaaaa 8 Dec 2008, 00:38
ოსტეო ხონდროზის მიზეზები გთხოვთ მომაწოდოთ ინფორმაცია ამ დავადების შესახებ

Posted by: SOFTT 8 Dec 2008, 17:42
ცხვირის ოპერაცია 16 წლის ასაკში გართულებებს არ გამოიწვევს?
სუნთქვაში ხელი არ მეშლება ურალოდ ფორმის შეცვლა მინდა 16 ვარ და გავიკეთო ამ ასაკში თუ 17 უნდა ველოდო აუცილებლად?

Posted by: daiv55 9 Dec 2008, 18:33
მგონი აქ უნდა ვიკითხო - როგორ მოვიშორო ხორცმეტები ( ბარადავკა)...ძმრის ესენცია მირჩია ერთმა, კაია? არადა მრავლდება და მრავლდება...წინასწარ მადლობთ... love.gif

Posted by: vano_t 9 Dec 2008, 21:26
daiv55
QUOTE
მგონი აქ უნდა ვიკითხო - როგორ მოვიშორო ხორცმეტები ( ბარადავკა)...ძმრის ესენცია მირჩია ერთმა, კაია? არადა მრავლდება და მრავლდება...წინასწარ მადლობთ... love.gif

მოჭრა, მოწვა ან მოყინვა ექიმის მიერ ოფისში. სამივე წამიანი პროცედურებია და ეფექტური. ექიმმა უნდა შეხედოს პრინციპში ყველა ვარიანტში, რომ დიაგნოზი დასვას. ხანდახან, რასაც ავადმყოფი ხორცმეტს ეძახის შეიძლება იყოფ ნეიროფიბრომა.

Posted by: SOFTT 9 Dec 2008, 21:28
vano_t
ცხვირის ოპერაცია 16 წლის ასაკში გართულებებს არ გამოიწვევს?
სუნთქვაში ხელი არ მეშლება ურალოდ ფორმის შეცვლა მინდა 16 ვარ და გავიკეთო ამ ასაკში თუ 17 უნდა ველოდო აუცილებლად?

Posted by: shtori 10 Dec 2008, 00:13
მუცელი მტკივა ხოლმე, მთლიანად მუცლის კანი შიდა მხრიდან (კუნთია თუ რაცაა თუ მუცლის ფარი თუ რა ჰქვია?), ძილის დროს, მხოლოდ მაშინ როცა გულაღმა მძინავს (ძილში ვერ ვაკონტროლებ აღმას და დაღმას), მაღვიძებს ისე მაგრად მტკივა თან. რის გამო შეიძლება იყოს? (კუჭთან პრობლემები არ მაქვს)
კიდე მარჯვენა მხრის ძვალთან, ზურგზე, კანი მაქვს დაბუჟებულისავით ან თითქოს რამე მაქვს გადაკრული. სულ ესე მაქვს. დისკომფორტს მიქმნის რითი მოვიშორო?
ხოდა ეს ისე - თავის კანი მიბუჟდება და ნორმალურია? არვისზე გამიგია მსგავსი

უზარო შეკითხვებს ვსვამ ალბათ, მაგრამ მაინტერესებს ხოლმე რა... თუ ძააან არ დაგეზარებათ.. smile.gif

გმადლობთ

Posted by: vano_t 10 Dec 2008, 00:23
SOFTT
QUOTE
ცხვირის ოპერაცია 16 წლის ასაკში გართულებებს არ გამოიწვევს?
სუნთქვაში ხელი არ მეშლება ურალოდ ფორმის შეცვლა მინდა 16 ვარ და გავიკეთო ამ ასაკში თუ 17 უნდა ველოდო აუცილებლად?
მაგ კითხვაზე პასუხს მე კარგად ვერ გაგცემ. ჯობია ეგ კითხვა პლასტიკურ ქირურგს (ან ყელ-ყურ-ცხვირის ექიმს) დაუსვა და იმათგან უფრო მიიღებ კომპეტენტურ პასუხს.

shtori
QUOTE
მუცელი მტკივა ხოლმე, მთლიანად მუცლის კანი შიდა მხრიდან (კუნთია თუ რაცაა თუ მუცლის ფარი თუ რა ჰქვია?), ძილის დროს, მხოლოდ მაშინ როცა გულაღმა მძინავს (ძილში ვერ ვაკონტროლებ აღმას და დაღმას), მაღვიძებს ისე მაგრად მტკივა თან. რის გამო შეიძლება იყოს? (კუჭთან პრობლემები არ მაქვს)
კიდე მარჯვენა მხრის ძვალთან, ზურგზე, კანი მაქვს დაბუჟებულისავით ან თითქოს რამე მაქვს გადაკრული. სულ ესე მაქვს. დისკომფორტს მიქმნის რითი მოვიშორო?
ხოდა ეს ისე - თავის კანი მიბუჟდება და ნორმალურია? არვისზე გამიგია მსგავსი
ასე ძალიან ძნელია რამის თქმა. ექიმმა უნდა გაგსინჯოს პირველ რიგში, რომ დაადგინოს სად გაქვს ტკივილი და რის გამო. შენს ადგილას ექიმს ვნახავდი.

Posted by: Fago† 10 Dec 2008, 01:38
ეს კითხვა არ ვიცი სად უნდა დამესვა მაგრამ მაინტერესებს შვრია (ე.წ. ჰერკულესი) რისთვის არის კარგი და როგორ უნდა მივირთვათ იგი? შეიძლება შაქართან ერთად?

Posted by: daiv55 10 Dec 2008, 14:02
vano_t მადლობთ! !


Posted by: EXHALATING 11 Dec 2008, 18:23
პატარა დაკვალიანება მჭირდება smile.gif ჩემს დას აქვს საშინელი თავის ტკივილები უკვე რამდნეიმე წელია.რაღაც პერიოდი დაუცხრება, მერე ისევ ხელახალი ტალღა. რამდენჯერმე იყო ექიმთან,გაიკეთა გამოკვლევები, ზოგმა უთხრა არაფერი არააო, 1მა შიდა ქალის წნევაო, დანიშნულება მისცა და წამლის ანოტაცია რომ წავიკითხე ანტიდეპრესანტი ყოფილა spy.gif
ახლა ისევ დაეწყო ტკივილები და იქნებ დამაკვალინოთ რომელ კლინიკაში მივიდეს ან სპეციალისტი მირჩიოთ კარგი, რა პროფილის ექიმს ენახვოს...
დიდი მადლობა წინასწარ

Posted by: =She= 11 Dec 2008, 21:54
ვინმემ დამეხმარეთ თუ შეგიძლიათ...
ერთი სიტყვით ვარ გერმანიაში, საშობაო ბაზრობაზე დავლიე "გლუვაინი" და სასტიკად მოვიწამლე, გული მერეოდა მთელი ღამე, გულზე თითქოს ლოდი მაწევდა ვერ ვსუნთქავდი და ძალიან ცუდად ვიყავი.
მას მერე ორი კვირა გავიდა დაახლოებით, საჭმელს რომ ვჭამ სულ გული მერევა, დილაობით რომ ვიღვიძებ გული მერევა, გრმად რომ ვისუნთქავ ტკივილის შეგრძნება მაქვს... ალბათ ნაღველი მაქვს გაღიზიანებული ან ღვიძლი, არ ვიცი, მირჩიეთ რა ვქნა ან რა მედიკამენტი მივიღო აქ რომ შესაძლებელი იყოს იმის ყიდვა ისეთი...
დიდი მადლობა!

Posted by: ana_nushka 12 Dec 2008, 16:25
გამარჯობა ყველას!!!!...
იქნებ დამეხმაროთ ვინმე. . . .

დღეს დამფხაჭნა პატარა ფისომ, აქამდე სულ სახლში იყო გარეთ არ უშვებდნენ. . .მაგრამ 2-3 დღის წინ ეზოში გაიქცა და საიდანღაც მკვდარი ვირთხა მოათრია. . . . drug.gif

კბენით არ უკბენია მაგრამ საკმაოდ დამფხაჭნა, სისხლამდე(აქამდეც დაუფხაჭნივარ თამაშის დროს მაგრამ ეხლა დავიზმენდი გარეთ რომ იყო გასული) . . . .

რაიმე აცრა გავიკეთო????ტეტანუსი ???? ცოფი????არის ამის აუცილებლობა???

Posted by: texasuri jleta benzoxerxit 13 Dec 2008, 00:21
QUOTE (ana_nushka @ 12 Dec 2008, 16:25 )
გამარჯობა ყველას!!!!...
იქნებ დამეხმაროთ ვინმე. . . .

დღეს დამფხაჭნა პატარა ფისომ, აქამდე სულ სახლში იყო გარეთ არ უშვებდნენ. . .მაგრამ 2-3 დღის წინ ეზოში გაიქცა და საიდანღაც მკვდარი ვირთხა მოათრია. . . . drug.gif

კბენით არ უკბენია მაგრამ საკმაოდ დამფხაჭნა, სისხლამდე(აქამდეც დაუფხაჭნივარ თამაშის დროს მაგრამ ეხლა დავიზმენდი გარეთ რომ იყო გასული) . . . .

რაიმე აცრა გავიკეთო????ტეტანუსი ???? ცოფი????არის ამის აუცილებლობა???

ტეტანუსიც და ცოფიც ყველა ვარიანტში!!! ძალიან მაღალი რისკი გაქვს დასნებოვნების...ვირთხა ერთ-ერთი პირველია ცოფის გადამტანთა შორის!!!

საშინელებაა ცოფით სიკვდილი...თანაც ადამიანი ბოლომდე გონზეა...

Posted by: ana_nushka 13 Dec 2008, 12:15
texasuri jleta benzoxerxit

მადლობაsmile.gif
ტეტანუსზე ავიცერიი. . . .აიი ცოფზე არა. . . .ნაკაწრია, ხომ არ უკბენიაა. . . .და რავი დავაკვირდები ფისოს ამ დღეებში. . .რაღაც არამგონია რამე ჭირდეს smile.gif

Posted by: Solveig 13 Dec 2008, 16:17
=She=

კი მაგრამ რა იყო მაგ გლუვაინში ასეთი, მთელი გერმანია მაგას სვამს ახლა და.....იქნებ არაა მაგისი ბრალი?

დაზღვევა არ გაქვს? თუ საიდუმლო არ არის, მანდ რა სტატუსი გაქვს? რაც ჩამოთვალე, მაგის მიხედვით უკეთესია, რომ ექიმთან მიხვიდე...თუ ღვიძლის ან ნაღვლის ბუშტის პრობლემაა, მე მგონი მაგისი წამლები გერმანიაში ურეცეპტოდ არ გაიცემა sad.gif.


Posted by: =She= 13 Dec 2008, 20:12
Solveig
არაა საიდუმლო რა თქმა უნდა, ვსწავლობ აქ...(გერმანულ ენას)
ხო რა ვიცი, მეც ვხედავ რომ მთელი გერმანია სვავს, მაგრამ სუნს რომ ვგრძნობ მაგისას ვკვდები...
კი მაქვს დაზღვევა, მაგრამ ამ დაზღვევით შორს ვერ გაფრინდები, უნდა გადავიხადო მაინც, ქალაქი სადაც ვცხოვრობ ძალიან ძვირი ქალაქია, მე კი ჩემი მწირი ბიუჯეტით ყველაფერზე ეკონომიის გაკეთება მიწევს, სამწუხაროდ.

Posted by: Solveig 13 Dec 2008, 20:56
=She=
QUOTE
უნდა გადავიხადო მაინც

იქნებ შენი სადაზღვევო კომპანიისათვის დაგერეკა და გაგერკვია-როგორ და რას გადაგიხდიან? ეცადე, ცოტა ხანს კვებითაც შეუწყო თავს ხელი-ანუ, შემწვარი, ცხიმიანი საკვები არ მიიღო.

უფ, ისე, საკუთარ ტყავზე მაქვს გამოცდილი, რომ გერმანიაში ექიმამდე მიღწევაც ძნელია sad.gif

Posted by: WuTheRinG_HeiGhts 14 Dec 2008, 15:10
Solveig

იქნებ იცოდეთ?!

მოკლედ დიაგნოზი მაქვს ასეთი, სტაფილოკოკი-ეპიდერმისი, ამასწინათ ჩემს სადაზღვევო კომპანიის კანის ექიმთან რომ ვიყავი პირველ კონსულტაციაზე და რომ ვუთხარი ესა და ესათქო, ეგ ყველა ადამიანს აქვს, ოღონდ ყველას არ გამოუვლინდებაო.... მეთქი ცხიანი კანი მაქვს ძალიანთქო.დამინიშნა Salve-ს ფირმის სახის თხევადი საპონი, რომელიც საპონს არ შეიცავს და კანს საკაიფოდ აშრობს(ეგ კი კარგია) იგივე ფირმის "ანტიაკნეალი" მაზი-(ეფექტი არ მიგრძვნია) და პლიუს სეპტრინი.(ნუ ვითომ რაღაცა მაგრამ ეგეც ეფექტის გარეშე)

მოკლედ ადრე ერთ ცნობილ დერმატოლოგთან დავდიოდი, კი ბატონო ეფექტური კი იყო, მაგრამ დროის გარკვეულ მონაკვეთებში.... ანუ ანტიბიოტიკურ მკურნალობას ვგულისხმობ და გარეგანი რაღაცის წასმას, როცა ანტიბიოტიკებს ვიღებდი ის პერიოდი გადატკიცინებული მქონდა სახე და მაგრამ მერე ისევ მეწყებოდა,,, მოკლედ იმდენი ანტიბიოტიკი მივიღე რომ შევეშვი რა მასთან სიარულს. მეთქი ღვიძლს რამე არ ვავნო.....

ახლა რა არის იცი? ანტიბიოტიკს არა, მაგრამ "სეპტრინს" რომელიც არ არის ანტიბიოტიკი-მაგას ვსვამ 5 დღის განმავლობაში, მერე დაახლოებით 2 - 3 კვირა ფანტასტიურად მაქვს კანი შემდეგ ისევ იჩენს თავს პატარ პატარა პრობლები.....უმეტესად პირის ირგვლივ და ცხვირის გვერდებში.

ცუდია სეპტრინის ამდენი მიღება???

და საერთოდ ნუთუ ჩემი ცხოვრების მძიმე ჯვარია ეს ? დავიღალე რაააა........ფსიქოლოგიურად ისე მოქმედებს რომ.....რა გითხრა

sad.gif
ერთხელ და საბოლოოდ რომ განვიკურნო რა იქნება......
* * *
QUOTE
დამინიშნა Salve-ს ფირმის სახის თხევადი საპონი, რომელიც საპონს არ შეიცავს და კანს საკაიფოდ აშრობს(ეგ კი კარგია) იგივე ფირმის "ანტიაკნეალი" მაზი-(ეფექტი არ მიგრძვნია) და პლიუს სეპტრინი.(ნუ ვითომ რაღაცა მაგრამ ეგეც ეფექტის გარეშე)


შემეშალა სეპტრინი კი არა ზინერიტი დამინიშნა


სეპტრინს ისე ვსვამ ხოლმე

Posted by: texasuri jleta benzoxerxit 14 Dec 2008, 15:44
[b]WuTheRinG_HeiGhts[/

სეპტრინის(სულფანილამიდების) ასეთი უსისტემო და ხშირი მიღება ყოვლად დაუშვებელია!!! ძალიან ცუდად მოქმედებს სისხლწარმოქმნაზე!!!

შეიქმნა ახალი მიმდინარეობა-დერმატოფსიქიატრია! სადაც დერმატოლოგიური პრობლემები განიხილება ფსიქიკური აშლილობების ნიღბად,ან მათ გამომწვევად ან მათ მიერ პროვოცირებულად...ასეა თუ ისე ამ ორი სფეროს პათოლოგლიური მდგომარეობები უხშირესად თანაარსებობენ(კომორბიდულები არიან) და ამძიმებენ ერთმანეთს.
ამ მიმდინაროებას საფუძველი ჩაუყარეს ფსიქიატრმა შოთა გამყრელიძემ, და დერმატოლოგმა ნინო ცისკარიშვილმა (კან.ვენ-ის დისპანსერი).
დერმატოლოგიური პრობლემების დროს აუცილებელია კომპლექსური მიდგომა ავადმყოფთან და მისი ფსიქიკური პროცესების დარეგულირება!

Posted by: WuTheRinG_HeiGhts 14 Dec 2008, 20:04
texasuri jleta benzoxerxit

დიდი მადლობა პასუხისთვის.

QUOTE
დერმატოლოგიური პრობლემების დროს აუცილებელია კომპლექსური მიდგომა ავადმყოფთან და მისი ფსიქიკური პროცესების დარეგულირება!


მართალი ხარ, ფსიქიურად ძაან დარხეული მაქვს, მაგალითად გუშინ მამა გაბრიელის საფლავზე ვიყავი, საფლავზე იმდენი ხალხი იყო ირგვლივ რომ ვერა და ვერ წავიწიე წინ რომ ხელი დამედო საფლავის მიწისთვის, და უცბად ისე განვიცადე წინ რომ ვერ მივდიოდი-ტყვიანაკრავივით შემოვტრიალდი და გამოვიქეცი მანქანისკენ, ჩავხტი და მივადგი გაზს, პატარა საცობი იყო იქვე და ისეთი სიძლიერით ვუჭერდი საჭეს ხელს(ვასიგნალებდი) რომ კინაღამ მოვტეხე.
თბილისის შესასვლელთან ბენზოგასამართ სადგურთან გავჩერდი და სინანულით კინაღამ ტირილი დავიწყე რომ მამა გაბრიელის საფლავზე ჩასული სულ 30 სანტიმეტრში მდგომი ვიყავი და ის ვერ მოვითმინე რომ კიდევ მომეცადა(ანუ რაღაც ისტერიულ განკითხვაში ჩავვარდი, ამდენი ხალხია და ის ვერ დაურეგულირებიათ რომ რიგი გაეკეთებინათთქო)


QUOTE
სეპტრინის(სულფანილამიდების) ასეთი უსისტემო და ხშირი მიღება ყოვლად დაუშვებელია!!! ძალიან ცუდად მოქმედებს სისხლწარმოქმნაზე!!!

ამის წაკითხვის შემდეგ ვიყიდე სეპტრინი 20 აბი და ხვალიდან დავიწყებ ისევ, ჯერ არ მაქვს არაფერი მაგრამ ვგრძნობ რომ ხვალიდან ისევ დამეწყება პრობლემები

ხოო ვიცი ფიქრობ რომ ეს ბიჭი გაგიჟებულაო, კი მართალიც ხარ! ასეა ალბათ yes.gif

Posted by: Solveig 15 Dec 2008, 14:04
რაკი დერმატოლოგიაზე ჩამოვარდა საუბარი-წითელ ქარს ქავილი ახლავს თან?

Posted by: jashusha 15 Dec 2008, 15:26
QUOTE (texasuri jleta benzoxerxit @ 30 Nov 2008, 14:18 )
rock in rose
გირჩევ ოტო-რინო-ლარინგოლოგთან და ალერგოლოგთან ვიზიტს

ვინ არის ყველზე კარგი? ან რომელ კლინიკაში ჯობს მისვლა? მეც სიცივეზე მაქვს ალერგია და რაიმე წვეთები მჭირდება.

Posted by: EXHALATING 15 Dec 2008, 16:16
QUOTE
პატარა დაკვალიანება მჭირდება smile.gif ჩემს დას აქვს საშინელი თავის ტკივილები უკვე რამდნეიმე წელია.რაღაც პერიოდი დაუცხრება, მერე ისევ ხელახალი ტალღა. რამდენჯერმე იყო ექიმთან,გაიკეთა გამოკვლევები, ზოგმა უთხრა არაფერი არააო, 1მა შიდა ქალის წნევაო, დანიშნულება მისცა და წამლის ანოტაცია რომ წავიკითხე ანტიდეპრესანტი ყოფილა spy.gif ახლა
ისევ დაეწყო ტკივილები და იქნებ დამაკვალინოთ რომელ კლინიკაში მივიდეს ან სპეციალისტი მირჩიოთ კარგი, რა პროფილის ექიმს ენახვოს... დიდი მადლობა წინასწარ




heeeeelp

Posted by: vano_t 15 Dec 2008, 21:02
EXHALATING
QUOTE
პატარა დაკვალიანება მჭირდება smile.gif ჩემს დას აქვს საშინელი თავის ტკივილები უკვე რამდნეიმე წელია.რაღაც პერიოდი დაუცხრება, მერე ისევ ხელახალი ტალღა. რამდენჯერმე იყო ექიმთან,გაიკეთა გამოკვლევები, ზოგმა უთხრა არაფერი არააო, 1მა შიდა ქალის წნევაო, დანიშნულება მისცა და წამლის ანოტაცია რომ წავიკითხე ანტიდეპრესანტი ყოფილა spy.gif ახლა
ისევ დაეწყო ტკივილები და იქნებ დამაკვალინოთ რომელ კლინიკაში მივიდეს ან სპეციალისტი მირჩიოთ კარგი, რა პროფილის ექიმს ენახვოს... დიდი მადლობა წინასწარ
რა სიხშირით აქვს თავის ტკივილები? რა ნაწილი ტკივა? რამდენ ხანს გრძელდება ტკივილი? გულსირევა ან ღებინება თუ აქვს ტკივილს დროს? სინათლეს და ხმაურს თუ გაურბის ტივილის შესამცირებლად? რამდენი წელია აქვს ტკვილი (შენ ახსენე რამედენიმე წელი-თუ მართლაც დიდი ხანი აქვს ტკივილები, მაშინ საშიში თავისთავად არაფერია)? თუ აქვს ცრემლდენა ან ცხვირიდან გამონადენი ტკივილის დროს? ოჯახში ვინმეს თუ ქონდა ანევრიზმა თავის ტვინის სისხლძაღვის?

თავის ტკივილები, საერთოდ, თუ დიდი ხანი გრძელდება, არ არის საშიში. ერთადერთი რამ რაც შეიძლება ქრონიკული თავის ტკივილის დროს საშიში იყოს, არის ე.წ. ცერებრალური ანევრიზმა-თავის ტვინის სისხლძაღვრების გაგანიერება. ამის დიაგნოზი ხდება ბირთვულ-მაგნიტური ანგიოგრაფიით და თუ საჭირო გახდა კონტრასტული ინტრაარტერიული ანგიოგრაფიით.

ახალგაზრდა გოგოებში და ქალებში, ყველაზე ხშირი მიზეზი თავის ტკივილსა არის მიგრენი და ე.წ. ტენზიური თავის ტკივილები.

თუ უფრო მეტ ინფორმაციას დადებ, უფრო დაგაკვალიანებ. ისე თავის ტკივილს იკვლევს თავიდან ტერაპევტი და თუ თერაპევტი დიაგნოზს ვერ დასვამს, ან დასვამს მაგრამ მის მიერ დანიშნული მკურნალობა ვერაფერს შვება, მაშინ ავადმყოფი იგზავნება ნვროლოგთან (ანუ ნევროპათოლოგთან).

Posted by: EXHALATING 15 Dec 2008, 21:07
vano_t
შენს მიერ ჩამოთვლილი ჩივილებიდან არ აქვს არც ერთი თითქმის. დიდი მადლობა ყურადღებისთვის,ნევროპათოლოგთან გავგზავნი..

Posted by: tatunia_22 15 Dec 2008, 21:39
24 თუ 25 ნოემბერს ვიყავი ექიმთან. ფეხმძიმედ ვარ და მიწევდა გეგმიური ვიზიტი, რომელშიც შედიოდა თერაპევტის კონსულტაციაც. მოკლედ გამსინჯა ამ თერაპევტმა. ფილტვებზეც მომისმინა და მითხრა, რომ აბსოლიტურად სუფთაა. (ისე ცნობისათვის, ვარ 24 წლის და არასოდეს არ მქონია არც ფილტვების ანთება არც ბრონქების რაიმე პრობლება).
დაახლოებით 10 დღის წინ დამეწყო ხველა, მაქვს ნახველი და სავარაუდოდ ბრონქები გამიცივდა (ჩემი დიაგნოზით).

ხოდა ეხლა რა მაინტერესებს, შეიძლება, რომ მქონდეს ტუბერკულიოზი sad.gif და 3 კვირის წინ არაფერი არ გამოჩენილიყო ფილტვებში?

ცოტა მშიშარა ვარ მიხვდებოდით ალბათ smile.gif

Posted by: vano_t 16 Dec 2008, 02:20
tatunia_22
QUOTE
24 თუ 25 ნოემბერს ვიყავი ექიმთან. ფეხმძიმედ ვარ და მიწევდა გეგმიური ვიზიტი, რომელშიც შედიოდა თერაპევტის კონსულტაციაც. მოკლედ გამსინჯა ამ თერაპევტმა. ფილტვებზეც მომისმინა და მითხრა, რომ აბსოლიტურად სუფთაა. (ისე ცნობისათვის, ვარ 24 წლის და არასოდეს არ მქონია არც ფილტვების ანთება არც ბრონქების რაიმე პრობლება).
დაახლოებით 10 დღის წინ დამეწყო ხველა, მაქვს ნახველი და სავარაუდოდ ბრონქები გამიცივდა (ჩემი დიაგნოზით).

ხოდა ეხლა რა მაინტერესებს, შეიძლება, რომ მქონდეს ტუბერკულიოზი  sad.gif და 3 კვირის წინ არაფერი არ გამოჩენილიყო ფილტვებში?

ცოტა მშიშარა ვარ მიხვდებოდით ალბათ smile.gif

მწვავე რესპირატორულ (ანუ სასუნთქი გზების) ინფექციაზე არავინ ფიქრობს ტუბერკულოზზე. უნდა დაელოდო და ნახო რამდენ ხანს გრძელდება სიმპტომები. რა თქმა უნდა, დალოდებამდე ექიმი გადაწყვიტავს არის თუ არა მკურნალობა საჭირო (ზემო სასუნთქი გზების ინფექციის მკურნალობაზეა საუბარი და არა ტუბერკულოზის). რაც შეეხება ტუბერკულოზს, ეგ ისეც უნდა შემოწმდეს ორსულში. იმიტომ კი არა, რომ მაგ ინფექციის ნიშნები გაქვს, არამედ ყველა ორსულს უკეთდება კანის ტესტი (მანტუს ეძახიან ამ ტესტს, ოღონდ დღესდღეობით აკეთებენ მანტუს უფრო დახვეწილ ფორმას PPD-ს) ტუბერკულოზის გამოსარიცხად.

Posted by: Solveig 16 Dec 2008, 09:29
jashusha
QUOTE
მეც სიცივეზე მაქვს ალერგია და რაიმე წვეთები მჭირდება.

სიცივეზე თუ გაქვს "ალერგია" , ყველაზე კარგი საშუალებაა ნორმალური და მუდმივი ტემპერატურა..რა წვეთებზეა საუბარი? ისე, მთავარი იმის დადგენაა, რომ მართლა სიცივითაა გამოწვეული და არა სხვა რამით...


Posted by: an1-)) 19 Dec 2008, 11:21
ხალხო ერთი თხოვნა მაქვს და აუცილებლად უნდა მიშველოთ. ჩემს კატას ცუდი სუნი აქვს პირში და მოშორება ან შემცირება არ შეიძლება? . ცხოველებისთვის არ არსებობს რამე საშუალება პირიდან სუნის გასაქრობად? თუ ვინმემ იცით სასწრაფოდ მიპასუხეთ რააააააააbiggrin.gif:D

Posted by: E-36 19 Dec 2008, 14:28
QUOTE (an1-)) @ 19 Dec 2008, 11:21 )
ხალხო ერთი თხოვნა მაქვს და აუცილებლად უნდა მიშველოთ. ჩემს კატას ცუდი სუნი აქვს პირში და მოშორება ან შემცირება არ შეიძლება? . ცხოველებისთვის არ არსებობს რამე საშუალება პირიდან სუნის გასაქრობად? თუ ვინმემ იცით სასწრაფოდ მიპასუხეთ რააააააააbiggrin.gif:D

http://forum.ge/?showforum=72 ...
http://forum.ge/?f=72&showtopic=33905479 აქ იკითხეთ smile.gif ....

Posted by: hirondelle 19 Dec 2008, 14:56
ტუჩის კუთხეში რაცხა გამომეზარდა, იქერცლებასავით, ერთი კვირა არ მშორდება, ვიღაცამ მითხრა ჰერპესი იქნებაო.

რა ჯანდაბაა ეს ჰერპესი ხომ ვერ მეტყოდით, საშიშია? და როგორ მოვიშორო?

Posted by: HAMZAT 19 Dec 2008, 16:01
მაინტერესებს რა დაავადებებს იწვევს გლანდები, შეიძლება გლანდებმა ვეგეტო ნერვოზის პროვოცირება მოახდინოს, ან ტაქიკარდიის
* * *
QUOTE (hirondelle @ 19 Dec 2008, 14:56 )
ტუჩის კუთხეში რაცხა გამომეზარდა, იქერცლებასავით, ერთი კვირა არ მშორდება, ვიღაცამ მითხრა ჰერპესი იქნებაო.

რა ჯანდაბაა ეს ჰერპესი ხომ ვერ მეტყოდით, საშიშია? და როგორ მოვიშორო?

ნერვოპათოლოგთან მიდი, ნერვების დაბოლოებების ანთებაა, ეგ ძაან ვრედნი დაავადებაა, მაგან ტკივილები იცის იმის დედა ვატირე, დიმედროლი , ბარალგინი ძვლივს აყუჩებს. სავარაუდოდ კასტელანის სითხეს დაგინიშნავს. და კიდევ რაგაც წამლებს.

Posted by: texasuri jleta benzoxerxit 19 Dec 2008, 17:05
HAMZAT
"გლანდებმა" შეიძლებ აგამოიწვიოს რევმატიზმი,რევმატული მიოკარდიტი(შესაბამისად ტაქიკარდიაც),მწვავე გომერულონეფრიტი...

თუ ქრონიკული ჩირქოვანი ტონზილიტია აუცილებლად ამოიჭერი

Posted by: HAMZAT 19 Dec 2008, 18:38
QUOTE (texasuri jleta benzoxerxit @ 19 Dec 2008, 17:05 )
HAMZAT
"გლანდებმა" შეიძლებ აგამოიწვიოს რევმატიზმი,რევმატული მიოკარდიტი(შესაბამისად ტაქიკარდიაც),მწვავე გომერულონეფრიტი...

თუ ქრონიკული ჩირქოვანი ტონზილიტია აუცილებლად ამოიჭერი

ჩირქიანი არაა, მარა აი, რევმატიზმები კი მაქვს yes.gif

Posted by: vano_t 19 Dec 2008, 22:48
hirondelle
QUOTE
ტუჩის კუთხეში რაცხა გამომეზარდა, იქერცლებასავით, ერთი კვირა არ მშორდება, ვიღაცამ მითხრა ჰერპესი იქნებაო.

რა ჯანდაბაა ეს ჰერპესი ხომ ვერ მეტყოდით, საშიშია? და როგორ მოვიშორო?

ჰერპესი საშიში არ არის. ეგ არის ვირუსული ინფექცია და როცა ადამიანი ამ ინფექციას შეიძენს, მერე ეს ვირუსი ყოველთვის ორგანიზმშია. სტრესის დროს (მაგალითად გაციების დროს) გააქტიურდება და ტუჩებთან გამონაყარს იძლევა. თავისით გადის ეგ გამონაყარი ისე. ხანდახან ადგილობრივმა ანტივირუსულმა წამლებმა შეიძლება შეამციროს ხანგრძლივობა. მარა, უფრო საჭიროა ადგილობრივი ანესთეიკის მაზი (მაგალითად ბენზოკაინის მაზი), რომელიც მწვავე ტკივილებს შეგიმცირებს.

Solveig
QUOTE
რაკი დერმატოლოგიაზე ჩამოვარდა საუბარი-წითელ ქარს ქავილი ახლავს თან?
უკაცრავად, ეს კითხვა ახლა ვნახე და გვიან გცემ პასუხს. წითელი ქარს, ანუ ერიზიპელასს, ეძახიან მემგონი სტრეპტოკოკულ ცელულიტს (cellulitis) რომელიც კანქვეშა ქსოვილის ინფექციაა. როგორც წესი ამას ახასიათებს ქავილიც და წვის შეგრძნებაც. cellulitis განსხვავდება cellulite-გან. cellulitis არის ინფექცია, რასაც მიუთითებს itis დაბოლოება. cellulite არის კიდევ კანის იმ მდგომარეობას, ქალებს რომ ადარდებთ ძალიან კოსმეტიკური თვალსაზრისით.

Posted by: giorgi33 20 Dec 2008, 00:20
მაქვს ქრონიკული ქოლეცისტიტი( დეფორმირებული ნაღველი ნაკეცით) და ც ჰეპატიტი ფიბროზის 1 სტადია, სიცივეზე მავქს მარჯვენა ფერდზე ტკივილი, რამოდენიმე ექიმს ვკითხე და მარტო ეხო ვინც გადამიღო მან მითხრა რომ ნაღველის ბრალი იქნებაო, სხვამ ზოგმა ხეხემალიო ზოგმა ნერვის ანთებაო. თქვენ ხომ ვერ მეტყოდით რისი ბრალი შეიძლება იყოს?
კიდევ თირკმელზე ასეთი რამ ჩამიწერეს და სურათიც გამომატანეს ეხოსი. ,, მარჯვენა თირკმლის შუა მესამედში დიფერენცირდება თიკმლის პარენქიმის, ბერტელის სვეტი, რომელიც ჰყოფს თირკმელს სამ ნაწილად,, ვინ ექოსკოპია გადამიგო მითხრა ასეთი რამ არასდროს შემხვედრიაო, მაგრამ საშიში არაფერიაო, იქნებ ბერტელის სვეტზეც მითხრათ ერთი ორი სიტყვით.

Posted by: EKIKULI 20 Dec 2008, 01:27
..................................

Posted by: hirondelle 20 Dec 2008, 04:37
vano_t
QUOTE
რომელიც მწვავე ტკივილებს შეგიმცირებს


ტკივილი საერთოდ არა მაქვს და სარკეში თუ არ ჩავიხედე ვერც ვგრძნობ რომ არსებობს

Posted by: vano_t 20 Dec 2008, 10:56
giorgi33
QUOTE
მაქვს ქრონიკული ქოლეცისტიტი( დეფორმირებული ნაღველი ნაკეცით) და ც ჰეპატიტი ფიბროზის 1 სტადია, სიცივეზე მავქს მარჯვენა ფერდზე ტკივილი, რამოდენიმე  ექიმს ვკითხე და მარტო ეხო ვინც გადამიღო მან მითხრა რომ ნაღველის ბრალი იქნებაო, სხვამ ზოგმა ხეხემალიო ზოგმა ნერვის ანთებაო. თქვენ ხომ ვერ მეტყოდით რისი ბრალი შეიძლება იყოს?
კიდევ თირკმელზე ასეთი რამ ჩამიწერეს და სურათიც გამომატანეს ეხოსი.  ,, მარჯვენა თირკმლის შუა მესამედში დიფერენცირდება თიკმლის პარენქიმის, ბერტელის სვეტი, რომელიც ჰყოფს თირკმელს სამ ნაწილად,, ვინ ექოსკოპია გადამიგო მითხრა ასეთი რამ არასდროს შემხვედრიაო, მაგრამ საშიში არაფერიაო, იქნებ ბერტელის სვეტზეც მითხრათ ერთი ორი სიტყვით.

ბევრი რამ შეიძლება იყოს და ასე გამოკლვევის (გასინჯვის) გარეშე ძნელია თქმა. მარტო სიცივის დროს გტკივა? ჭამის დროს? ჩასუნთქვა ამოსუნთქვისას? მაგ მხარეზე წოლისას?

ქრონიკული ქოლეცისტიტი თუ გაქვს, მაშინ მაგას შეუძლია მარჯვენა მხარეს ტკივილი მოგცეს. თუ ძვლების, კუნთების, პლევრის და სხვა პრობლემები გამოირიცხა, მაშინ შესაძლებელია მაგის გამო იყოს. საერთოდ, თუ მარტო სიცივეზე გაქვს ტკივილი და ლაბორატორიები ნორმალურია, მაშინ სერიოზული რამ არ არის ალბათ. მთავარია თბილად შეინახო თავი.

hirondelle
QUOTE
ტკივილი საერთოდ არა მაქვს და სარკეში თუ არ ჩავიხედე ვერც ვგრძნობ რომ არსებობს
მაშინ არარაფერია სამკურნალო. დაელოდე და გავა თავისით გამონაყარი.

Posted by: karaoke 20 Dec 2008, 11:18
giorgi33
QUOTE
კიდევ თირკმელზე ასეთი რამ ჩამიწერეს და სურათიც გამომატანეს ეხოსი. ,, მარჯვენა თირკმლის შუა მესამედში დიფერენცირდება თიკმლის პარენქიმის, ბერტელის სვეტი, რომელიც ჰყოფს თირკმელს სამ ნაწილად,, ვინ ექოსკოპია გადამიგო მითხრა ასეთი რამ არასდროს შემხვედრიაო, მაგრამ საშიში არაფერიაო, იქნებ ბერტელის სვეტზეც მითხრათ ერთი ორი სიტყვით.

თირკმელების დაავადებასთან რაღაც კავშირი აქვს ც ჰეპატიტს.
http://ndt.oxfordjournals.org/cgi/content/full/16/4/869-a
Hepatitis C-associated glomerulonephritis—a novel therapeutic approach

ეს შეამოწმე სულ მცირე .... თუ ც ჰეპატიტი აღმოჩნდა დამნაშავე მაშინ რაც სწრაფად დაიწყებ ანტივირუსულ მკურნალობას მით უკეთესი.

Posted by: vano_t 20 Dec 2008, 12:20
giorgi33
QUOTE
,, მარჯვენა თირკმლის შუა მესამედში დიფერენცირდება თიკმლის პარენქიმის, ბერტელის სვეტი, რომელიც ჰყოფს თირკმელს სამ ნაწილად,, ვინ ექოსკოპია გადამიგო მითხრა ასეთი რამ არასდროს შემხვედრიაო, მაგრამ საშიში არაფერიაო, იქნებ ბერტელის სვეტზეც მითხრათ ერთი ორი სიტყვით.

ბერტელის სვეტი თუ ბერტინის სვეტი? (Bertin column, column of Bertin). თუ ამაზეა საუბარი, ეს ნორმალური სტრუქტურაა.


user posted image


ამ სურათზე #17 არის თირკმლის სვეტი. #1 არის ე.წ. პირამიდები და მათ შორის არსებულ ნაწილებს ეძახიან თირკმლის სვეტს (ანუ ბერტინის სვეტს-ფრანგი ანატომის ბერტინის საპატივცემულოდ). ხანდახან ეს ნაწილები შეიძლება ნორმაზე დიდი იყოს და ექოზე გამოჩნდეს. თუ ეს გამოჩნდა ექოზე, მაშინ ეგ ნორმაა (და როგორც ჩანს ეგ გამოჩნდა).

giorgi33, გლომერულონეფრიტზე არ ინერვიულო.

karaoke
QUOTE
თირკმელების დაავადებასთან რაღაც კავშირი აქვს ც ჰეპატიტს.
http://ndt.oxfordjournals.org/cgi/content/full/16/4/869-a
Hepatitis C-associated glomerulonephritis—a novel therapeutic approach
მთლად ნუ შეაშინება ახლა ამ კაცს smile.gif გლომერულონეფრიტი თირკმელების უკმარისობით მიმდინარე დაავადებაა. მაგას ექოზე ვერ დაინახავ. ეგ ჩანს სისხლში, როცა შარდოვანა და კრეატინინი იწევს და შარდის ანალიზი ჭირდება ფიზიკურ გამოკვლევასთან ერთად. მაგას არაფერი აქვს საერთო ბერტინის სვეტებთან.

Posted by: karaoke 20 Dec 2008, 13:02
მე არ მიტქვამს ინერვიულე თქო. მეც ვიყავი თბლისში 2 წლის წინ ჰეპატიტის ცენტრში მთელ მუცელზე გამიკეთეს ექოსკოპია და თირკმელები არც მე მქონია მწყობრში. თუმცა აქაური ექიმი არც კი დაინტერესებულა თირკმელებით...
ბევრნაირ ანალიზს მიკეტებდა სისხლისას და არ ექნებოდა მიზეზი ალბათ.

ისე მაშინ მეც შემეშინდა , ინეტში მქონდა წაკითხული ... თუმცა ისიც წერია ინეტში რო ძალიან ცოტას ემარტება ც ჰეპატიტის ის მატარებლებს .

მე ვუთხარი შეამოწმებიე მეთქი ...თავის დაზღვევის მიზნით.
თუ საშიში არაფერია მაგ მხრივ მაშინ მაგას რა ჯობია.

Posted by: giorgi33 21 Dec 2008, 01:27
vano_t, დიდი მადლობა,კი ბერტინის ქნება, ორგან გავიკეთე ეხო, ერთგან ბერტილი ეწერა. კიდევ იყო . კიდევ ასეტი სხვაობა იყო ერთგან მაჯვენა თიკმლის ზომა 118/58 მმ. მეორეგან 138/53 მმ, შეიძლება ოც დღეში ასე შეცვილიყო? თუ უბრალოდ მექანიკური შეცდომაა?

Posted by: vano_t 21 Dec 2008, 01:49
giorgi33
QUOTE
vano_t, დიდი მადლობა,კი ბერტინის ქნება, ორგან გავიკეთე ეხო, ერთგან ბერტილი ეწერა. კიდევ იყო . კიდევ  ასეტი სხვაობა იყო ერთგან მაჯვენა თიკმლის ზომა 118/58 მმ. მეორეგან  138/53 მმ, შეიძლება ოც დღეში  ასე შეცვილიყო? თუ უბრალოდ მექანიკური შეცდომაა?

მექანიკური შეცდომაა. მასეთი ცვლილება 20 დღეში ვერ მოხდება. ერთადერთი როცა თირკმელი სწრაფად მოიმატებს ზომაში არის როცა შარდის მოძრაობის შეწყვეტა ხდება (ჰიდრონეფროზს ეძახიან ამას) და ამის დადგენა ექოთი ძაან ადვილია. შენს შემთხვევაში არის ან შეცდომით ჩაწერილი ზომა, ან სხვადასხვანაირად წაკითხული. ხანდახან, თU სხვადასხვა ექოზე გაიკეთებ გამოკვლევას (ან სხვადასხვა სპეციალისტი წაიკითხავს) განსხვავებას მოგცემს ზომის.ეს ნორმალური განსხვავებაა მარა ამხელა განსხვავებას არ გაძლევს. ამიტომ,ჩემი აზრით ჩაწერის დროს შეეშალათ რაღაც.

Posted by: giorgi33 21 Dec 2008, 01:49
ეხლა ანალიზებს გადავხედე , მეგონა რომ ყვლაფერზე ამარებუნა ექიმმა რაც ც ჰეპატიიტტან კავშირში იყო, კრეატინინზე არ მქონია, სხვა სახელით ხომ არ შეიძლება რომ იყოს ანალიზებში? რკინა მქონია აწეული 177 მდე, თიკმელთან ხომ არ არის კავშირში/?

Posted by: vano_t 21 Dec 2008, 11:48
giorgi33
QUOTE
ეხლა ანალიზებს გადავხედე , მეგონა რომ ყვლაფერზე ამარებუნა ექიმმა რაც ც ჰეპატიიტტან კავშირში იყო, კრეატინინზე არ მქონია, სხვა სახელით ხომ არ შეიძლება რომ იყოს ანალიზებში? რკინა მქონია აწეული 177 მდე, თიკმელთან ხომ არ არის კავშირში/?

ეგ არის ძალიან მცირედ მომატებული რიკინის დონე და არაფერს წარმოადგენს. რიკინის აწევას საერთოდ თირკმელთან კავშირი არ აქვს. რკინის დონე ძალიან თუ აიწევს მიუთითებს ჰემოქრომატოზზე, რაც ც ჰეპატიტთან და თირკმლის პრობლემებთან კავშირში არ არის (ყოველ შემთხვევისათვის მე არ გამიგია). ხადახან წამლებმა შეიძლება აწიოს რკინა მაღლა.

Posted by: geo-usa 22 Dec 2008, 05:31
კუჭის პირთან ნეკნები რომ იკვრება იქ დამეწყო ყრუ ტკივილები და სიმძიმის შეგრძნება, ხანდახან ტკივილი უკან ბეჭებშიც გადაჰკრავს ხოლმე, უმთავრესად ეს ჭამის შემდეგ ხდება, მაგრამ ხანდახან სრულიად მშირ კუჭზეც იგივე ტკივილები მაქვს, ტკივილებთან ერთად დამეწყო ჭინჭრის ციებასავით გამონაყარი, ხელებსა და ფეხებზე რომელიც ძალიან მექავება და უაზროდ ჩნდება და ქრება, იღებს სხვადასხვაგვარ ფორმას და წითელი ფერისაა. ხომ ვერ მირჩევთ რამეს რა ვქნა როგორ მოვიქცე? რჩევისთვის წინასწარ გმადლობთ.

Posted by: vano_t 22 Dec 2008, 22:14
QUOTE (geo-usa @ 22 Dec 2008, 05:31 )
კუჭის პირთან ნეკნები რომ იკვრება იქ დამეწყო ყრუ ტკივილები და სიმძიმის შეგრძნება, ხანდახან ტკივილი უკან ბეჭებშიც გადაჰკრავს ხოლმე, უმთავრესად ეს ჭამის შემდეგ ხდება, მაგრამ ხანდახან სრულიად მშირ კუჭზეც იგივე ტკივილები მაქვს, ტკივილებთან ერთად დამეწყო ჭინჭრის ციებასავით გამონაყარი, ხელებსა და ფეხებზე რომელიც ძალიან მექავება და უაზროდ ჩნდება და ქრება, იღებს სხვადასხვაგვარ ფორმას და წითელი ფერისაა. ხომ ვერ მირჩევთ რამეს რა ვქნა როგორ მოვიქცე? რჩევისთვის წინასწარ გმადლობთ.

ჩემი რჩევა იქნება ნორმალური ექიმი ნახო. ასე დიაგნოზი არ დაისმება. ეგ შეიძლება იყოს გასტრიტი, წყლული, ქოლეცისტიტი ან სხვა რამ. ამიტომ, საჭიროა გამოკითხვა და ფიზიკური გამოკვლევა. ჩემი რჩევა იქნება ვინმე კარგი ექიმი ნახო.

Posted by: EKIKULI 24 Dec 2008, 11:40
ვინმეს შეგიძლიათ ამიხსნათ რა არის ეს და გაიწოვება თუ არა მკურნალობით?


ნაღვლის ბუშტი საშუალო ზომის 6,0X2,9 სმ, კედლები უმნიშვნელოდ შესქელებულია, დეფორმაციის გარეშე, ყელისა და ტანის საზრვარზე მოსჩანს,
1,3X0,5 სმ ზომის კედელთან ფიქსირებული ჰიპერექოგენური წარმონაქმნი, სავარაუდოდ ქოლესტერინული პოლიპი.

Posted by: SPC 26 Dec 2008, 00:22
თუ შეიძლება დამაკვალიანეთ რა იწვევს ხელებში ძალის გამოცლას?
რამე ვიტამინით შეიძლება ამ პრობლემის გამოსწორება?

1 წელია ხორცს არ ვჭამ და აღარც შევჭამ,. შეიძლება ეს იყოს მიზეზი?

თუმცა ეს მანამდეც მაწუხებდა, ანუ ხელებში ვგრძნობ უღონობას თუ როგორაა

ფეხებზე ყველაფერი წესრიგშია ამ დროს.



და აქვე მეორე კითხვას დავსვამ, გრძელი თმები მაქვს და ბოლოები მეყოფა, რკინის ანალიზი უნდა გავიკეთო და მერე მინდა მისვლა ვინმესთან ვინც უშუალოდ თმებზე სპეციფირდება, რა ჰქვია თმის ექიმს? და არის თუ არა ასეთი საქართველოში? თუ კი სად?

Posted by: pao 26 Dec 2008, 21:54
ჩემს გოგონას დაუსვეს თირკმლის ანთების დიაგნოზი.

ორი დღე ტკიოდა ყრუთ კუჭის გასწვრივ მარჯვენა გვერდი და ღამით დაბალ ტემპერატურას (37.2-37.5) აძლევდა. დღეს (III დღეა) წავიყვანე ექიმთან და დავიწყეთ ანტიბიოტიკის მიღება(7-8 საათის წინ), მაგრამ ახლა სიცხე 38.5 აქვს.

ვხვდები რომ მკურნალობა ასე მალე შედეგს არ გამოიღებდა, მაგრამ რაღაც ბოლომდე ვერ დავრწმუნდი შეიძლებოდა თუ არა ამ დიაგნოზის დასმა მხოლოდ შარდის ანალიზზე დაყრდნობით? ასე ზეპირად თქმა ალბათ ძნელია, მაგრამ კიდევ სხვა რაიმეს რას შეიძლებოდა გამოეწვია მსგავსი სიმპტომები? (ინდონეზიაში ვიმყოფები და ბოლომდე ვერ ვენდე ექიმს, ეგებ დამაწყნაროთ).


Posted by: vano_t 27 Dec 2008, 00:43
pao
QUOTE
ჩემს გოგონას დაუსვეს თირკმლის ანთების დიაგნოზი.

ორი დღე ტკიოდა ყრუთ კუჭის გასწვრივ მარჯვენა გვერდი და ღამით დაბალ ტემპერატურას (37.2-37.5) აძლევდა. დღეს (III დღეა) წავიყვანე ექიმთან და დავიწყეთ ანტიბიოტიკის მიღება(7-8 საათის წინ), მაგრამ ახლა სიცხე 38.5 აქვს.

ვხვდები რომ მკურნალობა ასე მალე შედეგს არ გამოიღებდა, მაგრამ რაღაც ბოლომდე ვერ დავრწმუნდი  შეიძლებოდა თუ არა ამ დიაგნოზის დასმა მხოლოდ შარდის ანალიზზე დაყრდნობით? ასე ზეპირად თქმა ალბათ ძნელია, მაგრამ კიდევ სხვა რაიმეს რას შეიძლებოდა გამოეწვია მსგავსი სიმპტომები? (ინდონეზიაში ვიმყოფები და ბოლომდე ვერ ვენდე ექიმს, ეგებ დამაწყნაროთ).

მარტო შარდის ანალიზით შეუძლებელია მაგის დიაგნოზი. შარდის ანალიზი მხოლოდ უჩვენებს საშარდე გზების ინფექცია, რომელიც შეიძლება იყოს ურეთრიტი (შარდსადენის ინფექცია), ცისტიტი (შარდის ბუშტის ინფექცია) ან პიელონეფრიტი (თირკმლის ინფექცია). სიცხე საშარდე გზების ინფექციების დროს მიუთითებს უფრო პიელონეფრიტზე, მაგრამ არ ნიშნავს რომ პიელონეფრიტია. პიელონეფრიტის დროს ლეიკოციტების მომატებულია. თირკმლის ულტრაბგერა, როცა პიელონეფრტიზე ფიქრობ, აუცილებლად საჭიროა, რათა გამორიცხო საშარდე გზების "დაბლოკვა".

მე პედიატრი არ ვარ, მაგრამ ბავშვებში, როგორც მახსოვს ხშირად არის ე.წ. ვეზიკოურეთერალური რეფლუქსი, რომელთაც შეუძლიათ საშარდე ინფექციების ხშირი გამოწვევა. ამას ადგენენ სპეციალური გამოკვლევით: ცისტოურეთროგაფიით.

SPC
QUOTE
თუ შეიძლება დამაკვალიანეთ რა იწვევს ხელებში ძალის გამოცლას?
რამე ვიტამინით შეიძლება ამ პრობლემის გამოსწორება?

1 წელია ხორცს არ ვჭამ და აღარც შევჭამ,. შეიძლება ეს იყოს მიზეზი?

თუმცა ეს მანამდეც მაწუხებდა, ანუ ხელებში ვგრძნობ უღონობას თუ როგორაა

ფეხებზე ყველაფერი წესრიგშია ამ დროს.



და აქვე მეორე კითხვას დავსვამ, გრძელი თმები მაქვს და ბოლოები მეყოფა, რკინის ანალიზი უნდა გავიკეთო და მერე მინდა მისვლა ვინმესთან ვინც უშუალოდ თმებზე სპეციფირდება, რა ჰქვია თმის ექიმს? და არის თუ არა ასეთი საქართველოში? თუ კი სად?
ხელებში სისუსტე ბევრმა რამემ შეიძლება მოგცეს. ინტერნეტით ამის დიაგნოზი შეუძლებელია. ექიმი უნდა ნახო და ექიმმა დაწვრილებით უნდა გამოგკითხოს და გაგსინჯოს. შეიძლება ლაბორატორიებიც დაგჭირდეს.

თმის ექიმი არ არსებობს. თმის ზოგიერთი პრობლემა შეიძლება კანის ექიმის საქმე იყოს, ან თერაპევტის საქმე.

http://www.mindbodyfocused.com/articles/body/hair-and-skin-care/hair-split-ends.php არის თმის გაყოფის მიზეზები. თუ ინგლისურად კითხულობ, მაშინ შეიძლება საკმარისი რჩევა იყოს.

Posted by: pao 27 Dec 2008, 06:00
vano_t
უღრმესი მადლობა, თქვენი პასუხით ძალიან კმაყოფილი ვარ, ყურადღებიანი ბრძანდებით.

ერთი თვის წინ სინგაპურში ბავშვს დაუდგინეს საშარდე ბუშტის ანთება, მივედით ხშირი შარდვის და დასიებული თვალების ჩივილებით, იქ სისხლიც აუღეს, შარდიც და გადაუღეს "ეხო" (შავ დიდ ფირზე ბევრი პატარ-პატარა სურათია, ეგ არის ცისტოურეთროგრაფია?) თუმცა თირკმელი გამორიცხეს. მივიღეთ ანტიბიოტიკები დანიშნულებისამებრ და დღეს ასეთი მდგომარეობა გვაქვს,შეიძლება ვივარაუდოთ რომ მკურნალობამ წარმატებულად არ ჩაიარა თუ არ არის ერთმანეთთან კავშირში?

სიცხე თითქმის მთელი ღამე ჰქონდა, დღეს დილით ისევ 38.5 აქვს, თუმცა გვერდი ნაკლებად ტკივა... რას მირჩევთ აუცილებლად გავაკეთოთ ცისტოურეთროგრაფია??? (ამისათვის გადაფრენა მომიწევს მინიმუმ მალაიზიაში, რადგან აქ თითქმის არაფერი გააჩნიათ). და კიდევ ერთი, საერთოდ აწუხებს ღამის ენურეზი (7 წლისაა) და ესეც სამკურნალოა ამ ასაკში?

* * *
დიდ ბოდიშს ვიხდი, მაგრამ შემშლია ანთება არა ინფექცია. ესე იგი საშარდე ბუშტის ინფექცია ჰქონდა და ახლა დაუსვეს თირკმლის ინფექციის დიაგნოზი. რატომღაც ყურადღება არ მივაქციე, რომ ინფექცია და ანთება სხვადასხვა რამეა.

Posted by: vano_t 27 Dec 2008, 08:41
pao
QUOTE
ერთი თვის წინ სინგაპურში ბავშვს დაუდგინეს საშარდე ბუშტის ანთება, მივედით ხშირი შარდვის და დასიებული თვალების ჩივილებით,  იქ სისხლიც აუღეს, შარდიც და გადაუღეს "ეხო" (შავ დიდ ფირზე ბევრი პატარ-პატარა სურათია, ეგ არის ცისტოურეთროგრაფია?) თუმცა თირკმელი გამორიცხეს. მივიღეთ ანტიბიოტიკები დანიშნულებისამებრ და დღეს ასეთი მდგომარეობა გვაქვს,შეიძლება ვივარაუდოთ რომ მკურნალობამ წარმატებულად არ ჩაიარა თუ არ არის ერთმანეთთან კავშირში?

სიცხე თითქმის მთელი ღამე ჰქონდა, დღეს დილით ისევ 38.5 აქვს, თუმცა გვერდი ნაკლებად ტკივა... რას მირჩევთ აუცილებლად გავაკეთოთ ცისტოურეთროგრაფია??? (ამისათვის გადაფრენა მომიწევს მინიმუმ მალაიზიაში, რადგან აქ თითქმის არაფერი გააჩნიათ). და კიდევ ერთი, საერთოდ აწუხებს ღამის ენურეზი (7 წლისაა) და ესეც სამკურნალოა ამ ასაკში?

ცისტოურეთროგრაფიას ერთხელ აკეთებენ მგონი. ეგ ინფექციის დიაგნოზს არ სვამს. ეგ უბრალოდ გიჩვენებს ხშირი საშარდე ინფექციებს მიზეზს. ამიტომ, ჩემი აზრით არ უნდა დაჭირდეს. თუმცა პედიატრთან უნდა გადაამოწმო.

რაც შეეხება სიცხეს, თუ თირკმლის ინფექცია გამოირიცხა, მაშინ სიცხე შეიძლება იყოს უბრალოდ ვირუსული (რაც სიცხის ყველაზე ხშირი მიზეზია). მაგალითად, თუ შენს ბავშვს აწუხებს ყურისტკივილი, ცემინება, ცხვირიდან გამონადენი, ყელის ტკივილი, ხველა (არ არის აუცილებელი ყველაფერი ერთად ქონდას, შეიძლება ერთი ან ორი სიმტპომი ქონდეს აქედან), მაშინ ზემო სასუნთქის გზების ვირუსული ინფექცია შეიძლება იყოს. მთავარია თირკმლების ინფექცია გამოირიცხოს.

ენურეზი, როგორც მახსოვს, გარკვეულ ასაკამდე შეიძლება გაგრძელდეს. თუ ორგანული მიზეზები გამოირიცხა, მაშინ ყველაზე ხშირად სტრესს და ფსიქოლოგიურ ტრამვას აბრალებენ.

მე ზოგადი სახის ინფორმაცია მოგაწოდე და არავითარ შემთხვევაში არ არის კომპეტენტური პასუხი. ყველა ვარიანტში უნდა გადაამოწმო პედიატრთან.

Posted by: pao 27 Dec 2008, 13:03
vano_t
მეც კომპეტენტურის აზრი მაინტერესებდა, თქვენს რჩევებს აუცილებლად გავითვალისწინებ, უღრმესი მადლობა ოპერატიულობისთვის, ძალიან დამეხმარეთ.
* * *
და თქვენ საკმაოზე მეტად კომპეტენტური ბრძანდებით, თქვენნაირმა ექიმებმა იმრავლონ აქ და ყველგან....

Posted by: ხანთოზავრი 27 Dec 2008, 20:32
........................................

Posted by: pao 27 Dec 2008, 20:41
vano_t
QUOTE
ენურეზი, როგორც მახსოვს, გარკვეულ ასაკამდე შეიძლება გაგრძელდეს. თუ ორგანული მიზეზები გამოირიცხა, მაშინ ყველაზე ხშირად სტრესს და ფსიქოლოგიურ ტრამვას აბრალებენ.

დავუშვათ რომ გამოირიცხოს სტრესი და ფსიქოლოგიური ტრამვა, ხომ ვერ მეტყოდით რა ორგანული მიზეზები შეიძლება არსებობდეს ამ შემთხვევაში?

Posted by: vano_t 28 Dec 2008, 01:40
pao
QUOTE
დავუშვათ რომ გამოირიცხოს სტრესი და ფსიქოლოგიური ტრამვა, ხომ ვერ მეტყოდით რა ორგანული მიზეზები შეიძლება არსებობდეს ამ შემთხვევაში?

ორგანული მიზეზებიდან შეიძლება იყოს მაგალითად საშარდე გზების ინფექცია, მაგრამ ეს მოგცემს დროებით ენურეზს. ინფექციის მკურნალობის შემდეგ გადის. უშაქრო დიაბეტმაც (და შესაძლებელია შაქრიანმა დიაბეტმაც ალბათ) შეიძლება მოგცეს ეგ, მაგრამ ამის დიაგნოზი ადვილია. ამ დროს ბავშვები ძაან დიდი რაოდენობით წყალს სვამენ. შაქრიანი დიაბეტის დადგენაც ადვილია. კიდევ შეიძლება იყოს შარდსასქესო სისტემის თანდაყოლილი ან შეძენილი ანატომიური ცვლილებები (მაგალითად შარდსადენის შევიწროება რაიმე მიზეზის გამო). უროლოგმა წესით ასეთი რამ ადვილად უნდა დაიჭიროს. თუ ბავშვს მალაიზიაში გაუკეთდა ურეთეროცისტოგრაფია, მაშინ რარაცას დაინახავდა უროლოგი.

ენურეზის 95 პროცენტზე მეტ შემთხვევაში არავითარი ორგანული მიზეზი არ არსებობს. გოგოებში ღამის ენურეზი საშუალოდ ნორმად ითვლება 5 წლამდე და ბიჭებში 6 წლამდე. როცა ორგანული მიზეზები გამორიცხულია, მაშინ შეიძლება (ზოგიერთი პედიატრის და უროლოგის აზრით) ენურეზი გამოწვეული იყოს უბრალოდ გვიანი მომწიფებით (სასქესო ნიშნების მომწიფებაზე არ არის საუბარი).

მოკლედ, ორგანული მიზეზების გამორიცხვის შემთხვევაში, ალბათ გაუვლის მალე. წესით ბავშვი სხვანაირად თუ ჯანმრთელია, ალბათ გაუვლის ეგ მალე.

Posted by: pao 28 Dec 2008, 07:04
vano_t
ყველაფერი გასაგებია, დიდი მადლობა სრული პასუხისთვის.

Posted by: kuchu 29 Dec 2008, 19:50
ჩემი მეუღლე ორსულადაა, მეოთხე თვეშია და როე აქვს მაღალი >>55 ჰემოგლობინი >>57 სხვა მხრივ სისხლის სურათში არანაირი პათოლოგია არ აღინიშნება და ყველაფერი ნორმაშია.. შარდი სუფთა, მუცლის ღრუს ორგანოები ყველაფერი ნორმაშია.. ორსულობამდე ბავშვობიდან პერიოდულად ჰქონდა წითელი წერტილოვანი და ზოგჯერ ვარსკვლავისებრი კანქვეშა კაპილარები.. ორსულობის დასაწყისიდან დაეწყო ამ კაპილარების ზრდა ზომაში და ახალი კაპილარების წარმოქმნა.. ამ დღეებში მუცელზეც გაუჩნდა.. მყავდა ინფექციონისტთან და ჰემატლოგთან და ანგიოლოგთან მათ მხრივ პათოლოგია არ აღინიშნა.. რისი ბრალი შეიძლება იყოს ან ვის მივმართოთ იქნებ გვირჩიოთ

Posted by: vano_t 29 Dec 2008, 22:45
kuchu
QUOTE
ჩემი მეუღლე ორსულადაა, მეოთხე თვეშია და როე აქვს მაღალი >>55 ჰემოგლობინი >>57 სხვა მხრივ სისხლის სურათში არანაირი პათოლოგია არ აღინიშნება და ყველაფერი ნორმაშია.. შარდი სუფთა, მუცლის ღრუს ორგანოები ყველაფერი ნორმაშია.. ორსულობამდე ბავშვობიდან პერიოდულად ჰქონდა წითელი წერტილოვანი და ზოგჯერ ვარსკვლავისებრი კანქვეშა კაპილარები.. ორსულობის დასაწყისიდან დაეწყო ამ კაპილარების ზრდა ზომაში და ახალი კაპილარების წარმოქმნა.. ამ დღეებში მუცელზეც გაუჩნდა.. მყავდა ინფექციონისტთან და ჰემატლოგთან და ანგიოლოგთან მათ მხრივ პათოლოგია არ აღინიშნა.. რისი ბრალი შეიძლება იყოს ან ვის მივმართოთ იქნებ გვირჩიოთ

ორსულებში ეგ როე არ არის მაღალი. ეგ ორე ითვლება ნორმალურად. განსაკუთრებით ორსულობის მეორე ნახევარში. ორსულობის პირველ ნახევარში როე 50-მდე შეიძლება ავიდეს და მეორე ნახევარში 70-მდე. ეს საშუალო რიცხვებია.
http://www.bio-medicine.org/medicine-news/The-Relation-Between-Pregnancy-And-Erythrocyte-Sedimentation-Rate--5532-1/

ეგ ჰემოგლობინი სხვა ერთეულებშია და ვერ გეტვყი რამხელაა. ისე ორსულობის დროს მცირე ანემიები არ არსი იშვიათი.უფრო მეტიც, ასეთი ანემიებია ცრუ ანემიებია და არა ჭეშმარიტი ანემია. ორსულობის დროს ხდება მოცირკულირე სითხის მომატება დაახლოებით 50 % მდე და ეს იწვევს სისხლის განზავებას, რასაც მოსდევს ორსულობის ანემია. ამ ანემიას მკურნალობა არ ჭირდება. შეიძლება ორსულს დაემართოს რკინადეფიციტური ან B12 ვიტამინის ნაკლებობით გამოწვეული ანემია. ამას სპეციალური ლაბორატორიები ჭირდება. თუმცა თუ შენი მეუღლე კარგად გრძნობს თავს და სხვანაირად ყველაფერი ნორმაშია (და ნაყოფიც ნორმალურია), მაშინ უფრო სავარაუდო რომ ეგ ყველაფერი ფიზიოლოგიურია. საერთოდ, როცა ნაყოფი ვითარდება, მას ჭირდება დიდი რაოდენობით რკინა და B12 ვიტამინი. თუ საკმარისი რაოდენობით არ იღებს ორსული ამ ნაერთებს (B12-ისნ ნაკლებობა უფრო ხშირია ვეგეტარიანული დიეტის დროს), მაშინ შეიძლება ჩამოყალიბდეს ესეთი ანემიები. ამათ მარტო რკინის და B12 ვიტამინის მიღება ჭირდებათ.

რაც შეეხება გამონაყარს, ამას შეხედვა უნდა. მაგრამ, ორსულობის დროს ჩნდება გამონაყარები რომელიც ნორმალურია. შენ რასაც აღწერ შეიძლება იყოს ტელეანგიექტაზია. ეს გაგანიერებულ სისხლძარღვებია, რომელიც სისხლძარღვებში მომატებული წნევის გამო ჩნდება (რაც ნორმალური შეიძლება იყოს ორსულობის დროს). მოკლედ, ისე ჩანს, რომ ორსულობა ნორმალურად მიმდინარეობს, მარა შენს მეუღლეს მაინც ჭირდება გინეკოლოგის დაკვირვება და საბოლოოდ გინეკოლოგმა უნდა განსაზღვოს რა არის ფიზიოლოგიური და რა არის პათოლოგიური.

ისე რა ადგილას არის ეგ გამონაყარი?

ქვემო სურათზე მოცემულია ტიპიური გამონაყარი:
user posted image

Posted by: kuchu 30 Dec 2008, 01:12
vano_t
მადლობა გამოხმაურებისთვის smile.gif

რაც შეეხება გამონაყარს ესეთი აქვს მტევაზე ორი

ასევე წითელი წერტილოვანი აქვს მუცელზე და მკლავებზეც წერტილოვანები

Posted by: faustina 30 Dec 2008, 16:03
რა გარტულებები შეიძლება მოყვეს სწორი ნაწლავის ნახევარზე ამოკვეთIს შემდეგ ???

Posted by: cxovrebisazri 31 Dec 2008, 00:14
გამარჯობათ


რა მაინტერესებს იცით?შეიძლება თუ არა რომ რაიმე პრეპარატის გადამეტებულად მირებამ გამოიწვიოს ძლიერი ტოქსიკაცია და პაციენტის სიკვდილი?რა პრეპარატებია ასეთი და სიცოცხლის გადარცჰენის ხანგრძლივობა რამდენი შეიძლება იყოს მოწამვლის შემდგომ?ანუ პაციენტის გადასარჩენად რა დრო შეიძლება გვქონდეს?

Posted by: Solveig 1 Jan 2009, 17:51
მე მაქვს ასეთი კითხვა:

დაახლოებით თვენახევრის წინ გამიკეთეს კანის ბიოფსია ხელის მტევნიდან (მაწუხებს გამონაყარი და დიაგნოზი ვერ დასვეს sad.gif). რა თქმა უნდა, პროცედურა ძალიან მტკივნეული იყო და მომდევნო 1 კვირის განმავლობაში მქონდა შეშუპება და გარშემო-სილურჯე. შემდეგ შეშუპება დაცხრა, სილურჯემაც გაიარა ნელ- ნელა, მაგრამ დარჩა პატარა გამკვრივებული ადგილი ნაიარევიდან ცოტა მოშორებით (მგონი, მანდ გამაყუჩებელი გამიკეთა ექიმმა), რომელიც თვალითაც ჩანს და ხელითაც ისინჯება. შეხებისას ცოტა მტკივნეულია (ადრე უფრო მტკიოდა, ახლა-შემიმცირდა). რა შეიძლება იყოს?

Posted by: Guardian 1 Jan 2009, 19:05
Solveig
QUOTE
დაახლოებით თვენახევრის წინ გამიკეთეს კანის ბიოფსია ხელის მტევნიდან (მაწუხებს გამონაყარი და დიაგნოზი ვერ დასვეს sad.gif). რა თქმა უნდა, პროცედურა ძალიან მტკივნეული იყო და მომდევნო 1 კვირის განმავლობაში მქონდა შეშუპება და გარშემო-სილურჯე. შემდეგ შეშუპება დაცხრა, სილურჯემაც გაიარა ნელ- ნელა, მაგრამ დარჩა პატარა გამკვრივებული ადგილი ნაიარევიდან ცოტა მოშორებით (მგონი, მანდ გამაყუჩებელი გამიკეთა ექიმმა), რომელიც თვალითაც ჩანს და ხელითაც ისინჯება. შეხებისას ცოტა მტკივნეულია (ადრე უფრო მტკიოდა, ახლა-შემიმცირდა). რა შეიძლება იყოს?

შეიძლება ინფექცია შეიჭრა.
სურათი დადე.
გამონაყარისაც.
ბიოფსიით მიღებული მიკროსიც, თუ შესაძლებელია.

Posted by: Solveig 1 Jan 2009, 23:11
Guardian
QUOTE
შეიძლება ინფექცია შეიჭრა.



შეიძლება ინფექცია იყოს სიწითლის და შეშუპების გარეშე? შეშუპება აღარაა, მარტო ის ადგილია გამკვრივებული (როგორც ვთქვი, ხელით ისინჯება-მარცვალივითაა). ადრე უფრო იყო ამობურცული.

QUOTE
ბიოფსიით მიღებული მიკროსიც, თუ შესაძლებელია.

სამწუხაროდ, არ მაქვს ხელთ. შეღებვის საფუძველზე სოკოვანი ინფექცია, ასევე ტუბერკულოზი გამორიცხეს. გამოკვლევის მიხედვით, დერმა ინტენსიურად იყო ინფილტრირებული ლიმფოციტებით, მაკროფაგებით, ნეიტრო-და ეოზინოფილებით. არ იყო ვასკულიტისათვის დამახასიათებელი ცვლილებები და არ აღინიშნებოდა ბაზალური მემბრანის გასქელება. დასკვნის მიხედვით უნდა ყოფილიყო რომელიმე მათგანი-რეაქცია ფეხსახსრიანებზე, წითელი ქარის საწყისი სტადია ან რომელიმე ინფექცია (არ დაუკონკრეტებიათ).

მანამდე ჩატარდა სისხლის საერთო ანალიზი (ყველა მაშვენებელი ნორმაში იყო, მაგრამ ლეიკოციტური ფორმულა არ გაუკეთებიათ) და გამოკვლევა ანტისხეულებზე და ჰელიკობაქტერიაზე. ეს უკანასკნელი არ აღმოჩნდა, შრატში კი ნორმის ზედა ზღვარზე იყო სტრეპტოლიზინის საწინააღმდეგო ანტისხეულები. პარალელურად მაქვს აკნეც და ამისათვის ექიმმა დამინიშნა ცეფუროქსიმი, თუმცა უშედეგოდ. ასევე, არანაირი შედეგი არ გამოიღო ანტიჰისტამინურმა პრეპარატმა და ტკიპების საწინააღმდეგო მალამომ.

ახლა გამიშვა medizinische Hochschule-ს კლინიკაში-იქაც ნახონო. ბიოპტატის გამოკვლევაც იქ ჩატარდა და რა ვიცი, იქნებ იქ უფრო გაიგონ რამე. ცუდი ისაა, რომ პერიოდულად მწვავდება (მიზეზიც რომ არ არსებობს, ისე) და ძნელია, ექიმთან ვიზიტის დამთხვევა..ეს "სიკეთე" საკმაოდ ძნელად ხდება აქ sad.gif

გამონაყარი ძირითადად არის ზედა კიდურებზე და მხრებზე. ჩნდება და ქრება..მერე ისევ ჩნდება და ასე გრძელდება უკვე რამდნეიმე თვეა. თან ახლავს ძლიერი ქავილის შეგრძნებაც.


თავიდან ალერგია მეგონა რაღაც ქიმიკატზე (საქართველოში არ ვარ და აქ ხილი და ბოსტნეული არ არის სანდო მაგ მხრივ)-უცებ გამომაყარა თითქმის მთელ ტანზე, თან ეს ხდებოდა ნაშუადღევს, ამიტომაც დავუკავშირე საკვებს და აღარ შემიჭამია ის, რასაც ვაბრალებდი...უხვი გამონაყარი გაქრა, მაგრამ მაინც დარჩა ცალკეული ელემენტების სახით.
გამონაყარი ასე გამოიყურება:

Posted by: Guardian 1 Jan 2009, 23:37
Solveig
QUOTE
შეიძლება ინფექცია იყოს სიწითლის და შეშუპების გარეშე? შეშუპება აღარაა, მარტო ის ადგილია გამკვრივებული (როგორც ვთქვი, ხელით ისინჯება-მარცვალივითაა). ადრე უფრო იყო ამობურცული.

მანდ, როგორც ჩანს, რაღაც ანთებითი რეაქცია იყო და შედეგად ეგ დატოვა.
QUOTE
დერმა ინტენსიურად იყო ინფილტრირებული ლიმფოციტებით, მაკროფაგებით

ეს ქრონიკულ ანთებაზე მიუთითებს.
QUOTE
და ეოზინოფილებით

ეს - ალერგიაზე.
QUOTE
წითელი ქარის საწყისი სტადია

არ გავს მაგას - წითელი ქარი ძალიან მტკივნეულია და მწვავედ და სწრაფად ვითარდება.
QUOTE
პარალელურად მაქვს აკნეც

რეტინოიდები არ გიცდია?
QUOTE
ექიმმა დამინიშნა ცეფუროქსიმი, თუმცა უშედეგოდ. ასევე, არანაირი შედეგი არ გამოიღო ანტიჰისტამინურმა პრეპარატმა და ტკიპების საწინააღმდეგო მალამომ.

შეიძლება ამ წამლებიდან რომელიმეზე გაქვს ალერგიული რეაქცია - კიდევ რამეს ხომ არ ღებულობ?
QUOTE
გამონაყარი ძირითადად არის ზედა კიდურებზე და მხრებზე. ჩნდება და ქრება..მერე ისევ ჩნდება და ასე გრძელდება უკვე რამდნეიმე თვეა. თან ახლავს ძლიერი ქავილის შეგრძნებაც.

მანამდე ეგ არასდროს გქონია?
საერთოდ, ალერგიული ხარ?
კიდევ რა ახასიათებს?
კანი არ იქერცლება?
მოქავების შემდეგ სისხლი ხომ არ იწყებს ჟონვას წვრილი წერტილების სახით?
QUOTE
გამონაყარი ასე გამოიყურება:

ერითემული სახის გამონაყარია დიფუზური რეტიკულური სურათით.
ერთი შეხედვით, ალერგიულ რეაქციას ჰგავს.

Posted by: Solveig 2 Jan 2009, 00:12
Guardian
QUOTE
მანდ, როგორც ჩანს, რაღაც ანთებითი რეაქცია იყო და შედეგად ეგ დატოვა.

თავიდან ყურადღებას არ ვაქცევდი, უბრალოდ ახლა ამდენი დრო რომ გავიდა..

QUOTE
ეს ქრონიკულ ანთებაზე მიუთითებს.

არ შეგედავები, იმიტომ, რომ ანთებასა და ალერგიულ რეაქციაზე მეც ვიფიქრე..მაგრამ რა მაინტერესებს: ნეიტროფილები რჩება ქრონიკულის დროს? მე ისე მახსოვდა, რომ თავიდან მწვავე ანთების დროს ნეიტროფილები "იწყებდნენ" ინფილტრაციას და უკვე ქრონიკულში ხდებოდა მათი ჩანაცვლება ლიმფოციტებით და მაკროფაგებით...

QUOTE
რეტინოიდები არ გიცდია?

არა..არ დაუნიშნავს ექიმს.

QUOTE
შეიძლება ამ წამლებიდან რომელიმეზე გაქვს ალერგიული რეაქცია - კიდევ რამეს ხომ არ ღებულობ?

როცა ცეფუროქსიმს ვსვამდი, მაშინ დამინიშნა აკნესათვის მალამო "ეპიდუო" (მანდ თუ არის, არ ვიცი), რომელმაც საშინლად გამიღიზიანა კანი-გაწითლება, ქავილი, აქერცვლა და ა. შ,..მერე თავი დავანებე, მაგრამ კარგა ხანი დასჭირდა, რომ გაევლო..იმის მერე სკინორენზე გადამიყვანა და ახლა მას ვიყენებ, მაგრამ ვატყობ-არაფერი შედეგი არ აქვს. დალევით არაფერს არ ვსვამ ახლა. ბოლოს ექიმთან დეკემბრის შუა რიცხვებში ვიყავი და მითხრა-მე აღარაფერი ვიცი მეტი, სხვამაც გნახოსო, თუ გინდა, სტეროიდები ვცადოთო, მაგრამ მე უარი ვუთხარი-ჯერ იმ "სხვასთან" მივალ მეთქი...იქ 5 იანვარს მივდივარ, უფრო ადრე ვერ მიგიღებთო, მითხრეს. აქ ძალიან ძნელია ექიმთან მისვლა.

QUOTE
მანამდე ეგ არასდროს გქონია?

ასეთი სახის გამონაყარი მქონდა ერთხელ, როცა ვჭამე გაუთლელი ნაყიდი ვაშლი..მაგრამ 2 დღე მქონდა და მარტო რამდენიმე ელემენტი მუხლზე. უწამლოდ გაიარა. ერთ-ერთი მიზეზი ესეც იყო, რომ თავიდან ქიმიკატზე ალერგია მეგონა. საერთოდ, ალერგიული ვარ. ბავშვობაში აკრძალული მქონდა შავი ყურძენი, შოკოლადი, კვერცხის ცილა, ნიგოზი, თხილი..და მამაჩემი ვაშლებს არ წამლავდა ხოლმე..

QUOTE
კანი არ იქერცლება?

არა.

QUOTE
მოქავების შემდეგ სისხლი ხომ არ იწყებს ჟონვას წვრილი წერტილების სახით?

არა.
თუმცა კანი კი გადავიცალე რამდენიმე ადგილას და ახლა ნაიარევი დამრჩა.

დამატებით იმისი თქმაც შემიძლია, რომ იქ სადაც გამონაყარი იყო, ზოგჯერ წერტილივით სისხლჩაქცევა რჩება, მერე ქრება.

Posted by: Guardian 2 Jan 2009, 00:26
Solveig
QUOTE
ნეიტროფილები რჩება ქრონიკულის დროს? მე ისე მახსოვდა, რომ თავიდან მწვავე ანთების დროს ნეიტროფილები "იწყებდნენ" ინფილტრაციას და უკვე ქრონიკულში ხდებოდა მათი ჩანაცვლება ლიმფოციტებით და მაკროფაგებით...

გააჩნია, რა ტიპის ანთებაა.
ალერგიული ეტიოლოგიის ანთების დროს ნეიტროფილები, შესაძლოა, დარჩნენ.
QUOTE
არა..არ დაუნიშნავს ექიმს.

თუ სხვა არაფერი გშველის, ბოლოს მაგას ნიშნავენ ხოლმე აკნეს წინააღმდეგ - ძალიან ეფექტურია, მაგრამ სიფრთხილით გამოყენებაა საჭირო.
QUOTE
მერე სკინორენზე გადამიყვანა და ახლა მას ვიყენებ

სკინორენს ფოტოსენსიტიურობა ახასიათებს - სადაც გამოყრილი გაქვს, მაგ ადგილებზე მზე გხვდება?

Posted by: Solveig 2 Jan 2009, 00:38
Guardian
QUOTE
სკინორენს ფოტოსენსიტიურობა ახასიათებს - სადაც გამოყრილი გაქვს, მაგ ადგილებზე მზე გხვდება?

ანუ, სინათლის გავლენით იშლება? თუ პირიქით?

ზოგან მხვდება და ზოგან-არა.

Posted by: Guardian 2 Jan 2009, 00:42
Solveig
QUOTE
ანუ, სინათლის გავლენით იშლება? თუ პირიქით?

ჰო, არასწორად ვთქვი - სკინორენი ფოტოსენსიტიურობას იწვევს.
ანუ, მაგას თუ ხმარობ, შესაძლოა, ამან ანთებითი რეაქცია და გამონაყარი გამოიწვიოს, სადაც მზე მოგხვდება, იმ ადგილას.

Posted by: Solveig 2 Jan 2009, 00:52
Guardian
ეგ არ ვიცოდი..არც ექიმს უთქვამს და წამლის ანოტაციაც მხოლოდ გერმანულადაა, ალბათ ვერ მივაქციე ყურადღება.

გმადლობ, გავითვალისწინებ.

Posted by: pao 5 Jan 2009, 18:21
vano_t
7 წლის ბავშვს აქვს პირში სუნი ძალიან ხშირად, არა მარტო დილით... გლანდები ამოჭრილი აქვს, კბილების და ღრძილების პრობლემა არა აქვს, პირის ღრუს ჰიგიენას იცავს (ვიზიტი სტომატოლოგთან 6 თვეში ერთხელ და ხეხვა დილას საღამოს) კუჭ-ნაწლავი არ აწუხებს... ვეღარ გავიგე რა უნდა იყოს მიზეზი, რა უნდა გავუკონტროლო სხვა რომ ეს სუნი თავიდან მოვიშოროთ. შიძლება არც უნდა მივაქციო ყურადღება, არ ვიცი, ეგებ რამე მირჩოთ.

Posted by: EKIKULI 6 Jan 2009, 00:47
QUOTE
ნაღვლის ბუშტი საშუალო ზომის 6,0X2,9 სმ, კედლები უმნიშვნელოდ შესქელებულია, დეფორმაციის გარეშე, ყელისა და ტანის საზრვარზე მოსჩანს,
1,3X0,5 სმ ზომის კედელთან ფიქსირებული ჰიპერექოგენური წარმონაქმნი, სავარაუდოდ ქოლესტერინული პოლიპი


არის იმის შანსი რომ მკურნალობით გაიწოვოს?

Posted by: Tsico_Tsico 6 Jan 2009, 11:47
pao
ექიმი არ ვარ, მაგრამ შეიძლება ჭიები ჰყავდეს...

Posted by: lizofobi 6 Jan 2009, 16:52
თერმომეტრი რამდენი წუთით უნდა გაიჩერო იღლიაში? გამარკვიეთ რა user.gif

ხო ეს ერთი საკითხი და მეორე:

დღის განმავლობაში ტემპერატურა მითამაშებს - ანუ დილით შეიძლება 36.3 მქონდეს, შუადღეს 36.5, 1 საათის მერე რო გავიზომო ისევ 36.3...საღამოს საათებში, სადღაც 5-6 შუალედში, 36.8 მდე ადის, და როცა რაღაცაზე ვნერვიულობ შეიძლება 37.3 ზეც ავიდეს... რისი ბრალი შეიძლება იყოს? ვეგეტონევროზიაო მითხრეს, სხვა არაფერი არ მაწუხებს, ერთი ისაა, რომ სიბრძნის კბილები ამომდის და კიდევ, ლაბილური ფსიქიკის პატრონი ვარ თურმე gigi.gif

Posted by: texasuri jleta benzoxerxit 6 Jan 2009, 17:36
QUOTE (lizofobi @ 6 Jan 2009, 16:52 )
თერმომეტრი რამდენი წუთით უნდა გაიჩერო იღლიაში? გამარკვიეთ რა user.gif

ხო ეს ერთი საკითხი და მეორე:

დღის განმავლობაში ტემპერატურა მითამაშებს - ანუ დილით შეიძლება 36.3 მქონდეს, შუადღეს 36.5, 1 საათის მერე რო გავიზომო ისევ 36.3...საღამოს საათებში, სადღაც 5-6 შუალედში, 36.8 მდე ადის, და როცა რაღაცაზე ვნერვიულობ შეიძლება 37.3 ზეც ავიდეს... რისი ბრალი შეიძლება იყოს? ვეგეტონევროზიაო მითხრეს, სხვა არაფერი არ მაწუხებს, ერთი ისაა, რომ სიბრძნის კბილები ამომდის და კიდევ, ლაბილური ფსიქიკის პატრონი ვარ თურმე gigi.gif

სწორი უთქვამთ boli.gif givi.gif givi.gif givi.gif

Posted by: lizofobi 6 Jan 2009, 17:41
QUOTE (texasuri jleta benzoxerxit @ 6 Jan 2009, 17:36 )
სწორი უთქვამთ boli.gif givi.gif givi.gif givi.gif

ვერ მოვიშორე ეს გიჟის იარლიყი რაააა gigi.gif gigi.gif

კარგიი, მაშინ მესამე კითხვა, რა ვქნა, დავიკიდო, თერმომეტრი დავლეწო და დავიგინო რო აღარ გავიზომავ თუ სპორტს მივყო ხელი? gigi.gif

Posted by: texasuri jleta benzoxerxit 6 Jan 2009, 18:09
QUOTE (lizofobi @ 6 Jan 2009, 17:41 )
QUOTE (texasuri jleta benzoxerxit @ 6 Jan 2009, 17:36 )
სწორი უთქვამთ boli.gif  givi.gif  givi.gif  givi.gif

ვერ მოვიშორე ეს გიჟის იარლიყი რაააა gigi.gif gigi.gif

კარგიი, მაშინ მესამე კითხვა, რა ვქნა, დავიკიდო, თერმომეტრი დავლეწო და დავიგინო რო აღარ გავიზომავ თუ სპორტს მივყო ხელი? gigi.gif

ფსიქოთერაპიის (+/- ფსიქოფარმაკოთერაპიის) კურსი ჩაიტარე boli.gif

Posted by: vano_t 7 Jan 2009, 08:18
QUOTE (pao @ 5 Jan 2009, 18:21 )
vano_t
7 წლის ბავშვს აქვს პირში სუნი ძალიან ხშირად, არა მარტო დილით... გლანდები ამოჭრილი აქვს, კბილების და ღრძილების პრობლემა არა აქვს, პირის ღრუს ჰიგიენას იცავს (ვიზიტი სტომატოლოგთან 6 თვეში ერთხელ და ხეხვა დილას საღამოს) კუჭ-ნაწლავი არ აწუხებს... ვეღარ გავიგე რა უნდა იყოს მიზეზი, რა უნდა გავუკონტროლო სხვა რომ ეს სუნი თავიდან მოვიშოროთ. შიძლება არც უნდა მივაქციო ყურადღება, არ ვიცი, ეგებ რამე მირჩოთ.

პირის ღრუში სუნის მიზეზი ბევრი რამ შეიძლება იყოს, მათ შორის თვითონ პირის ღრუს პრობლემები და კუჭნაწლავის პრობლემები (რეფლუქსი ძირითადად). კიდევ ცხვირის ღრუდან გამონადენი, რომელიც ყელში ჩაედინება, იძლევა მაგას.

თუ ბავშვი სხვანაირად ჯანმრთელია (და თანაც სტომატოლოგი წელიწადში ორჯერ ამოწმებს), ალბათ არაფერია. დილაობით სუნი პირის ღრუში ბევრს აქვს ყოველგვარი დაავადების გარეშე.

Posted by: rock in rose 8 Jan 2009, 21:10
თვალის უპეები მუდმივად ჩაშავებული მაქვს ხოლმე, დასიებული არა ოღონდ. თირკმელები სუფთა მაქვს, რისი ბრალია და როგორ ვიმკურნალო?

Posted by: vano_t 9 Jan 2009, 21:22
QUOTE (rock in rose @ 8 Jan 2009, 21:10 )
თვალის უპეები მუდმივად ჩაშავებული მაქვს ხოლმე, დასიებული არა ოღონდ. თირკმელები სუფთა მაქვს, რისი ბრალია და როგორ ვიმკურნალო?

რამდენი წლის ხარ? სქესი? რამდენი ხანია გაქვს ეგ პრობლემა? სხვა რამ თუ გაწუხებს საერთოდ? თუ კი, მაშინ რა? და სურათს თუ დადებ ძალიან კარგი იქნება. (ისე თუ არაფერი არ გაწუხებს და მარტო თვალის გარშემო კანის ფერია შეცვლილი, სავარაუდოდ არაფერი იქნება)

Posted by: pao 10 Jan 2009, 23:02
QUOTE
პირის ღრუში სუნის მიზეზი ბევრი რამ შეიძლება იყოს, მათ შორის თვითონ პირის ღრუს პრობლემები და კუჭნაწლავის პრობლემები (რეფლუქსი ძირითადად). კიდევ ცხვირის ღრუდან გამონადენი, რომელიც ყელში ჩაედინება, იძლევა მაგას.

თუ ბავშვი სხვანაირად ჯანმრთელია (და თანაც სტომატოლოგი წელიწადში ორჯერ ამოწმებს), ალბათ არაფერია. დილაობით სუნი პირის ღრუში ბევრს აქვს ყოველგვარი დაავადების გარეშე.

თქვენი პასუხები ძალიან მეხმარება, გმადლობ.2kiss.gif

Posted by: giorgi33 10 Jan 2009, 23:11
მაქვს ც ჰეპატიტი ანუ შესაბამისად ღვიძლზე პრობლემები, გავცივდი და რა წამლის მიღება შეიძლება .
კიდევ ისეთი მსუბუქი დამამშვიდებლები როგორიცაა სომნა რიცი და გლიცინი ხომ არ ვნებს ღვიძლს?

Posted by: nikala papa 10 Jan 2009, 23:20
მაღალი სიმაღლიდან ჩამოვვარდი და ფეხი მაგრა დავარტყი უკვე ორი კვირაა მტკივა ეტყობა დამებეჟა და რამდენ ხანში გამივლის?

Posted by: texasuri jleta benzoxerxit 11 Jan 2009, 03:10
QUOTE (giorgi33 @ 10 Jan 2009, 23:11 )
მაქვს ც ჰეპატიტი ანუ შესაბამისად ღვიძლზე პრობლემები, გავცივდი და რა წამლის მიღება შეიძლება .
კიდევ ისეთი მსუბუქი დამამშვიდებლები როგორიცაა სომნა რიცი და გლიცინი ხომ არ ვნებს ღვიძლს?

კლატეგორიულად არ შეიძლება პარაცეტამოლის შემცველი მედიკამენტები!
თუ სისხლის შედედებაშიაც არის ცვლილებები,მაშინ ასიპირინს და მისი ჯგუფის მედიკამენტებსაც უნდა ერიდო.

რაც შეეხება ფსიქოტროპულებს...
იდეალური იქნება ჰეპტრალი,რომელიც ორმაგი მოქმედებითაა აღჭურვილი.ის ერთდორულად არის ჰეპატოპროტექტორიც და ანტიდეპრესანტიც.
ცოტა ძვირია ოღონდ... user.gif

Posted by: nikala papa 11 Jan 2009, 03:32

აუ მითხარით რამდენ ხანში გამივლის აღარ შემიძლია უკვე cry.gif

Posted by: texasuri jleta benzoxerxit 11 Jan 2009, 03:42
QUOTE (nikala papa @ 11 Jan 2009, 03:32 )
აუ მითხარით რამდენ ხანში გამივლის აღარ შემიძლია უკვე cry.gif

კიდევ დიდ ხანს არ გაგივლის...
თუ რამე სერიოზულია და რენტგენოლოგიურად და ტრავმატოლოგიურად არ ხარ გამოკვლეული კიდევ უფრო დიდი ხანი არ გაგივლის boli.gif givi.gif

Posted by: rock in rose 11 Jan 2009, 21:51
vano_t
QUOTE
QUOTE (rock in rose @ 8 Jan 2009, 21:10 )
თვალის უპეები მუდმივად ჩაშავებული მაქვს ხოლმე, დასიებული არა ოღონდ. თირკმელები სუფთა მაქვს, რისი ბრალია და როგორ ვიმკურნალო?


რამდენი წლის ხარ? სქესი? რამდენი ხანია გაქვს ეგ პრობლემა? სხვა რამ თუ გაწუხებს საერთოდ? თუ კი, მაშინ რა? და სურათს თუ დადებ ძალიან კარგი იქნება. (ისე თუ არაფერი არ გაწუხებს და მარტო თვალის გარშემო კანის ფერია შეცვლილი, სავარაუდოდ არაფერი იქნება)

18 წლის გოგო ვარ, უბრალოდ ეგ პრობლემა მაქვს მეტი არაფერი. დაახლოებით 3 ან 4 თვე იქნება, რაც ეგ მაწუხებს.
რამე სერიოზულია? პირდაპირ მითხარით თუ რამეა ისეთი. არაფერი დამიმალოთ sad.gif

Posted by: tete- 11 Jan 2009, 22:41
კალცის ნაკლებობა იწვევს თუ არა მეხსიერების ძლიერ დაქვეითებას smile.gif ?

Posted by: basa-ttt 11 Jan 2009, 22:47
QUOTE
18 წლის გოგო ვარ, უბრალოდ ეგ პრობლემა მაქვს მეტი არაფერი. დაახლოებით 3 ან 4 თვე იქნება, რაც ეგ მაწუხებს.
რამე სერიოზულია? პირდაპირ მითხარით თუ რამეა ისეთი. არაფერი დამიმალოთ

თეთრი და თხელი კანი გაქვს ეტყობა...
თუ სხვა არაფერი გაწუხებს -
საშიში არაფერია.

QUOTE
მაღალი სიმაღლიდან ჩამოვვარდი და ფეხი მაგრა დავარტყი უკვე ორი კვირაა მტკივა ეტყობა დამებეჟა და რამდენ ხანში გამივლის?

გააჩნია დაზიანებას -
თუ ვერ დადიხარ -
ჯობია რენტგენი გაიკეთო -
ან ტრავმატოლოგს აჩვენო -
შეიძლება მყესები იყოს დაზიანებული..

QUOTE
მაქვს ც ჰეპატიტი ანუ შესაბამისად ღვიძლზე პრობლემები, გავცივდი და რა წამლის მიღება შეიძლება .
კიდევ ისეთი მსუბუქი დამამშვიდებლები როგორიცაა სომნა რიცი და გლიცინი ხომ არ ვნებს ღვიძლს?

ჰომეოპათიური ანტიგრიპინი -
100% გარანტიას გაძლევ-
რომ არ გავნებს.

Posted by: rock in rose 12 Jan 2009, 12:53
basa-ttt
კი ძალიან თეთრი ვარ და თხელი კანიც მაქვს
მადლობა რომ დამამშვიდე. ე.ი. არაფერი მჭირს ისეთი


ხო და კიდევ სიცივეში ცხვირი მიწითლდება და ესეც იმის ბრალია, რომ ძალიან ფითქინა ვარ?

Posted by: addicted 13 Jan 2009, 13:11
რა პრობლემა მაქვს sad.gif 18 წლისას გამიკეთეს ქოლეცისტექტომია (ნაღვლის ბუშტი კენჭებით მქონდა სავსე, და სადინარში 2 კენჭი იყო გაჭედილი) ნუ ამას თან დაერთო პანკრეატიტი, ქრონიკული გასტრიტი და მწვავე კოლიტი 1 წლიანი. დიეტისა და მკურნალობის შემდეგ პანკრეატიტი მოვიშორე როგორც იქნა, მაგრამ კოლიტს და გასტრიტს ვერაფერი მოვუხერხე. ჩემს კოლიტს არანაირი დიეტა არ შველის, არც ჰერკულესის, არც მაწვნის, არც ჭარხლის. მკურნალობის პროცესში ყველაფერი ნორმალურად მიდის, დავამთავრებ თუ არა კურსს მაშინვე მეწყება ისევ. რა ვქნა აღარ ვიცი ამ ექიმებთან სირბილმაც დამღალა. ვის მივმართო ამ დარგში ყველაზე კომპეტენტურს (კოდუასთან ვმკურნალობდი)


Posted by: giorgi33 13 Jan 2009, 13:30
QUOTE
რაც შეეხება ფსიქოტროპულებს... იდეალური იქნება ჰეპტრალი,რომელიც ორმაგი მოქმედებითაა აღჭურვილი.ის ერთდორულად არის ჰეპატოპროტექტორიც და ანტიდეპრესანტიც.

ტკივილგამაყუჩებელი თუ არსებობს რვიძლს რომ არ ავნოს ისეთი?

Posted by: Neno_ 14 Jan 2009, 09:38
vano_t
გამარჯობათ, ამ ბოლო დროს დამჩემდა, დილით სანამ გამოვფხიზლდები კბილების კრაჭუნი და ნეტა რისი სიმპტომი შეიძლება იყოს ეს?
მადლობა

Posted by: vano_t 15 Jan 2009, 10:18
Neno_
QUOTE
გამარჯობათ, ამ ბოლო დროს დამჩემდა, დილით სანამ გამოვფხიზლდები კბილების კრაჭუნი და ნეტა რისი სიმპტომი შეიძლება იყოს ეს?
მადლობა

არ ვიცი, ვერ გეტყვი. საერთოდ, არ მგონია მხოლოდ კბილის კრაჭუნით ვინმემ დიაგნოზი დასვას, თანაც ინტერნეტით.

ისე, კბილის კრაჭუნი (ანუ ბრუქსიზმი სამედიცინო ენაზე) ძალიან ბევრ რამეს ახასიათებს. გარდა ამისა, ჯანმრთელ ადამიანსაც შეიძლება ქონდეს. იმიტომ რომ სხვადასვა დაავადებას ახასიათებს ეს პრობლემა, ამისათვის საჭიროა ავადმყოფისაგან დეტალური ისტორიის შეკრება და დეტალური გამოკვლევა (მათ შორის პირის ღრუს დეტალური დათვალიერება).

giorgi33
QUOTE
ტკივილგამაყუჩებელი თუ არსებობს რვიძლს რომ არ ავნოს ისეთი?
საერთოდ მასეთი წამალი არ არსებობს ბუნებაში. ყველა წამალს აქვს პოტენიცური ეფექტი, რომ ნებისმიერი ორგანო დააზიანოს. დაბალ დოზებში ტილენოლს (რაც იგივე აცეტამინოფენი ან პრაცეტამოლია) აშშ-ში ხმარობენ ც ჰეპატიტიანებში. ასევე ხმარობენ იბუპროფენსაც (და მისი ჯგუფის წამლებს) თუ ადამიანს არ აქვს მომატებული INR (INR-არის სისხლის "გათხელების" მაჩვენებელი ღვიძლის ავადმყოფობით დაავადებულებში და მომატებული INR ნიშნავს უფრო თხელ სისხლს). თუ ასეთ ავადმყოფში სისხლი გახელებულია, მაშინ იბუპროფენი სისხლდენის რისკს გაზრდის (ეს უკვე გითხრა ტეხასურმა ჟლეტამ ბენზოხერხით).

ისე, ტილენოლი დაბალ ზდოზებში (2 გრამამდე დღეში) შეიძლება მიიღოს როგორც ც ჰეპატიტით დაავადებულმა, ასევე ციროზით დაავადებულმა ადამიანმა. დაბალ დოზებში ამათ არ აქვთ გაზრდილი რისკი, რომ დაზიანებული ღვიძლის კიდევ უფრო დიდი დაზიანება გამოიწვიოს.

addicted
QUOTE
რა პრობლემა მაქვს sad.gif 18 წლისას გამიკეთეს ქოლეცისტექტომია (ნაღვლის ბუშტი კენჭებით მქონდა სავსე, და სადინარში 2 კენჭი იყო გაჭედილი) ნუ ამას თან დაერთო პანკრეატიტი, ქრონიკული გასტრიტი და მწვავე კოლიტი 1 წლიანი. დიეტისა და მკურნალობის შემდეგ პანკრეატიტი მოვიშორე როგორც იქნა, მაგრამ კოლიტს და გასტრიტს ვერაფერი მოვუხერხე. ჩემს კოლიტს არანაირი დიეტა არ შველის, არც ჰერკულესის, არც მაწვნის, არც ჭარხლის. მკურნალობის პროცესში ყველაფერი ნორმალურად მიდის, დავამთავრებ თუ არა კურსს მაშინვე მეწყება ისევ. რა ვქნა აღარ ვიცი ამ ექიმებთან სირბილმაც დამღალა. ვის მივმართო ამ დარგში ყველაზე კომპეტენტურს (კოდუასთან ვმკურნალობდი)
რას ეძახი კოლიტს? კოლიტის ბევრი ფორმა შეიძლება არსებობდეს. რა სიმპტომები გაქვს და როგორ დაგისვეს დიაგნოზი?

Posted by: Neno_ 15 Jan 2009, 15:27
vano_t
მადლობა! იმედია ჯანმრთელი ადამიანის კრაჭუნი მაქვს მხოლოდ, თუ რამის გაუარესებასაც შევატყობ თავს, მერე მივაკითხავ ექიმს!:D

Posted by: addicted 15 Jan 2009, 23:40
vano_t
QUOTE
რას ეძახი კოლიტს? კოლიტის ბევრი ფორმა შეიძლება არსებობდეს. რა სიმპტომები გაქვს


მშრალი კოლიტი მაქვს, სიმპტომები, მუცლის მუდმივი შებერილობა, თუ ძნელად მოსანელებელ საკვებს მივიღებ შეტევითი ხასიათის ტკივილები მეწყება. გამოკვლევები ტომოგრაფიაც გამიკეთეს და გასტროსკოპიაც

Posted by: vano_t 16 Jan 2009, 02:25
addicted
QUOTE
QUOTE
რას ეძახი კოლიტს? კოლიტის ბევრი ფორმა შეიძლება არსებობდეს. რა სიმპტომები გაქვს


მშრალი კოლიტი მაქვს, სიმპტომები, მუცლის მუდმივი შებერილობა, თუ ძნელად მოსანელებელ საკვებს მივიღებ შეტევითი ხასიათის ტკივილები მეწყება. გამოკვლევები ტომოგრაფიაც გამიკეთეს და გასტროსკოპიაც

მერე კოლონოსკოპიის აღწერაში რა ეწერა? ან ტომოგრაფიამ რა აღწერა? კოლონოსკოპიის დორს ბიოფსია თუ გაგიკეთდა და თუ გაგიკეთდა ბიოსფისაზე რა იქნა ნანახა.

არის ასეთი "დაავადება": irritable bowel syndrome, ანუ IBS (ადვილად გაღიზიანებადი ნაწლავების სინდრომი-უხეშად რომ ვთარგმნოთ). ოღონდ ამის დიაგნოზის ისმება გამორიცხვით: კოლიტების სხვადასხვა ფორმა და ნაწლავების სხვა დაავადებები უნდა გამოირიცხოს. IBS ყველაზე ხშირი მიზეზია ქრონიკული სიმპტომების.

Posted by: karaoke 16 Jan 2009, 02:40
rock in rose
QUOTE
18 წლის გოგო ვარ, უბრალოდ ეგ პრობლემა მაქვს მეტი არაფერი. დაახლოებით 3 ან 4 თვე იქნება, რაც ეგ მაწუხებს. რამე სერიოზულია? პირდაპირ მითხარით თუ რამეა ისეთი. არაფერი დამიმალოთ

არ შეგეშინდეს რაც მთავარია .... ზუსტ დიაგნოზს თვალების ჩაშავებით ვერავინ დაგისვამს . მე ერთ ანალიზზე გეტყვი რო აუცილებლად გაიკეთე.
მე მქონდა რგოლები თვალების გარშემო კანზე. აღმომაჩნდა ც ჰეპატიტი. მოკლედ არ დაიზარო და ჰეპათოლოგთან შეამოწმე. უფრო დიდი შანსია რო არ იყოს ეს. მაგრამ ჯობია რო თავი დაიმშვიდო. ისე კი სწორ განყოფილებაში მოხვდი მე მგონი. რაღაცა იქნება რაც ინტერნისტბს ეხებათ.

მოკლედ მე გირჩევ რო დაიმვიდო თავი და გამორიცხო ჰეპატიტები.

Posted by: lizofobi 16 Jan 2009, 10:34
შეკითხვა ინტერნისტებს - ელექტროთერმომეტრი (პირში რო იდებ biggrin.gif ), "ფილიპსის" წარმოების - რამდენად სანდოა და რამდენად ზუსტად ზომავს სიცხეს? თუ ისევ რუსული მამაპაპური თერმომეტრი ჯობია მაგას?

Posted by: karaoke 16 Jan 2009, 10:48
QUOTE
rock in rose

ხო დამავიწყდა რო მეთქვა . როგორც კი მკურნალობა დავიწყე ც ჰეპატიტებზე რგოლები მაშინვე გამიქრა. ერთ თვეში ... მახსოვს ყველა აღნიშნავდა ვინც მხვფებოდა გზაში...მოგვიანებით ანალიზებმაც აჩვენეს ღვიძლის ყველა მაჩვენებლის ფიზიოლოგიურ საზღვრებში დაბრუნება.
დღეს აღარ მაქვს რგოლები თვალებზე ... მიუხედავად იმისა რო ალტ და გამაGTცოტათი აწეულია ფიზიოლოგიურიდან .

მოკლედ რგოლები თვალებზე ერთ ერთი ნიშანია ც ჰეპატიტის . მაგრამ კიდე ვიმეორებ უბრალოდ შეამოწმე რო არ გაქვს ... მე არ ვარ ექიმი. მარა ვიცი რო სხვა უამრავმა რამემ შეიძლება გამოიწვიოს რგოლები . და ბევრი არცაა საშიში. მოკლედ შეამოწმე .....

Posted by: vano_t 16 Jan 2009, 10:50
lizofobi
QUOTE
შეკითხვა ინტერნისტებს - ელექტროთერმომეტრი (პირში რო იდებ  biggrin.gif ), "ფილიპსის" წარმოების - რამდენად სანდოა და რამდენად ზუსტად ზომავს სიცხეს? თუ ისევ რუსული მამაპაპური თერმომეტრი ჯობია მაგას?

ელექტროთერმომეტრი ყურშიც შეიძლება შეიდო (ასე აღარ ჟღერს ცუდად smile.gif). ელექტროთერმომეტრს სიზუსტით ვერცხლისწყლის თერმომეტრი ჯობია. ოღონდ ეს არ არის რუსული თერმომეტრი. რუსებსაც შეუძლიათ გააკეთონ (და აკეთებენ). მაგრამ ისე გერმანელი ფიზიკოსის, ფარენჰაიტის, შექმნილია. თანაც იღლიაში იდებ (არც პირში და არც ყურში). სხვაგანაც შეიძლება შედო ეს თერმომეტრი, მარა იღლია საკმაოდ კარგია smile.gif

rock in rose
არავითარი ც ჰეპატიტი. ყველაზე ხშირი მიზეზია უძილობა და სტრესი. ხშირად ალერგიულ ავადმყოფებს შეიძლება ქონდეთ. რაც უფრო თეთრი კანი გაქვს, ემით უფრო კონტრასტი იქმნება თვალის გარშემო კანის ფერსა და დანარჩენი სახის ფერს შორის. ასე რომ, ფითქინა კანიანებს ნორმაშიც შეიძლება ქონდეს გამოხატული.

Posted by: lizofobi 16 Jan 2009, 10:51
QUOTE (vano_t @ 16 Jan 2009, 10:50 )
lizofobi
QUOTE
შეკითხვა ინტერნისტებს - ელექტროთერმომეტრი (პირში რო იდებ  biggrin.gif ), "ფილიპსის" წარმოების - რამდენად სანდოა და რამდენად ზუსტად ზომავს სიცხეს? თუ ისევ რუსული მამაპაპური თერმომეტრი ჯობია მაგას?

ელექტროთერმომეტრი ყურშიც შეიძლება შეიდო (ასე აღარ ჟღერს ცუდად smile.gif). ელექტროთერმომეტრს სიზუსტით ვერცხლისწყლის თერმომეტრი ჯობია. ოღონდ ეს არ არის რუსული თერმომეტრი. რუსებსაც შეუძლიათ გააკეთონ (და აკეთებენ). მაგრამ ისე გერმანელი ფიზიკოსის, ფარენჰაიტის, შექმნილია. თანაც იღლიაში იდებ (არც პირში და არც ყურში). სხვაგანაც შეიძლება შედო ეს თერმომეტრი, მარა იღლია საკმაოდ კარგია smile.gif

აჰა, ანუ ელექტროთერმომეტრი სანდო არაა... და აბა ორალური ტემპერატურა როგორ ისინჯება? ვერცხლისწყლის თერმომეტრით?

Posted by: vano_t 16 Jan 2009, 10:57
lizofobi
QUOTE
აჰა, ანუ ელექტროთერმომეტრი სანდო არაა... და აბა ორალური ტემპერატურა როგორ ისინჯება? ვერცხლისწყლის თერმომეტრით?
სანდო არ არისთქო არ მითქვამს. ნაკლებად ზუსტია (უფრო დიდი ცვალებადობა ახასიათებს; ბატარიებზეა დამოკიდებული და ელექტრონული ნაწილების ჩვენება დამოკიდებულია ბატარიის ხარისხზე და ა.შ.). ძალიან გავრცელებულია ელექტრონული თერმომეტრი და შეგიძლია დაეყრდნო მის ჩვენებას. ისე ორივეთი შეიძლება გაზომო ტემპერატურა ორალურად.

Posted by: basa-ttt 16 Jan 2009, 11:07
QUOTE
basa-ttt
კი ძალიან თეთრი ვარ და თხელი კანიც მაქვს
მადლობა რომ დამამშვიდე. ე.ი. არაფერი მჭირს ისეთი


ხო და კიდევ სიცივეში ცხვირი მიწითლდება და ესეც იმის ბრალია, რომ ძალიან ფითქინა ვარ?

rock in rose
კი ალბათ ასეა -
ისე ჰემოგლობინი ინახე მერე -
ახლა ზამთარია და შეიძლება ცოტა დაიწია..

QUOTE
დიეტისა და მკურნალობის შემდეგ პანკრეატიტი მოვიშორე როგორც იქნა, მაგრამ კოლიტს და გასტრიტს ვერაფერი მოვუხერხე. ჩემს კოლიტს არანაირი დიეტა არ შველის, არც ჰერკულესის, არც მაწვნის, არც ჭარხლის. მკურნალობის პროცესში ყველაფერი ნორმალურად მიდის, დავამთავრებ თუ არა კურსს მაშინვე მეწყება ისევ. რა ვქნა აღარ ვიცი ამ ექიმებთან სირბილმაც დამღალა. ვის მივმართო ამ დარგში ყველაზე კომპეტენტურს (კოდუასთან ვმკურნალობდი)

ერთი კარგი რჩევა შემიძლია მოგცე -
უზმოზე როცა გაიღვიძებ 1 საათის განმავლობაში სვი წყალი - ყლუპობით, რამდენსაც შესძლებ,
მერე ერთი საათის მერე შეჭამე ან გახეხილი ვაშლი ან წყლიანი მაწონი, ან შავი ქლიავის ჩირი.
ძილის წინ ყოველდღე დალიე მაწონი.
და ხშირად ჭამე ჭარხლის ფხალი - თითქმის ყოველი სადილის წინ...
(ჭარხლის ფხალი ან ნიგვზით კეთდება ან მაიონეზით -
თუ გინდა რეცეპტს გეტყვი)
თუ ამანაც არ გიშველა -
მერე გამოკვლევა ჩაიტარე.
* * *
QUOTE
არ შეგეშინდეს რაც მთავარია .... ზუსტ დიაგნოზს თვალების ჩაშავებით ვერავინ დაგისვამს . მე ერთ ანალიზზე გეტყვი რო აუცილებლად გაიკეთე.
მე მქონდა რგოლები თვალების გარშემო კანზე. აღმომაჩნდა ც ჰეპატიტი. მოკლედ არ დაიზარო და ჰეპათოლოგთან შეამოწმე. უფრო დიდი შანსია რო არ იყოს ეს. მაგრამ ჯობია რო თავი დაიმშვიდო. ისე კი სწორ განყოფილებაში მოხვდი მე მგონი. რაღაცა იქნება რაც ინტერნისტბს ეხებათ.

მოკლედ მე გირჩევ რო დაიმვიდო თავი და გამორიცხო ჰეპატიტები

ჰეპატიტზე ეჭვი ჩნდება მაშინ, როცა ადამიანი უჩივის უმიზეზო სისუსტეს, ზოგჯერ გულისრევის შეგრძნებას
აქვს რაღაც დისკომფორტი - და ვერ ხვდება რა სჭირს.
ეს გოგო ამბობს არაფერი მაწუხებსო -
ანუ პირველ რიგში არა თვალის გარშემო რგოლებს,
არამედ სისუსტეს იტყოდა...

Posted by: addicted 16 Jan 2009, 13:38
vano_t
გამოკვლევების პასუხები დედაჩემთან მაქვს და როდესაც ვნახავ დავწერ. ზუსტად არ მახსოვს რომელი გამოკვლევით დამიდგინეს მაგრამ ერთ ერთ პასუხში ნაღვლის სადინარების მკვეთრი შევიწროვება ეწერა. და მარცვლოვანი და რაღაც ასეთი სახელის გასტრიტი.
თქვენ ვის მირჩევთ ვის მივმართო და სად მივიდე? ასე გაგრძელება რომ აღარ შეიძლება ვიცი. წამლებზე დამოკიდებული გავხდი და ეს ნორმალური რომ არაა მეც ვიცი sad.gif

Posted by: Dr-Rock 16 Jan 2009, 22:25
გამარჯობა ყველას. გულიკო ჩაფიძესთან მინდა სასწრაფოდ დაკავშირება და იქნებ ვინ მე დამეხმაროთ ან მისი პირადად ან კლინიკის ნომერი მომწეროთ. დზალიან მჩირდება და იქნებ რაიმე საკონტაქტო კორდინატები გამაგებინოთ. დიდი მადლობა წინასწარ ძალიან დამეხმარებით.

Posted by: KingOfSorrow 17 Jan 2009, 17:49
გამარჯობათ ... ხალხო მიშველეთ ღამე ყურებში მაქვს აუტანელი შუილი და რისი ბრალია?? იქნებ ვინმემ დამაკვალიანოს ან რა დავლიო ?

Posted by: zeppelina 17 Jan 2009, 20:24
vano_t
თუ შეგიძლიათ მიპასუხეთ,ყელის ტკივილის გარდა რამ შეიძლება გამოიწვიოს ყელში უსიანოვნო შეგრძნება, თითქოს რაღაც გაქვს გაჩხერილი.დილაობით სიმშრალე. დავღეჭე სტრეპტოციდი,მაგრამ არ მიშველა.უკვე ორი კვირაა მაწუხებს. თავიდან დავუკავშირე ნიგვზიანის და თხილის ჭამას. შეიძლება რამე სერიოზული იყოს?



Posted by: vano_t 18 Jan 2009, 01:20
zeppelina
QUOTE
თუ შეგიძლიათ მიპასუხეთ,ყელის ტკივილის გარდა რამ შეიძლება გამოიწვიოს  ყელში უსიანოვნო შეგრძნება, თითქოს რაღაც გაქვს გაჩხერილი.დილაობით სიმშრალე. დავღეჭე სტრეპტოციდი,მაგრამ არ მიშველა.უკვე ორი კვირაა მაწუხებს. თავიდან დავუკავშირე ნიგვზიანის და თხილის ჭამას. შეიძლება რამე სერიოზული იყოს?

ლორწოვანის ნებისმიერმა დაზიანება შეიძლება გამოიწვიოს ყელის ტკივილი. საერთოდ, 2 კვირა თუ გრძელდება, სერიოზული რაიმე არ მგონია რომ იყოს. სერიუზულ პრობლემებს სხვა სერიოზული სიმპტომებიც ახლავს ხშირად. შენს ადგილას, ჯერ მარტო სიმპტომს ვუმკურნალებდი (შეგიძლია პარაცეტამოლი ან იბუპროფენი მიიღო; ან კიდევ, თუ იშოვება, ლოკალური ანესთეტიკები პირის ღრუსათვის; შეგიძლია სოდიანი სავლები ივლო). თუ კიდევ ბევრ ხანს გაგრძელდა (2-4 კვირა შეიძლება კიდევ დაიცადო), მერე მიდი ექიმთან პირის ღრუს გამოკვლევისათვის.
* * *
KingOfSorrow
QUOTE
გამარჯობათ ... ხალხო მიშველეთ ღამე ყურებში მაქვს აუტანელი შუილი და რისი ბრალია?? იქნებ ვინმემ დამაკვალიანოს ან რა დავლიო ?

ყურებში შუილს (რასაც სამედიცინო ენაზე ტინიტუსი ქვია) აქვს აურაცხელი მიზეზი. ამ მიზეზების ჩამოთვლა ერთ ფურცელს გაავსებს.

მოკლედ რომ ავხსნა სმენი ფიზიოლოგია ასეთია: ბგერა, რომელიც წარმოადგენს ჰაერის რხევას გამტარი გზებით (ყურის გარეთა მილი) გადაეცემა დაფის აპკს (ანუ ყურის ბარაბანს), რომელიც თავის მხრივ იწყებს რხევას. შემდეგ ყურის ბარაბანის რხევა გადაეცემა პატარა სასმენ ძვლებს; ეს სასმენი ძვლები რხევას გადაცემენ შიგნითა ყურის სპეციალურ აპარატს. შემდეგ ამ აპარატში სითხის (ე.წ. ენდოლიმფის) რხევა იწყება; ეს სითხის რხევა გადაეცემა პატარა უჯრედებს (რეცეპტორებს) რომლებიც ამ რხევას საბოლოოდ გარდაქმნიან ელექტრულ სიგნალში; ეს სიგნალი სმენის ნერვის საშუალებით მიდი ტვინში და იქ ხდება ანალიზი ხმის. ნებისმიერ ამ უბანზე ცვლილებამ (ფიზიოლოგიურმა თუ დაავადებამ) შეიძლება მოგცეს ტინიტუსი.

უმეტეს შემთხვევაში ეს შეიძლება იყოს მარტივი დროებითი პრობლემა, რომელიც მალე გადის (მაგალითად ცოტა სითხე შუა ყურში, ან დაფის აპკის ან სასმენი ძვლების თვისებების დროებითი შეცვლა დავუშვათ გარემო წნევის ცვალებადობის ან ხმაურიან ადგილას ყოფნის გამო).

თითქმის ყველა ადამიანს ქონია ეს განცდა.

თუ დიაგნოზია საჭირო, მაშინ ექიმმა დეტალურად უნდა დაათვალიეროს ყური და დეტალური გამოკითხვა გააკეთოს. უემტეს შემთხვევაში არაფერი დიაგნოზი არ არის საჭირო და პრობლემა მალე გადის. სმენის ჰიგიენა დაიცავი, ეს ყველაზე კარგია.

Posted by: KingOfSorrow 18 Jan 2009, 19:13
შესაძლებელია თუ არა რომ ნერვოზული იყოს ან და სისხლის მიმოქცევის დარღვევის გამო?? უბრალოდ ჯერ 26 წლის ვარ და ... მაგრად დამანერვოზა მაგან ცალკე დაძინებაში მაგრად მიშლის ხელს ისედაც ცოტა მძინავს ;( მოკლედ რამე დამამშვიდებელი რო ვთხლიშო მიშველის?

Posted by: lgogokhia 19 Jan 2009, 08:55
vano_t
დღეს თხილამურებზე სრიალის დროს დროშას და ბადეს დავეჯახე და თავით ჩავერჭე თოვლში... არც გული ამრევია ან მითუმეტეს გრძნობა დამიკარგავს, უბრალოდ თავბრუ დამეხვა 1-2 წუთი, მერე ავდექი ჩვეულებრივად და ბოლომდე ჩამოვედი... მერე მანქანაც ვატარე დაახლოებით 1 საათი ჩვეულებრივად, მაგრამ თითქოს გაბრუებულივით ვიყავი... სახლში რო მოვედი ჩამეძინა და რო გავიღვიძე თითქოსდა თავი მტკივასავით... აი მაღალი წნევა რო გაქვს და შიგნიდან რო გაწვება რაღაცა... მსგავსი ტკივილი წინათაც მქონდა, როდესაც არ დავცემულვარ strenuous exercise-ს პონტში და მეორე დღეს ჩვეულებრივად ვიყავი... რას ფიქრობ, წავიდე ექიმთან? appointment მაინც 1 კვირაში მიწევს ჩემს PCP-სთან, მაგრამ რამე ვქნა უფრო ადრე?

Posted by: vano_t 19 Jan 2009, 11:26
lgogokhia
QUOTE
დღეს თხილამურებზე სრიალის დროს დროშას და ბადეს დავეჯახე და თავით ჩავერჭე თოვლში... არც გული ამრევია ან მითუმეტეს გრძნობა დამიკარგავს, უბრალოდ თავბრუ დამეხვა 1-2 წუთი, მერე ავდექი ჩვეულებრივად და ბოლომდე ჩამოვედი... მერე მანქანაც ვატარე დაახლოებით 1 საათი ჩვეულებრივად, მაგრამ თითქოს გაბრუებულივით ვიყავი... სახლში რო მოვედი ჩამეძინა და რო გავიღვიძე თითქოსდა თავი მტკივასავით... აი მაღალი წნევა რო გაქვს და შიგნიდან რო გაწვება რაღაცა... მსგავსი ტკივილი წინათაც მქონდა, როდესაც არ დავცემულვარ strenuous exercise-ს პონტში და მეორე დღეს ჩვეულებრივად ვიყავი... რას ფიქრობ, წავიდე ექიმთან? appointment მაინც 1 კვირაში მიწევს ჩემს PCP-სთან, მაგრამ რამე ვქნა უფრო ადრე?

ნაკლებ სავარაუდოა, რომ concussion/contusion იყოს. შენ ექიმი ხარ, როგორც ვიცი, და წასვლა/არწასლვა შენ უნდა გადაწყვიტო. მე შემიძლია რჩევა მოგცე. რა თქმა უნდა, მე ყველას ვეუბენები, რომ ასეთ შემთხვევაში ექიმთან უნდა მიხვიდე. მე რომ დამემართოს მასე და ცოტა თავის ტკივილის მეტი არაფერი მქონდეს, მე არ მივალ ექიმთან. მაგრამ, შენ რამდენად შეგიძლია გაერკვე შესაძლო პრობლემებში და მოცემული ტრამვის დროს მიღებულ გართულებებში, მაგ ძნელი სათქმელია. ამიტომ, ჩემი რჩევა იქნება, რომ მიხვიდე ექიმთან. საქმე იმაში, რომ თავის ქალას და ტვინის ტრამვის შანსი შეიძლება ნაკლებია (თუ ღრმა თოვლი იყო, მაგალითად, და ნიადაგს არ დაარტყი თავი), მაგრამ სამაგიეროდ, კისრის მალების დაზიანების შანსი არის. მოკლედ, თუ შენ თვითონ არ შეგიძლია გარკვე რა შეიძლება ასეთ ტრამვას მოყოლილიყო და რა სიმპტომები უნდა ქონდეს შესაძლო გართულებებს, მაშინ ჯობია მიხვიდე. PCP-ს ოფისს დაურეკე უბრალოდ და რისკეჯული გააკეთე.

ისე, ასეთ დროს რომ გამოიკვლევ ავადმყოფს, თუ რაიმე საეჭვო ვერ შენიშნე, სახლში შეგიძლია გაუშვა. კტ სკანირება შეიძლება გაგიკეთონ და შეიძლება არა. სახლში თუ გაგიშვეს, მაშინ გეტყვიან დააკვირდი ნევროლოგიურ სიმპტომებს და თუ რაიმე ახალი გამოჩნდა ან თავის ტკივილის აუტანელი გახდა, მაშინ მოდი უკან. საავადმყოფოში ობზერვაციაც შეიძლება გაგიკეთონ, თუ რაიმე საეჭვო შენიშნეს. მოკლედ, გამოკვლევის (ფიზკურის ჯერ) გარეშე ძნელია რაიმეს თქმა ინტერნეტით. აქ მხოლოდ ზოგადი რჩევა შეიძლება მიიღო.

Posted by: lgogokhia 19 Jan 2009, 12:25
vano_t
thnx უბრალოდ second opinion მინდოდა smile.gif
QUOTE
მე რომ დამემართოს მასე და ცოტა თავის ტკივილის მეტი არაფერი მქონდეს, მე არ მივალ ექიმთან.
ჩემ თავს მარტო არ დავაბრალებ რამე რო იყოს biggrin.gif JK..

QUOTE
თუ ღრმა თოვლი იყო, მაგალითად, და ნიადაგს არ დაარტყი თავი
სათხილამურო ტრასა იყო, არც რბილი თოვლი და არც ასფალტი... ხო იცი რა..

QUOTE
კისრის მალების დაზიანების შანსი არის
ამის ალბათ არ მეშინია, რადგან არ დავცემულვარ მაგ პროფილით... მოკლედ, ყველაზე ნაკლებად ალბათ...

QUOTE
PCP-ს ოფისს დაურეკე უბრალოდ და რისკეჯული გააკეთე
2 კვირაა რაც ველოდები physical-ებზე და აქეთ გადმოტანის შანსი არ იქნება... ვიცი რასაც მეტყვიან - if this is an emergency please call 911 biggrin.gif

QUOTE
კტ სკანირება შეიძლება გაგიკეთონ და შეიძლება არა. სახლში თუ გაგიშვეს, მაშინ გეტყვიან დააკვირდი ნევროლოგიურ სიმპტომებს და თუ რაიმე ახალი გამოჩნდა ან თავის ტკივილის აუტანელი გახდა, მაშინ მოდი უკან. საავადმყოფოში ობზერვაციაც შეიძლება გაგიკეთონ, თუ რაიმე საეჭვო შენიშნეს.
ეგ მეზარება ძალიან...

QUOTE
დააკვირდი ნევროლოგიურ სიმპტომებს
ხო რა ვნახავ ეს ერთი კვირა და თუ რამე მერე მომიწევს ალბათ

QUOTE
მოკლედ, გამოკვლევის (ფიზკურის ჯერ) გარეშე ძნელია რაიმეს თქმა ინტერნეტით.
გაიხარე, უკვე მითხარი რაც მინდოდა რო გამეგო smile.gif

Posted by: Solveig 19 Jan 2009, 13:11
მე მაქვს ასეთი კითხვა:

აღმოვაჩინე, რომ დედაჩემს ორივე ხელის წინამხარზე აქვს პიგმენტური ლაქები, დაახლოებით ასე 5X20სმ ფართობზე, მკაფიო საზღვრით,უსწორმასწორო კიდეებით..და ასე აქვს უკვე 1 წელია. რისი ნიშანი შეიძლება იყოს?

Posted by: lizofobi 19 Jan 2009, 16:29
36.8 სიცხეა? spy.gif და ისა, რამდენი წუთით უნდა გაიჩერო თერმომეტრი რო იზომავ?

Posted by: tete- 21 Jan 2009, 04:33
აქ რამე ბლატებია საჭირო კითხვაზე რო გიპასუხონ? down1.gif

Posted by: vano_t 21 Jan 2009, 09:58
Te0
QUOTE
აქ რამე ბლატებია საჭირო კითხვაზე რო გიპასუხონ? down1.gif

კალცის ნაკლებობასა და მეხსიერებას შორის კავშირი არ გამიგია. კალიცუმის ნაკლებობა უფრო ზვლებს აზიანებს.

lizofobi
QUOTE
36.8 სიცხეა? spy.gif და ისა, რამდენი წუთით უნდა გაიჩერო თერმომეტრი რო იზომავ?
36.8 ნრომალური სიცხეა. თერმომეტრს გააჩნია და გარემოს გააჩნია. 2-3 წუთი საკმარისია ალბათ.

Solveig
QUOTE
მე მაქვს ასეთი კითხვა:

აღმოვაჩინე, რომ დედაჩემს ორივე ხელის წინამხარზე აქვს პიგმენტური ლაქები, დაახლოებით ასე 5X20სმ ფართობზე, მკაფიო საზღვრით,უსწორმასწორო კიდეებით..და ასე აქვს უკვე 1 წელია. რისი ნიშანი შეიძლება იყოს?
ლაქას გააჩნია. შეიძლება ნორმალური ლაქაა, რომელიც ასაკთან დაკავშირებით ჩნდება: მაგალითად seborrheic keratosis ან ზოგიერთი ლენტიგო. თუ სურათს დადებ, უფრო კარგი იქნება.

Posted by: kasandra 22 Jan 2009, 11:49

ბოლო ხანია ისეთი შეგრძნება მაქვს, თითქოს ნერწყვს ვერ ვყლაპავ. ჭამის დროს მოულოდნელად ლუკმა მახრჩობს მგონია, ერთი-ორჯერ მართლა კინაღამ გავიგუდე. ეხლა როცა მემართება, თავს ვაწყნარებ, რომ არაფერია ცხვირით ხო ვსუნთქავ თქო და მეშვება. ყელი არ მტკივა. რის გამო შეიძლება მემართებოდეს ეს? ჰო, ნერვიული არ ვარ, საკმარისზე მეტად მშვიდი ადამიანი ვარ, არც გულში ვიცი პრობლემების ჩახვევა boli.gif

ვეწევი სიგარეტს user.gif

Posted by: texasuri jleta benzoxerxit 22 Jan 2009, 12:16
QUOTE (Solveig @ 19 Jan 2009, 13:11 )
მე მაქვს ასეთი კითხვა:

აღმოვაჩინე, რომ დედაჩემს ორივე ხელის წინამხარზე აქვს პიგმენტური ლაქები, დაახლოებით ასე 5X20სმ ფართობზე, მკაფიო საზღვრით,უსწორმასწორო კიდეებით..და ასე აქვს უკვე 1 წელია. რისი ნიშანი შეიძლება იყოს?

სხვა არანაირი სიმპტომი?ადვილად დაღლა,დაბალი წნევა...


ენდოკრინოლოგას აჩვენე,ადისონი არ გაიპაროს boli.gif

Posted by: shtori 22 Jan 2009, 23:09
2 თვეზე მეტია მახველებს,სველი ხველა მაქვს. ლაზოლვანს (სახელი თუ არ მეშლება) ვსვამდი არ მიშველა. ახლა პულმოლანს ვსვამ მე-2-ე კვირა გავიდა უკვე. თითქოს გამიარა. მაგრამ დღეს არ დავლიე და ისევ ისე დამეწყო. ყლაპვის დროს გულის რევის შეგრძნებას მიჩენს. ცივს არაფერს არ ვსვამ არც ვჭამ. რომელ წამალს მირჩევდით?

Posted by: Solveig 23 Jan 2009, 00:37
texasuri jleta benzoxerxit
QUOTE
სხვა არანაირი სიმპტომი?ადვილად დაღლა,დაბალი წნევა...


ენდოკრინოლოგას აჩვენე,ადისონი არ გაიპაროს 

საერთოდ, იმდენად სერიოზულად აქვს ჯანმრთელობა შეერყეული, რომ სულ რაღაც აწუხებს და...ადვილად დაღლა ყოველთვის ახასიათებდა, წნევა კი თურმე მაღლა უწევს ამ ბოლო ხანებში. სადაზღვევო კომპანიის ექიმს სჩვენებია და იმას უთქვამს-ასაკობრივიაო, არადა ჯერ 58 წლისაა. მე უფრო იმიტომ შემეპარა ეჭვი, რომ მთლიანი ლაქაა და არა ცალკეული.

ვერ იქნა და ექიმთან ვერ წავიყვანე, არადა, მეც მივდივარ უკვე 2 დღეში sad.gif ეს ყველაფერი ახლა აღმოვაჩინე, რომ ჩამოვედი. არაფერს მეუბნებოდა. არ ვარ საქართველოში და ყურადღებას ვეღარ ვაქცევ..


Posted by: shanyva 23 Jan 2009, 12:22
ყელში რაღაც თეთრი წარმონაქმნები რა შეიძლება იყოს?ჩირქი არ არის რაღაც ნაირია,მოშორების შემთხვევაში პატარა ორმოს smile.gif ტოვებს,რამდენიმე წლის წინ მეუღლეს ქონდა ზუსტად ასეთი და სტაფილოკოკი(თუ რაღაც ამდაგვარი)უთხრეს,მაინტერესებს ეგ უკანასკნელი რამდენად საშიშია ორსულობის დროს,ან ხომ არ უშლის ხელს ორსულობის დადგომას?
მადლობა წინასწარ

Posted by: shtori 25 Jan 2009, 00:38
ამ ბოლო დროს თავის ტკივილები მაწუხებს, როცა ვსვენებ მაშინ შაბათ-კვირას. კარგი დასვენება გამომდის ხოლმე biggrin.gif ვითომ რატომ უნდა იყოს ესე?

Posted by: shanyva 25 Jan 2009, 00:40
lgogokhia
vano_t
ყელში რაღაც თეთრი წარმონაქმნები რა შეიძლება იყოს?ჩირქი არ არის რაღაც ნაირია,მოშორების შემთხვევაში პატარა ორმოს ტოვებს,რამდენიმე წლის წინ მეუღლეს ქონდა ზუსტად ასეთი და სტაფილოკოკი(თუ რაღაც ამდაგვარი)უთხრეს,მაინტერესებს ეგ უკანასკნელი რამდენად საშიშია ორსულობის დროს,ან ხომ არ უშლის ხელს ორსულობის დადგომას?
მადლობა წინასწარ
* * *
lgogokhia
vano_t
ყელში რაღაც თეთრი წარმონაქმნები რა შეიძლება იყოს?ჩირქი არ არის რაღაც ნაირია,მოშორების შემთხვევაში პატარა ორმოს ტოვებს,რამდენიმე წლის წინ მეუღლეს ქონდა ზუსტად ასეთი და სტაფილოკოკი(თუ რაღაც ამდაგვარი)უთხრეს,მაინტერესებს ეგ უკანასკნელი რამდენად საშიშია ორსულობის დროს,ან ხომ არ უშლის ხელს ორსულობის დადგომას?
მადლობა წინასწარ

Posted by: vano_t 25 Jan 2009, 09:51
shanyva
QUOTE
ყელში რაღაც თეთრი წარმონაქმნები რა შეიძლება იყოს?ჩირქი არ არის რაღაც ნაირია,მოშორების შემთხვევაში პატარა ორმოს  ტოვებს,რამდენიმე წლის წინ მეუღლეს ქონდა ზუსტად ასეთი და სტაფილოკოკი(თუ რაღაც ამდაგვარი)უთხრეს,მაინტერესებს ეგ უკანასკნელი რამდენად საშიშია ორსულობის დროს,ან ხომ არ უშლის ხელს ორსულობის დადგომას?
მადლობა წინასწარ

ყელის დათვალიერების გარეშე ძნელია მაგის თქმა, მარა, რამდენია ეს წარმონაქმნები? 1, 2 თუ ბევრი? რა ადგილზეა? როცა იცილებ, სისხლი მოდის თუ არა? რამხელაა ეს წარმონაქმნი ზომაში და რანაირი ფორმა აქვს?

მოკლედ, თეთრი ლაქა ბევრმა რამემ შეიძლება მოგცეს: ვირუსულმა ინფექციებმა (ვირუსულმა ფარინგიტებმა), ლეიკოპლაკიამ (მაგრამ ამას ვერ მოიშორებ), პირის ღრუს კანდიდიოზმა (სოკოვანი ინფექციაა, რომელიც შეიძლება გაჩნდეს სტეროიდების ხმარების შემდეგ, ან ანტიბიოტიკების მიღების შემდეგ, ანუ როცა იმუნური სისტემა დათრგუნულია), დიფტერიამ (მარა ეს რთული ინფექციაა და ავადმყოფი ამ დროს ძალიან ცუდად არის).

მოკლედ, თუ გინდა რომ დიაგნოზი მიიღო, ექიმმა უნდა ჩაგხედოს (რა თქმა უნდა, ექიმს უნდა ესმოდეს რას ნახულობს).

shtori
QUOTE
2 თვეზე მეტია მახველებს,სველი ხველა მაქვს. ლაზოლვანს (სახელი თუ არ მეშლება) ვსვამდი არ მიშველა. ახლა პულმოლანს ვსვამ მე-2-ე კვირა გავიდა უკვე. თითქოს გამიარა. მაგრამ დღეს არ დავლიე და ისევ ისე დამეწყო. ყლაპვის დროს გულის რევის შეგრძნებას მიჩენს. ცივს არაფერს არ ვსვამ არც ვჭამ. რომელ წამალს მირჩევდით?
2 თვეზე მეტი თუ ახველებ, მაშინ ამას დიაგნოზი ჭირდება. ეს იმას ნიშნავს, რომ ექიმს უნდა ეჩვენო. ქრონიკულ ხველას ბევრი მიზეზი აქვს და მკურანლობას მიზეზის მიხედვით ინიშნება. ყველაზე ხშირი მიზეზებია: ასთმა, გასტროეზოფაგალური რეფლუქსი და postnasal drip (ეს არის ის სიმპტომი, როცა ავადმყოფს ცხვირის გამონადენი ყელში გადადის). საქართველოში ტუბერკულოზზეც უნდა იყოს აქცენტი, იმიტომ რომ გავრცელებულია საკმაოდ. არის კიდევ ქრონიკულ ხველის სხვა ბევრი მიზეზი.

Posted by: shtori 25 Jan 2009, 11:05
vano_t
გმადლობ.
ისე გაციებისგან დამრჩა. სურდო მოვირჩინე, ხველა ვერა
რამდენიმე დღეც და მერე ვეხალები ექმის. როგორ მიყვარს...

Posted by: shanyva 25 Jan 2009, 11:28
vano_t
მადლობა,აუცილებლად მივალ ექიმთან

Posted by: magdalina 27 Jan 2009, 12:50
ორი კვირის წინ წავიქეცი და თავი დავარტყი,მას შემდეგ მაქვს თაბრუსხვევა რო ვმოძრაობ განსაკუთრებით მაშინ,თავის ტკივილი,ყურებში რაღაც ხმა,საფეთქლებისა და კეფის ტკივილი,ხანდახან თვალებისაც(თითქოს რაღაც მიჭერს ისეთი).


ექიმმა მითხრა შერყევა არ არისო,მაგრამ შიდა ქალის წნევა არის მერყევიო.დანიშნულებაც მომცა თითქოს შვებას ვგრძნობ,მაგრამ თაბრუს ხვევა მაინც მაქვს რომ ვმოძრაობ და სუსტად ვარ. .დამეხმარეთ რა ....

Posted by: texasuri jleta benzoxerxit 29 Jan 2009, 01:50
QUOTE (magdalina @ 27 Jan 2009, 12:50 )
ორი კვირის წინ წავიქეცი და თავი დავარტყი,მას შემდეგ მაქვს თაბრუსხვევა რო ვმოძრაობ განსაკუთრებით მაშინ,თავის ტკივილი,ყურებში რაღაც ხმა,საფეთქლებისა და კეფის ტკივილი,ხანდახან თვალებისაც(თითქოს რაღაც მიჭერს ისეთი).


ექიმმა მითხრა შერყევა არ არისო,მაგრამ შიდა ქალის წნევა არის მერყევიო.დანიშნულებაც მომცა თითქოს შვებას ვგრძნობ,მაგრამ თაბრუს ხვევა მაინც მაქვს რომ ვმოძრაობ და სუსტად ვარ. .დამეხმარეთ რა ....

ესე იგი მაინც შერყევა ყოფილა fig.gif მიმარტე სხვა ნევროლოგს

Posted by: skoch 31 Jan 2009, 03:13
ოპერაციის სჰემდეგ ვენა მაკ გამაგრებული ზაან 2ტვე გავიდა უკვე და რატოომ???

Posted by: texasuri jleta benzoxerxit 31 Jan 2009, 03:25

თრომბოზია ალბათ და იმიტომ. ანგიოლოგთან სასწრაფოდ!!
skoch

Posted by: skoch 31 Jan 2009, 03:30
მაგრად ვვარჯიშობ 3კვირააა ვაპსჰე არ მიჯირს ამ ხელით აი როგორ ვტკვა გაბერილია მტეელ სიგრძეზე მაგრად....
* * *
მკურნალობას ხომ ვერ ვეტყვით რა სახის იქნება??

Posted by: texasuri jleta benzoxerxit 31 Jan 2009, 03:45
skoch
თრომბოფლებიტს გავს რასაც აღწერ.მკურნალობა კონსერვატიული(მედიკამენტოზური)და/ან ქირურგიული

Posted by: skoch 31 Jan 2009, 03:52
ანუ როარ ვინკურნალო ხელს სშემიშლის???/ლაზერული მკურნალობით ვერაა???

Posted by: ana_nushka 31 Jan 2009, 13:58
არ ვიცი ამ თემის შესაბამისი კითხვაა ათუ არა მაგრამ . . .

მაწუხებს თავის ტკივილები . . .
ვიყავი ექიმთან (ნევროლოგიურში) ექიმმა გამსინჯა ვერაფერს საგანგაშოს ვერ ვხედავოო ( ერთ წერტილში მტკიოდა თავი და რამე სერიოზილის მეშინოდა) . . .
დამინიშნა Milgamma- ვიტამინი B -ა თუ არ ვცდები და მსუბუქი ანტი დეპრესანტის სახით <<გრანდაკსინი>>

<<გრანდაკსინი>> -ს ინსტრუქცია არ მომეწონა. . . sad.gif ვითომ მსუბუქიაოო და უვნებელი მაგრამ უკუჩვენებებში უწერია დეპრესია და სუნთქვის უკმარისობა . . . ადრე მქონდა ნერვოზი და სწორედ დეპრესია, სუნთქვის უკმარისობა მაწუხებდა. . .(ეხლაც მაწუხებს იშვიათად. ჯერ კიდე ბოლომდე გასული არ მაქვს . . . cry.gif )

ვერ მივხვდი რა ვქნა??? დავლიო თუ ? ? ? იქნებ ვინმემ გამარკვიოთ უფრო დაწვრილებით ამ წამლის შესახებ? ? ? mo.gif

Posted by: ana_nushka 2 Feb 2009, 14:04
QUOTE
არ ვიცი ამ თემის შესაბამისი კითხვაა ათუ არა მაგრამ . . .

მაწუხებს თავის ტკივილები . . .
ვიყავი ექიმთან (ნევროლოგიურში) ექიმმა გამსინჯა ვერაფერს საგანგაშოს ვერ ვხედავოო ( ერთ წერტილში მტკიოდა თავი და რამე სერიოზილის მეშინოდა) . . .
დამინიშნა Milgamma- ვიტამინი B -ა თუ არ ვცდები და მსუბუქი ანტი დეპრესანტის სახით <<გრანდაკსინი>>

<<გრანდაკსინი>> -ს ინსტრუქცია არ მომეწონა. . .  ვითომ მსუბუქიაოო და უვნებელი მაგრამ უკუჩვენებებში უწერია დეპრესია და სუნთქვის უკმარისობა . . . ადრე მქონდა ნერვოზი და სწორედ დეპრესია, სუნთქვის უკმარისობა მაწუხებდა. . .(ეხლაც მაწუხებს იშვიათად. ჯერ კიდე ბოლომდე გასული არ მაქვს . . .  )

ვერ მივხვდი რა ვქნა??? დავლიო თუ ? ? ? იქნებ ვინმემ გამარკვიოთ უფრო დაწვრილებით ამ წამლის შესახებ? ? ? 



. . . . . . . . .. . . . .. . . . . . .. . . . mo.gif

Posted by: texasuri jleta benzoxerxit 2 Feb 2009, 15:14
ana_nushka
ჩემო კარგო. შენ სენესტო-იპოქონდრიული სინდროი აღგენიშნება.ეს არის დაავადებების შიში,გადაჭარბებული ყურადღება და შფითვა ჯანმრთელობის საკიტხების გარშემო,ექიმებისა და განსაკუთრებით წამლებისადმი უნდობლობა...და სხვა და სხვა უსიამოვნო სხეულებრივი შეგრძნებები,რომლებიც როგორც წესი ფუნქციური (არა-ორგანული)ხასიათისაა,მაგრამ ავადმყოფისთვის ძლიერ უსიამოვნოდ აღიქმება და ეძლევა დრამატული შინაარსი.

გრანდაქსინი არის უმსუბუქესი დღის ტრანქვილიზატორი და არა ანტიდეპრესანტი.(კსტაწი ანტიდეპრესანტი უფრო გჭირდება). მშვენიერი მედიკამენტია. ანოტაციების კიტხვას კი თავი დაანაბე. ამის ცოდნა ჩვენი საქმეა. მასე არც ერთი წამლის დალევა არ მოგინდება...ანალგინს უწერია გვერდით ეფექტებში თავის ტკივილი,დამამშვიდებელს-აგზნება...
ფარმაკოლოგიური ფირმა ვალდებულია,რომ თუ მილიონიდან ერთს მაინც აღენიშნა რამე თანამოვლენა,ჩაწეროს ანოტაციაში.
ამ თანამოვლენებიდან უმრავლესობას ისინი ან არ უვლინდება,ან პირველივე დღეებში ქრება...

ასე რომ მიირე გრანდაქსინი

Posted by: ana_nushka 2 Feb 2009, 15:32
texasuri jleta benzoxerxit



QUOTE
ჩემო კარგო. შენ სენესტო-იპოქონდრიული სინდროი აღგენიშნება.ეს არის დაავადებების შიში,გადაჭარბებული ყურადღება და შფითვა ჯანმრთელობის საკიტხების გარშემო,ექიმებისა და განსაკუთრებით წამლებისადმი უნდობლობა...და სხვა და სხვა უსიამოვნო სხეულებრივი შეგრძნებები,რომლებიც როგორც წესი ფუნქციური (არა-ორგანული)ხასიათისაა,მაგრამ ავადმყოფისთვის ძლიერ უსიამოვნოდ აღიქმება და ეძლევა დრამატული შინაარსი.


cry.gif მემგონი ვგიჟდები . . . biggrin.gif მასეთ რაღაცეებზე რომ ვფიქრობ . . . rolleyes.gif
მაგარამ წამლების ანოტაციას მას შემდეგ ვკითხულობ რაც ეგლონილი დავლიე( ექიმის დანიშნულებით) და უკუ ჩვენებები გამომივლინდა mo.gif

QUOTE
ასე რომ მიირე გრანდაქსინი


მადლობა დავიწყებ წამლის სმას . . . 2kiss.gif

Posted by: *hedgehog* 2 Feb 2009, 17:01
ვინმე კარგი სანდო თერაპევტი ან თირკმელები ექიმი მირჩიეთ რაააა

Posted by: _T_U_R_A_ 4 Feb 2009, 00:06
აუ ერთი ესეთი შეკითხვა მაქ. ტუალეტში მოსაქმების დროს დიდს თუ რაგაც ჯანდაბას რო ეძახიან (კულტურულად რომ ვთქვათ) და თან სისხლი რო მიყვება უკანა ხვრელიდან ეს მარტო კუჭის წყლულის დამახასიათებელი ნიშნებია და სხვა არაფრის? თუ კუჭის წყლულის გახეთქვის? თუ რის გამო შეიძლება მოხდეს ასე?

Posted by: texasuri jleta benzoxerxit 4 Feb 2009, 04:20
_T_U_R_A_
წყულიდან სისხლდენისას განავალი არის კუპრისფერი.თუ ფარული სისხლდენაა,ამას ანალიზი ადგენს...

განავალთან შერეული ალისფერი სისხლი ძირითადად არის ბუასილის,ანალური ნაპრალის ან სიმსივნის დროს.

Posted by: *hedgehog* 4 Feb 2009, 10:46
QUOTE
თერაპევტი ან თირკმელები ექიმი მირჩიეთ რაააა


Posted by: _T_U_R_A_ 4 Feb 2009, 13:55
QUOTE
კუპრისფერი.

ეს რა ფერია?
QUOTE
განავალთან შერეული ალისფერი სისხლი ძირითადად არის ბუასილის,ანალური ნაპრალის ან სიმსივნის დროს.

ექიმის გარესე ამის გარკვევა ვერ მოხერხდება?
* * *
ანუ მაგ რაგაცეებს სხვა დამახასიათებელი სიმპტომები რა აქ?

Posted by: texasuri jleta benzoxerxit 4 Feb 2009, 18:27
_T_U_R_A_
კუპრი არის ძალიან შავი...შავუ და ბზუნვარეა ამ დროს განავალი და ოდნავ თხელი.

რა თქმა უნდა ექიმის გარეშე დიაგნოზი ზოგადად არ ისმება...და არც მკურნალობა ტარდება boli.gif

Posted by: _T_U_R_A_ 4 Feb 2009, 19:10
texasuri jleta benzoxerxit
და მაგ რაგაცეებბს სხვა სმიპტომები არ აქვსს?? მარტო სისხლდენაა???
მაგალითად შარდის ფერი, ან რავი რამეე

Posted by: KABELI 4 Feb 2009, 19:19
კიტხვა მაქვს და თუ შეიძლება მიპსუხეთ, მაქვს დარღვეული ნივთიერებათა ცვლა. მირჩიეს დავლიო ნახევარ ჭიქაში გახსნილი რამდენიმე წვეთი პერეკისი,მითხრეს რო გააკეთებს დეზინფექციას და მოაგვარებს ნივთიერებათა ცვლასაც.
მე კიდევ ცოტა ვერ ვენდე,პერეკისი საშიში არ არის ორგანიზმისთვის?

Posted by: Impressive 4 Feb 2009, 19:27
სათესლე პარკებში, სათესლე ჯირკვლების გარდა კიდე რაღაცეებია სხვა რაღაცეები, ბრილიანტების მაგვარი smile.gif ანუ სათესლე ჯირკვლები არაა მარტო შიგნით, უფრო მარცხენაში, ისე ხელს არ მიშლის. და რა უნდა იყოს ?

Posted by: vano_t 5 Feb 2009, 02:41
KABELI
QUOTE
კიტხვა მაქვს და თუ შეიძლება მიპსუხეთ, მაქვს დარღვეული ნივთიერებათა ცვლა. მირჩიეს დავლიო ნახევარ ჭიქაში გახსნილი რამდენიმე წვეთი პერეკისი,მითხრეს რო გააკეთებს დეზინფექციას და მოაგვარებს ნივთიერებათა ცვლასაც.
მე კიდევ ცოტა ვერ ვენდე,პერეკისი საშიში არ არის ორგანიზმისთვის?
რას ნიშნავს ნივთიერებათა ცვლის დარღვევა და ვინ გირჩია პერეკისი დალიეო? პერეკისი იშლება საბოლოოდ წყლად და ჟანგბადის ატომად, რომელსაც გააჩნია ძლიერი მჟანგველი თვისებები და ძირითადად გამოიყენება როგორც ბაქტერიოციდულ (ანუ ბაქტერიების მკვლელი) ნაერთი. როგორ უნდა შეცვალოს ამან ნივთიერებათა ცვლა?

Impressive
QUOTE
სათესლე პარკებში, სათესლე ჯირკვლების გარდა კიდე რაღაცეებია სხვა რაღაცეები, ბრილიანტების მაგვარი smile.gif ანუ სათესლე ჯირკვლები არაა მარტო შიგნით, უფრო მარცხენაში, ისე ხელს არ მიშლის. და რა უნდა იყოს ?
სათესლე ჯირკვლების თავზე მდებარეობს ეპიდიდიმისი, რომელსაც მსგავი აღნაგობა აქვს, რაც შენ დაახასიათე. თუმცა, შენ რადგანაც არ იცი ეგ ყველაფერი, ჯობია ნორმალურ ექიმს გაასინჯებინო და გეტყვის გჭირდება თუ არა დამატებითი გამოკვლევები.

Posted by: unknown lady 5 Feb 2009, 14:18
vano_t


შეგიძლია ამიხსნა რა განსხვავებაა ბარალგეტასსა და ბარალგინს შორის

( ავერსმა შეუკვეთა ბარალგეტასი და ახლა ამას ასაღებენ თუ ეს ჯობია მართლა ვერ მივხვდი, თან ბარალგინი აღარ გამოდისო rolleyes.gif )

დავიბენი

Posted by: Goddess 5 Feb 2009, 15:01
vano_t
გამარჯობა smile.gif კითხვა მაქვს, მართლა დაბნეული ვარ და ეგებ დამეხმაროთ?
მოკლედ, ახალი წლიდან მოყოლებული ყურებში, ორივეში მაქვს გაბმული მოწუილო შუილი. დღის განმავლობაში იმდენად ვერ ვამჩნევ, უფრო ნერვების მომშლელია დილით და ღამით ძილის წინ, როცა სიწყნარეა გარშემო. ვიყავი მისული ექიმთან დაზღვევით. ოჯახის ექიმმა ზოგადად გამომკითხა, წნევა გამისინჯა (110/70) და გადამაგზავნა ყელ-ყურის ექიმთან. ყელ-ყურის ექიმმაც გამომკითხა, დამითვალიერა ყელი, ყურები, ცხვირი და ვერაფერი ვერ ნახა, ყველაფერი რიგზეაო და დიაგნოზის სავარაუდო გრაფაში ჩაწერა ნევროლოგიური სმენაჩლუნგობა (?) და გადამაგზავნა სმენის შესამოწმებლად.
სმენა შევიმოწმე მე-9-ე საავდმყოფოში. როდესაც ქალბატონმა, რომელიც მსინჯავდა, წინასწარ დიაგნოზს დახედა, გაბრაზდა, სმენის შემოწმების გარეშე, რატომ გაკადნიერდა და დაწერა რამეო. სმენა მქონდა 100%-იანი. ანუ მაგ მხრივაც არანაირი პრობლემა.
ახლა ის, რამაც ყველაზე მეტად დამაბნია.... ყურებზე პრობლემა არ მაქვს, მაგრამ აშკარად მაწუხებს და ცოტა ხანში ყურებს დავიჭრი ან ფანჯრიდან გადავხტები.... სხვა რა შეიძლება იყოს? რის ექიმთან სჯობს გადამოწმება (ასე პრაიზვოლნი პროგრამითაც არ მინდა ექიმებთან ბოდიალი, ვიცი მაგათი ამბავი, ათასი ჭირი შეიძლება გამომიგონონ და სულ სხვა, არარსებული რამეები მიმკურნალონ და მართლა არარსებული დამმართონ მერე).
კიდევ მეორე ფაქტორი, რამაც დამაბნია. მე-9-ში ქალმა გამომკითხა კისრის პრობლემა ხომ არ მაქვს, არადა მართლა მაწუხებს ამ ბოლო დროს, მატრასს ვაბრალებ...
მერე მითხრა, მაგი ოსტეოხონდროზი შეიძლება იყოს, მანაც იცის ეგეთი სიმპტომებიო (თავში გაფუჭებული ტელევიზორი). და მირჩია, რომ მივიდე ვიღაც ჰომეოპატთან, რომელიც მუშაობს დიღმის რკინიგზის საავადმყოფოში და რომელიც რაღაც ნემსების დახმარებით სვავს ზოგად დიაგნოზს სხეულის საერთო მდგომარეობაზე და იქ გამოჩნდება რეალურად რა გაწუხებსო....
ჰომეოპათიის კი მწამს, მაგრამ ჭეშმარიტი ჰომეოპათიის, აქ კი, ვურეკავ ამ ქალბატონს და მიხსნის როგორ მივიდე საავატყოფოში... აი ახლა კი მართლა აღარ ვიცი რა ვქნა....
გთხოვ, ეგებ დამეხმარო. აღნიშნული სიმპტომი კიდევ რამ შეიძლება გამოიწვიოს გარდა თავად ყურის პრობლემისა??

Posted by: unknown lady 5 Feb 2009, 15:23
Goddess


ნევროპათოლოგთან ყოფილხარ?

( იმიტომ გეკითხები, რომ ასეთი შემთხვევები ხანდახან ნევროლოგიურია)

Posted by: Goddess 5 Feb 2009, 15:39
unknown lady
QUOTE
იმიტომ გეკითხები, რომ ასეთი შემთხვევები ხანდახან ნევროლოგიურია

ნევროლოგიურში სავარუდოდ რა შეიძლება შედიოდეს??

მეშინია sad.gif

Posted by: unknown lady 5 Feb 2009, 16:21
Goddess
QUOTE
მეშინია



რაც შენ დაწერე ხშირად გამიგია ( ერთს დაემართა ყურის ანთების მერე, ნუ დარჩა რაა, ერთს ნერვიულობის ნიადაგზე )

ხოდა ნუ გეშინია, რაღაც არ გამიგია ცუდად დამთავრებულიყოს ...




vano_t მოვა და უფრო ზუსტ პასუხს გაგცემს smile.gif

Posted by: *hedgehog* 5 Feb 2009, 16:40
Goddess
ნუ გეშინია. სხვათაშორის ღამე რო ვწვები ყურებში რაღაც წუილი მეც მესმოდა და ნერვებს მიშლიდა. ნებისმიერ დროს, თუ სიჩუმეა და დავაკვირდები რა ხმა ხო არ ისმის ეგ წუილი სულ მესმის და მართლა ნერვებზე მოქმედებს თუ აყევი. მე ღამე მაგიტომ ტელევიზორს ვტოვებხოლმე ჩართულს, ან ტელევიზორთან ერთად ვიძინებ smile.gif თუ ყველაფერი წესრიგში გაქვს და მაგ წუილის მეტი არაფერი არ გაწუხებს, იქნებ ასესცადო, რამე სხვაზე გადაიტანო ყურადღება და .......


მე ვეძებ კარგ თერაპევტს ან თირკმელების სპეციალისტს, ექიმებმა ვინმე მირჩიეთ. ძალიან მტკივა, კრისტალები გაქვსო ეხოზე მითხრეს და ჩემი ოჯახის ექიმი კიდე მიმტკიცებს ეგ კრისტალები ყველას აქვს და ტკივილი ინდივიდუალურიაო. არადა ძალაინ დავიღალე თვეზე მეტია მტკივა + გულის რევის შეგრძნება მაქვს და მეშინია არ გავფშიკო ფეხები. ვინმე გულისხმიერი ექიმი მირჩიეთ რა smile.gif

Posted by: Goddess 5 Feb 2009, 18:48
unknown lady
*hedgehog*
ცოტა დავწყნარდი... არადა მართლა, ახალ წელს ავად ვიყავი და მას შემდეგ მაქვს (ყელი მტკიოდა ძალიან) და თან საშინელი თვე იყო დეკემბრის ბოლო იანვრის დასაწყისი.. პრაქტიკულად არ მძინებია იმდენი სანერვიულო მქონდა.... ალბათ მაგის ბრალია.
მართალია მეც სულ ფილმის ფონზე ვიძინებ, მაგრამ უკვე მაინც აღარ შემიძლია, მაგიჟებს.

*hedgehog*
QUOTE
ემი ოჯახის ექიმი კიდე მიმტკიცებს ეგ კრისტალები ყველას აქვს და ტკივილი ინდივიდუალურიაო

აი ოჯახის ექიმების მიმართ ძალიან ცუდი ნდობა მაქვს sad.gif სამწუხაროდ, მგონია, რომ ხშირად ღალატობენ პროფესიონალიზმს კომპანიის მიზნების გამო... მაგალითად გაგზავნიან მათ გავნო ექიმებთან და გზღუდავენ, რომ საუკეთესო სპეციალისტთან წახვიდე ვთქვათ. (ხშირ შემთხვევაში, თუ გათვითცნობიერებული არ ხარ, როგორ იურთიერთობო მათთან)
გავიკითხავ თირკმელებთან დაკავშირებით, თუმცა ვფიქრობ, გამოცდილი ინტერნისტები უკეთ დაგაკვალიანებენ.
ისე კი, მგონია ყველაზე გამოცდილები მაინც იქ არიან, არამიანცში რომ არის უროლოგიის განყოფილება, მგონი მეექვსე სართული??

Posted by: vano_t 5 Feb 2009, 22:26
unknown lady
QUOTE
შეგიძლია ამიხსნა რა განსხვავებაა ბარალგეტასსა და ბარალგინს შორის

( ავერსმა შეუკვეთა ბარალგეტასი და ახლა ამას ასაღებენ თუ ეს ჯობია მართლა ვერ მივხვდი, თან ბარალგინი აღარ გამოდისო  rolleyes.gif )

დავიბენი

ზუსტად ვერ გეტყვი რა განსხვავებაა. სწრაფი სერჩით ვნახე, რომ სხვადასხვა შემადგენლობა აქვთ და ორივე კომბინირებული (ანუ რამოდენიმი ნაერთის შემცველი) პრეპარატებია. აშშ-ში არ ხმარობენ ამ პრეპარატებს და ნაკლები ინფორმაცია გამაჩნია.

რაც შეეხება იმას რომელი ჯობია, ეს ყველაფერი სტატისტიკური ტერმინია. თუმცა მოცემულ პაციენტში სტატისტიკურად გაცილებით უკეთესმა ტკივილგამაყუჩებელმა შეიძლება გაიცლებით ნაკლები ეფექტი მოგცეს კერძო პაციენტში, ვიდრე სტატისტიკურად ნაკლებად ეფექტურმა ტკივილ გამაყუჩებელმა. იგივე ეხება გვერდით მოვლენებსაც.

Goddess
QUOTE
მოკლედ, ახალი წლიდან მოყოლებული ყურებში, ორივეში მაქვს გაბმული მოწუილო შუილი.
ამ პოსტს გაახედე (KingOfSorrow-ს გავეცი პასუხი) და თუ რაიმე კითხვა გექნება შეძლებისდაგვარად გიპასუხებ. მაგ სიმპტომს ქვია ტინიტუსი სამედიცინო ენაზე.
http://forum.ge/?f=43&showtopic=33759979&st=810#

Posted by: Goddess 5 Feb 2009, 22:30
vano_t
QUOTE
ამ პოსტს გაახედე (KingOfSorrow-ს გავეცი პასუხი) და თუ რაიმე კითხვა გექნება შეძლებისდაგვარად გიპასუხებ. მაგ სიმპტომს ქვია ტინიტუსი სამედიცინო ენაზე.

ეგ პოსტი წავიკითხე, მაგრამ საკმაო პასუხები ვერ მივიღე მაინც და იმიტომ იყო რომ ვიკითხე თავიდან. კარგი, გასაგებია, გმადლობ. მე რაც მაინტერესებდა უკვე წინა პოსტებში ყველაფერი დავწერე და აღარ გავმეორდები smile.gif

Posted by: unknown lady 6 Feb 2009, 18:20
vano_t
QUOTE
ზუსტად ვერ გეტყვი რა განსხვავებაა. სწრაფი სერჩით ვნახე, რომ სხვადასხვა შემადგენლობა აქვთ



გაიხარე, მეც ეს მაინტერესებდა ზუსტად ერთი და იგივე შემადგენლობა აქვთ თუ არა smile.gif

მადლობთ

Posted by: ikebana 6 Feb 2009, 21:16
vano_t
ერთი კითხვა მაქვს.
ბოლო ხანს შევნიშნე რომ ხელისგულები და მტევნის ზედაპირი მკვეთრად მოყვითალო შეფერილობის გამიხდა. უფრო ნარინჯისფერი ეთქმის ალბათ. არც ციტრუსებს ვიღებ დიდი რაოდენობით, კიარადა საერთოდ არ მიმიღია კაი ხანია. ასეთი რამ მქონდა 7 წლის წინაც, მაშინ სახეზეც გავყვითლდი. რა შეიძლება იყოს მიზეზი?

Posted by: ana_nushka 6 Feb 2009, 22:14
კითხვა ექიმებს !!!! smile.gif

ხომ არ იცით სად აკეთებენ თავის ტომოგრამას??? (ჩვეულებრივი ტომოგრამა მინდა, რაღაც მაგნიტურის და რაღაცეების გარეშე biggrin.gif )

და რა ფასებია??? chups.gif

Posted by: ikebana 6 Feb 2009, 22:35
ana_nushka
კომპიუტერულ ტომოგრამას თუ გულისხმობ, მაგალითად თოდუას ცენტრში. 300 ლარამდე.

Posted by: vano_t 6 Feb 2009, 22:45
ikebana
QUOTE
ერთი კითხვა მაქვს.
ბოლო ხანს შევნიშნე რომ ხელისგულები და მტევნის ზედაპირი მკვეთრად მოყვითალო შეფერილობის გამიხდა. უფრო ნარინჯისფერი ეთქმის ალბათ. არც ციტრუსებს ვიღებ დიდი რაოდენობით, კიარადა საერთოდ არ მიმიღია კაი ხანია. ასეთი რამ მქონდა 7 წლის წინაც, მაშინ სახეზეც გავყვითლდი. რა შეიძლება იყოს მიზეზი?

შეიძლება და ნორმალური პიგმენტაციაა. ყველაფერს პასუხს მაინც ვერ გასცემ. მთავარია, სერიოზული პათოლოგია არ გამოიპაროს. როგორც წესი, თუ სერიოზულია რამიე, მაშინ სხვა სიმპტომებიც უნდა გქონდეს. თავს თუ სხვანაირად კარგად გრძნობ და არაფერი გაწუხებს, ალბათ სანერვიულოც არაფერია. ექიმს შეგიძლია ეჩვენო და ყოველ შემთხვევისათვის ლაბორატორიები გადაამოწმებინო. ამაში შევა ბილირუბინიც. მომატებულიც რომ იყოს ცოტათ ბილირუბინი, არაფერს არ ნიშვავს, იმიტომ რომ ყველაზე ხშირი მიზეზი მცირედი არაპირდაპირი ბილირუბინემიის არის გილბერტის სინდრომი და ისედაც უწყანარი პრობლემაა ეს.

Posted by: ikebana 6 Feb 2009, 23:03
vano_t
დიდი მადლობა პასუხისთვის.
ბილირუბინის გარდა რა შეიძლება შევამოწმო? ღვიძლის და ნაღვლის ბუშტის ექოს ვაპირებ. ალბათ პანკრეასიც ხო?

Posted by: ana_nushka 7 Feb 2009, 00:02
ikebana
QUOTE
კომპიუტერულ ტომოგრამას თუ გულისხმობ, მაგალითად თოდუას ცენტრში. 300 ლარამდე.


mo.gif რა ამბავიაა . . .

მეც ვიცი მე-9 საავადმყოფოში 180 ლარი ღირს, მაგრამ ექიმმა მითხრა თუ ამაზე იაფად ნახავ გაიკეთეო. მაგას არ აქვსო მნიშვნელობა სად გაიკეთებო და იავიცი იავიცი . . . mo.gif

Posted by: vano_t 7 Feb 2009, 01:54
ikebana
QUOTE
ბილირუბინის გარდა რა შეიძლება შევამოწმო? ღვიძლის და ნაღვლის ბუშტის ექოს ვაპირებ. ალბათ პანკრეასიც ხო?

იმიჯინგი საერთოდ არ არის საჭირო. ჯერ ლაბორატორია შეგიძლია შეამოწმო მხოლოდ. თუ ბილირუბინი და ამინოტრანსფერაზებ ნორმაშია, მაშინ აღარფერი აღარ დაგჭირდება. თუ ლაბორატორია მკვეთრად ნორმიდან გადახრაა, მაშინ დაგჭირდება ღვიძლის/ნაღვლის ბუშტის გამოკვლევა. პანკრეასს საერთოდ იშვიათად ამოწმებენ (თუ პანკრეატიტზე ან პანკრეასის სიმსივნეებზე-ან სხვა რაიმე სპეციფიურ დიაგნოზზე- არ არის ეჭვი, არ აქვს პანკრეასის შემწმებას აზრი ალბათ)

Posted by: shanyva 7 Feb 2009, 16:37
vano_t
ხახის ბაქტერიული გამოკვლევის შედეგად ამოითესა უხვი ზრდით staphylococcu epidermidis (heamolytikus). მცირე ზრდით streptococcus pneumoniae.
ფაგზე მგრძნობელობა აქვს +3 და სხვადასხვა ანტიბიოტიკებზე +4(პენიცილინი,ამპიოქსი, ცეფაზოლინი, ზინაცეფი, კლაფორანი, როტაცეფი, და სხვ.)
რამდენად საშიშია ახლა ჩემთვის, რადგან ვგეგმავ დაორსულებას....
გამდლობ წინასწარ.... smile.gif

Posted by: vano_t 8 Feb 2009, 00:50
shanyva
QUOTE
ხახის ბაქტერიული გამოკვლევის შედეგად ამოითესა უხვი ზრდით staphylococcu epidermidis  (heamolytikus). მცირე ზრდით streptococcus pneumoniae.
ფაგზე მგრძნობელობა აქვს +3 და სხვადასხვა ანტიბიოტიკებზე +4(პენიცილინი,ამპიოქსი, ცეფაზოლინი, ზინაცეფი, კლაფორანი, როტაცეფი, და სხვ.)
რამდენად საშიშია ახლა ჩემთვის, რადგან ვგეგმავ დაორსულებას....

რატომ გაგიკეთეს საერთოდ ხახიდან ნაცხი?

მეორეც, ის რა ბაქტერიებიც ამოითესა, ნორმალური ფლორაა და მაგას არავინ მკურნალობს. staphylococcu epidermidis უფრო ხშირად კონტამინაციაა (ანუ კანიდან ხდება ნაცხების და სხვადასხვა კულტურების დაბინძურება მაგ ბაქტერიით) და არავითარი მკურნალობა ამას არ ჭირდება. ამ ბაქტერიით გამოწვეული ინფექციები არის მხოლოდ იმპლანტირებული ნაწილების (მაგალითად გულის სარქველის, ან სხვადასხვა ხელოვნური სახსრის) ინფექციები და სხვა შემთხვევაში არავითარი ღირებულება არ გააჩნია.

streptococcus pneumoniae (ანუ პნევმოკოკი) კიდევ ადამიანების 30-50 % ისედაც აქვს ნორმაში ხახის არეში. პნევმოკოკი ფარინგიტებს არ იწვევს და არაფერი მკურნალობა არ ჭირდება როცა მაგ არედან ამოთესავენ.

ნაყოფისათვის არ არის არაფერი საშიში (შეიძლება მუცლის მოშლის ცოტა რისკი იყოს). ასე რომ იყოს, მაშინ დედების 40 % (რომელიც ამ ბაქტერიის მატარებლები არიან) საერთოდ უნდა ერიდებოდნენ დაორსულებას. ახალშობილებისათვის გარკვეული რისკი არის, რომ პნევმოკოკმა მენინგიტი გამოიწვიოს. მაგრამ ამის შიშით დაორსულება იგივეა, რომ გარეთ არ გახვიდე ადამიანმა, იმიტომ რომ გარკვეული რისკი არსებობს, რომ მანქანა დაგვარტყამს.

Posted by: texasuri jleta benzoxerxit 8 Feb 2009, 01:13
QUOTE (vano_t @ 8 Feb 2009, 00:50 )
shanyva
QUOTE
ხახის ბაქტერიული გამოკვლევის შედეგად ამოითესა უხვი ზრდით staphylococcu epidermidis  (heamolytikus). მცირე ზრდით streptococcus pneumoniae.
ფაგზე მგრძნობელობა აქვს +3 და სხვადასხვა ანტიბიოტიკებზე +4(პენიცილინი,ამპიოქსი, ცეფაზოლინი, ზინაცეფი, კლაფორანი, როტაცეფი, და სხვ.)
რამდენად საშიშია ახლა ჩემთვის, რადგან ვგეგმავ დაორსულებას....

რატომ გაგიკეთეს საერთოდ ხახიდან ნაცხი?

მეორეც, ის რა ბაქტერიებიც ამოითესა, ნორმალური ფლორაა და მაგას არავინ მკურნალობს. staphylococcu epidermidis უფრო ხშირად კონტამინაციაა (ანუ კანიდან ხდება ნაცხების და სხვადასხვა კულტურების დაბინძურება მაგ ბაქტერიით) და არავითარი მკურნალობა ამას არ ჭირდება. ამ ბაქტერიით გამოწვეული ინფექციები არის მხოლოდ იმპლანტირებული ნაწილების (მაგალითად გულის სარქველის, ან სხვადასხვა ხელოვნური სახსრის) ინფექციები და სხვა შემთხვევაში არავითარი ღირებულება არ გააჩნია.

streptococcus pneumoniae (ანუ პნევმოკოკი) კიდევ ადამიანების 30-50 % ისედაც აქვს ნორმაში ხახის არეში. პნევმოკოკი ფარინგიტებს არ იწვევს და არაფერი მკურნალობა არ ჭირდება როცა მაგ არედან ამოთესავენ.

ნაყოფისათვის არ არის არაფერი საშიში (შეიძლება მუცლის მოშლის ცოტა რისკი იყოს). ასე რომ იყოს, მაშინ დედების 40 % (რომელიც ამ ბაქტერიის მატარებლები არიან) საერთოდ უნდა ერიდებოდნენ დაორსულებას. ახალშობილებისათვის გარკვეული რისკი არის, რომ პნევმოკოკმა მენინგიტი გამოიწვიოს. მაგრამ ამის შიშით დაორსულება იგივეა, რომ გარეთ არ გახვიდე ადამიანმა, იმიტომ რომ გარკვეული რისკი არსებობს, რომ მანქანა დაგვარტყამს.

სრული ჭეშმარიტებაა...
პირ-ხახიდან ერთი ზვიგენი და ნიანგი არ ამოითესება,თორემ დანარჩენს ყველაფერს ნახავ yes.gif

Posted by: aranormaluri 8 Feb 2009, 18:07
მე მაინტერესებს ნაღვლს ბუშტის მკურნალობა როგორ ხდება? ექვემდებარება მკურნალობას თუ უნდა ამოიკვეთოს თუ ადამიანს აწუხებს?

Posted by: tete- 11 Feb 2009, 03:06
ხვრინვაზე ყოველთვის მაინტერესებდა განკურნებადია თუ არა და მხოლოდ კაცები რატო ვრინავენ(ნუ ქალები იშვიათად)?biggrin.gif

Posted by: Goddess 11 Feb 2009, 11:36
მმ, ეგებ მირჩიოთ კარგი ნევროლოგი? აი ისეთი, ფანტომური დაავადებისათვის რომ არ გამჭყიპავს წამლებით.

Posted by: Solveig 11 Feb 2009, 19:53
aranormaluri
QUOTE
ნაღვლს ბუშტის მკურნალობა როგორ ხდება? ექვემდებარება მკურნალობას თუ უნდა ამოიკვეთოს თუ ადამიანს აწუხებს?

მაგაზე ვერავინ გიპასუხებს, თუ არ იტყვი-კონკრეტულად რა პრობლემაა smile.gif

Posted by: aranormaluri 11 Feb 2009, 20:12
Solveig იცი რა მჭირს? პრობლემა არ მაქვს ნაღვლის ბუშტზე მაგრამ ამეკვიატა რომ არ არის წესრიგში. ხოდა პარანოიაზე ვიმკურნალო თუ ნაღვლის ბუშტზე გავესინჯო აღარ ვიცი.
biggrin.gif
მაინც მინდა გამოკვლევა ჩავიტარო. რომელ ექიმს მივმართო და სად ჯობია მისვლა?
თან იმისაც მეშინია მართლა რაღაც რო აღმომაჩნდეს და საოპერაციო გავხდე. biggrin.gif

Posted by: texasuri jleta benzoxerxit 12 Feb 2009, 01:28
QUOTE (aranormaluri @ 11 Feb 2009, 20:12 )
Solveig იცი რა მჭირს? პრობლემა არ მაქვს ნაღვლის ბუშტზე მაგრამ ამეკვიატა რომ არ არის წესრიგში. ხოდა პარანოიაზე ვიმკურნალო თუ ნაღვლის ბუშტზე გავესინჯო აღარ ვიცი.
biggrin.gif
მაინც მინდა გამოკვლევა ჩავიტარო. რომელ ექიმს მივმართო და სად ჯობია მისვლა?
თან იმისაც მეშინია მართლა რაღაც რო აღმომაჩნდეს და საოპერაციო გავხდე. biggrin.gif

სენესტო-იპოქონდრია ქვია მაგ მდგომარეობას (თუ ნაღვლის ბუშტის ორგანული პატოლოგია გამოირიცხა)

ასე რომ ფსიქიატრთან!

Posted by: merry 12 Feb 2009, 04:50
არ ჩამქოლოთ! ლაზერული ეპილაციის უკუჩვენებები მაინტერესებს user.gif gigi.gif

Posted by: aranormaluri 12 Feb 2009, 13:05
QUOTE
ასე რომ ფსიქიატრთან


biggrin.gif უიმეეე.
ჯერ ნაღველზე მივიდე რა მოხდება.
ადრე აკვიატებული მქონდა რომ ჩიყვი მქონდა. მერე თავისით გადამიარა.

Posted by: addicted 12 Feb 2009, 13:59
aranormaluri

ჯერ ექოსკოპია გაიკეთე, ოღონდ კარგ კლინიკაში, და იქ გეტყვიან რასა იქმს შენი ნაღვლის ბუშტი smile.gif ხო პროფილაქტიკის მიზნით რა smile.gif

Posted by: lizofobi 12 Feb 2009, 16:09
იცით რა მაინტერესებს? არის თუ არა კავშირი ალერგიასა და კისერზე ლიმფური ჯირკვლების გადიდებას შორის? ან საერთოდ, რატომ დიდდება (სულ ოდნავ) და მერე ისევ პატარავდება ლიმფური ჯირკვლები? როცა რამე სიცოცხლისთვის საშიში ინფექციაა ორგანიზმში მაშინ ეს ჯირკვლები უნდა მოძრაობდნენ თუ არა?

Posted by: aranormaluri 12 Feb 2009, 19:08
addicted
კარგი biggrin.gif ეგ მინდოდა გამეგო.
დიდი დიდი მადლობა.

Posted by: shtori 14 Feb 2009, 22:01
იამს რა ჰქვია, რომ არ გინდა და მაინც ჭამ?
აი ახლა ნიგოზსს ვჭამ სულ ძალით, არც არავის დაუძალებია, გემოც საშინლად მუგემურება. მოკლედ საზიზღრობაა და მაინც ვჭამ. დილითაც სულ ძალით ვასაუზმე ჩემი თავი. გუშინ კიდე შოკოლადი....
მოვკვდები თუ მოვრჩები?

Posted by: Solveig 14 Feb 2009, 22:54
shtori
QUOTE
იამს რა ჰქვია, რომ არ გინდა და მაინც ჭამ?
აი ახლა ნიგოზსს ვჭამ სულ ძალით, არც არავის დაუძალებია, გემოც საშინლად მუგემურება. მოკლედ საზიზღრობაა და მაინც ვჭამ. დილითაც სულ ძალით ვასაუზმე ჩემი თავი. გუშინ კიდე შოკოლადი....

მგონი, უკეთესი იქნება, თუ ფსიქოთერაპევტს მიაკითხავ.
განსაკუთრებით ამ ფრაზის გამო
QUOTE
მოვკვდები თუ მოვრჩები?


Posted by: Tornike Alashvili 14 Feb 2009, 23:48
QUOTE (shtori @ 14 Feb 2009, 22:01 )
იამს რა ჰქვია, რომ არ გინდა და მაინც ჭამ?
აი ახლა ნიგოზსს ვჭამ სულ ძალით, არც არავის დაუძალებია, გემოც საშინლად მუგემურება. მოკლედ საზიზღრობაა და მაინც ვჭამ. დილითაც სულ ძალით ვასაუზმე ჩემი თავი. გუშინ კიდე შოკოლადი....
მოვკვდები თუ მოვრჩები?

საერთოდ რაც არ მოგწონს ის გიტაცებს ხშირად
რისიც გეშინია –იმისკენ მიისწრაფი და ზოგჯერ გსიამოვნებს კიდეც

ვისაც სიმაღლის ეშინოდა–პარაშუტიზმი უდიდეს სიამოვნებას ანიჭებს –ვინაიდან უამრავი ადრენალინი გამოიყოფა და ას კი ნარკომანიის ტოლფასია

შიში სიყვარულს ბადებს–ო
რაც გაგიჭირდეს –იმას დაეუფლები–ო

მოკლედ რაც საწამლავია იგივე –წამალია
მთავარია არ გადააჭარბო

Posted by: shtori 15 Feb 2009, 00:02
Solveig
QUOTE
მგონი, უკეთესი იქნება, თუ ფსიქოთერაპევტს მიაკითხავ.

ხო. მართალი ხარ. ალბათ უფრო ფსიქოლოგიურია.. წეღან არც დავუკვირდი

QUOTE
განსაკუთრებით ამ ფრაზის გამო

QUOTE
მოვკვდები თუ მოვრჩები?

ეს ფრაზა კი ისე მინაწერი, არასერიოზული კითხვაა smile.gif
* * *
Tornike Alashvili
QUOTE
საერთოდ რაც არ მოგწონს ის გიტაცებს ხშირად
რისიც გეშინია –იმისკენ მიისწრაფი და ზოგჯერ გსიამოვნებს კიდეც

სასიამოვნო არაა...
smile.gif biggrin.gif

Posted by: vano_t 15 Feb 2009, 05:19
merry
QUOTE
არ ჩამქოლოთ! ლაზერული ეპილაციის უკუჩვენებები მაინტერესებს user.gif gigi.gif

მაგის უკუშცნებებებათ ითვლება:

1) ორსულობა (ლაზერის გავლენა ჯერ-ჯერობით კარგად არ არის შესწავლილი ნაყოფზე);

2) ჰერპესვირუსული ინფექციის გააქტიურება თუ რამოდენიმე დღის წინ ქონდა ავადმყოფს და ახლახან ჩაცხრა (ლაზერმა შეიძლება გამოიწვიოს ვირუსის ხელახალი აქტვიაცია და ისევ გამოყრა);

3) კანის ადგილობრივი დაავადებები: განსაკუთრებით ჭრილობა/ინფექცია;

4) ფოტოსენტისიტიურობა (ანუ სინათლისადმი "ალერგია");

5) ალბინიზმი-უბრალოდ არ მუშუაობს ამ დროს ლაზერი, რადგანაც ლაზერის შტანთქმა პიგმენტის შემცველი კანის მიერ ხდება;

6) ნევროლოგიური დაავადებები უნდა გამოკვლეული იქნას ლაზერით მკურნალობამდე;

7) დიაბეტი, ლუპუსი და ეპილეფსია;

8) თუ ხალი არის დასხივები არეში, მაშინ ეს ხალი ჯერ უნდა მოიკვეთოს;

9) თუ მზის აბაზანები ახალი მიღებული აქვს პაციენტს და კანის ფერი ნორმას არ დაბრუნებია, მაშინ უნდა დაელოდო კანის ფერის ნორმაში დაბრუნებას;

lizofobi
QUOTE
იცით რა მაინტერესებს? არის თუ არა კავშირი ალერგიასა და კისერზე ლიმფური ჯირკვლების გადიდებას შორის? ან საერთოდ, რატომ დიდდება (სულ ოდნავ) და მერე ისევ პატარავდება ლიმფური ჯირკვლები? როცა რამე სიცოცხლისთვის საშიში ინფექციაა ორგანიზმში მაშინ ეს ჯირკვლები უნდა მოძრაობდნენ თუ არა?
ლიმფური ჯირკვალი არის იმუნიტეტის ორგანო. თუ ორგანიზმში რაიმე უცხო სხეული შეიჭრება, ამ უცხო სხეულის ნეიტრალიზაცია ხდება მისი საწინააღმდეგო ანტისხეულებით და შემდეგ ლიმფური სისხლძარღვებით ასე ნეიტრალიზებული უცხო სხეული (რაც შეიძლება იყოს ნებისმიერი რამ) ხვდება ლიმფურ კვანძებში და მიმდინარეობს ანთების პროცესი, რაც იწვევს ლიმფური კვანძის გადიდებას. ალერგიამაც (ალერგენებმა, რაც უცხო სხეულია) შეიძლება მოგცეს ლიმფური კვანძის გადიდება. კვანძის მოძრაობა/არმოძრაობა არ განსაზღვრავს ინფექციის სერიოზულობას. ინფექციის სერიოზულობა დამოკიდებულია ავადმყოფის სიმპტომებზე: განნსაკუთრებით ისეთი სიმპტომების არსებობაზე, რომელიც სეფსისს ("სისხლის მოწამვლას") მიუთეთებს. ეს სიმპტომები და ნიშნებია წნევის ვარდნა, გულისცემის სიხშირის გაზრდა, სუნთქვის უკმარისობა, ძალიან მაღალი სიცხეები და სხვადასხვა ორგანოების უკმარისობა.

Te0
QUOTE
ხვრინვაზე ყოველთვის მაინტერესებდა განკურნებადია თუ არა და მხოლოდ კაცები რატო ვრინავენ(ნუ ქალები იშვიათად)?
ქალები არ ხვრინავენ იშვიათად. ქალებშიც ძალიან ხშირია ხვრინვა; ხვრინვას აქვს სხვადასხვა მიზეზი და მიზეზის მიხედვით ზოგი განკურნებადია და ზოგი არა. თუ მიზეზი არის რაიმე ქირურგიულად მოცილებადი დაბრკოლება (მაგალითად პოლიპები, ცხვირის ტრამვის შედეგად ცხვირის დეფორმაცია), მაშინ განკურნებადი შეიძლება იყოს. თუ ხვრინვის მიზეზია ე.წ. ძილის ობსტრუქციული აპნეა (obstructive sleep apnea), მაშინ სპეციალური აპარატი ჭირდებათ ხოლმე ძილის დროს.

Posted by: shanyva 15 Feb 2009, 15:46
vano_t
texasuri jleta benzoxerxit

აბორტის შემდეგ სისიხლიანი განავალი აბორტთან არის კავშირში?თუ სწორი ნაწლავის პრობლემაა?აბორტი გაკეთდა ორი დღის უკან,ადრე სწორ ნაწლავთან არანაირი პრობლემა არა ჰქონდა,უთხრეს თითქოს ორგანიზმი იწმინდებაო,მაგრამ სწორი ნაწლავიდან?
მადლობა წინასწარ

Posted by: tete- 15 Feb 2009, 18:09
vano_t ვა საინტერესოაა.. მადლობ) ..

Posted by: Solveig 15 Feb 2009, 20:37
ეს
QUOTE
საერთოდ რაც არ მოგწონს ის გიტაცებს ხშირად

და ეს
QUOTE
რისიც გეშინია –იმისკენ მიისწრაფი და ზოგჯერ გსიამოვნებს კიდეც

სრულიად სხვადასხვა რამ არის და არც განზოგადებას ექვემდევარება..ზოგს-როგორ აქვს-ზოგს კიდევ-როგორ.

და კონკრეტულ შემთხვევასთან საერთოდ არაა შუაში, ჩემი აზრით.

QUOTE
მოკლედ რაც საწამლავია იგივე –წამალია
მთავარია არ გადააჭარბო

უკვე გასაგებია, საითკენაც უბერავს ქარი.


QUOTE
შიში სიყვარულს ბადებს–ო


War is Peace
Freedom is Slavery
Ignorance is Strength

იგივე ლოგიკაა.

ოფტოპიკისათვის ბოდიშს ვიხდი

Posted by: texasuri jleta benzoxerxit 15 Feb 2009, 22:45
QUOTE (shanyva @ 15 Feb 2009, 15:46 )
vano_t
texasuri jleta benzoxerxit

აბორტის შემდეგ სისიხლიანი განავალი აბორტთან არის კავშირში?თუ სწორი ნაწლავის პრობლემაა?აბორტი გაკეთდა ორი დღის უკან,ადრე სწორ ნაწლავთან არანაირი პრობლემა არა ჰქონდა,უთხრეს თითქოს ორგანიზმი იწმინდებაო,მაგრამ სწორი ნაწლავიდან?
მადლობა წინასწარ

givi.gif "იწმინდება" ნოუ რა givi.gif

საშოდან სისლდენის შემთხვევაში ისე შეიძლება ,ქალის ანატომირი თავისებურებების გამო სისხლმა შორისის და ანუსისკენ გაჟონოს და თეორიულად შეიძლბა შეერიოს განავას,მაგრამ ამდენ მკითხაობას ექიმ-გინეკოლოგს და პროქტოოგს რომ მიაკითხოს არა?

გულწრფელ,ბოროტ და რა თქმა უნდა არასწორ სიხარულს განვიცდი,როდესაც ადამიანს აბორსის სემდეგ რაიმე გართულება ემართება. ასე,რომ შენმან პოსტმა, ვაღიარებ,ეს არ არის,სწორი,ძალიან მასიამოვნა givi.gif

Posted by: chorven:) 15 Feb 2009, 22:51
თერაპევტი არ ვიცი, მაგრამ დამუხტული ვარ სასწაულად, დენი მარტყამს, თმას ვერ ვიშლი, მეკრობა, რა ვქნა
ადრე კი მქონდა ბიოდენები, მაგრამ ასე ცუდად არ ვიყავი

Posted by: merry 16 Feb 2009, 01:05
vano_t
დიდი მადლობა smile.gif ლაქტაცია შედის უკუჩვენებებში? და თუ შედის რატომ user.gif baby.gif

Posted by: shanyva 16 Feb 2009, 01:15
texasuri jleta benzoxerxit
აბორტი სამწუხაროდ სავალდებულო იყო,მოეშალა და გამოფხიკეს sad.gif ამიტომ ნამდვილად ბოროტულად გამოგივიდა
მეც ძალაინ მეეჭვება ეგ შეხედულება რომ იწმინდება,თან რაიონში ხდება ეგ ყველაფერიიიიი

Posted by: mtvareuli 16 Feb 2009, 02:48
texasuri jleta benzoxerxit

ვორნი


chorven:)
QUOTE
დამუხტული ვარ სასწაულად, დენი მარტყამს, თმას ვერ ვიშლი, მეკრობა, რა ვქნა ადრე კი მქონდა ბიოდენები, მაგრამ ასე ცუდად არ ვიყავი

http://forum.ge/?showtopic=33820267&hl=%E1%83%93%E1%83%94%E1%83%9C%E1%83%98

Posted by: vano_t 16 Feb 2009, 03:22
shanyva
QUOTE
აბორტის შემდეგ სისიხლიანი განავალი აბორტთან არის კავშირში?თუ სწორი ნაწლავის პრობლემაა?აბორტი გაკეთდა ორი დღის უკან,ადრე სწორ ნაწლავთან არანაირი პრობლემა არა ჰქონდა,უთხრეს თითქოს ორგანიზმი იწმინდებაო,მაგრამ სწორი ნაწლავიდან?
მადლობა წინასწარ

აბორტს უამრავი გართულება აქვს. მათ შორის არის საშვილოსნოს ყელის ან საშვილოსნოს პერფორაცია (ანუ ამ ორგანოების გახვრეტა). ამ ორგანოების უკან სწორი ნაწლავია და შესაძლებელია სწორი ნაწლავის ასე დაზიანება. ამიტომ, თუ სისხლდენა მართლა დაიწყო სწორი ნაწლავიდან, აუცილებლად ჭირდება ავადმყოფს სულ მცირე სიგმოიდოსკოპია (უკეთესია კოლონოსკოპია), რათა სისხლდენის მიზეზი დადგინდეს და ასევე, გინეკოლოგმა უნდა ხელახლა შეამოწმოს ყველიც და საშვილოსნოც, რათა პერფორაცია და სხვა გართულებები გამორიცხოს.

merry
QUOTE
ლაქტაცია შედის უკუჩვენებებში?
არა.

Posted by: Avastin 16 Feb 2009, 20:58
QUOTE (texasuri jleta benzoxerxit @ 15 Feb 2009, 22:45 )
QUOTE (shanyva @ 15 Feb 2009, 15:46 )
vano_t         
texasuri jleta benzoxerxit

აბორტის შემდეგ სისიხლიანი განავალი აბორტთან არის კავშირში?თუ სწორი ნაწლავის პრობლემაა?აბორტი გაკეთდა ორი დღის უკან,ადრე სწორ ნაწლავთან არანაირი პრობლემა არა ჰქონდა,უთხრეს თითქოს ორგანიზმი იწმინდებაო,მაგრამ სწორი ნაწლავიდან?
მადლობა წინასწარ

givi.gif "იწმინდება" ნოუ რა givi.gif

საშოდან სისლდენის შემთხვევაში ისე შეიძლება ,ქალის ანატომირი თავისებურებების გამო სისხლმა შორისის და ანუსისკენ გაჟონოს და თეორიულად შეიძლბა შეერიოს განავას,მაგრამ ამდენ მკითხაობას ექიმ-გინეკოლოგს და პროქტოოგს რომ მიაკითხოს არა?

გულწრფელ,ბოროტ და რა თქმა უნდა არასწორ სიხარულს განვიცდი,როდესაც ადამიანს აბორსის სემდეგ რაიმე გართულება ემართება. ასე,რომ შენმან პოსტმა, ვაღიარებ,ეს არ არის,სწორი,ძალიან მასიამოვნა givi.gif

აბორტის შემდეგ სისიხლიანი განავალი აბორტთან არის კავშირში?

ყველა შემთხვევაში საშვილოსნოს პერფორაცია და სწორი ნაწლავის დაზიანება უნდა იქნას გამორიცხული!!!
სასზრაფოდ გინეკოლოგთან, ოღონდ იმასთან არა, ვინც უთხრა ორგანიზმი იწმინდებაო... gigi.gif

[I]გულწრფელ,ბოროტ და რა თქმა უნდა არასწორ სიხარულს განვიცდი,როდესაც ადამიანს აბორსის სემდეგ რაიმე გართულება ემართება.

რატო? 2kiss.gif
იმ ვიღაცას თავისი გაჭირვება ეყოფა და გართულებების გარეშეც ცუდ დგეში იქნება, მიუხედავად იმისა, გააცნობიერებს თუ არა ჩადენილს...
რაც შეეხება იმ გოგონას, რომელმაც "აბორტი" გაიკეთა, თუ ეს ნამდვილად ინდიცირებული აბორტი იყო (Abortus incompletus, missed Abortion და ა.შ) და Abruptio არა, ძალიან ტყუილად განჰკიცხავ და ცოდვაში იგდებ თავს...
wink.gif wink.gif

Posted by: texasuri jleta benzoxerxit 17 Feb 2009, 00:28

mtvareuli
Avastin
კარგით რა ხალხნო...კაკიე სტრასწი


იუმორის დეფიციტის შემტხვევაშიც უნდა რიგდებოდეს ვორნები,მიუხედავა დ იმისა,ეს დეფიციტი მწვავეა,ქვემწვავე,ქრონიკული აქტიური,თუ ქრონიკული პერსისტიული... biggrin.gif

Posted by: Avastin 17 Feb 2009, 02:01
QUOTE (texasuri jleta benzoxerxit @ 17 Feb 2009, 00:28 )
mtvareuli
Avastin
კარგით რა ხალხნო...კაკიე სტრასწი


იუმორის დეფიციტის შემტხვევაშიც უნდა რიგდებოდეს ვორნები,მიუხედავა დ იმისა,ეს დეფიციტი მწვავეა,ქვემწვავე,ქრონიკული აქტიური,თუ ქრონიკული პერსისტიული... biggrin.gif

იუმორი ჩემი მოგონილია, გენც... ასე, რომ ნე სტოიტ! keh.gif

Posted by: texasuri jleta benzoxerxit 17 Feb 2009, 02:16
QUOTE (Avastin @ 17 Feb 2009, 02:01 )
QUOTE (texasuri jleta benzoxerxit @ 17 Feb 2009, 00:28 )
mtvareuli
Avastin
კარგით რა ხალხნო...კაკიე სტრასწი


იუმორის დეფიციტის შემტხვევაშიც უნდა რიგდებოდეს ვორნები,მიუხედავა დ იმისა,ეს დეფიციტი მწვავეა,ქვემწვავე,ქრონიკული აქტიური,თუ ქრონიკული პერსისტიული... biggrin.gif

იუმორი ჩემი მოგონილია, გენც... ასე, რომ ნე სტოიტ! keh.gif

რატომ ბრაზობ ავასტინ? 2kiss.gif ისე რამდენი ვიღაცის მოგონილია კაცო ეს იუმორი,ვერა და ვერ მივაკვლიე პირველწყაროს...

მე ჩვენი ცის მნათობთა სადარი მოდერი ვიგულისხმე უპირატესად... givi.gif
იმასაც ვაკოცებ,რომ არ გამიბრაზდეს 2kiss.gif

ანდა გამიბრაზდეს რა...ტაკ დაჟე ინწერესნეიე

Posted by: button nose 17 Feb 2009, 10:33
კითხვა მაქვს.უკვე დაახლოებით ერთი თვეა მარცხენა თირკმელი მტკივა.ადრეც მქონდა ასეთი ტკივილი ოღონდაც დილაობით როდესაც შარდის ბუშტი სავსე მქონდა,შემდეგ როცა ვშარდავდი მივლიდა,ახლა მთელი დღის განმავლობაში მაწუხებს. ვერ ვიტყვი რომ ძალიან მტკივა მაგრამ ხშირად მტკივა და ამიტომ ვწუხდები,რა უნდა იყოს და თუ შეგიძლიათ ასე ზეპირად დამეხმაროთ.წინასწარ მადლობტ

Posted by: vano_t 17 Feb 2009, 10:43
button nose
QUOTE
კითხვა მაქვს.უკვე დაახლოებით ერთი თვეა მარცხენა თირკმელი მტკივა.ადრეც მქონდა ასეთი ტკივილი ოღონდაც დილაობით როდესაც შარდის ბუშტი სავსე მქონდა,შემდეგ როცა ვშარდავდი მივლიდა,ახლა მთელი დღის განმავლობაში მაწუხებს. ვერ ვიტყვი რომ ძალიან მტკივა მაგრამ ხშირად მტკივა და ამიტომ ვწუხდები,რა უნდა იყოს და თუ შეგიძლიათ ასე ზეპირად დამეხმაროთ.წინასწარ მადლობტ

ასე ერთი ჩივილით რომ დიაგნოზი ისმებოდეს, რაღა უჭირს კაცობრიობას smile.gif მე გირჩევდი კარგ ექიმთნა მიხვიდე და გამოაკვლევინე პრობლემა.

ისე მაინც რამდენი წლის ხარ? რა სქესის? რაიმე პრობლემა თუ გაწუხებდა წარსულში? წამლებს თუ სვამ და თუ სვამ რას და რამდენი ხანი? სად არის ტკივილი ზუსტად? რა აძლიერებს ტკივილს და რა ანელებს ტკივილს (გარკვეული პოზა, საჭმელი, მოძრაობა)? ტკივილი თუ გადაეცემა სადმე? სხვა სიმპტომები თუ გაქვს (ღებინება, შარდვის პრობლემები, მუცლის ტკივილი და ა.შ. და ა.შ.)

Posted by: button nose 17 Feb 2009, 10:47
ვარ გოგო,26 წლის. არანაირი პრობლემა არასდროს არ მქონია,არანაირ წამალსაც არ ვღებულობ,ნუ რა ტქმა უნდა ტკივილგამაყუჩებელს იშვიათად,ანალგინს ან პენტალგინს.არფერი აძლიერებს ტკვილის,მხოლოდ როდესაც მუცელზე ვწევარ უფრო მეტად მტიკვა.და როდესაც შარდის ბუსტი მაქვს სავსე მაშინაც უფრო ძლიერად მტკივა.შარდის ანალიზი გავიკეთე და რაღაც მაქვს მომატებული.....მგონი რაგაც ლეიკოციტები აღარ მახსოვს ზუსტად სწორად ვიზახი თუ არაsmile.gif

Posted by: Avastin 17 Feb 2009, 13:15
QUOTE (texasuri jleta benzoxerxit @ 17 Feb 2009, 02:16 )
[/QUOTE]
რატომ ბრაზობ ავასტინ? 2kiss.gif ისე რამდენი ვიღაცის მოგონილია კაცო ეს იუმორი,ვერა და ვერ მივაკვლიე პირველწყაროს...

მე ჩვენი ცის მნათობთა სადარი მოდერი ვიგულისხმე უპირატესად... givi.gif
იმასაც ვაკოცებ,რომ არ გამიბრაზდეს 2kiss.gif

ანდა გამიბრაზდეს რა...ტაკ დაჟე ინწერესნეიე

არ ვბრაზობ გეტაყვა, პირიქით... მომწონს შენი ვირტუალური პიროვნება! იუმორი კი ნამდვილად ჩემი მოგონილია! vik.gif ცერტიფიცირებული იუმორისტი ვარ! boli.gif
* * *
QUOTE (button nose @ 17 Feb 2009, 10:33 )
კითხვა მაქვს.უკვე დაახლოებით ერთი თვეა მარცხენა თირკმელი მტკივა.ადრეც მქონდა ასეთი ტკივილი ოღონდაც დილაობით როდესაც შარდის ბუშტი სავსე მქონდა,შემდეგ როცა ვშარდავდი მივლიდა,ახლა მთელი დღის განმავლობაში მაწუხებს. ვერ ვიტყვი რომ ძალიან მტკივა მაგრამ ხშირად მტკივა და ამიტომ ვწუხდები,რა უნდა იყოს და თუ შეგიძლიათ ასე ზეპირად დამეხმაროთ.წინასწარ მადლობტ

რა გითხრა, ჩემო კარგო, ანამნეზი მთლად ვერ გაგვიშალე, მარა არ დაიდარდო...
ლეიკოციტები არიო შარდში?! წვა და ხშირი შარდვაც გაწუხებს? როცა სავსეა შარდის ბუშტი მაშინ თუ გტკივა ფერდის მიდამოში , გირჩევდი, უროლოგისთვის მიგემართა, ვესიკო-ურეტერალური რეფლუქსი გამორიცხონ!

Posted by: merry 17 Feb 2009, 14:05
ალაბათ ბევრჯერ უკითხავთ ამ თემაში, მარა - კარგი თერაპევტი ASAP user.gif არავირტუალური (gigi.gif), ვის მირჩევთ? ჩემთვის არ მინდა user.gif
პლსპლსპლსპლს


Posted by: button nose 17 Feb 2009, 15:00
Avastin
როცა სარდის ბუშტი სავსეა კი მაშინ უფრო მტკივა, მარცხენა მხარეს.წვა და ხშირი შარდვა არც ისე ძალიან.მაგრამ ხანდახან კიsmile.gif))და ვისთან მივიდე ან სად ხომ ვერ მირჩევდით?

Posted by: Avastin 17 Feb 2009, 15:14
QUOTE (merry @ 17 Feb 2009, 14:05 )
ალაბათ ბევრჯერ უკითხავთ ამ თემაში, მარა - კარგი თერაპევტი ASAP user.gif არავირტუალური (gigi.gif), ვის მირჩევთ? ჩემთვის არ მინდა user.gif
პლსპლსპლსპლს

ვირტუალურიდან მოძებნე და ცხადში მიაკითხე... რამდენს, თუმდაც ამ ფორუმში, მოაზროვნე ექიმს წააწყდება კაცი... მჟლეტელი გინდა თუ არამჯ- ვანო -ლეტელი... givi.gif gigi.gif givi.gif
* * *
QUOTE (button nose @ 17 Feb 2009, 15:00 )
Avastin
როცა სარდის ბუშტი სავსეა კი მაშინ უფრო მტკივა, მარცხენა მხარეს.წვა და ხშირი შარდვა არც ისე ძალიან.მაგრამ ხანდახან კიsmile.gif))და ვისთან მივიდე ან სად ხომ ვერ მირჩევდით?

უროლოგიურ კლინიკაში მიდი, იქვე ეხო-საც გადააგებინებ.

Posted by: merry 17 Feb 2009, 15:34
Avastin
ძვუირფასო დროში ვიწვი ძალიან. თუ შესაძლებელია ზუსტი კოორდინატები დამიწეროთ მადლობელი დაგრჩებით. ძებნის საშუალება რომ მქონოდა არ დავსვამდი ამ კითხვას. smile.gif

გავედი ახლა.

Posted by: la cioccolata calda 17 Feb 2009, 20:53
სამედიცინო ცენტრი ქირონი ვინმეს გაუგია?

თუ რამე იცით მის შეშახებ მომწერეთ რა პლს
* * *
-----------------

და ან ლია მანჯავიძე

Posted by: anonymouska 19 Feb 2009, 01:08
აქაც ვიკითხავ, იმედია აქ მაინც სუნთქავს ვინმე user posted image

დიდ დროს რომ ატარებ კომპთან(12 საათზე მეტს) და ამის გამო გადაღლილი ხარ(ანუ მისუსტებულია ორგანიზმი) რა უნდა ვქნააათ?

და კიდევ მაინტერესებს:

1.ვიტამინების მიღება მიზანშეწონილია?
2. რა საკვები უნა მივიღოთ რომ ორგანიზმი მოძლიერდეს?

Posted by: vano_t 19 Feb 2009, 01:32
QUOTE (anonymouska @ 19 Feb 2009, 01:08 )
აქაც ვიკითხავ, იმედია აქ მაინც სუნთქავს ვინმე user posted image

დიდ დროს რომ ატარებ კომპთან(12 საათზე მეტს) და ამის გამო გადაღლილი ხარ(ანუ მისუსტებულია ორგანიზმი) რა უნდა ვქნააათ?

და კიდევ მაინტერესებს:

1.ვიტამინების მიღება მიზანშეწონილია?
2. რა საკვები უნა მივიღოთ რომ ორგანიზმი მოძლიერდეს?

1) დიდი დრო არ უნდა გაატარო კომპიუტერთან და ხშირად დაისვენო.

2) ბუნებრივი საკვები: ხილი და ბოსტნეული-არაფერი ჯობია ამას.

Posted by: anonymouska 19 Feb 2009, 02:02
QUOTE (vano_t @ 19 Feb 2009, 01:32 )

1) დიდი დრო არ უნდა გაატარო კომპიუტერთან და ხშირად დაისვენო.

2) ბუნებრივი საკვები: ხილი და ბოსტნეული-არაფერი ჯობია ამას.

არ უნდა გავატაროთ, მაგრამ თუ სხვა გზა არ არის და უნდა გაატარო? როცა გიწევს მუშაობა 12 საათზე მეტი.





თუ შეიძლება მიპასუხეთ: როდესაც გრძნობ რომ ორგანიზმი დასუსტდა და ძალიან მალე იღლები, მიზანშეწონილია თუ არა ამ დროს ზრდასრულმა ადამიანმა მიიღოს ვიტამინები ?

და


ყველაზე სასარგებლო რა არის ხილიდან და ბოსტნეულიდან? (ასეთ სიტუაციაში smile.gif )

Posted by: anano333 19 Feb 2009, 09:13
გამარჯობათ
კითხვა მაქვს,პირველი(რეზუსუარყოფით)ჯგუფის სისხლს გარდა იმისა რომ იშვიათია სხვა უარყოფითი მხარეც ხომ არ აქვს?

Posted by: addicted 19 Feb 2009, 14:26
anano333
ამ თემაში თუ შეიხედავ კარგი გოგო იქნები 2kiss.gif http://forum.ge/?showtopic=33925556&st=15&hl=

Posted by: anano333 19 Feb 2009, 19:17
QUOTE (addicted @ 19 Feb 2009, 14:26 )
anano333
ამ თემაში თუ შეიხედავ კარგი გოგო იქნები 2kiss.gif http://forum.ge/?showtopic=33925556&st=15&hl=

გუშინ ვნახე ეგ თემა user.gif
შვილს აქვს მაგ ჯგუფის სისხლი,7წლისაა,თან არც საქართველოში არ ვართ თორე ცოტას მაინც გავცემდით,ზალიან განვიცადე იმ ბავშვის მდგომარეობა user.gif

Posted by: vano_t 19 Feb 2009, 20:07
anonymouska
QUOTE
არ უნდა გავატაროთ, მაგრამ თუ სხვა გზა არ არის და უნდა გაატარო? როცა გიწევს მუშაობა 12 საათზე მეტი.





თუ შეიძლება მიპასუხეთ: როდესაც გრძნობ რომ ორგანიზმი დასუსტდა და ძალიან მალე იღლები, მიზანშეწონილია თუ არა ამ დროს ზრდასრულმა ადამიანმა მიიღოს ვიტამინები ?

და


ყველაზე სასარგებლო რა არის ხილიდან და ბოსტნეულიდან? (ასეთ სიტუაციაში smile.gif )

ხვა გაზა ყოველთვის არის. მიუხედავად იმისა, გიწევს თუ არა კომპიუტერთან ხშირად ყოფნა, ყოველ საათში შეგიძლია 5-10 წუთი შესვენება გააკეთო. ფეხზე ადგე და გაიარგამოიარო; ღრმად ჩაისუნთქო და ამოისუნთქო. სუნთქვას დააკვირდე. ასეთი მცირე ვარჯიშიც საკმარისი იქნება, რომ მნიშვნელოვნად შეამცირო დატვირთვა. რა თქმა უნდა, კომპიუტერთან ჯდომის სწორი მანერაც არის საჭირო, რათა ხერხემლაზე დიდი დატვირთვა არ მიიღო.

რაც შეეხება დაღლილობას. ეგ სტრესისაგან მოდის. ვიტამინი სტრესს არ მოხსნის. სტრესს უნდა განტვირთვა. ვიტამინებს კიდევ ხილით და ბოსტნეულით უამრავს მიიღებ. ხილ-ბოსტნეულში შევა სხვადასხვა სახის ხილ ბოსტნეული რასაც ხილი და ბოსტნეული ქვია. ამას შეგიძლია მიამატო თხილეულიც.

anano333
QUOTE
გამარჯობათ
კითხვა მაქვს,პირველი(რეზუსუარყოფით)ჯგუფის სისხლს გარდა იმისა რომ იშვიათია სხვა უარყოფითი მხარეც ხომ არ აქვს?
სისხლის სხვადასხვა ჯგუფს არავითარი უარყოფითი მხარე არ აქვს გარდა იმისა, რომ რეზუს უარყოფითი სისხლი თუ გაქვს და ნაყოფს აქვს რეზუს დადებითი სისხლი, ამას შეიძლება მოყვეს ნაყოფის სერიოზული დაავადება. მაგრამ, ამის თავიდან აცილებაა შესაძლებელი დღევანდელი მედიცინის პირობებში. ყველა გინეკოლოგმა იცის ეს საკითხი.

Posted by: anano333 19 Feb 2009, 21:29
QUOTE (vano_t @ 19 Feb 2009, 20:07 )
[anano333
QUOTE
გამარჯობათ
კითხვა მაქვს,პირველი(რეზუსუარყოფით)ჯგუფის სისხლს გარდა იმისა რომ იშვიათია სხვა უარყოფითი მხარეც ხომ არ აქვს?
სისხლის სხვადასხვა ჯგუფს არავითარი უარყოფითი მხარე არ აქვს გარდა იმისა, რომ რეზუს უარყოფითი სისხლი თუ გაქვს და ნაყოფს აქვს რეზუს დადებითი სისხლი, ამას შეიძლება მოყვეს ნაყოფის სერიოზული დაავადება. მაგრამ, ამის თავიდან აცილებაა შესაძლებელი დღევანდელი მედიცინის პირობებში. ყველა გინეკოლოგმა იცის ეს საკითხი.

2kiss.gif მადლობ
კიდევ ერთი შეკითხვა 0 ,A ,B.AB
-0 პირველია ხომ,
და უარყოფითი რეზუსის ამღნიშვნელი როგორ იწერება -0 ?

Posted by: vano_t 19 Feb 2009, 21:48
anano333
QUOTE
კიდევ  ერთი შეკითხვა 0 ,A ,B.AB 
  -0 პირველია ხომ,
და უარყოფითი რეზუსის ამღნიშვნელი როგორ იწერება -0 ?

0 (ნოლი) არა, O (ასო "ო").

I, II, III და IV აღნიშნვნებს საზღვარგარეთ არ ხმარობენ და ზუსტად ვერ გეტყვი რომელი შეესაბამება რომელს. მარა, O ნიშნავს, რომ არც A და არც B ანტიგენები არ აქვს მოცემულ ადამიანს და შესაბამისად წარმოქმნის ანტისხეულებს ანტიგენიანი სისხლის ჯგუფების მიმართ, რაც გამოიწვევს გადასხმისას რეაქციებს. ასეთი სისხლი შეიძლება გადაუსხა ყველას, მაგრამ თვითონ ეს ჯგუფი არ მიიღებს არცერთ სისხლს გარდა O-სი. AB ყველას მიიღებს, მაგრამ არავის არ გადაესხმება რეაქციის გარეშე გარდა ისევ AB-სი. A მიიღებს მხოლოდ A-ს და O-ს და გადაესხმება A-ს და AB-ს. B-ც მიიღებს მხოლოდ B-ს და O-ს და გადაესხმება B-ს და AB-ს.

რეზუსის აღსანიშნავად მარჯვენა მხარეს ემატება ან "+" ან "-" ნიშანი. მაგალითად AB+, ან O-; ან ეს ნიშნები მარცხნივ შეიძლება დაამატო, სულერთია ეგ.

Posted by: anonymouska 19 Feb 2009, 23:12
vano_t

აჰ მადლობთ smile.gif

და სტრესისგან როგორ უნდა განიტვირთო? როცა თავისუფალი დრო თითქმის არ გაქვს.

Posted by: vano_t 20 Feb 2009, 00:06
QUOTE (anonymouska @ 19 Feb 2009, 23:12 )
vano_t

აჰ მადლობთ smile.gif

და სტრესისგან როგორ უნდა განიტვირთო? როცა თავისუფალი დრო თითქმის არ გაქვს.

ყოველ საათში ერთხელ 5-10 წუთი დაუთმე განტვირთვას. სხვანაირად ვერ განიტვირთები.

Posted by: ANNA_LOM 20 Feb 2009, 22:00
წნევა მაქვს 70/50 ზე და ფეხეში ვგრძნობ სიმძიმეს? რა შეიძლბა იყოს ამის მიზეზი და რომელ ექიმს უნდა მივმართო?

Posted by: texasuri jleta benzoxerxit 21 Feb 2009, 01:38
QUOTE (ANNA_LOM @ 20 Feb 2009, 22:00 )
წნევა მაქვს 70/50 ზე და ფეხეში ვგრძნობ სიმძიმეს? რა შეიძლბა იყოს ამის მიზეზი და რომელ ექიმს უნდა მივმართო?

თერაპევტს რაღა თქმა უნდა ა ტაკჟე ედნდოკრინოლოგს,ნევროპათოლოგს..და ა.შ. gigi.gif

Posted by: Solveig 21 Feb 2009, 16:30
QUOTE
უარყოფითი რეზუსის ამღნიშვნელი როგორ იწერება -0 ?


O (I) Rh-

Posted by: addicted 21 Feb 2009, 17:46
თანაფორუმელებო!! ძალიან აღშფოთებული ვარ და იქნებ თქვენ მითხრათ რამდენად საფუძვლიანია ჩემი აღშფოთება. ჩემი ოჯახის ერთერთი წევრი ამ კვირას იმყოფებოდა ვიზიტზე ერთერთ პრესტიჟულ კლინიკაში ასევე პრესტიჟულ ექიმთან. პაციენტს აწუხებდა კუჭის ტკივილი და მისვლის მიზეზიც ეს ოყო. ექიმმა არც მოუსმინა რა ჭირდა (ხელით გასინჯვაზე ლაპარაკი ზედმეტია) ისე ჩამოუწერა ჩასატარებელი გამოკვლევები (ექოსკოპია და ტომოგრაფია, სისხლის საერთო, ბიოქიმიური ანალიზი, შარდის ანალიზი) და არა გასტროსკოპია!!! სისხლში აღმოაჩნდა ლეიკოციტების დონე და როე ძალიან მაღალი. ჰოდა დაიწყეს მკითხაობა, სეფსისი, კანის ტუბერკულოზი და ა.შ. როგორც იქნა გუშინ დაიბარეს გასტროსკოპიის ჩასატარებლად და აღმოჩნდა რომ კვლევისთვის არ იყო სათანადოდ მომზადებული (წინასწარ არ გაუფრთხილებიათ) და დაიბარეს დღეს ხელმეორედ და რა თქმა უნდა მეორედ გადაახდევინეს 84 ლარი: აღმოაჩნდა თორმეტგოჯა ნაწლავის წყლულოვანი დაავადება (გართულებული) სუბკომპენსირებული, პილოროსტენოზით, მწვავე დისტალური ეროზიული ეზოფაგიტი.

ძალიან გთხოვთ გამარკვიოთ:
1. რამდენად სწორად მოიქცა ექიმი როდესაც ჩივილების უკითხავად ამდენი გამოკვლევა ჩაატარებინა პაციენტს (სისხლის და შარდის ანალიზებს არ ვგულისხმობ)
2. რამდენად საშიშია დასმული დიაგნოზი? (გულს გვიხეთქავენ ზერთული ფორმააო)
3. ექიმი ამბობს რომ კიდევ გაურკვეველია რატომ აქვს მომატებული ლეიკოციტების დონე და როე, შეიძლება გახეთქილი იყოს თორმეტგოჯაო. შესაძლებელია? (ექოსკოპიის მიხ. მუცელში თავისუფალი სითხის არსებობა არ შეინიშნება)

ძალიან გთხოვთ მიპასუხოთ, თუ უსაფუძვლოა ჩემი აღშფოთება დავწყნარდები მაინც და აღარც ოჯახში ვალანძღინებ სხვებს იმ ექიმს biggrin.gif

Posted by: ana_nushka 22 Feb 2009, 00:04
ხალხნო სიბრძნის კბილის ამოსვლამ იცის ყელის ტკივილი??

ისეთი შეგრძნება მაქვს თითქოს მთელი ყელი დასიებული მაქვს და მახრჩობს რაღაც, განსაკუთრებულად ყბის ქვემოთ . . . drug.gif არადა არ არის დასიებული . . .საჭმელი ნორმალურად ჩამდის givi.gif

იცის სიბრძნის კბილმა ესეთი რანცხები თუ რა ღაც ნერვოზულს გავს?????? cry.gif

Posted by: ბახარხალა 22 Feb 2009, 00:19
http://imghost.ge/out.php/i2792_untitled.JPG <<< ამ პრეპარატზე თუ გსმენიათ რამე : |

Posted by: anonymouska 22 Feb 2009, 02:44
ana_nushka შეიძლება გადავიდა ყელშიც. მეც ახლა ამომდის მაგრამ სადაც ამოდის ის ადგილი მტკივა biggrin.gif

ისე ჩაახედე ექიმს თუ არ გაგივლის )))

Posted by: basa-ttt 22 Feb 2009, 11:46
QUOTE
1. რამდენად სწორად მოიქცა ექიმი როდესაც ჩივილების უკითხავად ამდენი გამოკვლევა ჩაატარებინა პაციენტს (სისხლის და შარდის ანალიზებს არ ვგულისხმობ)

აი მედიცინა რომ უფასო იყოს -
ეს პრობლემა არ იქნებოდა.

Posted by: Solveig 22 Feb 2009, 15:33
basa-ttt
QUOTE
აი მედიცინა რომ უფასო იყოს -
ეს პრობლემა არ იქნებოდა.

უფასო ყველი მხოლოდ სათაგურშია (კოპირაით: ზარაზა). არსად არ არის უფასო მედიცინა, მით უმეტეს, თუ გამოკვლევები არასწორადაა დაგეგმილი. გადასაყრელი ფული არც სახელმწიფოს არ აქვს.


addicted
QUOTE
თორმეტგოჯა ნაწლავის წყლულოვანი დაავადება (გართულებული) სუბკომპენსირებული, პილოროსტენოზით, მწვავე დისტალური ეროზიული ეზოფაგიტი.

ექიმი არ ვარ, მაგრამ როგორც დაახლოებით იგივე დიაგნოზიანი (ეზოფაგიტი არ ჰქონიათ ოღონდ) ოჯახის წევრების პატრონმა, შემიძლია გითხრა, რომ მართლა სერიოზულია..იმათ ოპერაცია დასჭირდათ, ორივეს. ახლა გაცილებით მეტი შეუძლია მედიცინას მაგ საკითხში, ვიდრე 15-20 წლის წინ, მაგრამ არ ვიცი, სტენოზის (ანუ, შევიწროების) მკურნალობა შეიძლება თუ არა ოპერაციის გარეშე.

QUOTE
ექიმი ამბობს რომ კიდევ გაურკვეველია რატომ აქვს მომატებული ლეიკოციტების დონე და როე, შეიძლება გახეთქილი იყოს თორმეტგოჯაო.

ამაზე ვერაფერს გეტყვი ნამდვილად.

Posted by: ბახარხალა 22 Feb 2009, 19:41
როგორ გავიგო რომნელი სისხლის ჯგუფი მაქ?


ოღონდ ისე რო არ მეტკინოს : |

Posted by: SoulTaker 22 Feb 2009, 20:54
თერაპევტებო
მტკივა მარცენა გვერდი
ან ფილტვი
სადღაც შუაში
რამდენიმე დღეა მახველებს და 37 დან 37.5 მდე მაქვს სიცხეები
სუნთქვა მიჭირს
მახველებს
ჰაერი არ მყოფნის
პულსი შედარებით აჩქარებული მაქვს
თავბრუ მეხვევა
მადა არ მაქვს

ესეიგი:
შეიძლება თუ არა რომ ფილტვების ანთება მქონდეს უსიცხო??

მე გრიპი მეგონა მქონდა და ეს გვერდი იმის გამო მტკივა რომ ცოტა დავმძიმდი
(23 კვირის ფეხმძიმე ვარ)
და ეს სიმპტომებიც შეიძლება მქონდეს ალბათ ფილტვების ანთების გარეშე ხო?
რენტგენს ვერ გავიკეთებ
და
რა ვქნა?
მეშინია sad.gif

Posted by: baby-bobo 23 Feb 2009, 11:21
გიორგი კანდელაკი თუ იცით ვინმენ სად მუშაობს?

Posted by: shtori 23 Feb 2009, 21:36
მტვერზე მეწყება ხველა. ძალიან ცუდად გამხადა დღეს, ლამის გავიგუდე. აფთიაქში "ლიზლი" მირჩიეს და დავლიე +ნოშპა. ხველა კი გამიჩერა მალევე. მაგრამ ისე გამაბრუა ფეხზე ძლივს ვიდექი. ერთად არ უნდა დავლიო ხოლმე? ან სწრაფმოქმედი სხვა რამ მირჩიეთ რა

Posted by: ana_nushka 23 Feb 2009, 22:25
ყელის პრობლემა მაქვს იქნებ ვინმემ რამე მირჩიოთ??

ერთი კვირაა ყელზე, ყბის ქვემოთ თითქოს რაღაც მაზრჩობდა ისე მტკიოდა . . .ჯერ სიბრძნის კბილ დავაბრალე მაგრამ როგორც ჩანს არაფერ შუაშია . . .

ეხლა ესეთი ტკივილი გადავიდა უფრო ქვედა ნაწილში საყლაპავთან . . . ნახველი მაქვს მთელ ყელში და ნერწყვის ყლაპვა მიჭირს . . . (მაგრამ არ მახველებს) სისეთი შეგრძნება მაქვს თითქოს რაღაც მაქვს გაჩხერილი ყელში . . . sad.gif მაგრამ აშკარად არაფერია . . .

რას მირჩევთ . . .რამე ხველების წამალი (ნახველის გასაქრობად)თუ ანტიბიოტიკი მივიღო??? სავლებს ვივლებდი მაგრამ აარ გამიუმჯობესა სიტუაცია . . .

sad.gif

Posted by: texasuri jleta benzoxerxit 24 Feb 2009, 01:50
QUOTE (ana_nushka @ 23 Feb 2009, 22:25 )
ყელის პრობლემა მაქვს იქნებ ვინმემ რამე მირჩიოთ??

ერთი კვირაა ყელზე, ყბის ქვემოთ თითქოს რაღაც მაზრჩობდა ისე მტკიოდა . . .ჯერ სიბრძნის კბილ დავაბრალე მაგრამ როგორც ჩანს არაფერ შუაშია . . .

ეხლა ესეთი ტკივილი გადავიდა უფრო ქვედა ნაწილში საყლაპავთან . . . ნახველი მაქვს მთელ ყელში და ნერწყვის ყლაპვა მიჭირს . . . (მაგრამ არ მახველებს) სისეთი შეგრძნება მაქვს თითქოს რაღაც მაქვს გაჩხერილი ყელში . . . sad.gif მაგრამ აშკარად არაფერია . . .

რას მირჩევთ . . .რამე ხველების წამალი (ნახველის გასაქრობად)თუ ანტიბიოტიკი მივიღო??? სავლებს ვივლებდი მაგრამ აარ გამიუმჯობესა სიტუაცია . . .

sad.gif

გირჩევ ელემენტარულა ყელ-ყურ-ცხვირის სპაციალისტთან სასწრაფო კონსულტაციას!!!

Posted by: zviadcardio 25 Feb 2009, 00:33
baby-bobo
QUOTE
გიორგი კანდელაკი თუ იცით ვინმენ სად მუშაობს?

თუ სწორად მახსოვს ინფექციურ საავადმყოფოში.

Posted by: baby-bobo 25 Feb 2009, 14:12
zviadcardio
QUOTE
თუ სწორად მახსოვს ინფექციურ საავადმყოფოში.

იქ აღარ არის sad.gif

Posted by: SPC 25 Feb 2009, 15:40
აუ თქვენც ასე ხართ? ეს გაზაფხულის ბრალია ხო?

დავსუსტდი საშინლად, ეს დღეებია უბრალო რაღაცეების გაკეთება საშინლად მღლის, სახლამდე ძვლივს მივდივარ და ვიძინებ.....




ისედაც არ მიყვარდა რა გაზაფხული

Posted by: "CEREBRO" 25 Feb 2009, 22:37
თირკმლებთან დაკავშირებული პრობლემებს რა სიმპტომები აქვთ?? ეჭვი მაქვს რომ რაGაც პრობლემები უნდა მქონდეს. . . სიცარიელესავით მაქვს და მერე ჩაჭიმულობის შეგრძნება თირკმლებში. . ოდნავი წვა შარდვის მერე . . . საშინლად დაღლილი ვარ ყოველთვის . . . წნევა დაბალი მაქვს . . . შეიძლება თიქკმლის ბრალი იყოს>> ან ვის მივმართო?? მედიცინის ენაზე რა ქვია თირკმლის ექიმს?? გმადლობთ

Posted by: lizi11 26 Feb 2009, 10:38
თერაპევტებო შეკითხვა მაქვს - რა დაავადების სიმპტომებია სახის და ხელის თითების და ფეხის თითების შეშუპება ერთდროულად, როცა თირკმელები წესრიგშია. ამავდროულად აღინიშნება ხელებზე წვა და ქავილი და წითელი ლაქები ქავილის ადგილებზე.
* * *
თერაპევტებთან მაქვს შეკითხვა rolleyes.gif

Posted by: Solveig 27 Feb 2009, 22:46
მოკლედ, ასეთი კითხვა მაქვს:

გამონაყარი რომ ვირუსული წარმოშობის იყოს (ანუ, იყოს ვირუსული ინფექციის სიმპტომი), როგორაა შესაძლებელი დიაგნოსტიკა? შესაძლებელია, ეს დადგინდეს ბიოფსიით (კონკრეტულ ვირუსს აბათ ვერ დაადგენ, მაგრამ თუ სეიძლება ჰისტოპრეპარატზე იყოს ვირუსული ინფექციისათვის დამახასიათებელი სურათი), თუ ანტისხეულებზე სკრინინგი უნდა ჩატარდეს? თუ ორივე ერთად?

ძალიან ბევრი კითხვა კი გამომივიდა.

Posted by: LULA_QABABI 28 Feb 2009, 12:28
QUOTE (Solveig @ 27 Feb 2009, 13:46 )
მოკლედ, ასეთი კითხვა მაქვს:

გამონაყარი რომ ვირუსული წარმოშობის იყოს (ანუ, იყოს ვირუსული ინფექციის სიმპტომი), როგორაა შესაძლებელი დიაგნოსტიკა? შესაძლებელია, ეს დადგინდეს ბიოფსიით (კონკრეტულ ვირუსს აბათ ვერ დაადგენ, მაგრამ თუ სეიძლება ჰისტოპრეპარატზე იყოს ვირუსული ინფექციისათვის დამახასიათებელი სურათი), თუ ანტისხეულებზე სკრინინგი უნდა ჩატარდეს? თუ ორივე ერთად?

ძალიან ბევრი კითხვა კი გამომივიდა.

არსებობს ვირუსის კულტურა, უჯრედულ ნიადაგზე სადაც სპეციფიური ციტოპათიური ცვლილებების მიხედვით ადგენენ ვირუსის ტიპს; არის ასევე სწრაფი მეთოდებიც ძირითადად ჰერპეს სიმპლექსის და ვარიცელა (ჩუტყვავილას) დიაგნოსტირებისთვის - პირდაპირი ფლუორeსცენციით ანტიგენის (direct fluorescent antigen - DFA) დადგენა. არამგონია საქართველოში ეს მეთოდები არსებობდეს;
ზოგიერთ შემთხვევაში PCR-ს გამოყენებაც შეიძლება მაგრამ უნდა დააკონკრეტო რა ტიპის ვირუსები გაინტერესებს


Posted by: Solveig 28 Feb 2009, 17:28
LULA_QABABI
QUOTE
არამგონია საქართველოში ეს მეთოდები არსებობდეს;

მე თუ გერმ,ანიაში ვარ, რა ხეირი მერე sad.gif

QUOTE
უნდა დააკონკრეტო რა ტიპის ვირუსები გაინტერესებს

არაფრის დაკონკრეტება არ შემიძლია, სამწუხაროდ, ვმკითხაობ sad.gif იმიტომ, რომ ოქტომბრიდან მოყოლებული, დიაგნოზი ვერ დამისვეს..ჰოდა, ახლა ამ თვის ბოლოს დამიბარეს-ისევ თუ გექნება გამონაყარი, ბიოფსია უნდა გაკეთდეს ტავიდანო (ერთხელ გაკეთებული მაქვს უკვე, წინა გვერდებზე წერია).

Posted by: georgy 28 Feb 2009, 20:46
რას უნდა ნიშნავდეს მწვანე ენა? sleep.gif

Posted by: LULA_QABABI 28 Feb 2009, 23:54
QUOTE (Solveig @ 28 Feb 2009, 08:28 )
LULA_QABABI
QUOTE
არამგონია საქართველოში ეს მეთოდები არსებობდეს;

მე თუ გერმ,ანიაში ვარ, რა ხეირი მერე sad.gif

შენი ინფორმაციიდან გამომდინარე, ისიც საეჭვოა ვირუსი არის თუ არა; სურათი დადე თუ მოგიხერხდება, ან პმ-ზე გამოუშვი

Posted by: EXHALATING 1 Mar 2009, 01:14
მიშველეთ ვინმემ რა sad.gif დღეს ავიღე სისხლი ვენიდან. ჯერ მეტკინა, ნუ ეგ არაფერი... მაგრამ ამ ტკივილმა არ გამიარა. მკლავს ვერ ვხრი ისე მტკივა,შეხებაზეც კი ... თან შესიებულივით მაქვს. ნორმალურია? რით ვუშველო? sad.gif

Posted by: "CEREBRO" 1 Mar 2009, 13:47
EXHALATING
აცადე და ჩაცხრება .. . ცივის დადება შეიძლება. . პირველად აიღე?

Posted by: gigizura 1 Mar 2009, 18:18
ორი თვეა მუხლის ზემოთა კუნთი მტკივა, გამივლის და ისევ პატარა დატვირთვაზე მირთულდება, დიდხანს დგომით მტკივდება შემდეგ ფეხს რომ გავავარჯიშებ შედარებით ტკივილი გამივლის, გავიკეთე მოვალისი, გამიარა და ეხლა ისევ ამტკივდა, ეხლა ვოლტარენის მალამოს ვისვამ და პატარა ნაგრუსკაზე მტკივდაბა, დავიღალე ამის დედაც, მანქანის ცეპლენიის პედალს რომ ვაცვები მტკივდება, დაჭიმვით ვარჯიშებს ვაკეთებდი და იმას ვაბრალებ, ხომ არ იცით ამის წამალი ვინმემ

Posted by: Solveig 1 Mar 2009, 22:31
LULA_QABABI
http://forum.ge/?f=43&showtopic=33759979&st=765

აქ წერია.

Posted by: EXHALATING 1 Mar 2009, 22:32
"CEREBRO"
დღეს ოდნავ ჩაცხრა. პირველად არ ამიღია. ადრე ისე ავიღე არც მიგრძვნია, არანაირი ტკივილი და დისკომფორტის. ახლა რა დამემართა ან მე, ან იმ ქალს.....

Posted by: blblbl 1 Mar 2009, 23:16
ხელზე ვენა ამომებერა პატარაზე და მტკივა ხელს რომ ვადებ რისი ბრალია? sad.gif

Posted by: EXHALATING 2 Mar 2009, 00:28
blblbl
ვირუსია ალბათ ვენის დასიების lol.gif

Posted by: vano_t 2 Mar 2009, 03:42
lizi11
QUOTE
თერაპევტებო შეკითხვა მაქვს -  რა დაავადების სიმპტომებია სახის და ხელის თითების და ფეხის თითების შეშუპება ერთდროულად, როცა თირკმელები წესრიგშია. ამავდროულად აღინიშნება ხელებზე წვა და ქავილი და წითელი ლაქები ქავილის ადგილებზე.
* * *
თერაპევტებთან მაქვს შეკითხვა  rolleyes.gif

ექიმი გინდა, კაი ექიმი smile.gif

შეიძლება უბრალოდ ალერგიაა. სურათი დადი და უფრო დაწვრილებით აღწერე: როდის გემართები? გამონაყარი რამხელაა? რამდენი ხანი გრძელდება? რამესთან კავშირშია თუა (მაგალითად საკვებთან ან სარეცხ დეტერგენტებთან, ან კრემებთან და ა.შ.)?

georgy
QUOTE
რას უნდა ნიშნავდეს მწვანე ენა?
ეს არის ენის დრვილების არასრული აქერცვლის პრობლემა და ბევრმა რამემ შეუძლია გამოიწვიოს. არასრული ჰიგიენა პირის ღრუსი, სიგარეტი, ალკოჰოლი, ჩაი, საღებავიანი პირის ღრუს სავლებები ან მენთოლიანი "კანფეტები", ფართო სპექტრის ანტიბიოტიკები, მარტო რბილი (არაუხეში) საკვები და ა.შ.

gigizura
QUOTE
ორი თვეა მუხლის ზემოთა კუნთი მტკივა, გამივლის და ისევ პატარა დატვირთვაზე მირთულდება, დიდხანს დგომით მტკივდება შემდეგ ფეხს რომ გავავარჯიშებ შედარებით ტკივილი გამივლის, გავიკეთე მოვალისი, გამიარა და ეხლა ისევ ამტკივდა, ეხლა ვოლტარენის მალამოს ვისვამ და პატარა ნაგრუსკაზე მტკივდაბა, დავიღალე ამის დედაც, მანქანის ცეპლენიის პედალს რომ ვაცვები მტკივდება, დაჭიმვით ვარჯიშებს ვაკეთებდი და იმას ვაბრალებ, ხომ არ იცით ამის წამალი ვინმემ
პირველ რიგში ფიზიკური გამოკვლევა გჭირდება მუხლის. ტრამვატოლოგით დაიწყე.

EXHALATING
QUOTE
მიშველეთ ვინმემ რა sad.gif დღეს ავიღე სისხლი ვენიდან. ჯერ მეტკინა, ნუ ეგ არაფერი... მაგრამ ამ ტკივილმა არ გამიარა. მკლავს ვერ ვხრი ისე მტკივა,შეხებაზეც კი ... თან შესიებულივით მაქვს. ნორმალურია? რით ვუშველო? sad.gif
არაფერია. გაგივლის. თბილი საფენები დაიდე და იბუპროფენი (ან პარაცეტამოლი) დალიე და მალე გაგივლის.

Posted by: georgy 2 Mar 2009, 10:38
vano_t
QUOTE
georgy
QUOTE
რას უნდა ნიშნავდეს მწვანე ენა?
ეს არის ენის დრვილების არასრული აქერცვლის პრობლემა და ბევრმა რამემ შეუძლია გამოიწვიოს. არასრული ჰიგიენა პირის ღრუსი, სიგარეტი, ალკოჰოლი, ჩაი, საღებავიანი პირის ღრუს სავლებები ან მენთოლიანი "კანფეტები", ფართო სპექტრის ანტიბიოტიკები, მარტო რბილი (არაუხეში) საკვები და ა.შ.

დიდი მადლობა, როგორც მივხვდი საშიში არაფერია smile.gif

დღეს მივალ მაინც ექიმთან და ზუსტად დავადგენ smile.gif

Posted by: lizi11 2 Mar 2009, 11:21
vano_t
გმადლობ ,
smile.gif კაი ექიმი ნამდვილად მინდა. ე.ი ჯერ ალერგოლოგს მივაკითხავ. ალერტეკს ვსვამ ერთი თვეა უკვე და ეს ცოტა შეღავათს მაძლევს. ექიმი-ალერგოლოგიც მირჩიეთ, ვისთან მივიდე?

Posted by: gigizura 2 Mar 2009, 17:04
QUOTE (vano_t @ 2 Mar 2009, 03:42 )
gigizura
QUOTE
ორი თვეა მუხლის ზემოთა კუნთი მტკივა, გამივლის და ისევ პატარა დატვირთვაზე მირთულდება, დიდხანს დგომით მტკივდება შემდეგ ფეხს რომ გავავარჯიშებ შედარებით ტკივილი გამივლის, გავიკეთე მოვალისი, გამიარა და ეხლა ისევ ამტკივდა, ეხლა ვოლტარენის მალამოს ვისვამ და პატარა ნაგრუსკაზე მტკივდაბა, დავიღალე ამის დედაც, მანქანის ცეპლენიის პედალს რომ ვაცვები მტკივდება, დაჭიმვით ვარჯიშებს ვაკეთებდი და იმას ვაბრალებ, ხომ არ იცით ამის წამალი ვინმემ
პირველ რიგში ფიზიკური გამოკვლევა გჭირდება მუხლის. ტრამვატოლოგით დაიწყე.


უი, უი, არ ვიცოდი თუ ექიმთან უნდა მივსულიყავი, lol.gif

Posted by: georgy 2 Mar 2009, 18:19
vano_t
ვიყავი ექიმთან (ოტორინილარინგოლოგთან), ტატიშვილის კლინიკაში.
კონკრეტული დიაგნოზი ჯერ ვერ დასვა, ბაქტერიოლოგიის ანალიზის შემდეგ გვეცდინება რაშია საქმეო.

Posted by: giorgi33 4 Mar 2009, 13:45
მკურნალობის დროს ექოსკოპიის გაკეთება რამდენ ხანში ერთხელ არის რეკომენდირებული რომ ცვლილებები გამოჩნდეს?. მე როგორც მითხრეს სამი თვე აზრი არ აქვსო.
ასევე სისხლის ანალიზზე მაინტერესებს კონკრეტულად ალტ ასტ და ბილირუბინი. თვენახევრის ინტერვალით რომ ავიღო აქვს აზრი?

Posted by: addicted 4 Mar 2009, 15:03
giorgi33

ბილირუბინს მე ზოგჯერ ყოველდღე მიმოწმებდნენ biggrin.gif
რაზე მკურნალობ მაგაზეა დამოკიდებული როგორც ვიცი... თუმცა სპეციალისტების აზრს დაელოდე ჯობია smile.gif

georgy
QUOTE
ვიყავი ექიმთან (ოტორინილარინგოლოგთან), ტატიშვილის კლინიკაში. კონკრეტული დიაგნოზი ჯერ ვერ დასვა, ბაქტერიოლოგიის ანალიზის შემდეგ გვეცდინება რაშია საქმეო.


ილოცე რომ სხვა რამის ანალიზი არ მოგთხოვენ კიდევ biggrin.gif მანდ ისეთები იციაან...........

Posted by: EKIKULI 4 Mar 2009, 17:37
მაინტერესებს ხორხის შეშუპებას რა იწვევს და პაციენტს რა სიმპტომები აქვს ამ დროს?

Posted by: KABELI 4 Mar 2009, 19:53
არ ვიცი სწორ თემაში ვწერ თუ არა,იქნება მომწეროთ თუ არსებობს თუ არა ისეთი წამალი რომელიც ჭიების უმეტესობას მსპობს?

Posted by: shanyva 5 Mar 2009, 16:06
vano_t
გამარჯობა,
რისი ბარალი შეიძლება იყოს ხალების გაჩენა?განსაკუთრებით სახეზე,შეიძლება თუ არა რამენაირად თავი აარიდო ამ საშინელ ხალებს? sad.gif
მადლობა წინასწარ

Posted by: vano_t 6 Mar 2009, 08:06
EKIKULI
QUOTE
მაინტერესებს ხორხის შეშუპებას რა იწვევს და პაციენტს რა სიმპტომები აქვს ამ დროს?

ხორხის შეშუპებას ბევრი რამე იწვევს: ინფექციები (ბაქტერიული, ვირუსული, სოკოვანი), ფიზიკური ფაქტორები (ტრამვა, რადიაცია) და ქიმიური ფაქტორები (ტოქსიური ნაერთების ინჰალაცია).

სიმპტომები სერიოზული აქვს. ხორხის შეშუპებით ავადმყოფი მძიმედ ავადმყოფია და უპირველესი სიმპტომი არის სტრიდორი (ხმაურიანი, მსტვენავი სუნთქვა), სუნთქვის უკმარისობა და, შესაბამისად, ჟანგბადის უკმარისობა.

KABELI
QUOTE
არ ვიცი სწორ თემაში ვწერ თუ არა,იქნება მომწეროთ თუ არსებობს თუ არა ისეთი წამალი რომელიც ჭიების უმეტესობას მსპობს?
უმეტესობა ჭიის წამალი გარკვეული ჯგუფის ჭიებისათვის იხმარება. ამიტომ, ჯერ ექიმმა უნდა დაადგინოს რა ტიპის ჭია იწვევს დაავადებას.

shanyva
QUOTE
გამარჯობა,
რისი ბარალი შეიძლება იყოს ხალების გაჩენა?განსაკუთრებით სახეზე,შეიძლება თუ არა რამენაირად თავი აარიდო ამ საშინელ ხალებს? sad.gif
მადლობა წინასწარ
ჯერ უნდა დადგინდეს პიგმენტური წარმონაქმნის დიაგნოზი. სახეზე ჭორფლიც შეიძლება გაჩნდეს და ავადმყოფს ხალში შეეშალოს. თუ ნამდვილად ხალები ჩნდება, მაშინ ერთადერთი პრევენციული საშუალება, რაც მე ვიცი, არის ან მზისაგან თავის არიდება, ან სპეციალური კრემების ხმარება რომელიც ულტრიისფერი სხივებისაგან დაიცავს ავადმყოფს.

Posted by: natiko 6 Mar 2009, 13:00
გამარჯობათ ორსულად ვარ და დამინიშნეს ფოლიუმის მჟავა და ტრომბო აცც რამაც გამოიცვია კუჭის საშინელი გაღიზიანება და ტკივილები დროებიტ შემიწყვიტა გინეკოლოგმა ეს წამლები მაგრამ კუჭის სამკურნალოდ არაფერი დამინიშნა . ორსულობის დროს შეიზლება რაიმე მკურნალობის ჩატარება კუჭნაწლავზე?

Posted by: EKIKULI 6 Mar 2009, 15:29
vano_t
QUOTE
ხორხის შეშუპებას ბევრი რამე იწვევს

ალერგიული ფონიც?
სერეტიდი თუ ხსნის ამ სპაზმს?

Posted by: addicted 6 Mar 2009, 15:56
EKIKULI

მე სიგარეტის სუნზე მეწყება ყელის შეშუპება და ხველა sad.gif

Posted by: vano_t 7 Mar 2009, 02:23
natiko
QUOTE
გამარჯობათ ორსულად ვარ და დამინიშნეს ფოლიუმის მჟავა და ტრომბო აცც რამაც გამოიცვია კუჭის საშინელი გაღიზიანება და ტკივილები დროებიტ შემიწყვიტა გინეკოლოგმა ეს წამლები მაგრამ კუჭის სამკურნალოდ არაფერი დამინიშნა . ორსულობის დროს შეიზლება რაიმე მკურნალობის ჩატარება კუჭნაწლავზე?

ანტაციდები შეგიძლია სინჯო (კალციუმის შემცველი ან მაგნიუმის შემცველი, რაც შენთან იშოვება). აგრეთვე, თუ ანტაციდები არ მოაგვარებენ პრობლემას, მაშინ ომეპრაზოლი შეიძლება სინჯო. ომეპრაზოლზე არსებობს გარკვეული მონაცემები, რომ არ უნდა იყოს ტერატოგენული და არ უნდა ახასიათებდეს გაზრდილი რისკი სხვა არასასურველი გამოსავალის (მაგალითად, სპონტანუტი აბორტები, ნაყოფის მცირე წონა). რამოდენიმე წყაროდან შევამოწმე და ერთ-ერთი ქვემოთ არის მოცემული.

http://www.pubmedcentral.nih.gov/articlerender.fcgi?artid=1781096

EKIKULI
QUOTE
ალერგიული ფონიც?
სერეტიდი თუ ხსნის ამ სპაზმს?
ალერგიის დროს რომ ხორხი შუპდება, ეს უკვე არის ანაფილაქსიური რეაქცია. ამ დროს ავადმყოფები რეანიმაციაში იმყოფებიან.

შენ სულ სხვა რამეზე საუბრობ, მე მოგნი, საერთოდ. რას ეძახი ხორხის შეშუპებას?

Posted by: EKIKULI 7 Mar 2009, 11:24
vano_t
შეგრძნება რომ ყელი შეშუპებულია და ნერწყვის ყლაპვაში უშლის ადამიანს ხელს ამის მიზეზი რა შეიძლება იყოს?
ჯირკვლები შესიებულია და ყელის უკანა კედელიც შეწითლებული, ანტიბიოტიკებით მკურნალობის შემდეგ შEდეგი არ არის. პაციენტი განიცდის დისკომფორტს.


Posted by: vano_t 8 Mar 2009, 08:31
EKIKULI
QUOTE
შეგრძნება რომ ყელი შეშუპებულია და ნერწყვის ყლაპვაში უშლის ადამიანს ხელს ამის მიზეზი რა შეიძლება იყოს?
ჯირკვლები შესიებულია და ყელის უკანა კედელიც შეწითლებული, ანტიბიოტიკებით მკურნალობის შემდეგ შEდეგი არ არის. პაციენტი განიცდის დისკომფორტს.

ეს სხვა რამ არის. ეს არ არის ხორხის შეშუპება. მაგას ქვია ფარინგიტი (და ტონზილიტი, თუ ნუშურა ჯირკლვებიც გაწითლებულია). მაგის ყველაზე ხშირი მიზეზია ვირუსები და ეგ არის ძაან ხშირი პრობლემა. საერთოდ მაგას ანტიბიოტიკი არ ჭირდება. წესით, როცა ავადმყოფს მაგ ჩივილი აქვს, ავადმყოფს უკეთდება ხახის და ტონზილების ნაცხი და თუ სტრეპტოკოკული ფარინგიტი გამოირიცხა, მაშინ ანტიბიოტიკი არ არის საჭირო თავიდან. ვირუსულ ფარინგიტს ანტიბიოტიკი მაინც ვერაფერს ვერ უზამს-ანტიბიოტიკი არ არის ვირუსისათვის შექმნილი წამალი. ვირუსული ფარინგიტი შეიძლება 1-2 კვირა გაგრძელდეს და მერე თავისით წავა. მკურნალობა არის მხოლოდ სიმპტომატური, სხვა არაფერი უნდა მაგას. დიდი რაოდენობით სითხეები, ტვივილ გამაყუჩებელი (ან პარაცეტამოლი ან ტაილენოლი ან იბურპოფენი) და სავლებები (სოდიანი ან მარილიანი წყლის). თუ 2 კვირაზე მეტი გაგრძელდა ეს ყველაფერი, ან ავადმყოფი ტოქსიურია, ან სუნთქვის უკმარისობა ეწყება, მაშინ ექიმი უნდა სასწრაფოდ.

Posted by: EKIKULI 8 Mar 2009, 11:30
vano_t

ტკივილები საერტოდ არ აქვს უბრალოდ აწუხებს შეგრზნება რომ შეშუპებული აქვს.

აი ეს არის ნაცხის პასუხი

Posted by: nanian 9 Mar 2009, 14:59
არ ვიცი, სწორ განყოფილებაში ვწერ თუ არა, მაგრამ მაინც უნდა დავწეროsmile.gif)) 1 თვეა, რაც მაქვს მაღალი წნევები 160/100-ზე. საერთოდ ვატარებ 110-70-ს. მყავს 3 თვის შვილი. ბავშვი ბუნებრივ კვებაზეა. დღეში დაახლოების 1.5 ლიტრამდე სითხეს ვღებულობ. ვარ 32 წლის. რით გავუმკლავდე ამ წნევებს.

Posted by: shanyva 10 Mar 2009, 13:16
vano_t
როცა 23 წლის გოგოს აქვს წნევა 100-80-ზე რისი ბრალია?და რა წამალი შეიძლება მიიღოს?
მადლობააა 2kiss.gif

Posted by: vano_t 11 Mar 2009, 09:31
EKIKULI
QUOTE
ტკივილები საერტოდ არ აქვს უბრალოდ აწუხებს შეგრზნება რომ შეშუპებული აქვს.

აი ეს არის ნაცხის პასუხი

ეგ ნაცხი ნიშნავს მხოლოდ ერთ რამეს: ნაცხი დაბინძურებულია და არასწორად არის აღებული. საერთოდ, პირის ღრუდან ანცხის აღება არის არასწორი. ასეთ რამეს დასავლეთში არ აკეთებენ. ნაცხს იღებენ ტონზილებიდან და ხახიდან, ისე რომ საერთოდ არ უნდა მოვიდეს ნაცხის აღებისას რასაც იყენებ პირის ღრუსთან კონტაქტში. თანაც, არსებობს ტესტი (quickstrep), რომელიც ძალიან სწრაფია და სტრეპტოკოკულ ანტიგენზე ამოწმებს. მთელი დიაგნოზის არსიც ის არის, რომე სტრეპტოკოკი გამოირიცხოს როგორც ფარინგიტის/ტონზილიტის გამომწვევი.

მოკლედ, ეგ ნაცხი არაფერს ნიშნავს გარდა დაბინძურებული ნაცხისა.

nanian
QUOTE
არ ვიცი, სწორ განყოფილებაში ვწერ თუ არა, მაგრამ მაინც უნდა დავწეროsmile.gif)) 1 თვეა, რაც მაქვს მაღალი წნევები 160/100-ზე. საერთოდ ვატარებ 110-70-ს. მყავს 3 თვის შვილი. ბავშვი ბუნებრივ კვებაზეა. დღეში დაახლოების 1.5 ლიტრამდე სითხეს ვღებულობ. ვარ 32 წლის. რით გავუმკლავდე ამ წნევებს.
ასე რაიმეს თქმა ძნელია. "კითხვა კარდიოლოგთან" თემაში არის ბევრი რამ მაღალი წნევის დიაგნოზის და მკურანობის შესახებ. იქ შეგიძლია მოიპოვო ზოგადი ინფორმაცია. საბოლოოდ თუ ეს პრობელმა გაწუხებს ექიმმა უნდა გამოგიკვლიოს.

shanyva
QUOTE
როცა 23 წლის გოგოს აქვს წნევა 100-80-ზე რისი ბრალია?და რა წამალი შეიძლება მიიღოს?
მადლობააა
ახალგაზრდებში ასეთი წნევა უმეტესად ნორმალურია და არავითარ პრობლემას არ წარმოადგენს თუ ავადმყოფს არ აქვს დაბალი წნევის სიმპტომები: თავბრუსხვევები და გულის წასვლა. თუ ეს სიმპტომები აქვს, მაშინ უამრავი მიზეზი შეიძლება იყოს დაბალი წნევის. ყველაზე ხშირი ამ ასაკში არის არასაკმარისი რაოდენობით საკვებისა და წყლის მიღება. თუ არ აწუხებს სიმპტომები, მაშინ არაფერი არ არის საჭირო.

Posted by: *hedgehog* 13 Mar 2009, 13:29
რა მაინტერესებს იცით? თავს რო ვატრიალებ, ან თვალებს რო ვწევ გვერდით ბოლომდე, თავბრუმესხმის და მხედველობა გაურკვეველი ხდება. ეს ყველაფერი რამდენიმე წამს გრძელდება, მაგრამ მაინტერესებს საშიშია რამე? ანუ რამის დასაწყისი შეიძლება იყოს? თუ უბრალოდ გაზაფხულის ბრალია? smile.gif

Posted by: chxira 17 Mar 2009, 20:22
შარდის ანალიზზე, შარდი სახლიდან უნდა წაიღო?

რაღაც უაზრობად მეჩვენება და მართლა ესეა? spy.gif




ვერ გავიგე სად უნდა მეკითხა....


Posted by: m_i_s_h_a_tt 18 Mar 2009, 13:03
გამარჯობათ ტუ სჰეგიძლიათ ამიხსნათ რა არის ოსტეოქონდროზი , რამდენათ საშიშია ,კიდე განკურნებადია თუ არა? როგორ იკურნება და მალე იკურნება თუ არა..ანუ ყველაფერი მაინტერესებს .


წინასწარ დიდი დიდი მადლობა პასუხისთვის

Posted by: ტანკე 20 Mar 2009, 02:37
სალამი ყველას!
დახმრება მჭირდება და თუ მიპასუებთ,ძალიან დამავალებთ smile.gif

მოკლედ,მარჯვენა იდაყვის კანზე მაქვს პრობლემა,გაუხეშებულია და გაშავებული(მუქი ყავისფერი).დასისხლიანებამდე გავიხეხე მაჩალკით და საპნით,სისხლი ვიდინე,მაგრმა არ მცილდება.(ახლახან შევამჩნიე,ადრე არ მქონდა).მარცხენაზე არ მაქვს,ჩვეულებრივი ფერია,მარტო მარჯვენაზე.ზაფხული მოდის და გარეთ ვერ გავალ,თუ ეგ პრობლემა ვერ მოვაგვარე biggrin.gif მკვდარი უჯრედებია თუ რა არის,არ ვიცი,მარა ...
რამე ექსფოლიაციური(ქართულად როგორაა,ეგეც არ ვიცი) კრემები ან რამე არ იყიდება? ან რა ვიყიდო? ლიმონისა და ხახვის დადება არ მინდა,რამე უფრო თანამედროვე მედიკამენტი მინდა და ვერ მეტყვით,ვერ მირჩევთ რამეს? აფთიაქში რომ იყიდებოდეს და შველოდეს,ეეგთს user.gif

Posted by: Ni-L 20 Mar 2009, 13:54
vano_t
ორსულობის დროს ციტრუსი თუ შეიძლება?ძაან მინდებაააა

Posted by: Leoner 20 Mar 2009, 18:55
მოკელდ ფილტვების და კისრის მალების რენტგენოგრაფია გავიკეთე ტატიშვილის კლინიკაში , სულ 4 სურათი გამოვიდა რამდენად მავნებელი ჯანმრთელობისთვის რენტგენოგრაფია და შეილებოდა თუ არა გადაღება ერთ დღეს მაგდენის...
* * *
აქ შეკითXვებს ვინმე პასუხ ცეემს ? biggrin.gif

Posted by: SPC 21 Mar 2009, 22:46
დავიტანჯე უკვე, რისი ბრალია რომ სულ მცივა???

სამსახურში უკვე ერთი-ორი თანამშრომელი მეზიზღება, სულ ფანჯარას აღებენ და მყინავენ!
წითელი სიფათებით დადიან სულ სცხელათ



Posted by: rock in rose 21 Mar 2009, 23:04
SPC
QUOTE
დავიტანჯე უკვე, რისი ბრალია რომ სულ მცივა???


დაბალწნევიანი ხომ არ ხარ?

ან სისხლძარღვები გექნება შევიწროვებული
მე მასე მქონდა და სულ მციოდა, მაგრამ მკურნალობის კურსი ჩავიტარე და ახლა ჩვეულებრივად ვარ

Posted by: chxira 21 Mar 2009, 23:17
ზინატის და იუნიდოქსის მიღება ერთად
არ შეიძლება დიზბაქტერიოზს გამოიწვევსო, მართლა ესეა?

Posted by: SPC 21 Mar 2009, 23:20
rock in rose
QUOTE
დაბალწნევიანი ხომ არ ხარ?

100-60 რავიცი აბა


Posted by: rock in rose 21 Mar 2009, 23:38
SPC
QUOTE
100-60 რავიცი აბა


არც ისე დაბალი წნევა გქონია

ხელ-ფეხი ხშირად გეყინება თუნდაც მაშინ როცა თბილად გაცვია?

Posted by: SPC 21 Mar 2009, 23:51
rock in rose
QUOTE
ხელ-ფეხი ხშირად გეყინება თუნდაც მაშინ როცა თბილად გაცვია?

biggrin.gif ფეხები ამ წუთასაც რადიატორზე მიწყვია

Posted by: mangustik 22 Mar 2009, 01:22
გამარჯიბათ, ძალიან გტხოვთ მიპასუხეტ რა იწვევს შიდა ქალის მაღალ წნევას და საშიშია თუ არა? მოკლედ რამდენიმე თვის წინ მაწუხებდა თავის ტკივილი, საშინლად და დიდი ხნის განმავლობაში მტკიოდა, ვიყავი თოდუას კლინიკაში ნევროპათოლოგთან, გადამიღო თავზე იქვე კაბინეტში და შიდა ქალის წნევა გაქვსო, თვალზე გამიშვა და აღმოჩნდა რომ თვალი მქონდა გაღიზიანებული, ვსვამდი წამლებს და გამიარა, მაგრამ ახლა ისევ დამეწყო 2 დღეა და კიდევ ნევროპათოლოგმა მაგნიტურ რეზონანს თუ გადაიღებ კარგს იზამო, რა ჩანს ამ მაგნიტური რაზონანსით? ანუ რამემ დააეწვა და იმიტომ მითხრა? მიპასუხეთ რა, ძალიან მეშინია

Posted by: texasuri jleta benzoxerxit 22 Mar 2009, 02:54
QUOTE (SPC @ 21 Mar 2009, 23:51 )
rock in rose
QUOTE
ხელ-ფეხი ხშირად გეყინება თუნდაც მაშინ როცა თბილად გაცვია?

biggrin.gif ფეხები ამ წუთასაც რადიატორზე მიწყვია

ენდოკრინოლოგს ესტუმრე და ფარისებრი ჯირკვლის დაბალი ფუნქცია (ჰიპოტირეოზი) გამორიცხე

Posted by: ROWAN ATKINSON 22 Mar 2009, 14:08
....................................................................

Posted by: isida8 23 Mar 2009, 09:55
ყოველი ოდნავი გაციების დროსაც კი მიჩნდება ნახველი,დავიტანჯე sad.gif რისი ბრალი შეიძლება იყოს?მე სიგარეტს ვაბრალებ.

Posted by: Rogue 23 Mar 2009, 22:51
გამარჯობა
არ ვიცოდი სად დამეწერა ეს კითხვა და თუ შეიძლება აქ დავსვავ
ესეიგი შეიძლება თუ არა მივიღო მეთანი (ტაბლეტების სახით)
ვვარჯიშობ ბოდიბილდინგზე და ტრენერმა მითხრა იყიდე 50 ტაბლეტიო. . .
დაახლოებით როგორც ვიცი მამაკაცის ჰორმონებს შეიცავს მგონი

Posted by: shtori 24 Mar 2009, 01:52
ყურის ტკივილი შეიძლება რომ გულის ბრალი ან სისცხლის მიმოქცევის ბრალი იყოს? არც მახსოვდა ყური, უბრალოდ წამოვწექი და გულის ცემა გაძლიერდა, რიტმიდან ამოვარდა და თითქოს ყურში გირტყავსო ყოველი შეკუმშვისას, თან ხმაც ისმოდა და მოკლედ შედეგად ყური ამტკივდა. ადრე დამემართა კიდევ, რაღაც ხმაური მესმოდა ამავე ყურში, ხოდა აი ეს ხმაური იყო რაც წეღან sad.gif
როგორ მტკივაა (((
რა ჩავიწვეთო???

Posted by: lizofobi 26 Mar 2009, 17:24
ჩხირბირთვიანი ნეიტროფილები მაქვს 7 % , როცა ნორმაა 0–4 % user.gif და რა დაავადებების დროს შეიძლება იყოს მომატებული?
ხო ნუ თუ იმასაც დავამატებ რომ ბოლო 3 კვირაა სიცხე 37.1–37.3 მდე ადის და ჩამოდის. ფილტვები სუფთაა, სისხლის საერთოში კიდე მარტო ეს ნეიტროფილებია ნორმაზე მეტი sad.gif სანამ თერაპევტთან მივალ, იქნებ აქაურმა თერაპევტებმა მითხრათ რისი ბრალი შეიძლება იყოს ეს სიცხეები? დავიტანჯე უკვე...

Posted by: Kobalty 29 Mar 2009, 14:35
ინტერნეტში რაც მოძიVიე, დაზუსტებული პასუხი მაინც ვერ დავადგინე და რა მაინტერესებს: ფილტვის სიმსივნემ ფილტვის მიმდებარე ტერიტორიაზე ღრმად სუნთქვის დროს ტკივილი თუ იცის?
2 დღეა როცა ღრმად ამოვისუნთქებ ხოლმე მაშინ მტკივა ფილტვთან და რისი ბრალი შეიძლება იყოს?
ან სად და ვისთან მირჩევთ რომ მივიდე ექოსკოპიაზე?

Posted by: skoch 29 Mar 2009, 20:28
QUOTE
სად და ვისთან მირჩევთ რომ მივიდე ექოსკოპიაზე?

რესპუბლიკურში სპარტაკა ქვია ეგაც კაია და თოდუაშიც

Posted by: texasuri jleta benzoxerxit 30 Mar 2009, 02:01
QUOTE (koba4ever @ 29 Mar 2009, 14:35 )
ინტერნეტში რაც მოძიVიე, დაზუსტებული პასუხი მაინც ვერ დავადგინე და რა მაინტერესებს: ფილტვის სიმსივნემ ფილტვის მიმდებარე ტერიტორიაზე ღრმად სუნთქვის დროს ტკივილი თუ იცის?
2 დღეა როცა ღრმად ამოვისუნთქებ ხოლმე მაშინ მტკივა ფილტვთან და რისი ბრალი შეიძლება იყოს?
ან სად და ვისთან მირჩევთ რომ მივიდე ექოსკოპიაზე?

ექოსკოპიაზე მაქსიმუმ გამონაჟონი (სითზე,"წყალი",როგორც უწოდებენ)ნახო პლევრის ღრუში.
ფილტვების პათოლოგიები რენტგენოლოგიურად (მათ შორის ყველაზე კარგად კომპიუტერულ-ტომოგრაფიულად) დგინდება.

თქვენი ასაკი?
ვვარაუდობ,რომ ახალგაზრდა ხართ და ფილტვის კიბო კი არა იპოქონდროული ნევროზი გაწუხებთ. 2kiss.gif

ისე კი,ფილტვის კიბომ,პლევრაში ჩაზრდის შემთხვევაში იცის ზემოაღწერილი ტკივილი boli.gif biggrin.gif

Posted by: Kobalty 30 Mar 2009, 04:15
texasuri jleta benzoxerxit
QUOTE
თქვენი ასაკი?

24

QUOTE
ისე კი,ფილტვის კიბომ,პლევრაში ჩაზრდის შემთხვევაში იცის ზემოაღწერილი ტკივილი

ხოდა გამარჯობა შენი, მეც მანდ ვარ. biggrin.gif

skoch
მადლობა. smile.gif

Posted by: texasuri jleta benzoxerxit 30 Mar 2009, 12:29
QUOTE (koba4ever @ 30 Mar 2009, 04:15 )
texasuri jleta benzoxerxit
QUOTE
თქვენი ასაკი?

24

QUOTE
ისე კი,ფილტვის კიბომ,პლევრაში ჩაზრდის შემთხვევაში იცის ზემოაღწერილი ტკივილი

ხოდა გამარჯობა შენი, მეც მანდ ვარ. biggrin.gif

skoch
მადლობა. smile.gif

gigi.gif და დიფერენციალურ-დიაგნოსტიკურ ჭრილში მაგალითად ნეკნთაშუა კუნთების,ნეკნთაშუა ნერვებისმიერი ტკივილი რომ განვიხილოთ,არა? ეგრევე ფილტვის კიბო? თანაც უკვე პლევრაში ჩაზრდილი? givi.gif
იპოქონდრია ისეთი რამეა,რომ რენტგენოლოგიურად ვერაფერს რომ ვერ აღმოაჩენ, მაინც ვერ მოისვენებ...
მაგრამ თუ ასე ნერვიულობ, ელემენტარულად მოასმენინე ფილტვებზე თერაპევტს,ტყუილად დასხივეაბას არ გირჩევ,მაგრამ გადაიღე რენტგენოგრაფია, რომ დამშვიდდე (თუმცა მაინც ვერ დამშვიდდები givi.gif ).ექოსკოპიას,როგორც ვთქვი ფილტვენის შემთხვევაში აზრი არ აქვს...თორემ სპარტაკი მართალაც ძალიან კარგია.თუმცა არანაკლები რენტგენოლოგები გვყავს მანდ...
გამოირიცხოს ყველა ორგანული მიზეზი. მაგრამ მერე იპოქონდრიას უნდა მიეხედოს.

თორემ პო ირონიი სუძბი, რისიც ყველაზე მეტად გვეშინია, სწორედ ის გვემართება ადრე თუ გვიან fig.gif gigi.gif

Posted by: tikope 30 Mar 2009, 17:21
ყბაყურას მკურნალობაზე დაწერეთ რა რამე, გთხოვთ
ჩემს 12 წლის ბიჭს აკვს და რა უნდა გავაკეთო?

Posted by: Xoca Nasreddin 30 Mar 2009, 19:13
ბოდიში რომ გაწუხებთ, არ მეგონა ამ განყოფილებაში შემოხედვა თუ დამჭირდებოა როდისმე. ერთი კითხვა მაწუხებს, ფილტვის ციროზი თუ იკურნება, ან თუ არა რამდენ ხანში კლავს კაცს?

Posted by: melano-melano 30 Mar 2009, 20:37
გამარჯობათ! სამი წლის წინ შემთხვევით აღმოვაჩინე,რომ ყოველ ლუწ საათში სიცხე 37,2 მაქვს(მანამ კი ჩემი სიცხე დაბალი იყო -35,5),სისხლის ანალიზი ჩავაბარე,ფილტვიც გადავიღე,რევმატიულიც,არსად არაფერი არ აღმოჩნდა,გინეკოლოგმაც ჩემი სიცხე არააო.მირჩიეს თუ არ გაწუხებს ყურადღებას ნუ აქცევო. ახლახან ისევ გავიზომე და ყოველ ორ საათში უკვე 37,4 მაქვს სიცხე.ძალიან ვნერვიულობ,რა შეიძლება იყოს და ვის მივმართო?

Posted by: lizofobi 31 Mar 2009, 14:33
melano-melano
+1 მეც სულ ეგრე ვარ sad.gif აი ეხლა 37.1 ,აქვს და ზემოჩამოთვლილმა ანალიზებმა არაფერი აჩვენა. გარდა იმისა რომ ჩხირბირთვიანი ნეიტროფილები მაქ მომატებული ოდნავ sad.gif შეიძლება ნევროზი გვაქ spy.gif

Posted by: melano-melano 31 Mar 2009, 16:25
აუუუუ,აქ ექიმი არავინაა? თორემ lizofobi ,შენი არ იყოს,მასეთ დიაგნოზს მეც კი ვსვამ wink.gif არა,არც ნევროზი მაქვს,ნევროპათოლოგთანაც ვიყავი,რადგან როცა დიაგნოზს ვერ გეტყვიან ან ვირუსია,ან ნევროზი biggrin.gif

Posted by: lizofobi 31 Mar 2009, 17:22
melano-melano
ფსიქოგენური სიცხეებიო gigi.gif მაშინ უნდა გამოვეკვლიოთ სპიდზე, ტუბერკულოზზე, მონონუკლეოზზე, მუკოვისციტზე თუ რაღაც უბედურებაზე და ასე უსასრულოდ gigi.gif

Posted by: gulua1 31 Mar 2009, 17:47
ერთი ასეთი კითხვა მაქვს

შეიძლება ადამიანის მუცლის ღრუში 24 წლის განმავლობაში იყოს ჩირქის კერა და არავითად გართულებას არ იწვევდეს?

Posted by: melano-melano 31 Mar 2009, 17:52
lizofobi
სპიდზე ანალიზი ჩაბარებული მაქვს
lol.gif ტუბერკუზი არ მაქვს,სხვა დანარჩენის რა გითხრა idea.gif ისე,ფაქტია,რომ რაღაც არის,აბა სიცხე საიდან? wow.gif ეეეხ,ისევ ჯობდა სამედიცინოზე ჩამებარებინა,ხომ არ გამიხდებოდა დიაგნოზი სხვისი სახვეწარი? biggrin.gif biggrin.gif biggrin.gif
ექიმებო,თუ ხარ აქ,გაგვეცით რა პასუხიიიიიიიიიიიიიიი cry.gif cry.gif cry.gif

Posted by: lizofobi 31 Mar 2009, 18:00
melano-melano
ხო მე ტუბერკულოზი არ მაქვს და სპიდზე ანალიზი ადრე მაქვს ჩაბარებული gigi.gif თან სისხლის საერთოში როე და ლეიკოციტები ხო აჩვენებდნენ რო რაღაც ინფექციური პათოლოგიაა ორგანიზმში? baby.gif გვიპასუხეთ რა ექიმებო baby.gif

Posted by: ancho 31 Mar 2009, 20:01
მოკლედ ჩემ ახლობელს ტკივა ყური 1 კვირაა, შეტევითი საშინელი ტკივილები აქვს. ექიმთან არ მიდის (ცოცხალი თავით), მითხარით რისი მიღება ვურჩიო? დაყრუებას ესე დისტანციურად ნათქვამი მკურნალობა ჯობს მგონი. boli.gif

Posted by: melano-melano 2 Apr 2009, 21:04
QUOTE
გამარჯობათ! სამი წლის წინ შემთხვევით აღმოვაჩინე,რომ ყოველ ლუწ საათში სიცხე 37,2 მაქვს(მანამ კი ჩემი სიცხე დაბალი იყო -35,5),სისხლის ანალიზი ჩავაბარე,ფილტვიც გადავიღე,რევმატიულიც,არსად არაფერი არ აღმოჩნდა,გინეკოლოგმაც ჩემი სიცხე არააო.მირჩიეს თუ არ გაწუხებს ყურადღებას ნუ აქცევო. ახლახან ისევ გავიზომე და ყოველ ორ საათში უკვე 37,4 მაქვს სიცხე.ძალიან ვნერვიულობ,რა შეიძლება იყოს და ვის მივმართო?

ამ ტექსტს,ისევ ვდებ,გთხოვთ,მიპასუხოოოოოოოოოთ!!! cry.gif

Posted by: KABELI 2 Apr 2009, 23:08
ჩიებზე უნდა დავლიო ჰელმინტოქსი.იქნებ მიტხრატ რა დოზით და როგორ?

Posted by: la cioccolata calda 3 Apr 2009, 18:04
ancho
QUOTE
ტკივა ყური 1 კვირაა, შეტევითი საშინელი ტკივილები აქვს.

გამნადენი ხო არა აქვს,ჩიქროვანი ან ნებისმიერი?
ან კიდე 1 კვირრის წინ რა მოხდა უეცრად ხო არ დაეწყო ეს ტკივილი, ან რაიმეს აფეთქების ფონზე?

ყური ძალიან სათუთი რაღაცაა და საფრთხილო ესე რომ ესე დისტანციით ნათქვამით ვერ ჩააწვეთებ ვერაფერს

:ჩემი მოკრძალებული რჩევა-მივიდეს ოტორინოლარინგოლოგთან დროზე:

Posted by: Romina 3 Apr 2009, 22:21
melano-melano
ჩემი გურწფელი რჩევაა თავი გაანებე სიცხის ზომვას.

Posted by: la cioccolata calda 3 Apr 2009, 22:25
აქ არის ისეთი ხალხი ვისაც ძვლები ,სახსრები და კუნთებიც ეხება? smile.gif

Posted by: Romina 3 Apr 2009, 22:28
la cioccolata calda
მე მეხება mo.gif

Posted by: lila2009 3 Apr 2009, 22:45
QUOTE (Romina @ 3 Apr 2009, 22:28 )
la cioccolata calda
მე მეხება mo.gif

user.gif
მეც მაინტერესებს,
21 წლისა ვარ, და სახსრები მუდმივად მაწუხებს, რისი ბრალია??

მართლა არაფერი არ შველის? sad.gif

Posted by: Romina 3 Apr 2009, 23:16
როგორ არ შველის, უსაშველო არაფერია. უბრალოდ არსებობს ქრონიკული დაავადებები, რომლებსაც მუდმუვად მკურნალობა ჭირდება. რევმატოლოგთან ყოფილხარ? რომელი სახსრები გტკივა?

Posted by: la cioccolata calda 4 Apr 2009, 13:09
Romina
ჩემი ჩივილი არის ტკაცუნები, ვარჯიშის დროს
საერთოდ ცოტას ვვარჯიშობ ამ ბოლო დროს და დიდ ხანს ვზივარ კომპთან და ამიტომ მარჯვენა მხარი მეღლება ხოლმე,
მერე უცებ წამოვხტები და ვავარჯიშებ ხელებს ,მხრებს და ტკაცუნებს sad.gif

ანუ ვფიქრობ უნდა გავაგრძელო ვარჯიში ინტენსიურად რომ სახსრები გავამაგრო და კომპთანაც ნაკლები დრო ვიჯდე

ადრე ლექტორმა აგვიხსნა მაგრამ კარგად არ მახსოვს--რაღაც ჰაერი შედისო სახსარში რაც არ უნდა იყოსო წესითო და ის იწვევს ამ ტკაცუნსო
და ხო ვერ შემახსენებდით მექანიზმს? smile.gif

Posted by: Romina 4 Apr 2009, 20:07
la cioccolata calda
სახსარში ჰაერის შესვლაზე და შესაბამისად მექანიზმზე ნამდვილად არაფერი მსმენია smile.gif
თუ უბრალოდ ტკაცუნს გულისხმობ და არა ხრაშუნს (რომელიც ართროზის დროს არი), ეგ ხშირად თან ახლავს ხოლმე ე.წ. სახსრების ჰიპერმობილობის სინდრომს. მაგ დროს მართლა ყველაზე ეფექტური ვარჯიშია, ოღონდ არა ძალიან დიდი დატვირთვა, მსუბუქი აერობიკის მსგავსი.

Posted by: lila2009 4 Apr 2009, 21:18
QUOTE (Romina @ 3 Apr 2009, 23:16 )
როგორ არ შველის, უსაშველო არაფერია. უბრალოდ არსებობს ქრონიკული დაავადებები, რომლებსაც მუდმუვად მკურნალობა ჭირდება. რევმატოლოგთან ყოფილხარ? რომელი სახსრები გტკივა?

user.gif
რავი, უფრო მეტედ ფეხების ალბათ, სიარულის დროს, გადაგლის შემდეგ მაწუხებს.. ამინდის არევის დროსაც საშინლად მღრღნის მუხლები.. უმოძრაო ცხოვრების წესი მაქვს და ხშირად ყველა სახსრის ტკივილის შეგრძნებაც მაქვს..
ხელების, მხრების, კისერის , ბეჭებისაც,
ტკაცუნიც მუდმივად, და ხრაშუნის ხმებიც ...
ოდნავ დატვირთვის დროს- ან კიბეზე არბენის , ან პატარა სიმძიმის დაჭერის დროს-მაქვს შეგრძნება რომ , ფეხი უნდა გადამიტყდეს.. sad.gif არც ისეთი მოუქნელი და სუსტი ვარ, მაგრამ რაღაც სახსრები თუ ძვლები მაქვს სუსტი ეტყობა

რევმატოლოგთან მისვლას მირჩევთ?თუ ვისთან?smile.gif

Posted by: Romina 4 Apr 2009, 21:42
lila2009
ნუ გეშინია, სერიოზული არაფერი არ იქნება. ართროზული ტიპის გამოვლინებები ბევრ ადამიანს აქვს, ახალგაზრდა ასაკშიც კი, მათ შორის მეტეოპათიკური ტიპის (ამინდის ცვლილებაზე რო გტკივა). ზოგში მემკვიდრული წინასწარგანწყობაა, ზოგს როგორც ზემოთ დავწერე ჰიპერმობილობის სინდრომის გამო უვითარდება და პასიური ცხოვრების წესი გამოვლინებას ხელს უწყობს. ყველაზე ეფექტური ზომიერი ფიზიკური დატვირთვა და მასაჟია. რევმატოლოგთან უნდა მიხვიდე, მუხლების და კისრის რენტგენი საჭირო იქნება ალბათ.

Posted by: lila2009 4 Apr 2009, 21:59
Romina

QUOTE (Romina @ 4 Apr 2009, 21:42 )
lila2009
ნუ გეშინია, სერიოზული არაფერი არ იქნება. ართროზული ტიპის გამოვლინებები ბევრ ადამიანს აქვს, ახალგაზრდა ასაკშიც კი, მათ შორის მეტეოპათიკური ტიპის (ამინდის ცვლილებაზე რო გტკივა). ზოგში მემკვიდრული წინასწარგანწყობაა, ზოგს როგორც ზემოთ დავწერე ჰიპერმობილობის სინდრომის გამო უვითარდება და პასიური ცხოვრების წესი გამოვლინებას ხელს უწყობს. ყველაზე ეფექტური ზომიერი ფიზიკური დატვირთვა და მასაჟია.  რევმატოლოგთან უნდა მიხვიდე, მუხლების და კისრის რენტგენი საჭირო იქნება ალბათ.


ასე მოვიქცევი.
დიდი მადლობა ყურადღებისათვის.

Posted by: la cioccolata calda 4 Apr 2009, 23:45
Romina
QUOTE
სახსარში ჰაერის შესვლაზე და შესაბამისად მექანიზმზე ნამდვილად არაფერი მსმენია

ე.ი. ისევ იმას მივაკითხავ და ვთხოვ შემახსენოს smile.gif

QUOTE
თუ უბრალოდ ტკაცუნს გულისხმობ

აჰა
QUOTE
ეგ ხშირად თან ახლავს ხოლმე ე.წ. სახსრების ჰიპერმობილობის სინდრომს. მაგ დროს მართლა ყველაზე ეფექტური ვარჯიშია, ოღონდ არა ძალიან დიდი დატვირთვა, მსუბუქი აერობიკის მსგავსი.

ხოო, გაიხარე

ე.ი. ვავარჯიშებ და ეგაა smile.gif

Posted by: andriana 5 Apr 2009, 00:56
QUOTE
გამარჯობათ! სამი წლის წინ შემთხვევით აღმოვაჩინე,რომ ყოველ ლუწ საათში სიცხე 37,2 მაქვს(მანამ კი ჩემი სიცხე დაბალი იყო -35,5),სისხლის ანალიზი ჩავაბარე,ფილტვიც გადავიღე,რევმატიულიც,არსად არაფერი არ აღმოჩნდა,გინეკოლოგმაც ჩემი სიცხე არააო.მირჩიეს თუ არ გაწუხებს ყურადღებას ნუ აქცევო. ახლახან ისევ გავიზომე და ყოველ ორ საათში უკვე 37,4 მაქვს სიცხე.ძალიან ვნერვიულობ,რა შეიძლება იყოს და ვის მივმართო?

ეტყობა ეს საკმაოდ ხშირია ანალოგიური პრობლემა მეც მაქვს sleep.gif და რა უნდა გამოიკვლიო ასეთ დროს? sad.gif საშიშია? eek.gif

Posted by: kasandra 6 Apr 2009, 15:31
რისი ბრალია ტკივილი მარჯვენა ფერდსქვეშ რომელსაც ახლავს გულისრევა? მთლად ფერდქვეშაც არაა, უფრო ქვემოთ. ხელის დაჭერის დროს ტკივილი არ ძლიერდება, არც ელვისებური ტკივილია. თან ახლავს კიდურების კანკალი. ძალიან გთხოვთ მიპასუხეთ რა.

Posted by: Kaifistka2 7 Apr 2009, 22:26
ანტიბიოტიკების გაკეთება მიწევს ceftriaxone-ის და 4 გრამი ლიდოკაინი მოყვება, მაგრამ მაინც ძალიან მტკივა ხოლმე, გაკეთების მეორე დღესაც კი ხოდა როგორ შეიძლება რომ არ მეტკინოს ხოლმე ან რით გავიჩერო ტკივილი? sad.gif და ეს კითხვა აქ უნდა დავსვა თუ სადმე სხვაგან?

Posted by: LULA_QABABI 8 Apr 2009, 07:28
QUOTE (Kaifistka2 @ 7 Apr 2009, 13:26 )
ანტიბიოტიკების გაკეთება მიწევს ceftriaxone-ის და 4 გრამი ლიდოკაინი მოყვება, მაგრამ მაინც ძალიან მტკივა ხოლმე, გაკეთების მეორე დღესაც კი ხოდა როგორ შეიძლება რომ არ მეტკინოს ხოლმე ან რით გავიჩერო ტკივილი? sad.gif და ეს კითხვა აქ უნდა დავსვა თუ სადმე სხვაგან?

ჯერ ის თქვი (თუ ითქმება) რატომ დაგინიშნეს ცეფტრიაქსონი;
შესაძლოა საერთოდ ტაბლეტირებული ანტიბიოტიკიც საკმარისი იყოს; თუ არადა პმ-ში მომწერე დიაგნოზი;

Posted by: kasandra 8 Apr 2009, 13:31
QUOTE
რისი ბრალია ტკივილი მარჯვენა ფერდსქვეშ რომელსაც ახლავს გულისრევა? მთლად ფერდქვეშაც არაა, უფრო ქვემოთ. ხელის დაჭერის დროს ტკივილი არ ძლიერდება, არც ელვისებური ტკივილია. თან ახლავს კიდურების კანკალი. ძალიან გთხოვთ მიპასუხეთ რა.


ამაზე არავინ მიპასუხებს? user.gif

Posted by: addicted 8 Apr 2009, 15:38
kasandra
ნაღვლის ბუშტის ტკივილს გავს sad.gif მე მასე დამეწყო ქოლეცისტიტი, მაგ ნიშნებით. დააკვირდი რა დროს გტკივა, ჭამის შემდეგ? თუმცა ჯობია ექიმს დაელოდო და ჩემი დილეტანტური პასუხით გული არ გაიხეთქო smile.gif


Posted by: Kaifistka2 8 Apr 2009, 16:25
LULA_QABABI
რატომ და ლაიმას ბორელიოზი მაქვს უკვე მე9 თვეა, გვიან დამისვეს ლაიმაზე დიაგნოზი, მანამდე ვსვამდი და მიკთებდნენ ანტიბიოტიკებს, მაგრამ უმიზნოდ, ამ პერიოდში შემთხვევითობის წყალობით დამალევინეს ამოქსაცილინი 7 დღე, მერე რაღაც დროის შემდეგ10 დღე როცეფინი იგივე ცეფტრიაქსონი და ასევე დოქსაცილინი 20 დღე, ესეც რომელიღაც პერიოდში და დანარჩენ დროს ფართო სპქეტრის ანტიბიოტიკები მიკეთეს, უბრალოდ არ იცოდნენ რა მჭირდა მერე მიკეთებდნენ 4 თვე ბიცელინს და ძალიან ცოტათი დაიკლო, მაგრამ იმდენი ანტიბიოტიკები მქონდა სხვადასხვა სიძლიერის მინაღები არ შეიძლებოდა, ახლა ბოლო პერიოდში დავლიე 20 დღე დოქსაცილინი და ახლა დამინიშნეს ათი დღე ცეფტრიაქსონი. და დამწეყო თავის ტკვილებით, სიცხით, გულზე უსაიმოვნო შეგრძნებებით და ძალიან ბევრი სხვა სიმპტომი მოქნდა ლაიმასი, ვერ ხვდებოდნენ რა იყო და ათას რაღაცაზე აირეს ანალაიზი, ახლა ლაიმას გამო პერიკარდიტი მაქვს, თავიდან უფრო მეტი აღმომაჩნდა მერე სითხე დავიშრე წამლების საშუალებით, მაგრამ ისევ ჩადგა გულში და ახლა ისევ ვმკურნალობ, მოკლედ ამოქსაცილინი ძირითადად ბავშვები ინიშნება და დოქსაცილინი და ცეფრტიაქსონი, ზუსტად მაგის წამლებია, ამ მომენტში ცეფრაიქსონი ჩემი ნომერ პირველი წამალია იმიტომ რომ მე დაგვიანებული სტადია მაქვს და არ დავაჩქარე მისი მკრნალობა მერე ამის ორგანიზმიდან გამოდევნა რთული იქნება. მოკლედ მე ის მინდა გაიგო რომ ერთი ადგილი საშინლად მტკივა ხოლმე ამ ნემსის გაკეთების მერე და როგორ შეიძლება ისე გავაკეთო რომ არ მეტკინოს? sad.gif ან თუ არის რაიმე კრემი, რომ წავისვა და აღარ მტკიოდეს.... ეს ნემსი ვენაშიც კეთდება, მისი ანოტაცია წავიკითხე, მაგრამ ვენაში უარესია გართულებები შეძლება მოყვეს ამიტომ კუნთში გაკეთება ჯობია და იქნებ რაიმე მირჩიოთ რომ არ მეტკინოს... user.gif

Posted by: Romina 9 Apr 2009, 09:21
Kaifistka2
სად ცხოვრობ? ლაიმის დაავადება (იგივე ბორელიოზი) ძალიან იშვიათად გვხვდება საქართველოში. ტყეში ისეირნე სადმე? რა სიმპტომებით დაიწყო, ზომაში მზარდი გამონაყარი გქონდა?

Posted by: Kaifistka2 9 Apr 2009, 12:19
Romina
შენ წარმოიდგინე არც ისე იშვიათი ყოფილა უკვე, ჩემმა ექიმმა მითხრა 4 მაინც მაკითხავს ამ მიზეზითო თვეშიო (პაატა წიკლაურია ჩემი ექიმი). სად მიკბინა და სოფელში, ერთი კვირით ვიყავი, სახლს პლოშადი აქვს და იქ ვიჯექი და ტილოს თხელი შარვალი მეცვა და ზევიდან მიკბინა, ძალიან მწარედ, მაგრამ მაშინ ვერ დავინახე რამ მიკბინა და ნაკბენის შესახებ მალე მივივიწყე, იმიტომ რომ არ იყო დიდი, საშუალოდ იყო ნაკბენის ირგვლივ რგოლად აწითლებული და მალევე გამიარა ასე 2 კვირაში აღარაფერი მეტყობოდა, დაჟე უფრო ნაკლებ დროში და ტერიტორიულად მიკბინა სამტერდიის რაიონში, კერძოდ სოფელ მარანში, იმერეთში. და დამეწყო დაბალი სიცხეებით და დღემდე მიგრძელდება, 38ს არ ამცდენია, დაახლოებით მაქვს ხოლმე 37.0 - 37.5. ასევე მქონდა ძალიან ძლიერი თავის ტკივილები. მე თავის შიდა ქალის წნევაც მაქვს და მეგონა თავიდან მისი ბრალი იყო თავის ტკივილი. ახლაც მაქვს ხშირად ლაიმას გამო ტკივილი, იმიტომ რომ თავის შიდა ქალის წნევას განსხვავებული ტკივილი აქვს ცოტა. ასევე გული მქონდა ხოლმე ძალიან ცუდად და დაბალი წნევა, რომელიც ახლაც ხშირად მაქვს ხოლმე, მაგალითად 70 40ზე ან 90 50ზე, ხანდახან ესეც მაქვს ხოლმე, ისე მაღალ წნევას არ ვატარებ. პლიუს ამას მქონდა ზურგის ალაგ-ალაგ ყრუ ტკივილები და ეგონათ ფილტვები და გადამიღეს ფილტვებზე და არაფერი არ აღმოჩნდა რა თქმა უნდა. მქონდა ასევე კისრის კუნთების გაკავება, სინათლისადმი აღგზნებადობა, ვერ ვიტანდი რაღაც პერიოდი შუქს და ვთხოვდი ჩაექროთ, გამძაფრებული ყნოსვა კიდევ, სუნები მაწუხებდა და ასევე ხმაური, ასევე ძილის დარღვევები მქონდა, უფრო სწორად საერთოდ მიჭირდა დაძინება და თეთრად გამითენებია ღამეები, ასევე მქონდა დეპრესია უმიზეზო და ეს სიმპტომებიც იყო თავიდან, მაქვს ცოტათი მეხსირების დარღვევა და აი მიჭირს კონცენტრაცია. საუბრის დროს განსაკუთრებით და უცებ შეიძლება დამავიწყდეს რაზე ვლაპარაკობდი. ნუ ზოგადი მეხიერების პრობლემები იმდენად არ მაქვს, უფრო აი კონცენტრაცია მიჭირს და მეხსიერების გასაუმჯობესებელი წამლები თავს მატკიებს და ვერ ვსვამ მაგიტომ. ახლა ამ დროისთვის ბევრი სიმპტომი აღარ მაქვს, მაქვს ხანდახან ტავის ტკივილები, მაგრამ ისე ძლიერ აღარ და გამაყუჩებლებს ვსვამ, ნიმესილს მაგალითად, რომელიც სიცხესაც წევს ან მაგენოლს, ეს ახლა მოვსინჯე და ესეც მშველის, , გული მაწუხებს სითხის გამო, განსაკუთრებით მაშინ თუ მაქვს სიცხე და ნუ სიცხეც მაქვს პერიოდულად და ნუ მასმევენ კიდე წამლებს გულში სითხის დასაშრობად. და წამლების მერე დიდ უკეთესობას ვერ ვხედავ სიმართლე რომ ვთქვა. თავი ისევ მტკივა, გულში სითხე ისევ გროვდება... და ახლა ათი ნემსი მაქვს ცეფტრიაქსონი დანიშნული, და მერე მივდივარ ანალიზებზე უნდა ვნახო ბორელიას დონემ დაიკლო თუ არა სისხლში.

ხოდა მირჩიეტ ვინმემ ბოლოსდაბოლოს ნემსზე sad.gif

Posted by: Romina 9 Apr 2009, 13:38
Kaifistka2
QUOTE
ჩემმა ექიმმა მითხრა 4 მაინც მაკითხავს ამ მიზეზითო თვეშიო


sad.gif sad.gif ნამდვილად არ ვიცოდი ეს. უბრალოდ ერთი უზუსტობაა, მე როგორც ვიცი ტკიპი 2-3 დღე უნდა იყოს კანზე მიმაგრებული რომ დაავადება გამოიწვიოს. თან დაავადების პირველ ეტაპზე ერითემა ზომაში მატულობს და ნაკბენიდან ზოგჯერ დიდ მანძილზე ვრცელდება, შემდეგ თუ არ იმკურნალა ადამიანმა ჩნდება სხვა სიმპტომები, მათ შორის ქრონიკული მენინგოენცეფალიტის (რაც შენ ჩამოთვალე). ამ ეტაპზე დოქსაციკლინის ან ცეფტრიაქსონის 1 თვიანი კურსი ინიშნება, მე ასე მახსოვს. ანტიბიოტიკების მასე ცვლა და ხანმოკლე კურსის ჩატარება არ შეიძლება. ნუ გეშინია, სრულიად განკურნებადი დაავადებაა. შენ შეკითხვაზე პასუხი არ მაქ, ნემსთან დაკავშირებით sad.gif იქნებ სხვამ გიპასუხოს.

Posted by: zanduki 9 Apr 2009, 14:06
ე.ი მაინტერესებს,
1. რა დადებითი და უარყოფითი შედეგი მოყვება თაფლიანი წყლის დალევას დილით უჭმელზე
2. რამდენად მართალია, რომ სამი დღის განმავლობაში ბორჯომისა და ხლებცის მიღება წმენდს ორგანიზმს, რისგან წმენდს და რა უარყოფითი შედეგი შეიძლება მოყვეს და რა დადებითი, კონკრეტულად (იმის გარდა, რომ შეიძლება დაიკლო წონაში)

წინასწარ მადლობა და მირჩევნია სამედიცინო განათლების მქონე ადამიანმა მიპასუხოს smile.gif

Posted by: Kaifistka2 9 Apr 2009, 18:01
Romina
QUOTE
მე როგორც ვიცი ტკიპი 2-3 დღე უნდა იყოს კანზე მიმაგრებული რომ დაავადება გამოიწვიოს. თან დაავადების პირველ ეტაპზე ერითემა ზომაში მატულობს და ნაკბენიდან ზოგჯერ დიდ მანძილზე ვრცელდება, შემდეგ თუ არ იმკურნალა ადამიანმა ჩნდება სხვა სიმპტომები, მათ შორის ქრონიკული მენინგოენცეფალიტის (რაც შენ ჩამოთვალე).

არვიცი, მე მიკბინა შარვალზე ვიჯექი, ბარძაყზე ქვემოდან ანუ იატაკის მხარეს და თან მიკბინა შარვალზე და როგორც კი მიკბინა ძალიან მემწარა, ფუტკრის ნაკბენივით მწარე იყო და უცებ მოვისვი ხელი და ვნახე და აღარაფერი აღარ იყო, ანუ ტკიპა არ მომკრობია, შეიძლება შარვალმა გადამარჩინა სწორედ, ამიტომაც ნაკბენი დიდი არ მქონდა და ნეტავ მქონოდა იმიტომ რომ მიხვდებოდი რის გამო ვიყავი ცუდად, აი ძალიან ცოტათი გაწითლდა, დაახლოებით თითი რომ მოხარო საჩვენებელი და ცერა თითის შუა ნაწილთან რომ მიიტანო, რომ შეგეხედა კოღოს ნაკბენს მიამსგავსებდი და არც მიმიქცევია ყურადღება მაგიტომ sad.gif ხოდა თუ გვიანი სტადიაა მაგის გარდა ბევრი რამ ჩნდება ეგ დაავადება ნერვულ სისტემას, ტვინს, სახსრებს და გულს აზიანებს, შესაბამისად იწვევს გართულებებს, ნერვული სისტემის დაზიანებისას ადამიანს შიზოფრენიაც კი შეიძლება დაემართოს, ტვინს როცა აზიანებს მეხსიერების გაუარესებას უჩივის ადამიანი, გულზე სხვადასხვა გართულებაა, ჩემს შემთხვევაში პერიკარდიტი მივიღე, სახსრების დაზიანებისას იწვევს ართრიტს და ნუ მოკლედ მართლა საშინელი რამეა მე მხოლოდ ის მამშვიდებს, რომ რაღაც პერიოდში მივიღე ის ანტიბიოტიკები რაც ამისთვის იყო განკუთვნილი და მთელი ამ დროის გნმავლობაIშ ფართო სპექტრი ანტიბიოტიკებს მასმევდნენ, გარდა ბორელისისა ტკიპის ემენციფალიტეტი მაქვს, რიკ კონორი თუ რაღაც ესე ეძახიან, მე არ ვარ მედიცინაში გარკვეული პირი და მისი სახელი ზუსტად არვიცი. რა ინფორმაციასაც ვწერ ეს თვითონმოვიძიე ინტერნეტში მას შემდეგ რაც ავად გავხდი.
QUOTE
ამ ეტაპზე დოქსაციკლინის ან ცეფტრიაქსონის 1 თვიანი კურსი ინიშნება, მე ასე მახსოვს. ანტიბიოტიკების მასე ცვლა და ხანმოკლე კურსის ჩატარება არ შეიძლება.

დოქსაცილინი მე მივიღე უკვე 20 დღე მანამდე, ახლა ვარ 10 დღიან ცეფტრიაქსონზე ანუ ეს ერთი თვე, მარამ მე მკურნალობას არ ვწყვეტ ვაგრძელებ იგივე ანტიბიოტიკებს სანამ არ დაიწევს დონე სისხლში და პლიუს ამას მერეც უნდა გაგრძელდეს მკურნალობა, ეგ ექიმი სწორად მკურნალობს, მე ინტერნეტში მოვიძიე ერთ-ერთი საკამოდ ცნობილი ექიმის გვარი მაგ ლაიმას დაავადების მკურნალობაში და აქვს თავისი საიტი სადაც განთავსებულია წამლების დასახელებები და დოზები დაავადების გართულებებისდა მიხედვით. ჩემი ექიმი სწორად მმკურნალობს. მაგ ექიმს ძალიან რთულად მივაგენი და მან დამისვა დიაგნოზი ლაიმაზე, ისეთი რთულად გამოსაცნობი დაავადება, რომ მე ვნახე სხვა ხალხის ისტორიები ინტერნეტში და უცხოელები არიან ძირითადად და უცხოელი ექიმები ვერ უდგენდნენ ზოგს 8 წელი ქონდა, ზოგს ორი და ვერ ხვდებოდნენ რა ჭირდათ, მადლობელი ვარ ამ კაცის რომ სწორად მიმიხვდა რაც მჭირდა და მანამდე მომატყუეს სეფსოლოგებმა, დამისვეს სეფსის დიაგნოზი, ვიწექი სეფსის ცენტრშიც, ასევე გულის კლინიკაში ეგონათ გულის პთოლოგია პერიკარდიტი, მაგრამ სითხეს რომ ვერ აშრობდნენ და სისხლში მომატებული იყო სტრეპტოლიზინის დონე მერე მიხვდნენ რომ ეს არ იყო გულის ბრალი. მანამდე კიდე ნერვოპათოლოგთან ვმკურნალობდი თავზე და ნერვოპათოლოგმა გამაგზვნა გულის შესამოწმებლად, იმიტომ რომ ძალიან არეული პულსით და დაბალი წნევით მივედი მასთან და პლიუს სიცხით.
ეს იმ ექიმის ვებ საიტია სადაც წერია მკურნალობა და წამლების ზუსტი სახელები და ცეფტრიაქსონს თვითონ ანოტაციაში უწერია რომ 14 დღეზე მეტი ერთ ჯერზე არ შეიძლება და უმაღლესი დოზა 2მგ. დღე-ღამეში, ლაიმას მკურნალობის დროს.
http://www.drerniemurakami.com/main/page_clinical_facts_on_lyme_treatment.html
QUOTE
ნუ გეშინია, სრულიად განკურნებადი დაავადებაა. შენ შეკითხვაზე პასუხი არ მაქ, ნემსთან დაკავშირებით sad.gif იქნებ სხვამ გიპასუხოს.

sad.gif sad.gif sad.gif
* * *
zanduki
QUOTE
1. რა დადებითი და უარყოფითი შედეგი მოყვება თაფლიანი წყლის დალევას დილით უჭმელზე

დიაბეტიკებისთვის არ ვარგა მინუსი ეგ აქვს და საერთოდ თაფლიანი წყალი კარგია მეხსიერებისთვის, როგორც ვიცი, ისევე როგორც შაქრიანი წყალი, მაგრამ თაფლიან წყალზე ამბობენ აძლიერებს იმუნიტეტსო, მაგრამ რამდენად მართალია არვიცი. და წყალი კარგია დილით საერთოდ უზმოზე კუჭისთვის, ოღონდ ნახევარი საათით ადრე ჭამამდე, ჩაწმინდავს ყველაფერს.
QUOTE
2. რამდენად მართალია, რომ სამი დღის განმავლობაში ბორჯომისა და ხლებცის მიღება წმენდს ორგანიზმს, რისგან წმენდს და რა უარყოფითი შედეგი შეიძლება მოყვეს და რა დადებითი, კონკრეტულად (იმის გარდა, რომ შეიძლება დაიკლო წონაში)

წონაში დაკლების ეს მეთოდი ძალიან ცუდია, იმიტომ რომ შეიძლება ორგანიზმი გამოგიფიტოს, თუნდაც სამდღიანმა და მინერალები მარტო ორგანიზმს ვერ უშველის და ბორჯომის, ნაბეღლავის, საირმის თუ ლიკანის მიღება არ შეიძლება დიდი რაოდენობით არაფრით, კუჭის გარსისთვის არის ძალიან ცუდი მე როგროც ვიცი და თვითონ კურორტზე ჩასული დამსვენებლებისთვისაც არ შეიძლება დღეში ერთი ჭიქაზე მეტი. და ორგანიზმს ეს ვერ გაწმინდავს, შეიძლება გატვირთვის დღის მოწყობა რაც სასურველია ადამიანმა გააკეთოს კვირაში ერთხელ მაინც, ამ დროს უნდა ჭამო მარტო ერთი სახის საკვები მაგალIტად საზამთრო მთელი დღე და კეთდება მსგავსი ტიპის განტვირთვის დღეები, კუჭის მოქედების მოწესრიგება და დაკლება თუ გინდა გირჩევ შენს რაციონში გერკულელის ფაფა ჩართო, ძალიან სასარგებლოა ორგანიზმისთის და არც მაღლი კალორიულობის. და დიეტას რაც შეეხება არ უნდა იყოს ძალიან მკაცრი და არ უნდა ეფუძნებოდეს შენი ორგანიზმის გამოფიტვას და უნდა დანიშნოს ენდკრინოლოგმა და ასვე უნდა მიახმარო შენი ფიზიკური აქტივობაც. და რაც ყოველდღიურად დაგეხმარება კალორიების წვაში და თან ძალიან გემრიელია არის თუნდაც გრეიფუტი, ერთი ცალი გრეიფუტი 800 კალორიას წვავს, თვითონ 38 აქვს. და გრეიფუტის ჭამისას არ მიირთვა თეთრი კანი, ს ხდის მერე მწარეს, ისე ძალიან ტკბილია.
ხო და მე არ ვარ ექიმი, იმედია ჩემს ნათქვამში შესწორებებს დაამატებენ ექიები ან სულ სხვა რამეს გირჩევენ და ეს შეკიტხვები ძირთიადად უნდა დაუსვა ენდოკრინოლოგს. მე მგონი არის აქ ეს განყოფიელბა, აპინულია, შეკითხვები ენდოკრინოლოგს. smile.gif

Posted by: zanduki 9 Apr 2009, 18:55
Kaifistka2
გაიხარე მადლობა შენ smile.gif

Posted by: mangustik 10 Apr 2009, 15:36
დაახლოებით ერთი თვეა სიცხეს მაძლევს ხოლმე, 37 ან 37,2. ერთი საათის მერე აღარ მქონდა ხოლმე, მერე ისევ ამიწევდა. ერთი 2 დღეა მთელი დღე 37 მაქვს, სხვა არაფერი მაწუხებს, სისხლის საერთო ანალიზში ყველაფერი ნორმალურად იყო, რისი ბრალი შეიძლება იყოს და რომელ ექიმთან უნდა მივიდე? დამეხმარეთ რააააა user.gif

Posted by: marikuza78 10 Apr 2009, 17:09
მყავს ორი თვის გოგონა. ექოსკოპიური გამოკვლევით დაუსვეს მარცხენა მენჯ-ბარძაყის სახსრის დისპლაზია. აცეტაბულური ინდექსი მარცხენა სახსრის არის ალფა-52 და ბეტა-56, მარჯვენასი ალფა-60 და ბეტა-54. ისახება ბარძაყის თავის წერტილოვანი გაძვალების ბირთვი. ინსპექციით ნაოჭები სიმეტრიულია, კიდურები სიგრძეში ტოლია, სახსრის გადაშლისას ტკაცუნი არ აღინიშნება, მაგრამ მარცხენა მენჯ-ბარძაყის სახსარში ოდნავ არის გადაშლის შეზღუდვა. ორთოპედთან აუცილებლად მივიყვან, მაგრამ მანამდე იქნებ დამეხმაროთ და გამაგებინოთ როგორი მკურნალობა იქნება საჭირო. ინტერნეტი გადავქექე და პირველადი მკურნალობა ამ ასაკში პავლიკის ქამრებიაო. პედიატრმა თქვა ალბად მასაჟები საკმარისი იქნებაო. არ მინდა არასწორად ვუმკურნალოთ თავიდანვე და მერე დაგვიანდეს. წინასწარ გიხდით მადლობას!

Posted by: Nuna 11 Apr 2009, 10:39
გამარჯობა, თრომბოფილიის შესახებ შეგიძლიათ რამე ინფორმაცია მომაწოდოთ? მაინტერესებს როგორ მკურნალობენ, თუ არის საქართველოში მეთოდები დანერგილი და საერთოდ რა იწვევს ამ დაავადებას და რამდენად ექვემდებარება მკურნალობას. დიდი მადლობა წინასწარ

Posted by: texasuri jleta benzoxerxit 11 Apr 2009, 14:10
Kaifistka2
დასაწყისში უსისტემოდ,მაგრამ მაინც მიღებულმა ანტიბიოტიკებმა ცოტა წაშალა დაავადების კლასიკური სურთი. მართლა გაგიმართლა ,ისეთი ექმინი შეგხვდა. ჯერ ცოტა დრო გავიდა იმისთვის,რომ შეუქცევადი და მძიმე გართულებები მიიღო,ასე რომ მოითმინე ცოტა ხანი.აუცილებლად განიკურები.

Posted by: Nuna 11 Apr 2009, 14:12
texasuri jleta benzoxerxit
იქნებ მეც მიპასუხო რა, ჩემსმეგობარს დაუდგინეს თრომბოფილია და მაგის ბრალია, რომ მუცელი მოეშალა sad.gif

Posted by: Kaifistka2 11 Apr 2009, 19:17
texasuri jleta benzoxerxit
sad.gif მე9 თვეა ვითმენ, რომც არ მოვითმინო სხვა რა გზა მაქვს
cry.gif

Posted by: Ze Moreira 12 Apr 2009, 16:35
აუ ხალხო ესეთი რაღაც მჭირს
მარჯვენა ხელის თითებზე გამომდის კოღოს ნაკბენივით რაღაცეები
მისივდება და მეფხანება ძალიან
განსაკუთრებით ცხელი წყალი რომ მომხვდება მაშინ
ერთგან რომ ჩამიცხრება სხვაგან გამომდის
მთელი ზამთარი ასე მაქ, სიცივის ალერგია მეგონა, მაგრამ მარტო ცალ ხელზე მცივა?
ეხლა შევამჩნიე კიდევ რო ფრჩხილები მეკუთხება რაღაცნაირად, არ ვიცი მაგის ბრალი თუა
რა შეიძლება მჭირდეს?

Posted by: la cioccolata calda 12 Apr 2009, 17:51
ისა პატივცემულო კოლეგებო, მეგობრებო და თანაფორუმელებო smile.gif

ხანშიშესული ქალბატონი სახსრების პრობლემებით (ტკვილებით) სად მივიყვანო, ვისთან?

(+ შარდის ანალიზით საკმაოდ ბევრი მარილია, რამოდენიმეჯერ უმკურნალა ამ პრობლემას)

რას მირჩევთ?

Posted by: Doloress 12 Apr 2009, 20:52
მემგონი აქ უნდა ვიკითხო:

ყელი მტკივა, სუსტად ვარ ზოგადად და ცოტ-ცოტას მამტვრევს. ხანდახან მახველებს, მაგრამ ფილტვებიდან არა, ყელიდან.

მტკივა ბეჭები და ვერ გავიგე რატომ..

ფეხმძიმედ ვარ და მხოლოდ მცენარეულ საწუწნ აბებზე და ლიმნიან ჩაიზე ვარ ჯერ.
სიცხე არ მაქვს.
მაგრამ ეს ზურგის ტკივილი არ მომწონს.
ფილტვებთან შეიძლება იყოს რამე კავშირში? baby.gif

არც სურდო მაქვს და არც გრიპის სხვა სიმფტომები.

Posted by: shvancki 14 Apr 2009, 21:06
სისხლის საერთო ანალიზში როე და ლეიკოციტები თუ ნორმაშია, შეიძლება ვივარაუდოთ რომ ორგანიზმში რამე სერიოზული ქრონიკული ინფექციური დაავადება არაა? კერძოდ, აივ ინფექცია.ნუ ხომ უნდა აჩვენოს რამე ცვლილება სისხლმა თუ შიდსი მაქვს?

Posted by: texasuri jleta benzoxerxit 14 Apr 2009, 22:50
QUOTE (shvancki @ 14 Apr 2009, 21:06 )
სისხლის საერთო ანალიზში როე და ლეიკოციტები თუ ნორმაშია, შეიძლება ვივარაუდოთ რომ ორგანიზმში რამე სერიოზული ქრონიკული ინფექციური დაავადება არაა? კერძოდ, აივ ინფექცია.ნუ ხომ უნდა აჩვენოს რამე ცვლილება სისხლმა თუ შიდსი მაქვს?

აივ ინფიცირებულს როგორც წესი დასაწყისში არანაირი ცვლილება არ აღენიშნება სისხლის საერო ანალიზში.

ასე,რომ მიზეზი არ გაქვს დასამშვიდებლად. თუ ეჭვი გეპარება,გაიკეთე სათანადო ანალიზი. თუ ანტისხეულებს იკვლევ,საეჭვო ინფიცირების მომენტიდან (ინტრავენული მანიპულაცია,სექსი...) გასული უნდა იყოს 3 თვე.
თუ PCR-ი იკეთებ (რაც უფრო სპეციფიკურია,მაგრამ ასევე ბევრად უფრო ძვირი)-1 კვირა boli.gif

Posted by: Remediosi 15 Apr 2009, 00:59
ტუჩები მიბუჟდება საშინლად და მტკივა. მთელი სისხლი გროვდება ტუჩებში.

რის ექიმთან უნდა მივიდე?

Posted by: anonymouska 15 Apr 2009, 01:05
Doloress წადი გენაცვალე გაესინჯე, მაინც სიფრთხილეს თავი არ სტკივაოო

2kiss.gif

Posted by: misha-htc 15 Apr 2009, 01:25
უცნაური რაღაც მჭირს- ძილის დროს მეღვიძება თითქოს რაღაც საშიში სიზმარი მენახოს თუმცა სიზმარს არ ვხედავ. ვალერიანის ტაბლეტებმა ცოტა მიშველა მაგრამ რო აღარ დავლიე ისევ დამიწყო. წესიერად ვერ დამიძინია სანამ ჩამთვლემს მეღვიძება. იქნებ რამე მირჩიოთ.

Posted by: giorgi33 15 Apr 2009, 20:29
ორგანიზმში რკინის და ტრანსფერინის სატურაციის მომატება რისი ბრალი შეიძლება იყოს?

Posted by: TETISA 16 Apr 2009, 12:03
ექიმებო, ცხვირიდან სისხლდენა გამიხშირდა ამ ბოლო 2 თვის მანძილზე. და რამ იცის? A ვიტამინის ნაკლებობაა? და რა შეიცავს ამ ვიტამინს?

Posted by: womanly 16 Apr 2009, 23:14
ექიმები არიან მგონი ამ პოსტში უმრავლესობა ხოდა დამისვით ეხლა ონლინე დიაგნოზი სანამ არ გავრეკე rolleyes.gif
მარცხენა ხელის საჩვენებელი თითი პერიოდულად მისივდება და მტკივა თითქოს მიბუჟდება კიდევაც რა შეიძლება რო მჭირდეს? ისე ალერგია მქონდა ერთი 2 თვის წინ , გრიპიც. მითხარით რამე რა არ დამტოვოთ უკომენტაროდ

Posted by: SPC 17 Apr 2009, 10:56
ეს დღეებია წნევა სულ 100-80, ზე მაქვს, სხვაობა მცირეა ხო , ლამის ერთი დარტყმაა ხოლმე biggrin.gif
ცუდია ეს თუ დავიკიდო?

Posted by: texasuri jleta benzoxerxit 17 Apr 2009, 12:04
QUOTE (SPC @ 17 Apr 2009, 10:56 )
ეს დღეებია წნევა სულ 100-80, ზე მაქვს, სხვაობა მცირეა ხო , ლამის ერთი დარტყმაა ხოლმე biggrin.gif
ცუდია ეს თუ დავიკიდო?

ისე ხომ არ ხდება ხოლმე,რომ "ზედამ" დაიწიოს ძალიან, "ქვედამ" ("გულის წნევამ" gigi.gif ) პირიქით აიწიოს და "ზედას" გაუსწროს? givi.gif givi.gif

Posted by: SPC 17 Apr 2009, 15:55
texasuri jleta benzoxerxit
QUOTE
ისე ხომ არ ხდება ხოლმე,რომ "ზედამ" დაიწიოს ძალიან, "ქვედამ" ("გულის წნევამ"  ) პირიქით აიწიოს და "ზედას" გაუსწროს? 

lol.gif lol.gif lol.gif ისე ეგრეც შეიძლება smile.gif)

Posted by: Blaugraner 20 Apr 2009, 19:10
იქნებ მიპასუხოთ მწვავე ქსუდატიურ ოტიტს და მწვავე ლარინგიტს რა სიმპტომები ახასიათებს?

1. შეიძლება თუ არა პაციენტი ამ დროს ვერ ხედავდეს? (ტკივილისგან ან სხვა მიზეზეით)
2. შეიძლება თუ არა რომ არ ესმოდეს?
3. შეიძლება თუ არა რომ გული წაუვიდეს?
4. ადექვატური კომუნიკაცია ვერ შეძლოს?

რა რეკომენდაცია ეძლევა ამ დროს პაციენტს? (დაწოლა, მოძრაობის შეზღუდვა და ა.შ.

წინასწარ გმადლობთ კვალიფიციური პასუხებისთვის

Posted by: GITANA 22 Apr 2009, 14:28
სიმფიზის ძვლების ტკივილი რამ იცის?

Posted by: pteranouka 23 Apr 2009, 00:33
womanly
QUOTE
მარცხენა ხელის საჩვენებელი თითი პერიოდულად მისივდება და მტკივა თითქოს მიბუჟდება კიდევაც რა შეიძლება რო მჭირდეს? ისე ალერგია მქონდა ერთი 2 თვის წინ , გრიპიც. მითხარით რამე რა არ დამტოვოთ უკომენტაროდ


დაახლოებით ასეთივე რამ მჭირს. ორი თვის წინ სახსარში გამისივდა მარჯვენა ხელის საჩვენებელი თითი, მიხრა მიჭირდა, მტკიოდა. ვიყავი რევმატოლოგთან. სისხლის ანალიზი გავიკეთე, აბსოლიტურად ყველაფერი ნორმაში იყო, არც რევმატიულია და არც არაფრიო. და დამისვა დიაგნოზი რომ სიცივის ალერგია იყო. ალერგიისთვის აბები დამინიშნა.

თითი თავისით ჩამიცხრა, მაგრამ მარცხენა ხელთან შედარებით მაინც გასიებულია სახსარში, ხანდახან მტკივა, მუდმივად არის წითელი და ცივი. გავიკეთე დამატებითი ანალიზები, როგორც ექიმმა მირჩია, უფრო ფართო სპექტრის, მაგრამ ყველაფერი ნორმაშია. ეხლა ამბობს, სიცივის ალერგია რომ ყოფილიყო აქამე უნდა გაევლოო.

მოკლედ რა ვქნა აღარ ვიცი, ძალიან განვიცდი, როგორ დავიბრუნო ჩემი ლამაზი თითი? რისი ბრალი უნდა იყოს?

Posted by: Ze Moreira 24 Apr 2009, 16:00
QUOTE (Ze Moreira @ 12 Apr 2009, 16:35 )
აუ ხალხო ესეთი რაღაც მჭირს
მარჯვენა ხელის თითებზე გამომდის კოღოს ნაკბენივით რაღაცეები
მისივდება და მეფხანება ძალიან
განსაკუთრებით ცხელი წყალი რომ მომხვდება მაშინ
ერთგან რომ ჩამიცხრება სხვაგან გამომდის
მთელი ზამთარი ასე მაქ, სიცივის ალერგია მეგონა, მაგრამ მარტო ცალ ხელზე მცივა?
ეხლა შევამჩნიე კიდევ რო ფრჩხილები მეკუთხება რაღაცნაირად, არ ვიცი მაგის ბრალი თუა
რა შეიძლება მჭირდეს?

ამაზე მიპასუხეთ რა თუ იხედება ვინმე
მაწუხებს ძალიან, მექავება სულ.
და მაგრა მაქ თითები დასიებული

Posted by: pteranouka 24 Apr 2009, 19:51
ამ თემაში არავინ შემოდის ხო? spy.gif



როდინდელი კითხვებია უპასუხოდ........

Posted by: Alpinistka123 25 Apr 2009, 22:07

გამარჯობატ! ზალიან გტხოვტ დამეხმაროტ.
მტელი ზამტარი გუმელტან ვიჯექი და შევნიშნე რომ პეხზე კანი ამიჩრელდა და აგარ გადამდის. რა მეშველება? ვის მივმარტო?
ტორემ დამიმახინჯდა პეხები. წინასწარ გიხდიტ მადლობას
* * *

ჩვენი ექიმები მიტინგზე არიან ალბატ, ჩვენტვის ვერ იცლიაააააან biggrin.gif

Posted by: NewName 26 Apr 2009, 01:50
მეც დავპოსტავ და ერთ კვირაში შემოვიხედავ იქნებ ვინმემ რამე მიპასუხოს biggrin.gif
ყველას შეკითხვას ერთი ნიკით ვპოსტავ.
1.მოკლედ რამდენიმე კვირის წინ თავი მივარტყი მაგრად რაღAცას, ძალიან მეტკინა და მთელი დღე მტკიოდა. იმის მერე ხშირად მტკივა ყრუდ თავი. ერთი ორჯერ მაგრად ამტკივდა (Aდრე თავის ტკივილები ცხოვრებაში არ მქონია) რა გავაკეთო გადავიღო რამე? რა ქვია და სად გადავიღო ან რა ანალიზი ავიღო ან ვის მივმართო რომ გავარკვიო რა მჭირს.

2. ბოლო დროს ვარ სულ სუსტად, თითქოს დილას რომ ვიღვიძებ უკვე დაღლილი ვარ, ირგვლივ არაფერი არ მაინტერესებს, ყველაფერი მეზარება და ერთი სული მაქვს დღე როდის ჩაივლის საწოლამდე მივიდე და დავიძინო. ეს გრძელდება უკვე რამდენიმე წელია, ჯერ ორსულობაო, მერე მშOბიარობაო, მერე მშOბიარობის მერე იცისო, მერე ბავშვი გღლისო.. მოკლედ გავიდა 2-3 წელი და ისეთი შეგრძნება მაქვს ცხოვრება სადღაც გვერდით მიდის და მე ნახევრად მძინავს.. ხანდახან ვცდილობ თავი ხელში ავიყვანო და ისე აქტიურად ვიცხოვრო როგორც სხვები ცხოვრობენ, მაგრამ უბრალოდ ფიზიკურად არ შემიძლია, ოთახებში დავლასლასებ და თითქოს ყველაფერი მტკივა, გამოფიტული ვარ, საღამოობით ცუდად ვხედავ. სახე სულ დაღლილი მაქვს. მოკლედ ვგრძნობ რომ ასე არ უნდა იყოს ყველაფერი როგორც არის. აშკარად რაღAც მჭირს. არ ვიცი რამე დაავდების სიმპტომებია თუ ფსიქოლოგთან ვარ მისასვლელი.. რითი დავიწყო და რა გავარკვიო ისიც არ ვიცი.. ხოდა აქ ვკითხულობ. იქნებ ვინმემ დამაკვალიანოს საიდან დავიწყო თავის გამოკვლევა.

3. ჩემს ახლობელს გაუკეთეს ოპერაცია და სრულიად შემთხვევით აღმოაჩIნეს სწორ ნაწლავზე სიმსივნე. რომელიც მანამდე გამოკვლევებში (ოპერაციამდე რაც გაუკეთეს) არსად არ გამოჩნდა და არ იყო ნახსენები. ამ შემთხვევის მერე დამებადა ეს შეკითხვა,.მაინტერესებს რა გამოკვლევა უნდა ჩაიტარო (გარდა იმისა რომ გაგჭრან და შემთხვევით იპოვონ) რომ ასეთი სიმსივნე აღმოაჩIნო (სწორ ნაწლავზე) ადრეულ სტადიაში. თუ არსებობს საერთოდ რამე ასეთი გამოკვლევა.

4. მაინტერესებს რა იცით ბუასილის სკლეროზირების შესახებ. 8 საავადმყოფოში აკეთებენ ასეთ რაღAცას და აინტერესებს ადამიანს რომელიც გატანჯულია კოლონოსკოპიის კეთებით biggrin.gif იმიტომ რომ ეს საკითხი აშკარად ხშირად აწუხებს და ასეთი რამე ურჩიეს და საერთოდ ესე კეთდება თუ ახალი ხილია რამე? იმიტომ რომ პირველად გაიგო ვისაც ვკითხეთ (პრინციპშI ბევრისთვისაც არ მიკითხავს) და იქნებ აქ იცოდეთ ვინმემ რამე...

5. მაინტერესებს საშვილოსნოს ოპერაციის მერე, როდესაც ამოკვეთავენ მთლიანად ყველაფერს ყელს და ა.შ ეს იმას ნიშნავს რომ საერთოდ ვეღარ გააგრძელებ სქესობრივ ცხოვრებას? ნუ შვილს რომ ვეღარ გააჩენ ეგ თავისთავად smile.gif (მაპატიეთ თუ შეკითხვა უვიცობის განსახიერებაა: ) ამ შეკითხვას გინეკოლოგიაშIც დავაკოპირებ ისე.,

წინასწარ დიდი მადლობა ნებისმიერი პასუხისთვის.

Posted by: pteranouka 26 Apr 2009, 02:14
NewName

მე უკვე რამოდენიმე დღეა ვემოდები პასუხს და სხვა პოსტებსაც რომ შევხედე, იმედი გადამეწურა რომ აქ ვინმე გვიპასუხებს.

მე ექიმი არ ვარ, მაგრამ ვიცი რომ სისუსტე რომ გაწუხებს მაგის მიხედვა ჯობია სისხლის საერთო ანალიზით დაიწყო. ოღონდ მერე რა უნდა ქნა არ ვიცი smile.gif მოკლედ ჯერ ანალიზი გაიკეთე და ამასობაშI იქნებ ვინმე შმოვიდეს ტემაში

ბოლო კითხვას რაც შეეხება, ნპ, სექსისთვის არ აქვს მნიშვნელობა. მე ასე გამიგია rolleyes.gif

Posted by: GITANA 26 Apr 2009, 02:38
NewName

1. მიაკითხე ნევროპათოლოგს და გადაიღე ნეიროსონოსკოპია თუ საჭიროდ ჩათვალა.

2. ნევროზი, დეპრესია, ქრონიკული დაღლილობის სინდრომი ან ჰიპოთირეოზი (აქ გააჩნია სხვა სიმპტომებსაც) უნდა მიაკითხო ფსიქოთერაპევტს ან ჰიპოთირეოზის შემთხვევაში ენდოკრინოლოგს.

3. უნდა მიაკითხო პროქტოლოგ მუხაშავრიებს მამას ან შვილს რესპუბლიკურში და ისინი გეტყვიან რა გამოკვლევები უნდა ჩაიტარო.

4. იხილე მესამე კითხვის პასუხი.

5. გინეკოლოგიაში მაყვალას ჰკითხე პირადად. ყელში რას გულისხმობ საშოს თუ ცერვიკალურ არხს?


Ze Moreira

რატომ არ მიაკითხავ დერმატოლოგს, აქ ვინ გეტყვის ზეპირად რა გჭირს?

Alpinistka123

ესე მალე არ გადაგივა, დრო უნდა. თუ არადა ანგიოლოგს მიმართე.

Posted by: pteranouka 26 Apr 2009, 02:52
GITANA
მე არ მისაპსუხებ? user.gif

QUOTE
ორი თვის წინ სახსარში გამისივდა მარჯვენა ხელის საჩვენებელი თითი, მoხრა მიჭირდა, მტკიოდა. ვიყავი რევმატოლოგთან. სისხლის ანალიზი გავიკეთე, საერთო, და კიდევ რაღაც, აბსოლიტურად ყველაფერი ნორმაში იყო, არც რევმატიულია და არც არაფერიო. და დამისვა დიაგნოზი რომ სიცივის ალერგია იყო. ალერგიისთვის აბები დამინიშნა.

თითი ოდნავ ჩამიცხრა, მაგრამ მარცხენა ხელთან შედარებით მაინც გასიებულია სახსარში, ხანდახან მტკივა, მუდმივად არის წითელი და ცივი. გავიკეთე დამატებითი ანალიზები, როგორც ექიმმა მირჩია, უფრო ფართო სპექტრის, კრიო და კიდევ ადნმ თუ რაღაც ამდაგვარი, გადავიღე რენდგენი, მაგრამ ყველაფერი ნორმაშია. ეხლა ამბობს, სიცივის ალერგია რომ ყოფილიყო აქამე უნდა გაევლოო.

მოკლედ რა ვქნა აღარ ვიცი, ძალიან განვიცდი, როგორ დავიბრუნო ჩემი ლამაზი თითი? რისი ბრალი უნდა იყოს?


Posted by: GITANA 26 Apr 2009, 03:10
pteranouka

მოდი აბა შენც კაციტაძესთან მიდი მაგ ანალიზებსი პასიხებით. დერმატოლოგია, კონსტიტიციის ქუჩაზე კანისა და ვენ. დაავადებების დისპანსერში.

Posted by: pteranouka 26 Apr 2009, 10:20
pteranouka

მადლობა smile.gif

კაციტაძე კი ვიცი ვინც არის, მაგრამ ეგ დერმატოლოგია, მე ლო სახსარი მაქვს გასიებული. შეიძლება ეს ორი რამ ერთმანეთთან კავშირში იყოს?

Posted by: GITANA 26 Apr 2009, 10:50
pteranouka

თუ სიცივის ალერგიის ალბათობაც იყო, რატო არ შეიძლება კნაის პორბლემაც იყოს. მითუმეტეს თუსხვა ყველაფერი გამოირიცხა. თან ამბობ მთელი თითი წითელი მაქვსო. ყოველშემთხვევაში მაგ ანალიზებიკთ მიხვდება მაინც სად უნდა გაგიშვას.

Posted by: ბაჭი24 26 Apr 2009, 10:57
ხელ-ფეხი მიბუჟდება ხშირად, ფეხის კუნთები მტკივა, მეჭიმება, და კიდე ამ ბოლო დროს სუ მწყურია

მეც მიპასუხეთ

Posted by: NewName 26 Apr 2009, 11:03
pteranouka
გამიმართლა აშკარად biggrin.gif
GITANA
დიდი მადლობა პასუხებისთვის !!
QUOTE
3. უნდა მიაკითხო პროქტოლოგ მუხაშავრიებს მამას ან შვილს რესპუბლიკურში და ისინი გეტყვიან რა გამოკვლევები უნდა ჩაიტარო.

კონკრეტულად ამათ? ანუ სხვა პროქტოლოგთან რომ მიხვიდე ვერ გეტყვის? რამე კონკრეტული სპეციალიზაცია აქვთ თუ უბრალოდ მირჩიე როგორც კლარგი ექიმები.

QUOTE
. გინეკოლოგიაში მაყვალას ჰკითხე პირადად. ყელში რას გულისხმობ საშოს თუ ცერვიკალურ არხს?

ოკ. მადლობთ smile.gif ვკითხავ. რას ვგულისხმობ არ ვიცი. მე ყელი გამიგია და ეგეც თუ რამდენიმეა არ ვიცოდი smile.gif)))))))))

Posted by: GITANA 26 Apr 2009, 11:06
NewName

ნუ რა გითხრა აბა პროქტოლოგია მუხაშავრიასთან ასოცირდება საქართველოში. biggrin.gif მაგაზე უკეთესი არა მგონია მონახო.

Posted by: ბაჭი24 27 Apr 2009, 09:07
არ ვიცოდი სად მეკითხა, ამიტომ ვწერ აქ და იქნებ ის მაინც მითხრათ სად წავიდე და რა ანალიზები ავიღო

მოკლედ რა ხდება, წინა პოსტში ვწერდი რომ მიბუჟდება ხელ ფეხი და კუნთები მტკივა, გუშინ ღამე დავაფიქსირე რომ მარჯვენა ფეხი მაქ ძაან გასიებულია, თავიდან მეგონა მეჩვენება, მერე ჩემებმაც დაამიდასტურეს, ანუ გასიებულია კოჭთან, და თითებთან პერიოდულად კანი ლურჯდება, დღეს რომ ავდექი, დღესაც დასიებული მაქ, სილურჯე სიწითლეშია, რა მჭირს? აი ფეხი რომ გადაგიბრუნდება და დაგისივდება მასე მაქ, არადა არ მტკივა, თანაც არ გადამბრუნებია

Posted by: GITANA 27 Apr 2009, 10:45
ბაჭი24

ჯერ ნევროპათოლოგთან მიდი და მერე ანგიოლოგთან

Posted by: ბაჭი24 27 Apr 2009, 11:09
GITANA
QUOTE
ჯერ ნევროპათოლოგთან მიდი და მერე ანგიოლოგთან

მადლობ პასუხისთვის

Posted by: shanyva 28 Apr 2009, 19:03
მოგესალმებით ყველას!მაინტერესებს B ჰეპატიტი რა საშიშროებას წარმოადგენს ფეხმძიმობისას,როგორ მკურნალობენ და რამდენად არის ალბათობა რომ ბავშვს გადაედება.მადლობა ყველას წინასწარ smile.gif

Posted by: Miss_Incredible 29 Apr 2009, 08:38
სად მეკითხა, არ ვიცოდი...
დაახლოებით 10 დღეა, დავრბივარ დილაობით 10-10 წუთი, მე-4 დღიდან კი მტკივა წვივის შუა ნაწილი წინა მხრიდან, ისეთი შეგრძნება მაქვს, თითქოს ძვალი მტკივა. ვხვდები, რომ ვარჯიშის დროს კუნთი იჭიმება და ა.შ. მაგრამ ვერ ვხვდები, ასეთ უცნაურ ადგილას ტკივილი კუნთის ბრალია? თუ შეიძლება მცირე ბრტყელტერფიანობას უკავშირდებოდეს ან რამე სხვას?

Posted by: Sex Narkomanka 1 May 2009, 00:25
TETISA
QUOTE
ექიმებო, ცხვირიდან სისხლდენა გამიხშირდა ამ ბოლო 2 თვის მანძილზე. და რამ იცის? A ვიტამინის ნაკლებობაა? და რა შეიცავს ამ ვიტამინს?

ცხვირიდან სისხლდენა იცის სისხლში შაქრის მომატების დონემ ან სუსტმა კაპილარებმა. მე უფრო შაქარი მგონია და A ვიტამინი თუ გინდა შეგიძლია იყიდო აფთიაქში წვეთების სახითაა მგონი.

ბაჭი24
ვენების გაგანიერებას გავს ეგ სიმპტომები.

Posted by: M-U-S(ic)E-X 1 May 2009, 00:37
cxviridan sisxldena magalma cnevamac icis + smile.gif eg aris organizmis egret damcvelobiti reakcia..magali cnevis dros...kapilari roca skdeba cxvirshi da roca aris sisxldena cxviridan eg araperi...roca tavshi...insultis shemtxvevashi..eg zalian mzime shedegi sad.gif ase rom asaks gaachnia...marto diabetma da amatuim vitaminis naklebbam ara :L)

Posted by: Romina 1 May 2009, 08:01
Miss_Incredible
QUOTE
თუ შეიძლება მცირე ბრტყელტერფიანობას უკავშირდებოდეს ან რამე სხვას?

შეიძლება. ცოტა ხანი არ ირბინო, თუ არ გაგიარა მერე რენტგენი უნდა გადაიღო ალბათ.

Posted by: just smile19 1 May 2009, 19:20
დისფუნქციური სტერეოგრამიტი რას ნიშნააავს? user posted image

Posted by: EXHALATING 1 May 2009, 20:26
ბოლო დროს, პერიოდულად მიბუჟდებასავით ზურგის მარცხენა ნაწილი, ბეჭის ქვემოთ. რასთან შეიძლება იყოს დაკავშირებული ან ვის მივმართო?

Posted by: giorgi_g 2 May 2009, 17:23
ახლობელმა შენიშნა თავიდან, თმის ბუდობრივი ცვენა, დაახლოებით 2 სმ.-ის დიამეტრის. აქვს 2 ადგილას, კეფასთან (ცოტათი ზემოთ) და საფეთქელთან.
რისი ბრალი შეიძლება იყოს და რა ნაბიჯები უნდა გადადგას?

გთხოვთ მიპასუხოთ PM-ში

Posted by: lurdes 2 May 2009, 21:10
რამდენიმე დღეა, ფეხები - კოჭების მიდამოებში შემისივდა და მტკივა. გარეგნულად საკმაოდ შესამჩნევია. მანამდე, ორი კვირის მანძილზე, ყოველდღიურად სირბილი დავიწყე - წონის კორექციის მიზნით. ფეხი არ გადამბრუნებია, არ, მიღრძვია.
მხოლოდ ფიზიკური დატვირთვის გამო შესაძლებელია შესიება? და რას მირჩევთ..
გმადლობთ..

Posted by: GITANA 2 May 2009, 21:17
shanyva
QUOTE
მოგესალმებით ყველას!მაინტერესებს B ჰეპატიტი რა საშიშროებას წარმოადგენს ფეხმძიმობისას,როგორ მკურნალობენ და რამდენად არის ალბათობა რომ ბავშვს გადაედება.მადლობა ყველას წინასწარ


სამშობიარო გზებით შეიძლება გადაედოს ბავშვს ამიტომ B ჰეპატიტის დროს საკეისრო კვეთა ხდება. საერთოდ ინფექციური დაავადებების მატარებელი დედები მეხუთე სამშობიაროში მშობიარობენ სპეციალური განყოფილებაა ამისთვის. მეან-გინეკოლოგ ბიძინა სიხარულიძესთან უნდა მიხვიდე, ამავე სამშობიაროს მთავარი ექიმია. ძირითადად ის ამშობიარებს ამ განყოფილებაში.

Posted by: just smile19 2 May 2009, 22:54
QUOTE
დისფუნქციური სტერეოგრამიტი რას ნიშნააავს?



არავინ იცით? user posted image

Posted by: EXHALATING 3 May 2009, 01:47
QUOTE
ბოლო დროს, პერიოდულად მიბუჟდებასავით ზურგის მარცხენა ნაწილი, ბეჭის ქვემოთ. რასთან შეიძლება იყოს დაკავშირებული ან ვის მივმართო?


user.gif

Posted by: makuna 4 May 2009, 16:31
მოგესალმებით. ძალიან მჭირდება თქვენიდახმარება და რჩევა. ჩემს მეუღლეს დილით ორი დღეა უკვე ამოხველებისას წითელი სისხლის მაგვარი გამონადენი აქვს ყელიდან. (უკაცრვად და ამოხველების დროს). არაფერი არ აწუხებს 37 წლისაა. რა შეიძლება იყოს, და ვის შეიძელბა მივმართო რომ სწორად დამისვან დიაგნოზო, ძალიან ვნერვიულობ.

Posted by: texasuri jleta benzoxerxit 4 May 2009, 17:19
QUOTE (makuna @ 4 May 2009, 16:31 )
მოგესალმებით. ძალიან მჭირდება თქვენიდახმარება და რჩევა. ჩემს მეუღლეს დილით ორი დღეა უკვე ამოხველებისას წითელი სისხლის მაგვარი გამონადენი აქვს ყელიდან. (უკაცრვად და ამოხველების დროს). არაფერი არ აწუხებს 37 წლისაა. რა შეიძლება იყოს, და ვის შეიძელბა მივმართო რომ სწორად დამისვან დიაგნოზო, ძალიან ვნერვიულობ.

პირველ რიგში თერაპევტს. გამოკვლევებია საჭირო. მწეველია?

Posted by: makuna 5 May 2009, 09:56
გმადლობთ გამოხმაურებისთვის. კი მწეველია, კარგი თერაპევტს ვის მირჩევთ?

Posted by: pin700 5 May 2009, 17:46
მოგესალმებით კველას.. ერტი შეკითხვა მაქვს ასი სახეზე სიწითლე ცხვირტან და ლოყებთან რამ იცის რატომ არის გამოწვეული ??? ეს შეიზლება იკოს ალერგია რაამე ვიტამინების ნაკლებობა ტუ რა??? აი სურატზე ნაჩვენებია ის ადგილები eek.gif user.gif რისგან არის გამოწვეული ??? წინასწარ გმადლობთ yes.gif

Posted by: Cousteau 5 May 2009, 17:54
QUOTE (pin700 @ 5 May 2009, 17:46 )
მოგესალმებით კველას.. ერტი შეკითხვა მაქვს ასი სახეზე სიწითლე ცხვირტან და ლოყებთან  რამ იცის რატომ არის გამოწვეული ??? ეს შეიზლება იკოს ალერგია რაამე ვიტამინების ნაკლებობა ტუ რა??? აი სურატზე ნაჩვენებია ის ადგილები  eek.gif  user.gif  რისგან არის გამოწვეული ??? წინასწარ გმადლობთ  yes.gif

იმას რაც შენ გიხატია ქვია Malar Butterfly Rash და ყველაზე ხშირად არის სისტემური წითელი მგლურას დროს (Systemic Lupus Erythematosus - SLE) თუმცა სხვა დაავადებების დროსაც არის

user posted image

ნორმა არ არის! ისერო თუ ეგ გაქვს (ან ვინმე შენ ნაცნობს აქვს) უნდა ეჩვენო ექიმს სასურველია კარგ თერაპევტს

შეიძლება იყოს გამოწვეული წამლების მიღებით, რამე წამალს ხო არ იღებ(ს)? თუ წამლებს იღებს გადაამოწმეთ წამლის დანართი იმიტომრომ არ მახსოვს, შეიძლება ამ დროს საჭირო იყოს წამლის ეგრევე შეწყვეტა (გადაამოწმე წამლის დანართში, ან დაუკავშირდი ექიმს)

Posted by: pin700 5 May 2009, 18:08
Cousteau
QUOTE
იმას რაც შენ გიხატია ქვია Malar Butterfly Rash და ყველაზე ხშირად არის სისტემური წითელი მგლურას დროს (Systemic Lupus Erythematosus - SLE) თუმცა სხვა დაავადებების დროსაც არის

გმადლობთ მარა ეგეტი სიწითლეც არა ნუ ოდნავ ... ექიმს მივმართAვ... გაიხარეთ

Posted by: Romina 5 May 2009, 19:34

pin700

მზეზე ჩნდება თუ ისედაც არის გამონაყარი? სიცხეც ხო არ აქვს პაციენტს?

Posted by: lurdes 5 May 2009, 19:55
QUOTE
რამდენიმე დღეა, ფეხები - კოჭების მიდამოებში შემისივდა და მტკივა. გარეგნულად საკმაოდ შესამჩნევია. მანამდე, ორი კვირის მანძილზე, ყოველდღიურად სირბილი დავიწყე - წონის კორექციის მიზნით. ფეხი არ გადამბრუნებია, არ, მიღრძვია.
მხოლოდ ფიზიკური დატვირთვის გამო შესაძლებელია შესიება? და რას მირჩევთ..
გმადლობთ..

იქნებ მირჩიოთ რამე
ვფიქრობ, სირბილი გამაღიზიანებელი ხომ არ იყო და შეშუპება უფრო სერიოზული მიზეზით ხომ არ ხდება
რა იწვევს ფეხების ასე შეშუპებას?
რამდენიმე დღეა მდგომარეობა არ შემცვლია

Posted by: Romina 5 May 2009, 19:58
lurdes
მარტო შეშუპებულია? არ გტკივა? ფერი ხომ არ აქვს შეცვლილი?

Posted by: lurdes 5 May 2009, 20:03
ტკივილით მტკივა, ისე, რომ სიარულიც მიჭირს. ფერი არ შეუცვლია. განსაკუთრებით მარცხენა ფეხი. კოჭის მიდამოში გამობერილი მაქვს.

აქვე - არ ვიცი მნიშვნელობა თუ აქვს. სამსახურში ძირითადად ვზივარ და სამუშაო დღის მერე მოძრაობა უფრო მტკივნეულია.

Posted by: Romina 5 May 2009, 20:10
lurdes
იცი რა მასე ცოტა ძნელია გაგება ვენების პრობლემა თუ კოჭ-წვივის სახსარია დაინტერესებული, გასინჯვისას ადვილი მისახვედრია. ანგიოლოგთან და რევმატოლოგთან ჯობია მიხვიდე. ხანგრძლივად ჯდომის ან დგომის მერე ფეხები ხო არ გიმძიმდება?

Posted by: lurdes 5 May 2009, 20:16
გმადლობ დაკვალიანებისთვის.

კი, მიმძიმდება ხოლმე.


Posted by: Romina 5 May 2009, 20:19
lurdes
QUOTE
კი, მიმძიმდება ხოლმე.

მაგით ვენების პრობლემას უფრო გავს, თუმცა გამორიცხული არ არის კოჭ-წვივის სახსრის ართროზი-ართრიტი. მიდი ექიმთან.

QUOTE
გმადლობ დაკვალიანებისთვის.


2kiss.gif

Posted by: lurdes 5 May 2009, 20:23
მივალ აუცილებლად
მადლობა 2kiss.gif

Posted by: pin700 5 May 2009, 21:13
Romina
QUOTE
მზეზე ჩნდება თუ ისედაც არის გამონაყარი? სიცხეც ხო არ აქვს პაციენტს?

არა მზეზე არა სულ ისე უბრალოდ... ან კიდე რო დაცხება და რამე სიმხურვალე სახეზე და სიწითლე .. სიცივის დროსაც იგივე.. ანუ ეს ალერგიაა თუ რამე დაავადებაა???

Posted by: Romina 5 May 2009, 21:58
pin700
შეიძლება ალერგიაც იყოს, მაგრამ როგორც უკვე მოგწერეს შეიძლება ერთ-ერთი სისტემური დაავადების გამოვლინება იყოს. მზეზე იმიტომ გკითხე, რომ როგორც წესი ამ დაავადების დროს მზეზე გამონაყარი ძლიერდება, ან მარტო მზეზე ყოფნისას ჩნდება. ექიმთან უნდა მიხვიდე. თუ გინდა თერაპევტი იყოს, მაგრამ მე ვფიქრობ ჯობია რევმატოლოგთან მიხვიდე, მის კომპეტენციაში შედის სისტემური დაავადებები. რამდენიმე ანალიზი დაგჭირდება.



Posted by: pin700 6 May 2009, 13:05
Romina
QUOTE
შეიძლება ალერგიაც იყოს, მაგრამ როგორც უკვე მოგწერეს შეიძლება ერთ-ერთი სისტემური დაავადების გამოვლინება იყოს. მზეზე იმიტომ გკითხე, რომ როგორც წესი ამ დაავადების დროს მზეზე გამონაყარი ძლიერდება, ან მარტო მზეზე ყოფნისას ჩნდება. ექიმთან უნდა მიხვიდე. თუ გინდა თერაპევტი იყოს, მაგრამ მე ვფიქრობ ჯობია რევმატოლოგთან მიხვიდე, მის კომპეტენციაში შედის სისტემური დაავადებები. რამდენიმე ანალიზი დაგჭირდება.

აჰაა.. გასაგებია კარგი.. ექიმს მივმარტავ კველა ვარიანტში... გმადლობთ

Posted by: Lacrimosa_ 7 May 2009, 20:19
ჩემს მეგობარს ექიმმა უთხრა, რომ ფილტვზე აქვს ჯირკვალი
რას ნიშნავს ეს? spy.gif

Posted by: texasuri jleta benzoxerxit 8 May 2009, 02:03
QUOTE (Lacrimosa_ @ 7 May 2009, 20:19 )
ჩემს მეგობარს ექიმმა უთხრა, რომ ფილტვზე აქვს ჯირკვალი
რას ნიშნავს ეს? spy.gif

შენი მეგობრის ექიმი გამოუშვით ჩემთან,როგორც ფსიქიატრთან ანუ yes.gif givi.gif givi.gif

Posted by: SPC 8 May 2009, 23:49
მომატებული ერითროციტების დალექვის სიჩქარე - 40, როცა ნორმაა 2-10
და ამასთანავე მომატებული ლეიკოციტები შეიძლება იყოს ტუბერკულიოზის ჩხირთან დაკავშირებული? ჩემს ძველ ანალიზებს გადავხედე და რაღაც მაინტერესებდა, ბოლომდე ვერ დავადგინე რა მჭირდა მაშინ,
ნუ ანალიზის სხვა მაჩვენებლებიც არ იყოს თითქმის არცერთი ნორმაში მაგრამ ყველაზე დიდი ამოვარდნა ამათ ჰქონდათ
ხო +სიცხეები 37-37.3
და მანტუზე რეაქცია იყო , ოღონდ ზუსტად რა არვიცი

:დაბნეულიპაციენტი:

Posted by: Nnino 9 May 2009, 00:04
ძვირფასო ექიმებო, ჩემმა თანამშრომელმა "ტიანშის" რაღაც ობში კალციუმი ( smile.gif ) შემომტენა, გინდა თუ არა, უნდა დალიოო, იმის გადაკიდებას პოლკის გადაკიდება მირჩევნია და ამიტომ ვიყიდი, მაგრამ ღირს, რომ დავლიო? თუ მორიგი აფერისტობაა (საკმაოდ ძვირი ჩემი ჯიბისთვის – 55 ლარი) არ სჯობია, 67 წლის დედას დავალევინო, ხერხემალი აწუხებს მაინც. ჰო, კითხვის აზრი ისაა, რომ გოლდ რეისა და პოლიჟენს რომ ვსვამდი, არ ვიცი, დამთხვევა იყო თუ არა, მაგრამ სწორედ იმ პერიოდში გამოვკარი დეპრესიას ხელი, და ჩემმა ექიმმა ფსიქონევროლოგმა მითხრა, დაანებე თავი მესამე სამყაროსთვის გამოსულ საკვებ დანამატებსო. ყუთზე მაინცადამაინც არაფერი აწერია ისეთი. ნეტში კიდევ საეჭვო და უსაგნო ქება–დიდების მეტი ვერაფერი ვნახე. აი, ესაა: http://www.tiens-shop.net/product_info.php?products_id=13&osCsid=diiie284vim3rgh28t9iu587p6

მიპასუხეთ, რა... 2kiss.gif

Posted by: texasuri jleta benzoxerxit 9 May 2009, 01:15
Nnino
პოლიჟენს და გოლდ-რეის ხშირად ანერგიული დეპრესიების დროს ვნიშნავთ. ტიანშის პროდუქციას უკიდურესი სკეპტიციზმით ვუყურებ.( ცისფერი მთები ანუ ტიანშანი )SPCმანტუზე სუსტი რეაქცია თითქმის ყველას გვექნება...ტუბერკულოზს ყველაფერი შეიძლება ახასია\ტებდეს,თუმცა უფრო დამახასიათებელია ლიმფოციტოზი.

Posted by: Nnino 9 May 2009, 01:24
texasuri jleta benzoxerxit

დიდი მადლობა... მეც ვიცი, რომ სისულელეა ეგ ტიენსია თუ ქლიავისფერი მთები, მაგრამ იმ ქალს უარს ვერ ვეტყვი (უფროსია ჩემი და თან საშიშიც), ეს ის შემთხვევაა, წამლის არმიღების უარყოფითი შედეგი რომ მიღების უარყოფით შედეგს აჭარბებს. biggrin.gif. ყოველ შემთხვევაში, რომ დავლიო, "ბაშმაკებს არ გადამაყრევინებს", არა?


Posted by: texasuri jleta benzoxerxit 9 May 2009, 01:47
QUOTE (Nnino @ 9 May 2009, 01:24 )
texasuri jleta benzoxerxit

დიდი მადლობა... მეც ვიცი, რომ სისულელეა ეგ ტიენსია თუ ქლიავისფერი მთები, მაგრამ იმ ქალს უარს ვერ ვეტყვი (უფროსია ჩემი და თან საშიშიც), ეს ის შემთხვევაა, წამლის არმიღების უარყოფითი შედეგი რომ მიღების უარყოფით შედეგს აჭარბებს. biggrin.gif. ყოველ შემთხვევაში, რომ დავლიო, "ბაშმაკებს არ გადამაყრევინებს", არა?

lol.gif givi.gif givi.gif givi.gif givi.gif givi.gif ნუ თუ მასეა საქმე,დალიე რა გაეწყობა. თუ გავნო,შეატყობ... ასე რომ givi.gif

Posted by: skoch 9 May 2009, 02:26
ბევრ ლიმონს ვჭამ ყოველ დღე ...რამე მჭირს?? cry.gif

Posted by: just smile19 9 May 2009, 15:42
თერაპევტთან მინდა მისვლა და სად მირჩევთ?

Posted by: skoch 9 May 2009, 17:53
QUOTE
თერაპევტთან მინდა მისვლა და სად მირჩევთ?

კოტე მაისაია რესპუბლიკურში 4ე სარტულზეა მაგარი თეაპევტია

Posted by: Jayson 9 May 2009, 20:46
გამარჯობათ

კითხვას თერაპევტთან დავსვამ თუ შეიძლება, სხვა განყოფილებებზე მეტად ამას შეეხება თუ არვცდები

ძალიანმ ჭირდება თქვენი დახმარება :>

ხომ არ იცით თბილისში რამე ცენტრი ან ბავშვთა სახლი

დაუნის სინდრომით დაავადებული ბავსვებისთვის ? sad.gif


Posted by: just smile19 9 May 2009, 22:46
skoch
მადლობა user posted image

Posted by: ბაჩუკი 12 May 2009, 16:55
skoch
კოტე მაისაია ძაან მაგარი გასტროენტეროლოგია მაგრამ თერაპევტი არ არის...

Posted by: skoch 12 May 2009, 16:58
QUOTE
კოტე მაისაია ძაან მაგარი გასტროენტეროლოგია მაგრამ თერაპევტი არ არის...

ხოო..ლექტორმა თერაპევტად გამაცნო smile.gif

Posted by: madonna_Girl 12 May 2009, 21:38
ერთი თხოვნა მაქვს,გამეცით პასუხი თუ შეგიძლიათ- მაინტერესებს ექიმი ასკურავას კოორდინატები და საკონტაქტო ტელეფონის ნომერი.წინასწარ დიდი მადლობა

Posted by: goga_ivana 13 May 2009, 12:09
გამარჯობათ. ერთი კითხვა მაქვს თქვენთან. უკვე ერთი წელი იქნება სადღაც შუადღისთვის ვგრძნობ ხოლმე ძლიერ მოთენთილობას ანუ ძილი მომერევა ხოლმე. შეიძლება ეს სიმპტომები ჭარბი წონით იყოს გამოწვეული? დიდი მადლობა წინასწარ.

Posted by: ana_nushka 13 May 2009, 12:24
გამარჯობათ . . .ჩემი კითხვის შესაბამისი თემა ვერ ვნახე და იქნებ თქვენ დამეხმაროთ . . .

თუ ადამიანს ხალი აქვს გაღიზიანებული . . .ექავება და ლიმფა სდის . . . მელანომის საშიშროება არის?? თუ შეიძლება კიდევ რაიმე მიზეზეი იყოს??? ლაპარაკია 50 წლის მამაკაცზე . . . sad.gif

Posted by: keti8181 16 May 2009, 11:48
გამარჯობა! თუ იცის ვინმემ რისი ბრალი შიძლება იყოს მამაჩემს 75 წლის ასაკშია და დაეწყო წელის ტკივილი, უფრო სწორად როცა სკამზე ზის და ადგომის პროცესში სანამ გასწორდება. 10 წლის წინ აწუხებდა მარილები კისრის არეში და იმკურნალა შედეგიანად.

Posted by: Romina 16 May 2009, 12:59
keti8181
შეიძლება იგივე პროცესის ბრალი იყოს რაც კისერში ქონდა. რევმატოლოგთან ჯობია მივიდეს. წელის მალები უნდა გადაიღოს ისე რომ გავა-თეძოს შესახსრებაც კარგად გამოჩნდეს და დენსიტომეტრიის გაკეთებაც კარგი იქნება. იმიტომ რომ ზოგჯერ ოსტეოპოროზული კომპრესიული მალები იწვევენ ტკივილს. ამის კარგი აპარატი 8-ე სავადმყოფოში აქვთ, ელენე გიორგაძის კლინიკაში. DEXA ქვია. მარა ცოტა ძვირია, მგონი 80 ლარი ღირს.

Posted by: keti8181 16 May 2009, 14:11
Romina
უღრმესი მადლობა. ვერაფრით ვაიძულე ექიმთან წასვლა ეშინია რას ეტყვის და მგონი მეც გადმომდო ეხლა შიში. თან ვარწმუნებ რომ რაიმე ონკოლოგირი რომ იყოს მარტო ადგომის დროს არ ეტკინებოდა

Posted by: Miss_Manette 16 May 2009, 14:11
მგონი აქ უნდა დავწერო

3 დღეა საჭმელი შემზიზღდა,ვერაფერს ვჭამ,დანახვაც არ მინდა,სუნიც მაღიზიანებს,მაგრამ მშია sad.gif თუ შევჭამე,ცოტა მაინც,ღებინების შეგრძნება მაქვს sad.gif
დღეს ცოტა მაწონი ვჭამე,ვიფიქრე იქნებ ამან არაფერი დამმართოს მეთქი ,მაგრამ იგივე შედეგია.


არაფერზე მინერვიულია,არც ორსულად ვარ gigi.gif

ჰოდა,რით გამოვასწორო მდგომარეობა?

Posted by: Romina 16 May 2009, 15:20
keti8181
QUOTE
უღრმესი მადლობა.

არაფერს
smile.gif
საშიში არაფერი არ არის, დარწმუნებული ვარ. წამოდგომისას და პირველი ნაბიჯების გადადგმისას ტკივილი, ართროზის დამახასიათებელი სიმპტომია და "სტარტული ტკივილი" ქვია. ექიმთან აუცილებლად უნდა მივიდეს, ოსტეოპოროზიც არ იყოს, თორე წელში მოიხრება.



Posted by: shanyva 20 May 2009, 12:44
პროთრომბინის დასაწევი საშუალებები მაინტერესებს,მცენარეებს ვგულისხმობ,ან ბოსტნეულს ან ხილს რავიციი...(კომშის ფოთლების გარდა)ორსულობის დროს სულ მარალი მაქვს და ხშირად მიწევს წამლების მიღებაsad.gifმადლობა ყველას წინასწარ

Posted by: gio256 21 May 2009, 13:01
უკაცრავად მაინტერესებს კენჭის დაშლა თუ შეიძლება 4 დღეში 16 ნემსის გაკეთებით??? შესაძლებელია თუ არა... ყველა ნაცნობი მეუბნება როპმ კენჭი ესე მალე არ იშლება და რომ მცდარი დიაგნოზი დამისვეს??

Posted by: ninchancho 21 May 2009, 13:14
გამარჯობათ..როგორც მითხრეს რაც მე მაწუხებს თერაპევტის საქმეოაო და აბა გეტყვით რა მაწუხებს ეხლა smile.gif
ესეიგი ძალიან ვოფლიანობ ოდნავ რომ დამცხება მაშინვე მთელ სხეულზე ვსველდები რაც ძალიან მაწუხებს და დისკომფორტს მიქმნის,ხელის გულები სულ ნამიანი მაქვს.. დიაბეტი მაქვს ეს თუ წაადგება დიაგნოზის დასმას იქნება რამე მირჩიოთ?საქართველოში არ ვარ და ექიმთან წასვლას ვერ ვახერხებ..ხო წლების წინ ჩიყვიც მქონდა მაგრამ უკვე ძალიან დიდი ხანია აღარ გამომიკვლევია შეიძლება ეს იყოს მიზეზი?
მადლობა წინასწარ...

Posted by: aladini 21 May 2009, 13:30
მოკლედ გამიჩნდა ოსტეოქონდროსთან დაკავშირებული პრობლემა

გასულ ზაფხულში ფეხბურთის ემრე ნაბანი მივუჯექი კონდიციონერს და მხარზე ნერვი გაივცივე
ხოდა კაი ხნაი მოვუნდი მკურნალობას

ამ 4 კვირის უკან ისევ დამეწყო ტკივილები, ვიფიქრე ნერვია თქო და იგივე მეთოდით დოკლოფეროლის და ვურდონის ნემსები, მაზები, ტაბლეტებიდ ა ეგეთები ვიკეთე და ვსვი (ნუ ტკივლი განსხვავებულია, ამშინ განძრევაზე მეგონა დამიწყდა კუნთები თქო და ეხლა კისრის და ბეჭის მკდამოში მხტკივა , მჩხვლეტს და თითქოს სიმძიმეა , და ეს ყველაფერი მარჯვენა მხარს, სახსარ და ხელის თიოტებსაც კი აგდაეცემა)
მერ ემივედი ექიმთან
დმაინIშნა რაღაც ბლოკადა (გორმონის ნემსები), მარ აშეედეგი სიევ ნული
მერე გადავიიღე რენტგენი
კისრის ფიზიოლოგიური ლორდოზი ფასწორებულია, მე-4-5-6 მალთშორისი სივრცეები შევიწროებუილი (განსაკუთრებით მე-4-5), რაღაც ოსტეოფიტები, ფიზიოლოგიური ყიფოზი,მალების სუბქონდრალური ჩამკეტი ფირფიტები გამკვრივებულია, და ასეტები დამიწერეს, დიაგნოზი - კისრის და გულმკერდის მალთაშუა დისკების ოსტეოქონდროზი

ხოდა შაბათს უდნა მივიდე დაზღვევის ნერვოპათოლოგთან
როგორც ვიცი ეგენი არ ანაზღაურებენ ასეთ რამეებს, ქრონიკულიაო-ეგრევე ასკვნიან
მარა ის მრტ თუ მრი თუ რაღაც მასეთი თუ არ გადამიღეს, ზუსტი დიაგნოზი როგორ უნდა დასვან და წინაწსარ როგორ საზღვრავენ-ქრონიკულია თუ არა
არადა ეგ პროცედურა კაი ძვირი ღირსო
რას მირჩევთ,ს ად წავიდე, ვის მივმართო, რა ფასებია, როგორ მოვიქცე?
სამსახური სიეთი ამქვს ძირითადად ჯდომა მიწევს და მაგან გაამწვავა ალბათ
მოკელდ შარში ვარ მაგარში რა

Posted by: A_I 21 May 2009, 15:04
მიპასუხეთ რა მჭირს სანამ გამირეკია biggrin.gif

დღეს დილას გავიღვიძე და... თითქმის აღარფერი მესმის მარჯვენა ყურში. ასევე როდესაც ვეხები თავის/სახის (ასევე კეფის მიდამოებში) მარჯვენა მხარეს ისეთი შეგრძნება მაქვს თითქოს სხვას ვეხები და არა ჩემს თავს.

სიცხე არ მაქვს, სხვა მხვრივ თავს მშვენივრად ვგრძნობ, კბილებიც მაქეთ მხარეს თითქოს დამაკმაყოფილებელ მდგომარებაში არიან.

მაგდავარი შეგრძნება 3 დილის განმავლობაში მაქვს უკვე, თუმცა პირველ ორ შემთხვევაში, შეგრძნებამ პირველ 5-10 წუთში გაიარა. დღეს კი, უკვე რომელი საათია და ისევ ისე არის ყველაფერი... user.gif

Posted by: aranormaluri 21 May 2009, 15:52
ცხივრს რომ ვიხოცავ პატარა წინწკლები მაქვს სისხლის და რისი ბრალი შეიძლება იყოს?
ერთი თვის წინ 4 საათი ვიფრინე და ვითომ ამან გამოიწვია?

ტუბერკულოზს აქვს თუ არა ასეთი სინდრომები იქმებ მითხრათ.

Posted by: Solveig 22 May 2009, 14:11
ninchancho
დაზღვევა არ გაქვს? სად ხარ ასეთ ღვთისგან დავიწყებულ ადგილას? იდეაში, ინტერნეტი ყოფილა და...

Posted by: homoSacer 23 May 2009, 01:32
სწორ განყოფილებაში ვსვამ კითხვას იმედია .. მარტივ და ბანალურს, მაგრამ თუ მიპასუხებთ ძალიან დამეხმარებით ...

გონებრივი გადაღლის დროს (ძილს ვცდილობ არ მოვიკლო) ხომ არის საკვები რაც უკეთ აამუშავებს ტვინს... თანაც ასეთ დროს ჭამა აღარ გინდება და ძალით ჭამ.. მოკლედ, ყველაზე სასარგებლო რა პროდუქტებია ასეთ შემთხვევაში?


Posted by: aladini 23 May 2009, 13:09
დღეს ვიყავი ნერვოპათოლოგთან
დამინიშნა ემ-ერ-აი
მერე ოჯახის ექიმმა ძვირიანი პოროცედურაა და სათაომ უნდა გადაწყვიტოს აფინანსებენ თუ არაო
ისინი კდიე ორშაბათამდე არ მUშაობენ და მოკლედ ისევ უდნა გავუძლო ტკივილს რა

Posted by: blblbl 23 May 2009, 21:11
გამარჯობათ.დედაჩემს ერთი კვირაა სტკივა თავი არანაირი წამალი არ შველის,გუსინ იყო ნერვოპატოლოგთან,ხელიტ რაღაცეები გაუსიმჯა თAვზე დაახრევია გადაახრევინა biggrin.gif და ა.ს. და უთხრა ნერვისგან იქნებაო და დაუნისნა წამლები მაგრამ შედეგი არაა არანაირი.რას მირჩევტ რა გავაკეთოთ და სად მივიდეს?

Posted by: texasuri jleta benzoxerxit 24 May 2009, 02:22
QUOTE (blblbl @ 23 May 2009, 21:11 )
გამარჯობათ.დედაჩემს ერთი კვირაა სტკივა თავი არანაირი წამალი არ შველის,გუსინ იყო ნერვოპატოლოგთან,ხელიტ რაღაცეები გაუსიმჯა თAვზე დაახრევია გადაახრევინა biggrin.gif და ა.ს. და უთხრა ნერვისგან იქნებაო და დაუნისნა წამლები მაგრამ შედეგი არაა არანაირი.რას მირჩევტ რა გავაკეთოთ და სად მივიდეს?

გირჩევთ ნეიროქირურგთან კონსულტაციას და თავის ტვინის მაგნიტურ რეზონანსს (MRT), რამე საშეინელება რომ არ გაგვეპაროს. თუმცა MRT-ს გაკეთება არგაკეთება ჯობია კვალიფიციურმა ნევროპათოლოგმა და/ან ნეიროქირურგმა გადაწყვიტოს.

P.S. ისე წნევა თუ გაუზომეთ ამ კვირის განმავლობაში?

Posted by: blblbl 24 May 2009, 11:14
QUOTE
ისე წნევა თუ გაუზომეთ ამ კვირის განმავლობაში

კი და მაღალი აქვს ,მაგის გამო ვფიქრობდით ტავისტკივილებს მაგრამ ნერვოპათოლოგმა მაგის ბრალი არ იქნებაო,ან ცუდათ გედო თAვი ან ნერვი გაგიცივდა,მაგრამ ფაქტია რომ არ შველის წამლები რაც გამოუწერა.

Posted by: TETISA 24 May 2009, 11:42
A_I
QUOTE
დღეს დილას გავიღვიძე და... თითქმის აღარფერი მესმის მარჯვენა ყურში. ასევე როდესაც ვეხები თავის/სახის (ასევე კეფის მიდამოებში) მარჯვენა მხარეს ისეთი შეგრძნება მაქვს თითქოს სხვას ვეხები და არა ჩემს თავს.

სასწრაფოდ მიდი ექიმთან. მგონი ნევროპათოლოგი გჭირდება. თუკი მთელი მარჯვენა მხარე კარგავს შეგრძნებას, ეს სეიძლება ტვინიდან იყოს გამოწვეული.
ამიტომ დროზე უნდა მიხედო. აღარ გადადო ეგ საქმე და მიდი კარგ სპეციალისტთან

Posted by: Dead Silence 25 May 2009, 02:48
ერთი კვირაა სიცხე მაქ ვერაფრით ვერ ვუშველე,წამალსაც ვსვამ სიცხის დამწევს ( C ვიტამინით ) არაფერი მეშველა და სიცხე სულ ცვალებადია 37 დან 38 მდე მერყეობს,ერთი ორჯერ მეტიც იყო. თავიდან ძალიან მახველებდა,იმდენი ვახველე,მთელი კუნთები მტკივა ნეკნებთან და მარცხენა მხარიც კი. ღრმად რომ ვსუნთქავ მაგ დროსაც მტკივა სასუნთქი მილი (ყელი არა). რამე სერიოზული ხომ არაა, ? რაღაც დიდიხანს გაგრძელდა...თან უცნაური ტკივილები და ხველა აქ.


დაწერეთ თუ ვინმემ რამე იცით.რისი სიმპტომები შეიძლება იყოს ?

მადლობა წინასწარ.

Posted by: texasuri jleta benzoxerxit 25 May 2009, 13:13
QUOTE (Dead Silence @ 25 May 2009, 02:48 )
ერთი კვირაა სიცხე მაქ ვერაფრით ვერ ვუშველე,წამალსაც ვსვამ სიცხის დამწევს ( C ვიტამინით ) არაფერი მეშველა და სიცხე სულ ცვალებადია 37 დან 38 მდე მერყეობს,ერთი ორჯერ მეტიც იყო. თავიდან ძალიან მახველებდა,იმდენი ვახველე,მთელი კუნთები მტკივა ნეკნებთან და მარცხენა მხარიც კი. ღრმად რომ ვსუნთქავ მაგ დროსაც მტკივა სასუნთქი მილი (ყელი არა). რამე სერიოზული ხომ არაა, ? რაღაც დიდიხანს გაგრძელდა...თან უცნაური ტკივილები და ხველა აქ.


დაწერეთ თუ ვინმემ რამე იცით.რისი სიმპტომები შეიძლება იყოს ?

მადლობა წინასწარ.

შეიძლებსა იყოს პნევმონია. აუცილებელია გაშუქება (რენტგენი)რადგანაც ვისურული პნევმონიების დროს ხშირად ფილტვების აუსკულტაციით (მოსმენით) არაფერი არ ისმის.

Posted by: aladini 25 May 2009, 17:38
მოკლედ ემ-ერ-აი გავიკეთე და მე-5 დისკის თუ მალის (კისერზე) თიაქარიაო
ნეიროქირურგმა უდნა ნახოსო , მარა სავარაუდოდ საოპერაციოაო
შარში ვარ მაგარშI
ნეტა სხვა საშველია რ იქნება, მასაჟებით ან რამეთი?

Posted by: Romina 26 May 2009, 12:03
aladini
ნეიროქირურგთან მიდი აუცილებლად. მე გირჩევდი 9-ე საავადმყოფოში გველესიანს. ნამდვილად არ გეტყვის ოპერაციას თუ არ გჭირდება. არის სხვადასხვა პროცედურები, რაც მდგომარეობას შეამსუბუქებს, მაგრამ მხოლოდ გარკვეულ სტადიაზე. ამიტომ ჯერ ნეიროქირურგთან მიდი.

Posted by: Solveig 26 May 2009, 15:34
თმის ღერების დათხელება და გამყიფება და ამასთანავე, თმის ცვენაც, რით შეიძლება იყოს გამოწვეული?
(ფარისებრი ჯირკვლის პრობლემა არ იუნდა იყოს, ყოველ შემთხვევაში, ჰორმონების დონე ნორმაშიაო).

Posted by: minerva 26 May 2009, 15:54
Solveig
QUOTE
თმის ღერების დათხელება და გამყიფება და ამასთანავე, თმის ცვენაც, რით შეიძლება იყოს გამოწვეული?

როცა ლოკალიზებული არ არის და დიფუზურ თმის ცვენაზეა ლაპარაკი, რამოდენიმე მიზეზი შეიძლება იყოს:
- სტრესი
- ფარისებრი ჯირკვლის ფუნქციის მოშლა (ეს შენს შემთხვევაში გამოირიცხა უკვე)
- სხვადასხვა მედიკამენტების მიღება
- ინფექციური დაავადებები
- ჰორმონალური ცვლილებები( კლიმაქტერიული პერიოდი, მშობიარობა, ჩასახვის საწინააღმდეგო პრეპარატების შეწყვეტა)
- არასრულფასოვანი კვება (მაგ. ქალებში რკინის ნაკლებობა ხშირია)

Posted by: Solveig 26 May 2009, 16:44
minerva
ლოკალიზებული ნამდვილად არ არის.
QUOTE
- სტრესი

ოოო...ეს- იცოცხლე sad.gif
2006 წლის ბოლოდან-ყოველდღე sad.gif

QUOTE
სხვადასხვა მედიკამენტების მიღება

არაფერს ვსვამ, მათ შორის არც ჩსპ-ს.
QUOTE
ინფექციური დაავადებები

მე მგონი, არ მაქვს ასეთი.
QUOTE
არასრულფასოვანი კვება (მაგ. ქალებში რკინის ნაკლებობა ხშირია)

შეიძლება ესეც იყოს..აქ რა საკვებია, კაცმა არ იცის..მიუხედავად იმისა, რომ ნახევარფაბრიკატებს და "სწრაფ საკვებს" არ ვიღებ.
QUOTE
ჰორმონალური ცვლილებები

მეც მაგას ვფიქრობდი, მაგრამ ექიმის დასკვნით (ჯერ-ჯერობით)-სამკურნალო არაფერიაო და,,,რა ვიცი.

ბოლო 2-3 წელია, მაქვს. არადა, მანამდე ლამის ერთი ღერი არ მაკლდებოდა ხოლმე.

Posted by: texasuri jleta benzoxerxit 26 May 2009, 17:10
QUOTE (aladini @ 25 May 2009, 17:38 )
მოკლედ ემ-ერ-აი გავიკეთე და მე-5 დისკის თუ მალის (კისერზე) თიაქარიაო
ნეიროქირურგმა უდნა ნახოსო , მარა სავარაუდოდ საოპერაციოაო
შარში ვარ მაგარშI
ნეტა სხვა საშველია რ იქნება, მასაჟებით ან რამეთი?

არა,სხვა საშველი არ არის.

თუმცა მასაჟებმა შეიძლება დროებიტი შვება მოგცეს. ოღონდ ნეიროქირურგთან შეთანხმებით

Posted by: minerva 27 May 2009, 02:17
Solveig
რადგან ექიმმა მასე გითხრა, ორგანული დაავადებები და ჰორმონული ამბები გამოირიცხა. ალბათ უფრო საკვებისმიერი იქნება.
რკინის რაოდენობა რომ გამოიკვლიო, კარგი იქნება...
B ჯგუფის ვიტამინებს(+ ბიოტინს) კარგი შედეგი აქვსო ამბობენ. გარდა ამისა ცინკის შემცველ პრეპარატებს ნიშნავენ ხშირად და ხანდახან კომბინაციაში გოგირდშემცველ ამინომჟავებსაც (მეთიონინი, ცისტეინი).

ადრე რადგან არ უჩიოდი, რამე მანიპულაცია ხო არ ჩაგიტარებია ამ ბოლო დროს თმაზე, რასაც შეიძლება დაეზიანებინა?
ერთხელ დამემართა მასე... მეც ძალიან ჯანმრთელი თმა მაქვს საერთოდ, არც ვიღებავ და ქიმიურ პროცედურებსაც ვერიდები. ფენითაც როცა აუცილებელია ან მეჩქარება, მაშინ ვიშრობ. ერთი-ორი წლის წინ ქიმიურად დატალღვა გავაკეთებინე( არადა რად მინდოდა აზრზე არა ვარ, ისედაც არ მაქვს სწორი თმა), ჯერ იქვე ისეთი ალერგია მომცა წამალმა, კინაღამ გადავრიე პარიკმახერი. მერე არც დამეხვა. მოკლედ, "რაც შევიტანე, ისევ ის გამოვიტანე იმ სახელგანთქმულ ვერსალიდან" gigi.gif. უფრო სწორად უარესი გამოვიტანე, თმა დასუსტდა რაღაცნაირად და გამოშრა. მას მერე რამდენჯერმე შევიჭერი და მერე ჩადგა ნელ-ნელა ძველ ფორმაში.

Posted by: Solveig 27 May 2009, 14:20
minerva
არა, თმა ბოლოს 2006 წლის ზამთარში შევიღებე და მას შემდეგ ხელი აღარ მიხლია.

ისე, თმის მოვლის ზოგიერთი საშუალება მეც ალერგიას მაძლევს.

Posted by: Cor-toni 27 May 2009, 15:02
Solveig

რკინის რაოდენობა გამოიკვლიე შრატში. ქალებში ხშირია. Mით უფრო, არასრულფასოვან კვებას თუ უკავშირებ. ფრჩხილებიც ხომ არ გიშრევდება? ან მენზის შემდეგ ხომ არ ძლიერდება?

Posted by: Solveig 27 May 2009, 19:00
Cor-toni
არა, არანაირი ციკლური ცვლილებები არ ახასიათებს. ფრჩხილები ნორმალურადაა..


უშუალოდ რკინა არ გამომიკვლევია, მაგრამ ბოლოს რომ გავიკეთე სისხლის ანალიზი, ერითროციტები, ჰემოგლობინი, ჰემატოკრიტი ნორმაში იყო.


მადლობა ყველას რჩევებისათვის smile.gif

Posted by: sofi-18 27 May 2009, 21:11
გემოგლობინი რამდენია ნორმა? smile.gif

გემოგლობინი რამდენია ნორმა? smile.gif

Posted by: Unico 27 May 2009, 23:38
კითხვა მაქ: ანტი სტაფილოკოკური ფაგების გამოყენება რამდენად ეფექტურია?
smile.gif

Posted by: aladini 28 May 2009, 14:41
Romina
QUOTE
aladini
ნეიროქირურგთან მიდი აუცილებლად. მე გირჩევდი 9-ე საავადმყოფოში გველესიანს. ნამდვილად არ გეტყვის ოპერაციას თუ არ გჭირდება. არის სხვადასხვა პროცედურები, რაც მდგომარეობას შეამსუბუქებს, მაგრამ მხოლოდ გარკვეულ სტადიაზე. ამიტომ ჯერ ნეიროქირურგთან მიდი.

დიდი მადლობა
თითქმის გადაწყვეტილია ოპერაცია, თოდუაში, მამაშვილ ჭეიშვილებთან
მაგ გველესიანზეც მითხრეს, გაესინჯე აუცილებლადო და ალბათ შაბათს ყოველი შემთხვევსათვის მივალ მაიჰნც
მადლობა smile.gif

Posted by: Solveig 28 May 2009, 14:51
Unico
QUOTE
კითხვა მაქ: ანტი სტაფილოკოკური ფაგების გამოყენება რამდენად ეფექტურია?

გააჩნია, ფაგოგრამა როგორია. შეიძლება, საერთოდ რეზისტენტული იყოს ფაგების მიმართ.

ასე რომ, ექიმის და გამოკვლევის გარეშე თვითმკურნალობა არ ღირს.

Posted by: teoj 28 May 2009, 19:21
საერთოდ თუ ძალიან ვინერვიულებ რამეზე, ვხდები ავად
რამდენიმე დღის წინ ვინერვიულე, დილით გამონაყარი მქონდა, კოღოს ნაკბენივით, მერე რომ მოიქავებ და ღილებივით დაგაჯდება ეგრე, ხოდა ეს რამდენიმე დღეა ახალ ახალი გამომდის, სახეზე არა
მაგრად მექავება , არადა ძაან ღიზიანდება თუ მოვიფხან
ხოდა, არ ვიცი მართლა ნერვიულობის ბრალია, არ ვიცი რამემ მაწყინა,არ ვიცი რამე სხვა.. ფაქტია რომ შემაწუხა ძალიან
ეს კიდევ არ ჩერდება,
რა შეიძლება იყოს? და რას მირჩევდით rolleyes.gif




Posted by: natalialuka 28 May 2009, 22:46
რა მაინტერესებს იცით, პატარა მყავს 10 თვის ბიჭი, 6თვის იყო გავუკეთეთ თიაქრის ოპერაცია, მალევე გაგვიცივდა, იმის მერე საკმაოდ ხშირად ცივდება, იმუნიტეტი ვერ აღიდგინა, თან ძუძუზეა და ვიტამინები დავალევიო თუუუ?? ყელი უცივდება ადვილად და უბრალო სურდოზეც კი სიცხეს აძლევს sad.gif

Posted by: Solveig 29 May 2009, 20:29
teoj
რამე ხომ არ გკბენს?

ბაღლინჯო, მაგალითად.

Posted by: teoj 29 May 2009, 20:55
QUOTE
teoj რამე ხომ არ გკბენს?

ბაღლინჯო, მაგალითად.

დღეს უკეთ ვარ რა შედარებაა, არა კბენა გამორიცხულია, თუმცა მაგასაც ვფიქრობდი.
კომპთან, ეზოში, აივანზე, მომენტალურად დამეწყებოდა ქავილი და გამომაყრიდა, მადლობა ყურადღებისთვის, ალბათ რამემ მაწყინა 2kiss.gif

Posted by: izabel 30 May 2009, 08:29
Solveig
ერთი პერიოდი მე მცვიოდა თმა ძალიან და მერე დამიდგინდა ანემია. სისხლი შეამოწმებინე.

Posted by: riosweetie 31 May 2009, 11:19
გამარჯობათ, დახმარება მჭირდება ორსულად ვარ და ყელი მტკივა საშინლად. დღეს დილით ხმაც შემეცვალა. რა სავლები ან რაიმე საშუალება შეიძლება რომ გამოვიყენო? სახლში მაქვს ბიოპაროქსი, კი უწერია ორსულებში შეიძლებაო მაგრამ მეშინია

Posted by: WUDU 1 Jun 2009, 10:49
არ ვიცი სწორ თემაში ვწერ თუ არა, მაგრამ ამ ბოლო დროს ძალიან მაწუხებს გაზები და შებერილობა და იქნება მირჩიოთ რამე.

Posted by: shanyva 1 Jun 2009, 12:54
WUDU
ესპუმიზანი მიიღე,მე ძალაინ მიშველააა smile.gif)

Posted by: GT.ge 1 Jun 2009, 13:48
მოკლედ, ახლა რა ხდება...
მქონდა კონტაქტი ბავშვთან, რომელსაც აქვს წითურა (დღეს არის მერვე დღე).
გამომიცხადეს 21 დღიანი კარანტინი, საკუთარ ბავშვს არ მაკონტაქტებენ... 18-ში აცრა უნდა გაუკეთდეს და ვირუსები არ გვინდაო. ორგანიზმში 21 დღე რჩება წითურაო.
წითურიანი ბავშვის ექიმმა გვითხრა, 9 დღის მერე ორგანიზმიდან ეს ვირუსი გადისო...
ჩავთვალე საჭიროდ, აქ მაინც ვიკითხო და დავაზუსტო: ვის დავუჯერო, ოჯახს თუ ექიმს?

Posted by: WUDU 1 Jun 2009, 14:22
shanyva
QUOTE
ესპუმიზანი მიიღე,მე ძალაინ მიშველააა

რა დოზებით მივიღო?

Posted by: Ze Moreira 1 Jun 2009, 22:37
დიაზეპამმა შეიძლება სუნთქვის გაძნელება და სპაზმური შეტევა გამოიწვიოს?

Posted by: texasuri jleta benzoxerxit 2 Jun 2009, 02:38
QUOTE (Ze Moreira @ 1 Jun 2009, 22:37 )
დიაზეპამმა შეიძლება სუნთქვის გაძნელება და სპაზმური შეტევა გამოიწვიოს?

თუ ალერგიული ხარ მის მიმართ- კი... თუმცა იშვიათია ბენზოდიაზეპინებზე ალერგია

Posted by: ninulia 2 Jun 2009, 13:09
ერთი კვირააა დავიწყე სირბილიიიიი,ბევრიცცც არა 15წუთი დავრბოდიი საღამოობითთ,ორივე ფეხზე კოჭთან შესიებული მაქვსს და თAნ ხელსაც რო ვიდებ მტკივაა,ალბათ ერთი კვირააა ესე ვარრრ და სიარულიცც მიჭირსსსს help.gif help.gif help.gif help.gif

Posted by: GT.ge 2 Jun 2009, 15:06
ninulia
დაისვენე ცოტა ხანს. გაგივლის და მერე დაიწყე 3-5 წუთიდან ძაან ნელი ძუნძულით. smile.gif
საღამოს დაღლილიც იქნებოდი...

Posted by: katka 4 Jun 2009, 12:29
GT.ge
ოჯახს ოჯახს დაუჯერე.

Posted by: sofi-18 4 Jun 2009, 23:45
გემოგლობინი რამდენია ნორმა?



-----------------------------------------

Posted by: shanyva 5 Jun 2009, 10:55
WUDU
2 აბი სამჯერ დღეში,ერთი კვირა,მე ასე დამინიშნეს და მიშველაა

Posted by: oups 5 Jun 2009, 11:15
ninulia
ე.ი. საღამოობით შეიძლება სირბილი?


Posted by: GT.ge 5 Jun 2009, 12:50
katka
გმადლობთ smile.gif

p.s. რაფერ ხარ? 2kiss.gif
* * *
oups
QUOTE
საღამოობით შეიძლება სირბილი?

ზოგის აზრით, საღამოს უფრო სასარგებლოა ორგანიზმისთვის ვარჯიში... დილით, ახალ გამოღვიძებული ორგანიზმის დატვირთვა არ შეიძლებაო. მე, რომ საღამოს ველოდო, სამსახურის მერე სახლში მისვლის თავი მაქვს მხოლოდ... biggrin.gif
ვარჯიში საერთოდ სასარგებლოა მთელი დღის განმავლობაში, ოღონდ ნორმა. შესაძლ;ებლობებზე გადამეტება არ შეიძლება.
უნდა დაიწყო სიარულით 1-2-3 კმ. მერე, შიგადაშიგ ძუნძულიც უნდა გაურიო ცოტა-ცოტა... და ა.შ.
სირბილის (ანუ, ძუნძული. სირბილის წინააღმდეგი ვარ, არაა საჭირო. ირბინონ ეთიოპიელებმა) დაწყების წინ, ცურვაზე ერთი თვე სიარული მთლა კარგია, ჩემი გამოცდილებით.

Posted by: lizofobi 5 Jun 2009, 17:15
მოგესალმებით ამ ეტაპზე gigi.gif გთხოვთ გამარკვიოთ, რაზე შეიძლება მეტყველებდეს ბოლო 4-5 თვის განმავლობაში ტემპერატურის მატება 37.5 მდე და მერე ისევ ნორმაზე დაბრუნება? ანალიზები ნორმალურია, ფილტვები თავისუფალიუ...მერე მეგობარი ცუდად გამეღადავა შიდსი გაქო biggrin.gif ხოდა რაღაც ჭკუაში დამიჯდა ეს აზრი biggrin.gif 5 თვე ეს იდიოტური ხტუნვა ტემპერატურის 36 დან 37.5 მდე და მერე ისევ 36 ზე ჩამოსვლა შიდსს შეიძლება ახასიათებდეს? spy.gif gigi.gif სარისკო ქცევებიდან აღვნიშნავ უსიამოვნო შემთხვევას მანიკიურშასთან და სტომატოლოგთან...

Posted by: ninulia 6 Jun 2009, 00:36
oups
კიიი ესე 11ისტვის დავრბივარ ხოლმე ბავშვს რო მივაძინებ მერეე, გახდომისთვისაც დაა ჯანმრთელობისთვისაც ძალიან კარგიაა,ეხლა ამინდების გამოოო ვეღარ დავრბივარრრრრრრ user.gif user.gif

Posted by: oups 6 Jun 2009, 10:42
GT.ge
ხო, მე დილით ვერ ვიტან ადგომას, სამსახური რომ არა 3-4 საათზე გავიღვიძებდი.

ხოდა ძალიან გამიხარდა რომ შეიძლება სირბილი საღამოობითაც bis.gif მშვენიერია

Posted by: Nensi_1988 8 Jun 2009, 20:01
არ ვიცი ამაზე თერაპევტს უნდა მივმართო თუ არა მაგრამ მოკლედ რა მაინტერესებს : იღლიაში ერთ მხარეს მთელს სიგრძეზე მაქვს სულ ოდნავ სესიებული და გამაგრებული. აშკარად არაა ისეთივე სირბილე როგორც მეორე იღლიაში. საქმე იმაშია რო ტკივილები არ მაწუხებდა აქამდე და "საროჩკის" ვიწრო მკლავმა გამიღიზიანა და აშკარად ვიგრძენი რო ამტკივდა, ხელი რო მივიდე მთელ სიგრძეზე გამაგრებული იყო (საშინლად არა მარა საშუალოდ) ხელი რომ მივიჭირე ცოტაზე ოფლმა დამასხა და ოდნავი აკანკალება ვიგრძენი. რა შეიძლება იყოს ეს? და ვის მივმართო? კვანძი არ არის (ხელში არ მომხვდა კვანძისმაგვარი არაფერი) გთხოვთ იქნებ იცოდეთ რამე ამის შესახებ

თუ ეს თერაპევტის საქმე არაა იქნებ მითხრათ ვის მივმართო?

Posted by: shtori 8 Jun 2009, 23:08
თავის ქალა მიტკაცუნებს თუ რავი, რაღაც ტკაცუნის ხმებს გამოსცემს. დღეს დილით იე შემეშინდა მეთქი თავი არ გამისკდეს biggrin.gif უკანა მხარეს დამიწყო ტკაცატკუსი და თითქოს ჰარეი დაგროვდა თავშიო, ცოტაც და გასკდება წნევისგანო. რისი ბრალი შეიძლება იყოს? ისე წყალში თუ ჩავყვინთავ ხოლმე მაშინაც ასე ტკაცუნობს ხოლმე, ყინული რომ იბზარება ისეთი ხმა აქვს წვრილი და სუფთა. მართლა შეიძლება ისე თავი გამისკდეს? rolleyes.gif ან რის გამოა ესე? საშიშია?
ხო და კიდევ წნევა მივარდება ხოლმე თუ სულ დავარდნილი მაქვს ამ ბოლო დროს. რამოდენიმე თვეა უკვე. ხან კარგად ვარ და უცებ გავბრუვდები, თვალები მებინდება და მაშინ თუ შევამოწმებ 80-60 ან 70-50 ან სადღაც ასე მაქვს ხოლმე. ხან გაღვიძებისთანავე ესე ვარ დარეტიანებული. ამასთან გულისრევის შეგრძნებაც და გულისწასვლის... ჟენშენის ნაყენი დალიეო მითხრეს დღეში 2-3ჯერო. კაია ვითომ? სხვა რა შეიძლება რომ მივიღო წნევის დასარეგულირებლად? და საერთოდ რა იწვევს წნევის ვარდნას?

Posted by: ana_nushka 9 Jun 2009, 00:46
ექიმებოო !!!! თქვენი რჩევა მჭირდება . . . rolleyes.gif

რამოდენიმე ხნის წინ მქონდა ნერვოზიი . . . drug.gif (საკმაოდ გამოხატული, ფორმით. ნერვული სპაზმებით და მთელი ამბებით . . . )

იმის მერე ძალიან მგრძნობიარე გავხდიი . . .ყველაფერი გულთან ახლოს მიმაქვს და ყველა წვრილმანზე ვნერვიულობ . . . baby.gif მეშინია ამ ჩემმა ნერვიულობამ ისევ არ დამიბრუნოს ნერვოზი . . . drug.gif

იქნებ მირჩიოათ რაიმე მსუბუქი დამაშვიდებელიიი (ვალერიანის და ვალოსედის გარდაsmile.gif ) პროფილაქტიკისთვის . . . .

წინასწარ მადლობა . . . smile.gif

Posted by: RAULA 9 Jun 2009, 11:56
lizofobi

ნერვოზი , სხვა სახის ანთებები smile.gif ანალოგიური აქვს 2 კვირაა და ეხლა კარდიოლოგთან უნდა წავიდე , ( ფილტვები სუფთა მაქვს ) ეჭვი მაქვს გულის კუნთის ანთება უნდა იყოს , როცა სიცხე გაქვს უმიზეზოდ არ შეიძლება იყოს , რაღაცის ბრალია და უნდა მიხედო wink.gif

Posted by: xoze 10 Jun 2009, 09:11
ძალიან მაინტერესებს, რამდენად ეფექტურია ,,სალხინო'' რეალურად და არა ის, რასაც ტელევიზორით ვისმენთ. მაინტერესებს იმათი აზრი, ვინც ეს პრეპარატი პრაქტიკულად გამოიყენა.

Posted by: lilucia 10 Jun 2009, 22:01
მაინტერესებს თუ შეიძება ვიტამინების და ალერგიის წამლის ერთად მიღება?
გთხოვთ მიპასუხეთ...

Posted by: cvanca 10 Jun 2009, 23:57
ესე იგი დღეს გავიკეთე სისხლის საერთო ანალიზი , და ნორმებში არ ჯდება ეს სამი რამ: ჰემოგლობინი მაქ - 112 (67) ;ლეიკოციტები - 9.5 ; ედს - 18 . ექიმმა მითხრა რომ ანთებითი პროცესია, მაგრამ რეალურად ამის მიზეზი ვერ მითხრა .მერე რაღაცეები მკითხა და გამოვლინდა ფარისებრი ჯირკვლის პრობლემა და ექომაც აჩვენა მისი შემცირება .ხოდა რა მაინტერესებს სისხლი რა შუაშიია ფარისებრთან ?შეიძლება გამოიწვიოს მისი ცვლილება? და რას ნიშNავს სისხლის ანთებითი პროცესი?ხომ არ ჯობია გადავამოწმო სხვაგანაც ? ;( ვეღარ გავიგე.ცოტა შემეშინდა კიდეც sad.gif

Posted by: jashusha 11 Jun 2009, 14:16
კატარა რა არის იქნებ ამიხსნათ?
ყელის ამბავი რომაა ვიცი და მეტი ვერ გავიგე კარგად. ოდონორმი გამომიწერეს.
საშიში ხომ არაა ?

Posted by: skoch 13 Jun 2009, 11:16
QUOTE
კიიი ესე 11ისტვის დავრბივარ ხოლმე ბავშვს რო მივაძინებ მერეე,

ninulia
biggrin.gif მე 4სთვის ვვარჯიშობ ყოველ დრე შუადღეს ამ დროს კუნთი ყველაზე აქტიურია მარა შენი დროც კაია დილით არ შაილება tongue.gif
* * *
QUOTE
თავის ქალა მიტკაცუნებს თუ რავი, რაღაც ტკაცუნის ხმებს გამოსცემს. დღეს დილით იე შემეშინდა მეთქი თავი არ გამისკდეს

lol.gif lol.gif lol.gif lol.gif lol.gif lol.gif :lolთავის კალის ძვლები მოძრაობსana_nushka
QUOTE
იქნებ მირჩიოათ რაიმე მსუბუქი დამაშვიდებელიიი (ვალერიანის და ვალოსედის გარდა ) პროფილაქტიკისთვის . . . .

ყველაზე კაი რელაXანტი ცურვა ვარჯიში სუფთა ჰაერია
QUOTE
ძალიან მაინტერესებს, რამდენად ეფექტურია ,,სალხინო''

ჩევი ფლორის საოცრებაა და როგორ შეილება ეფექტური არ იყოს boli.gif

Posted by: freeandhappy 13 Jun 2009, 21:57
ე.ი ვეგეტო დისტონია მჭირს ნუ როგორც ექიმმა დაასკვნა. ყლაპვა მიძნელდება, სუნთქვა. ამ ბოლო დროს, მაქვს ასევე შაკიკის სიმპტომები და მიგრენი ანუ ციმციმი, ამ ბოლო დროს კი, როცა ადრე ნახევარი მხარე მიბუჟდებოდა და ა.შ, ახლა მაქვს ჰაერის უკმარისობა, სპაზმის შეგრძნება კუჭის თავზე. ხელის გულზე კი აი მაისურზე რომ დავისვავ ან რამე საგანს ისე ვერ ვგრძნობ დაბუჟებას, მხოლოდ რომ ვეხები რამეს, რისი ბრალი იქნება. ალბათ ისევ ეს დისტონიაა არ ვიცი,ან რითი ვიმკურნალო. ხანდახან მგონია რომ შევიწროების მაგივრად პირიქით მაქვს. ნუ არ ვიცი, თუ საკმარისად გადმოვეცი თუ რა მჭირს, იქნებ მირჩიოთ რა ვქნა. მადლობა წინასწარ

Posted by: love_u 16 Jun 2009, 02:27
ვინმეს რჩევა მჭირდება სასწრაფოდ. ფილტვები მტკივა საშინლად. ზამთარში ამტკივდა პირველად და მერე პერიოდულად მტკიოდა. სუნთქვა მიჭირდა ხოლმე მაგ დროს. ზუსტად ერთი კვირის წინ კუს ტბაზე საშინლად დავსველდი და იმის მერე მხველებს. ყელის ტკივილის წამლები ვსვი, ხველების საწინააღმდეგოც, მაგრამ არ მშველის. ჩემით დავასკვენი რომ ფილტვების ბრალია. მოკლედ ექიმთან ვერ მივდივარ, რეენდგენზე. დედაჩემმა არ იცის რომ ვეწევი და მაგიტომ ვერ ვეუბნები. ვინმემ რამე მირჩიეთ ან ის მაინც მითხარით , დაახლოებით რა ჯდება რენდგენი ფილტვებზე. ჩემით რო მივიდე და გადავიღო sad.gif

Posted by: texasuri jleta benzoxerxit 16 Jun 2009, 22:05
QUOTE (love_u @ 16 Jun 2009, 02:27 )
ვინმეს რჩევა მჭირდება სასწრაფოდ. ფილტვები მტკივა საშინლად. ზამთარში ამტკივდა პირველად და მერე პერიოდულად მტკიოდა. სუნთქვა მიჭირდა ხოლმე მაგ დროს. ზუსტად ერთი კვირის წინ კუს ტბაზე საშინლად დავსველდი და იმის მერე მხველებს. ყელის ტკივილის წამლები ვსვი, ხველების საწინააღმდეგოც, მაგრამ არ მშველის. ჩემით დავასკვენი რომ ფილტვების ბრალია. მოკლედ ექიმთან ვერ მივდივარ, რეენდგენზე. დედაჩემმა არ იცის რომ ვეწევი და მაგიტომ ვერ ვეუბნები. ვინმემ რამე მირჩიეთ ან ის მაინც მითხარით , დაახლოებით რა ჯდება რენდგენი ფილტვებზე. ჩემით რო მივიდე და გადავიღო sad.gif

გაშუქება 15 ლარი,გადაღება 30. გაეშუქე

Posted by: magdalina 17 Jun 2009, 11:15
გამარჯობა 2 დღეა მარცხენა ხელი მიბუჟდება და მტკივა იღლიის მიმდებარე ადგილას,ზოგჯერ სუნთქვაც მიჭირს,თითქოს მკერდი მტკივა თითქოს გული sad.gif ძაან გაურკვევლობაში ვარ,ვერც იმას ვხვდები რის ექიმთან მივიდე,
ძაან გთხოვთ მირჩიეთ რამეეეეეე help.gif

Posted by: Sattelite 18 Jun 2009, 17:05
20 წლის ვარ.
ბუღალტერი ვარ და სამუშაოც ძირითადად მჯდომარე მაქვს თან საკმაოდ დაძაბულიც...

1.ყლაპვა მაქვს გართულებული...
2.ყელი ძალიან წითელი მაქვს...
3.ღამე მეღვიძება შიშით და გულის აჩქარებით...
4.ხმა მიწყდება ხშირად...
5.ბოლო დროს მადა მიქვეითდება...
6.წვას ვგრძნობ ზოგჯერ გულმკერდის არეში...
7.მჟავიანობა მაქვს მომატებული დასწვრივი კოლინჯის პრობლემები მაქვს რის გამოც გაზებიც მიგროვდება...
8.მარცხენა ხელი და ფეხი მიბუშდება ზოგჯერ(მარცხენა მხარეს ყელიც შესიებული მაქვს ყურიც მაწუხებს და თვალიც)...
9.ყბაყურა და სხვა სანერწყვე ჯირკვლები მაქვს გადიდებული...
10.ეს ყველაფერი ხდება პერიოდულად...

თერაპევტმა მითხრა ანგინა და ვდს-გაქვსო...
ყელ-ყურ-ცხვირმა მწეველის ანგინა დამიდგინა და მოწევა ამიკრძალა რასაც კარგად ვასრულებ...

რა უფრო შეიძლება იყოს?

smile.gif.

* * *
magdalina

ზუსტად იგივე მჭირს მეც კარგად მესმის შენი სიტუაცია...
საშინელებაა...


Posted by: varda 18 Jun 2009, 21:10
პრეპარატ асд 2 -ზე თუ გსმენიათ რამე? თუ იცით მართლა აჩერებს კიბოს?
გთხოვთ შემატყობინოთ. მეგობარს მკერდის სიმსივნე აქვს და ძალიან მინდა დავეხმარო.
წინასწარ გიხდით მადლობას.

Posted by: *hedgehog* 19 Jun 2009, 10:43
ასეთი რაღაც მინდა გკითხოთ.

მარცხენა მხარეს, ლავიწის ძვლის ქვემოთ მტკივა ძალიან, საკმაოდ დიდი ხანია. მუდმივად არ მტკივა, პერიოდულად და რაგაცნაირად მისივდება. მერე ეს ტკივილი გადადის მთლიანად მარცხენა ხელში. რისი ბრალი შეიზლება იყოს და რომელ ექიმთან შეიძლება მივიდე ამ პრობლემიტ?

Posted by: kent055 19 Jun 2009, 12:33
მოგესალმებით, ვარ 41 წლის მაქვს ფილტვების პრობლემა, კერძოდ ქრონიკული პნევმონია, იყო ეჭვი ტუბერკულოსზე, მაგრამ ბოლო გამოკვლევებით გამოირიცხა. ვიზუალურად მაქვს ზოგჯერ ჰაერის უკმარისობა, აგრეთვე მაქვს c ჰეპატიტი, მაქვს ძგიდის გამრუდება და ამის გამო მიწევს მთელი ცხოვრება პირით სუნთქვა. ანალოგიური პრობლემით დაავადებულმა ჩემმა ახლობელმა მიმართა 2-3 წლის წინ იოგას ვარჯიშებს, იოგას გეგმით კვებას და სუნთქით პროცედურებს. შედეგი განსაცვიფრებელი ადამიანს სიცოცხლის ხალისი დაუბრუნდა და შეიძლება ითქვას განიკურნა იოგას მეშვეობით. თქვენ როგორც პროფესიონალებს რას მირჩევდით იოგას ვარჯიშები შესახებ. გთხოვთ მიპასუხოთ, წინასწარ გიხდით მადლობას, უფალი იყოს ჩვენი მფარველი.

Posted by: Kaifistka 19 Jun 2009, 16:36
texasuri jleta benzoxerxit
LULA_QABABI
Romina

ამ თემაში ადრე დავწერე, ჩემს დაავადებაზე ლაიმას ბორელიოზზე და ახლა მკურნალობა გამიუარესდა, მიუხედავად მიღებული ანტიბიოტიკებისაც ცუდი შედეგები მაქვს, რაც დამიკლო ნელ-ნელა ისევ იწევს და ვიღაცამ რომელიღაცა თქვენგანმა მირჩია ინფექციონისტი მგონი თუ რაღაც მაგ საქმეში გარკვეული ექიმი, სამწუხაროდ მაშინც არ მივედი და აღარ დამრჩა PM სახელი მახსოვს ალეკო და გვარი არა მგონი მეგრელიშვილი არ მახოვს და შეგიძლიათ თავიდან მომწეროთ თუ შეძლებთ? წინასწარ დიდი მადლობა.

Posted by: Cousteau 20 Jun 2009, 00:33
QUOTE (Kaifistka @ 19 Jun 2009, 16:36 )
ალეკო

ნანუაშვილი

(ამაზე განათლებული ადამიანი სამედიცინო სფეროში მე პირადად არ მინახავს) მუშაობს ინფექციური საავადმყოფოს ტერიტორიაზე (მგონი სეფსისის ცენტრია, ბოლო შენობა)

Posted by: aladini 22 Jun 2009, 13:56
http://www.aversi.ge/annotations.php?id=73...o&drug_id=21972
ა მ პრეპარატზე გსმენიათ ვინმეს რამე?

Posted by: Romina 23 Jun 2009, 09:26
Kaifistka

სამწუხაროდ გვიან ვნახე პოსტი. ძალიან მაინტერესებს და განვიცდი შენ ამბავს :-( ზემოთ ხსენებული ექიმი მართლა ერთ-ერთი ყველაზე კომპეტენტურია, აუცილებლად მიდი.

Posted by: Sattelite 23 Jun 2009, 21:00
19 წლის გოგოს რომელმაც 1 თვის წინ საკეისრო კვეთით გააჩინა ბავშვი აღენიშნება ჭრილობიდან გამჭვირვალე სითხის დენა. სამკურნალოდ იყენებს წყალბადის ზეჟანგს და ბრილიანტის მწვანეს. შეიძლება თუ არა ადგილობრივად ანტიბიოტიკის გამოყენება იმის გათვალისწინებით რომ ბავშვი ბუნებრივ კვებაზეა? (ხომ არ დაემართება ჩვილს დისბაქტერიოზი?)

წინასწარ მადლობა!

Posted by: Cousteau 24 Jun 2009, 09:05
QUOTE (Sattelite @ 23 Jun 2009, 21:00 )
19 წლის გოგოს რომელმაც 1 თვის წინ საკეისრო კვეთით გააჩინა ბავშვი აღენიშნება ჭრილობიდან გამჭვირვალე სითხის დენა. სამკურნალოდ იყენებს წყალბადის ზეჟანგს და ბრილიანტის მწვანეს. შეიძლება თუ არა ადგილობრივად ანტიბიოტიკის გამოყენება იმის გათვალისწინებით რომ ბავშვი ბუნებრივ კვებაზეა? (ხომ არ დაემართება ჩვილს დისბაქტერიოზი?)

წინასწარ მადლობა!

1 თვის თავზე ჭრილობა უკვე უნდა იყოს შეხორცებული. უნდა მიხვიდეთ ექიმთან, გაარკვიოთ რა არის ეს ''გამჭირვალე სითხე'', მერე უნდა აიღონ ამ ''სითხის'' ნაცხი, დათესონ, ნახონ მანდ რამე ხო არ იზრდება (სავარაუდოდ სტაფილოკოკი, ან სხვა რამე რაც ''ქირურგიული ჭრილობის ინფექციას'' იწვევს) ხო არ არის ინფიცირებული უფრო ღრმა შრეები და ა.შ. ანტიბიოტიკი უნდა შეგირჩიოთ ექიმმა + მან უნდა გადაწყვიტოს (ან პედიატრმა) რომელი ჯობია ბავშვი ბუნებრივ კვებაზე ''შიანარჩუნო'' თუ ანტიბიოტიკო (ან რაიმე სხვა სახის) თერაპია ჩაუტარო.

P.S. თერაპევტი არ ვარ

Posted by: Lacrimosa_ 28 Jun 2009, 14:01
თერმომეტრი გამიტყდა საწოლ ოთახში, მაქსიმალურად ვეცადე ამეკრიფა ვერცხლისწყლის პატარა ბურთულაკები, მაგრამ ბევრი ვერ მოვნახე.
როგორ მოვიქცე და არის თუ არა სიცოცხლისათვის საშიში ერთ თერმომეტრში არსებული ვერცხლისწყლის რაოდენობა?

საშინლად მეშინია

Posted by: Romina 28 Jun 2009, 18:26
Lacrimosa_
არა უშავს, თერმომეტრში ძალიან ცოტაა და საშიში არ არის. მშვიდად იყავი
smile.gif

Posted by: Lacrimosa_ 28 Jun 2009, 21:27
Romina

გაიხარე
დავმშვიდდი smile.gif

Posted by: shtori 29 Jun 2009, 23:31
რისი ბრალი შეიძლება იყოს იღლიაში (რა სიტყვაა...) პატარა ადგილას კანი გამიმუქდა დანარჩენ ნაწილთან შედარებით. კანის პრობლემაა თუ რისი? კავშირში რამდენადაა ამასთან არ ვიცი, მაგრამ რამეს რომ დავეყრდნო(კუთხოვანს ვთქვათ) თუნდაც 5 წუთით მერე ის ნაკვალევი (ჩაზნექილი კანი) 2 საათი და მეტიც გამყვება. თან ფეხები მილურჯდება თუ მოძრაობაში, არ ჰორიზონტალურ მდგომარეობაში არ ვარ. და კიდევ ჭრილობა კი მიხორცდება ადვილად მაგრამ ლაქა მრჩება ხოლმე დიდ ხანს (1წლამდეც კი).იქნებ ერთი მიზეზით იყოს ყველაფერი?
ხოდა რა შეიძლება იყოს მიზეზი და რის ექიმდათნ უნდა მივიდე?

Posted by: nini_blini 1 Jul 2009, 02:53
რამდენად შეესაბამება თემას არ ვიცი მაგრამ იქნებ მითხრათ??

რამდენიმე დღის წინ დავიწვი ბათუმში, ახლაც წითელი ვარ, კანი მაქვს დაჭიმული, ნუ ეგ გამივლის ნელ ნელა, მაგრამ აი ფეხის კოჭებთან მაქვს ცოტა შესიებულები და ეგ მაფიქრებს, ვენებისთვის საშიში ხო არაა? და რატო მაქვს ასე? სხვა არაფერი არ დამსიებია... მესამე დღეა ესე ვარ..
* * *
ვინმემ რამე მითხარით რა sad.gif

_______________________

Posted by: lizofobi 2 Jul 2009, 12:29
nini_blini
არა, ეგ პირდაპირ დამწვრობის ბრალია smile.gif მეც ეგრე ვარ ხოლმე მზეს რო მივაფიჩინდები, მერე გაღიმებაც კი მიჭირს, კოჭებზე და მაჯების მოძრაობაზე აღარაფერს ვამბობ biggrin.gif

Posted by: jashusha 2 Jul 2009, 16:01
მედიკამენტოზური ყელის სიმშრალე ასეთი ძნელი მოსარჩენია? aba.gif მთელი დღე მაწუხებს მოჭერის შეგრძნება ყელში და ფიზიოლოგიური ინგალაცია გიშველისო, ექიმმა მითხრა და ღირს რომ გავიკეთო, მირჩიეთ რა... ევკალიპტის ნაყენები, ტრავისილები და შალვეი არ მარჩენს და არც ორონორმი... help.gif

Posted by: Blind_Torture_Kill 4 Jul 2009, 17:03
დერმატოლოგი არ გვყავს ფორუმზე ?

Posted by: Tatra 5 Jul 2009, 01:53
სტიმულატორებზე ცალკე თემა არ არის, ამიტომ გადავწყვიტე აქ ვიკითხო:
არავინ იცით ისეთი სტიმულატორი, რომელიც ძილს ჩაანაცვლებს? ისეთი კი არა, რომელიც უძილობას გამოიწვევს, არამედ ისეთი, რომელიც ზრდის შრომის ნაყოფიერებას და ყურადღების კონცენტრაციას და ამცირებს ძილის აუცილებლობას.

Posted by: jashusha 6 Jul 2009, 11:35
აღარ პოსტავენ ექიმები? no.gif გვიპასუხეთ რა

Posted by: tkemali 6 Jul 2009, 12:30
nini_blini
lizofobi

ზუსტად ამ პრობლემის წინაშე ვდგავარ ახლა... ვარ საშინლად დამწვარი... მივიღე მზის დარტყმა... ზურგის გარდა ყველაფრის წვამ გამიარა... ზურგი მეწვის არანორმალურად,ვისვამ მაწონს... და ფეხებზე,კოჭებთან უკვე მეორე დღეა დასიებული მაქვს...

ხომ არ მოვკვდები??? cry.gif

დედაჩემმა დამალევინა ლერონი... მაგრამ მაგანაც არ მიშველა... მგონი თანდათან ვსივდები cry.gif ოდესმე გავხდები კარგად??? კერძოდ ზეგ.... ზეგ მაქვს გამოცდა უნარებში და ასე როგორ წავიდე... cry.gif

Posted by: Romina 6 Jul 2009, 14:56
tkemali
QUOTE
ხომ არ მოვკვდები???

არა smile.gif ნუ გეშინია.
ჰიდროკორტიზონის მალამო წაისვი იქ სადაც ძალიან შესიებულია და ბუშტუკებია.

Posted by: baby-bobo 7 Jul 2009, 15:48
მითხარით რა ეს რას ნიშნავს, რამდენად საშიშია და ვის უნდა მივმართო???

ღვიძლის IV სეგმენტში, პერიპორტულად ისახება არაკონტურირებული ექოგენური წარმონაქმნი,
რომელიც ექოსემიოტიკით შეესაბამება კეროვანი ფიბროლიპომატოზის ზონას.


Posted by: LUKA-BRAZI 8 Jul 2009, 23:34
baby-bobo
QUOTE
ღვიძლის IV სეგმენტში, პერიპორტულად ისახება არაკონტურირებული ექოგენური წარმონაქმნი,
რომელიც ექოსემიოტიკით შეესაბამება კეროვანი ფიბროლიპომატოზის ზონას.

ფიბრომა და ლიპომა კეთილთვისებიანი სიმსივნეებია და სხვადასხვა ორგანოებში ვითარდება ხოლმე. ფიბროლიპომატოზი ესაა ლიპომა, რომელიც შეიცავს ფიბრინულ ქსოვილსაც. მანდ შენ გიწერია რომ რა წარმონაქმნიც დააფიქსირეს ექოზე, შეესაბამება ფიბროლიპომატოზის ზონას და არა იმას რომ ეს წარმონაქმნი ფიბროლიპომაა, ამიტომ აუცილებელია გაარკვიო ფიბროლიპომაა მართლაც თუ სხვა რამე..... იდეაში რახან ფიბროლიპომის ადგილზეა, შესაძლებელია მართლაც ფიბროლიპომა იყოს, მაგრამ შეიძლება არც იყოს. ამიტომ უნდა გაარკვიო yes.gif

Posted by: baby-bobo 8 Jul 2009, 23:43
LUKA-BRAZI
QUOTE
ფიბრომა და ლიპომა კეთილთვისებიანი სიმსივნეებია და სხვადასხვა ორგანოებში ვითარდება ხოლმე. ფიბროლიპომატოზი ესაა ლიპომა, რომელიც შეიცავს ფიბრინულ ქსოვილსაც. მანდ შენ გიწერია რომ რა წარმონაქმნიც დააფიქსირეს ექოზე, შეესაბამება ფიბროლიპომატოზის ზონას და არა იმას რომ ეს წარმონაქმნი ფიბროლიპომაა, ამიტომ აუცილებელია გაარკვიო ფიბროლიპომაა მართლაც თუ სხვა რამე..... იდეაში რახან ფიბროლიპომის ადგილზეა, შესაძლებელია მართლაც ფიბროლიპომა იყოს, მაგრამ შეიძლება არც იყოს. ამიტომ უნდა გაარკვიო


დამატებითი გამოკვლევაა საჭირო.
და რა ან როგორი?

თუ ეს ისაა რაც დაწერე, შეიძლება ტკივილს იწვევდეს? ან რა ღონისძიებებია საჭირო?
ოპერაცია?

მადლობა დიდი.

Posted by: BadbadGirl 9 Jul 2009, 14:39
ხალხო, რევმატიზმის საშველი არის რამე?

Posted by: Romina 9 Jul 2009, 19:40
BadbadGirl
QUOTE
ხალხო, რევმატიზმის საშველი არის რამე?

როგორ არ არის საშველი smile.gif რა იცი რომ რევმატიზმი გაქვს, ექიმმა დასვა ეს დიაგნოზი?

Posted by: BadbadGirl 9 Jul 2009, 21:28
Romina
არა ექიმმა არა ამინდმა და შესაბამისად საკუთარ თავს დავუსვი დიაგნოზი biggrin.gif biggrin.gif


Posted by: Romina 9 Jul 2009, 21:50
BadbadGirl
smile.gif რევმატიზმი არ არის ეგ. მეტეოპათიური ტიპის ართრალგია ბევრს ახასიათებს. თუ მარტო ამინდზე გაქვს სახსრების ტკივილი და ეს შემაწუხებელია, შეგიძლია იბუპროფენი ან კეტონალი დალიო. მასაჟი, ვარჯიში კარგია მაგ დროს.
აუცილებლად მიდი ექიმთან (რევმატოლოგთან) თუ: ტკივილი ძლიერია, შესიებულია რომელიმე სახსარი, ამინდის ცვლილებების გარდა სხვა დროსაც თუ არის ტკივილი და მოძრაობის შეზღუდვა.

Posted by: BadbadGirl 9 Jul 2009, 22:35
Romina
ვვარჯიშობ ყოველთვის. მაგრამ ცუდი ამინდის დროს - წვიმა ძალიან უსიამოვნო შეგრძნებაა. ყველა ძვალი მტკივა ხოლმე biggrin.gif მაგრამ ასატანად, შესაბამისად არ მინდა გამაყუჩებლის დალევა.

სხვა დროს არანაირი ტკვილი არ ამქვს, არც მოძრაობის შეზღუდვა. მხოლოდ წვიმიან ამინდში, (თოვლიანში არ მახსოვს სამწუხაროდ)

გმადლობ smile.gif

Posted by: Ni-L 10 Jul 2009, 14:35
VANO T
27 წლის ქალიშვილს აწუხებს წონა,საგანგაშო არა მაგარამ აშკარად ცოტას ჭამს მაინც სუქდება,რისი ბრალი შეიძლება იყოს?და რა ანალიზები იქნება საჭიროო,მადლობა წინასწარ 2kiss.gif
* * *

ასევე აწუხებს ჭარბთმიანობა,არეული აქვს მენსტრუალური ციკლიც.
texasuri jleta benzoxerxit smile.gif smile.gif smile.gif

Posted by: kokogorgo 10 Jul 2009, 23:36

აქაც დავწერ. სასწრწფოდმჭირდება ჩინური ნემსები.

იქნებ ვინმე დამეხმაროს სად ვიშოვო.

წინასწარ , მადლობას გიხდით.

Posted by: LUKA-BRAZI 10 Jul 2009, 23:56
Ni-L
ჰორმონული ცვლა აქვს მოშლილი სავარაუდოდ. ენდოკრინოლოგის კონსულტაცია ჭირდება. ცხიმის გადანაწილება ორგანიზმში როგორ აქვს? არის ასეთი დაავადება, კუშინგის სინდრომი და დაავადება. ახასიათებს ბევრი რამ, ერთ-ერთია ცხიმის დაგროვება ტანზე, გულმკერდის ზემო მიდამოში, სახეზე, ხოლო კიდურებზე ცხიმი არ გროვდება + მენსტრუაციული ციკლის არევა. ჭარბთმიანობა ჰირსუტიზმის ნიშანია. უფრო ზუსტად ენდოკრინოლოგი და სისხლის ანალიზი ჰორმონებზე გამოავლენს დანარჩნს.

smile.gif

Posted by: texasuri jleta benzoxerxit 11 Jul 2009, 03:20
LUKA-BRAZI
და ჰიპოთირეოზი... boli.gif მოკლედ ემდოკრი(მი)ნოლოგი ყველა ვარიანტში

Posted by: პერწკლი 20 Jul 2009, 16:21
გულისრევის შეგრძნება მაქვს მთელი დღეა გაურკვევლი მიზეზის გამო fingal.gif
საზამთრო ვჭამე გუშინ და მაგის ბრალი შეიძლება იყოს?
ან სიცხის mo.gif

Posted by: ikebana 20 Jul 2009, 23:50
ტრავმატოლოგი მირჩიეთ რა.

ჩემს ქმარს ნეკნი აქვს მგონი მოტეხილი, დაეცა.

Posted by: Natuka NGN 21 Jul 2009, 01:20
ikebana
დაიტიო სადაა, იმას დაეკონტაქტე და ის გირჩევს... ადრე თუ მოძებნი ადრე მის პოსტებს იქ უწერია...

Posted by: ikebana 21 Jul 2009, 01:22
Natuka NGN
თამ წავიკითხე, მაგრამ ვერ ვიპოვე. ღუდუშაურშია მემგონი ეგ კლინიკა.

Posted by: MALI 21 Jul 2009, 09:47
ტუჩები მექერცლება და სისხლი მდის, თუ პომადას წავისვამ...
ზამთარში სიცივის ბრალი მეგონა და ჰიგიენურს ვხამრობდი..მაგრამ ეხლა გამირთულდა ...
ქიმიურ პომადებს არ ვხმარობ ...
ა ვიტამინიც ვიხმარე,, მაგრამ არ მიშველა..
.რომელ ექიმს უნდა მივაკითხო?

Posted by: chxira 24 Jul 2009, 17:18
რა ვქნა აღარ ვიცი... სახლში მეუბნებიან რომ ოპერაცია არ არის საჭირო, ქირურგი ამბობს რომ აუცილებელი არ არის,
მაგრამ მაინც ჯობია ოპერაციის გაკეთება. ქირურგებს მართლა უყვართ ოპერაციები? mo.gif

Posted by: SPC 24 Jul 2009, 21:19
უცნაური რაღაც დამემართა, 5 წუთის წინ დაბლა მაღაზიიდან საზამთრო ამოვიტანე და 8 კილო იყო მძიმე,,, ნუ ორივე ხელით კი ამოვიტანე, მაგრამ რომ დავდე მაგიდაზე მაგის მერე ხელები მიკანკალებს (აიი უფრო მკლავის არეში) და ძალიან დაძაბულია. რავიცი რამე მყესი ხო არ გაწყდა თქო...
ლაშქრობებში დავდივარ და 10 კგიანი ზურგჩანთებიც მითრევია მაგრამ ასეთი არ დამმართნია რაღაც დავიგრუზე ცუდი შეგრძნებაა. 1 საათში თუ არ გაიარა რამე უნდა ვიღონო ასე არ დამრჩეს

Posted by: sofi-18 25 Jul 2009, 19:58
თვალების გარშემო ჩამიშავდა,,, (ისე როგორც მენსტრუაციის დროს იცის ხოლმე, მაგრამ ახლა არ ველოდები)..
აგერ ორი კვირაა ასე ვარ. რისი ბრალი შეიძლება იყოს?

Posted by: სკაინეტა 27 Jul 2009, 08:24
ხალხო ტვინის შერყევის დიაგნოზი როგორ დავსვა სახლის პირობებში ?
მამაჩემი ამტკიცებს შერყევა მაქვსო
თავი აარტყა რაღაცას ...
ექიმებზე ალერგია აქვს და ყოველთვის მე მიწევს ხოლმე მკურნალობა
გულის რევის შეგრძნება აქვს და თავი სტკივა

პ.ს. ექიმი არ ვარ wink.gif

Posted by: chxira 27 Jul 2009, 15:33
ამ თემაში პასუხის გასაცემად შემოდის ვინმე? არა?
აბა რის მაქნისია გზა, თუკი ტაძართან არ მიმიყვანს?! biggrin.gif

Posted by: texasuri jleta benzoxerxit 27 Jul 2009, 15:44
სკაინეტა
სავარაუდოდ ტვინის შერყევა ნამდვილად აქვს,თუ გულისრევას და ტავის ტკივილს უჩივის. სასწრაფოდ ნევროლოგის კონსულტავია. შესაძლებელია გამოუსწორებელ შედეგებამდე მივიდეს უმკურნალებელი ტვინის შერყევა

Posted by: Zotz 27 Jul 2009, 15:57
იქნებ ვინმემ მირჩიოთ სად დავსვა ეს შეკითხვა: დაბალი წნევა როგორ დავირეგულირო და რა შეიძლება იყოს გამომწვევი მიზეზი?!

Posted by: texasuri jleta benzoxerxit 27 Jul 2009, 16:13
QUOTE (Zotz @ 27 Jul 2009, 15:57 )
იქნებ ვინმემ მირჩიოთ სად დავსვა ეს შეკითხვა: დაბალი წნევა როგორ დავირეგულირო და რა შეიძლება იყოს გამომწვევი მიზეზი?!

მიზეზი შეიძლება ძალიან ბევრი იყოს, მათ შორის ეს შეიძლება შენთვის ნორმის ვარიანტიც კი იყოს.

ფარისებრი ჯირკვლის ჰიპოფუნქციაა გამოსარიცხი პირველ რიგში. და ელემენტარულად,კონსულტაცია თერაპევტთან

Posted by: Zotz 27 Jul 2009, 17:17
texasuri jleta benzoxerxit


QUOTE
მიზეზი შეიძლება ძალიან ბევრი იყოს, მათ შორის ეს შეიძლება შენთვის ნორმის ვარიანტიც კი იყოს.

ფარისებრი ჯირკვლის ჰიპოფუნქციაა გამოსარიცხი პირველ რიგში. და ელემენტარულად,კონსულტაცია თერაპევტთან


ჩემს მეგობარს აწუხებს და მაინტერესებდა, ჩვეულებრივზე დაბალი წნევა აქვს, რაც გამოიხატება სისუსტეში და მომენტებში თავბრუსხვევაში. მოკლედ ნევროზის მკურნალობას მორჩა და ეხლა თითქოს უკეთ არის, მაგრამ ეს დაბალი წნევა უშლის ხელს. რა შეიძლება გააკეთოს?!

პ.ს. წინასწარ მადლობთ!

Posted by: სკაინეტა 27 Jul 2009, 22:11
texasuri jleta benzoxerxit
QUOTE
სავარაუდოდ ტვინის შერყევა ნამდვილად აქვს,თუ გულისრევას და ტავის ტკივილს უჩივის. სასწრაფოდ ნევროლოგის კონსულტავია. შესაძლებელია გამოუსწორებელ შედეგებამდე მივიდეს უმკურნალებელი ტვინის შერყევა

ექიმთან არ მივა no.gif
მე რა შემიძლია რომ გავუკეთო ?
მე ვარ მაგისი პირადი ექიმი (პროგრამისტის დიპლომით)

სისხლდენა შევუჩერე ერთხელ კუჭიდან იკაიფეთ რა drug.gif
ქირურგთან გავიარე კონსულტაცია და ამიხსნა რა წამლები უნდა გამეკეთებინა და როგორ უნდა ჩამეყენებინა დრენაჟი

Posted by: lurdes 27 Jul 2009, 22:51
როგორც ჩანს ოფთალმოლოგი ბოლო დროს კითხვებს არ პასუხობს და ამიტომ აქ დავწერ, იქნებ მიპასუხოთ

ოთხი დღეა თვალები მაწუხებს. მედ.ჯი-ზე წავიკითხე, რომ ქრონიკული კონიუნქტივიტის ნიშნები მაქვს. ეს დღეები ვიწვეთებდი ალბუციდის ხსნარს. შედეგი არ მაქვს, თვალები მეტად მექავება და გამონადენიც მომემატა. ექიმთან მისვლას უახლოეს ორ დღეს ვერ მოვახერხებ და ექიმის გარეშე თუ შემიძლია რაიმე პრეპარატის მიღება?

Posted by: RAULA 28 Jul 2009, 09:36
შეკითხვა : დაახლოებით რამდენხანში გადის გლანდების ოპერაციის მერე ის შეგრძნებები რაც იყო მანამდე /?? სახსრები და ა.შ smile.gif სრულად როდის გაიწმინდება ორგანიზმი ??

Posted by: Cousteau 28 Jul 2009, 12:17
რკინადეფიციტური ანემიის მკურნალობის გაიდლაინი ხო არ გაქვთ ვინმეს?

რომელ ''რკინას'' უნიშნავთ პაციენტებს?

რა დოზებით? ჰარისონში წერია 300 მგ უნდა მიიღოს სულ დღეღამეში...
+ ჰემოგლობინის კონტროლი რამდენ ხანში ერთხელ არის საჭირო?

Posted by: jashusha 29 Jul 2009, 15:12
გოლდ რეის ვიტამინმა შეიძლება ხელის დაბუჟება გამოიწვიოს? ოდნავ დამიბუჟდა მარცხენა ხელი და მეშინია.

Posted by: skoch 29 Jul 2009, 16:25
jashusha
გარდა დაბუჟებისა რამე ხომ არ გტკივა ან შემაწუხებლად გექავება???

Posted by: jashusha 29 Jul 2009, 17:17
არა, ალერგიული რეაქცია არ მაქვს. sad.gif

Posted by: ქეთო 5 Aug 2009, 23:30
ვერ ვერკვევი რომელ ექიმს რა ჰქვია და ამიტომ აქ დავსვამ შეკითხვას თუ არ გამიბრაზდებით smile.gif


ბოლო დროს ასე უაზროდ დამიტრიალდება თავში რაღაც და გონებას ვკარგავ sad.gif ვითიშები წამიერად..

გადაღლილობას ვაბრალებ..ან იმას,რომ კვირაში 1-2ჯერ მიწევს მგზავრობა მაღალმთიან რაიონებში სამსახურის გამო..ზღვის დონიდან 2000მეტრზე. შეიძლება წნევის ცვალებადობის ბრალია, შეიძლება დაღლილობის, შეიძლება ნერვიულობის, რადგან მაქვს რაღაც განსაცდელები..

რას იტყვით თქვენ?

ვნერვიულობ, ვკვდები ვითომ? sad.gif

ჰო და კიდევ: როგორც კი რამეზე დავიწყებ ნერბვიულობას,სიცხე მიწევს sad.gif

გადავრჩები? biggrin.gif

Posted by: Botanic1 7 Aug 2009, 23:27
გამარჯობათ ე.ი ჯირკვალი მაქვს აი ამ ადგილას user posted image

მხოლოდ მარჯვენა მხარეს მაქ...მარჩხენაზე რომ გავსინჯე იყო ხტილის სახით მარა,მარჯვენაზე რომ დავიზმენდი ხელს რო ვუსვამდი გაღიზიანდა შეილება? დიდი არაა ძალიან,თკივილით არ მტკივა..

ძალიან საშისია? დაახლოებით 1 თვის მწინ მეძავთან მქონდა სექსი,2 კვირის შემდეგ ჩემს ექიმს მივაკითხე ვიტამინების დალევა მინდოდა და მომცა "Bio-Ritmo" მეორე Dრეს მარჯვენა ხელის იღლიასი Gამეჩითა ჯირკვალი რა,თავიდან დავაბრალე იმ ვიტამინს,მერე გამახსენდა 2 Dრით ადრე მივიღე მარჯვენა ხელის იდაყვზე ჭრილობა,რომელსაც ყურაDრება არ მივაქციე და რამოდენიმე დღის შემდეგ გაწითლდა და სითხე წამოვიდა და სუნიც ჰქონდა ცუდი რა...მერე დავიზმენდი რო მეძავმა ხომ არ გადამდო რამე,მარა დაცული ვიყავი ნაღდათ...
და რას Mირჩევთ რა ვქნა მივიდე ექიმთან და რომელ ექიმს მირჩევთ?
ისე ეს ვიტამინების ბრალი ხომ არააა ნახევრად? ის ბიო-რიტმო მოვრჩი და ეხლა დავიწყე დუო-ვიტის სმა....

Posted by: maknatuna 8 Aug 2009, 00:21
გამარჯობათ.
წნევის ცვალებადობის პრობლემა, ადრე არასდროს მქონია.
ვატარებდი ნორმალურ წნევას 110-70-ზე.
ეს ერთი თვეა, ყოველ მეორე დღეს წნევა მეცემა 90-70 ან 80-60 მდე და საკმაოდ ცუდათ ვგრძნობ თავს.
არ ვდიეტობ. არ ვშიმშილობ.
რა შეიძლება იყოს გამომწვევი მიზეზი წნევის დავარდნის?
ან რა ვუშველო?
შევწუხდი უკვე user.gif user.gif
გმადლობთ წინასწარ.

Posted by: ana_maniani 8 Aug 2009, 15:09
არ ვიცი ამ თემას ეკუთვნის თუ არა ჩემი შეკითხვა.თუ არა_არ გაბრაზდეთ smile.gif
რა ხდება_ 22 წლის ადამიანისთვის თუა ნორმალური ესეთი წნევა_102/62, პულსი_82...წნევას არ გავიზომავდი მთელი კვირა თავის ტკივილს რომ არ შევწუხებინე,მაგრამ აი მთლიანად თავი კი არ მტკივა,მარცხენა საფეთქელის არე,სამი გამაყუჩებელი თუ დავლიე მერე კი ჩერდება.
გადაღლის ბრალი შეიძლება იყოს თუ რამე სერიოზულის სიმპტომებია? user.gif

Posted by: Romina 8 Aug 2009, 18:07
maknatuna
ana_maniani

საერთოდ სიცხეში წნევა დაბალია, ჩვეულებრივთან შედარებით. ვფიქრობ ჯობია ბევრი სითხე მიიღოთ. თუ ძალიან დიდხანს გაგრძელდა და შემაწუხებელი გახდა მერე მიმართეთ ექიმს.

Posted by: ana_maniani 8 Aug 2009, 22:26
Romina
სადაც ვარ აქ არ ცხელა,ანუ ამაზე სიცხე ყოფილა და ტკივილების გარეშე მივლია.მგონი უძილობის ბრალია,დგეს ნორმალურად გამოვიძინე და თავის ტკივილების გარეშე ავდექი,სამაგიეროდ რაღაცაზე გავღიზიანდი თუ არა დამეწყო ისევ აუტანლად...idea.gif
გმადლობთ smile.gif

Posted by: RAULA 9 Aug 2009, 09:34
ვინმე პასუხობს აქ საერთოდ ?? biggrin.gif

თუ პასუხობს მაშინ იმუნიტეტის გასაძლიერებელი მირჩიეთ რამე ძლიერი და კარგი

პოლიჟენი ვიცი მარა ვიცნ სვავს ყველას გული ერევა sad.gif

Posted by: fsiqoterapevti 9 Aug 2009, 10:35
კარგი საშუალებაა ჰომეიპათიური წამლები, გამოცდილი და ეფექტურია................შენი გადასაწყვეტია რამდენად ენდობი..........აბსოლიტურად უსაფრთხოა

Posted by: paichadze 9 Aug 2009, 20:53
ტახიკარდია მაქვს დრო გამოშვებით და ტუ არის შესაძლებელი რომ რაიმე ცუდი შედეგები მოყვეს ან ვის მივაკითხო?

Posted by: RAULA 9 Aug 2009, 21:21
paichadze

ქვემოთ იკითხე კარდიოლოგების თემაში smile.gif იქ უფრო გირჩევენ ისე ექიმს უნდა მიაკითხო კარდიოლოგს smile.gif

Posted by: წრიპა 9 Aug 2009, 22:27
paichadze
ტაქიკარდია არა არის ნორმალური მდგომარეობა და ბუნერივია, რომ შესაძლებელია სხვადასხვა გართულებები გამოიწვიოს. ტაქიკარდია დამოუკიდებლად დაავადება არ არის, იგი რაღაცის შედეგია, გამომდინარე აქედან სათანადო ყურადღება უნდა მიაქციო, მიაკითხე კარდიოლოგს.
თუ შენი ტაქიკარდია დაკავშირებულია ფიზიკურ დატვირთვასთან და ამასთანავე ხარ ახალგაზრდა შეგიძლია წყნარად იყო....

Posted by: jashusha 10 Aug 2009, 15:45
QUOTE
ვინმე პასუხობს აქ საერთოდ ?? 

თუ პასუხობს მაშინ იმუნიტეტის გასაძლიერებელი მირჩიეთ რამე ძლიერი და კარგი

პოლიჟენი ვიცი მარა ვიცნ სვავს ყველას გული ერევა 


ექიმი არ ვარ, მაგრამ ექიმმა დამინიშნა ვიტამინი გოლდ რეი, ძაან მომწონს, ისე იმუნალიც არის და იმუნო რიცი.

Posted by: texasuri jleta benzoxerxit 15 Aug 2009, 13:18
QUOTE (maknatuna @ 8 Aug 2009, 00:21 )
გამარჯობათ.
წნევის ცვალებადობის პრობლემა, ადრე არასდროს მქონია.
ვატარებდი ნორმალურ წნევას 110-70-ზე.
ეს ერთი თვეა, ყოველ მეორე დღეს წნევა მეცემა 90-70 ან 80-60 მდე და საკმაოდ ცუდათ ვგრძნობ თავს.
არ ვდიეტობ. არ ვშიმშილობ.
რა შეიძლება იყოს გამომწვევი მიზეზი წნევის დავარდნის?
ან რა ვუშველო?
შევწუხდი უკვე user.gif user.gif
გმადლობთ წინასწარ.

ბევრი მიზეზი შეიძლება ჰქონდეს,დაწყებული მეტეომგრძნობელობით და ნეიროცირკულატორული დისტონიით (ეს სახელწოდება აღარ იხმარება ოღონდ)დამთავრებული ენდორკინული პათოლოგიებით.

ფარისებრი ჯირკვლის ჰორმონებია პირველ რიგში სანახავი.
რუტინულად ელექტროკარდიოგრამას თუ გადაიღებ და ექოკკარდიოსკოპიას,არ იქნება ზედმეტი,
ასევე სისხლის საერთო ანალიზი რა თქმა უნდა.

Posted by: maknatuna 15 Aug 2009, 23:26
texasuri jleta benzoxerxit
QUOTE
ფარისებრი ჯირკვლის ჰორმონებია პირველ რიგში სანახავი.

ანალიზმა (ერთი წლის წინ მაქვს გაკეთებული) და ექოსკოპიამ (ამ თვეში) აჩვენა რომ, არ მაქვს ფარისებრი ჯირკვლის პრობლემა.
QUOTE
ექოკკარდიოსკოპიას

ყველაფერი წესრიგში მაქვს, მხოლოდ დამატებითი ქორდა ამეღნიშნება.
QUOTE
რუტინულად ელექტროკარდიოგრამას

კარდიოგრამაზე პატარა დისტროფიული ცვლილებები მქონდა, მაგრამ ესეც გამოსწორდა. ერთ თვიანი მკურნალობის შედეგად.
QUOTE
ასევე სისხლის საერთო ანალიზი რა თქმა უნდა.

სისხლის საერთო ანალიზი კი არ გამიკეთებია. კოაგულოგრამა გავიკეთე და პროთრომბინი მქონდა მომატებული, ესეც ნორმაში მაქვს ეხლა.
წნევა აღარ დამვარდნია ასე კატასტროფულად ამ დღეებში, მაგრამ გულის არეში ვგრძნობ დისკომფორტს user.gif
გმადლობთ ყურადღებისთვის smile.gif

Posted by: Natuka NGN 17 Aug 2009, 10:56
maknatuna
QUOTE
სისხლის საერთო ანალიზი კი არ გამიკეთებია.

ერთ-ერთი ყევლაზე მთავარი ანალიზი არ გაგიკეთებია. გემოგლობინი რამდენი გაქვს? მსგავსი პრობლემა ჰქონდა ჩემს მეოგბარს. აქ რას არ უჩხიკინებდნენ... ჰოდა, გერმანიაშია ახლა და დაუდგინეს რკინადეფიციტური ანემია. ერთი სიტყვით. რაც რკინის პრეპარატები მიიღო, მას შემდეგ მსგავს პრობლემას აღარ შეუწუხებია. ერთი სიტყვით ნახე სისხლის საერთო ანალიზი და გემოგლობინი.

Posted by: EXHALATING 18 Aug 2009, 18:22
საერთო ანალიზში მომატებული როე და 37 -37.3 სიცხეები რას დავაბრალო, ვის ვეჩვენო cry.gif
პლუს მომატებული TSH ...
ჩემი ჯანმრთელობა ქარს მიაქვს მგონი... sad.gif

Posted by: faustina 19 Aug 2009, 15:05
ხელის თითების დაბუჯება რამ იცის და რომელ ექიმს მივმართO???

Posted by: skoch 19 Aug 2009, 16:57
ჩემს ახლობელს ვენა გაუხეტქეს როგორ იმკურნალოს რას მირჩევთ??

Posted by: BeDelicious 20 Aug 2009, 14:05
მირჩიეთ, რის/რომელ ექიმთან უნდა მივიდეს 27 წლის მამაკაცი, რომელსაც მუცლის მარჯვენა მხარეს ჩხვლეტითი შეგრძნებები აწუხებს?

ბარემ სახელი და გვარიც რომ დაწეროთ და კოორდინატები, თუ ვინმე კარგი სპეციალისტი იცით.

გმადლობთ.

Posted by: ketina 20 Aug 2009, 15:52
გამარჯობა,
83 წლის ქალს ტკივა ფეხები. რამდენიმე თვეა წევს და ძალიან იშვიათად დგება, დამოუკიდებლად ვერ დადის. თავიდან ჰქონდა სხვა პრობლემები, ოპერაცია გაიკეთა და იმის მერე სულ წევს, იყო ძალიან ენერგიული და აქტიური. შეიძლება თუ არა, რომ ტკივილი უმოძრაობით იყოს გამოწვეული და რას გვირჩევდით სამკურნალოდ, ასევე მაინტერესებს ხალხური მედიცინიდანაც, იქნებ სამკურნალო აბაზანებმა მოუხსნას ტკივილი. დიდი მადლობა

Posted by: BadbadGirl 24 Aug 2009, 16:22
ექიმებო, ღორის გრიპის საწინაამღგდეგო აცრა არსებობს? (პარასკვეს გვიპირებენ სტუდენტების ვაქცინაციას და მე არ ვიცი ავიცრა თუ არა. აცრა ნებაყოფლობითია)

Posted by: skoch 24 Aug 2009, 16:50
ეს თემა მიტოვებულია ბლიად................

Posted by: texasuri jleta benzoxerxit 26 Aug 2009, 13:00
BadbadGirl ლაპარაკია ალბათ ჩვეულებრივი გრიპის საწინააღმდეგო ვაქცინაზე (რასაც ნამდვილად გირჩევთ,). ღორის გრიპის ვაქცინა რამდენადაც ვიცი ჯერ არც არის შემუშავებული.

Posted by: BadbadGirl 26 Aug 2009, 15:23
texasuri jleta benzoxerxit
საქმეც ეგაა რომ ღორის გრიპის ვაქცინაზეა საუბარი. არ ვიცი რას ცრიან, რომ გამიკვირდა იმიტომ ვიკითხე აქ. (ჩვეულებრივი გრიპის საწინაამღდეგოდ აცრა გაკეთებული მაქვს უკვე ამ წელს)

Posted by: Natuka NGN 27 Aug 2009, 22:23
BadbadGirl
აქ ხარ თუ გერმანიაში? ღორის გრიპის ვაქცინაზეა საუბარი. სადღაც მაკიდმა დაწერა. დაათვალიერე თემები. მე რომ შნე ვიყო, არ ავიცრიდი... თავად აცრას ქონია საკამოდ უკუშედეგები.
texasuri jleta benzoxerxit

QUOTE
რამდენადაც ვიცი ჯერ არც არის შემუშავებული.

არი,ს არის მოასწრეს უკვე... ეს კი იმსი ეჭვის საფუძველს ამზლევს, რომ ეს ყველაფერი გადამაშებული დიდი ფინანსური აფიორაა. რომელიღაც კომპანიამ გამოიმუშავა ვაქცინა wink.gif

Posted by: 2PAC 28 Aug 2009, 15:39
дизентрин - ამ წამალზე ვერაფერი ინფორმაცია ვერ მოვიძიე :|

არადა ექიმმა გუშინ დამინიშნა, მოწამლული ხარო და ეგ და კიდევ რაღაც ვიტამინი და არ მახსოვს დღეში რამდენჯერ უნდა დავლიო

sad.gif


help.gif

Posted by: SnowFlakes 29 Aug 2009, 02:55
აბა,პირველად დახმარებას ვინ გამიწევს ))

მარცხენა ყური თითქმის უწყვეტად მექავება ((( უკვე ყურის ჩხირი ჩანთით დამაქვს იმდენად შემაწუხებელია. ვერაფრით გავიჩერე და თან რამის ჩაწვეთებაც არ მინდა,ექიმს მივაკითხო ამის გამო?

მადლობა!

Posted by: skoch 29 Aug 2009, 03:07
QUOTE
აბა,პირველად დახმარებას ვინ გამიწევს ))

მარცხენა ყური თითქმის უწყვეტად მექავება ((( უკვე ყურის ჩხირი ჩანთით დამაქვს იმდენად შემაწუხებელია. ვერაფრით გავიჩერე და თან რამის ჩაწვეთებაც არ მინდა,ექიმს მივაკითხო ამის გამო?

მადლობა!

მოიქავე შენც..... wink.gif

Posted by: SnowFlakes 29 Aug 2009, 11:09
skoch

როდემდე ? შეიძლება რამე დაზიანდეს !

Posted by: BadbadGirl 29 Aug 2009, 14:35
Natuka NGN
არც მანდ არც გერმანიაში biggrin.gif biggrin.gif
არ მინდა აცრა მეც, რადგან გართულების მეშIნია.

კიდე ტუპერკულიოზზე უნდათ აცრა. მახსოვს რომ ავიცერი მაგრამ დარწმუნებული ვარ. როგორ გაიგო ნეტა?

Posted by: Natuka NGN 29 Aug 2009, 14:48
BadbadGirl
მგონი არის რაღAც კანონმდცებლობა, რომლის მიხედვით ვერ დაგაავლდებულებს ევრავინ აიცრა. ადრე ეწერა რომელიღაც თემაში. და ტუბერკკულოზზე ტარდებოდა ვაქციანცია? არა მგონია. მხოლოდ მანთუს დ აპირკეს აკეთებდნენ მგონი. რა ვიცი, მე არა ვარ საერთოდ აცრილი და მდიონ ახლა... მოვიყარე 37 და ვარ ცოცხალი smile.gif

Posted by: BadbadGirl 30 Aug 2009, 16:54
Natuka NGN
სავალდებულო არაა, სასურველია. რადგან ბევრ პაციენტთან მექნება საქმე და რა იცი რა ხდება.
მაგრამ მაინც არ მხიბლავს ვაქცინაციის იდეა, მეშინია biggrin.gif გარდა ამისა არ ვიცი გინდაც იგივე ტუპერკულიოზზე აცრილი ვარ თუ არა. თუ აცრილი ვარ და კიდევ გავიკეთებ ხელმეორედ, რამე მომივა?

Posted by: vano_t 31 Aug 2009, 03:02
BadbadGirl
QUOTE
სავალდებულო არაა, სასურველია. რადგან ბევრ პაციენტთან მექნება საქმე და რა იცი რა ხდება.
მაგრამ მაინც არ მხიბლავს ვაქცინაციის იდეა, მეშინია biggrin.gif  გარდა ამისა არ ვიცი გინდაც იგივე ტუპერკულიოზზე აცრილი ვარ თუ არა. თუ აცრილი ვარ და კიდევ გავიკეთებ ხელმეორედ, რამე მომივა?

დარწმუნებით რომ გაიგო გქონდა თუ არა ვაქცინა გაკეთებული, უნდა ნახო შენი ანკეტის ჩანწერები იმ საავადმყოფოში, სადაც შესაძლოა გაკეთებულიყო BCG ვაქცინა. მეორე ვარიანტია, მანტუს ტესტი (ან უფრო მოდიფიცირებული ტესტი, რასაც PPD ქვია) გაიკეთო. თუ ტესტი დადებითია და გულმკერდის რენტგენი უარყოფითია, ამასთან არ გაქვს ქრონიკული ხველა, მაშინ დიდი შანსია, რომ გაკეთებული გქონდეს ეგ ვაქცინა. თუმცა, ტესტი დადებითი შეიძლება აღმოჩნდეს მაშინაც კი, როცა არც დაავადება გაქვს და არც აცრა გქონია.

ხელმეორედ აცრას იგივე რისკები აქვს, რაც თავდაპირველ აცრას. აცრის ადგილას შეიძლება ჩამოყალიბდეს სიწითლე, ანთება, შეიძლება ტემპერატურამ აიწიოს, შეიძლება ალერგიული რეაქცია მოგცეს. ბოლოს, მცირე შანსია ასევე ტუბერკულოზის განვითარების. ტუბერკულოზის აცრა კეთდება ცოცხალი ტუბერკულოზის ბაქტერიებით, რომელიც შესუსტებულია სპეციალური მეთოდებით. იდეაში, ხელმეორედ აცრისას კიდევ უფრო ნაკლები შანსი უნდა იყოს დაავადების შეძენის, ვიდრე თავდაპირველი აცრისას-აცრის არსი ზუსტად ეს არის, რომ მომავალში დაგიცვას ამათუიმ დაავადებისაგან, მითუმეტეს თუ დაავადების გამომწვევი აგენტი კიდევ უფრო შესუსტებულია, ვიდრე მისი ბუნებრივი ბიძაშვილი.

Posted by: BadbadGirl 31 Aug 2009, 10:56
vano_t
დიდი მადლობა. საქმეც ეგაა რომ ჩემ ანკეტას უბრალოდ ვერ ვნახავ, იმიტომ რომ არ ვიცი სადაა. მანტუს ტესტის გაკეთება კიდე შესაძლებელია.

ფიქრობ ჯობია ავიცრა ხო? მიუხედავად იმისა მაქვს თუ არა აცრა ჩატარებული smile.gif

ღორის გრიპზე რა ვქნა? ავიცრა? აი მაგი ევრ გადავწყვიტე. არ მინდა რაღაც ექპერიმენტის მონაცილე "ზღვის გოჭი" ვიყო. თან ყოველ დღე წერენ როგორ დაავადდა ვირაც და როგორ მოხვდა სავადმყოფოში. მოკლედ საკმაოდ ინტენსიური სტრესია ჩემთვის.

დიდი მადლობა კიდევ ერთხელ smile.gif

Posted by: texasuri jleta benzoxerxit 31 Aug 2009, 13:26
მე აუცილებლად ავიცრები ღორის გრიპზე.
ტუბერკულოზზე აცრა ახლა ყოველგვარ აზრს მოკლებულია

Posted by: BadbadGirl 31 Aug 2009, 13:33
texasuri jleta benzoxerxit
QUOTE
ტუბერკულოზზე აცრა ახლა ყოველგვარ აზრს მოკლებულია

რატომ?

Posted by: magdalina 31 Aug 2009, 15:40
ორი თვეა მაწუხებს ხერხემალი ხან კისერთან მტკივა და ხან გულ-მკერდის არეში ძირითადათ დღე განსაკუთრებით სიმძიმეს რომ ავწევ მაშინ(ბავშვი მყავს 1,10 თვის),მკერდზეც გადამდიოდა ტკივილი,მაგის გამო მამოლოგთანაც კი ვიყავი,ხოდა მან მითხრა რომ ხერხემლის ბრალიაო,კისერთან რომ მტკივა მაშინ თავზეც ამდის თითქოს,როცა დავწვები მაშინარანაირი ტკივილი აღარ მაქვს.ვისმევ ვოლტარენის მაზს და თითქოს მშველის.

იქნებ მირჩიოთ რა განხრის ექიმს მივაკითხო(ხერხემალს ვინ მკურნალობს)?
მადლობა წინასწარ.

Posted by: NetDevil 1 Sep 2009, 21:05
magdalina
სავარაუდოდ იქნება დისკის თიაქარი, ანუ მალთაშუა დისკი კარგად არ ზის და ნერვს აწვება... ესე დარწმუნებით ვერ იტყვი...

ნევროლოგს უნდა მიაკითხოთ

Posted by: Aniberia 2 Sep 2009, 13:34
Romina
QUOTE
ანტიბიოტიკების მასე ცვლა და ხანმოკლე კურსის ჩატარება არ შეიძლება. ნუ გეშინია, სრულიად განკურნებადი დაავადებაა. შენ შეკითხვაზე პასუხი არ მაქ, ნემსთან დაკავშირებით  იქნებ სხვამ გიპასუხოს.

საიდან მოიგონე რომ ლიმა იკურნება? ეგ არ იკურნება.
* * *
Kaifistka2
QUOTE
დოქსაცილინი მე მივიღე უკვე 20 დღე მანამდე, ახლა ვარ 10 დღიან ცეფტრიაქსონზე ანუ ეს ერთი თვე, მარამ მე მკურნალობას არ ვწყვეტ ვაგრძელებ იგივე ანტიბიოტიკებს სანამ არ დაიწევს დონე სისხლში და პლიუს ამას მერეც უნდა გაგრძელდეს მკურნალობა, ეგ ექიმი სწორად მკურნალობს, მე ინტერნეტში მოვიძიე ერთ-ერთი საკამოდ ცნობილი ექიმის გვარი მაგ ლაიმას დაავადების მკურნალობაში და აქვს თავისი საიტი სადაც განთავსებულია წამლების დასახელებები და დოზები დაავადების გართულებებისდა მიხედვით. ჩემი ექიმი სწორად მმკურნალობს. მაგ ექიმს ძალიან რთულად მივაგენი და მან დამისვა დიაგნოზი ლაიმაზე, ისეთი რთულად გამოსაცნობი დაავადება, რომ მე ვნახე სხვა ხალხის ისტორიები ინტერნეტში და უცხოელები არიან ძირითადად და უცხოელი ექიმები ვერ უდგენდნენ ზოგს 8 წელი ქონდა, ზოგს ორი და ვერ ხვდებოდნენ რა ჭირდათ, მადლობელი ვარ ამ კაცის რომ სწორად მიმიხვდა რაც მჭირდა და მანამდე მომატყუეს სეფსოლოგებმა, დამისვეს სეფსის დიაგნოზი, ვიწექი სეფსის ცენტრშიც, ასევე გულის კლინიკაში ეგონათ გულის პთოლოგია პერიკარდიტი, მაგრამ სითხეს რომ ვერ აშრობდნენ და სისხლში მომატებული იყო სტრეპტოლიზინის დონე მერე მიხვდნენ რომ ეს არ იყო გულის ბრალი. მანამდე კიდე ნერვოპათოლოგთან ვმკურნალობდი თავზე და ნერვოპათოლოგმა გამაგზვნა გულის შესამოწმებლად, იმიტომ რომ ძალიან არეული პულსით და დაბალი წნევით მივედი მასთან და პლიუს სიცხით.
ეს იმ ექიმის ვებ საიტია სადაც წერია მკურნალობა და წამლების ზუსტი სახელები და ცეფტრიაქსონს თვითონ ანოტაციაში უწერია რომ 14 დღეზე მეტი ერთ ჯერზე არ შეიძლება და უმაღლესი დოზა 2მგ. დღე-ღამეში, ლაიმას მკურნალობის დროს.
http://www.drerniemurakami.com/main/page_c..._treatment.html


მგონი ჩემს შვილს აქვს ლიმა(არტრიტიული),http://forum.ge/?showtopic=34007113&st=0&#entry15320993
გთხოვ მაგ შენი ექიმის მონაცამები მომცე

Posted by: Solveig 2 Sep 2009, 19:45
BadbadGirl
მარცხენა მკლავზე დაიხედე (მხრის ცოტა ქვემოთ)-გეტყობა ნაიარევი? თუ კი, ე. ი. ბცჟ გაქვს გაკეთებული.

vano_t
QUOTE
დარწმუნებით რომ გაიგო გქონდა თუ არა ვაქცინა გაკეთებული, უნდა ნახო შენი ანკეტის ჩანწერები იმ საავადმყოფოში, სადაც შესაძლოა გაკეთებულიყო BCG ვაქცინა.

ოი, რა ფანტაზიის სფეროს მიეკუთვნება მაგის გაგება საქართველოში.. sad.gif

ეგ კი არა, 2005-.ში ტეტანუსის ვაქცინა გამიკეთეს და ჟურნალში საერთოდ არ დამარეგისტრირეს თურმე...რამე რომ მომსვლოდა და პრეტენზია წაეყენებინათ ჩემებს, იტყოდნენ-ვინ ხართ, ვის დაკარგვიხართ, ჩვენ რა შუაში ვართო.

Posted by: BadbadGirl 2 Sep 2009, 22:22
Solveig
კიიი მაგი მაქვს მრგვალი შრამიააა smile.gif
ესეიგი აღარ გავიკეთებ smile.gif

Posted by: lazy 4 Sep 2009, 13:23
ხალის გაღიხიანება ან გაჭრა მართლა იწვევს კიბოს?

Posted by: BadbadGirl 4 Sep 2009, 17:29
მოგაბეზრეთ თავი მაგრამ კიდევ უნდა ვიკითხო რადგან ჩემი ანკეტა სადაა არ ვიცი.
წესით პოლიომელითის და მსხგავსი აცრები ხო უნდა მქოდეს გაკეთებული? საიდან გავიგო?

Posted by: shtori 4 Sep 2009, 17:36
მმასტიკის სუნით მოვიწამლე, თუ რაც ჰქვია. გეგონება არც ვარსობობო კაიფი კია
რა დავლიო რპმ გამიაროს? ადრე მქონდა და ექიმმა არ ვიცი რა დამალევინა

Posted by: psyxo 7 Sep 2009, 01:00
Natuka NGN
[quote]QUOTE
რამდენადაც ვიცი ჯერ არც არის შემუშავებული.


არი,ს არის მოასწრეს უკვე... ეს კი იმსი ეჭვის საფუძველს ამზლევს, რომ ეს ყველაფერი გადამაშებული დიდი ფინანსური აფიორაა. რომელიღაც კომპანიამ გამოიმუშავა ვაქცინა [/quote]
მეც ეგრე მეფიქრება, სულ შეთქმულების თეორიის იდეა მაწუხებს!...მართლა boli.gif
SnowFlakes
[quote]skoch
[quote]აბა,პირველად დახმარებას ვინ გამიწევს ))

[quote]skoch

როდემდე ? შეიძლება რამე დაზიანდეს ! [/quote]
დიდი დიდი ყურწულობა gigi.gif დაკარგო biggrin.gif
wink.gif

Posted by: merry 7 Sep 2009, 14:55
ხალხო კარგ ონკოლოგს ვერ მირჩევთ? ჩემს ძალიანძალიან ახლობელს ესაჭიროება აზაპ cry.gif

Posted by: lazy 8 Sep 2009, 21:32
ვკვდები რამე მიშველეთ და გამეცით პასუხი

Posted by: freeandhappy 8 Sep 2009, 23:49
აღმენიშნება ტკივილი ნეკნის არეში, რომელიც 4დღეა უფრო მიძლიერდება. ტრავმა მივიღე რაც არ უნდა სასაცილო იყოს ჩახუტებისგან... მაინტერესებს გაბზარული ან რამე ორგანო ხომ არ დამიზიანდებოდა? ყრუ ტკივილები მაქვს

Posted by: zura198630 10 Sep 2009, 00:53
გამარჯობათ გავიკეტე ძაღლის ნაკბენზე სამი შრატის ნემსი ბოლო ნემსი გავიკეთე 2 აგვისტოს, ძაღლს ცოფი არ აღმოაჩნდა და ჯანმრთელია. ექიმებმა მითხრეს 6 თვე არ დალიოო ალკოჰოლური სასმელიო. საერთოდ არ ვეტანები დასალევს მაგრამ მაინტერესებს შეიძლება თუ არა, ან საშიშროება არსებობს დალევის შემთხვევაში 6 თვეზე ადრე. მადლობთ წინასწარ

Posted by: Natuka NGN 11 Sep 2009, 00:21
შეკითხვა ექიმებს:
რამდენად ძლიერი დამაყუჩებელია ქსეფოკამი, რა კატეგორიის ტკივილს აყუჩებს და აქვს თუ არა უკუჩვენებები ქიმიოთერაპიის დროს. თუ გამოგიყენებიათ პრაქტიკაში და რა ჩივილები ქონია თქვენს პაციენტებს.

Posted by: blblbl 11 Sep 2009, 00:23
zura198630
არ შეიძლება,მცირე ალერგიიდან დაწყებული სიკვდილით დამთAვრებული არსებობს საშიშროება.ასე რომ ნუ დალევ 6 თვე.

Posted by: Tamu-bebichka 11 Sep 2009, 15:31
"კითხვა კარდეოლოგთან"–ში ვიკითხე უკვე და იმედია მალე მივიღებ პასუხს. აქაც დავდებ ჩემს პოსტს, რადგან ეჭვი მაქვს, რომ კარდეოლოგი კი არა, ნევროლოგი მჭიდრება smile.gif))

თითქმის სამი თვეა გულზე სუნთქვა არ მყოფნის და ღრმად ჩასუნთქვა მჭირდება დროდადრო. ეს ბოლო ხანები აშკარად შემიმცირდა ეს პრობლემა.

თავიდან სიცხეს და ტენიანობას ვაბრალებდი. მერე ვიფიქრე ალერგია გამირთულდა მეთქი (ივლისის სიცხეში ყოველთვის ბევრს ვაცემინებ და თვალები მიცრემლიანდება). კიდევ, არ ვიცი მეჩვენება თუ მართლა ეგრეა, მაგრამ ვარჯიშით თუ ცოტათი დავიტვირთე მაშინაც მიმატებს ღრმად ჩასუნთქვის მოთხოვნილება.

კარდეოლოგს გავესინჯო თუ კატაბალახა დავლიო? wink.gif
მირჩიეთ რა რამე smile.gif

Posted by: aladini 14 Sep 2009, 09:49
უკაცრავად
არ ვიცი სწორედ ვსვამ თუა რა კითხვას
გუშIნ გამფხაჭნა მეგობრის ძაღლმა
ნასვამი ვეთამაშებოდი და შუბლზე მოუხდა კბილი
ნუ აცრილია ცხადია
მაშინვე საპნით ჩამოვიბანე და მერე სპირტიც გადავისვი
მარა რავი,ე საო ისაო მაინც სახეაო
ნუ ძაღლს კი მაინც დავაკვირდები, ამრა ამის გულისთვის ნემსების დარჭობა ღირს?

Posted by: Frankens†ein 14 Sep 2009, 18:47
აქ უნდა დამეწერა თუ არა არ ვიცი... შოკში ვარ...

ტუჩებზე, მკერდზე, თითებს შორის და სასირცხვო ადგლებში 2ო თეთრიანის ხელა წყლულები გამიჩნდა და რა ხდება?

Posted by: the_lizard_king 15 Sep 2009, 11:53
aladini
QUOTE
ამის გულისთვის ნემსების დარჭობა ღირს?

yes.gif

თავის არე ყველაზე საშიშია.

Posted by: Zuraba 17 Sep 2009, 01:29
როგორ ხდება ექიმების რეგისტრაცია და სად შეიძლება ამის შემოწმება, მაგალითად მინდა გავიგო თუ ჩემი ექიმი ფლობს სათანადო კვალიფიკაციას და რეგისტრერებულია იმუშაოს ექიმად საქართველოს ხელისუფლების მიერ?

აგრეთვე სად შეიძლება ინტერნეტისტების სიის მოძებნა, მაგალითად თუ ვინმემ იმიშავა ექიმ რეზიდენტად საქართველოში ვის შეუძლია ამის დადასტურენა, არის თუ არა ასეთი საბუთები ცენტრალიზებული?

დიდი მადლობა

Posted by: თეოგოგო 17 Sep 2009, 09:39
Zuraba
ინფექციურის წინ არის ლიცენზირების დეპარტამენტი, მაგრამ ეს ინფორმაცია საჯაროა თუ არა - არ ვიცი.

Posted by: Zuraba 18 Sep 2009, 20:58
QUOTE (თეოგოგო @ 17 Sep 2009, 08:39 )
Zuraba
ინფექციურის წინ არის ლიცენზირების დეპარტამენტი, მაგრამ ეს ინფორმაცია საჯაროა თუ არა - არ ვიცი.

მეც ეგრე ვიცი, როცა ამთავრებ სამედიცინო სასწავლებელს ექიმის დიპლომით (საჯარო, ზოგადი მედიცინა) აუცილებელია რომ გაიარო ლიცენზირება საექიმო საქმიანობისათვის რომ დასაქმდე, ასე რომ 6 წლიანი სწავლა და სალინცენზიო გამოცდები არის აუცილებელი პირობა რომ ექიმად იმუშავო დამოუკიდებლად, ვინ დამიდასტურებს?

პს. ყველა ლიცენზირებული ექიმი არის რეგისტრირებული სამედიცინო საქმიანობის რეგულირების სააგენტოს მიერ, ასევეა რეგისტრირებული ინტერნისტების საქმიანობა...

Posted by: Mr Adam 19 Sep 2009, 11:37
ოსტეოქონდროზი და ცალმხრივი თავის ტკივილი..

უკვე კარგა ხანია ბებიაჩემს აწუხებს ეს ორი პრობლემა , სად აღარ იყო , მაგრამ ვერავინ უშველა..
იქნებ თქვენ მირჩიოთ ვისთან მივიყვანო ? რომელ კლინიკაში ?..

Posted by: SPC 21 Sep 2009, 23:19
ხომ ვერ მეტყვით რისი ბრალი შეიძლება იყოს გახშირებული სუნთქვა? 2 თვეა სადღაც მივხვდი რომ ჩქარა ვსუნთქავ....და დღის განმავლობაში რამოდენიმეჯერ აი ღრმა ჩასუნთქვა რომ არის, შენგან დამოუკიდებლად.
თუ იცით იცით
თუ არადა არაუშავს ვისუნთქებ ჩქარ ტემპში smile.gif

Posted by: RAULA 22 Sep 2009, 10:49
1 შეკითხვა დაბალი სიცხეები ნევროზისგან შეიძლება იყოს გამოწვეული ?


Posted by: Mr Adam 22 Sep 2009, 17:09
მგონი აქ ყველას ფეხებზე კიდია .. ონლაინ დახმარებას არავინ იძლევა

Posted by: BeDelicious 24 Sep 2009, 16:14
ხელის მტევანი მტკივა უკვე თითქმის ერთი თვეა, და რის ექიმთან უნდა მივიდე? ნერვოპათოლოგთან? ტრავმატოლოგთან?
არ მიღრძვია, არც მივარტყი.


Posted by: BadbadGirl 24 Sep 2009, 23:56
ესეიგი ასეთი კითხვა მაქვს :
რომელ ექიმს მივმართო თუ: უკვე მერამდენე თვეა არ ვიცი ყოვედღიურად მითამაშებს მარჯვენა თვალის ქუთუთოები (ხანდახან მარცხენაც)

დიდი მადლობა.

Posted by: anonymouska 25 Sep 2009, 01:36
BadbadGirl ეგ ნერვის ბრალია.
* * *
RAULA
დაბალი სიცხეები ფილტვებმა იცის. გაესინჯე ყოველიშემთხვევისთვის.
* * *
lazy
ონკო-დერმატოლოგს აჩვენე. საერთოდ ხალი უმეტესად კეთილთვისებიანი წარმონაქმნია და არაა კარგი მისი დაზიანება.
* * *
BeDelicious
ისე ღუდუშაურში არიან ხელის მტევნის სპეციალისტები.

Posted by: Natalie:) 27 Sep 2009, 12:35
რა მინდა გკითხოთ biggrin.gif

სიცხე როგორ ავიწიო? givi.gif
ახლა საერთოდ არ მაქვს და შესაძლებელია, რომ საღამოს სულ მცირე 37 მაინც მქონდეს? user.gif


წინასწარ მადლობა დახმარებისთვის smile.gif

Posted by: anonymouska 27 Sep 2009, 14:20
Natalie:)

lol ))

შენც ადექი და შეათბე თერმომეტრი biggrin.gif

Posted by: ANGELofDEATH 28 Sep 2009, 04:50
გამარჯობათ... sad.gif იქნებ ვინმემ მიშველოთ ... ვარ 21 წლის.(გოგოbiggrin.gif ) დაახლოებით ორი თვეა დამეწყო საშინელი თმის ცვენა... ბღუჯა ბღუჯა ჩამომდის თმა ერთი ხელის მოსმითაც... ძალიან შეთხელებულია, კანი მიჩანს sad.gif ექიმთან ვიყავი არანაირი ანალიზები არ გაუკეთებია ჩემთვის თმაზე ხელი მომკიდა და გამოიტანა დასკვნა რომ ნერვოზისგან არის ეს ყველაფერი... დამინიშნა ნერვოზის წამლები, რაღაც ვიტამინები და + თმის ზეთი... ერთი თვე გავიდა დაახლოებით და არანაირი შედეგი... შეიძლება ჰორმონალურ წამლებს გამოეწვია? ისე დაემთხვა რომ სანამ დამეწყებოდა თმის ცვენა ვმკურნალობდი პატარა ჰორმონალურ დარღვევას დამინიშნეს იოდბალანსი და ბრომოკრიპტინი... ჩიყვზეც მაქვს ეჭვი(მითხრეს რომ თმის ცვენას ჩიყვიც იწვევსო მართალია? ... მოკლედ არ ვიცი რა ვიფიქრო sad.gif ვინმემ რამე მირჩიეთ უკვე პანიკაში ვარ ჩავარდნილი... ესე თუ გაგრძელდა მალე თავზე თმა აღარ მექნება... sad.gif sad.gif ასევე ცუდი სიმპტომები მაქვს... წამოხურებები კლიმაქსიან ქალებს რომ ემართებათ... უცებ ცივი ოფლი დამასხავს და ვერ ვსუნთქავ ხოლმე... sad.gif და ბარემ აქ ვარ კიდევ ერთი კითხვა მაქვს: მარცხენა იღლია მიღიზიანდება ძალიან ხშირად პატარა სიმაგრე ჩნდება ხოლმე რამოდენიმე დღე მიგრძელდება და ხანდახან ისე მტკივა ხოლმე ხელს ვერ ვამოძრავებ... sad.gif მაგაზე ვისთან მივიდე? sad.gif

რამდენი დამიწერია biggrin.gif


Posted by: axx 28 Sep 2009, 13:37
ANGELofDEATH
QUOTE
დაახლოებით ორი თვეა დამეწყო საშინელი თმის ცვენა...

QUOTE
გამოიტანა დასკვნა რომ ნერვოზისგან არის ეს ყველაფერი

QUOTE
შეიძლება ჰორმონალურ წამლებს გამოეწვია?

QUOTE
ჩიყვზეც მაქვს ეჭვი

QUOTE
ასევე ცუდი სიმპტომები მაქვს... წამოხურებები კლიმაქსიან ქალებს რომ ემართებათ

QUOTE
უცებ ცივი ოფლი დამასხავს და ვერ ვსუნთქავ ხოლმე...

QUOTE
მარცხენა იღლია მიღიზიანდება ძალიან ხშირად პატარა სიმაგრე

QUOTE
რამოდენიმე დღე მიგრძელდება და ხანდახან ისე მტკივა ხოლმე ხელს ვერ ვამოძრავებ... 

შენ ნახვას მოვასწრებ სანამ ცოცხალი ხარ? ეს რეები დაწერე ცუდად ვარ mad.gif , თმის ცვენა კიდევ ხო, გადაიპარსე რა ვაბშე თმები , მაგრად მოგიხდება შენ yes.gif აი დანარჩენებზე არ ვიცი , ნუ ყოველშემთხვევაში უნდა მიხედო სასწრაფოდ, 2kiss.gif

Posted by: texasuri jleta benzoxerxit 28 Sep 2009, 15:24
ANGELofDEATH
ენდოკრინოლოგს ეჩვენე აუცილებლად და გამორიცხეთ ყველა ჰორმონალური მიზეზი. თუ თმის ცვენის ნევროზული გენეზი დადგინდა მიმართე ფსიქიატრს და არა ნევროლოგს.

Posted by: button nose 29 Sep 2009, 00:35
კითხვა მაქვს ერთი და მგონი მეჩქარება.ეხლა უცებ დამეცკო ტკივილი მუცლის არეში მარჯვენა მხარე დაახლოებით საკვარცხეებთან,ნუ მთლად საკვერცხესთა არა,წელთან დაახლოებით,მტკივა მარჯვენა მხარეს წელიც და მთელ მარჯვენა ფეხში მგონია რომ ძალა არ მაქვს და მქაჩავს.დავწექი ზურგზე და ხელით მაგა დავაჭირე მტკივან ადგილ და უცბად რომავიუშვი ამეწვა,როგორ უნდა მოვიქცე და რისი ბრალია

Posted by: texasuri jleta benzoxerxit 29 Sep 2009, 01:35
QUOTE (button nose @ 29 Sep 2009, 00:35 )
კითხვა მაქვს ერთი და მგონი მეჩქარება.ეხლა უცებ დამეცკო ტკივილი მუცლის არეში მარჯვენა მხარე დაახლოებით საკვარცხეებთან,ნუ მთლად საკვერცხესთა არა,წელთან დაახლოებით,მტკივა მარჯვენა მხარეს წელიც და მთელ მარჯვენა ფეხში მგონია რომ ძალა არ მაქვს და მქაჩავს.დავწექი ზურგზე და ხელით მაგა დავაჭირე მტკივან ადგილ და უცბად რომავიუშვი ამეწვა,როგორ უნდა მოვიქცე და რისი ბრალია

სასწრაფოდ უნდა გაესინჯო ქირურგს მწვავე აპენდიციტის ან მწვავე მუცლის სხვა გამომწვევი მიზეზების გამოსარიცხად. და საერთოდ უნდა დადგინდეს მწვავე მუცელია თუ არა. ამ შემთხვევაში ტკივილგამაყუჩებლებმა შეიძლება წაშალონ სურათი

Posted by: Harmpy_Harmpy 29 Sep 2009, 02:28
გამარჯობათ
დაახლოებით თვენახევრისწინ ცალ მხარეს ძუძუსთავი ვაწვალე და ამის შედეგად ერთი წვეთი გამონადენი მქონდა, მოთეთრო ფერის.
ამის შემდეგ ხელის მოჭერის მიუხედავად გამონადენი აღარ მქონია.
დღეს ისევ ისე მოხდა რომ ისევ იმ ადგილზე დავიჭირე ხელი მაგრად და ისევ ასეთი გამონადენი მქონდა ოღონდ უფრო ნაკლები.
და საერთოდ, შეიძლება თუ არა ამას გამონადენი დაექვას? ანუ სპონტანურად არ ჩნდება.
რისი ბრალი შეიძლება იყოს ეს? მკერდის კიბოს დროს როგორც ვიცი სისხლნარევი ან უფერული გამონადენი იცის.
გამკვრივება არ მაქვს მკერდში არანაირი.
პ.ს მე 21 წლის ვარ.

Posted by: texasuri jleta benzoxerxit 29 Sep 2009, 02:39
QUOTE (Harmpy_Harmpy @ 29 Sep 2009, 02:28 )
გამარჯობათ
დაახლოებით თვენახევრისწინ ცალ მხარეს ძუძუსთავი ვაწვალე და ამის შედეგად ერთი წვეთი გამონადენი მქონდა, მოთეთრო ფერის.
ამის შემდეგ ხელის მოჭერის მიუხედავად გამონადენი აღარ მქონია.
დღეს ისევ ისე მოხდა რომ ისევ იმ ადგილზე დავიჭირე ხელი მაგრად და ისევ ასეთი გამონადენი მქონდა ოღონდ უფრო ნაკლები.
და საერთოდ, შეიძლება თუ არა ამას გამონადენი დაექვას? ანუ სპონტანურად არ ჩნდება.
რისი ბრალი შეიძლება იყოს ეს? მკერდის კიბოს დროს როგორც ვიცი სისხლნარევი ან უფერული გამონადენი იცის.
გამკვრივება არ მაქვს მკერდში არანაირი.
პ.ს მე 21 წლის ვარ.

გინეკოლოგ-ენდოკრინოლოგს უნდა მიმართოთ. პროლაქტინის დონეა სანახავი
რა თქმა უნდა მამოლოგთანაც უნდა მიხვიდეთ

Posted by: Harmpy_Harmpy 29 Sep 2009, 02:42
QUOTE (texasuri jleta benzoxerxit @ 29 Sep 2009, 02:39 )
QUOTE (Harmpy_Harmpy @ 29 Sep 2009, 02:28 )
გამარჯობათ
დაახლოებით თვენახევრისწინ ცალ მხარეს ძუძუსთავი ვაწვალე და ამის შედეგად ერთი წვეთი გამონადენი მქონდა, მოთეთრო ფერის.
ამის შემდეგ ხელის მოჭერის მიუხედავად გამონადენი აღარ მქონია.
დღეს ისევ ისე მოხდა რომ ისევ იმ ადგილზე დავიჭირე ხელი მაგრად და ისევ ასეთი გამონადენი მქონდა ოღონდ უფრო ნაკლები.
და საერთოდ, შეიძლება თუ არა ამას გამონადენი დაექვას? ანუ სპონტანურად არ ჩნდება.
რისი ბრალი შეიძლება იყოს ეს? მკერდის კიბოს დროს როგორც ვიცი სისხლნარევი ან უფერული გამონადენი იცის.
გამკვრივება არ მაქვს მკერდში არანაირი.
პ.ს მე 21 წლის ვარ.

გინეკოლოგ-ენდოკრინოლოგს უნდა მიმართოთ. პროლაქტინის დონეა სანახავი
რა თქმა უნდა მამოლოგთანაც უნდა მიხვიდეთ

დიდი მადლობა რჩევისთვის, მაგრამ მამოლოგთან რა მინდა? biggrin.gif

Posted by: texasuri jleta benzoxerxit 29 Sep 2009, 02:47
QUOTE (Harmpy_Harmpy @ 29 Sep 2009, 02:42 )
QUOTE (texasuri jleta benzoxerxit @ 29 Sep 2009, 02:39 )
QUOTE (Harmpy_Harmpy @ 29 Sep 2009, 02:28 )
გამარჯობათ
დაახლოებით თვენახევრისწინ ცალ მხარეს ძუძუსთავი ვაწვალე და ამის შედეგად ერთი წვეთი გამონადენი მქონდა, მოთეთრო ფერის.
ამის შემდეგ ხელის მოჭერის მიუხედავად გამონადენი აღარ მქონია.
დღეს ისევ ისე მოხდა რომ ისევ იმ ადგილზე დავიჭირე ხელი მაგრად და ისევ ასეთი გამონადენი მქონდა ოღონდ უფრო ნაკლები.
და საერთოდ, შეიძლება თუ არა ამას გამონადენი დაექვას? ანუ სპონტანურად არ ჩნდება.
რისი ბრალი შეიძლება იყოს ეს? მკერდის კიბოს დროს როგორც ვიცი სისხლნარევი ან უფერული გამონადენი იცის.
გამკვრივება არ მაქვს მკერდში არანაირი.
პ.ს მე 21 წლის ვარ.

გინეკოლოგ-ენდოკრინოლოგს უნდა მიმართოთ. პროლაქტინის დონეა სანახავი
რა თქმა უნდა მამოლოგთანაც უნდა მიხვიდეთ

დიდი მადლობა რჩევისთვის, მაგრამ მამოლოგთან რა მინდა? biggrin.gif

რა ვიცი,ძუძუ წესით მამოლოგის კომპეტენცია იყო აქამდე

Posted by: Harmpy_Harmpy 29 Sep 2009, 02:49
QUOTE (texasuri jleta benzoxerxit @ 29 Sep 2009, 02:47 )
QUOTE (Harmpy_Harmpy @ 29 Sep 2009, 02:42 )
QUOTE (texasuri jleta benzoxerxit @ 29 Sep 2009, 02:39 )
QUOTE (Harmpy_Harmpy @ 29 Sep 2009, 02:28 )
გამარჯობათ
დაახლოებით თვენახევრისწინ ცალ მხარეს ძუძუსთავი ვაწვალე და ამის შედეგად ერთი წვეთი გამონადენი მქონდა, მოთეთრო ფერის.
ამის შემდეგ ხელის მოჭერის მიუხედავად გამონადენი აღარ მქონია.
დღეს ისევ ისე მოხდა რომ ისევ იმ ადგილზე დავიჭირე ხელი მაგრად და ისევ ასეთი გამონადენი მქონდა ოღონდ უფრო ნაკლები.
და საერთოდ, შეიძლება თუ არა ამას გამონადენი დაექვას? ანუ სპონტანურად არ ჩნდება.
რისი ბრალი შეიძლება იყოს ეს? მკერდის კიბოს დროს როგორც ვიცი სისხლნარევი ან უფერული გამონადენი იცის.
გამკვრივება არ მაქვს მკერდში არანაირი.
პ.ს მე 21 წლის ვარ.

გინეკოლოგ-ენდოკრინოლოგს უნდა მიმართოთ. პროლაქტინის დონეა სანახავი
რა თქმა უნდა მამოლოგთანაც უნდა მიხვიდეთ

დიდი მადლობა რჩევისთვის, მაგრამ მამოლოგთან რა მინდა? biggrin.gif

რა ვიცი,ძუძუ წესით მამოლოგის კომპეტენცია იყო აქამდე

ხო მარა ეს ამისტვის არ დამიწერია რომ თქვენ გეთქვათ მამოლოგტან წადიო biggrin.gif

Posted by: texasuri jleta benzoxerxit 29 Sep 2009, 02:52
QUOTE (Harmpy_Harmpy @ 29 Sep 2009, 02:49 )
QUOTE (texasuri jleta benzoxerxit @ 29 Sep 2009, 02:47 )
QUOTE (Harmpy_Harmpy @ 29 Sep 2009, 02:42 )
QUOTE (texasuri jleta benzoxerxit @ 29 Sep 2009, 02:39 )
QUOTE (Harmpy_Harmpy @ 29 Sep 2009, 02:28 )
გამარჯობათ
დაახლოებით თვენახევრისწინ ცალ მხარეს ძუძუსთავი ვაწვალე და ამის შედეგად ერთი წვეთი გამონადენი მქონდა, მოთეთრო ფერის.
ამის შემდეგ ხელის მოჭერის მიუხედავად გამონადენი აღარ მქონია.
დღეს ისევ ისე მოხდა რომ ისევ იმ ადგილზე დავიჭირე ხელი მაგრად და ისევ ასეთი გამონადენი მქონდა ოღონდ უფრო ნაკლები.
და საერთოდ, შეიძლება თუ არა ამას გამონადენი დაექვას? ანუ სპონტანურად არ ჩნდება.
რისი ბრალი შეიძლება იყოს ეს? მკერდის კიბოს დროს როგორც ვიცი სისხლნარევი ან უფერული გამონადენი იცის.
გამკვრივება არ მაქვს მკერდში არანაირი.
პ.ს მე 21 წლის ვარ.

გინეკოლოგ-ენდოკრინოლოგს უნდა მიმართოთ. პროლაქტინის დონეა სანახავი
რა თქმა უნდა მამოლოგთანაც უნდა მიხვიდეთ

დიდი მადლობა რჩევისთვის, მაგრამ მამოლოგთან რა მინდა? biggrin.gif

რა ვიცი,ძუძუ წესით მამოლოგის კომპეტენცია იყო აქამდე

ხო მარა ეს ამისტვის არ დამიწერია რომ თქვენ გეთქვათ მამოლოგტან წადიო biggrin.gif

რა პასუხს ელოდით საინტერესოა gigi.gif

Posted by: Harmpy_Harmpy 29 Sep 2009, 02:59
QUOTE (texasuri jleta benzoxerxit @ 29 Sep 2009, 02:52 )
QUOTE (Harmpy_Harmpy @ 29 Sep 2009, 02:49 )
QUOTE (texasuri jleta benzoxerxit @ 29 Sep 2009, 02:47 )
QUOTE (Harmpy_Harmpy @ 29 Sep 2009, 02:42 )
QUOTE (texasuri jleta benzoxerxit @ 29 Sep 2009, 02:39 )
QUOTE (Harmpy_Harmpy @ 29 Sep 2009, 02:28 )
გამარჯობათ
დაახლოებით თვენახევრისწინ ცალ მხარეს ძუძუსთავი ვაწვალე და ამის შედეგად ერთი წვეთი გამონადენი მქონდა, მოთეთრო ფერის.
ამის შემდეგ ხელის მოჭერის მიუხედავად გამონადენი აღარ მქონია.
დღეს ისევ ისე მოხდა რომ ისევ იმ ადგილზე დავიჭირე ხელი მაგრად და ისევ ასეთი გამონადენი მქონდა ოღონდ უფრო ნაკლები.
და საერთოდ, შეიძლება თუ არა ამას გამონადენი დაექვას? ანუ სპონტანურად არ ჩნდება.
რისი ბრალი შეიძლება იყოს ეს? მკერდის კიბოს დროს როგორც ვიცი სისხლნარევი ან უფერული გამონადენი იცის.
გამკვრივება არ მაქვს მკერდში არანაირი.
პ.ს მე 21 წლის ვარ.

გინეკოლოგ-ენდოკრინოლოგს უნდა მიმართოთ. პროლაქტინის დონეა სანახავი
რა თქმა უნდა მამოლოგთანაც უნდა მიხვიდეთ

დიდი მადლობა რჩევისთვის, მაგრამ მამოლოგთან რა მინდა? biggrin.gif

რა ვიცი,ძუძუ წესით მამოლოგის კომპეტენცია იყო აქამდე

ხო მარა ეს ამისტვის არ დამიწერია რომ თქვენ გეთქვათ მამოლოგტან წადიო biggrin.gif

რა პასუხს ელოდით საინტერესოა gigi.gif

ნუ ალბათ იმას, რისი ბრალი შეიძლება იყოს biggrin.gif
თორემ ექიმთან ვიზიტები საჭიროაო ეს ისედაც ვიცი biggrin.gif

Posted by: texasuri jleta benzoxerxit 29 Sep 2009, 03:08
მიზეზი უპირველეს ყოვლისა შეიძლება იყოს პროლაქტინის სიჭარბე,რაც იწვევს გალაქტორეას, "რძის დენას". ჰიპერპროლაქტინემია შეიძლება ჰიპოფიზის პათოლოგიით იყოს განპირობებული.
შეიძლება მიზეზი ადგილობრივია და შეიძლება გამონადენს არაფერი კავშირი არ ქონდეს რძესთან.
დავწვრილმანდე რა პათოლოგიები შეიძლება იდგეს ამ ყველა პროცესის უკან? biggrin.gif fig.gif boli.gif pop.gif givi.gif

Posted by: Harmpy_Harmpy 29 Sep 2009, 03:16
QUOTE (texasuri jleta benzoxerxit @ 29 Sep 2009, 03:08 )
მიზეზი უპირველეს ყოვლისა შეიძლება იყოს პროლაქტინის სიჭარბე,რაც იწვევს გალაქტორეას, "რძის დენას". ჰიპერპროლაქტინემია შეიძლება ჰიპოფიზის პათოლოგიით იყოს განპირობებული.
შეიძლება მიზეზი ადგილობრივია და შეიძლება გამონადენს არაფერი კავშირი არ ქონდეს რძესთან.
დავწვრილმანდე რა პათოლოგიები შეიძლება იდგეს ამ ყველა პროცესის უკან? biggrin.gif fig.gif boli.gif pop.gif givi.gif

კი დაწვრილმანდი biggrin.gif
მაგრამ რაგაც პროფესიონალობას ვერ გატყობ და შენი აზრი დიდად არ მაინტერესებს biggrin.gif
როცა არაფერი იცი უნდა გაჩუმდე ზოგჯერ

Posted by: texasuri jleta benzoxerxit 29 Sep 2009, 03:20
QUOTE (Harmpy_Harmpy @ 29 Sep 2009, 03:16 )
QUOTE (texasuri jleta benzoxerxit @ 29 Sep 2009, 03:08 )
მიზეზი უპირველეს ყოვლისა შეიძლება იყოს პროლაქტინის სიჭარბე,რაც იწვევს გალაქტორეას, "რძის დენას". ჰიპერპროლაქტინემია შეიძლება ჰიპოფიზის პათოლოგიით იყოს განპირობებული.
შეიძლება მიზეზი ადგილობრივია და  შეიძლება გამონადენს არაფერი კავშირი არ ქონდეს რძესთან.
დავწვრილმანდე რა პათოლოგიები შეიძლება იდგეს ამ ყველა პროცესის უკან?  biggrin.gif  fig.gif  boli.gif  pop.gif  givi.gif

კი დაწვრილმანდი biggrin.gif
მაგრამ რაგაც პროფესიონალობას ვერ გატყობ და შენი აზრი დიდად არ მაინტერესებს biggrin.gif
როცა არაფერი იცი უნდა გაჩუმდე ზოგჯერ

მაშინ არ დავწვრილმანდები. პროფესიონალის პასუხს დაელოდე givi.gif

Posted by: axx 29 Sep 2009, 17:19
QUOTE
ცალ მხარეს ძუძუსთავი ვაწვალე და ამის შედეგად ერთი წვეთი გამონადენი მქონდა

და ცალ მხარეს რა უბედურებაა? 1 ხელი გაქვს მარტო? rolleyes.gif (ალბათ მეორეთი სხვა რამეს აწვალებდი) eek.gif ღმერთმა შეგინდოს pop.gif

Posted by: texasuri jleta benzoxerxit 30 Sep 2009, 02:12
QUOTE (axx @ 29 Sep 2009, 17:19 )
QUOTE
ცალ მხარეს ძუძუსთავი ვაწვალე და ამის შედეგად ერთი წვეთი გამონადენი მქონდა

და ცალ მხარეს რა უბედურებაა? 1 ხელი გაქვს მარტო? rolleyes.gif (ალბათ მეორეთი სხვა რამეს აწვალებდი) eek.gif ღმერთმა შეგინდოს pop.gif

givi.gif givi.gif givi.gif givi.gif givi.gif givi.gif

Posted by: ANGELofDEATH 30 Sep 2009, 02:28
lol.gif lol.gif lol.gif lol.gif lol.gif lol.gif lol.gif lol.gif lol.gif lol.gif axx ra cudi xar

Posted by: ana_nushka 2 Oct 2009, 12:27
არ ვიცი ეს კითხვა ამ თემას შეესაბამება თუ არა მაგრამ მინც დავწერ . . . rolleyes.gif

თუ ხართ ინფორმირებულები რამდენად საშიშია ლაზერული ეპილაცია??? rolleyes.gif იღლიაში . . . ან ხალზე??? spy.gif

spy.gif

Posted by: ფარსმანი 2 Oct 2009, 20:33
მგონი კარგად ვერ არის ჩემი საქმე:

ზურგი მტკივა, უფრო სწორად მარჯვენა მხრის ქვედა ნაწილი. როცა წელში ვიხრები ტკივილს ვგრძნობ. ასევე ტკივილს ვგრძნობ ღრმად ჩასუნთქვის დროს. ბოლო ორი დღეა რაც ეს ტკივილები დამეწყო.
სამსახურში ფანჯარა და კარები ხანდახან ღია მაქვს, ასევე კონდიციონერიც ჩართულია მუდმივად.
ეს ხომ არ არის მიზეზი? რამეს ხომ ვერ მირჩევდით?
იმედია რაიმე სერიოზული არ არის

Posted by: anonymouska 3 Oct 2009, 02:24
ana_nushka
ნამდვილად არაა კარგი. //
ფარსმანი
ნევროლოგიური გამოკვლევა ჩაიტარე. მგონი რადიკულიტია, თან გულმკერდის რადიკულიტმა იცის სუნთქვის დროს ტკივილი.

Posted by: ფარსმანი 3 Oct 2009, 10:15
anonymouska
რა დროს ჩემი რადიკულიტია : (
გაციებას არ შეუძლია მსგავსი ტკივილების გამოწვევა?

Posted by: lexomd 3 Oct 2009, 21:26
ANGELofDEATH
QUOTE
მარცხენა იღლია მიღიზიანდება ძალიან ხშირად პატარა სიმაგრე ჩნდება ხოლმე რამოდენიმე დღე მიგრძელდება და ხანდახან ისე მტკივა ხოლმე ხელს ვერ ვამოძრავებ...


ანტიპერსპირანტს იყენებთ?
Harmpy_Harmpy
QUOTE
გინეკოლოგ-ენდოკრინოლოგს უნდა მიმართოთ. პროლაქტინის დონეა სანახავი რა თქმა უნდა მამოლოგთანაც უნდა მიხვიდეთ

სწორია. და მარცხენა იღლიაში ხომ არაფერი გამოგდით? კვანძი ან რამე?

* * *
RAULA
QUOTE
1 შეკითხვა დაბალი სიცხეები ნევროზისგან შეიძლება იყოს გამოწვეული ?

ნაკლებსავარაუდოა. სხვა არაფერი ჩივილია?
Mr Adam
QUOTE
ოსტეოქონდროზი და ცალმხრივი თავის ტკივილი..

QUOTE
მგონი აქ ყველას ფეხებზე კიდია .. ონლაინ დახმარებას არავინ იძლევა

სტეოქონდროზის შველა არაა ეგრე ადვილი, ამიტომაც არავინ იძლევა პასუხს. ცალმხრივი თავის ტკივილი კი თუ ასე ვთქვათ ''მარილების'' ნერვზე ზეწოლითაა გამოწვეული, მაშინ სხვანირი მკურნალობა უნდა, ან შეიძლება იყოს შაკიკი, ან ფრონტიტი, დაწერეთ ტკივილზე დაწვრილებით...

Posted by: ana_nushka 4 Oct 2009, 00:11
anonymouska
QUOTE
ნამდვილად არაა კარგი. //



დღეს ვიქექებოდი გუგლში და ვერანაირი უკუჩვენება ვერ მოვუნახე ლაზერულ ეპილაციას გარდა იმისა რომ შეიძლება კანზე ლაქები ან ფუფხი დატოვოს ისიც არაკვალიფიცირებული სპეციალისტის გამო . . .

და რაც შეეხება ხალებს ეწერა რომ ხალები და ჭორფლი შეიძლება გაგიღავოთო . . . ups.gif

რამდენად სანდო ინფორმაციაა არ ვიცი . . .



მე ის მაწუხებს რომ ლაზერით ფავიკეთე საფეთქელთან და იმის მერე პატარა ხალი გამომივიდა იმ ადგილზე, იმ დღესვე შევამჩნიე, ადრე არ მქონდა ნამდვილად ვიცი . . .sad.gif

ძალიან უმნიშვნელო 1მმ დიამეტრის . . .მაგრამ მაინც ვნერვიულობ . . .საშიში ხომ არ არის? და ვის უნდა მივმართო ??? user.gif იქნებ მირჩიოთ რამე . . .mo.gif

Posted by: mainc 5 Oct 2009, 17:33
მმმ
უმიზეზოდ ფეხის გული კიარაა ტერფი რომ გიხურს
სინამდვილეში არ გიხურს , მარა ასეთი შეგრძნება როა, ცოტა ხნით და მერე გადაივლის
ეს ნერვების გამოა
?

მმმმ

rolleyes.gif

Posted by: -LesTaT- 5 Oct 2009, 19:52
ხერხემლის 4-5- დისკები მაქ ჩაჩ###ტილი,, ეგგრე მითრრა 1-მა ეიქმმა მერე სადაზღვეოში მივედი იქ ეს სხვა რაღAაცს გავსო და ჩამწერს შემდეგ ორშაბატს ნევროპათოლოგტან ამ დღეებში სხვა ექიმთანაც მივალ ,მარა დაივაღAლე უკვე,,
3 დღეა ვიკეთEბ ამ ნემსებს მარა დიდი შედეგი არ მოაქვს...

იქნებ ვინმემ იცდოეს რამე კარგი გზა რომ დავდივარ წელის ქვედა ნაწილსი რავი ალბატ კუდუსინის ძვლსი დაბოლოება მტკვია ცოტატი, წელიც ცოტატი, და დისკონმპორტი მაქ სიარულის დროს და რა ვქნა sad.gif

Posted by: Solveig 6 Oct 2009, 13:55
ვინმემ იქნებ მიპასუხოს sad.gif

რამდენად მართებულია, თუ ადამიანს აქვს ვენების უკმარისობა (დიაგნოზში ასე ეწერა) და ექიმი უნიშნავს ასეთ რამეს: ვოლტარენი კოჭიდან მუხლამდე და 2 ფენა ელასტიკური ბანდი, უნდა ატაროს 24 საათის განმავლობაში, 4 დღე (მერე ექიმთან მვიიდეს).

ღამით შეხსნა მაინც არ შეიძლება ნეტავ? საშინლად ამწუხებს sad.gif

თან დღეს კიბეებზე მიწევს სიარული და სულ მიფეთქავს sad.gif

Posted by: pravinia 7 Oct 2009, 00:56
მგონი ამ თემაში უნდა ვიკითხო:
მითხრეს რომ ნუროფენი არის სიცხის დამარეგულირებელი, ანუ რომც არ ჰქონდეს ბავშვს (1 წლისაზე მაქვს საუბარი), არ დააგდებს და თან მატებასაც ხელს შეუშლის, ამას იმიტომ ვკითხულობ რომ 39 სიცხიდან 35 და 5 ხაზზე დაუგდო კალპოლმა, მიუხედავად დოზების და მიღების ინტერვალების ზუსტი დაცვისა, ამიტომ სამომავლოდ მინდა რაღაც უფრო უსაფრთხო შევარჩიო, იქნებ მიპასუხოთ smile.gif თუ ბავშვზე ვერ მიპასუხებთ, რადგან ეს პედიატრის პრეროგატივა უფროა, მაშინ უბრალოდ ნუროფენის თაობაზე გამოცდილება მაინც გამიზიარეთ smile.gif

Posted by: kukurukuku 7 Oct 2009, 15:51
მოკლედ მაინტერესებს, მთელ თბილისში როა მოდებული რაღAც საშინელი ვირუსი, ჩემს ქმარს აქვს უკვე 6 დღეა 37- 37.4 სიცხეები, უფრო მაღლა არ ადის, ცოტა ახველებს, და ზოგჯერ აქვს გულის რევის შეგრძნება. რა წამალი შეიძლება დალიოს? თუ საჭიროა ექიმთან მისვლა ?

Posted by: texasuri jleta benzoxerxit 11 Oct 2009, 21:17
QUOTE (kukurukuku @ 7 Oct 2009, 15:51 )
მოკლედ მაინტერესებს, მთელ თბილისში როა მოდებული რაღAც საშინელი ვირუსი, ჩემს ქმარს აქვს უკვე 6 დღეა 37- 37.4 სიცხეები, უფრო მაღლა არ ადის, ცოტა ახველებს, და ზოგჯერ აქვს გულის რევის შეგრძნება. რა წამალი შეიძლება დალიოს? თუ საჭიროა ექიმთან მისვლა ?

აუცილებლად საჭიროა ტერაპევტთან ვიზიტი. ფილტვების ანთების გამოსარიცხად(რითიც ძალიან ხშირად რთულდება ეს ვირუსი)

Posted by: SPC 15 Oct 2009, 15:51
1-2 თვეა და დღეს განსაკუთრებით დამეწყო ხელის კანკალი.
დღეს რაღაცას ვწერდი და საწერ კალამს ძვლივს ვიმორჩილებდი. რისი ბრალი შეიძლება იყოს?

Posted by: Romina 15 Oct 2009, 19:05
SPC

ცალ ხელშია მარტო?

Posted by: SPC 15 Oct 2009, 19:25
Romina
არა ორივე მგონი
წერისას მარჯვენა უფრო შესამჩნევი იყო

Posted by: Romina 15 Oct 2009, 19:30
SPC

ნევროლოგმა აუცილებლად უნდა გაგსინჯოს. აუცილებლად! სახლში რაიმე ალკოჰოლური სასმელი გაქვს? ცოტა რომ მოსვა (ნახევარი ჭიქა მაგალითად) და ნახე თუ გაგივლის.

Posted by: SPC 15 Oct 2009, 19:37
Romina
დიდი მადლობა

ვეცდები აბა

Posted by: ana_maniani 17 Oct 2009, 16:29
თუ სხვა თემის კითხვაა მაპატიეთ
იქნებ ვინმემ იცოდეთ სხეულზე სისხლჩაქცევები რისგან შეიძლება იყოს გამოწვეული???
გმადლობთ

Posted by: BaBajanka 18 Oct 2009, 10:11
ვერაფრით ვერ მივხვდი რის ექიმს უნდა მივმართო.

აი რაშია საქმე.

მაქვს პაპილომა ვირუსი 16-18 შტამი, რამდენადაც ვიცი საშვილოსნოს ყელს, სწორ ნაწლავს და პირის ღრუს ემუქრება ეს ვირუსი კიბოთი. რამდენიმე წლის წინ ყელში ვგრძნობდი რაღაც წანაზარდს, ხორცმეტივით, რომელიც ხან იყო, ხან არა. მაშინ მე-9-ში ვიყავი სუხიაშვილთან, რომელმაც ფაქტიურად დამცინა, ნევროზული (თუ ნერვოზული) გჭირს, არაფერი არ გაქვსო, გეჩვენებაო, მაგრამ მაინც დამიბარა, მერე მითხრა, ფარინგიტია, მაგრამ მაინც არაფერიო, მოკლედ ვერაფერი საინტერესო მისგან ვერ გავიგე.
მერე რამდენიმე წელი აღარ მქონია, შარშან დაიწყო სასაზე გამოსვლა ერთმა წყლულმა, კარგ ადგილას იყო , ყველა ხედავდა და ჩემს ფანტაზიის ნაყოფს ვეღარ დაარქმევდი უკვე. ისიც ხან ჩნდებოდა, ხან ქრებოდა, დღეში რამდენჯერმე. ვერ მივხვდი რა პრინციპით გამოდიოდა და ჩადიოდა. smile.gif

ახლა უკვე ისევ ყელში მაქვს გამონაზარდი და მაწუხებს ძალიან, ყლაპვაშიც მიშლის. ტკივლით არ მტკივა.. მგონია, რომ ონკოლოგთან უნდა მივიდე, თუ არა? .... აი ვერ გავერკვიე ვის მივაკითხო, იქნებ ვინმემ იცოდეს რა მოვლენას გავს და რომელ ექიმთან მივიდე>?

უღრმესი მადლობა წინასწარ.

Posted by: Solveig 18 Oct 2009, 20:08
BaBajanka
ახლა გამონაზარდი გაქვს თუ წყლული?

Posted by: BaBajanka 18 Oct 2009, 22:03
Solveig

აზრზე არ ვარ, ახლა ყელში მაქვს ღრმად და არ ჩანს
sad.gif

Posted by: Solveig 18 Oct 2009, 22:31
BaBajanka
სად ხარ, საქართველოში თუ სხვაგან?

სხვაგან თუ ხარ, ჯერ ოჯახის ექიმთან მოგიწევს მისვლა, ყველა ვარიანტში...

თუ არადა, მე მგონი, აჯობებს, ჯერ ყელ-ყურ-ცხვირის ექიმთან მიხვიდე. თუკი ღრმადაა, მაგას უფრო ექნება კარგად გასინჯვის საშუალება.


ana_maniani
QUOTE
იქნებ ვინმემ იცოდეთ სხეულზე სისხლჩაქცევები რისგან შეიძლება იყოს გამოწვეული???

ძალიან ბევრი რამით.

ასე რომ, უმჯობესია, ექიმს აჩვენო, როგორი სისხლჩაქცევებია, სადაა ლოკალიზებული (მთელ ტანზე, ქვედა კიდურებზე, წელზე და ა. შ.).

Posted by: BaBajanka 18 Oct 2009, 23:29
Solveig

საქართველოში ვარ და მაგიტომ ვკითხულობ. აი სუხიაშვილმა ისე ამიგდო, რომ ვიფიქრე ეს ყელ-ეურ-ცხვირს ეტყობა არ ეხება-თქო. smile.gif
დიდი მადლობა რჩევისთვის!

Posted by: Breaky 18 Oct 2009, 23:34
BaBajanka
eek.gif eek.gif eek.gif eek.gif eek.gif

შენ აქ ხარ??? და ასე ჩუმად ხარ და არ პოსტავ????
მოყევი ჩქარა როგორ ხარ, რას შვები, შენი პატარა და ალბათ უკვე დიდი კაცი როგორაა????

Posted by: BaBajanka 18 Oct 2009, 23:38
Breaky


smile.gif) ხო მთელი ზაფხულია აქეთ ვარ და ფორუმზე საუკუნეში ერთხელ შემოვდივარ. სურათს იქ დავდებ, ბაღნებში. თქვენ როგორ ხართ??? გუფთა რას შვრება smile.gif

Posted by: Breaky 18 Oct 2009, 23:42
BaBajanka
QUOTE
მთელი ზაფხულია აქეთ ვარ და ფორუმზე საუკუნეში ერთხელ შემოვდივარ


მერე, სინდისი და ნამუსი დაკარგე ხო სულ??? დატოვე ხო იქ? biggrin.gif არადა რამდეჯერ გამახსენდი და ვფიქრობ ხოლმე, სად დაიკარგათქო user.gif

გუფთა არის რა, გუფთობს biggrin.gif გუფთას გარდა კიდევ ბევრი სხვა ეპთეტი შეიძინა.
ველოდები სურათს!!!! 2kiss.gif

Posted by: grafinia 19 Oct 2009, 18:09
ძალიან მაწუხებს ერთი საკიტხი და იქნებ დამეხმაროთ. ვიცი ასე ინტერნეტით ჯანმრტელობაზე და მითუმეტეს დიაგნოზე საუბარი სისულელა, მაგრამ იქნებ ვინმემ მიახლოებითი პასუხი მაინც გამცეთ.
დედა არის 55 წლის. და უჩვეულო რამ სჭირს. გულმკერდის არეში, ზუსტად შუაში (მისახვედრად: მძივი რომ მთავრდება) დროთა განმავლობაში გამოებერება რაღაც მრგვალად, რომელიც წითლდება და ერთი სამი ან ერთი კვირის განმავლობაში აქვს. მერე ქრება. ხომ არ იცით რა უნდა იყოს?

უბრალოდ ექმითან არადა არ მიდის და არადა მე ძალიან ვნერვიულობ და იქნებ მიახლოებით მაინც დამეხმაროთ.

წინასწარ დიდი მადლობა.

Posted by: ana_nushka 20 Oct 2009, 18:33
ხლახნო მაქვს ბრტყელი ტერფი . . . საშუალო სტადიის . . .თუ როგორც ქვია . . .


მარცხენა ფეხზე ცოტათი უფრო ბრტყელტერფიანი ვარ ვიდრე მარჯვენაზე . . . ხოდა ეს მარცხენა ფეხი მისივდება და მტკივა . . .ძალიან მაწუხებს . . .სუპინატორებს ვხმარობ მაგრმა არ მშველის . . .

მარჯვემა ფეხი კი საერთოდ არ მაწუხებს, მიუხედავად იმისა რომ ბრტყელი ტერფი მაქვს მანდაც, ნუ მაგრამ ოოდნავ ცოტათი მსუბუქ ფორმით ვიდრე მარცხენაზე . . . . . . .

ხოოოდა რომელ ექიმს უნდა მიივმართო??? რომ ცოტა მაინც შევიმსუბქო მდგომარეობა და გავიგო რატომ მისივდება და მტკივა ეს დამპალი მარცხენა ფეხი ესე ზალიან . . . .

Posted by: keen 21 Oct 2009, 12:20
vano_t-ამ თემის ავტორი აქ რატომ აღარ შემოდის? spy.gif

მაინც ვიკითხავ... პერიოდულად მაქვს სიმძიმის და უსიამოვნო შეგრძნება კუჭის და მარჯვენა ფერდქვეშა არეში... ტკივილით არ მტკივა. რა შეიძლება იყოს? baby.gif

Posted by: Lacrimosa_ 22 Oct 2009, 22:46
ძვირფასო მეგობრებო, ყოველ 6 თვეში ერთხელ ვიღებ ხოლმე ანალიზებს. ზოგადად ყველაფერი რიგზე მაქვს ხოლმე. მაგრამ დღეს აღებულ ანალიზზე მითხრეს, რომ სისხლის ფორმულაში ცვლილებებია. ექიმთან მინიმუმ ნოემბრის შუა რიცხვებამდე ვერ მივალ, კვირაობით არ მუშაობენ, სხვა დროს კი მე არ მცალია და იქნებ დამეხმაროთ. რასაც გავიგებ, დავწერ აქ და დიდი მადლობელი დაგრჩებით თუ მეტყვით საშიშია თუ არა
მოკლედ,

ჰემოგლობინი 130 (78)
ერითროციტები 4.3*10 12/3
ფერადობის მაჩვენებელი 0.9
ტრომბოციტი 62%
ლეიკოციტი 7.9*10 4/3
მეტამიელოციტი 1
ჩხირბირთვიანი 5
სეგმენტი 57
ეოზინოფილები 5
ლიმფოციტები 28
მონოციტები 2
პლაზმატიური უჯრედები 2
ერითროციტების დალექვის სიჩქარე 5



Posted by: Solveig 23 Oct 2009, 03:09
Lacrimosa_
გაწუხებს რამე? რაიმე ავადმყოფობა ხომ არ გადაგიტანია?


Posted by: gorski*lud*lover 23 Oct 2009, 23:44
გადავისანე ვირუსი რა მოდებული რომ იყო უკვე 2კვირაზე მეტია და მაინც მახველებს უაზროდ ძაან მშრალად რა ვქნა შევწუხდი კაცი მეთქი გაივლის არადა არ გადის რარაც უცნაურად მახველებს თან mad.gif

Posted by: texasuri jleta benzoxerxit 24 Oct 2009, 01:47
QUOTE (gorski*lud*lover @ 23 Oct 2009, 23:44 )
გადავისანე ვირუსი რა მოდებული რომ იყო უკვე 2კვირაზე მეტია და მაინც მახველებს უაზროდ ძაან მშრალად რა ვქნა შევწუხდი კაცი მეთქი გაივლის არადა არ გადის რარაც უცნაურად მახველებს თან mad.gif

საჭიროა ფილტვების აუსკულტაცია(მოსმენა) და/ან რენტგენოსკოპია პნევმონიის გამოსარიცხად.

Posted by: Lacrimosa_ 25 Oct 2009, 11:48
Solveig

არაფერი არ მაწუხებს გარდა ყელისა, რომელიც სამი წელია, ყოველ შემოდგომაზე ერთი-ორი კვირით მტკივდება

Posted by: addicted 26 Oct 2009, 21:26
თქვენი დახმარება მჭირდება smile.gif
ჩემს მეუღლეს აუცნაური შეგრძნება აქვს თავში. ანუ უკეთ რომ ვთქვა მუდმივი სიმძიმის შეგრძნება აქვს. ვის უნდა მივმართოთ? ნევროლოგს თუ თერაპევტს? და რა შეიძლება ჭირდეს?

Posted by: miriani777 30 Oct 2009, 23:22
მოკლედ მქონდა ციტომეგალოვირუსი. ხოდა ექიმმა დამინიშნა ციკლოფერონი და ვიუსიდი გუშინ დავამთავრე. და ამ ბოლო კვირას რაღაც უცნაური შეგრძნებები მაქვს, გამოუფხიზლებელი დავდივარ ანუ რაღაც აღქმის უნარი დავკარგე რა გეგონება არ ვარ ჩემს სხეულში? რისი ბრალია ამ წამლების? მეშინია და ნევროზიც ავიკიდე მგონია რო ვკდები ვარ 21 წლის იქნებ მირჩიოთ რამე? წეღან სასმელიც დავლიე ბევრი და არ დავთვერი რაღაც უცნაურ განცდებში ვარ ანუ არ ვზივარ ჩემში რარაც გაორებულად ვარ არ მინდა ჯერ სიკვდილი და მეფიქრება ამ სიტუაციიდან გამომდინარე

Posted by: martata 31 Oct 2009, 13:43
გეხვეცებით ერთ რამეში გამარკვიეთ,სოკომ შეიძლება გამოიწვიოს მწვავე ცისტიტი
და სისხლში ლიმფოციტების მომატება?
* * *
miriani1987
ციკლოფერონს მეც ვსვავდი და და მეც დაახლოვებით მასეთი შეგრძნება მქონდა,თავბრისხვევა,თითქოს
გათიშული დავდიოდი.არ შეგეშინდეს მგონი ნორმალური უნდა იყოს.მე მალევე გამიარა smile.gif

Posted by: The Factory 2 Nov 2009, 13:55
აუფ, გუშინ მთელი ღამე და დღეს თავბრუ მესხმის საშინლად. ფეხზე ვერ ვდგები ისე. მეტი არაფერი არ მჭირს და რა ჯანდაბაა?

Posted by: _SisterofNight_ 3 Nov 2009, 10:44
ბავშვს ამ თვეში 4 ჯერ ამოუვარდა ხელი და ტრამვატოლოგიურში მეუბნებიან რომ ჩვეული ამოვარდნაა და ასე იცისო 5 წლამდეო, 5 წლამდე ესე ვიყო? სულ შიშის ქვეს? რამე ხო არიცით ან სად მივიყვანო ან რამე წამალი ან მალამო თუ არსებობს? ოსტეოქეარი დაუნიშნა პედიატრმა და სვავს მაგრამ ჯერჯერობით შედეგი არ არის.

Posted by: khatia88 3 Nov 2009, 14:20
არ ვიცოდი სად მეკითხა და აქ დავწერ იქნება მიპასუხოს ვინმემ.
პატარა მყავს 4 თვის, დაახლოებით 2 კვირის წინ ისეთი თმის ცვენა დამეწყო რომ ბღუჯებად ჩამომდის. ვიცოდი რომ ასეთი რამე ლაქტაციის დროს ხდებოდა, მაგრამ მე ძუძუ საერთოდ არ მიჭმევია, არ მქონია რძე და ამ დროსაც ხდება ასეთი რამე? რით ვუშველო?

Posted by: vano_t 4 Nov 2009, 09:47
khatia88
QUOTE
არ ვიცოდი სად მეკითხა და აქ დავწერ იქნება მიპასუხოს ვინმემ.
პატარა მყავს 4 თვის, დაახლოებით 2 კვირის წინ ისეთი თმის ცვენა დამეწყო რომ ბღუჯებად ჩამომდის. ვიცოდი რომ ასეთი რამე ლაქტაციის დროს ხდებოდა, მაგრამ მე ძუძუ საერთოდ არ მიჭმევია, არ მქონია რძე და ამ დროსაც ხდება ასეთი რამე? რით ვუშველო?

შეგიძლია კარგ პედიატრს ანახინო, მაგრამ თუ ბავშვს სხვა არაფერი ჭირს (მაგალითად თავზე არ აქვს გამონაყარი ან რაიმე დაზიანება), მაშინ არაფერია სანერვიულო. ახალშობილებში (6-12 თვემდეც კი) ხშირია თმის ცვენა და სერიოზული მიზეზები ამ ასაკში იშვიათიაო-პედიატრების თქმით.

http://www.babycenter.com/0_hair-loss_85.bc არის ინგლისური სტატია და საკმაოდ კარგად ახსნილი მშობლებისათვის.

Posted by: khatia88 4 Nov 2009, 13:55
vano_t
გმადლობთ პასუხისათვის, მაგრამ ჩემს ბიჭს კი არა მე მცვივა თმა smile.gif

Posted by: kolhaun 6 Nov 2009, 22:32
შეიძლება ამ თემას არ ეხება, მაგრამ დახმარებისათვის წინასწარ გიხდით მადლობას - წამალი ესპერალი გამოდის თუ არა ამპულებში და თუ კი სურათი ხომ არ იცით სად შეიძლება ვნახო, გამიკეთეს მაგრამ რაღაც ვერსად ვერ ვნახე ინტერნეტში რომ ამპულებში გამოდიოდეს და რაღაც ეჭვი ავიღე, იქნებ მომაწოდოთ რამე ინფორმაცია

Posted by: Miss_Nata 8 Nov 2009, 19:31
შეგიძLიათ ამიხსნათ, სისხლში ფერიტინის მეტობა რას ნიშნავს???
წინასწარ დიდი მადლობა.

Posted by: Goddess 11 Nov 2009, 15:49
ძალიან გთხოვთ დამეხმაროთ...

უკვე მესამე დღეა მტკივა წელის არეში გვერდები... ტკივილი ცოტა წინ გადმომდის, მუცლის ღრუზე. გუშინ მივედი ექიმთან (დაზღვევის ოჯახის ექიმი). ხელით გამსინჯა და დამიწერა ორი წამალი:

ფურამაგი 1კაფსულა ორჯერ დღეში (აუცილებლად დააყოლე ერთი ჭიქა წყალიო)

პესტო 2 აბი ორჯერ დღეში...

ცოტა სწერვა ექიმი იყო (ჩემი გამოძახებაზე იყო) და ჩემი მოთხოვნის მიუხედავად არ ამიხსნა ნორმალურად არაფერი... ანუ.. მხოლოდ ხელით გამსინჯა და დამინიშნა პირდაპირ. ეხო არ გადამაღებინა... ანალიზის (ს/ს, შ/ს) პასუხები ჯერ არ არის... ამიტომ ეჭვით ვუყურებ დანიშნულებას, როცა ჯერ მიზეზი არ ვიცი ტკივილის...

ეგებ რამენაირად გამათვითცნობიეროთ?? თორემ საღამო ახლოვდება და ტკივილიც იმატებს... გამაგიჟა უკვე..
* * *
ისა... ამ თემაში როგორ არის? კაცი რომ გადატრიალდება და გაფშიკავს ფეხებს პასუხებს მერე გაღმა უგზავნით??? boli.gif

Posted by: niniaa 11 Nov 2009, 18:30
Goddess
თირკმელებზე ან საშარდე სისტემაზე მიიტანა ეჭვი? გქონდათ როდისმე საშარდე სისტემის პათოლოგია გამოვლენილი?

Posted by: Goddess 11 Nov 2009, 19:25
niniaa
თირკმელებზე მიიტანა ეჭვი როგორც მივხვდი. ხელით გამისინჯა და მანდ მტკიოდა თუმცა კომენტარი არ გაუკეთებია.
ადრე არანაირი პათოლოგია არ მქონია. ჩემით მაინც წავედი ექო გადავიღე და ყველაფერი კარგად იყო... ანალიზის პასუხიი ხვალ ოქნება..

არადა ეს წამალი რომელიც დამინიშნარმ წავიკითხე ანოტაცია არ შეიძება მიღება თუ თირკმლის ფუნქცია მოშლილიაო თუ რაღაც და არადა თირკმელზე მაქვს მგონი პრობლემა

Posted by: chxira 12 Nov 2009, 02:43
მკერდის კიბოს შემთხვევაში, როცა ამბობენ ხოლმე მკერდი მოკვეთესო, რა იგულისხმება ამაში,
ვუზუალურად, გარედან იცვლება რამე, თუ შიგნიდან ხდება გარკვეული მცირე ქსოვლიების გამოღება? sad.gif

Posted by: gviriluka 12 Nov 2009, 04:44
გამარჯობა!79 წლის კალბატონს გაუსივდა მარცხენა თვალი და ლოკა. აკვს გამონადენი წყალივით.დროში ირევა და არა ადეკვატურია ხოლმე.2 დგეა თითქმის არაპერს ჭამს. იკნებ მითხრათ,რა შეიძლება იკოს ეს?სჰეუსივდა სადგაც 20 დგის ცინ.5-6 საათიანი მგზავრობის შემდეგ.გმადლობთ!

Posted by: chxira 12 Nov 2009, 16:16
QUOTE
ისა... ამ თემაში როგორ არის? კაცი რომ გადატრიალდება და გაფშიკავს ფეხებს პასუხებს მერე გაღმა უგზავნით???


მეც დავინტერესდი მაგით, ადრეც და ეხლაც, მემგონი სურათი უნდა დადო კითხვასთან ერთად, ან საკუთარი იდეოლოგიის შესახებ დაწერო რამე ორი სიტყვით, მერე შემოვლენ, გადაათვალიერებენ, მოეწონებათ-გიპასუხებენ, არ მოეწონებათ-არ გიპასუხებენ, მოკლედ , ისე უფრო გაქვს შანსი boli.gif

Posted by: niniaa 12 Nov 2009, 19:10
Goddess
QUOTE
არადა ეს წამალი რომელიც დამინიშნარმ წავიკითხე ანოტაცია არ შეიძება მიღება თუ თირკმლის ფუნქცია მოშლილიაო თუ რაღაც და არადა თირკმელზე მაქვს მგონი პრობლემა

ფუნქციის მოშლაში იგულისხმება თირკმლის უკმარისობა, რაც თქვენ შემთხვევაში გამოირიცხა. საინტერესოა ანალიზების პასუხი...
* * *
gviriluka
QUOTE
სჰეუსივდა სადგაც 20 დგის ცინ

სიცხე არ ჰქონია? შესიებამდე არაფერს უჩიოდა?გამონადენი თვალიდან აქვს?ექიმს არ უნახავს?

Posted by: gviriluka 12 Nov 2009, 19:44
[B] niniaa [/ B]
სიცხე არ ვიცი არ ა მგონია. უცჰვეულოს, ახალს არაპერს, უცჰიოდა. გამონადენი დვალიდან აკვს, მგონი ცოტა სჰეუმცირდა .. მომენტებსჰი რაგაცებს ამბობს რაც არ ხდება. რამოდრნიმე დგეა. უმეტესად სცორად აზროვნებს. ვპიკრობ, სკლეროზულია. მაგრამ ეს ტვალი და ლოკა მასჰინებს. გმადლობტ კურადგებისატვის,

Posted by: niniaa 12 Nov 2009, 20:38
gviriluka
QUOTE
ვპიკრობ, სკლეროზულია.

თუ მანამდე აზროვნებაში პრობლემები არ ჰქონდა და ახლა დაეწყო თვალის და ლოყის გასიებასთან დაკავშირებით, მაშინ აუცილებელია ექიმი!

Posted by: RadioShack 12 Nov 2009, 20:44
ძალიან ხშირად ავადვყოფობ სიცხეებს მაძლევს მაგრამ მალე მივლის სულ დაღლილობის და უძილობის შეგრძნება მაქ რამდენიც არ უნდა მეძინოს sleep.gif
რას მირჩევთ ? rolleyes.gif

Posted by: Lizikuna3 13 Nov 2009, 16:58
ერთი კითხვა მაქვს!

ხშირად თითქმის მთელი შემოდგომა ზამთარი და გაზაფხული გაყინული მაქვს ხოლმე ხელები. მაინტერესებს რისი ბრალი შეიძლება რომ იყოს და რამდენად საშიშია.

Posted by: niniaa 13 Nov 2009, 19:16
RadioShack
ჩაიტარე გამოკვლევები: საერთო ანალიზი სისხლის, შარდის,თერაპევტის, ყელ-ყურის ექიმის , ენდოკრინოლოგის კონსულტაცია... უნდა დადგინდეს მიზეზი, რომ სწორი რჩევა მიიღოთ.

Posted by: RadioShack 13 Nov 2009, 23:24
რამე სხვა რჩევა ხომ არგაქვთ? (ანალიზებზე უკიდურეს შემთხვევაში წავალ, რადგან ფობია მაქ ნემსის)

Posted by: niniaa 14 Nov 2009, 12:53
RadioShack
შეუძლებელია გამოკვლევების გარეშე ვინმემ სწორი და დაზუსტებული დიაგნოზი დასვას.შეიძლება , შენი ჩივილები ზედმეტ ფსიქიურ ან ფიზიკურ გადაღლასთან იყოს დაკავშირებული, შეიძლება სისხლნალლებობასთან, ან ფარისებური ჯირკვლების ფუნქციის დარღვევასთან, ან ვიტამინების ნაკლებობასთან,ან იმუნიტეტის დაქვეითებასთან, ან ნუშისებური ჯირკვლების ანთებასთან,ან ....ბევრი მიზეზია ,რამაც ანალოგიური ჩივილები შეიძლება გამოიწვიოს და ამიტომ , რომ მოხდეს დიფერენცირება, აუცილებელია ანალიზები.

Posted by: skoch 14 Nov 2009, 13:40
თმის ჩაბრუნება კუდუსუნში მგონი ესე ქვია,,რა ხდება ამ დროს??აუცილებელია ოპერაცია თუ შეილება თვითონაც დაიშალოს და გაიწოვოს??

Posted by: Alpinistka123 16 Nov 2009, 22:15

ზალიან გტხოვტ დამეხმარეტ!!!!!

გამომეხმაურეტ, რა ნევროპატოლოგები დიდ გასაჩირში ვარტ. 18 წლის ახალგაზრდას წლების განმავლობაში ტკიოდა ყური, მერე აგმოჩნდა რომ დაზიანებული ქონდა ყურის ბარაბანი. გაუკეტდა ოპერაცია, ოპერაციის დასრულებისტანავე აგმოვაჩინეტ რომ ვერ ხუჩავდა ტვალს იმავე მხარეს და ტუჩებიც გვერდზე მოექცა. ჩემის აზრიტ დაუზიანეს სახის ნერვი. ტვიტონ ექიმებმა კი შემოგვტავაზეს ტომოგრაპიის გაკეტება იმ მიზეზიტ, რომ ეს ნაკლი ტავიდან არის გამოწვეულიო, მაგრამ რაგა მაინცდამაინც ეხლა ოპერაციისტანავე აგმოჩნდა ეს ყველაპერი. გვეპარება ეჩვი ეს ექიმების შეცდომაა, რომლის აგიარებაც არ უნდატ. როგორ მოვიქცეტ?
ზალიან გტხოვტ მომაწოდეტ ინპორმაცია რა შეიზლებოდა მომხდარიყო.

Posted by: Mariii_555 16 Nov 2009, 22:31
გამარჯობათ. ესეთი შეკითხვა მექნება და მოუთმენლად დაველოდები თქვენს პასუხებს smile.gif 53 წლის კაცს აქვს კუჭისაშლილობა უკვე ერთი კვირაა და ტანზე ქავილი. რენდგენი გაკეთებული აქვს და ყველაფერი წესრიგში აქვს, მუცლის ღრუ ექოსკოპიურად გადაგებული აქვს და მანდაც წესრიგია. ისე გადატანილი აქვს ც ჰეპატიტი რომელიც უკვე იმკურნალა და მაგის ბრალი ხომ არ შეიძლება რომ იყოს ტანზე ქავილი გინდაც ეს კუჭისაშლილობა? წინასწარ მადლობთ.
* * *
ძალიან გთხოვთ რაც მოგცერეთ მაგაზე იქნებ მიპასუხოთ? მადლობთ.

Posted by: Dr Pepper 17 Nov 2009, 03:22
skoch
ოპერაციას საჭიროებს თითქმის ყოველთვის

Alpinistka123
გამარჯობათ!
საერთოდ ბევრი ქირურგიული ჩარევის დროს შესაძლებელია მოხდეს
ცენტრალური ან პერიფერიული ნერვული სისტემის მამოძრავებელი ცენტრებისა და გამტარი გზების დაზიანება რაც იმ კონკრეტული არეალის ან ორგანოსა და ორგანოიდის გათიშვით (დამბლით) მთავრდება...
როგორც ჩანს თქვენ მართალი ბრძანდებით რადგან სახის ნერვი მიმიკურ კუნთებს აინერვირებს, შესაბამისად პირის ირგვლივ კუნთს, თვალის დაღუჭვა გახელაც მასზეა დამოკიდებული.... გადაიღეთ CT უბრალოდ უნდა იცოდეთ რომ არც ისე იაფი ღირს smile.gif რაც შეეხება ამ ნერვის გადანერგვას მინდა გითხრათ რომ ხდება ასეთი რამე... ეს იგივე პლასტიკური ქირურგიაა, მაგრამ არა ესთეთიკური. ამიტომ ისინი ვინც პლასტიკურ ქირურგიაში არიან საქართველოში ნაკლებად მგონია ეს შეძლონ... სახის ნერვის გადანერგვა, კუნთის ტრანსპორტიზაცია 10 ბალიანი სისტემით რომ შევაფასოთ, მე-9 კატეგორიის სირთულის ოპერაციაა, იმიტომ, რომ მუშაობა მიდის კუნთებსა და ნერვებზე...
ჩემი აზრით კარგი იქნებოდა თუ საქმის კურსში სხვა, უკეთეს და უფრო მეტად გამოცდილ ექიმსაც ჩააყენებდით...

Posted by: Alpinistka123 17 Nov 2009, 14:04
Dr Pepper

დიდი მადლობა შემოხმიანებისტვის. ოპერაცია გაკეტდა 4 დგის წინ, დგეს დაბარებულია მკურნალ ექიმტან. საქმე ისაა, რომ ამ ყველაპერს ფინანსები ჩირდება რაც არა გვაქვს და ეს ტუ ექიმის შეცდომაა რატომ არ უნდა იყოს პასუხისმგებელი შედეგებზე, ტვიტონ არ უნდა დაპაროს შემდგომი მკურნალობის ხარჯები? ვის უნდა მოვტხოვოტ პასუხი? იქნებ დამაკვალიანოტ. ცნობისტვის ოპერაცია გაკეტდა 9 საავადმყოპოში

Posted by: gorski*lud*lover 17 Nov 2009, 14:04
Dr Pepper
QUOTE
ოპერაციას საჭიროებს თითქმის ყოველთვის

ადგილობრივი ანესთეზიით შეილება გაკეთდეს თუ აუცილებელია ვენური???
Dr Pepper
რაც შეეხება ამ ნერვის გადანერგვას მინდა გითხრათ რომ ხდება ასეთი რამე... ეს იგივე პლასტიკური ქირურგიაა, მაგრამ არა ესთეთიკური. ამიტომ ისინი ვინც პლასტიკურ ქირურგიაში არიან საქართველოში ნაკლებად მგონია ეს შეძლონ... სახის ნერვის გადანერგვა, კუნთის ტრანსპორტიზაცია 10 ბალიანი სისტემით რომ შევაფასოთ, მე-9 კატეგორიის სირთულის ოპერაციაა,
ზურა ჩიჩუა ალბათ გაგიგონია არ შეუძლია მას ეს გააკეთოს??

Posted by: Mariii_555 17 Nov 2009, 14:36
მეც მიპასუხეთ რააა ძალიან გთხოვთ sad.gif((((((((((((((((((((((((((((((((

Posted by: niniaa 17 Nov 2009, 17:12
Mariii_555
ქავილი და კუჭის აშლილობა შეიძლება იყოს დაკავშირებული ჰეპატიტთან. ღვიძლის ფუნქციური სინჯები გადაამოწმეთ.

Posted by: Dr Pepper 17 Nov 2009, 17:16
Mariii_555
გასტროსკოპიაც გაიკეთოს ყველა ვარიანტში smile.gif

Alpinistka123
საინტერესოა ვინ გააკეთა ოპერაცია.. მე9ში ასე თუ ისე ვიცნობ ოტორინოლარინგოლოგებს smile.gif
ნუ რათქმაუნდა თუ ეს მათი შეცდომაა მათვე უნდა გამოასწორონ მე ასე მგონია.. იურიდიულ საკითხებში ვერ ვერკვევი სამწუხაროდ..

gorski*lud*lover
არ ვიცნობ სამწუხაროდ სად მუშაობს?

Posted by: ANGELofDEATH 17 Nov 2009, 17:23
სისხლის ანალიზი ავიღე დღეს და არ ვიცი ცუდია თუ არა biggrin.gif
ჰემოგლობინი 120
ერითროციტები 3.7
ფერადობის მაჩვენებელი 0.97
ტრომბოციტები 260
ლეიკოციტები 5.5
ჩხირბირთვიანი 1
სეგმენტბირთვიანი 50
ეოზინოფილები 4
ლიმფოციტები 37
მონოციტები 8
ეს ლიმფოციტები მგონი მაღალია არა? რას უნდა ნიშნავდეს? rolleyes.gif

Posted by: Alpinistka123 17 Nov 2009, 18:27
Dr Pepper

QUOTE
საინტერესოა ვინ გააკეთა ოპერაცია.. მე9ში ასე თუ ისე ვიცნობ ოტორინოლარინგოლოგებს
ნუ რათქმაუნდა თუ ეს მათი შეცდომაა მათვე უნდა გამოასწორონ მე ასე მგონია.. იურიდიულ საკითხებში ვერ ვერკვევი სამწუხაროდ..



მე ჯერჯერობიტ ტავს ვიკავებ ექიმის ვინაობის დასახელებისაგან. ტქვენ ეჩვი გამოტქვიტ, ტუ მატი შეცდომააო და აშკარა არ არის რომ შეცდომაა?? ოპერაციამდე არ იდგა ეს პრობლემა. ამწუტას პრობლემა უსახსრობაა და ბავშვი რჩება ინვალიდი და ვისი წყალობიტ? მიხედონ მაინც ტუ დააშავენ, ტორემ ჩივილი იდეაპიქსად არა მაქვს. დგეს გავიგებ რა გაირკვა და ტქვენც იქნებ გამიგოტ რაიმე sad.gif

Posted by: Dr Pepper 17 Nov 2009, 19:26
Alpinistka123
რა არის იცით?
მე ვერავის ხელს ვერ დავადებ ოქეი? უბრალოდ ვამბობ რომ ის პრობლემა რაც სამწუხაროდ თქვენს ახლობელს აქვს ჩემი ვარაუდით ოპერაციის შემდგომ წამოჭრილი პრობლემაა და აქ თავის ტვინი არაფერ შუაშია... ზემოთ პოსტში გითხარით რომ სახის ნერვით სწორედ ის არეალები ინერვირდება რომელიც დაზიანებული აქვს თქვენს ახლობელს ასე რომ აქ ფიზიკურად ვერ შეიძლება ყოფილიყო პრობლემა მანამდე და სახელდობრ ოპერაციის მერე ასე "სასტიკად" აღმოცენილიყო...

Posted by: Mariii_555 17 Nov 2009, 19:47
niniaa
დიდი მადლობა პასუხისთვის. და ეს ღვიძლის ფუნქციური სინჯები სად უნდა აიგოს ხომ ვერ მეტკვი? და რამდენას საშიშია?

Posted by: niniaa 17 Nov 2009, 19:54
Mariii_555

QUOTE
ეს ღვიძლის ფუნქციური სინჯები სად უნდა აიგოს ხომ ვერ მეტკვი?

ყველა ნორმალური ლაბორატორია გააკეთებს მაგ ანალიზს!საიმედოდ სად აკეთებენ ვერ გეტყვით, იქნებ სხვამ გიპასუხოთ ამ კითხვაზე.
QUOTE
და რამდენას საშიშია?

სერიოზული მისახედია, რადგან ღვიძლის ფუნქციის დარღვევა შეიძლება იყოს მაგ სიმპტომების მიზეზი.

Posted by: Mariii_555 17 Nov 2009, 19:59
მადლობთ პასუხისთვის. ისე თქვენ ექიმი ხართ?

Posted by: niniaa 17 Nov 2009, 20:17
Mariii_555
QUOTE
მადლობთ პასუხისთვის. ისე თქვენ ექიმი ხართ?

დიახ.

Posted by: Mariii_555 17 Nov 2009, 21:53
მადლობთ. შემთხვევით ფასი ხომ არ იცით მაგ ანალიზების?

Posted by: niniaa 17 Nov 2009, 22:28
Mariii_555
ყველგან თავისი ფასები ექნებათ. მიახლოებითაც ვერ გეტყვი, სამწუხაროდ.იქნებ სხვა გამოგეხმაუროს!

Posted by: Mariii_555 17 Nov 2009, 22:30
კარგი, უგრმესი მადლობა პასუხებისთვის.

Posted by: Dr Pepper 17 Nov 2009, 22:33
მარი ჩვენთან რესპუბლიკურში არის ახალი ლაბორატორია საკმაოდ მაგარი

Posted by: Mariii_555 17 Nov 2009, 22:57
ხოო? და იქნებ ფასები მითხრათ რომ ვიცოდე?
* * *
Dr Pepper
იქნებ პასუხი მომწეროთ რა ეღირება მაგ ანალიზის ჩაბარება? მადლობთ................

Posted by: Dr Pepper 17 Nov 2009, 23:51
მარი ნამდვილად არ ვიცი sad.gif
ვეცდები გავიგო smile.gif

Posted by: Mariii_555 18 Nov 2009, 00:11
ხო რა იქნებ გამიგო? დიდი მადლობელი ვიქნები smile.gif

Posted by: elloellonatia 22 Nov 2009, 13:39
იქნებ მიპასუხოთ რაა...

თვალებს შორის,ცხვირზე გამომივიდა რაღაცარის გამაგრებული და მთლიანად გამისივდა ცხვირის ზედა ნაწილი...რამდენიმე დღეა მაქვს და არმივლის..პირიქით დაჯე უფრო ემატება სიწითლე და გასიება

რა შეიძლება წავისვა,ან საერთოდ რა შეიძლება იყოს...

მითხარით რაააააა ;(

Posted by: texasuri jleta benzoxerxit 23 Nov 2009, 02:47
QUOTE (elloellonatia @ 22 Nov 2009, 13:39 )
იქნებ მიპასუხოთ რაა...

თვალებს შორის,ცხვირზე გამომივიდა რაღაცარის გამაგრებული და მთლიანად გამისივდა ცხვირის ზედა ნაწილი...რამდენიმე დღეა მაქვს და არმივლის..პირიქით დაჯე უფრო ემატება სიწითლე და გასიება

რა შეიძლება წავისვა,ან საერთოდ რა შეიძლება იყოს...

მითხარით რაააააა ;(

ყბა-სახის ქირურგს მიაკითხე. მაგ მიდამოს ინფექცია საშიშია,რადგანაც შეიძლება ტვინის გარსებისა და ტვინისკენ გავრცელდეს fig.gif

Posted by: Oscar Schindler 23 Nov 2009, 03:18
ვინმემ კარგი თერაპევტი მირჩიეთ რაა გთხოვთთთ cry.gif
ჩემი ქმრის ხველება არ მომწონს და ვინმეს მინდა ოვასმენიო,მე ვინც მყავს ნანახი თერაპევტები გონზე არ არიან და ისეთი მირჩიეთ რო არ შემომაკვდეს

Posted by: grigolich 23 Nov 2009, 16:50
აუ მე მხარი დავძაბე რა მძიმე წონა ავწიე და იმის მერე მიტკაცუნებს და ნერვებს მიშლის..გამოვლის?????? რამე შეიძლება რომ ვუშველო?

Posted by: იტალიელი 23 Nov 2009, 22:48
ტავი მისკდება და მეზობელმა კეტოროლი მომცა დალიეო და ვაბშე თავის ტკივილის წამალია?

Posted by: Mariii_555 25 Nov 2009, 17:18
Oscar Schindler
ჩაჩავაში მიდით მანდ არის თერაპევტი ქალი ოგონდ გვარი და სახელი არ მახსოვს სამწუხაროდ sad.gif

Posted by: lala 29 Nov 2009, 12:34
გამარჯობათ
ბოლო დღეებია გულისრევა და თავის ტკივილი მაწუხებს
ხელებზე გამიჩნდა ფრჩხილებთან მოყვითალო მოწითალო ფერის ლაქები, აი იოდს რო აქვს ხელზე ისეთი ფერის, არადა საერთოდ ხელი არაფრისთვის არ მიხლია, დილას მქონდა, რას შეეძლო გამოეწვია და თუ იცის რამემ?
განსაკუთრებით ცერა თითებთან,
ძაან ვნერვიულობ

Posted by: gviriluka 2 Dec 2009, 20:01
გამარჯობა.რა ვქნა არ ვიცი დედა მიკვდება და ვერაპერს ვუკეტებ უსახსრობის გამო.მტავრობა ბრმაა და კრუ.საავადმკოპო ესაცჰიროება რეანიმაცია.1 დგე მინიმუმ 600 ლარია.მე კი პატარა სჰვილიტ, სულ მარტო ვარ.ისიც კი არ სჰემიზლია გავიდე და რამე ვიგონო.სახელმციპოს ვტხოვო დახმარება.დავიგუპე.იკნებ მირჩიოტ რა ვკნა. გავლენიანს ვერავის ვაცვდენ ხმას.ეხლა ვზივარ და ჩემი ტავი მეზიზგება, ასეტი უზხლური რომ ვარ.რა ვკნა.

Posted by: წრიპა 2 Dec 2009, 20:37
gviriluka
რა აწუხებს დედას და რამდენი წლის არის?

Posted by: gviriluka 2 Dec 2009, 22:22
75 ცლისაა.რაჩასჰი მგზავრობის შემდეგ გაუსივდა მარცხენა ტვალი და ლოკა.მეგონა ინპეკცია იკო.მაგრამ,მერე დამზიმდა და რეანიმაციაში გადავიკვანეტ.დიაგნოზი;კრონიკული პნევმონია და ობსტრუკციული ბრონხუტი.ხშირად არა ადეკვატურია შაბატსგამოვედიტ სახლში.იმის გამო რომ 6 დგე დამტავრდა და მიტხრეს ტუ გადაიხდიტ იკოს,ტუ არა ცაიკვანეტო ტან კატეტერიც დაუტოვეს.რამაც ინფეკცია გამოიცვია და სიცხემ აუცია .სადგაც 3 საატის ცინ ამოვუგეტ. ახლა37 აკვს.მინდა ისევ რეანიმაციაში დავაცვინო,მაგრამ ტანხა არ გამაჩნია.

Posted by: taamo 4 Dec 2009, 23:43
მოკლედ ასეთი რამ მაწუხებს, ზაფხულში კეფაზე გამიჩნდა სველი გამონაყარი, უფრო სწორად გამონაყარი არაა, ქერქები კეთდება სისველის შედეგად. მერე ეს ქერქები მცვივა და ისევ ახლიდან იწყება. დერმატოლოგმა რაღაც მალამო დამიმზადა, მაგრამ არ მიშველა. თან საქმე ისაა, რომ 2 წლის წინაც დამემართა დაახლოებით ასეთი რამ, სახეზე გამიჩნდა მსგავსი რაღაც, ოღონდ მაშინ წყლის მიკარება არ შეიძლებოდა, რადგან ლამის მთელ სხეულზე მომედო, მაშინაც იმავე დერმატოლოგმა დამიმზადა მალამო და იმან მიშველა. და ბოლოს რაც მთავარია, ეხლანდელი გამონაყარი მექავება და ხშირად თავს ვერ ვიკავებ. იქნებ რამე მირჩიოთ.

Posted by: almada 5 Dec 2009, 00:23
საკმაო ხანია მაწუხებს კიდურების რევმატიული ტკივილები, განსაკუთრებით მტკივა მარცხენა მხარე, იქნებ მირჩიოთ სად მივიდე გამოსაკვლევად, და რა დაავადება შეიძლება მჭირდეს. ვარ 23 წლის. მადლობა წინასწარ

Posted by: almada 6 Dec 2009, 11:54
არ მიპასუხებთ?.... user.gif
...

Posted by: LUKA-BRAZI 7 Dec 2009, 17:10
იქნებ დავეხმაროთ ამ ადამიანს? user.gif

http://forum.ge/?f=25&showtopic=34046008&st=0

* * *
QUOTE
საკმაო ხანია მაწუხებს კიდურების რევმატიული ტკივილები, განსაკუთრებით მტკივა მარცხენა მხარე, იქნებ მირჩიოთ სად მივიდე გამოსაკვლევად, და რა დაავადება შეიძლება მჭირდეს. ვარ 23 წლის. მადლობა წინასწარ

Romina თუ მოიცლი იქნებ გასცე პასუხი ამ კითხვას smile.gif

Posted by: kasandra 7 Dec 2009, 21:22
მჭირდება წესიერი სამედიცინო დაწესებულების მისამართი თბილისში, სადაც შეიძლება გაიკეთო სისხლის ანალიზი, გაიარო ლარინგოლოგის, რევმატოლოგის, ენდოკრინოლოგის კონსულტაცია. პაციენტი 18 წლისაა და პირველად მიდის მარტო სამედიცინო დაწესებულებაში, ასე რომ პირანიებთან ნუ გამაშვებინებთ. user.gif

Posted by: shtori 7 Dec 2009, 21:58
ბოლო დროს თავის ტკივილი მომეჩვია. საღამოობით კომპიუტერთან თუ დავჯდები ცოტა ხანში თავი მტკივდება, მიმძიმდება, საცაა გასკდებაო (ასატანი კია მარა..). ან შაბათ-კვირას სახლში როცა ვარ მთელი დღე მტკივა. ანუ გამოდის სახლი მატკიებს თავს. საღამოს გადაღლილობას ვაბრალებ. შაბათ-კვირას კი დასვენებას. რა გავიგო ახლა რის გამოა?
თუ კარგად დავაკვირდი უმოძრაობისას, ფიზიკურად როცა არ ვარ დატვირთული, ეგ მატკიებს


კიდე, ეს ისე მაინტერესებს, წყალში ჩაყვინთვისას თავში ხმაური (ტკაცუნი) ნორმალურია? ხან მეშინია ამდენი ტკაცუნის. გეგონება იბზარებაო : )) biggrin.gif

Posted by: kaxa111 7 Dec 2009, 22:50
ვინმეს შეგიძლიათ მითხრათ რენტგენზე გამოკვლევისას კლოიბერის ცრუ ფიალები რა შემთხვევაში იქნება?

Posted by: ირაკლო 7 Dec 2009, 23:41
აუ ხალხო 2 დღეა თვალები უსაშინლესად მტკივა მონიტორის ყურებისას. ტელევიზორზე რეაქცია არ მაქვს და რა გზას დავადგე? კაი ოკულისტი ვინ იცით?

Posted by: katusia86 9 Dec 2009, 15:27
გამარჯობათ, ვუჩივი მარჯვნივ მკერდის ტკივილს ეს ტკივილი გადაეცემე მხარს და ყურს თან ყურში მაქვს ქავილი, ზუსტად იქ სადაც მტკივა მკერდი შიგნიდან მიღუტუნებს და მახველებს, მაქვს ქრონიკული ფარინგიტი და თუ შეძლება ამის ბრალი იყოს? თან ფილტვების მეშინია smile.gifwink.gif

Posted by: Mariii_555 9 Dec 2009, 16:32
ამიტომ თავი რო დაიზღვიო ჯობია მიმართო ექიმს smile.gif აქ კი გირჩევენ მარა დახმარებით ვერ დაგეხმარებიან.

Posted by: Bluetooth_a 11 Dec 2009, 04:13
ძილის საკითხები რომელ ექიმს ეხება? yawn.gif user.gif

Posted by: Mariii_555 11 Dec 2009, 14:53
.Bluetooth_a
თუ არ ვცდები ნევროფათოლოგს ეხება.

Posted by: ბახარხალა 11 Dec 2009, 15:41
პირს რო ვაღებ მარჯვენა ყური მტკივა ისე გეგონება ყბის კუნთი მაქ დაჭიმულიო (სერიოზულად) და რა ვქნა? givi.gif




ოღონდ არ დამიწეროთ ნუ გააღებ პირსო :|

Posted by: axx 16 Dec 2009, 16:49
დღეს გავიკეთე ეხოსკოპია თუ რაღაც ჯანდაბა მუხლზე და მენისკი აღმომაჩნდა sad.gif მითხრეს საოპერაციოაო, მარა ეხლა ოპერაციის არც დრო მაქვს და არც ნერვები, და რომ დავიკიდო და არ გავიკეთო ოპერაცია რა დონემდე შეიძლება გართულდეს? boli.gif

Posted by: vajiko_1 16 Dec 2009, 17:34
კისერზე გამომივიდა ბურთივით რაღაც, კანის ქვეშ არის და მარტო ხელის დადებით იგრძნობა. მითხრეს ჯირკვალია და ვიტამინების ნაკლებობამ იცისო, რა არის და რამდენად საშიში შეიძლება იყოს, ან რისი ბრალია, ან ვის უნდა მივმართო იქნებ მითხრათ?

Posted by: ტანკე 16 Dec 2009, 21:56
გაუმარჯოს კაი ხალხს smile.gif

თქვენი რეკომენდაცია მჭირდება,წავიდე თუ არა ექიმთან.
მოკლედ,5 დღის წინ გრიპი მქონდა,ტემპერატურა 37.5 და ვირუსისთვის დამახასიათებელი ყველა თანმხლები ნიშნით.

ტერაფლუს ვსვამდი და მიშველასავით,მარა ბოლო სამი-ოთხი დღეა,37-37.1 მაქვს სიცხე.ოდნავ სუნთქვა მიჭირს,სხვა არაფერი.ხანდახან ყელის ჩაწმენდა მიხდება.დღეში 1-2-ჯერ თუ წამომამხველებს ლაითად,1 წამით.როგორც მეუბნებიან,ფერი მაქვს ცუდი.სხვა არაფერი მაწუხებს...ოდნავ გაბრუებული ვარ.

მივაკითხო ექიმს?

ფილტვებზე მოსმენა დამჭირდება?
ეს დაბალი სიცხე ვერ მოვიცილე ვერაფრით.


Posted by: lolita88 17 Dec 2009, 16:34
აუ ხალხო მიშველეთ რა...

ფეხის ფრჩხილი დამიჩირქდა და მტკივა... გუშინ მოვიჭერი ფრცხილები და ეტყობა რაღაც ინფექცია შეეპარა...

რა ვქნა?

Posted by: Mariii_555 17 Dec 2009, 19:24
მე 18 კვირის ორსული ვარ და კუჭის მოქმედება მაქვს ძალიან ცუდი sad.gif იქნებ რამე წამალი მირჩიოთ და კიდე ბუტალაქს ვსვავ ხოლმე ზოგჯერ 5 წვეთს და რამეს ხომ არ ავნებს ნაყოფს? მადლობთ წინასწარ.

Posted by: Sunny_Lovely 17 Dec 2009, 21:04
თუ შეგიძლიათ დამეხმარეთ რა,

ვარ 19 წლის, მაქვს მუდმივი თავის ტკივილები, ხშირად თავბრუმეხვევა და წნევა მაქვს ხოლმე დაბალი 80/60ზე ან უფრო დაბალი. ამ ბოლო დროს ამას დაემატა ისიც, რომ ყურადღება მეფანტება, სწავლა მიჭირს და მეხსიერებაც გამიუარესდა. ამას გარდა სულ მეძინება, მიუხედავად იმისა რომ 8 საათზე ნაკლები არ მძინავს.
ორი თვის წინ გავიკეთე ფარისებრი ჯირკვლის ჰორმონის ანალიზი და ნორმაში იყო.
გაკეთებული მაქვს ინტრაკრანიალური დოპლეროსკოპია და თავის ტვინის ორივე ნაწილში მაქვს სპაზმები. ნევროპათოლოგმა დამინიშნა ვაზობრალი, სინკაპტონი, ვაგოსტაბილი და აქტივოგენი მონაცვლეობით. ათი დღეა რაც ვსვავ წამლებს, პირველი სამი დღე უკეთესად ვიკავი, ეხლა ისევ იგივე სიმპტომები მაწუხებს თავის ტკივილის გამოკლებით. ძილიანობას და მეხსიერებას ვერაფრით ვშველი. რისი ბრალი შეიძლება იყოს და რა ვქნა?

Posted by: axx 18 Dec 2009, 01:08
ქ არავინ არაფერზე გვპასუხობს mad.gif

Posted by: texasuri jleta benzoxerxit 18 Dec 2009, 04:33
Sunny_Lovely
შეიძლება შენიღბული,ფარული დეპრესია იყოს.
lolita88
სასცრაფოდ ქირურგთან!
ტანკე
რაღა თქმა უნდა საჭიროა ფილტვების აუსკულტაცია და საჭიროებისამებს რენტგენოსკოპია.
vajiko_1
პირველ რიგში უნდა მიმართო თერაპევტს,რომელიც მოგცემს დანიშნულებას.

Posted by: pouet 18 Dec 2009, 15:02
გამარჯობა; გუშინ ამიღეს სისხლი ვენიდან, ანალიზისთვის. დიდი ნემსი იყო და მეტკინა, მაგრამ სახლში რომ მოვედი, ისევ მტკიოდა და გასიებული მქონდა, დღეს ცოტა ჩაცხრა მგონი, მაგრამ მაინც მტკივა თითქმის იდაყვამდე. საშიშია თუ დავიცადო და გამივლის? ნორმალურია ესეთი რეაქცია.? იმედია, მიპასუხებთ, ცოტა დავშინდიsad.gif
* * *
ექიმოოოოოოოოო მიშველე!!!! გთხოვ!

Posted by: gorski*lud*lover 18 Dec 2009, 15:59
pouet
QUOTE
გამარჯობა; გუშინ ამიღეს სისხლი ვენიდან, ანალიზისთვის. დიდი ნემსი იყო და მეტკინა, მაგრამ სახლში რომ მოვედი, ისევ მტკიოდა და გასიებული მქონდა, დღეს ცოტა ჩაცხრა მგონი, მაგრამ მაინც მტკივა თითქმის იდაყვამდე. საშიშია თუ დავიცადო და გამივლის? ნორმალურია ესეთი რეაქცია.? იმედია, მიპასუხებთ, ცოტა დავშინდი
* * *
ექიმოოოოოოოოო მიშველე!!!! გთხოვ!

ამ ...ყ ექომების დ..რო შევ....მე გამიხვრიტეს ვენა და თრომბი მკონდა შენ შეილება გაგიხეთქეს და დიდი ლურჯი ლაქასავით გექნება ეს თუა გაივლის მალე..

Posted by: pouet 18 Dec 2009, 16:06
gorski*lud*lover
ლურჯი მაქვს უკვე, გუშინაც ლურჯი მქონდა, მაგრამ გასიებულია საკმაოდsad.gif თრომბი არ ჩანს ჯერჯერობით... თან მტკივა. ახალგაზრდა იყო, რას მერჩოდა...

Posted by: gvelis_wiwila 19 Dec 2009, 10:44
ესეიგი სრული სერიოზულობით ვაცხადებ რომ გუშინ წინ თავზე გადამამტვრიეს შუშის გრაფინი. ვიყავო მეცხრეში და ნაკერები დამადეს. გული არ წამსვლია და არც გავთიშულვარ. თავბრუც დიდად არ დამხვევია.

ეს ჰემატომას და ძალიან საშიშ რაღაცეებს რისგანაც კვდებიან ან ინვალიდდებიან გამორიცხავს ხო ავტომატურად?

Posted by: cruel__woman 19 Dec 2009, 20:18
მაინტერესებს სისხლის და შარდის ანალიზი ყველა პოლიკნინიკაში კეთდება? და რა ღირს თუ ვინმემ იცის? პასუხების შემდეგ ვინ იძლევა მედიკამენტურ დანიშნულებას?

Posted by: unknown lady 21 Dec 2009, 16:28
ერთი გამაცნობიერეთ ნარკოლოგიურში ცნობა როგორ აიღება

Posted by: maknatuna 26 Dec 2009, 01:01
გამარჯობათ
უმეტესად (თითქმის ყოველ საღამოს) ამეღნიშნება 37.2 ტემპერატურა.
არ მაქვს სურდო. სისხლის საერთოში მხოლოდ ლიმფოციტები მაქვს ძალიან დაბალი. 10 (ნორმა 20-40)
რომელ ექიმს მივმართო? ან რა გამოკვლევები გავიკეთო?

Posted by: asiko 26 Dec 2009, 23:21
შეკითხვა: ექიმმა წამალი დანიშნა ჭამის წინ. წამლის ანოტაციაში წერია, რომ წამალი მიიღება ჭამის შემდეგ. რომელს დავუჯეროთ?

Posted by: anonymouska 27 Dec 2009, 01:45
maknatuna
ფილტვები ? იცის დაბალი სიცხე. აუცილებლად შეამოწმე

Posted by: lost_for_words 27 Dec 2009, 17:01
ცხვირიდან სისხლის წამოსვლა მოულოდნელად ძალიან ცუდია ?

Posted by: Randolph Engineering 27 Dec 2009, 18:19
გამარჯობა/თ

თვალის ქვეშ რო სილურჯეებია საერთოდ მოცილება ან შედარებით გაუფერულება შეიძლება?
არ ვგულისხმობ ტონალურ კრემებს და მსგავს საშუალებებეს.

რა შეიძლება იყოს გამომწვევი მიზეზი? (ბებიაჩემს ქონდა დედაჩემს და უფრო მგონია რო გენეტიკურია)
რამე უფრო სერიოზულზე ხო არ უნდა ვიფიქრო?

Posted by: anonymouska 27 Dec 2009, 19:04
lost_for_words
ხშირი სისხლდენა გაქვს ? ასპირინს და ტკივილგამაყუჩებელს თუ სვამ?

ყოველშემთხვევაში თუ კიდე გამეორდა თერაპევტს მიმართე.

Randolph Engineering
ეგ თვალის ირგვლის მდებარე კანის სისხლძარღვთა გაფართოების და მათი გამჭვირვალობის მომატების შედეგია.

კი გენეტიკური არის ხოლმე და მიზეზიც ბევრი რამ შიეძლება იყოს... სტრესი, კომპთან თუ დიდ დროს ატარებ, გადაღლა, დეპრესია, ნიკოტინი, ალკოჰოლი და ა.შ

თუ დატვირტული/გადაღლილი ხარ ეცადე დაისვენო და საკმარისად იძინო. ც ვიტამნს თუ მიიღებ და მასაჟებს გაიკეთებ კარგია.

Posted by: lost_for_words 27 Dec 2009, 19:52
anonymouska
არ ვსვავ არცერთს. პმ ნახე აბა mo.gif

Posted by: shota_xuc 28 Dec 2009, 15:07
ხომ ვერავინ მეტყვის, დერმატოლოგი იოსებ კობახიძე რომელ კლინიკაში მუშაობს?

Posted by: loli-lola 29 Dec 2009, 15:28
ინფექციონისტს რომელ განყოფილებაში შემიძლია დავეკონტაქტო?

Posted by: Randolph Engineering 29 Dec 2009, 20:14
anonymouska
მადლობ
__________

Posted by: niniaa 29 Dec 2009, 23:22
maknatuna
QUOTE
უმეტესად (თითქმის ყოველ საღამოს) ამეღნიშნება 37.2 ტემპერატურა.
არ მაქვს სურდო. სისხლის საერთოში მხოლოდ ლიმფოციტები მაქვს ძალიან დაბალი. 10 (ნორმა 20-40)
რომელ ექიმს მივმართო? ან რა გამოკვლევები გავიკეთო?

თუ აბსოლუტური ლიმფოპენიაა,მაშინ მიზეზი ლიმფურ სისტემაში უნდა ვეძებოთ, რასაკვირველია, ტუბერკულოზზეც ვიფიქროთ .გადაამოწმეთ ერთხელ კიდევ სისხლის ანალიზი და დაითვალონ ლიმფოციტები,აბსოლუტური რაოდენობა განსაზღვრონ.

Posted by: maknatuna 30 Dec 2009, 00:13
niniaa
QUOTE
გადაამოწმეთ ერთხელ კიდევ სისხლის ანალიზი და დაითვალონ ლიმფოციტები,აბსოლუტური რაოდენობა განსაზღვრონ

ამ წელს სამჯერ მაქვს გადამოწმებული.
გაზაფხულით სისიხლის გადასმის ინსტიტუტში გავიკეთე, მაშინ ლიმფოციტები მქონდა 12
აგვისტოში მე-6 პოლიკლინიკაში, ლიმფოციტები ნორმა მქონდა 22
ნოემბერში კი ისევ სისხლის გადასხმის ინსტიტუტში და ისევ დაბალი მქონდა ლიმფოციტები.10
user.gif

Posted by: taso 31 Dec 2009, 00:40
QUOTE
და ცოტ-ცოტა ყველაფერი.

სადაც საჭირო იყო, იქ არ მიპასუხეს და იქნებ აქ მიპასუხოთ sad.gif

რითი შეიძლება დავადგინო, რომ ადამიანი ნარკომანია, თუ წელიწადში რამდენჯერმე იკეთებს?

Posted by: niniaa 31 Dec 2009, 02:24
maknatuna
რატომ გაიკეთეთ ანალიზი, რამე გაწუხებდათ?
საინტერესოა ასევე, ლიმფოციტების რაოდენობის შემცირება შეფარდებითი იყო/ანუ ნეიტროფილების მომატების ხარჯზე/ თუ აბსოლუტური/1ლ სისხლში ლიმფოციტების რაოდენობს შემცირება/?
QUOTE
გაზაფხულით სისიხლის გადასმის ინსტიტუტში გავიკეთე, მაშინ ლიმფოციტები მქონდა 12
აგვისტოში მე-6 პოლიკლინიკაში, ლიმფოციტები ნორმა მქონდა 22
ნოემბერში კი ისევ სისხლის გადასხმის ინსტიტუტში და ისევ დაბალი მქონდა ლიმფოციტები.10

რომელიმე ლაბორანტი ხომ არ ცდება?

Posted by: maknatuna 31 Dec 2009, 14:32
niniaa
QUOTE
რატომ გაიკეთეთ ანალიზი, რამე გაწუხებდათ?

გაზაფხულით კი, დავსივიდი და ვერ გამიგეს ვერაფერი. რამდენიმე ანალიზი გავიკეთე და ყველაფერი ნორმაში მქონდა.
დანარჩენი კი ჩემი სურვილით გავიკეთე.
QUOTE
საინტერესოა ასევე, ლიმფოციტების რაოდენობის შემცირება შეფარდებითი იყო/ანუ ნეიტროფილების მომატების ხარჯზე

მხოლოდ მონოციტები მქონდა მომატებული, უფრო სწორად ზედა ზღვარი.
QUOTE
რომელიმე ლაბორანტი ხომ არ ცდება?

არ ვიცი user.gif
შეგიძლიათ რომელიმე ლაბორატორია მირჩიოთ, სადაც ნორმალურად აკეთებენ ანალიზებს?

Posted by: niniaa 1 Jan 2010, 00:37
maknatuna
QUOTE
გაზაფხულით კი, დავსივიდი და ვერ გამიგეს ვერაფერი.

მთლიანი სხეული იყო შესიებული?სიცხეები თუ გქონდათ მაშინ? ექოსკოპია თუ გაიკეთეთ? ელენთა რა მდგომარეობაში იყო?
ორ სხვადასხვა ლაბორატორიაში ერთსა და იმავე დღეს გაიკეთეთ, თუ სხვაობა მოგვცა , ე.ი. ცდებიან / აუცილებლად ლიმფოციტების აბსოლუტური რაოდენობა განსაზღვრონ/.

Posted by: dadn 1 Jan 2010, 17:07
გამარჯობათ. მოგილოცავთ ახალ წელს ,გისურვებთ ჯანმრთელობას და ბედნიერებას .....
მაინტერესებს Tinnitus-ს სმენის დაქვეითებით საქართველოში თუ მკურნალობენ ? გერმანიაში ვცხოვრობ და აქაურმა ექიმებმა 2 კვირიანი სისხლის გამათხელებელ (უშედეგო ინფუზიონების) შემდეგ მირჩიეს რომ ჩემ ახალ დაავადებას (სიმპტომს) მივეჩვიო ,რადგან ჯერ ამ დაავადების წინააღმდეგ წამალი არ გამოუგონიათ.

Posted by: Breaky 2 Jan 2010, 14:43
ანტიბიოტიკების მიღებისას, ბოლო აბის მიღების შემდეგ რამდენ ხანში გამოდის ის ორგანიზმიდან?
ანუ როდის შეუძლია ადამიანს დაუბრუნდეს ჩვეულ ცხოვრებას? მიიღოს ალკოჰოლი, ჰქონდეს დაუცველი სექსი და ა.შ.

Posted by: LUKA-BRAZI 2 Jan 2010, 14:58
Breaky
QUOTE
ანტიბიოტიკების მიღებისას, ბოლო აბის მიღების შემდეგ რამდენ ხანში გამოდის ის ორგანიზმიდან?

ყველა მედიკამენტი ინდივიდუალურია. ყველას საკუთარი ნახევარდაშლის პერიოდი აქვს. მგონი ყველა წამალს უნდა ეწეროს ანოტაციაში ეგ ინფო.

Posted by: Breaky 2 Jan 2010, 15:07
LUKA-BRAZI
ანოტაციაში კარგად ვერ გავიგე, თუ ღვიძლის პრობლემა არ გაქვს, ნახევრადგამოყოფის პერიოდი 3-5 საათიაო. ეგაა ორგანიზმიდან
სრულად გამოტანის დრო?
ინტერნეტში რაც ვნახე, წერია - Half Life: 4 hours

Posted by: LUKA-BRAZI 3 Jan 2010, 15:28
Breaky
QUOTE
ანოტაციაში კარგად ვერ გავიგე, თუ ღვიძლის პრობლემა არ გაქვს, ნახევრადგამოყოფის პერიოდი 3-5 საათიაო. ეგაა ორგანიზმიდან სრულად გამოტანის დრო?

სრულად არა, ნახევარი yes.gif

სწორედ მაგას ნიშნავს რომ ორგანიზმში 3-5 საათში მაგ წამლის კონცენტრაცია იქნება მაქსიმალურის ნახევარი. ანუ წამლის რაოდენობა ორგანიზმში განახევრდება.

Posted by: miau 3 Jan 2010, 23:26
ფორუმელო ექიმებო, თქვენ განსაკუთრებულად გაფასებთ და იქნებ რაიმე მირჩიოთ.

მქონდა გრიპი სიცხეებით, სიცხეებმა გამიარა და მერ დაბალი სიცხეები მქონდა - 35.5 და ა.შ> მერე დაბალი წნევა, ახლა ეს ორივე გამომისწორდა და ისე საშინლად მაქვს ფეხების კუნთები დაჭიმული, აღარ ვიცი რა ვქნა sad.gif
რა შეიძლება გავაკეთო?

Posted by: remixtedi 4 Jan 2010, 12:06
(კითხვა ოფთალმოლოგთან) პოსტში.
კითხვა დავსვი ოფთალმოლოგთან და რატომღაც არ მპასუხობენ არ გვყავს ოფთალმოლოგები საქართველოში თუ რაც მე მაინტერესებს და არა მარტო მე!!! არ უნდათ საიდუმლოს გამჟღავნება? იქნებ აქ მაინც მითხრას ვინმემ რა უკუჩვენება აქვს ლაზერს? (თვალის ლაზერს ვგულისხმობ) იქნებ ვინმემ გაგვანათლოს fig.gif

Posted by: LUKA-BRAZI 4 Jan 2010, 12:15
remixtedi
ინტერნეტში ნახე შენც. ეგ ისეთი საკითხია რომ არაკომპეტენტურ რჩევას ჯობია არ მოუსმინო და კომპეტენტურ რცევაზე გარანტიას ფორუმზე ვინ მოგცემს? ეს ჯანმედის წესებსიც კი წერია...... თან გააჩნია რისთვის გინდა ეგ ლაზერი, მხედველობის კორექციისათვის თუ ბადურის გასამაგრებლად..... თუ სხვა რამისთვის კიდე....

Posted by: remixtedi 4 Jan 2010, 12:21
მე უკვე ვიცი მარა ამათი პასუხი მაინტერესებს იმიტო მე თვითონ მაქ ამ საკითხზე აქ თბილიში შეხება თვალთან დაკავშირებით და მინდა ვიცოდე ვისთან მაქვს საქმე რა დონეზე არიან ჩვენი ექიმები საზღვარგარეთ წასვლის საშუალება არ მაქ თორე დღეს ყველაზე ხელმისაწვდომი და ასე თუ ისე გარანტიას უკრაინაში იძლევიან

Posted by: LUKA-BRAZI 4 Jan 2010, 12:29
remixtedi
ნუ მაშინ არ ვიცი. არიან თუ არა ამ ფორუმზე ოფთალმოლოგები ეგეც არ ვიცი...
QUOTE
მე უკვე ვიცი მარა ამათი პასუხი მაინტერესებს

ვინმე კონკრეტულზე ამბობ?

Posted by: Mariii_555 4 Jan 2010, 15:45
გამარჯობათ. ყველას გილოცავთ ახალ წელს მრავალს დაესწარით smile.gif ესეთი შეკითხვა მაქვს 21 კვირის ორსული ვარ და კუჭის მოქმედებამ ძალიან გამაწვალა sad.gif ბოდიში ამ თემაზე მაგრამ კი გავდივარ კუჭში მარა ჭინთვებით და რამდენად საშიშია ეს? sad.gif წინასწარ მადლობთ.

Posted by: da_ra 4 Jan 2010, 20:41
ე.ი. რჩევა მჭირდება..

ახალ წელს რაკეტა არ გავარდა და იმენნო ყურთან ახლოს ამიფეთქდა. ძალიან ძლიერი ხმა ქონდა.

უკვე 4 დღეა ყურში მიწუის. ცოტა წუილმა დაიწია ან ტვინი შეეჩვია.
მარა მაგრად მაწუხებს.

ხოდა ექიმთან მისვლა საჭიროა?

თუ თავისით გამივლის?

თუ მთელი ცხოვრება წუილს უნდა ვუსმინო? sad.gif(

Posted by: LUKA-BRAZI 4 Jan 2010, 20:55
da_ra
QUOTE
ხოდა ექიმთან მისვლა საჭიროა?

QUOTE
უკვე 4 დღეა ყურში მიწუის.

მგონი აჯობებს რომ მიხვიდე yes.gif

Posted by: Natuka NGN 4 Jan 2010, 21:13
დღეს გამომიწერეს ერიუსი, და სადღაც 3 საათის შემდეგ შევნიშნე თითქოს პირის სიმშრალე... ჩავხედე "პობოჩკებს" და ბავშვებში არის მსგავსი. რა ვქნა? ხვალაც დავლიო და თუ გაძIლერდა შევწყვიტო? მარა ქავილი მაქვს და ვწუხდები ისე ანტიჰისტამინურის გარეშე...

Posted by: niniaa 4 Jan 2010, 22:43
Mariii_555
QUOTE
გავდივარ კუჭში მარა ჭინთვებით და რამდენად საშიშია ეს?

შეეცადე სითხეები მიიღო,ზეთიანი საკვები, ბოსტნეული, ვაშლი და ყველაფერი ის, რაც კუჭის მოქმედებას გაგიადვილებს,თორემ შეიძლება ბუასილი აიკიდო.

Posted by: Mariii_555 4 Jan 2010, 23:13
niniaa
მადლობთ რჩევისთვის. ისე ეს ახლახანს დამეწყო მანამდე მთლად ესე არ ვიყავი sad.gif თუ იცით ბუტალაქსი რამდენად მისაღებია ორსულისთვის?

Posted by: maknatuna 4 Jan 2010, 23:57
niniaa
QUOTE
მთლიანი სხეული იყო შესიებული?სიცხეები თუ გქონდათ მაშინ? ექოსკოპია თუ გაიკეთეთ? ელენთა რა მდგომარეობაში იყო?

მთლიანი სხეული მქონდა შესივებული. (სხვათაშორის მიხდებოდა biggrin.gif ) ექოსკოპია იმ პეროდში არ მახსოვს გავიკეთე თუ არა, მაგრამ ელენთა სხვა დროს ყოველთვის ფორმაში ყოფილა.
ვერაფერი რომ ვერ გამიგეს, არც წამალი დაუნიშნიათ. მხოლოდ წოლითი რეჟიმი დამინიშნეს და თბილად უნდა ვყოფილიყავი.
QUOTE
ორ სხვადასხვა ლაბორატორიაში ერთსა და იმავე დღეს გაიკეთეთ

და როგორ გავიგო რომელი ტყუის? spy.gif მომიწევს ალბათ მესამეჯერ თითზე ჩხვლეტა user.gif

Posted by: nino_sh 5 Jan 2010, 17:13
დედაჩემის მიყვანა მინდა ექიმთან-ალბათ თერაპევტთან, ამ ბოლო დროს კუჭი აწუხებს ძალიან, ხან ამბობს ღვიძლიო, ხან თირკმელებიო, მოკლედ ვერ გავიგე რა ტკივა. ფაქტია ცუდადაა ქალი.
ხომ ვერ მეტყვით ვისთან შეიძლება მივიყვანო-თან კარგი ექიმი რომ იყოს და თან შაბათს რომ მუშაობდეს???? ისე დღეებში სამსახურიდან ვერ გავდივარ, თვითონ დედაჩემი კი ცოცხალი თავით არ მიდის ექიმთან, შაბათს ძალით უნდა წავიყვანო.

Posted by: dadn 5 Jan 2010, 18:13
QUOTE (da_ra @ 4 Jan 2010, 20:41 )
ე.ი. რჩევა მჭირდება..

ახალ წელს რაკეტა არ გავარდა და იმენნო ყურთან ახლოს ამიფეთქდა. ძალიან ძლიერი ხმა ქონდა.

უკვე 4 დღეა ყურში მიწუის. ცოტა წუილმა დაიწია ან ტვინი შეეჩვია.
მარა მაგრად მაწუხებს.

ხოდა ექიმთან მისვლა საჭიროა?

თუ თავისით გამივლის?

თუ მთელი ცხოვრება წუილს უნდა ვუსმინო? sad.gif(

არ მინდა შეგაშინო მაგრამ ზაან სტრაფად ყურის ექიმთან უნდა მიხვიდე.. მეც დაახლოებით იგივე სიმპტომი მაქ უკვე თვე ნახევარი. თუ წუილი არ გავიდა მთელი ცხოვრება შეიზლება გაგყვეს...სასტრაფოდ ექიმტან ........ყურის ნიჯარას არ მიეწოდება საკმარისი ჯანგბადი სისხლით და ტვინი არასტორად აგიტკვავს იმპულსებს და ტუილის სახიტ გესმის მხოლოდ შენ! ვენაში უნდა გაგიკეთონ სისხლის ჯანგბადის დამაშკარებლებს მინიმუმ 5 ჯერ 250 გრამი..დამიჯერე ტორე შემსავიტ გაწვალდები ..მაგას კვია TINNITUS isi დაავადება....დროზე მიხედე!

Posted by: zeppelina 5 Jan 2010, 20:50
QUOTE (dadn @ 5 Jan 2010, 18:13 )
QUOTE (da_ra @ 4 Jan 2010, 20:41 )
ე.ი. რჩევა მჭირდება..

ახალ წელს რაკეტა არ გავარდა და იმენნო ყურთან ახლოს ამიფეთქდა. ძალიან ძლიერი ხმა ქონდა.

უკვე 4 დღეა ყურში მიწუის.  ცოტა წუილმა დაიწია ან ტვინი შეეჩვია.
მარა მაგრად მაწუხებს.

ხოდა ექიმთან მისვლა საჭიროა?

თუ თავისით გამივლის?

თუ მთელი ცხოვრება  წუილს უნდა ვუსმინო? sad.gif(

არ მინდა შეგაშინო მაგრამ ზაან სტრაფად ყურის ექიმთან უნდა მიხვიდე.. მეც დაახლოებით იგივე სიმპტომი მაქ უკვე თვე ნახევარი. თუ წუილი არ გავიდა მთელი ცხოვრება შეიზლება გაგყვეს...სასტრაფოდ ექიმტან ........ყურის ნიჯარას არ მიეწოდება საკმარისი ჯანგბადი სისხლით და ტვინი არასტორად აგიტკვავს იმპულსებს და ტუილის სახიტ გესმის მხოლოდ შენ! ვენაში უნდა გაგიკეთონ სისხლის ჯანგბადის დამაშკარებლებს მინიმუმ 5 ჯერ 250 გრამი..დამიჯერე ტორე შემსავიტ გაწვალდები ..მაგას კვია TINNITUS isi დაავადება....დროზე მიხედე!

შემაშინე უკვე.
სხვისი ექაუნთიდან მქონდა დაწერილი. მე მაწუხებს ეგ პრობლემა.

--

დავძებნე ინფორმაცია ტინიტუსზე ინტერნეტში და არსად არ წერია რომ ტინნიტუსს იწვევს ძლიერი ხმა.

თქვენც ძლიერი ხმისგან დაგემართათ???

Posted by: niniaa 5 Jan 2010, 21:14
Mariii_555
QUOTE
თუ იცით ბუტალაქსი რამდენად მისაღებია ორსულისთვის?

არ არის სასურველი საფაღარათოების მიღება ორსულობის დროს. პრეპარატების მიღება გინეკოლოგთან უნდა იყოს შეთანხმებული.
* * *

maknatuna
QUOTE
მთლიანი სხეული მქონდა შესივებული.

ძალიან საინტერესო შემთხვევაა.ამიტომ ბევრი კითხვა მექნება:1.შეშუპება უცბად განვითარდა ?2.რამდენ ხანს გაგრძელდა შეშუპება?3.მაგ პერიოდში მარხვაზე ან დიეტაზე ხომ არ იყავით?
3.სიცხე მაშინაც გქონდათ?4.ხელის დაჭერით შეშუპებულზე ღრმული რჩებოდა?

Posted by: dadn 5 Jan 2010, 21:56
zeppelina
მე გერმანულ საიტებზე ვეზებ სადაც საკმაოდ დიდი ინფორმაციაა მაგის შესახებ .. Tinnitus durch Knalltrauma (ზლიერი ხმისაგან )...მე ჯერ ვერც დამიდგინეს რისგან მაქ (90 ზე მეტი გამომწვევი მიზეზი აქ .) ..ყველაზე ცუდი ისაა რომ თუ 6 თვეში
(წუილის, ბზუილის ხან პულსის ) ხმის შეგრზნება არ გავიდა ქრონიკულში გადადის და მკურნალობას ვეგარ ექვემდებარება ...უკვე 2 თვე ვწვალობ და უშედეგოდ ,ჯერ მარცხენა ყურში დამეწყო ახლა მარჯვენაც ...აქვე ტხოვნა მაქ ტუ რამე მკურნალობა არსებობს საქარტველოში Tinnitus ის გამაგებინე ...

Posted by: maknatuna 5 Jan 2010, 22:54
niniaa
QUOTE
შეშუპება უცბად განვითარდა

არა.
QUOTE
2.რამდენ ხანს გაგრძელდა შეშუპება?

2 კვირაზე მეტი, რაც უკვე შესამჩნევი გახდა. (თავიდან მეგონა რომ ძილი არ მყოფნიდა და თვალების შეშუპებას უძილობას ვაბრალებდი. შემდეგ ვიფიქრე რომ მოვსუქდი მეთქი.)
QUOTE
მაგ პერიოდში მარხვაზე ან დიეტაზე ხომ არ იყავით?

დიეტაზე ზაფხულში ვიყავი (შეშუპება ზამთარში განვითარდა)
QUOTE
3.სიცხე მაშინაც გქონდათ?

აღარ მახსოვს sad.gif (თუმცა ორი კვირით ადრე სურდო მქონდა)
QUOTE
ხელის დაჭერით შეშუპებულზე ღრმული რჩებოდა?

კიიი, თან საკმაოდ დიდი. ჩემი წონა 52 კგ იყო, შეშუპების პერიოდში კი 65 გავხდი.

Posted by: Solveig 5 Jan 2010, 23:34
maknatuna

თირკმელები წესრიგში გქონდა? გული?

ან ნივთიერებათა ცვლის რაიმე დარღვევა ხომ არ გაქვს?

Posted by: maknatuna 6 Jan 2010, 01:01
Solveig
QUOTE
თირკმელები წესრიგში გქონდა

კი. ანალიზები კარგი მქონდა.
QUOTE
ან ნივთიერებათა ცვლის რაიმე დარღვევა ხომ არ გაქვს?

გემახსოვრება საბახტარაშვილთან რომ დავდიოდი. ბაზალური ინსულინი მქონდა მომატებული 2 ერთეულით და რაღაც წამლები დამინიშნა. ჰორმონალურიც სხვათაშორის.. 3 თვიანი მკურნალობის შემდეგ, ბაზალური ინსულინის მაჩვენებელმა უფრო მოიმატა და ჩემი სურვილით დავანებე მკურნალობას თავი, რადგან ამ ექიმს მოსმენის კულტურა არ გააჩნია.
თუ ამ წამლებმა გამოიწვია რამე, არ ვიცი.
თუმცა წამლების ჩამონათვალი რომ მივეცი ექიმს, მითხრა არაფერ შუაშია შეშუპებასთანო. თან 6-7 თვე იყო გასული უკვე.

Posted by: niniaa 6 Jan 2010, 12:39
maknatuna
1 .შეშუპების ზემოთ კანის ფერი?
2. შარდვა ნორმალური იყო?შარდის ანალიზი გაკეთდა, გამოირიცხა თირკმლის პათოლოგია,თუნდაც, კენჭის მიერ შარდსადენის სანათურის დახშობა?
3.შეშუპებამდე რა მედიკამენტებს იღებდით?
4.ფარისებრი ჯირკვლის ფუნქცია თუ შემოწმდა შეშუპების დროს?

Posted by: Mariii_555 6 Jan 2010, 14:54
niniaa
გინეკოლოგმა მითხრა მიიღეო მრა მე მაინც არ მინდოდა და მაგიტომ მაინტერესებს ყველას აზრი.

Posted by: niniaa 6 Jan 2010, 18:09
Mariii_555
QUOTE
გინეკოლოგმა მითხრა მიიღეო მრა მე მაინც არ მინდოდა და მაგიტომ მაინტერესებს ყველას აზრი.

რასაკვირველია, თუ უწამლოდ მოაგვარებ პრობლემას, უკეთესია.

Posted by: Mariii_555 6 Jan 2010, 18:27
niniaa
ნუ ეგ თავისთავად მარა ......... sad.gif((

Posted by: maknatuna 7 Jan 2010, 14:12
niniaa
QUOTE
.შეშუპების ზემოთ კანის ფერი?

ჩვეულებრივი, ყველა აღნიშნავდა რომ კარგ ფერზე გამოვიყურებოდი smile.gif
QUOTE
2. შარდვა ნორმალური იყო?შარდის ანალიზი გაკეთდა, გამოირიცხა თირკმლის პათოლოგია,თუნდაც, კენჭის მიერ შარდსადენის სანათურის დახშობა?

კი, გავიკეთე ანალიზები. 2 ტიპის. ორივე ანალიზის პასუხები იდეალური იყო.
QUOTE
3.შეშუპებამდე რა მედიკამენტებს იღებდით?

რამდენიმე თვით ადრე ვსვმდი წამლებს, ზუსტად აღარ მახსოვს მედიკამენტების ჩამონათვალი sad.gif
QUOTE
4.ფარისებრი ჯირკვლის ფუნქცია თუ შემოწმდა შეშუპების დროს?

კი. აქაც ყველაფერი წესრიგში იყო.

პ.ს. შობას გილოცავთ ყველას! ყველა კეთილი სურვილი აგსრულებოდეთ bis.gif 2kiss.gif 2kiss.gif

Posted by: Teri7 10 Jan 2010, 12:12
გამარჯობათ, იქნებ მითხრათ კარგი თერაპევტი ან ჰეპატოლოგი ღვიძლისა და ნაღვლის ბუშტის პრობლემების, უფრო ზუსტად ღვიძლის სტეატოზის სამკურნალოდ. და კიდევ რომელ ლაბორატორიას და ეხოსკოპისტს მირჩევდით?
წინასწარ დიდი მადლობა გულისხმიერებისათვის

Posted by: shtori 12 Jan 2010, 00:29
ჭარბი ძილიანობა რითი შეიძლება იყოს გამოწვეული? რომ არ გააღვიძო 24საათი ეძინება ისე რომ არც ფიზიოლოგიური მოთხოვნილებები შეაწუხებს. თან გაღვიძება არ კმარა. შეიძლება გაიღვიძოს დაგელაპარაკოს, და ისევ, წამიც არ უნდა, ჩაეძინება. ეს 15 საათის ძილის მერეც კი.
თუ ჩვეულებრივი რამეა და ზოგს 6 საათი ყოფნის ზოგს 24 სჭირდება?

Posted by: SPC 13 Jan 2010, 12:50
დაახლოებით ერთი კვირაა გულის რევის შეგრძნება მაქვს...დამიქვეითდა მადა, ტკბილს საერთოდ ვეღარ ვჭამ....
დილაობით განსაკუთრებით მაქვს ეს შეგრძნება.... ერთადერთი იოგურტის ჭამა მისწორდება...საღამოობით ცოტა მშIვდება მაგრამ რომ შევჭამ მერე ისევ იგივე....


რისი ბრალი შეიძLება იყოს, რის ექიმთან უნდა მივიდე....


ანუ 1-2 დღე ემსმის მაგრა 7-9 დღე სულ სგულსი რევის შეგრძნება sad.gif(((((

პირველ დღეებშI თან ერთვოდა სისუსტე, მეგონა სიცხე მქონდა და ვიზომავდი არ მქონდა....

ეხლაც სუსტად ვარ ,,, არადა ბაკურიანშI მივდივარ და თიხლამურები და მჭIრდება რა ენერგია ...სწრაფმოქმედი წამალი ხომ არ იცით ასე ვთქვათ ძალის მოსაცემი? ენერგიის მოსამატებელი?

Posted by: pleita 13 Jan 2010, 14:38
ხალხო, ავექსონს (ანტიბიოტიკს) ვიკეთებ და ცოოტა ვისკი რომ დავლიო რამე??? rolleyes.gif
მალე მიპასუხეთ რა user.gif

Posted by: LUKA-BRAZI 13 Jan 2010, 15:39
pleita
QUOTE
ხალხო, ავექსონს (ანტიბიოტიკს) ვიკეთებ და ცოოტა ვისკი რომ დავლიო რამე???

ცოოტა ვისკიც კი საკმაო რაოდენობით ეთანოლს შეიცავს. მე როგორც ვიცი არ შეიძლება ანტიბიოტიკი და სპირტიანი სასმელი ერთად..

Posted by: pleita 14 Jan 2010, 14:34
LUKA-BRAZI
დავლიე mo.gif და გული მიმდიოდა საღამოს, არ ვიცი რამდენად ამის ბრალი იყო,

Posted by: mtoria 14 Jan 2010, 16:07
რით შეიძლება იყოს გამოწვეული მეზენტერიალური ლიმძფური კვანძების ჰიპერპლაზია?

Posted by: texasuri jleta benzoxerxit 14 Jan 2010, 16:39
QUOTE (mtoria @ 14 Jan 2010, 16:07 )
რით შეიძლება იყოს გამოწვეული მეზენტერიალური ლიმძფური კვანძების ჰიპერპლაზია?

ბევრი რამით. დაწყებული რეაქტიული გადიდებისგან,გამოწვეული ნაწლავების და მუცლის ღრუს ორგანოების ანთებითი დაავადებებით,დამავრებული ლიმფომებით და მეზენტერიალური ჯირკვლების ტუბერკულოზით.
აუცილებელია ზუსტი დიაგნოზის დასმა, რაც შეიძლება სწრაფად

Posted by: lizofobi 15 Jan 2010, 11:48
ლიმფოციტების მომატებას რა იწვევს?

Posted by: miriani777 15 Jan 2010, 12:48
ეს დღეებია მიჭირს კუჭში გასვლა და მოშარდვაც მიძნელდება, ანუ დიდხანს ველოდები სანამ შარდი წამოვა, ვსვავ ფსიქოტროპულ წამლებს ამის ბრალი ხომ არაა?

Posted by: lala 16 Jan 2010, 11:27
იცით რა მაინტერესებს? გამაყუჩებელი ნემსი და სანთელი ერთნაირად მოქმედებს ცუდად კუჭზე/>? თუ სათელი ნაკლებად?
მიპასუხეთ რა მეჩქარება

Posted by: Bella Moretta9 17 Jan 2010, 18:20
ხომ ვერ მეტყვით, რამ შეიძლება გამოიწვიოს მუდმივი შეგრძნება, თითქოს ლუკმა გულზე გადგას (აი ძალიან გვიან არ გიჭამიათ ბევრი? მაგ დროს რომ იცის ხოლმე?

Posted by: Nnino 17 Jan 2010, 19:14
ახალი თემა რომ გამეხსნა, მაინც რაღაცას შეუერთებდნენ და იქნებ აქ იყოს ამ შეკითხვის ადგილი:

დანიშნული მქონდა რაციოცეფი კუნთში, პირველივე ნემსზე კანი ამოიბურცა და გაწითლდა, ცოტა ხანში სიწითლე ალაგდა, მაგრამ სიმაგრე და სილურჯე დარჩა. ახლა არ მტკივა, მაგრამ გალურჯებულია და "პადოშივით" გამაგრებული. გავაგრძელეთ მეორე კუნთზე გაკეთება. მე-4 ნემსზე ანალოგიური მოხდა იქაც.

მოკლედ, ამ სიმაგრეების დასაშლელი ეფექტური საშუალება არაფერი იცით? იოდის კლეტკები გავიკეთე უკვე და კომბოსტოები მადევს. ასე თუ დარჩა, ცხოვრებაში ნემსს ვეღარ გავიკეთებ, არადა, დამჭირდება ჩემი ჭირკოტიანი თავის ამბავი რომ ვიცი... smile.gif

მადლობა წინასწარ დიდი...

miriani1987
ექიმი არ ვარ, მაგრამ მაგისი ბრალია, კი... ისე, ექიმს უნდა გაეფრთხილებინე ამის თაობაზე წესით.

Posted by: Solveig 17 Jan 2010, 20:31
Nnino
QUOTE
სიმაგრე და სილურჯე დარჩა. ახლა არ მტკივა, მაგრამ გალურჯებულია და "პადოშივით" გამაგრებული

მეც მქონდა ეგ და გაქრება თავისით.

მე კანი ამაჭრეს ხელიდან და ასე 2 თვე გამყვა შედეგი. ჯერ ოდნავი ტკივილის შეგრძნება და სიმაგრე მქონდა. მერე ტკივილი გაქრა, მაგრამ ამობურცულობა კიდევ დიდხანს ისინჯებოდა.

Posted by: n10n10-87 18 Jan 2010, 15:37
მაინტერესებს ვარუსულ ტერფს თუ შეუძლია ტერფის ზურგის და კოჭ-წვივის სახსრის შეშუპების განვითარება?..

Posted by: Examiner 18 Jan 2010, 17:54
გაციებული ვიყავი ერთი კვირა, თითქოს გამიარა მაგრამ ისევ მახსენებს თავს, რა ვქნა რა ვჭამო რა დავლიო?

Posted by: n10n10-87 18 Jan 2010, 21:31
QUOTE
გაციებული ვიყავი ერთი კვირა, თითქოს გამიარა მაგრამ ისევ მახსენებს თავს, რა ვქნა რა ვჭამო რა დავლიო?

რას გულისხმობ ისევ მახსენებს თავსო?.. რა გაწუხებს გააჩნია...

Posted by: Nnino 18 Jan 2010, 22:38
Solveig
მადლობა... 2kiss.gif

Posted by: texasuri jleta benzoxerxit 24 Jan 2010, 18:53
QUOTE (miriani1987 @ 15 Jan 2010, 12:48 )
ეს დღეებია მიჭირს კუჭში გასვლა და მოშარდვაც მიძნელდება, ანუ დიდხანს ველოდები სანამ შარდი წამოვა, ვსვავ ფსიქოტროპულ წამლებს ამის ბრალი ხომ არაა?

კი, ტრიციკლურმა ანტიდეპრესანტებმა (მაგალითად ამიტრიპტილინი,მელიპრამინი,ანაფრანილი და ა.შ.) და ნეიროლეფსიურმა პრეპარატებმა შეიძლება მსგავსი გვერდითი ეფექტი მოგცეს,მათი ე.წ. ანტიქოლინერგული (ატროპინისმსგავსი) ეფექტების გამო.
გაიარე კონსულტაცია შენ ექიმთან,შეიძლება შეცვალოს დანიშნულება, შეიძლება ეს გვერდითი მოვლენები თავისითაც შემცირდეს და გაქრეს.

Posted by: Gooduna 25 Jan 2010, 02:52
2 თვეა ყოველ დღე საღამოს 7-დან 10 საათამდე სიცხე მაქვს 37.2 - 37.5 არანაირი ტკივილები, ან რამე ჩივილები... 6 ექიმთან ვიყავი უკვე ამ ორი თვის მანძილზე... რენდგენიც გადავიღე ანალიზებიც გავიკეთე მაგრამ არც არაფერი არ მჭირს...თუ გაგიგიათ ასეთი რამე ხალხო?

Posted by: dianeee 25 Jan 2010, 17:30
გეხვეწებით, მირჩიეთ კარგი თერაპევტი, სად მივიდე? აი, ეხლავე, დღესვე მინდა მივიდე, 6 ის მერე თავისუფალი ვარ სამსახურიდადნ. ავერსის კლინიკაში ვფიქრობ და ღირს, რომ მივიდე? იქნებ რამე უკეთესი იცით? უმორჩილესად გთხოვთ, მირჩიეთ რა სასწრაფოაააა sad.gif

Posted by: Achmed the dead terrorist 25 Jan 2010, 18:03
უკაცრავად, მაინტერესებს დაკაწვრით (ფრჩხილით) და კბენით რაიმე სქესობრივი დაავადების აკიდების რამდენად დიდი შანსია ?

Posted by: miriani777 26 Jan 2010, 16:09
ჩავიტარე კტ კვლევა, და კეროვანი დაზიანებები არ აღმომაჩნდა, უბრალოდ მარჯვენა პარკუჭი ზომიერად დილატირებულიაო, ეს რა ნიშნავს? გაჟონვა ხომ არ მომივა? სულ არა სასიამოვნო შეგრძნება მაქვს ხოლმე მარჯვენა თავის ქალაში და მტკივა ხოლმე თვალის ბუდიანად

Posted by: Eka1967 26 Jan 2010, 16:18
ხალხნო რისი ბრალია ხელები რომ გიბუჟდება და დაკარგული გაქვს მგრძნობელობა მსუბუქი საგნების მიმართ. (ნემსი , ფურცელი 67წლის ასაკში)

Posted by: miriani777 26 Jan 2010, 21:27
ჩავიტარე კტ კვლევა, და კეროვანი დაზიანებები არ აღმომაჩნდა, უბრალოდ მარჯვენა პარკუჭი ზომიერად დილატირებულიაო, ეს რა ნიშნავს? გაჟონვა ხომ არ მომივა? სულ არა სასიამოვნო შეგრძნება მაქვს ხოლმე მარჯვენა თავის ქალაში და მტკივა ხოლმე თვალის ბუდიანად

Posted by: nino_sh 27 Jan 2010, 15:24
ლამისაა 3 წელია ვსვავ უამრავ წამას, მათ შორის ჰორმონალურს (უშვილობას ვმკურნალობ), ვგრძნობ რომ ორგანიზმი მაქვს საშინლად გამოფიტული, რაც გამოიხატება მუდმივად მოთენთილობაში, დაღლილობაში, უხასიათობაში, მხედველობის დაქვეითებაში, ადვილად ვცივდები და ადვილად ვიწამლები. ხომ ვერ მირჩევდით ვიტამინების რომელი კომპლექსი სჯობს რომ დავლიო პროფილაქტიკისთვის რომ ორგანიზმი მოვაძლიერო??? წინასწარ მადლობები

Posted by: ia_Baggins 30 Jan 2010, 11:47
ძალინ გთხოვთ ვინმე ექიმმა მომწერეთ რა პმ


კითხვა მაქვს

Posted by: ia_Baggins 30 Jan 2010, 13:15
ძალიან გთხოვთ ვინმემ მომწერეთ რა პმ

Posted by: miriani777 2 Feb 2010, 11:39
ჩავიტარე კტ კვლევა, და კეროვანი დაზიანებები არ აღმომაჩნდა, უბრალოდ მარჯვენა პარკუჭი ზომიერად დილატირებულიაო, ეს რა ნიშნავს? გაჟონვა ხომ არ მომივა? სულ არა სასიამოვნო შეგრძნება მაქვს ხოლმე მარჯვენა თავის ქალაში და მტკივა ხოლმე თვალის ბუდიანად


Posted by: Breaky 2 Feb 2010, 16:18
6 წლის ბავშვის სისხლის საერთო ანალიზის პასუხში წერია:

აღინიშნება შეფარდებითი ნეიტროპენია, ერითროციტებში - ჰიპოქრომია, მიკრო - მაკროანიზოციტოზი (2+), ლიმფოციტებში სჭრაბობს ფართოციტოპლაზმიანი ფორმები.

მაჩვენებლები, რომლებიც ნორმის გარეთაა:
ერითროციტის საშუალო მოცულობა - 69.4 (ნორმა 72-88)
ჰემოგლობინის საშ. შემცველობა ერითროციტში - 22.4 (ნორმა 23-31)
ერიტროციტისგანაწილების ფართი - 22.8 (ნორმა 12-16)

ლეიკოციტალური ფორმულა
პლაზმური - 1 (ნორმა 0-0.5)

დანარჩენი მაჩვენებლები ასე თუ ისე ნორმის ფარგლებშია.


ავიღო 2 აგური??? სანამ ექიმს ვნახავ, ვნერვიულობ user.gif

Posted by: Solveig 2 Feb 2010, 22:31
Breaky
რამდენადაც მახსოვს, ხანგრძლივად მიმდინარე ან ახლადგადატანილი ვირუსული ინფექციის მაჩვენებელია ეგ-ვგულიხსმობ ფართოციტოპლაზმიან ლიმფოციტებს და პლაზმურ უჯრედებს.
და ანემიაცაა. რკინადეფიციტურს გავს (ანიზოციტოზი, ჰიპოქრომია, მომატებული განაწილების ფართი).

QUOTE
ავიღო 2 აგური???

no.gif

Posted by: Breaky 2 Feb 2010, 22:34
Solveig
ველაპარაკე ექიმს და იგივე მითხრა.
ანემია არ უთქვამს ოღონდ, თან ჰემოგლობინიც ნორმაშია, მართალია, ქვედა ზღვართან, მაგრამ ჯერ არაფერი არ გინდათო.
2კვირაში გადაამოწმეთ და მერე ვნახოთო.

Posted by: laluka18 2 Feb 2010, 22:39
მახველებს საშინლად, 2 ნაირი წამალი დავლიე მარა მაინც არაფერი გაციებულიც ვარ და იმდენ წამლებს ვსვავ კუჭი გამიღიზიანდა უკვე, ამ ხველებამ კიდე მომკლა, რას მირჩევთ? sad.gif

Posted by: Solveig 2 Feb 2010, 23:44
Breaky
იმედია, აღარ ფიქრობ აგურებზე smile.gif

ყველაფერი კარგად იქნება.

Posted by: miriani777 3 Feb 2010, 01:55
იქნებ ჩემო პოსტი ნახოთ smile.gif.......... დიდი მადლობა წინასწარ

Posted by: Masked 3 Feb 2010, 03:46
QUOTE (Breaky @ 2 Feb 2010, 22:34 )
Solveig
ველაპარაკე ექიმს და იგივე მითხრა.
ანემია არ უთქვამს ოღონდ, თან ჰემოგლობინიც ნორმაშია, მართალია, ქვედა ზღვართან, მაგრამ ჯერ არაფერი არ გინდათო.
2კვირაში გადაამოწმეთ და მერე ვნახოთო.

გამოცვალეტ ექიმი, როგორ უნდა დალოდება 14 დღე?
mad.gif mad.gif mad.gif
რა სისულელე არის, არ არის კარგი ბეფუნდ ეს რაც დაწერეტ!

როგორ არის ნორმა ჰემოგლობინ?
ბავშვს აქვს ანემიი და ნოიტროპენიი, ლოიკოპენიის ფორმა.
და ამას აქვს ენ რაოდენობა მიზეზი.

ნახეტ ჰäმატოლოგე.

უნდა გადამოწმდეს სასწრაფოტ კველა მიზეზი რატომ არის ეს!.
დაწკებული კნოხენმარკიდან და დამთავრებული რკინა, ელენთა კუჩი, ფარის. ჯირკვალი უნდ უნდ უნდ.
და ტუ ექიმა ეს არ იცის, დაგიწერტ პმ რა უნდა გადამოწმდეს.
აუცილებელი არ არის ცუდი რამე იკოს...
რამე ინფფეკტ ქონდა ბოლო დროს ბავშვს? გრიპპალე ინფფეკტ.


Posted by: LUKA-BRAZI 3 Feb 2010, 09:49
აქ არის სისხლის ნორმალური შემადგენლობა ბავშვებში, ასაკის მიხედვით:

user posted image


Posted by: keen 3 Feb 2010, 10:24
Masked
QUOTE
გამოცვალეტ ექიმი, როგორ უნდა დალოდება 14 დღე? mad.gif mad.gif mad.gif

ჩემს შვილსაც ქონდა ცვლილებები სისხლის საერთო ანალიზში ვირუსული ინფექციის მერე, ორ საუკეთესო ექიმს ვაჩვენე და არაა ეგ პრობლემაო, დამახასიათებელია ამ შემთხვევისთვისო smile.gif

QUOTE
რამე ინფფეკტ ქონდა ბოლო დროს ბავშვს? გრიპპალე ინფფეკტ.

შემოვა ალბათ Breaky და გიპასუხებთ. რამდენადაც ვიცი, ვირუსული ინფექციის მერე ბრონქიტიც ქონდა მის შვილს yes.gif

Posted by: miriani777 3 Feb 2010, 10:34
ყველაფერი კარგად იქნება უფლის წყალობით

Posted by: keen 3 Feb 2010, 10:40
miriani1987
QUOTE
იქნებ ჩემო პოსტი ნახოთ smile.gif.......... დიდი მადლობა წინასწარ

ვინ უნდა ნახოს? აქ ფაქტობრივად აღარ შემოდიან თერაპევტები და მათ შორის თემის ავტორიც yes.gif უკეთესია, რჩევა ექიმს კითხო და თუ მაინც ფორუმიდან გინდა პასუხის გაგება, ამ თემის ავტორს კითხე პმ-ში, არ მგონია, პასუხს დაგზარდეს smile.gif

Posted by: Solveig 3 Feb 2010, 11:11
Masked
QUOTE
ნოიტროპენიი, ლოიკოპენიის ფორმა.
და ამას აქვს ენ რაოდენობა მიზეზი.

როგორც ჩანს, ბავშვს აქვს შეფარდებითი ნეიტროპენია ლიმფოციტების მომატების ხარჯზე (და ასევე, ციტოტოქსიკური ლიმფოციტები). ალბათ ექიმმა მხედველობაში მიიღო, რომ მან საკმაოდ რთულად გადაიტანა გრიპი ცოტა ხნის წინ და ამიტომ გასცა მშობელს მაგგვარი პასუხი.

Posted by: patriciia 3 Feb 2010, 17:42
miriani1987
QUOTE
ჩავიტარე კტ კვლევა, და კეროვანი დაზიანებები არ აღმომაჩნდა, უბრალოდ მარჯვენა პარკუჭი ზომიერად დილატირებულიაო, ეს რა ნიშნავს? გაჟონვა ხომ არ მომივა? სულ არა სასიამოვნო შეგრძნება მაქვს ხოლმე მარჯვენა თავის ქალაში და მტკივა ხოლმე თვალის ბუდიანად

ზუსტად ეგეთი რამ აწუხებს ჩემს ვაჟს, გარდა ამისა, იმავე მხარეს, სადაც თVალი აწუხებს, მუხლის სისუსტე. 17წლის ხდება მალე
მივიყვანე თVალის ექიმთან, ორი დღე იქ დავაღამეთ, აღმოუჩინეს ასტეგმატიზმი (მამამისს და მის ძმას აქვს),

მარჯვენა თვალში მხედველობა 60% , მარცხენა თვალში _ 40%,
მარცხენა გუგა ოვალური ფორმის, მაგრამ რის გამოც მივედით, არც მოგვისმინა, ბოლოს, როგორც იქნა, ვუთხარი, მეორე დღისთვის დაგვიბარა, მივედით, ჩახედა და ნევრალგიააო......ახლა ვურეკავ ნევროლოგს გამწარებული და ვერ მივკვლიე, არც სამსახურშია და არც მობილზე პასუხობს..
მოკლედ, მე თვითონაც ნევრალგიაზე ვიეჭვე, მაგრამ ჯერ მაინც ოფთალმოლოგს ვაჩვენე
ეს ბუნდოვანი მზერა, თVალის ბუდის ტკივილი არის ნევრალგიური, რასაც იწვევს
1) დაბადებიდანვე სუსტი სისხლძარღვები (ჩემი შვილი ჰიპოსქიით დაიბადა)
2) გაზით გამთბარ ოთახში დიდხანს ჯდომა (ცეო მაინც გამოიყოფა, თუმცა კარმა გვიდგას და ყოველ წელიწადს მოგვყავს ხელოსანი, რომელიც ამოწმებს, ამ წელიწადს გავაწმენდინეთ თავიდან ბოლომდე)
3) კომპზე თამაში ღამის სამ საათამდე, ცუდი ძილი
4) ზოგადად უმოძრაობა, ფიზიკური კულტურის უქონლობა
5) ზემოთჩამოთვლილიდან გამომდინარე, ჟანგბადით ცუდი მომარაგება ზოგადად ორგანიზმის და განსაკუთრებით ტვინის.......

გარდა ამისა, შენი პოსტები სხვაგანაც მინახავს, ძაან უცნაურ შეგრძნებებს აღწერ, ჩემს შვილსაც ეგეთი აზრები აწუხებს,
out of body experience-ები და რამე, როგორც იციან ხოლმე თინეიჯერებმა.......
თუ ვინმე შემოვა და საბჭითა ნევროპათოლოგიისგან განსხვავებულ, პროგრესულ აზრს მოგვაწოდებს, ძალიან მადლობელი დავრჩები, ალბათ შენც..........

Posted by: kriketi-1985 3 Feb 2010, 22:28
გადაკუზულ მდგომარეობაში დიდხანს ვერ ვძლებ, რომ ვსწორდები, წელი საშინლად მტკივა, რა მჭირს, ექიმო? biggrin.gif დიდი მადლობა მას, ვინც მიპასუხებს, იმედია სწორად მოვხვდი biggrin.gif

Posted by: laluka18 4 Feb 2010, 22:19
პოზა შეცვალეთ biggrin.gif
.................

Posted by: kriketi-1985 4 Feb 2010, 23:11
QUOTE (laluka18 @ 4 Feb 2010, 22:19 )
პოზა შეცვალეთ biggrin.gif
.................

ჭურჭელს წამოწოლილი ხომ ვერ გავრეცხავ wink.gif tongue.gif

Posted by: miriani777 5 Feb 2010, 13:49
patriciia
გაიხარე დიდი მადლობა ამომწურავი პასუხისათვის

Posted by: shota_xuc 6 Feb 2010, 12:21
გამარჯობათ, შეუძლია ვინმეს მითხრას, რომალ კლინიკაში მუშაობს დერმატოლოგი იოსებ კობახიძე. წინასწარ დიდი მადლობა

Posted by: makauka 7 Feb 2010, 20:05
გამარჯობათ .საშინელი თავისტკვილი მაწუხებს 1 თვეა უკვე გაუჩერებლად 24/7. ღამეც რო მეღვიძება იგივეს ტკივილს ვგრძნობ ,მაგრამ მთლად ტკივილი არ მაქვს ,ცოტა რთული ასახსნელია ,მაგრამ ვეცდები ავხსნა ,მოკლედ.შეგრძნება მაქვს თითქოს ტვინი "მიყანყალებს" ვიბრაციას ვგრძნობ ქალის შიგნით და არის გაუჩერებელი ხან ისეტი სიძლიერით რომ კონცენტრირება მიჭირს ,ტავბრუს ხვევას და გულის რევას ვგძნობ ,(ვიკვებები ჩემთვის ნორმალურად ანუ არაპერი შემიცვლია) .ამავდროულად თვალებზე მაწვება და ხან ისეტი შეგრძნება მაქვს თითქოს ქვიშიტ მაქვს თვალები სავსე.მხედველობა ორივე ტვალზე 100% მაქვს გასინჯული ვარ რამოდენიმე თვის წინ ,თერაპევტთან ვაპირებ წასვლას ამ დღეებში ,მაგრამ მანმადე იქნებ რამე მირჩიოთ .ტკივლიგამაყუჩებელი საერთოდ არ მშველის , ვცადე.

Posted by: patriciia 9 Feb 2010, 12:32
გადავუღე ჩემს შვილს მაგნიტურ-რეზონანსული ტომოგრაფია, ტვინი აბსოლუტურად საღი აქვს, არავითარი სამწვერა ნერვი და ეგეთები.......არც თვალ-ბუდე, მოკლედ, აბსოლუტურად ჯანსაღი ტვინი აქვს....ყველაფერი რაც სჭირდა, ფუნქციონალური დარღვევა აღმოჩნდა........
კვირას ვიყავი ძეობაზე ნათესავთან, უფროსი ბIჭი 17 წლის ჰყავთ. დღე და ღამე კომპთან აქვს გასწორებული ჩემი შვილივით, ისეთ მდგომაეობაშია ტიპი, ჩემს შვილს იმის ნახევარიც კი არ აწუხებს: ორივე თვალს შაში-ბეშივით აკვარკვალებს, მეტყველება გაძნელებული აქვს, სახე ატკრეცილი, გარტყმაში არაა რა ხდება დედამიწაზე, მე-11 კლასშია და გარკვევით ვერ ამბობს რა პროფესიის არჩევა და საერთოდ რა უნდა ცხოვრებაში.......
მოკლედ, დავღუპე შვილი ამ ეროვნული გამოცდებით და ბოლო წლებში სკოლიდან გაყვანით, მართალია შაშკინი, ვეტანხმები 100%-ით, აკადემიურიც რომ ავაღებინო ან პირველივე წელს ვერ ჩააბაროს, არ დავეძებ, იაროს სკოლაში და 10 გამოცდა ჩააბაროს........

დასკვნა: კომპიუტერის ბოროტად გამოყენება არ შეიძლება.......
ესაა ძირითადად თავის ტკივილების და სხვა ჩივილების მიზეზი

Posted by: teoj 10 Feb 2010, 06:30
მარცხენა ხელი მიბრუჟდება , უკვე მალ მალე ეს ბოლო დღეებია.. პროტრომბინის ბრალი იქნება? rolleyes.gif

Posted by: DoNxN1ka 10 Feb 2010, 09:45
არვიცი სად უნდა დამესვა ეს კითხვა და გავრისკავ აქ დავსვამ user.gif

მოფსმის სამედიცინო ტერმინს ხომ ვერ მეტყოდით? mo.gif


Posted by: LUKA-BRAZI 10 Feb 2010, 14:40
QUOTE (makauka @ 7 Feb 2010, 20:05 )
გამარჯობათ .საშინელი თავისტკვილი მაწუხებს 1 თვეა უკვე გაუჩერებლად 24/7. ღამეც რო მეღვიძება იგივეს ტკივილს ვგრძნობ ,მაგრამ მთლად ტკივილი არ მაქვს ,ცოტა რთული ასახსნელია ,მაგრამ ვეცდები ავხსნა ,მოკლედ.შეგრძნება მაქვს თითქოს ტვინი "მიყანყალებს" ვიბრაციას ვგრძნობ ქალის შიგნით და არის გაუჩერებელი ხან ისეტი სიძლიერით რომ კონცენტრირება მიჭირს ,ტავბრუს ხვევას და გულის რევას ვგძნობ ,(ვიკვებები ჩემთვის ნორმალურად ანუ არაპერი შემიცვლია) .ამავდროულად თვალებზე მაწვება და ხან ისეტი შეგრძნება მაქვს თითქოს ქვიშიტ მაქვს თვალები სავსე.მხედველობა ორივე ტვალზე 100% მაქვს გასინჯული ვარ რამოდენიმე თვის წინ ,თერაპევტთან ვაპირებ წასვლას ამ დღეებში ,მაგრამ მანმადე იქნებ რამე მირჩიოთ .ტკივლიგამაყუჩებელი საერთოდ არ მშველის , ვცადე.

მე ვერაფერ კონკრეტულს ვერ გირცევ, გარდა იმისა რომ ექიმთან უნდა მიხვიდე აუცილებლად!!! ნევროპათოლოგთან ან თერაპევტი იყოს თუ გინდა, მაგრამ მაინც მიდი აუცილებლად!

Posted by: teoj 11 Feb 2010, 11:22
QUOTE
მარცხენა ხელი მიბრუჟდება , უკვე მალ მალე ეს ბოლო დღეებია.. პროტრომბინის ბრალი იქნება?

უკვე ფეხიც rolleyes.gif
* * *
QUOTE (teoj @ 11 Feb 2010, 11:22 )
QUOTE
მარცხენა ხელი მიბრუჟდება , უკვე მალ მალე ეს ბოლო დღეებია.. პროტრომბინის ბრალი იქნება?

უკვე ფეხიც rolleyes.gif

გარდავიცვალე, გმადლობთ rolleyes.gif

Posted by: dadn 12 Feb 2010, 18:24
QUOTE (teoj @ 11 Feb 2010, 11:22 )


გარდავიცვალე, გმადლობთ  rolleyes.gif

lol.gif ar vici tore getkodi rames

Posted by: keen 12 Feb 2010, 18:55
LUKA-BRAZI
ისე ამ თემის არსებობის არსს ვერ ვხდები, პასუხის გამცემი ფაქტობრივად არავინაა და ხალხი კი ამაოდ ელის smile.gif

Posted by: LUKA-BRAZI 12 Feb 2010, 19:26
keen
მასეთი რამე ნებისმიერ თემაზე შეიძლება თქვა... ფორუმი საავადმყოფო ხომ არ არის? smile.gif

teoj
QUOTE
QUOTE
მარცხენა ხელი მიბრუჟდება , უკვე მალ მალე ეს ბოლო დღეებია.. პროტრომბინის ბრალი იქნება?

უკვე ფეხიც rolleyes.gif
* * *
QUOTE (teoj @ 11 Feb 2010, 11:22 )
QUOTE
მარცხენა ხელი მიბრუჟდება , უკვე მალ მალე ეს ბოლო დღეებია.. პროტრომბინის ბრალი იქნება?

უკვე ფეხიც rolleyes.gif

გარდავიცვალე, გმადლობთ rolleyes.gif


და აქ პოსტვას ექიმთან მისულიყავი არ ჯობდა? smile.gif აქ ვინ მოგცემთ რაიმეს გარანტიას, რომ სწორს გეტყვიან ან არასწორს? კი შემოდის რამდენიმე ექიმი, მაგრამ ბევრიც ისეთია, ძალიან შორს რომ დგას მედიცინისგან.....

http://forum.ge/?f=43&showtopic=34075354

Posted by: keen 12 Feb 2010, 19:34
QUOTE
keen მასეთი რამე ნებისმიერ თემაზე შეიძლება თქვა... ფორუმი საავადმყოფო ხომ არ არის? smile.gif

ეგ გასაგებია, მაგრამ სხვა ანალოგიურ თემებში შედიან შესაბამისი პროფილის ექიმები, ნუ რამდენად სანდოა, ეგ მეორე საკითხია

აქ რომელია აბა თერაპევტი და ბოლოს როდის დაპოსტა? smile.gif

Posted by: teoj 12 Feb 2010, 20:16
LUKA-BRAZI
QUOTE
და აქ პოსტვას ექიმთან მისულიყავი არ ჯობდა?

რა თქმა უნდა. თქვენ მხოლოდ უნდა შეგეშინებინეთ, სანამ გარდავიცვლებოდი, იქნებ მიმესწრო rolleyes.gif




dadn
QUOTE
ar vici tore getkodi rames

სამაგიეროდ, სიმართლით წავედი 2kiss.gif

Posted by: LUKA-BRAZI 12 Feb 2010, 20:50
teoj
smile.gif

keen
რა ვქნა? სად ვიშოვო თერაპევტი ფორუმისთვის? შენზე მეტად მე მინდა რომ ექიმები მრავლად იყვნენ აქ..... user.gif

Posted by: keen 13 Feb 2010, 14:57
QUOTE
რა ვქნა? სად ვიშოვო თერაპევტი ფორუმისთვის? შენზე მეტად მე მინდა რომ ექიმები მრავლად იყვნენ აქ..... user.gif

რა პროფილის ექიმებიც არიან, შესაბამისი საკონსულტაციო თემები კი არის და ვინც არაა -არა, ამას ვამბობ მხოლოდ. კი გვყავს ფორუმელი თერაპევტი vano_t, მაგრამ რატომღაც მიატოვა ეს განყოფილება... ალბათ მოუცლელობის გამო user.gif

Posted by: LUKA-BRAZI 13 Feb 2010, 16:10
QUOTE
vano_t

user.gif

ვანო, აკლიხარ აქაურობას yes.gif

Posted by: dadn 14 Feb 2010, 15:24
ღმერთმა დაგიფაროთ და Thalassämie (minor)-თან შეხება ხომ არავის გქონიათ?გერმანიაში სისხლის შემოწმების დროს შემთხვევით აღმომაჩნდა,დამეტებითი გამოკვლევის მერე არანაირი მკურნალობა არ გინდაო (ვერც გიშველისო) რადგან გენეტიკურიაოო,ხოდა მაინტერესებს იკნებ რაგაცა მაინც არსებობს ქართული მამაპაპური ცოტა რომ წამშველებოდა?? ისე რაარი ცხოვრება დადიხარ ,ჭამ-სვავ და ...

Posted by: miriani777 14 Feb 2010, 21:02
makauka
ტომოგრამა გადაიღეთ

Posted by: natiagogona 15 Feb 2010, 12:16
ჩემს მეგობარს 8 თვეა ახველებს და ეს პერიოდია სულ მკურნალობს, ექიმები ეუბნევიან რომ ყელში აქვს ბაქტერია. დაახლოებით ერთი თვის წინ გაუკეტეს ანალიზი და აღარ არისო მაგრამ ისევ ახველებდა, კიდევ გაუკეთეს ანალიზი და იყო. ასე ახსნეს რომ ზოგჯერ არ ჩნდება ანალიზშიო, მაინტერესებს შეიზლება ასეთი რამ მოხდეს? ან ვინმეს თუ გქონიათ?

Posted by: asharashi 16 Feb 2010, 14:23
იქნებ აქ მაინც....


შეიძლება თუ არა რომ ჰორმონალური დისბალანსმა გამოიწვიოს ალერგიისმაგვარი სიმპტომები...ან არსებობს ალერგია ჰორმონების წარბობით ან ნაკლებობით გამოწვეული? ჰა?

Posted by: inesa1936 19 Feb 2010, 15:34
ყველაფრის მეშინია: სიცოცხლის, სიკვდილის , დღევანდელი დღის, ხვალინდელის...მოკლედ, გავაფრინე - ყველაფრის. თან ვფიქრობ ,შეიძლება ასეთი შიშის ქვეშ ცხოვრება?გარეთ_ არაფერი მეტყობა, შეიძლება ბევრი გაცინოთ კიდევაც სხვადასხვა თემებზე, მაგრამ ეს ,,ფობიები" ვერ მივიშორე! ყველა ექიმის მეშინია!ადრე სხვებს რომ დავცინოდი- ყველას კრეტინიზმში გავასწარი !კარგის გაფიქრების მეშინია. არა, მაქვს კიდევაც მიზეზი- სრულიად ჯანმრთელი მამა, უცებ გარდამეცვალა... და იმის შემდეგ, ჭკუაზე არ ვარ! სულ შიშშში ვარ! მაინც ვფიქრო,ბრომ კორეგირებადი ვარ და ამიტომ არანაირ ტრანკვილიზატორს არ ვიღებ, მაგრამ ვინ იცის ,ფსიქიატრიულის კონტინგენტიც დასაწყისში ასე ფიქრობდა?!, ალბათ მქირდება : ნევროლოგ-ფსიქიატრ-ფსიქოლოგი, მაგრამ სად არის ასეთი უნივერსალი?წავედი სხვა შიშებიც მაქვს მისახედი.

Posted by: LUKA-BRAZI 19 Feb 2010, 16:10
inesa1936
QUOTE
ალბათ მქირდება : ნევროლოგ-ფსიქიატრ-ფსიქოლოგი,

და ენდოკრინოლოგიც yes.gif

Posted by: Solveig 19 Feb 2010, 18:17
dadn
QUOTE
ღმერთმა დაგიფაროთ და Thalassämie (minor)-თან შეხება ხომ არავის გქონიათ?გერმანიაში სისხლის შემოწმების დროს შემთხვევით აღმომაჩნდა,დამეტებითი გამოკვლევის მერე არანაირი მკურნალობა არ გინდაო (ვერც გიშველისო) რადგან გენეტიკურიაოო,ხოდა მაინტერესებს იკნებ რაგაცა მაინც არსებობს ქართული მამაპაპური ცოტა რომ წამშველებოდა?? ისე რაარი ცხოვრება დადიხარ ,ჭამ-სვავ და ...

არა, არ არსებობს მაგის არანაირი მკურნალობა. მძიმე შემთხვევებში ერითროციტების მასას უსხამენ ხოლმე, რომ ჰემოგლობინის უკმარისობა და აქედან გამოწვეული ანემია დააკომპენსირონ..

თალასემია მინორის დროს ჰემოგლობინის ცილაში მარტო ერთი ბეტა-ჯაჭვია შეცვლილი (მარტო 1 ალელშია მუტაცია) და ამის გამო ანემია მსუბუქი ფორმითაა. საშიშიც არაფერია..ერთი ეგაა, რომ მემკვიდრეობით გადადის. სხვა არაფერი.

Posted by: dadn 19 Feb 2010, 18:57
Solveig
აქაც იგივეს მეუბნებიან, მაგრამ თქვენმა პასუხმა უფრო დამაწყნარა . დიდი მადლობა პასუხისთვის.

Posted by: miriani777 23 Feb 2010, 15:24
Solveig
ჭკვიან..................................................................................................

Posted by: სოფი 23 Feb 2010, 21:24
გამარჯობა.
დაბალი სიცხეები მაქვს-37-37,3 კვირაზე მეტია. ყელისგან არ არის, ფილტვები სუფთაა. რენტგენი გავიკეთე დღეს და არაფერიაო. ერთადერთი რაც მითხრეს-შეხორცება მაქვს პლევრასა და დიაფრაგმას შორის. (იქნებ ესეც განმიმარტოთ რას ნიშნავს).
თირკმელებიც მტკივა, რა სიმპტომები აქვს თირკმელების ანთებას?

Posted by: LUKA-BRAZI 23 Feb 2010, 21:48
სოფი
QUOTE
დაბალი სიცხეები მაქვს-37-37,3 კვირაზე მეტია.

http://forum.ge/?f=43&showtopic=34064843

QUOTE
შეხორცება მაქვს პლევრასა და დიაფრაგმას შორის.

არ ვიცი, შეიძლება ანთება გქონდა გადატანილი.... პლევრიტი ან პლევრო-პნევმონია. გახველებდა? ჭვალი გქონდა გვერდებში?

QUOTE
თირკმელებიც მტკივა, რა სიმპტომები აქვს თირკმელების ანთებას?

თირკმლის ანთებას ყველაზე კარგად სისხლის და შარდის ანალიზები გამოავლენს yes.gif

Posted by: სოფი 24 Feb 2010, 01:05
LUKA-BRAZI
QUOTE
შეიძლება ანთება გქონდა გადატანილი.... პლევრიტი ან პლევრო-პნევმონია. გახველებდა? ჭვალი გქონდა გვერდებში?

არა ექიმო, ეხლა მახველებს და ჭვალიც ეხლა მაქვს ცოტა user.gif
მადლობა პასუხისთვის

Posted by: LUKA-BRAZI 24 Feb 2010, 15:36
სოფი
ექიმი არ ვარ..... ჯერ smile.gif)
შარდის ანალიზი მაინც გაიკეთე თუ ტკივილი წელის მიდამოში გაქვს (თირკმლების საპროექციო არეში)... და ზემოთ რომ ლინკი მიგითითე, მაგასაც გადახედე...

Posted by: სოფი 24 Feb 2010, 18:22
LUKA-BRAZI
QUOTE
ზემოთ რომ ლინკი მიგითითე, მაგასაც გადახედე...

გადავხედე უკვე და გასკდა გული user.gif
მადლობა დიდი smile.gif

Posted by: miriani777 26 Feb 2010, 19:42
სოფი
სოფი მეც მასეთი სიცხეები მაქვს, დღეს საერთო გავიკეთე სისხლის, და არაფერი გაქვსო, გემოგლობინი მქონდა ოდნავ დაბალი, სუსტად ვარ, ფილტვები სუფთა მაქვს არ მახველებს, უბრალოდ გაბრუებული ვარ ეს დრეებია, და თავში სიმზძიმეს ვგრძნობ.

Posted by: tatooo 27 Feb 2010, 20:36
თEრაპევტებთან გამომგზავნეს ენდოკრინოლოგებმა biggrin.gif ბრმა ნაწლავთან დაკავშირებით. მარჯვენა ფეხი მიჩერდებასავით ხანდახან ნუ არ ვიცი მიჩერდერბა თუ აი რაღაცნაირად მაქვს და ხანდახან ნაღვლის ბუშტი მტკივა ოდნავ თვითონ ბრმა ნაწლავი არ მტკენია ჯერჯერობით და გადაცემითი ტკივილია თუ რა არის? ამიხსენით რა ვინმემ ბრმა ნაწლავის ტკივილები როგორია და ოპერაცია აუცილებელია? ისე რომ განიკურნოს არ შეიძლება?

Posted by: prince_meo 28 Feb 2010, 19:05
37.5 სიცხე მაქვს და ცხვირი მაგარს მატყუებს ... და მგონი კიდე მიწევს .... პარაცეტამოლი კი დავლიე მაგრამ რავიციი ... რა ვქნა დაწოლის გარდა ?biggrin.gif

Posted by: miriani777 28 Feb 2010, 19:06
prince_meo
თავის ტკივილები ან სიმძიმეები თავში?

Posted by: prince_meo 28 Feb 2010, 19:10
miriani1987
ეხა არა მარა ისე საერთოდ რო გავირბენ ან დავიგლები სწრაფად შუბლში და ცოტა უკან საშნელი ტკივილი მაქ ხოლმე წვასავით და იქნებ მითხრა რაარის ? მადლობა წინასწარ smile.gif

Posted by: miriani777 28 Feb 2010, 19:18
prince_meo
შიიგნიდან გრძნობთ მაგას?

Posted by: prince_meo 28 Feb 2010, 19:18
miriani1987
კი და რაც უფრო დავიგლები მით უფრო დიდი ხანი და მწარედ sad.gif
* * *
miriani1987
და კიდე ეხა სახე მიხურს თავზე სიმძიმეს ვგრძნობ

Posted by: miriani777 28 Feb 2010, 19:44
რამდენი წლის ხართ?.........................................................

Posted by: prince_meo 28 Feb 2010, 19:47
miriani1987
17 biggrin.gif თ ზედმეტია tongue.gif
* * *
და კიდე პატარაობაში სიცხე რო მქონდა ყოველთვის ყელის ტკივილი მქონდა ხოლმე ანგინა რა ... და ეხა კიდე არა ... რავი მითხარით რა რამე biggrin.gif

Posted by: miriani777 28 Feb 2010, 20:45
არანაირი პრობლემა არ გაქვს, ვფიქრობ სიგარეტს ეწევი, და ბევრს ნერვიულობ smile.gif

Posted by: prince_meo 28 Feb 2010, 20:53
miriani1987
= ))) არა არ ვეწევი ... 1 წელი გახდება მალე რაც არ ვეწევი ... კარგად კი ვუბერავდი ადრე biggrin.gif და ნერვულობით კიდე შეიძლება ცოტა სუსტი ნერვები მაქ და lol.gif

Posted by: miriani777 1 Mar 2010, 11:10
prince_meo
გასაგებია.................... არაფერი არაა დამშვიდდი და მსუბუქი დამაწყნარებელი მიიღე ხოლმე, ვალერიანი ან რამე მსგავსი

Posted by: kriketi-1985 3 Mar 2010, 12:24
მწოლიარე ავადმყოფს ეწყება კანკალი, შემცივნება და შემდეგ უწევს სიცხე 39მდე,ვუკეთებთ დამწევებს, მერე ეწყება ოფლიანობა, რისი ბრალი შეიძლება იყოს?
თბილისში კარგი ინფექციონისტის კოორდინატები მიმასწავლეთ მწოლიარე ავადმყოფისთვის, თუ იცით, გმადლობთ smile.gif

Posted by: LUKA-BRAZI 3 Mar 2010, 12:31
kriketi-1985
QUOTE
მწოლიარე ავადმყოფს ეწყება კანკალი, შემცივნება და შემდეგ უწევს სიცხე 39მდე,ვუკეთებთ დამწევებს, მერე ეწყება ოფლიანობა, რისი ბრალი შეიძლება იყოს?

ეგ სერიოზულია! 39 გრადუსამდე ტემპერატუსრის მატება, შემცივნებით, სასწრაფოდ საჭიროებს ყურადღების მიქცევას!

QUOTE
თბილისში კარგი ინფექციონისტის კოორდინატები მიმასწავლეთ მწოლიარე ავადმყოფისთვის, თუ იცით, გმადლობთ

ეს არ ვიცი სამწუხაროდ....

Posted by: tevdore 3 Mar 2010, 16:53
ღვიძლის ფუნქციურ სინჯს სად აკეთებენ ნორმალურად?
გული რომ არ გამიხეთქონ.. smile.gif
იუნონაში აკეთებენ თუ იცით.........

Posted by: kriketi-1985 3 Mar 2010, 16:58
ტრაქეოსტომის შეძენა მინდა და სად უნდა ვნახო? არ ვიცი სწორად ვკითხულობ აქ თუ არა, მაგრამ მჭირდება sad.gif
გმადლობთ

Posted by: LUKA-BRAZI 3 Mar 2010, 17:44
kriketi-1985
აბა აქ ნახე, თუ არადა აფთიაქში იკითხე, შეიძლება მათ იცოდნენ სად იშოვება ეგეთი სამედიცინო მასალები..
http://www.ivermedi.ge/index.php?&sl=GE

Posted by: Cousteau 4 Mar 2010, 09:50
რამოდენიმე დღეა მარჯვენა მხარეს მტკივა ის ადგილი რომელიც მონიშნულია სურათში

user posted image

დამეწყო უცებ, ტრავმა არ მქონია, არ მქონა ზედა სასუნთქი გზების ინფექცია, სიცხე არ მაქვს, შეშუპებული არ მაქვს (ან შეიძლება მაქვს მაგრამ ძალიან ოდნავ), თავიდან პალპაციით ეს არე ოდნავ მტკივნეული იყო, ეხლა თითქმის აღარ მტკივა (თითქმის) მაგრამ მტკივა ყბის დაკეტვისას (მაგრად რომ მოვუჭირო კბილებს). სტომატოლოგიური პრობლემები არ მაქვს და ყბაყურა გადატანილი მაქვს ბავშვობაში.

იდეები არის რამე?


Posted by: LUKA-BRAZI 4 Mar 2010, 10:48
Cousteau
მეც მტკიოდა მარჯვენა მხარეს ქვედა ყბის ქვემოთ პალპაციით უფრო მტკივნეული იყო... რამდენიმე დღე, მერე გაიარა. მასე ხან რა ამტკივდებ ხან რა და ცოტა ხანში გაივლის ხოლმე smile.gif

Posted by: SamaraT 4 Mar 2010, 19:23
ზურგისთის (ხერხემლისთვის),
უფრო სწორად სახსრებისთვის რომელია უკეთესი წამალი,მაზი,
"ფასტუმ გელ" ისნაირი რამე, ოღონდ კარგი, რო შველოდეს.
მკურნალობისთვის რაა კარგი, ასაკი 60 წელი.

Posted by: lolita55 4 Mar 2010, 20:07
გამარჯობათ,

შეკრულობა რისი ბრალი შეიძლება იყოს?
როცა ადამიანი კვირაში ერთხელ გადის მხოლოდ კუჭში?
რა უნდა ქნას?

ექიმმა თქვა ჯერ ახალგაზრდაა, და ამიტომ სპეციალურ წამლებს ვერ დავინიშნავ,
კვებით უნდა მოაგვაროს თვითონ ორგანიზმმაო,
და არ გვარდება თვითონ და როდემდე იყოს ასე?
რამე სპეციალური დიეტა ხომ არ არსებობს რაც ხელს შეუწყობდა კუჭში გასვლას?
ან მკურნალობა ან რამე სხვა საშველი?



Posted by: izabel 5 Mar 2010, 20:32
lolita55

სულ წამლებზე ხომ არ იქნება მერე? მშრალი საკვები უნდა შეზღუდოს მაქსიმალურად...მიიღოს წვნიანები... ძალიან კარგია შავი ქლიავის ჩირი...ჭარხალი...



მაინტერესებს აქ ექიმი თუ პასუხობს...რაღაც კითხვა მაქვს...

Posted by: paradise_seeker 6 Mar 2010, 16:19
ინტერნისტი რა არის არ ვიცი დანარჩენები ვიცი & ამიტომ აქ ვსვამ კიტთხას ( რომ თუ რამეა გულუბრყვილო ოFტოპიკში ჩამეთვალოს)
ahm..
იდაყვთან მაქვს ტრავმა- დარტყმული უხეშ ზედაპირზე რამაც გამოიწვია დაკაწრვაც, სისხლდენაც, კანის ადღლეზაც & დალურჯებაც.

ბავშვობაში ასეთი ტრავმები სულ მქონდა ფეხებზე & მიჩნდებოდა შემდგომ რაღაც დამცავი გარსი (რომელსაც მერე წვალებით & ტირილით , მაგრამ მაინც ვიგლეჯდი biggrin.gif) მგონი მიხვდით რაზეცაა საუბარი..

ხოდა უკვე ბევრი დრო გავიდა ამ ბოლო ტრავმიდან , მაგრამ ეგ რაღაცა არ გამიჩნდა.. უბრალოდ ვარდისფერია & რაღაც ადგილებში წითელი სისხლის წერტილებივით, სისხლდენ არ მაქვს.. ხოდა რამეს რომ ედება მწარეა, მინდა დავიკრო რამე & დედაჩემი მიკრძალავს & რა ვქნა??? თავისით გამაგრდებაო & ვერ გამაგრდება იდაყვის ადგილია & ეგ კანი სულ მოძრაობაშია , ალბათ მაგიტო. ხო?

კიდე დაველოდო როდის გამაგრდება გარედან იმ ყავისფერი რაღაცით თუ დავიკრო რამე??

Posted by: merry 6 Mar 2010, 23:54
აზრზე არ ვარ სად უნდა დავსვა ეს შეკითხვა და ან გადამამისამართეთ ან მიპასუხეთ user.gif

პენსიონერისთვის ოპერაციის ხარჯების ანაზღაურება როგორ ხდება ვინმემ იცის? ვის უნდა მივმართოთ, რა საბუთებია საჭირო, რა პროცედურაა და ა.შ. baby.gif

წინასწარ მადლობა!

პ.ს. გულს ეხება მაშდამე, ე.წ. pacemaker უნდა ჩაიდგას user.gif baby.gif

Posted by: khatia88 7 Mar 2010, 19:56
არ ვიცოდი ეს კითხვა სად დამესვა და იქნებ მიპასუხოთ რა. რას ნიშნავს ჭიპიდან უეცრად სითხის წამოსვლა, რომელსაც აქვს არასასიამოვნო სუნი? და როცა სპირტს ისმევს ეწვის.

Posted by: donvaso 8 Mar 2010, 18:37
რამე ისეთი მირჩიეთ გრიპის დასამარცხებლად რაც არ ვიცი... sad.gif
არაყი+ნიორიდან დაწყებული ჰომეოპათიური წამლებით დამთავრებული ყველაფერი ნაცადი მაქვს....
სითხე მიღებული მაქვს 2 ლიტრამდე, ვსვამ ტერაფლუს(ცხელ წყალში გასახსნელს), და ყელის ტკივილისთვის სტრეპსილს, თუმცა ჯერჯერობით უშედეგოდ sad.gif
თვალებს ვერ ვახელ, სურდო მომდის ლიტრობით, ყელი მტკივა, თავი სკდება.....

მიშველეეეეეეეეეეეეეეეეეეთ

sad.gif weep.gif war.gif vis.gif help.gif fingal.gif mad.gif

Posted by: Solveig 8 Mar 2010, 21:51
donvaso
QUOTE
რამე ისეთი მირჩიეთ გრიპის დასამარცხებლად რაც არ ვიცი...
არაყი+ნიორიდან დაწყებული ჰომეოპათიური წამლებით დამთავრებული ყველაფერი ნაცადი მაქვს....
სითხე მიღებული მაქვს 2 ლიტრამდე, ვსვამ ტერაფლუს(ცხელ წყალში გასახსნელს), და ყელის ტკივილისთვის სტრეპსილს, თუმცა ჯერჯერობით უშედეგოდ
თვალებს ვერ ვახელ, სურდო მომდის ლიტრობით, ყელი მტკივა, თავი სკდება.....

ეჰ, მაგ ოხერი სანამ თავისას არ ამოიჭამს, მანამ არ მოგეშვება sad.gif ეგრე ვიყავი მეც ამ ცოტა ხნის წინ...3-4 დღეში გადაივლის >38 სიცხე და მერე ნელ-ნელა ალაგდება ყველაფერი-ჯერ ყელის ტკივილი, მერე ხველა და მერე სურდო...


Posted by: izabel 8 Mar 2010, 22:02
წელს ქვევით დაბუჟებული ვარ და ფეხებშიც სიმძიმეს ვგრძნობ. წნევას შეიძლება დაბუჟება გამოეწვია??/

Posted by: donvaso 8 Mar 2010, 23:05
QUOTE (Solveig @ 8 Mar 2010, 21:51 )
donvaso
QUOTE
რამე ისეთი მირჩიეთ გრიპის დასამარცხებლად რაც არ ვიცი...
არაყი+ნიორიდან დაწყებული ჰომეოპათიური წამლებით დამთავრებული ყველაფერი ნაცადი მაქვს....
სითხე მიღებული მაქვს 2 ლიტრამდე, ვსვამ ტერაფლუს(ცხელ წყალში გასახსნელს), და ყელის ტკივილისთვის სტრეპსილს, თუმცა ჯერჯერობით უშედეგოდ
თვალებს ვერ ვახელ, სურდო მომდის ლიტრობით, ყელი მტკივა, თავი სკდება.....

ეჰ, მაგ ოხერი სანამ თავისას არ ამოიჭამს, მანამ არ მოგეშვება sad.gif ეგრე ვიყავი მეც ამ ცოტა ხნის წინ...3-4 დღეში გადაივლის >38 სიცხე და მერე ნელ-ნელა ალაგდება ყველაფერი-ჯერ ყელის ტკივილი, მერე ხველა და მერე სურდო...

3 დღე ვგრძნობდი რომ ვირუსი ორგანიზმში ფეხს იკიდებდა , მხოლოდ ყელი მტკიოდა გუშინ უკვე სურდოც დაემატა, დღეს კიდევ ყველაფერი ერთად ძალიან მწვავე ფორმებში. სიცხე არც გავიზომე(რა აზრი აქვს რო), ნუ, 38 ზე ზევით თუ ავიდა მერე სხვანაირად მივხედავ....




cry.gif

Posted by: Solveig 9 Mar 2010, 02:04
donvaso
QUOTE
3 დღე ვგრძნობდი რომ ვირუსი ორგანიზმში ფეხს იკიდებდა , მხოლოდ ყელი მტკიოდა გუშინ უკვე სურდოც დაემატა, დღეს კიდევ ყველაფერი ერთად ძალიან მწვავე ფორმებში. სიცხე არც გავიზომე(რა აზრი აქვს რო), ნუ, 38 ზე ზევით თუ ავიდა მერე სხვანაირად მივხედავ....

ზუსტად ეგრე ვიყავი მეც.

თავიდან ყელის ტკივილით და დაბალი სიცხით დაიწყო. მე-4 თუ მე-5 დღეს მომცა 38 და ხველა დამაწყებინა..2 დგის განმავლობაში მქონდა ეგრე, მერე 37-მდე დაიწია, ყელმა გამიარა, მაგრამ ძალიან მახველებდა (განსაკუთრებით, ღამით) და სურდო მქონდა სასწაული.
სითხეების და ცხვირის შესასუნთქის გარდა არაფერი არ გამომიყენებია, 38-ის დროს დავლიე პარაცეტამოლი..სადღაც 10 დღე გაგრძელდა საერთო ჯამში.

Posted by: donvaso 9 Mar 2010, 18:51
Solveig
დღეს შედარებით უკეთ ვარ, ყელი არ მტკივა, უბრალოდ მახველებს(მშრალად), სურდოც საღამოს უკვე არ მაქვს, ხოდა მოკლედ უკეთ ვარ რა...
არაფერი ჯობია ჯანმრთელობას რა.... biggrin.gif biggrin.gif

Posted by: SamaraT 9 Mar 2010, 19:50
ზურგისთის (ხერხემლისთვის),
უფრო სწორად სახსრებისთვის რომელია უკეთესი წამალი,მაზი,
"ფასტუმ გელ" ისნაირი რამე, ოღონდ კარგი, რო შველოდეს.
მკურნალობისთვის რაა კარგი, ასაკი 60 წელი.

Posted by: Xato38 12 Mar 2010, 08:42
გამარჯობათ .... მოკლედ მე ასეთი რაგაც მჩირს...როცა ვდიეტობ მეწკება გამონაკარი მკერდთან ..შემდეგ სახეზე..და კურის უკან..ეს ხდება დაახლოებით , როგორც 5 კგ..ს მოვიკლებ..ამ დროს ვსვამ კლარიტინს და მშველის რამოდენიმე დგით.... დიეტის დროს მნშვნელოა არ აქვს რა საკვებს ვიგებ /// + ვიტამინებსაც ვიგებ, მაგრამ არაპერი არ მშველის....ერტი პერიოდი ვიკვლევდი ინპექცურში, მაგრამ უშედეგოდ...იქნებ ვინმეს მსგავსი გსმენიათ... ....... როგორც კი ვცკვეტ დიეტას მილაგდება კველაფერი.... გთხოვთ მირჩიოთ.

Posted by: vano_t 12 Mar 2010, 12:06
Xato38
QUOTE
გამარჯობათ .... მოკლედ მე ასეთი რაგაც მჩირს...როცა ვდიეტობ მეწკება გამონაკარი მკერდთან ..შემდეგ სახეზე..და კურის უკან..ეს ხდება დაახლოებით , როგორც 5 კგ..ს მოვიკლებ..ამ დროს ვსვამ კლარიტინს და მშველის რამოდენიმე დგით.... დიეტის დროს მნშვნელოა არ აქვს რა საკვებს ვიგებ /// + ვიტამინებსაც ვიგებ, მაგრამ არაპერი არ მშველის....ერტი პერიოდი ვიკვლევდი ინპექცურში, მაგრამ უშედეგოდ...იქნებ ვინმეს მსგავსი გსმენიათ... ....... როგორც კი ვცკვეტ დიეტას მილაგდება კველაფერი....  გთხოვთ  მირჩიოთ.

აბა სინჯე წონის დაკლება გარკვეული დიეტის გარეშე, მაგალითად ვარჯიშები+ნაკლები ჭამა, მაგრამ ჩვეულებრივი დიეტა. თუ მაინც დაგემართა, მაშინ შეიძლება წონის კლებასთან იყოს კავშირში. თუ აღარ დაგემართა, მაშინ შეიძლება გაკრვეული საკვები ელემენტების დეფიციტის გამო იყოს.

Posted by: keen 12 Mar 2010, 12:18
vano_t
bis.gif ეს უკვე პროგრესია, თუნდაც ერთი პოსტი...

Posted by: Xato38 12 Mar 2010, 12:39
vano_t მადლობა რჩევისთვის....
yes.gif yes.gif yes.gif

Posted by: ia_Baggins 13 Mar 2010, 11:38
ესეთი რაღაც მჭირს, ძირითადად ნერწყვის გადაყლაპვის შემდეგ ვეღარ ამოვისუნქებ ხოლმე მეკვრება სუნთქვა და ვიგუდები

თებერვლის დასაწყისში დამეწყო, გლანდები მაწუხებდა და მაგის ბრალი მეგონა
სადღაც 18 დღე გავიდა რაც გლანდები ამოვიჭერი და ამ გუდვებმა არ გამიარა

რა შეიძლება რო იყოს?

შეიძლება რო ჩიყვი მქონდეს?

Posted by: vano_t 14 Mar 2010, 11:15
ia_Baggins
QUOTE
ესეთი რაღაც მჭირს, ძირითადად  ნერწყვის გადაყლაპვის შემდეგ ვეღარ ამოვისუნქებ ხოლმე მეკვრება სუნთქვა და ვიგუდები

თებერვლის დასაწყისში დამეწყო, გლანდები მაწუხებდა და მაგის ბრალი მეგონა
სადღაც 18 დღე გავიდა რაც გლანდები ამოვიჭერი და ამ გუდვებმა არ გამიარა


QUOTE
შეიძლება რო ჩიყვი მქონდეს?
არა მგონია. ჩეიყვი ძალიან დიდი უნდა იყოს, რაიმე პრობლემა რომ გამოიწვიოს ყლაპვასთან ან სუნთქვასთან დაკავშირებით (იმდენად დიდი, რომ საკუთარი ხელითაც შეიძლება მოსინჯო ალბათ და თვალითაც დაინახო).

QUOTE
რა შეიძლება რო იყოს?
ამისათვის ექიმმა უნდა გამოგიკვლიოს. ინტერნეტით ამის თქმა ძნელია. მინიმუმ საჭიროა პირის ღრუს და ყელის კარგი დათვალიერება ამისათვის (შეიძლება ხორხის და ფილტვების მოსმენაც). ასევე ექიმმა უნდა დაგაკვირდეს ყლაპვის მომენტში. ამის შემდეგ შეიძლება რაიმეს თქმა. სხვანაირად შეუძლებელია.

SamaraT
QUOTE
ზურგისთის (ხერხემლისთვის),
უფრო სწორად სახსრებისთვის რომელია უკეთესი წამალი,მაზი,
"ფასტუმ გელ" ისნაირი რამე, ოღონდ კარგი, რო შველოდეს.
მკურნალობისთვის რაა კარგი, ასაკი 60 წელი.
უკეთესი წამალი ამ ქვეყანაზე არ არსებობს. არსებობს წამლები. ერთისთვის რაც უკეთესია, მეორესათვის შეიძლება სასიკვდილო იყოს საერთოდ. უკეთესი წამალის გაგებისათვის, ავადმყოფმა უნდა მიიღოს წამალი და თუ წამალს ეფექტი გააჩნია და ამასთანავე უარყოფითი მოვლენები არ გააჩნია (ან მინიმალურად აქვს გამოხატული), მაშინ ასეთი წამალი არის უკეთესი მოცემული ავადმყოფისათვის. ისე, ტკვილის დიაგნოზირება უნდა-ანუ გარკვევა რა არის ტკვილის მიზეზი. თუმცა ზურგის ტკვილების უმრავლესობა პრობლემას არ წარმოადგენს და ჭირდება მხოლოდ ტკივილის მკურნალობა. ამისათვის არსებობს სხვადასხვა ჯგუფის აბები, მისაკრავი წამლები და მაზები. გარდა ამისა, შეიძლება სითბოს გამოყენებაც, რომელიც ხშირად ამცირებს ზურგის ტკივილს. ასევე ზურგისათვის გარკვეული ტიპის ვარჯიშებიც კარგია (ამას თუმცა განსაზღვრავს ფიზიკური თერაპიის სპეციალისტი).

თუ ტკვილი აუტანელი არ არის და მოთმენა შეიძლება, მაშინ ყველაზე უკეთესია ტკივილთან ერთად ცხოვრება-ამას არცერთი წამალი არ შეედრება, რაოდენ უცნაურადაც არ უნდა ჟღერდეს.

Posted by: ia_Baggins 14 Mar 2010, 16:57
vano_t
მადლობა პასუხისთვის smile.gif

რამე კონკრეტული სპეციალიზაციის ექიმთან უნდა მივიდე თუ თერაპევტთან?

Posted by: earth 14 Mar 2010, 20:22
გამარჯობათ
მაინტერესებს 2წლის წინ თუ ადამიანი ნარკოტიკების მომხმარებელი იყო დღეს რა ანალიზით შემიძლია ამის შემოწმება?

მადლობა წინასწარ

Posted by: konkretula 14 Mar 2010, 22:11
ესეიგი რა მაინტერესებს : )


მქონდა დაბალი სიცე, საერთოდ სულ ყელი პრობლემა მაქვს... მაგრამ ექიმი რო გამოვიძახე მიტხრა რომ მაქვს ღორის გრიპი : ) გრიპი გაქვს და უკვე ანალიზს აზრი არ აქვს, რაც კი გრიპია ყველა ღორისააო, 5 დღე ვსვავდი ამ ტამეFლუს : )

და მართლლა მარტო ღორის გრიპია??

Posted by: Masked 14 Mar 2010, 22:25
QUOTE (konkretula @ 14 Mar 2010, 22:11 )
ესეიგი რა მაინტერესებს : )


მქონდა დაბალი სიცე, საერთოდ სულ ყელი პრობლემა მაქვს... მაგრამ ექიმი რო გამოვიძახე მიტხრა რომ მაქვს ღორის გრიპი : ) გრიპი გაქვს და უკვე ანალიზს აზრი არ აქვს, რაც კი გრიპია ყველა ღორისააო, 5 დღე ვსვავდი ამ ტამეFლუს : )

და მართლლა მარტო ღორის გრიპია??

არა გაქვს შენ "ღორისგრიპპი" და მოხსენი ეს "შხამი" ტამიფლუ. yes.gif

შენი Dრ. mad.gif

Posted by: anuka2009 14 Mar 2010, 23:53
გამარჯობა
ღამე მიბუჟდება ხელები და ფეხები, მარვიძებს გაბუჟების შეგრძნება. ვის უნდა მივმართო? იქნებ მიპასუხოთ. წინასწარ გიხდით მადლობას

Posted by: vano_t 15 Mar 2010, 00:09
konkretula
QUOTE
ესეიგი რა მაინტერესებს : )


მქონდა დაბალი სიცე, საერთოდ სულ ყელი პრობლემა მაქვს... მაგრამ ექიმი რო გამოვიძახე მიტხრა რომ მაქვს ღორის გრიპი : ) გრიპი გაქვს და უკვე ანალიზს აზრი არ აქვს, რაც კი გრიპია ყველა ღორისააო, 5 დღე ვსვავდი ამ ტამეFლუს : )

და მართლლა მარტო ღორის გრიპია??

შეიძლება, შეიძლება არა. ღორის გრიპის დიაგნოზი კლინიკურად (ავადმყოფის სიმპტომების და ფიზიკური გამოკვლევის მიხედვით) შეუძლებელია. ღორის გრიპი არაფრით არ განსხვავდება სხვა გრიპებისაგან საერთოდ. გარდა ამისა, გრიპებისათვის (ყველანაირი გრიპისათვის), როგორც წესი, დამახასიათებელია მაღალი სიცხეები, თავის ტკივლი, მთელი სხეულის "მტრევა" და ტკვილიები (ინგლისურად ამის ეძახიან body aches), ხველა. თუ იმ ექიმმა სპეციალური ტესტი არ ჩაატარა ღორის გრიპზე, მაშინ ვერანაირად ვერ დასვამს მაგ დიაგნოზს. სპეციალურ ტესტს გამოძახებული ექიმი სახლში ვერ ჩაატარებს. აშშ-ში, ჩრდილო დაკოტის შტატში, ამ ტესტს ატარებს ცენტრალური ლაბორატორია და ისიც მაშინ როცა ავადმყოფს გააჩნია ზემოჩამოთვლილი სიმპტომები, პლიუს ყელის ტკივილი.

ტამიფლუს დალევას არ გირჩევ საერთოდ. არამგონია მაგ წამალს რაიმე ღირებულება ქონდეს (განსაკუთრებით გრიპის მარტივი ფორმების დროს).

earth
QUOTE
გამარჯობათ
მაინტერესებს 2წლის წინ თუ ადამიანი ნარკოტიკების მომხმარებელი იყო დღეს რა ანალიზით შემიძლია ამის შემოწმება?
სისხლის ტესტით ვერცერთი ტესტი ვერ დაადგენს ასეთ რამეს. ყველაზე დიდი ხანი ორგანიზმში რჩება მარიხუანა და ისიც 4-6 კვირის შემდეგ შეუძლებელია მისი დადგენა სისხლში. ყველაზე დიდი ხანი ნარკოტიკული ნივთიერება რჩება თმაში და როგორც ვიცი შეუძლიათ დაადგინონ ასეთი ნივთიერების მიღება თმის ანალიზით 6 თვის განმავლობაში. 2 წელი არ გამიგია.

ia_Baggins
QUOTE
რამე კონკრეტული სპეციალიზაციის ექიმთან უნდა მივიდე თუ თერაპევტთან?
წესით თერაპევტმა უნდა დაიწყოს დიაგნოზის დასმაც და მკურნალობაც. თუ თერაპევტი ვერ სვამს დიაგნოზს, ან სვამს დიაგნოზს მაგრამ მკურნალობას შედეგი არ აქვს, მაშინ აგზავნის სპეციალისტთან იმის მიხედვით თუ რა არის ტკივილის მიზეზი.

anuka2009
QUOTE
ღამე მიბუჟდება ხელები და ფეხები, მარვიძებს გაბუჟების შეგრძნება. ვის უნდა მივმართო? იქნებ მიპასუხოთ. წინასწარ გიხდით მადლობას
მსგავსი კითხვა დასვა ia_Baggins-მა. ყველანაირი პრობლემის გამკვლევა უნდა იწყებოდეს თერაპევტის მიერ და თერაპევტმა უნდა გადაწყვიტოს რა სახის სპეციალიტს უნდა მიმართოს ავადმყოფმა, იმის მიხედვით თუ რა შეიძლება იყოს სავარაუდო დიაგნოზები.

Posted by: keen 15 Mar 2010, 10:16
vano_t
QUOTE
ტამიფლუს დალევას არ გირჩევ საერთოდ. არამგონია მაგ წამალს რაიმე ღირებულება ქონდეს (განსაკუთრებით გრიპის მარტივი ფორმების დროს).

აბა, ამ კონკრეტულ შემთხვევაში, რეალური ექიმის დანიშნულებას უფრო აქვს ფასი თუ გაცილებით კვალიფიციური მედიკოსის ვირტუალურ რჩევას? smile.gif

Posted by: konkretula 16 Mar 2010, 00:11
vano_t
დავლიე უკვე...
ესეიგი, თავიდან მტელი მკერდი და ზურგი მტკიოდა, ალბათ შემცივდა. მერე დაბალი სიცხე მომცა და ექიმმა რო გამსინჯა ყელზეც მქონდა პრობლემა მაგრამ ატომღაც მაინც ღორის გრიპი დამისვა დიაგნოზი და ტამეფლუ დამალევინა. : )

ამის ფონზე : D 4 თვის წინ გულის ოპერაცია გავიკეთე, და ვითომ ტამეფლუს დანიშვნით მოსალოდნელი საფრთხისაგან დაიზღვია თავი (?) და ტან დარწმუნებით თქვა რო ღორის გრიპი მაქვს :|

და მე ისიც არ ვიციი, ამ ოპერაციის მერე მაგ ტამეფლუს მიღება თუ შიეძლება biggrin.gif
კარდიოლოგს ვკითხე და იმან, რასაც ეგ დაგინიშნავს დალიეო biggrin.gif არადა სადაც გავიკეთე ოპერაცია, მიტხრეს კარდიოლოგის ტანხმობის გარეშე არაფერი დალიოო biggrin.gif ყველამ დამიკიდა ხო? : D

Posted by: vano_t 16 Mar 2010, 19:14
konkretula
QUOTE
და მე ისიც არ ვიციი, ამ ოპერაციის მერე მაგ ტამეფლუს მიღება თუ შიეძლება biggrin.gif
კარდიოლოგს ვკითხე და იმან, რასაც ეგ დაგინიშნავს დალიეო biggrin.gif არადა სადაც გავიკეთე ოპერაცია, მიტხრეს კარდიოლოგის ტანხმობის გარეშე არაფერი დალიოო biggrin.gif ყველამ დამიკიდა ხო? : D

არა, ტამიფლუს დალევა გულით ავადმყოფობის დროს არ არის უკუჩვენება. საერთოდ გულით და ფილტვებით დაავადებულებს უფრო ჭირდებათ თანამედროვე მედიცინის მიდგომით. შენ შემთხვევაშიც ალბათ სწორი გითხრა შენმა ექიმმა. მე ზოგადად არ ვურჩევ ავადმყოფს დალიოს ტამიფლუ. მაგრამ, ექიმების უმეტესობა თანამედროვე რეკომენდაციების მიხედვით ურჩევენ ავადმყოფს დალიოს ტამიფლუ ყველანაირი გრიპის დროს როცა ავადმყოფს გულის და ფილტვების დაავადებები აქვთ.

Posted by: texasuri jleta benzoxerxit 16 Mar 2010, 19:16
konkretula
ძალიან კარგი პრეპარატია ტამიფლუ. გადასარევად გადააქვთ ავადმყოფებს (მათ შორის ბავშვებს,ასევე მოხუცებს),მინიმალური გვერდითი ეფექტებით. ტამიფლუს ზედმეტი გაპიარება,ისევე როგორც რაღაც მისტიკური რანგის შიშის დანერგვა მის მიმართ ფარკმაკოლოგიური ომის შედეგია სხვა და სხვა ფირმებს შორის... უბრალოდ მისი რემანტადინივით პროფილაქტიკურად მიღების (გრიპის სეზონის დროს)არ გვირჩევენ, ვირუსმა რეზისტენტობა რომ არ გამოიმუშავოს ამ მედიკამენტის მიმართ. თუმცა პროფილაქტიკური მიღება რეკომენდებულია დადასტურებული ღორის გრიპით ავადმყოფთან კონტაქტის შემდეგ 10 დღე.
პირადად მე რამდენიმე ასეთი პაციენტის გასინჯვა მომიწია და შესაბამისად ორჯერ ჩავიტარე ტამიფლუს პროფილაქტიკური 10 დღიანი კურსი. (ორივეს პაბორატორიულად დაუდასტურდათ H1N1)
ბებიას კი ყოველგვარი ლაბ.ანალიზის გარეშე, ეგრევე დავუნიშნე. იმთავითვე მაღალი სიცხით,გულისრევა-ღებინებით,და მარჯვენამხრივი მწვავე პნევმონიით დაეწყო. 84 წლის არის. მშვენივრად გადაიტანა. boli.gif თუმცა შეიძლება არც ქონდა H1N1 biggrin.gif მაგრამ ტამიფლუ სხვა შტამების მიმართაც აქტიურია,თუმცა მხოლოდ ღორის გრიპისთვის არის არჩევის პრეპარატი. ან შეიძლება H1N1 ქონდა ბებიას და ტამიფლუს გარეშეც განკურნებულიყო. biggrin.gif ყველაფერი ხდება მედიცინაში. მაგრამ თავი დავიზღვიე და მიმაჩნია,რომ სწორად მოვიქეცი yes.gif boli.gif

Posted by: Tornike Alashvili 16 Mar 2010, 19:54
vano_t
QUOTE
მე ზოგადად არ ვურჩევ ავადმყოფს დალიოს ტამიფლუ

+1
გაუმარჯოს იმ ექიმებს, რუმლებიც თავისი თავით ფიქრობენ
და არა როგორც ფარმაკომაფიას უნდა
ფასობს ჰედლაინები კი არა
ადამიანი !

Posted by: Natuka NGN 16 Mar 2010, 19:59
texasuri jleta benzoxerxit
QUOTE
ძალიან კარგი პრეპარატია ტამიფლუ. გადასარევად გადააქვთ ავადმყოფებს

აი, ახლა ცრუობ... ბავშვს ბოდვა დააწყბინა და მაშინვე მოვუხსენით. არ მინდა დეტალებში ჩაკირკიტება. სორედაც რომ რომელირაც კომპანიამ დიდი ფული მოტეხა და მსოფლიო ჯანდაცვის ორგანიზაციამ ამ თღორის გრიპებიტა და ტამი-ფლის რეკლამით.

Posted by: TETISA 17 Mar 2010, 01:17
თხევად აზოტს რა უკუჩვენებები აქვს კანზე გამოსაყენებლად?

Posted by: vano_t 17 Mar 2010, 05:46
TETISA
QUOTE
თხევად აზოტს რა უკუჩვენებები აქვს კანზე გამოსაყენებლად?

აბსოლუტურ უკუჩვენებას წარმოადგენს მისი გამოყენება თვალთან ახლოს (ანუ, თუ სამკურნალო ადგილი თვალთან ახლოს არის, არ უნდა გამოიყენო, რადგანაც თვალზე თუ მოხვდა სიბრმავის შანსი დიდია). აბსოლიტური უკეჩვენება ნიშნავს აბსოლუტურ უარს.

შედარებითი უკეჩვენებაა სიცივისადმი მგრძნობელობის დაქვეითება (cold intolerance), სიცივისადმი "ალერგია" (cold urticaria), კრიოგლობულინებმია, რეინოს დაავადება/სინდრომი. შეფარდებითი უკუჩვენების დროს ცდილობ არ გააკეთო დანიშნული მკურნალობა, მაგრამ გარკვეულ შემთხვევებში შესაძლებელია გაკეთება.

Posted by: TETISA 17 Mar 2010, 22:33
vano_t
ისე მოხდა, რომ ძაან მცირე ნაკადი თვალთან ახლოს მომხვდა, მაგრამ მხედველობაზე არანაირი გავლენა არ მოუხდენია. რამდენიმე დღეა გასული და კარგად ვარ.

რაც შეეხება შედარებით უკუჩვენებებს, არც ამათი სიმპტომები მაქვს. ეს ნიშნავს, რომ ჩემთვის არ აქვს ამას უკუჩვენება?

უბრალოდ ექიმს ხელი აუცდა თუ რა მოუვიდა არ ვიცი, მაგრამ საღ ადგილას დამასხა და მიზანს ააცდინა. ხოდა მაგ უბანში კანს ხომ არ გამიფუჭებს?

Posted by: vano_t 17 Mar 2010, 22:45
TETISA
QUOTE
რაც შეეხება შედარებით უკუჩვენებებს, არც ამათი სიმპტომები მაქვს. ეს ნიშნავს, რომ ჩემთვის არ აქვს ამას უკუჩვენება?
კი, ეგ ნიშნავს, რომ არ აქვს უკეჩვენება შენთვის.

QUOTE
უბრალოდ ექიმს ხელი აუცდა თუ რა მოუვიდა არ ვიცი, მაგრამ საღ ადგილას დამასხა და მიზანს ააცდინა. ხოდა მაგ უბანში კანს ხომ არ გამიფუჭებს?
არა მგონია. საერთოდ დაბალი ტემპერატურა იწვევს ჯერ ანთებას, მერე ბუშტუკის წარმოქმნას ექსპოზიციის ადგილას და მერე ბუშტუკის შიგთავსი ხმება (ან გასკდომის შემდეგ, ან გაუსკდომლად). იმ ადგილას ცოტა ხანი შეიძლება იყოს პიგმენტის ნაკლებობა. მე თვითონ ბევრჯერ მიხმარია თხევადი აზოტი და ყოველთვის მესხმება ხელზე მცირე ნაწილი. ჯერ ჯერობით არ დავზიანებულვარ.

Posted by: Solveig 18 Mar 2010, 01:18
vano_t
საერთოდ, რისთვის იყენებენ თხევად აზოტს?


Posted by: Nunziatina 18 Mar 2010, 01:21
QUOTE
საერთოდ, რისთვის იყენებენ თხევად აზოტს?

მაცივრებში. yes.gif

Posted by: Solveig 18 Mar 2010, 01:25
Nunziatina
QUOTE
მაცივრებში.

გმადლობთ, ზოგადად რისთვის იყენებენ, ეგ მეც ვიცი....მაგალითად, ჩვენ უჯრედებს ვინახავთ შიგ.

კითხვას ადრესატი ჰყავდა მგონი, იქნებ გვეცლია პასუხი? smile.gif


Posted by: Nunziatina 18 Mar 2010, 01:52
Solveig
QUOTE
კითხვას ადრესატი ჰყავდა მგონი, იქნებ გვეცლია პასუხი?

კი, მეც მაინტერესებს.

Posted by: vano_t 18 Mar 2010, 06:31
Solveig
QUOTE
საერთოდ, რისთვის იყენებენ თხევად აზოტს?

კანი სხვადასხვა lesion-ისთვის (lesion-ის ზუსტი შესატყვისი არ ვიცი ქართულად). მაგალითად კანის სიმსივნეებითვის (squamous cell carcinoma of the skin, basal cell carcinoma of the skin) გარდა მელანომებისა. მელანომას ჭირდება ამოკვეთა აუცილებლად. კედევ aktinic keratosis, seborheic keratosis, მეჭეჭის, skin tag-ის დროს (მე მგონი ხორცმეტს ეძახიან ამას). ეს მე რაც გამომიყენებია იმათ ვასახელებ. ყველაზე ხშირად მეჭეჭების დროს გამომიყენებია. კიდევ შეიძლება კანის სხვა წარმონაქმნებისათვის.

Posted by: TETISA 18 Mar 2010, 13:30
vano_t
გასაგებია.
მადლობა smile.gif

Posted by: MissFrance 18 Mar 2010, 22:10
ან დამამშვიდეთ, ან რამე მიშველეთ.... sad.gif 3 თვეა სიცხე მაქვს 37. 37.1. ბოლო დროს თავბრუ მეხვევა. ფილტვები კი არა, თუ რამე ორგანო არსებობს ყველაფერი გამოვიკვლიე და არაფერი მჭირს. ანალიზებიც გავიკეტე უამრავი და იქაც ნორმალურადაა ყველაფერი. თერმომეტრს რომ ვხედავ, მაკანკალებს უკვე. რა ვქნა, სად წავიდე?

Posted by: vano_t 18 Mar 2010, 22:26
MissFrance
QUOTE
ან დამამშვიდეთ, ან რამე მიშველეთ.... sad.gif    3 თვეა სიცხე მაქვს 37.    37.1.    ბოლო დროს თავბრუ მეხვევა.  ფილტვები კი არა, თუ რამე ორგანო არსებობს ყველაფერი გამოვიკვლიე და არაფერი მჭირს. ანალიზებიც გავიკეტე უამრავი და იქაც ნორმალურადაა ყველაფერი.  თერმომეტრს რომ ვხედავ, მაკანკალებს უკვე. რა ვქნა, სად წავიდე?

37 არ არის სიცხე. თუ ყველაფერი ნორმაში გაქვს (გამოკვლევები ჩაგიტარა ექიმმა და ვერაფერი ნახა), აღარაფერია სანერვიულო. 37 სიცხეს საერთოდ არ ჭირდება არაფერი გამოკვლევა, თუ სიცხის გარდა ავადმყოფს სხვა სიმპტომები არ აქვს.

Posted by: MissFrance 18 Mar 2010, 22:40
QUOTE
37 არ არის სიცხე. თუ ყველაფერი ნორმაში გაქვს (გამოკვლევები ჩაგიტარა ექიმმა და ვერაფერი ნახა), აღარაფერია სანერვიულო. 37 სიცხეს საერთოდ არ ჭირდება არაფერი გამოკვლევა, თუ სიცხის გარდა ავადმყოფს სხვა სიმპტომები არ აქვს

თავბრუს ხვევა მაქვს, 1 თვეა
* * *
შეიძლება ნევროზული იყოს? ავერსის კლინიკაში დავდივარ და ვერაფერი გაუგიათ, რა მჭირს. ბოლოს ნევროპათოპლოგთან გამგზავნეს და ხვალ მივდივარ. მგონია, რომ სიცხემ დამმართა ნერვოზი და არა პირიქით yes.gif

Posted by: vano_t 18 Mar 2010, 23:39
MissFrance
QUOTE
შეიძლება ნევროზული იყოს?
თუ გამუდმებით გაქვს თავბრუსხვევა, მაშინ ალბათ არ შეიძლება ნევროზს მიაწერო. ისე პანიკური შეტევების დროს შესაძლებელია თავბრუსხვევა. ავადმყოფი სწრაფად სუნთქავს და სისხლში ეცემა ნახშირორჟანგის კონცენტრაცია, რამაც შეიძლება დაბუჟების შეგრძნება და თავბრუსხვევა გამოიწვიოს.

ხშირი თავბრუსხვევების დროს ექიმს უნდა ეჩვენო. ექიმი გასინჯავს წნევას დაწოლილ და ფეხზე დამდგარ მდგომარეობაში, გასინჯავს პულსს, სისხლში ჟანგბადის შემცველობას, გააკეთებს მარტივ ლაბორატორიულ ტესტებს (რომ გამორიცხოს ძალიან დაბალი ჰემოგლობინი, გაუწყლოება) და თუ ყველაფერი ნორმაში, მაშინ ცოტა ხანი დაკვირვება შეიძლება სხვა უფრო ძვირი ტესტების გაკეთების გარეშე. ეს სავარაუდი სცენარია. საბოლოო ჯამში ექიმმა სრული ინფორმაციის შეგროვებით უნდა დასვას დიაგნოზი და დაუნიშნოს მკურნალობა.

Posted by: MALI 19 Mar 2010, 16:17
ექიმებო
თქვენი რჩევა მჭირდება

ტუჩები მეწვის უკვე რამდენი ხანია ზამთარ -ზაფხულ და პომადას ხმარების შემდეგ კი კანი მექერცლება.
შევცვალე ფირმა. ვხმარობდი ვიშის ჰიგიენურსაც მაგრამ მაინც ასე მაქვს..
ა ვიტამინოზე გექნებაო და ვისვამდი ამ ვიტამინს ტუჩზე მაგრამ არ მეშველა
რა შეიძლება იყოს ან რის ექიმს უნდა მივაკითხო?

მადლობა წინასარ

Posted by: pouet 19 Mar 2010, 22:06
ალბათ დიდ რჩევას ვერავინ მომცემს, მაგრამ მაინც...
სათხილამუროდ ვიყავი და სახე მაქვს საკმაოდ დამწვარი, თან ნიღბის თეთრი კვალი... მოკლედ ვიზუალური მხარე მხდის ცუდად მეც და გარშემომყოფებსაც.. თუ იცით რამე საშუალება გარუჯვის მოსაცილებლად, გისმენთ. smile.gif
მადლობა

Posted by: vano_t 20 Mar 2010, 13:43
MALI
QUOTE
ტუჩები მეწვის უკვე რამდენი ხანია ზამთარ -ზაფხულ და პომადას ხმარების შემდეგ  კი კანი მექერცლება.
შევცვალე ფირმა.  ვხმარობდი ვიშის ჰიგიენურსაც    მაგრამ მაინც ასე მაქვს..
ა ვიტამინოზე გექნებაო და ვისვამდი  ამ ვიტამინს ტუჩზე მაგრამ არ მეშველა
რა შეიძლება იყოს ან რის ექიმს უნდა მივაკითხო?

თუ მარტო პომადის ხმარების დროს გიჩნდება ეგ, პომადის ბრალია-არ აქვს მნიშვნელობა სხვადასხვა ფირმისას იხმარ თუ არა. არ იხმარო პომადა ცოტა ხანი და თუ ყველაფერმა გაიარა, მაშინ მიხვდები რომ პომადის ბრალია.

pouet
QUOTE
თუ იცით რამე საშუალება გარუჯვის მოსაცილებლად, გისმენთ.
დრო და მოთმინება smile.gif

მომავალში შეგიძლია მზისგან დასაცავი კრემები იხმარო და თავიდან აიცილებ მაგ პრობლემას.

Posted by: MALI 20 Mar 2010, 18:53
vano_t
მადლობა ვანო

პომადის გარეშე რომ ვერ შევძლებ??? aba.gif .... ეს ისე ნახევრად ხუმრობით

.
QUOTE
არ იხმარო პომადა ცოტა ხანი


კი ასეც ვიქცევი , მაგრამ მგონი საქმე უფრო სერიოზულად მაქვს.........

ვიცი,, შეგიძლიათ კიდევ რაიმე სხვა რჩევის მოცემა

Posted by: რაჟდენ 20 Mar 2010, 20:25
vano_t
გამარჯობა,ერთი კითხვა მაქვს და იქნებ იცოდე smile.gif

მოკლედ,კორტიზოლის რაოდენობას ვიკვლევ სისხლში,9:00(am)-ზე რომ ავიღე ვენიდან სისხლი,4.5-ჯერ იყო ნორმის მაქსიმალურ ზღვარს აცდენილი კორტიზოლის ოდენობა.
ახლა ვიკეთებ 12:00(შუაღამეს),მაგრამ კითხვა რაში მდგომარეობას: ყველა ადამიანში ასეა,რომ დილის ცხცრაზეა მაქსიმალური რაოდენობა და შუაღამისას ქვემოთ იწევს? იმიტომ ვკითხულობ,რომ მეშინია,არასწორი მონაცემები არ მივიღო.ძირითადად,მე დღე-ღამის რეჟიმი არეული მაქვს და იმ ღამით საერთოდ არ მეძინა,როცა დილის 9-ზე ამიღეს სისხლი.ვერ ვიტყვი,რომ უძილარი ვიყავი და სტრესის გამო მოიმატა კორტისოლმა.წინა დღეს საღამოს 8 საათზე გავიღთვიძე და მთელი ღამე რომ არ მეძინა,ამაში არაფერია გასაკვირი.

დღესაც ვიკეთებ ანალიზს 12:00-ზე, ღამე,მაგრამ დღესაც 6 საათზე გამეღვიძა(საღამოს)დავიძინე დილის 8 საათზე.ტყუილად ხომ არ ვიკეთებ? sad.gif
მოგვცემს ეს რეალურ სურათს?

ისეთი რეჟიმი მაქვს,რომ კვირაში 3 დღე ჩვეულებრივად მძინავს ღამე და ვიღვიძებ დილის 10-ზე,ხოლო დარჩენილი 4 დღე კი შებრუნებული მაქვს დღე-ღამის რეჟიმი - დღე მძინავს და ღამე ფხიზლად ვარ.ძილი არ მაკლდება,უბრალოდ,შეტრიალებული მაქვს რეჟიმი კვირაში 4 დღე(გარკვეული მიზეზების გამო).

მაინტერესებს,ტყუილად ხომ არ მივდივარ დღეს ანალიზის გასაკეთებლად?

ახლადგაღვიძებული როცა ხარ,მაშინ გაქვს კორტიზოლის რაოდენობა მომატებული სისხლში თუ იმენნა დილის 9:00 საათზე იმატებს,გინდაც გეძინოს?

წინასწარ მადლობა.

პ.ს. კუშინგის სინდრომს ვიკვლევ.
პ.პ.ს. ტანკე ვარ



Posted by: vano_t 20 Mar 2010, 23:00
რაჟდენ
გაგაიმარჯოს ტანკე
QUOTE
მაგრამ კითხვა რაში მდგომარეობას: ყველა ადამიანში ასეა,რომ დილის ცხცრაზეა მაქსიმალური რაოდენობა და შუაღამისას ქვემოთ იწევს?
ყველა ადამიანში ასე ვერ იქნება. საერთოდ რაღაც ტესტს რომ იკვლევ, უამრავ ჯანმრთელ ადამიანზე აკეთებენ გამოკვლევას და მის განაწილებას სწავლობენ. მერე ადგენენ ე.წ. cut-off value-ს, იმის მიხედვით ჭირდებათ ტესტის სენსიტიურობის თუ სპეციფიურობის გაზრდა. ყოველთვის იქნება ადამიანი, რომელიც ჯამრთელია და ტესტი ნორმის ზღვრიდან არის გამოსული, რადგანაც ნორმა-არანომრა გარკვეულწილად ხელოვნური არჩევანია.

QUOTE
მაინტერესებს,ტყუილად ხომ არ მივდივარ დღეს ანალიზის გასაკეთებლად?
აქ სხვანაირად ადგენენ კუშინგს. პირველ რიგში კლინიკური ნიშნები უნდა ქონდეს ავადმყოფს ამისა (მთვარესებური სახე, buffalo hump, კანის ადვილი დაზიანებები, დიაბეტი და ა.შ.). ამის მერე აკეთებენ ტესტებს. ყველაზე სპეციფიურ ტესტად ითვლება 24 საათში შარდის შეგროვება და კორტიზოლის ანალიზი 24 საათში გამოყოფილ შარდში. მეორე ტესტი, რომელიც დაშვებულია არის ღამის ნერწყვში კორტიზოლის რაოდენობის განსაზღვრა. დანარჩენი სპეციფიური ტესტებია: მაგალითად, დექსამეტაზონით დათრგუნვის ტესტი და პეტროზული სინუსის სისხლის ანალიზი.

აგერ ქვემოთ დავდებ კარგ ლინკს. ექიმი კაცი ხარ და ინგლისურიც იცი. გადაიკითხე, საკმაოდ კარგად არის ყველაფერი მოცემული. თუ რაიმე ვერ გაიგე, იკითხე და შევეცდები გიპასუხო:
http://www.aafp.org/afp/20000901/1119.html

MALI
QUOTE
კი ასეც ვიქცევი , მაგრამ მგონი საქმე უფრო სერიოზულად მაქვს.........

ვიცი,, შეგიძლიათ კიდევ რაიმე სხვა რჩევის მოცემა
ზუსტად ვერ გავიგე: როცა პამადას არ ხმარობ, მაინც გაქვს პრობლემა?

თუ არ გაქვს პრობლემა როცა პომადას არ ხმარობ, მაშინ სერიოზული არაფერი უნდა იყოს. უბრალოდ პომადის რაღაც ნაერთზე ან ნაერთებზე გაქვს რაღაცა სახის რეაქცია. და თუ ასეა და მაინცდამაინც გინდა პომადის ხმარება, მაშინ შეგიძლია სინჯო ტუჩების დამატენიანებლის ხმარება პომადის ქვეშე. მოგიხდება თუ არა, მაგას ვერ გეტყვი, მაგრამ მოსინჯვამ არ უნდა გავნოს წესით.

Posted by: lala 21 Mar 2010, 01:09
გამარჯობათ
ფეხები მტკივა ეს ბოლო დროა ძაან, ნუ როგორ სახსრები მტკივა.
რისი ბრალი შეიძლება იყოს და რა ანალიზები ჩავიტარო რო გავიგო
ძაან მტკვია თითქოს მჭამს რაღაცა....

Posted by: TETISA 21 Mar 2010, 01:14
იუნიდოქსი დამინიშნა ექიმმა. 10 დღე სვიო.
ვიზიტზე მიXთრა კიდე 2 კვირა გააგრძელეო.
ინსტრუქციაში რომ წავიკითხე, ეს ანტიბიოტიკი 10 დღეზე მეტი არაა მითითებული, რომ შეიძლება დაილიოს. აი ზოგჯერ ხომ წერენ, ექიმის დანიშნულებს მიზხედვით შეიძლება ხანგრძლივობის ან დოზის მომატებაო, აქ ეგ არ წერია.
დავიჯერო ესე კიდია მაგ ექიმს პაციენტის ჯანმრთელობა. მიკროფლორას დედა აღარ ეყოლება ესეთი ტემპით.

Posted by: რაჟდენ 21 Mar 2010, 01:43
vano_t
მადლობა ამომწურავი აპსუხისთვის.
QUOTE
აქ სხვანაირად ადგენენ კუშინგს. პირველ რიგში კლინიკური ნიშნები უნდა ქონდეს ავადმყოფს ამისა

მაქვს სპეციფიური სიმსუქნე,ატროფიული სტრიები(მელნისფერი),ჰიპერტენზია და ა.შ.
ვიყავი პროფესორ ენდოკრინოლოგთან და მაგან მირჩია მაგ ტესტების გაკეთება.

მარჯვენა თირკმელზედა ჯირკვლის MRT გავიკეთე,არაფერი ისეთი,მარტო რაღაც კედლის გასქელება იყო.
QUOTE
ყველაზე სპეციფიურ ტესტად ითვლება 24 საათში შარდის შეგროვება და კორტიზოლის ანალიზი 24 საათში გამოყოფილ შარდში

გავიკეთე, ნორმაშია.

ხო,კუშინგი თითქმის გამორიცხეს, დღეს რომ ვიყავი.დღეს ორჯერ ამიღეს ვენიდან სისხლი,ნატრიუმის რაოდენობა გაქვს ძალიან მომატებეულიო.ახლა 12:00 საათიან კორტიზოლს ელოდებიან.ვნახოთ,მომატებული იქნება თუ არა 9:00-ის მსგავსად.
თავზე მაქვს MRT გაკეთებული.ჰიპოფიზის სუბატროფია მაქვს,განლეული თურქული კეხისადმი მიდრეკილებაო. კიდევ თადაყოლილი ანომალია - კეფის დიდი ცისტერნის დილატაცია.
2მმ იყო ჰიპოფიზის კრანიოკაუდალური ნაწილი.ძალიან არის განლეული.

ახლა ჰიპოფიზზე ეჭვობენ და თირკმელზედას შეეშვნენ პრაქტიკულად.

მოკლედ,ყველა მხრებს იჩეჩავს და თვალებს ატრიალებს,ურთიერთგამომრიცხავი რაღაცეებია და დიაგნოზს ვერ ვსვამთო sad.gif ყველაფერი მე როგორ უნდა დამემართოს sad.gif ექიმი ჰაუსი მჭირდება biggrin.gif

უკვე აღარ ვიცი,რა ვქნა


Posted by: lala 21 Mar 2010, 01:49
vano_t
იქნებ მეც მიპასუხოთ
დიდი მადლობა

Posted by: vano_t 21 Mar 2010, 01:58
ტანკე
QUOTE
მაქვს სპეციფიური სიმსუქნე,ატროფიული სტრიები(მელნისფერი),ჰიპერტენზია და ა.შ.
ვიყავი პროფესორ ენდოკრინოლოგთან და მაგან მირჩია მაგ ტესტების გაკეთება.
სიმსუქნეს ბევრი რამ ახლავს თავისთავად (ჰიპერკოტიზოლიზმის გარეშეც რომ იყოს სიმსუქნე) შენი ჩამთვლილი ნიშნების ჩათვლით: სტრიები, ჰიპერტენზია, გლუკოზისადმი ტოლერანტობის დაქვეითება და ა.შ. სპეციფიურ სიმსუქნეში რას გულისხმობ?

QUOTE
QUOTE
ყველაზე სპეციფიურ ტესტად ითვლება 24 საათში შარდის შეგროვება და კორტიზოლის ანალიზი 24 საათში გამოყოფილ შარდში

გავიკეთე, ნორმაშია.
თუ ეს ნორმაშია, მაშინ რაღას ეძებენ და რატომ? შეიძლება მიზეზი აქვთ, მაგრამ მე არ ვიცი ასეთი რამ.

წონის დაკლება თუ გიცდია ვარჯიშებით და დიეტით?

QUOTE
ახლა ჰიპოფიზზე ეჭვობენ და თირკმელზედას შეეშვნენ პრაქტიკულად.
კუშინგის სინდრომი არის ნებისმიერი მიზეზით გამოწვეული ჰიპერკორტიზოლიზმი. მე ვერ ვხვდები რატომ უნდა ეს ყველა იმიგიჯინგ კვლევები, თუ კორტიზოლი ნორმაში გაქვს (24-სათიანი ტესტი ნორმაშია). ჩემს ახლობელ ენდორკინოლოგს შევეკითხები და ვნახავ რას მეტყვის.

lala
QUOTE
გამარჯობათ
ფეხები მტკივა ეს ბოლო დროა ძაან, ნუ როგორ სახსრები მტკივა.
რისი ბრალი შეიძლება იყოს და რა ანალიზები ჩავიტარო რო გავიგო
ძაან მტკვია თითქოს მჭამს რაღაცა....
ამაზე ინტერნეტით შეუძლებელია საუბარი. ამისათვის მინიმუმ დეტალური გამოკითხვა და ფიზიკური გამოკვლევაა საჭირო, რაც ექიმის მიერ უნდა ჩატარდეს.

TETISA
QUOTE
  იუნიდოქსი დამინიშნა ექიმმა. 10 დღე სვიო.
ვიზიტზე მიXთრა კიდე 2 კვირა გააგრძელეო.
ინსტრუქციაში რომ წავიკითხე, ეს ანტიბიოტიკი 10 დღეზე მეტი არაა მითითებული, რომ შეიძლება დაილიოს. აი ზოგჯერ ხომ წერენ, ექიმის დანიშნულებს მიზხედვით შეიძლება ხანგრძლივობის ან დოზის მომატებაო, აქ ეგ არ წერია.
დავიჯერო ესე კიდია მაგ ექიმს პაციენტის ჯანმრთელობა. მიკროფლორას დედა აღარ ეყოლება ესეთი ტემპით
შენს კონკრეტულ შემთხვევაზე ვერაფერს ვიტყვი, მაგრამ ექიმის მიერ მკურნალობის ხანგრძლივობის გაგრძელება ან შემოკლება ხშირია ყველა ქვეყანაში. პრინციპში, ინსტურქციაში წერია მხოლოდ რეკომენდირებული ხანგრძლივობა.

Posted by: TETISA 21 Mar 2010, 02:10
vano_t
QUOTE
შენს კონკრეტულ შემთხვევაზე ვერაფერს ვიტყვი, მაგრამ ექიმის მიერ მკურნალობის ხანგრძლივობის გაგრძელება ან შემოკლება ხშირია ყველა ქვეყანაში. პრინციპში, ინსტურქციაში წერია მხოლოდ რეკომენდირებული ხანგრძლივობა.

უცხო ექიმია ის ჩემთვის. დაზღვევით მომიწია მასთან მისვლა და არ ვიცი მისი დონე. ამიტომ ვიფიქრე გადავამოწმო თქო.
და როცა რეკომენდირებულია 10 დღე და კიდევ 14 დღეს დაგიმატებენ, ეგ მოსულა ექიმებთან?
ანტიბიოტიკოფობია მაქვს

Posted by: რაჟდენ 21 Mar 2010, 02:14
vano_t
QUOTE
სიმსუქნეს ბევრი რამ ახლავს თავისთავად (ჰიპერკოტიზოლიზმის გარეშეც რომ იყოს სიმსუქნე) შენი ჩამთვლილი ნიშნების ჩათვლით: სტრიები, ჰიპერტენზია, გლუკოზისადმი ტოლერანტობის დაქვეითება და ა.შ. სპეციფიურ სიმსუქნეში რას გულისხმობ?

მარტო მუცლის არეში და კისრის ირგვლივ.

კუნთები ატროფირებული მაქვს და ძვლები მყიფე.კუშინგმა როგორც იცის
QUOTE
თუ ეს ნორმაშია, მაშინ რაღას ეძებენ და რატომ? შეიძლება მიზეზი აქვთ, მაგრამ მე არ ვიცი ასეთი რამ.

სისხლშია მომატებული ზატო,თან ნორმა არის 10-48,მე მაქვს 180-ზე მეტი.ენდოკრინული მდგომარეობის მხრივ რომ რაღარ წესრიგში არ გაქვს,ეს ცხადიაო,მაგრამ ვერ მივაგენით ჯერ,კონკრეტულად რაო

QUOTE
წონის დაკლება თუ გიცდია ვარჯიშებით და დიეტით?

არა,არ მიცდია.ვარჯიში გამორიცხულია,მომენტალურად მიწევს წნევა და ცუდად ვხდები.ეგ კი არა,გევრდზე რომ გადავიხარო,მაშინაც ვგრძნობ წნევის მომატებას.იატაკიდან რომ რაღაც ავიღო და თავი დავხარო,თავი სისხლით მევსება 1 წამში და ყურები მეგუბება.გავსწორდები თუ არა,გადის მომენტალურად.სხეულის ოდნავ დაძაბვა(ვთქვათ,რაღაცას მინდა შევწვდე და უხერხუალდ გადავიხარე),ეგრევე საშინელებები მემართება.რა ჭირია ეს,წარმოდგენა არ მაქვს.უმოძრაოაბს ვაბრალებ,მარა ასეთი რაღაც?
დიეტაზეც ვერ ვჯდები,შიმშილზეც მიწევს წნევა,ტოესწ,ვგრძნობ მაგ დროს ჩემს "ნორმალურ" წნევას...შეიძლება არც მიწევს.
ხო,ჩემი წნევა 150(160)-110-ია ნორმა(თავს გადასარევად რომ ვგრძნობ მოსვენებულ მდგომარეობაში).ამაზე დაბლა არ ჩამოდის,გამორიცხულია.
ოდნავი დატვირთვაც და...
დაახლოებით 20 წლიდან დამეწყო ეს წნევა,ადრე 140-100 იყო ნორმა,8 წელიწადში 10-ით მომატა მაჩვენებლებმა.პულსი 80-90 მაქვს ნორმა.
QUOTE
ჩემს ახლობელ ენდორკინოლოგს შევეკითხები და ვნახავ რას მეტყვის.

გაიხარე smile.gif
QUOTE
მე ვერ ვხვდები რატომ უნდა ეს ყველა იმიგიჯინგ კვლევები,

ზუსტი ლოკალიზაციის მოსაძებნად,როგორც თქვეს.


Posted by: vano_t 21 Mar 2010, 02:21
TETISA
QUOTE
უცხო ექიმია ის ჩემთვის. დაზღვევით მომიწია მასთან მისვლა და არ ვიცი მისი დონე. ამიტომ ვიფიქრე გადავამოწმო თქო.
და როცა რეკომენდირებულია 10 დღე და კიდევ 14 დღეს დაგიმატებენ, ეგ მოსულა ექიმებთან?
ანტიბიოტიკოფობია მაქვს

ეგ ექიმი როგორ უდგება საკითხს არ ვიცი. მე ავადმყოფს ვუხსნი ხოლმე რას ვმკურნალობ, როგორ და რა საფუძველი გააჩნია მკურნალობას.

რას მკურნალობ, სიმპტომები და ობიექტური მონაცემები მოიხსნა თუ არა, specimen-ის დათესვა და მგრძნობელბაზე ანალიზი მოხდა თუ არა, ეს ყველაფერი გავლენას ახდენს. თუ 10 დღე მკურნალობ რაღაცას და შედეგი არ არის, მაშინ მე დავეჭვდებოდი მკურნალობის მიზანშეწონილობაში. დავსვამდი კითხვას, ინფექციასთან მაქვს თუ არა საქმე საერთოდ და ან გადავსინჯავდი დიაგნოზს, ან მკურნალობას შევცვლიდი სხვა ანტიბიოტიკით. ეს ყველაფერი კონკრეტული შემთხვევებისათვის სპეციფიურია და ძნელია შენს შემთხვევაში კერძოდ რაიმე თქვა. შენი ექიმი რას ამბობს ამაზე? რა დიაგნოზის დაგისვა და რატომ მკურნალობს 24 დღე?

რაჟდენ
გაგირკვევ უფრო დაწვრილებით და PM-ში მოგწერ.

Posted by: Solveig 21 Mar 2010, 02:23
lala
QUOTE
ფეხები მტკივა ეს ბოლო დროა ძაან, ნუ როგორ სახსრები მტკივა.
რისი ბრალი შეიძლება იყოს და რა ანალიზები ჩავიტარო რო გავიგო
ძაან მტკვია თითქოს მჭამს რაღაცა....

რევმატოლოგთან მიდი და გაგსინჯავს კიდეც, გამოკვლევებსაც გაგაკეთებინებს.

Posted by: MALI 21 Mar 2010, 11:05
vano_t

quote]როცა პამადას არ ხმარობ, მაინც გაქვს პრობლემა?[/quote]

რა თქმა უნდა მაქვს, მაინც პრობლემა მგრამ პომადის შემთხვევაში განსაკუთრებით, სისხლიც კი მდის

QUOTE
მაშინ შეგიძლია სინჯო ტუჩების დამატენიანებლის ხმარება 


პირველ პოსტში დავწერე რომ ვიშის ფირმის ჰიგიენურს ვხმარობდი
QUOTE
როცა პამადას არ ხმარობ, მაინც გაქვს პრობლემა?


რის ექიმი იკვლევს ამგვარ დაავადებებს?

Posted by: vano_t 21 Mar 2010, 11:50
MALI
QUOTE
რის ექიმი იკვლევს ამგვარ  დაავადებებს?

დერმატოლოგს უნდა ეჩვენო. ისე წესით თერაპევტმა უნდა ნახოს პირველად (თუ ესმის ცოტა დერმატოლოგია).

Posted by: lala 21 Mar 2010, 14:33
Solveig
აუ სად ვნახო კარგი რევმატოლოგი
იქნებ მიმასწავლოთ

Posted by: Solveig 21 Mar 2010, 15:31
lala
რევმატოლოგიის ცენტრში მიდი, უზნაძის ქუჩაზე. მიხეილის საავადმყოფოს უკანაა.

ან კიდევ:
http://www.medportal.ge/pg2.php?act=profs&backact=profs&cid=125&mp_specislistebiid=13

ფორუმზე თემა

http://forum.ge/?f=43&showtopic=33746280&st=0

Posted by: MALI 21 Mar 2010, 18:50
vano_t

დიდი მადლობა მივალ აუცილებლად

Posted by: aranormaluri 21 Mar 2010, 21:10
ერთი კითხვა მაქვს: ალკოჰოლის მოკიდება რაზეა დამოკიდებული? ზოგს რომ ძალიან ეკიდება და ზოგს საერთოდ არა.
ვთქვათ დავლიე ერთი ჭიქა კონიაკი და სხვამაც დალია. მე რატომ მეკიდება ძალიან და ის სხვა კი ფხიზლად არის? (დავუშვათ რომ ორივე ერთნაირად ვართ ალკოჰოლის მომხმარებლები, ანუ იმისი ორგანიზმი არც მეტად და არც ნაკლებად არ არის მიჩვეული ალკოჰოლს)

Posted by: shtori 22 Mar 2010, 02:22
მარჯვენა მხარეს, ნეკნები რომ მთავრდება, პატარა ნეკნების ქვეშ საშინლად ამტკივდა და მერე უკან წავიდა ტკვილი, თითქოს გადაცოცდა რაღაცო : (( ვაიმე, რა ცუდი იყო : ( ისე მეტკიოდა ვერ ვინძრეოდი. 2 წუთის მერე ისევ განმეორდა ოღოდ ისე მძლავრად არა.
ღამე რომ ამტკივდეს ასე გავაფრეენ
რა ვქნა? რა გავმზადო, რომ მერე უცებ დავილო და გამიაროს? help.gif
* * *
აააა sad.gif(( ისევ მეტკინაა
ფუ, რა ჯანდაბაა რაა არის ეს რა cry.gif
ნუთუ ყველას გძინავთ?

Posted by: Solveig 22 Mar 2010, 02:40
shtori
ნაღვლის ბუშტს გავს...რა ჭამე საღამოს?

aranormaluri
QUOTE
ერთი კითხვა მაქვს: ალკოჰოლის მოკიდება რაზეა დამოკიდებული? ზოგს რომ ძალიან ეკიდება და ზოგს საერთოდ არა.
ვთქვათ დავლიე ერთი ჭიქა კონიაკი და სხვამაც დალია. მე რატომ მეკიდება ძალიან და ის სხვა კი ფხიზლად არის? (დავუშვათ რომ ორივე ერთნაირად ვართ ალკოჰოლის მომხმარებლები, ანუ იმისი ორგანიზმი არც მეტად და არც ნაკლებად არ არის მიჩვეული ალკოჰოლს)

ფერმენტ ალკოჰოლდეჰიდროგენაზას აქტივობაზეა, როგორც მე ვიცი...და მემკვიდრეობით გადადის. ზოგში უფრო აქტიურია, ზოგში-უფრო ნაკლებად.


ჩრდილო ამერიკის ინდიელებში, ასევე ჩუქჩებში, ესკიმოსებში ძალიან დაბალი იყო მისი აქტივობა და მაგიტომ იყო, ადვილად რომ თვრებოდნენ და ლოთდებოდნენ.


Posted by: shtori 22 Mar 2010, 02:50
Solveig
QUOTE
ნაღვლის ბუშტს გავს...რა ჭამე საღამოს?

შემწვარი კვერცხი + ტყემალი

Posted by: Solveig 22 Mar 2010, 04:32
shtori
QUOTE
შემწვარი კვერცხი + ტყემალი

ჰოდა, სწორედ მაგიტომ...ორივე საშინლად არ უხდება.
დიეტაზე გადადი რამდენიმე დღე და შემწვარი, ცხიმიანი, ცხარე, ნივრიანი, ალკოჰოლური სასმელი, კვერცხის გული არ მიიღო..და ექიმთან მიდი. ნაღვლის ბუშტს ხუმრობა არ უყვარს. რომ აგტკივდა, ესე იგი, რაღაცაში იქნება საქმე.

პს. არ მკითხო ახლა-აბა, შიმშილით მოვკვდეო? biggrin.gif

Posted by: shtori 22 Mar 2010, 05:24
Solveig
QUOTE
პს. არ მკითხო ახლა-აბა, შიმშილით მოვკვდეო?

biggrin.gif
შიმშილი მირჩევნია კიდე ასეთ შემოტევებს

დიდი მადლობა დახმარებისთვის 2kiss.gif

Posted by: aranormaluri 22 Mar 2010, 17:56
Solveig
QUOTE
ფერმენტ ალკოჰოლდეჰიდროგენაზას აქტივობაზეა, როგორც მე ვიცი...და მემკვიდრეობით გადადის. ზოგში უფრო აქტიურია, ზოგში-უფრო ნაკლებად.


აჰ გასაგებია.ესე იგი ამ ფერმენტის გამომუშავების აქტივობა დამოკიდებულია არაფერზე. უბრალოდ მემკვიდრეობით გადადის.
დიდი მადლობა 2kiss.gif

Posted by: Solveig 22 Mar 2010, 18:06
shtori
QUOTE
იმშილი მირჩევნია კიდე ასეთ შემოტევებს

ნაღვლის ბუშტს არც შიმშილი უხდება, რადგან ამ დროს ბუშტი ივსება ნაღვლით და დაცლა (რაც გულისხმობს ნაღვლის ევაკუაციას თორმეტგოჯაში საკვების მიღების შემდეგ, რეფლექსურად) არ ხდება.#

ყველაზე კარგია დიეტური კვება დღეში რამდენჯერმე.

Posted by: izabel 24 Mar 2010, 21:22
FERRITIN სისხლში ძალიან დაბალი მაქვს. ექიმი თუ შემოდის აქ თუ შეიძლება ამიხსნას ცოტა მაინც მისი არსებობის შესახებ.




წინასწარ მადლობთ.

Posted by: Solveig 24 Mar 2010, 23:49
izabel
ისევ რკინადეფიციტური ანემია ხომა რ გაქვს?

Posted by: izabel 25 Mar 2010, 08:58
Solveig

კი, ამჯერად გემოგლობინი დაბალი კია, მაგრამ საგანგაშო არაა...FERRTIN-ია კატასტროფულად დაბალი.

Posted by: vano_t 25 Mar 2010, 19:16
izabel
QUOTE
კი,    ამჯერად    გემოგლობინი    დაბალი  კია,  მაგრამ  საგანგაშო      არაა...FERRTIN-ია    კატასტროფულად    დაბალი.

დაბალ ფერიტინს სადიაგნოსტიკო მნიშვნელობის გარდა სხვა რაიმე მნიშვნელობა არ აქვს. ფერიტინი მიუთითებს რკინის მარაგზე მხოლოდ. სესაბამისად, რკონადეფიციტური ანემიის დროს, რაც რკინის მარაგის დაქვეითებას იწვევს, ფერიტინიც დაბლა ეცემა. ამიტომ საგანგაშო არაფერია. უმკურნალებენ რკინადეფიციტურ ანემიას და ფერიტინიც აიწევს. ასე რომ, რკინის პრეპარატების გარდა არაფერი საჭირო არ არის.

Posted by: izabel 26 Mar 2010, 00:35

vano_t

დიდი მადლობა.

Posted by: miscolin 26 Mar 2010, 12:31
გამარჯობათ,ესეთი კითხვით გაწუხებთ,პირის ღრუში ზედა სასაზე მქონდა წამონაზარდი,ექიმებმა ზუსტად ვერ დაადგინეს პაპილომა იყო თუ არა,გამიკეთეს ოპერაცია,ამომიჭრეს და მიმიწვეს,ამის შემდეგ რამოდენიმე დღის შემდეგ ძალიან გამიღიზიანდა ეს ადგილი,მაგარი სისხლდენაც მქონდა და წავედი ისევ იმ ექიმთან,და მარგნეცი წამისვა,მითხრა მარგანეცის ძლიერი ხსნაი ისვიო დღეში ორჯერო,მაგრამ შედეგი არ არის,საერთოდ ვერაფერს ვჭამ ტკივილისგან და საშინლად გაღიზიანებულია ეგ ადგილი,რა შეგიძლიათ მირჩიოთ???? sad.gif sad.gif sad.gif

Posted by: Romina 26 Mar 2010, 21:53
miscolin
ვინ არის თქვენი ექიმი პროფესიით, სტომატოლოგი?

მე არ ვარ ამ საკითხში კომპეტენტური. პირში როცა მტკივნეული გამონაყარია შემომგარსველები(ალმაგელი), ლიდოკაინის შემცველი სავლებები და სტეროიდული სავლებები კარგად მოქმედებს, მაგრამ ოპერაციის შემდგომი ჭრილობის შემთხვევაში არ ვიცი ნამდვილად.

იქნებ ვინმე იყოს კომპეტენტური, 3 თემაში იკითხა უკვე ამ ადამიანმა. ეტყობა მართლა ძალიან აწუხებს.

Posted by: vano_t 26 Mar 2010, 23:20
miscolin
QUOTE
გამარჯობათ,ესეთი კითხვით გაწუხებთ,პირის ღრუში ზედა სასაზე მქონდა წამონაზარდი,ექიმებმა ზუსტად ვერ დაადგინეს პაპილომა იყო თუ არა,გამიკეთეს ოპერაცია,ამომიჭრეს და მიმიწვეს,ამის შემდეგ რამოდენიმე დღის შემდეგ ძალიან გამიღიზიანდა ეს ადგილი,მაგარი სისხლდენაც მქონდა და წავედი ისევ იმ ექიმთან,და მარგნეცი წამისვა,მითხრა მარგანეცის ძლიერი ხსნაი ისვიო დღეში ორჯერო,მაგრამ შედეგი არ არის,საერთოდ ვერაფერს ვჭამ ტკივილისგან და საშინლად გაღიზიანებულია ეგ ადგილი,რა შეგიძლიათ მირჩიოთ???? sad.gif  sad.gif  sad.gif

Romina-ს ნათქვამს ვერაფერს დავამატებდი, გარდა იმისა, რომ ექიმმა უნდა ნახოს და დაათვალიეროს ეგ ადგილი. შეიძლება ინფექციაა და მკურნალობაც უნდა. სიმპტომების მოსახსნელად, რაც უკვე აღნიშნა რომინამ, ლიდოკაინის სავლებია კარგი. თუ ინფექცია არ არის და არის მაგალითად აფთოზური წყლული, მაშინ ადგილობრივი სტეროიდები შეიძლება მოუხდოს.

Posted by: miscolin 27 Mar 2010, 00:25
რამდენჯერაც ვიყავი ექიმთან და დაურეკე,გაგივლისო არაუშავსო,მაქსიმუმ ერთ თვეში შეგიხორცდებაო,ასე მაბოლებს თავის ჩკუითsad.gifარადა რანაირად უნდა შეხორცდეს,ისეთ ცუდ ადგილასაა,ხან ნერწყვი ედება,ხან წყალი,პირის ღრუა ბოლო-ბოლო,ზედა სასაsad.gifsad.gifsad.gifმოკლედ დავიტანჯე,ნუთუ არავინ არაა სპეციალისტი???

Posted by: Solveig 28 Mar 2010, 14:29
მე მაქვს ასეთი კითხვა:

ვენების ვარიკოზული დაავადების დროს რა სიხშირითაა რეკომენდებული ელასტიკური კომპრესიული წინდების ტარება? მუდმივად უნდა ეცვას ადამიანს, თუ შეიძლება ატარო 1 კვირა, მერე 1 კვირა შეისვენო და ა. შ.?

Posted by: vano_t 29 Mar 2010, 02:39
Solveig
QUOTE
ვენების ვარიკოზული დაავადების დროს რა სიხშირითაა რეკომენდებული ელასტიკური კომპრესიული წინდების ტარება? მუდმივად უნდა ეცვას ადამიანს, თუ შეიძლება ატარო 1 კვირა, მერე 1 კვირა შეისვენო და ა. შ.?

მე პირადად არ გამიგია ვარიკოზული ვენების დროს ელასტიკური კომპრესიული წინდების ტარების აუცილებლობა. შეიძლება რამეს აკეთებდეს ეს, მაგრამ მე არ ვიცი. ვარიკოზების დროს შეიძლება იხმარო ესენი, როცა ვარიკოზებს თან ახლავს ქვემო კიდურების შეშუპება. კომპრესიული წინდები, როგორც ვიცი, ხელს უწყობს შეუშუპების მოხსნას. ბევრჯერაც დამინიშნავს და ვინშნავ ავადმყოფებში ქვემო კიდურების შეშუპების დროს. ამის გარდა შეიძლება AceWrap-ის ხმარებაც. შეშუპების დროს თუ გამოიყენებ, მაშინ პირადი ინდივიდუალურია იმის გადაწყვეტა რა სიხშირით უნდა იხმარო. ზოგს მუდმივად ჭირდება, ზოგს შეიძლება ერთხელ დაუნიშნო რამოდენიმე დღე და მერე აღარ დაჭირდეს. ზოგს შეიძლება ხანგამოშვებით გაუკეთო. დამოკიდებული იმაზე, როგორ ხდება ქვემო კიდურების შეშუპების კონტროლი.

Posted by: Mr Adam 30 Mar 2010, 17:41
ცხვირი მაქვს გაჭედილი , ხანდახან ვახველებ , თავი მტკივა .. ყელი მეფხანება
რომელი წამალი დავლიო?.. sad.gif


Posted by: IkaMac 30 Mar 2010, 18:41
გამარჯობა, ტერფზე აი სურათზე სადაც არის აღნიშნული , მაგ ადგილას მაქ შიგნი შარიკივით და ხომ ვერ მეტყვით რა არის და როგორ ვიმკურნალო?
(ისე არ მაწუხებს არც სიარულისას და არც ისე)

user posted image

Posted by: vano_t 31 Mar 2010, 02:05
Mr Adam
QUOTE
ცხვირი მაქვს გაჭედილი , ხანდახან ვახველებ , თავი მტკივა .. ყელი მეფხანება
რომელი წამალი დავლიო?..  sad.gif
tylenol cold and cough-თუ იშოვება მაქეთ, ან nyquil ღამით და dayquil დღისით. ან შეგიძლია მარტო სითხეები მიიღო, კარგად დაისვენო და გადაიტან მაგ ყველაფერს. მე პირადად გაციების დროს არაფერს არ ვღებულობ სითხეების გარდა.

IkaMac
QUOTE
გამარჯობა, ტერფზე აი სურათზე სადაც არის აღნიშნული , მაგ ადგილას მაქ შიგნი შარიკივით და ხომ ვერ მეტყვით რა არის და როგორ ვიმკურნალო?
(ისე არ მაწუხებს არც სიარულისას და არც ისე)
თუ არ გაწუხებს, არაფერი მკურნალობა არ უნდა. შეგიძლია დააკვირდე იზრდება თუ არა. ყველაზე კარგია გამოცდილ ექიმს აჩვენო. აქედან ვერ გიეტყვი რა არის-ექიმმა უნდა დაათვალიეროს. მაგ ადგილას ხშირად სესამოიდური ძვალი არის, რაც ნორმალურია. მეტატარსო-ფალანგეალური სესამოიდური ძვალი ქვია როცა მასეთი ლოკალიზაციის არის. თუმცა, სხვა რაიმეც შეიძლება იყოს.

Posted by: Mr Adam 31 Mar 2010, 06:59
vano_t
QUOTE
tylenol cold and cough-თუ იშოვება მაქეთ, ან nyquil ღამით და dayquil დღისით. ან შეგიძლია მარტო სითხეები მიიღო, კარგად დაისვენო და გადაიტან მაგ ყველაფერს. მე პირადად გაციების დროს არაფერს არ ვღებულობ სითხეების გარდა.


გმადლობ, ახლა უკვე სიცხეც მაქვს sad.gif


Posted by: vano_t 31 Mar 2010, 07:21
ტაილენოლი სიცხესაც შველის. შეგიძლია იბპუროფენიც (მოტრინი, ადვილი) მიიღო.

Posted by: IkaMac 31 Mar 2010, 13:13
vano_t
QUOTE
თუ არ გაწუხებს, არაფერი მკურნალობა არ უნდა. შეგიძლია დააკვირდე იზრდება თუ არა. ყველაზე კარგია გამოცდილ ექიმს აჩვენო. აქედან ვერ გიეტყვი რა არის-ექიმმა უნდა დაათვალიეროს. მაგ ადგილას ხშირად სესამოიდური ძვალი არის, რაც ნორმალურია. მეტატარსო-ფალანგეალური სესამოიდური ძვალი ქვია როცა მასეთი ლოკალიზაციის არის. თუმცა, სხვა რაიმეც შეიძლება იყოს.

მადლობ გაიხარე!


>...............

Posted by: niniaa 31 Mar 2010, 13:58
სალიცენზიო ტესტები ზოგად თერაპიაში თუ დევს ინეტში,იქნებ მითხრათ, ფლიიზ...

Posted by: Mr Adam 31 Mar 2010, 15:32
QUOTE (vano_t @ 31 Mar 2010, 07:21 )
ტაილენოლი სიცხესაც შველის. შეგიძლია იბპუროფენიც (მოტრინი, ადვილი) მიიღო.

რამე გვერდითი მოვლენები ხო არ ახასიათებს?..
ისე ბევრად ჯობია ხო წამლის მიღების გარეშე თავისით განიკურნო?..

Posted by: vano_t 31 Mar 2010, 22:44
Mr Adam
QUOTE
რამე გვერდითი მოვლენები ხო არ ახასიათებს?
გვერდითი მოვლენა ყველა წამალს აქვს-არ არსებობს წამალი გვერდითი მოვლენის გარეშე.

QUOTE
ისე ბევრად ჯობია ხო წამლის მიღების გარეშე  თავისით განიკურნო?..
რა თქმა უნდა ჯობია. ვირუსულ დაავადებებზე არც არსებობს რაიმე ეფექტური საშუალება ისედაც. ვირუსული დაავადებების დროს მკურნალობენ მხოლოდ სიმპტომებს (სიცხეს, ტკვილის). "ვირუსს" ორგანიზმმა უნდა მოერიოს ისედაც.

Posted by: ikebana 1 Apr 2010, 01:11
ქვემოდან ბიძგები რომ გეჩვენება რას ნიშნავს? აი ტერფებით ვგრძნობ, თავბრუსხვევის მსგავსია.

Posted by: Mr Adam 1 Apr 2010, 07:29
vano_t
QUOTE
გვერდითი მოვლენა ყველა წამალს აქვს-არ არსებობს წამალი გვერდითი მოვლენის გარეშე.


ვიტამინებსაც?..


Posted by: vano_t 1 Apr 2010, 08:03
Mr Adam
QUOTE
QUOTE
გვერდითი მოვლენა ყველა წამალს აქვს-არ არსებობს წამალი გვერდითი მოვლენის გარეშე.


ვიტამინებსაც?..

ვიტამინებსაც, თუ დიდი რაოდენობით მიიღებ. გარდა ამისა, ვიტამინები ქიმიურად მზადდება და არავიტამინურ მინარევებზე შეიძლება სერიოზული რეაქციები.

ikebana
QUOTE
ქვემოდან ბიძგები რომ გეჩვენება რას ნიშნავს? აი ტერფებით ვგრძნობ, თავბრუსხვევის მსგავსია.
ვერტიგო ხომ არ გაქვს შემთხვევით? ვერტიგო ორნაირია: 1) ტრიალის შეგრძნება; 2) მოძრაობის შეგრძნება ტრიალის გარეშე.

ვერტიგო თუ არის, მაშინ ან ყელყუცხვირის ექიმმა უნდა გაგსინჯოს ან ნევროპათოლოგმა. თუ ქრონიკული სიმპტომები გაქვს, მაშინ სავარაუდოა მენიერის დაავადება ან რაღაც მსგავსი. ვერტიგოს MRI ჭირდება. ნახე კარგი ნევროპათოლოგი. მე შეიძლება მიმართულება მოგცე-სხვანაირად აქედან ვერც დიაგნოზს დასვამ მასე და ვერც მკურნალობას ჩაატარებ.

Posted by: gadya 1 Apr 2010, 11:21
მაქვს წელი ტკივილი, პირველად დაახლოებით 1-2 თვის წინ შევამჩნიე ვარჯიშის დროს უკან რო გადავიხარე;
ვერ ვიტყვი რო რამე განსაკუთრებულად მაწუხებს მაგრამ უკან გადახრისას ისევ მაქვს ტკივილი და მგონი ნელნელა მიუარესდება მდგომარეობა...
სამსახურში უმეტესად კომპიუტერთან მიწევს ჯდომა და უფრო ამას ვაბრალებ sad.gif

ხო ვერ მეტყვით კონკრეტულად სად და ვის მივმართო, ან რა შეიძლება მჭირდეს და ბოლომდე განკურნებადი თუა?

Posted by: ikebana 1 Apr 2010, 20:31
vano_t
QUOTE
2) მოძრაობის შეგრძნება ტრიალის გარეშე.

ანუ უნდა მეჩვნენებოდეს რომ მე ვმოძრაობ თუ საგნები მოძრაობს ჩემს ირგვლივ?

2 თვეა რაც შევნიშნე. პირველად როცა დამეწყო ორსულად ვიყავი და პროგესტერონის ორალურად მიღებას ვუკავშირებდი, მსუბუქ თავბრუსხვევას ვგრძნობდი, თითქოს მთვრალი ვიყავი. ახლა უბრალოდ ეს ბიძგისმაგვარი შეგრძნება მაქვს პერიოდულად. წნევა მაქვს ხშირად დაბალი და სრულფასოვნად ვერ ვიკვებები, მარხვას ვინახავ. მუდმივად მაქვს დაღლილობის შეგრძნება და ვგრძნობ სისუსტეს. დილით განსაკუთღებით.

გასაგებია, ვესტუმრები ნევროპათოლოგს. დიდი მადლობა.

Posted by: MITIATO 2 Apr 2010, 14:06
ეხლა რა მჭირს. დაახლოებით 5-6 წლის წინ დამეწყო.
სულ მეძინება. რამდენი ხანიც არ უნდა მეძინოს,მაინც. ყოფილა შემთხვევა,დღის განმავლობაში ორჯერ დამიძინებია. მაინც მეძინებოდა. დღის რეჟიმი არეული არ მქონია და ღამეები არ მითენებია. ერთი პერიოდი 9 საათზეც კი ვიძინებდი. არა და ბევრ რამეში მიშლის ხელს. სულ ხომ არ შეიძლბა ადამიანს ეძინოს!
რკინადეფიციტური ანემია არ მაქვს და არც ჰიპოთირეოზი.
კიდევ რა შეიძლება ამის მიზეზი იყოს?



Posted by: Workgroup 2 Apr 2010, 15:55
42 წლის ქალს აქვს სიარულის პრობლემა..ფეხები სტკივა..კერძოდ ქუსლებზე შიგნით აქვს წანაზარდები....ექიმის თქმით ეს წანაზარდები არ იკურნება და დაუნიშნა 5 თVიანი მკურნალობის კუსრი..რომელიც ამ წანაზარდების კიდევ უფრო გაზრდას შეაჩერებდა...გავიდა 5 თვე(მკურნალობის 5 თვე) და სიარულისას ტკივილები უფრო გაუმძაფრდა.......................................მე მაინტერესებსს ოპერაცია თუ არის შესაძლებელი და დადებითი პასუხის შემთხვევაში არსებობს თუ არა ამ წანაზარდების თავიდან წარმოქმნის საფრთხე??....... წინასწარ გიხდით მადლობას........

Posted by: Comprachicos 2 Apr 2010, 16:36
დასიების საწინააღმდეგო რამეს ვერ მირჩევთ? ან მალამო ან რამე ეგეთი,ოღონდ ჰიპერტონული ხსნარი(მარილიანი წყალი) არ გამაგონოთ.ეგ ვარიანტი არაა.სხვა რამე ალტერნატივა არის?

Posted by: nanana 2 Apr 2010, 17:46
ერთი კვირაა ბრმანაწლავი მტკივა და მგონი უნდა გამისკდეს bis.gif
ოპერაციას ვერ გავიკეთებ ეხლა და რამე საშუალება არ არსებობს რომ გადამიაროს?

Posted by: vano_t 2 Apr 2010, 23:34
ikebana
QUOTE
ანუ უნდა მეჩვნენებოდეს რომ მე ვმოძრაობ თუ საგნები მოძრაობს ჩემს ირგვლივ?
ორივე ვერტიგოს სახეობაა.

QUOTE
წნევა მაქვს ხშირად დაბალი და სრულფასოვნად ვერ ვიკვებები, მარხვას ვინახავ. მუდმივად მაქვს დაღლილობის შეგრძნება და ვგრძნობ სისუსტეს. დილით განსაკუთღებით.
იმდენად არასპეციფიური სიმპტომებია ეგ, რომ შეუძლებელია ინტერნეტით რაიმეს თქმა. უამრავ დაავადებას ახლავს დაღლილობა და სისუსტე (დეპრესიების მსუბუქი ფორმების ჩათვლით). რას შეეხება მარხვის შენახვას, თუ ბოსტნეულს და ხილეულს მიიღებ, სრულფასოვანი დიეტაა თავისთავად.

MITIATO
QUOTE
ეხლა რა მჭირს. დაახლოებით 5-6 წლის წინ დამეწყო.
სულ მეძინება. რამდენი ხანიც არ უნდა მეძინოს,მაინც. ყოფილა შემთხვევა,დღის განმავლობაში ორჯერ დამიძინებია. მაინც მეძინებოდა. დღის რეჟიმი არეული არ მქონია და ღამეები არ მითენებია. ერთი პერიოდი 9 საათზეც კი ვიძინებდი. არა და ბევრ რამეში მიშლის ხელს. სულ ხომ არ შეიძლბა ადამიანს ეძინოს!
რკინადეფიციტური ანემია არ მაქვს და არც ჰიპოთირეოზი.
კიდევ რა შეიძლება ამის მიზეზი იყოს?
თითქმის ყველანაირ ქრონიკულ დაავადებას შეიძლება ახლდეს ძილიანობა. გარდა ამისა, ეს შეიძლება ახლდეს ძილის ობსტრუქციულ აპნეას (ეს დაავადება ხშირია წონაში მომატებულთათვის, ვინც ხვრინავს ღამით), ქრონიკული დაღლილობის სინდრომს (Chronic fatigue syndrome) და ფიბრომიალგიას.

Workgroup
QUOTE
42 წლის ქალს აქვს სიარულის პრობლემა..ფეხები სტკივა..კერძოდ ქუსლებზე შიგნით აქვს წანაზარდები....ექიმის თქმით ეს წანაზარდები არ იკურნება და დაუნიშნა 5 თVიანი მკურნალობის კუსრი..რომელიც ამ წანაზარდების კიდევ უფრო გაზრდას შეაჩერებდა...გავიდა 5 თვე(მკურნალობის 5 თვე) და სიარულისას ტკივილები უფრო გაუმძაფრდა.......................................მე მაინტერესებსს ოპერაცია თუ არის შესაძლებელი და დადებითი პასუხის შემთხვევაში არსებობს თუ არა ამ წანაზარდების თავიდან წარმოქმნის საფრთხე??....... წინასწარ გიხდით მადლობას........
ეგ წანაზარდები ძალიან ბევრს აქვს საერთოდ და თანაც მათი უმრავლესობა უსიმპტომოდ მიმდინარეობს (მაგათ ეძახიან დეზებს). აშშ-ში არ გამიგია, რომ წანაზარდებზე ოპერაციას აკეთებდნენ. თუ ტკვილები დილით აქვს განსაკუთრებით (როცა პირველ ნაბიჯებს დგას ლოგინიდან ადგომის შემდეგ) და შემდეგ ცოტა უმჯობესდება, მაშინ მაგ ტკივილები შეიძლება იყოს ე.წ. პლანტარული ფასციიტი. თუმცა, ამის გარდა სხვა რამეებმაც შეიძლება მოგცეს ტკვილი. პლანტარულ ფასციიტს უნდა სპეციალური ვარჯიშები ქუსლისათვის.

აგერ დავდებ ვიდეო ლინკს, რომელიც საუბრობს ამ ვარჯიშებზე. უმარტივესი ვარჯიშებია და ყველას შეუძლია გააკეთოს. თანაც, რომელი ვარჯიშჯი რა დროს უნდა გაკეთდეს და რამდენი ხნით, ყველაფერი კარგად არის გადმოცემული:


თუ ვარჯიშები არ შველის, მაშინ შეიძლება გარკვეულ ადგილას ექიმმა გააკეთოს სტეროიდების ინფექცია, რასაც ხშირად შედეგია აქვს. რა თქმა უნდა, ექიმს უნდა ქონდეს ამის გამოცდილება პირველ რიგში.


Comprachicos
QUOTE
დასიების საწინააღმდეგო რამეს ვერ მირჩევთ? ან მალამო ან რამე ეგეთი,ოღონდ ჰიპერტონული ხსნარი(მარილიანი წყალი) არ გამაგონოთ.ეგ ვარიანტი არაა.სხვა რამე ალტერნატივა არის?
დასიების საწინააღდეგო რას ქვია? რა გაქვს დასიებული? რატომ გაქვს დასიებული? დასიების გარდა თუ გაქვს სხვა სიმპტომები (სიცხე, ღებინება, დასიებულ ადგილას სიწითლე, გახურება და ა.შ.)?

nanana
QUOTE
ერთი კვირაა ბრმანაწლავი მტკივა და მგონი უნდა გამისკდეს bis.gif
ოპერაციას ვერ გავიკეთებ ეხლა და რამე საშუალება არ არსებობს რომ გადამიაროს?
ბრმა ნაწლავის დანამატის ტკივილი უნდა დაადგინოს ექიმმა. შენ ალბათ გტკივა მარჯვენა ქვემო მუცლის არეში, რაც შეიძლება გამოწვეული იყოს სხვადასხვა მიზეზებით. ექიმთან თუ ვერ მიხვალ, მაშინ შეუძლებელია მიზეზის დადგენა. მიზეზის დადგენა აუცილებელია, რათა სერიოზული მიზეზი გამოირიცხოს.

Posted by: Comprachicos 3 Apr 2010, 00:02
vano_t
QUOTE
დასიების საწინააღდეგო რას ქვია? რა გაქვს დასიებული? რატომ გაქვს დასიებული? დასიების გარდა თუ გაქვს სხვა სიმპტომები (სიცხე, ღებინება, დასიებულ ადგილას სიწითლე, გახურება და ა.შ.)?

ტერფის ზურგი.სიწითლე არ მაქვს,არც გახურება,არც ინფექცია და არც სიცხე.
ოპერაციის შემდგომ დასივდა და ...ძვალი ფხიკა ქირურგმა.ანტიბიოტიკები და რაღაცეები ვიკეთე,ჭრილობა ნორმალურადაა,მაგრამ ეს დასიება...მარილი გახსენი წყალში და 1 კვირა დაიდე დღეში სამჯერო.მე მაგ პროცედურებს ვერ გავაკეთებ(რავ ვაკეთე,მეყო,მაინც არ შველის),საშინალდ მეხამუშება ეს მარილი,თან ზაფხული მაინც იყოს,წინდას არ ჩავიცვამდი და ნაკლები დისკომფორტი იქნებოდა.სულ სველი უნდა იყოს მარლა,არ დააშროო და ეს წარმოუდგენელია...1 კვირა თეთრი(მარისლისგან) და "გაკრახმალებული" წინდებით სიარული არ შემიძლია,ტანჯვაა.1 დღე რომ იყოს,კიდევ ჰო...ზეწარიც დაისვრება ძილის დროს და არ მინდა, რა...

Posted by: vano_t 3 Apr 2010, 01:14
Comprachicos
QUOTE
QUOTE
დასიების საწინააღდეგო რას ქვია? რა გაქვს დასიებული? რატომ გაქვს დასიებული? დასიების გარდა თუ გაქვს სხვა სიმპტომები (სიცხე, ღებინება, დასიებულ ადგილას სიწითლე, გახურება და ა.შ.)?

ტერფის ზურგი.სიწითლე არ მაქვს,არც გახურება,არც ინფექცია და არც სიცხე.
ოპერაციის შემდგომ დასივდა და ...ძვალი ფხიკა ქირურგმა.ანტიბიოტიკები და რაღაცეები ვიკეთე,ჭრილობა ნორმალურადაა,მაგრამ ეს დასიება...მარილი გახსენი წყალში და 1 კვირა დაიდე დღეში სამჯერო.მე მაგ პროცედურებს ვერ გავაკეთებ(რავ ვაკეთე,მეყო,მაინც არ შველის),საშინალდ მეხამუშება ეს მარილი,თან ზაფხული მაინც იყოს,წინდას არ ჩავიცვამდი და ნაკლები დისკომფორტი იქნებოდა.სულ სველი უნდა იყოს მარლა,არ დააშროო და ეს წარმოუდგენელია...1 კვირა თეთრი(მარისლისგან) და "გაკრახმალებული" წინდებით სიარული არ შემიძლია,ტანჯვაა.1 დღე რომ იყოს,კიდევ ჰო...ზეწარიც დაისვრება ძილის დროს და არ მინდა, რა...

მაში, მაგისათვის შეგიძლია რამოდენიმე რამ სცადო: 1) ფეხები ხშირად აამაღლო, ისე რომ ტერფი მენჯზე მაღლა იყოს და სითხეს შეუწყო ხელი სიმძიმის ძალით ჩამოვიდეს დაბალ; 2) ადგილობრივად იხმარო ყინული. შეგიძლია ყინული გაახვიო რამე წყალგაუმტარ პარკში და 15-20 წუთით დაიდო შესიებულ ადგილას 2-3 ჯერ დღეში; 3) მიიღო ანთების საწინააღმდეგო პრეპარატები (მაგალითად იბუპროფენი/ადვილი/მოტრინი), თუ, რა თქმა უნდა, არ გაქვს კუჭის წყლული ან კუჭიდან სისხლდენის ისტორია და შეგიძლია კარგად გადაიტანო ეს წამალი; 4) კომპრესიული წინდები იხმარო. შეგიძლია ეს ყველაფერი (ან რამოდენიმე რამ) ერთად გააკეთო.

არაფერიც რომ არ გააკეთო, ისედაც გადის შეშუპება რამოდენიმე დღეში. თუმცა, თუ ზედმეტი შეშუპების მოცილებას ხელს შეუწყობ, მაშინ ჭრილობის ინფექციის შანსი ეცემა და მისი შეხერცებაც უკეთესია.

Posted by: nanana 3 Apr 2010, 12:00
vano_t
QUOTE
QUOTE
ერთი კვირაა ბრმანაწლავი მტკივა და მგონი უნდა გამისკდეს bis.gif
ოპერაციას ვერ გავიკეთებ ეხლა და რამე საშუალება არ არსებობს რომ გადამიაროს?

ბრმა ნაწლავის დანამატის ტკივილი უნდა დაადგინოს ექიმმა. შენ ალბათ გტკივა მარჯვენა ქვემო მუცლის არეში, რაც შეიძლება გამოწვეული იყოს სხვადასხვა მიზეზებით. ექიმთან თუ ვერ მიხვალ, მაშინ შეუძლებელია მიზეზის დადგენა. მიზეზის დადგენა აუცილებელია, რათა სერიოზული მიზეზი გამოირიცხოს.


ექიმთან რასაკვირველია წავალ, მაგრამ ქვეყნიდან გავდივარ 4 დღით და რამე დროებითი შემამსუბუქებელი არ არსებობს? ან დიეტა ან რამე

Posted by: woulstoke 3 Apr 2010, 13:20
მაქვს ასტიგმატიზმი მაღალ სტადიაზე არ მაქვს ორივე თვალსში +0,5 ცოტა სრაპად მეღლება თვალები ადრე ეგ პრობლემა არ მქონია მაგრამ 1 ცლის ცინ კომპტან დიდხანს ჯდომის გამო გამიცჰნდა ეგ პრობლემა ეხლა ისე ტავისუპლად უკვე ვეხარ ვზივარ კომპტან როგორც ადრე მაქვს თვალების სიმშრალე და სიწითლე.ექიმი მეუბნება საშიში არაპერიაო ვატარებ სათვალეებს. მაინტერესებს ბოლომდე ახმიდგება თვალი თუ არა ანუ როგორც ადრე და რა დრო არის ამისტვის საჭირო პ.ს. ვარ 16 წლის

Posted by: Comprachicos 3 Apr 2010, 15:50
vano_t
უღრმესი მადლობა ამომწურავი პასუხისთვის 2kiss.gif

Posted by: Workgroup 4 Apr 2010, 21:01
QUOTE (vano_t @ 2 Apr 2010, 23:34 )


Workgroup
QUOTE
42 წლის ქალს აქვს სიარულის პრობლემა..ფეხები სტკივა..კერძოდ ქუსლებზე შიგნით აქვს წანაზარდები....ექიმის თქმით ეს წანაზარდები არ იკურნება და დაუნიშნა 5 თVიანი მკურნალობის კუსრი..რომელიც ამ წანაზარდების კიდევ უფრო გაზრდას შეაჩერებდა...გავიდა 5 თვე(მკურნალობის 5 თვე) და სიარულისას ტკივილები უფრო გაუმძაფრდა.......................................მე მაინტერესებსს ოპერაცია თუ არის შესაძლებელი და დადებითი პასუხის შემთხვევაში არსებობს თუ არა ამ წანაზარდების თავიდან წარმოქმნის საფრთხე??....... წინასწარ გიხდით მადლობას........
ეგ წანაზარდები ძალიან ბევრს აქვს საერთოდ და თანაც მათი უმრავლესობა უსიმპტომოდ მიმდინარეობს (მაგათ ეძახიან დეზებს). აშშ-ში არ გამიგია, რომ წანაზარდებზე ოპერაციას აკეთებდნენ. თუ ტკვილები დილით აქვს განსაკუთრებით (როცა პირველ ნაბიჯებს დგას ლოგინიდან ადგომის შემდეგ) და შემდეგ ცოტა უმჯობესდება, მაშინ მაგ ტკივილები შეიძლება იყოს ე.წ. პლანტარული ფასციიტი. თუმცა, ამის გარდა სხვა რამეებმაც შეიძლება მოგცეს ტკვილი. პლანტარულ ფასციიტს უნდა სპეციალური ვარჯიშები ქუსლისათვის.

აგერ დავდებ ვიდეო ლინკს, რომელიც საუბრობს ამ ვარჯიშებზე. უმარტივესი ვარჯიშებია და ყველას შეუძლია გააკეთოს. თანაც, რომელი ვარჯიშჯი რა დროს უნდა გაკეთდეს და რამდენი ხნით, ყველაფერი კარგად არის გადმოცემული:


თუ ვარჯიშები არ შველის, მაშინ შეიძლება გარკვეულ ადგილას ექიმმა გააკეთოს სტეროიდების ინფექცია, რასაც ხშირად შედეგია აქვს. რა თქმა უნდა, ექიმს უნდა ქონდეს ამის გამოცდილება პირველ რიგში.





ძალიან დიდი მადლობა ამ კონსულტაციისთვის...იცით დილით ადგომიდან 20 წუთში უსივდება-უმძიმდება ფეხები..ღამითაც დაწოლისას 15 წუთს ვერ აჩერებს ფეხებს ერთ მდგომარეობაში...ამჯამად ხალხურ მედიცინას მიმართავს სოდის აბაზანებს იკეთებს(რასაც შვება მოაქვს)..და ეს ვარჯიშებიც რომ დაამატოს მემგონი ურიგო არ იქნება ამ კონკრეტულ შემთხვევაში ..კიდევ ერთხელ უღრმესი მადლობა.

Posted by: MITIATO 5 Apr 2010, 01:02
QUOTE
თითქმის ყველანაირ ქრონიკულ დაავადებას შეიძლება ახლდეს ძილიანობა. გარდა ამისა, ეს შეიძლება ახლდეს ძილის ობსტრუქციულ აპნეას (ეს დაავადება ხშირია წონაში მომატებულთათვის, ვინც ხვრინავს ღამით), ქრონიკული დაღლილობის სინდრომს (Chronic fatigue syndrome) და ფიბრომიალგიას.

წონის პრობლემა ნამდვილად არ მაწუხებს. ფიბრომიალგიაც არა მგონია მქონდეს. ძილიანობის გარდა სხვა არც ერთი სიმპტომი არ ემთხვევა. ქრონიკული ფარინგიტი მაქვს და ეს ყველაფერი ამის ბრალი შეიძლება იყოს?
* * *
ქრონიკული დაღლილობის სადიაგნოსტიკო კრიტერიუმებს გადავხედე და ეგეც გამოირიცხა...

Posted by: vano_t 5 Apr 2010, 05:40
MITIATO
QUOTE
წონის პრობლემა ნამდვილად არ მაწუხებს. ფიბრომიალგიაც არა მგონია მქონდეს. ძილიანობის გარდა სხვა არც ერთი სიმპტომი არ ემთხვევა. ქრონიკული ფარინგიტი მაქვს და ეს ყველაფერი ამის ბრალი შეიძლება იყოს?
* * *
ქრონიკული დაღლილობის სადიაგნოსტიკო კრიტერიუმებს გადავხედე და ეგეც გამოირიცხა...
შეიძლება. ნებისმიერმა ქრონიკულმა ინფექციამ შეიძლება მოგცეს ეგ. რადგან ძილიანობა ძალიან არასპეციფიური სიმპტომია და ბევრ დაავადებას ახასიათებს, პირველ რიგში თერაპევტმა უნდა გააკეთოს დეტალური ანამნეზი, ფიზიკური გამოკლვევა და გარკვეული ბაზისური ლაბორატორიები.

Posted by: MITIATO 6 Apr 2010, 01:01
კარგი. ძალიან დიდი მადლობა.
biggrin.gif

Posted by: ზაზა_147 6 Apr 2010, 03:07
vano_t

მოკლედ ჩემს დას აქვს უკვე 3 წელია ალერგია მტვერზე, აქვს ცემინება , ცხვირიდან გამონადენი . ...
თითქმის ყველა ექიმთან იყო, საკმაოდ ბევრაირი მედიკამენტი მიიღო, აცრებსაც იკეთებდა 2 თვის განმავლობაში , მაგრამ არაფერი შველის.
მაინტერესებს თქვენი აზრი, რა შეიძლება იყოს მიზეზი და მკურნალობა რა სქემით უნდა წარიმართოს? ,,,

და მეორე შეკითხვა : ჩემი მეგობარი 5 წელია სვამს ცეტირიზინის ჯგუფის მედიკამენტს , ფაქტიურად დამოკიდებულია ამ წამალზე, რომ არ დალიოს ეწყება ალერგიული სიმტომები: ტანზე გამონაყარი , ხელებზე და მუცელზე უჩნდება ლაქები და ნერვებზეც მოქმედებს. ...
რისი ბრალი შეიძლება იყოს და რა არის გამოსავალი?

წინასწარ დიდი მადლობა.

Posted by: vano_t 6 Apr 2010, 08:12
ზაზა_147
QUOTE
მოკლედ ჩემს დას აქვს უკვე 3 წელია ალერგია მტვერზე, აქვს ცემინება , ცხვირიდან გამონადენი . ...
თითქმის ყველა ექიმთან იყო, საკმაოდ ბევრაირი მედიკამენტი მიიღო, აცრებსაც იკეთებდა 2 თვის განმავლობაში , მაგრამ არაფერი შველის.
მაინტერესებს თქვენი აზრი, რა შეიძლება იყოს მიზეზი და მკურნალობა რა სქემით უნდა წარიმართოს? ,,
მიზეზი ალერგია ყოფილა და, როგორც შენ აღწერ, ალერგიულ რინიტს გავს. ალერგიული რინიტის დროს კარგია ცხვირში ჩასაწვეთებელი ადგილობრივი სტეროიდები. ამათში შედის flonase, nasonex, veramyst. ესენი აშშ-ში გავრცელებული ბრენდებია. გენერიული ვერსიებია ფლუტიკაზონი და მომეტაზონი. გარდა ამისა, არის პერ ოს მისაღები ანტიჰისტამინური წამლებიც (ალეგრა, ზირტეკი, დიმედროლი). ზოგადად ალერგიის მკურნალობა შედგება შემდეგი ეტაპებისაგან: 1) ალერგიის დიაგნოზი და ალერგენის იდენტიფიკაცია; 2) თუ შესაძლებელია ალერგენისაგან თავის არიდება; 3) ანტიჰისტამინური და ადგილობრივი სტეროიდული წამლები; 4) თუ ხდება ალერგენის იდენტიფიკაცია, მაშინ ალერგოლოგი ხანდახან შეეცდება პატარა დოზების შეყვანას და მერე თანდათან გაზრდას-ამას ეძახიან დესენსიტიზაციას; 5) კუნთებში ხანგრძლივი მოქმედების სტეროიდების გაკეთება.

QUOTE
და მეორე შეკითხვა : ჩემი მეგობარი 5 წელია სვამს ცეტირიზინის ჯგუფის მედიკამენტს , ფაქტიურად დამოკიდებულია ამ წამალზე, რომ არ დალიოს ეწყება ალერგიული სიმტომები: ტანზე გამონაყარი , ხელებზე და მუცელზე უჩნდება ლაქები და ნერვებზეც მოქმედებს. ...
რისი ბრალი შეიძლება იყოს და რა არის გამოსავალი?
იგივე პასუხია ამ კითხვაზეც. გამოცდილ ალერგოლოგთან მიყვანაც შესაძლებელია.

თუ ტრადიციული მეთოდები არ შველის, არატრადიციული მეთოდების გამოყენებაც შეიძლება. ამისათვის ამ დარგის სპეციალისტს უნდა მიაკითხო. აქაც არის ბატონი თორნიკე ალაშვილი და იმასაც შეგიძლია კითხო. თუმცა, შენს დამოკიდებულებას ამ მეთოდებისადმი დიდი მნიშვნელობა აქვს ალბათ.

Posted by: ზაზა_147 6 Apr 2010, 11:57
vano_t
დიდი მადლობა ბატონო ვანო.
QUOTE
მიზეზი ალერგია ყოფილა და, როგორც შენ აღწერ, ალერგიულ რინიტს გავს. ალერგიული რინიტის დროს კარგია ცხვირში ჩასაწვეთებელი ადგილობრივი სტეროიდები. ამათში შედის flonase, nasonex, veramyst. ესენი აშშ-ში გავრცელებული ბრენდებია. გენერიული ვერსიებია ფლუტიკაზონი და მომეტაზონი. გარდა ამისა, არის პერ ოს მისაღები ანტიჰისტამინური წამლებიც (ალეგრა, ზირტეკი, დიმედროლი). ზოგადად ალერგიის მკურნალობა შედგება შემდეგი ეტაპებისაგან: 1) ალერგიის დიაგნოზი და ალერგენის იდენტიფიკაცია; 2) თუ შესაძლებელია ალერგენისაგან თავის არიდება; 3) ანტიჰისტამინური და ადგილობრივი სტეროიდული წამლები; 4) თუ ხდება ალერგენის იდენტიფიკაცია, მაშინ ალერგოლოგი ხანდახან შეეცდება პატარა დოზების შეყვანას და მერე თანდათან გაზრდას-ამას ეძახიან დესენსიტიზაციას; 5) კუნთებში ხანგრძლივი მოქმედების სტეროიდების გაკეთება.


ის წამლები რომლებიც ჩამოთვალეთ ყველა აქვს მიღებული , როგორც ადგილობრივი ასევე სისტემური.
4) თუ ხდება ალერგენის იდენტიფიკაცია, მაშინ ალერგოლოგი ხანდახან შეეცდება პატარა დოზების შეყვანას და მერე თანდათან გაზრდას-ამას ეძახიან დესენსიტიზაციას; - ესეც აქვს გაკეთებული 2 კურსი. რაც შეეხება პროფილაქტიკას მაქსიმალურად ცდილობს მაგრამ აბსოლიტურად სტერილურ გარემოში ვერ იცხოვრებს და შესაბამისად მუდმივად აქვს ეს პრობლემა, ჩემს მეგობარს ქონდა ყვავილის მტვერზე ალერგია გაიკეთა დესენსიბილიზაცია და 1 კურსით, აბსოლიტურად მოეხსნა ყველა ის სიმპტომი რომელიც ქონდა.

QUOTE
იგივე პასუხია ამ კითხვაზეც. გამოცდილ ალერგოლოგთან მიყვანაც შესაძლებელია.
თუ ტრადიციული მეთოდები არ შველის, არატრადიციული მეთოდების გამოყენებაც შეიძლება. ამისათვის ამ დარგის სპეციალისტს უნდა მიაკითხო. აქაც არის ბატონი თორნიკე ალაშვილი და იმასაც შეგიძლია კითხო. თუმცა, შენს დამოკიდებულებას ამ მეთოდებისადმი დიდი მნიშვნელობა აქვს ალბათ.


საინტერესო იქნებოდა ბატონი თორნიკეს აზრი ამ საკითხზე. ჰომეოპათიას დიდი ისტორია აქვს და ვფიქრობ მისი უგულებელყოფა გაუმართლებელია მითუმეტეს ჩემი პროფესიის ადამიანისათვის. smile.gif

Posted by: terdzi 6 Apr 2010, 18:22
QUOTE
შეკითხვები ინტერნისტებს(თერაპევტებს)


Posted by: pawking 6 Apr 2010, 18:25
მაინტერესებს, რისი ბრალი შეიძლება იყოს როდესაც ადამიანი შარდავს ძალიან ცოტას და ამავე დროს უსივდება მთლიანი ტანი და აქვს თირკმელების ყრუ ტკივილი? დიდი მადლობა წინასწარ .

Posted by: vano_t 6 Apr 2010, 19:01
pawking
QUOTE
მაინტერესებს, რისი ბრალი შეიძლება იყოს როდესაც ადამიანი შარდავს ძალიან ცოტას და ამავე დროს უსივდება მთლიანი ტანი და აქვს თირკმელების ყრუ ტკივილი? დიდი მადლობა წინასწარ .

როცა ადამიანი შარდავს ძალიან ცოტას და ტანი უსივდება, ეს მიუთითებს თირკმელების ფუნქციის უკმარისობაზე, რასაც თავის მხრივ ძალიან ბევრი რაიმე იწვევს: თირკმელების სხვადასხვა სახის დაავადებები, შარდსაწვეთების და შარდის სადინარის ობსტრუქცია სხვადასხვა მიზეზების გამო, გულის დაავადებები, დაბალი წნევა. ამ ყველაფერს უნდა დიაგნოზი, რასაც უნდა აკეთებდეს თერაპევტი საწყის ეტაპზე და შემდეგ ნეფროლოგი ან ნეფროლოგს შეუყძლია თავიდან ბოლომდე გამოიკვლიოს ავადმყოფი. ასეთ ავადმყოფს ჭირდება ექიმი აუცილებლად. ინტერნეტით არაფერი გამოვა.

Posted by: -LesTaT- 7 Apr 2010, 16:54
ნერვოპათოლოგი ტუ არის ვინმე, მითხარით რა ვქნა
მაქ მგონი საჯდომი ნერვის გაციება .. ექიმის ტქმით. კაი ხანი მქონდა, და მეგონა თვიტონ გამივლიდა ტუ ძალიან რო შემახუწა მერე მივედი, რესკ მოძრაობებზე მქონდა ტკივილიები, და რო ვჯდებოდი დიდი ხანი, სხვა არაფერი, გამომიწერე წამლები 2 კვირაა ვიკეთებ ნემსებს, მარა ავი ისეთი ტკივილები არა მარა გეგონება ფეხზე გადამდის, ფეხის კუნტზე პულსაცია სავით მაქვს ხოლმე და გეგონება მაგას აწვება სიმძიმე სიარულის დროს.

რა გადავიღო ? მირჩიეთ ვისთან მივიდე

Posted by: Workgroup 7 Apr 2010, 17:37
მუხლს ზემოთ ფეხზე კაპილარების გაჩენა რისი ბრალი შეიძლება იყოს???..... შესაძლებელია რაიმე საშუალებით მათი გაქრობა??

Posted by: SPC 8 Apr 2010, 09:51
დაბალ სიცხიანი გრიპი არსებობს? 37-37.1 ?


ხუთშაბათს ყელი ამტკივდა, მერე ყელმა გამიარა და იმის მერე მახველბს სულ, რაც მაინტერესებს ისაა რომ სიცხე სულ 37. -37.1, ან 36.9 მაქვს ......ეს რანაირი გრიპია? თუ გრიპი არაა და რამე გართულებაა თუ რა არის საერთიდ? გრიპის დროს მე ვიცი რომ უფრო მაღალი სიცხეებია .

Posted by: Nasty 8 Apr 2010, 15:07
მოკლედ ხმა მაქვს დაკარგული უკვე რამოდენიმე კვირაა, 3 ალბათ
არც ბევრი მიმღერია, არც მიყვირია არც მინერვიულია, არც გაციების ნიშნები მაქვს. ზოგადად სუსტი ყელი და იოგები მაქვს, მაგრამ პირველად "ხავილი" დამეწყო ბერლინის აეროპორტში, ძალიან დიდი თოვლი მოვიდა და ციოდა, სტამბულში კი სიცხე იყო და ვიფიქრე აკლიმატიზაციის ან ტემპერატურის ცვლილების გამო წამივიდა ხმა მეთქი, მაგრამ აქამდე ხომ უნდა აღმდგარიყო?
იქნება მირჩიოთ რამე? ან ვინმე? ყელ-ყურთან უნდა მივიდე?

Posted by: vano_t 8 Apr 2010, 19:22
Workgroup
QUOTE
მუხლს ზემოთ ფეხზე კაპილარების გაჩენა რისი ბრალი შეიძლება იყოს???..... შესაძლებელია რაიმე საშუალებით მათი გაქრობა??

არ ვიცი რა კაპილარებზე საუბრობ. უფრო სავარაუდოა ვენების ვარიკოზული გაგანიერება. შეიძლება ტელეანგიექტაზიებიც იყოს.

user posted image
ეს არის ვარიკოზები და თუ ამის მსგავსია, ე.ი. ვარიკოზებია. ამათი მოცილება კი შეიძლება სხვადახვა ქირურგიული და არაქირურგიული მეთოდებით. ანგიოლოგი აკეთებს ამას.

SPC
QUOTE
დაბალ სიცხიანი გრიპი არსებობს? 37-37.1 ?


ხუთშაბათს ყელი ამტკივდა, მერე ყელმა გამიარა და იმის მერე მახველბს სულ, რაც მაინტერესებს ისაა რომ სიცხე სულ 37. -37.1, ან 36.9 მაქვს ......ეს რანაირი გრიპია? თუ გრიპი არაა და რამე გართულებაა თუ რა არის საერთიდ? გრიპის დროს მე ვიცი რომ უფრო მაღალი სიცხეებია .
გრიპს მაღალი სიცხეები ახასიათებს; გარდა ამისა, გრიპისათვის დამახასიათებელია თავის ტკივილები, კუნთებში მტვრევა და ტკივილები, სისუსტე. შენ რაც გაქვს, ეგ არის უბრალოდ გაციება. ინგლისურად ქვია common cold და კიდევ მაგას უწოდებენ ქემო სასუნთქვი გზების ინფექციას (ვირუსული ინფექციაა და თავისით რჩება).

Nasty
QUOTE
მოკლედ ხმა მაქვს დაკარგული უკვე რამოდენიმე კვირაა, 3 ალბათ
არც ბევრი მიმღერია, არც მიყვირია არც მინერვიულია, არც გაციების ნიშნები მაქვს. ზოგადად სუსტი ყელი და იოგები მაქვს, მაგრამ პირველად "ხავილი" დამეწყო ბერლინის აეროპორტში, ძალიან დიდი თოვლი მოვიდა და ციოდა, სტამბულში კი სიცხე იყო და ვიფიქრე აკლიმატიზაციის ან ტემპერატურის ცვლილების გამო წამივიდა ხმა მეთქი, მაგრამ აქამდე ხომ უნდა აღმდგარიყო?
იქნება მირჩიოთ რამე? ან ვინმე? ყელ-ყურთან უნდა მივიდე?
ამ ეტაპზე არავისთან მისვლ არ გჭირდება. შენ ალბათ ვირუსული ლარინგიტი გაქვს. ხანდახან ვირუსის მოცილებიდან დაკარგული ხმა რამოდენიმე კვირა შეიძლება გაგრძელდეს. შეგიძლია 1-2 კვირა დააკვირდე და თუ არ გაუმჯობესდი, მაშინ მიდი ყელყურის ექიმთან. მანამდე აარაიდე თავი ყველაფერს, აც კიდევ უფრო გააღიზიანებს ყელს და ხმის იოგებს (სიგარეტი, კვამლი და ა.შ.)

Posted by: SPC 8 Apr 2010, 20:12
vano_t
დიდი მადლობა
......

Posted by: Workgroup 8 Apr 2010, 22:40
vano_t
ფოტოზე ალბათ გართულებული ფორმაა..მაგრამ გავს.....შეიძლება მაგ ვენების გაგანიერებას მაღალ ქუსლებზე სიარული იწვევდეს ან ვარჯიში?

Posted by: kriketi-1985 9 Apr 2010, 02:24
არ ვიცი რატომ, მაგრამ დამებადა შეკითხვა, რომ მჭირს ფრონტიტი, შუბლზე სიცივეს ვერ ვიტან, მაშIნვე თავის ტკივილი მეწყება, როგორც ვიცი, ფრონტიტს ახასიათებს ცხვირიდან გამონადენი და მასთან ერთად უსიამოვნო სუნი, მაგრამ ეს არ ხდება ჩემს შემთხვევაში, ოდნავ სისველე მაქვს, ქუდის დახურების მერე კი მითბება შუბლი და თავის ტკივილიც გამივლის ხოლმე... ეს მხოლოდ სიცივის ბრალია თუ ფრონტიტი მაქვს? spy.gif თვალის ბუდის ტკივილიც სცოდნია და ასე ვარ მეც, თვალის პრობლემები პატარაობიდან კი მაქვს... წნევაც მეცემა ხოლმე ამ დროს...
გმადლობთ წინასწარ, ვინც მიპასუხებს...

Posted by: mincepie 9 Apr 2010, 05:22
როგორც ჰიპოქონდრიაში შემჩნეული ადამიანი, ძალიან გთხოვთ, ჩამახშეთ თუ ეს სერიოზულად არ ჯღერს smile.gif, მაგრამ ამჯერად მე მგონი მართლა რაღაც დავიმართე:
დაახლოებით ერთი წელია შეზღუდულად ვიკვებები - ფაქტიურად ჯანსაღ, უმი საკვების, დიეტაზე ვარ და 10 კილოზე მეტი დავიკელი. ამხელა წონის დაკლების გამო, გარდა ჰორმონალური და ათასი 'სასიამოვნო' პრობლემებისა ამ ბოლო დროს დამეწყო კიდევ უფრო გადასარევი სიმპტომები: მშივდება უგონოდ, პირდაპირ სადილის ან საუზმის მერეც, მეღვიძება 4-5 ხანდახან 6 საათი ძილის მერე. თაბრუსხვევა, ბუნდოვანი მხედველობა, ყველაფერი მაქვს რაც სისხლში დაბალ შაქარს ახასიათებს. ჰოდა რა უნდა ვქნა არ ვიცი - ექიმთან უნდა წავიდე? თუ ვჭამო ყოველ ნახევარ საათში როცა თავბრუსხვევა დამეწყება? ან იქნებ სულაც არ მაქვს დაბალი შაქარი და გაზაფხულის ბრალია?
დიდი მადლობა წინასწარ ვინც მიპასუხებს. საქართველოში არ ვარ და სანამ აქაურ სამთვიან რიგში ჩავეწერები ექიმთან ვინმეს რჩევა მაინტერესებდა.

Posted by: vano_t 9 Apr 2010, 20:30
Workgroup
QUOTE
ფოტოზე ალბათ  გართულებული ფორმაა..მაგრამ გავს.....შეიძლება  მაგ ვენების გაგანიერებას მაღალ ქუსლებზე სიარული იწვევდეს ან ვარჯიში?

ვარიკოზული ვენების მიზეზად ეგენი არ ითვლება.



mincepi
QUOTE
როგორც ჰიპოქონდრიაში შემჩნეული ადამიანი, ძალიან გთხოვთ, ჩამახშეთ თუ ეს სერიოზულად არ ჯღერს smile.gif, მაგრამ ამჯერად მე მგონი მართლა რაღაც დავიმართე:
დაახლოებით ერთი წელია შეზღუდულად ვიკვებები - ფაქტიურად ჯანსაღ, უმი საკვების, დიეტაზე ვარ და 10 კილოზე მეტი დავიკელი. ამხელა წონის დაკლების გამო, გარდა ჰორმონალური და ათასი 'სასიამოვნო' პრობლემებისა ამ ბოლო დროს დამეწყო კიდევ უფრო გადასარევი სიმპტომები: მშივდება უგონოდ, პირდაპირ სადილის ან საუზმის მერეც, მეღვიძება 4-5 ხანდახან 6 საათი ძილის მერე. თაბრუსხვევა, ბუნდოვანი მხედველობა, ყველაფერი მაქვს რაც სისხლში დაბალ შაქარს ახასიათებს. ჰოდა რა უნდა ვქნა არ ვიცი - ექიმთან უნდა წავიდე? თუ ვჭამო ყოველ ნახევარ საათში როცა თავბრუსხვევა დამეწყება? ან იქნებ სულაც არ მაქვს დაბალი შაქარი და გაზაფხულის ბრალია?
დიდი მადლობა წინასწარ ვინც მიპასუხებს. საქართველოში არ ვარ და სანამ აქაურ სამთვიან რიგში ჩავეწერები ექიმთან ვინმეს რჩევა მაინტერესებდა.
ექიმთან მიდი. როცა ბევრი სხვადასხვა და არასპეციფიური სიმპტომი გაქვს ერთად, ამის მიზეზი ბევრი რამ შეიძლება იყოს. მიზეზის დასადგენად დაწვრილებით გამოკითხვა და გამოკვლევაა საჭირო.

kriketi-1985
QUOTE
არ ვიცი რატომ, მაგრამ დამებადა შეკითხვა, რომ მჭირს ფრონტიტი, შუბლზე სიცივეს ვერ ვიტან, მაშIნვე თავის ტკივილი მეწყება, როგორც ვიცი, ფრონტიტს ახასიათებს ცხვირიდან გამონადენი და მასთან ერთად უსიამოვნო სუნი, მაგრამ ეს არ ხდება ჩემს შემთხვევაში, ოდნავ სისველე მაქვს, ქუდის დახურების მერე კი მითბება შუბლი და თავის ტკივილიც გამივლის ხოლმე... ეს მხოლოდ სიცივის ბრალია თუ ფრონტიტი მაქვს? spy.gif თვალის ბუდის ტკივილიც სცოდნია და ასე ვარ მეც, თვალის პრობლემები პატარაობიდან კი მაქვს... წნევაც მეცემა ხოლმე ამ დროს...
გმადლობთ წინასწარ, ვინც მიპასუხებს...
არ გავს მაგი ფრონტიტს. სიცივის ბრალი შეიძლება იყოს. დაიხურ ქუდი თუ პრობლემას ხსნის smile.gif არაფერი ჭირდება მაგას ჩემი აზრით.

Posted by: LUKA-BRAZI 9 Apr 2010, 21:09
vano_t
ვანო, დიდი მადლობა smile.gif

Posted by: Workgroup 10 Apr 2010, 00:08
vano_t
გასაგებია..მადლობთ smile.gif))

Posted by: kriketi-1985 10 Apr 2010, 02:08
vano_t
მეც გიხდი მადლობას smile.gif

Posted by: E N J I 10 Apr 2010, 20:51
vano_t
თუ შეიძლება მითხარით სისხლში დიდი რაოდენობით ლეიკოციტები რისგან შეიძლება იყოს გამოწვეული?

Posted by: Nasty 10 Apr 2010, 20:54
QUOTE
NastyQUOTE
მოკლედ ხმა მაქვს დაკარგული უკვე რამოდენიმე კვირაა, 3 ალბათ
არც ბევრი მიმღერია, არც მიყვირია არც მინერვიულია, არც გაციების ნიშნები მაქვს. ზოგადად სუსტი ყელი და იოგები მაქვს, მაგრამ პირველად "ხავილი" დამეწყო ბერლინის აეროპორტში, ძალიან დიდი თოვლი მოვიდა და ციოდა, სტამბულში კი სიცხე იყო და ვიფიქრე აკლიმატიზაციის ან ტემპერატურის ცვლილების გამო წამივიდა ხმა მეთქი, მაგრამ აქამდე ხომ უნდა აღმდგარიყო?
იქნება მირჩიოთ რამე? ან ვინმე? ყელ-ყურთან უნდა მივიდე?

ამ ეტაპზე არავისთან მისვლ არ გჭირდება. შენ ალბათ ვირუსული ლარინგიტი გაქვს. ხანდახან ვირუსის მოცილებიდან დაკარგული ხმა რამოდენიმე კვირა შეიძლება გაგრძელდეს. შეგიძლია 1-2 კვირა დააკვირდე და თუ არ გაუმჯობესდი, მაშინ მიდი ყელყურის ექიმთან. მანამდე აარაიდე თავი ყველაფერს, აც კიდევ უფრო გააღიზიანებს ყელს და ხმის იოგებს (სიგარეტი, კვამლი და ა.შ.)


მადლობა პასუხისთვის,
ორ კვირაზე მეტი არის რაც ასე მაქ ხმა, ხან ვითომ უკეთ ხან უარესად. პარასკევს მისული ვიყავი ექიმთან და ორშაბათს დამიბარა სხვა კლინიკაში, აქ იოგებში რომ ჩაგხედო ეგ აპარატი გაპუჭდაო.. იმედია სერიოზული არაფერია

Posted by: E N J I 10 Apr 2010, 20:59
და ქრონიკულმა ტონზილიტმა თუ შეიძლ;ება ეს გამოიწვიოს? ანუ ლეიკოციტების და როეს საკმაო მომატება
წინასწარ მადლობა

Posted by: vano_t 11 Apr 2010, 01:28
E N J I
QUOTE
და ქრონიკულმა ტონზილიტმა თუ შეიძლ;ება ეს გამოიწვიოს? ანუ ლეიკოციტების და როეს საკმაო  მომატება
წინასწარ მადლობა

რამხელაზეა მომატებული ლეიკოციტები? რამდენი პროცენტი ნეტიროფილებია, ბაზოფილებია, ჩხირბირთვიანია, ეოზინოფილებია და კიდევ სხვა რაიმე სახის უჯრედები თუ არიას? სხვა რაიმე სიმპტომები თუ გაქვს?

ისე ლეიკოციტოზი ათასობით დაავადებას ახასიათებს და მარტო მისი მომატებით ვერაფერ დიაგნოზს ვერ დასვამ. ზოგჯერ ადამიანს შეიძლება მომატებული ქონდეს როგორც მისთვის ნორმა.


Posted by: E N J I 11 Apr 2010, 21:08
vano_t
ლეიკოციტებია 10200, ,მგონი ნორმა 9000ია
როე 27
დანარჩენი ყველაფერი წესრიგშია
თერაპევტმა ნახა ყელი და გლანდების ბრალია ქრონიკული ტონზილიტი გაქვსო
სულ სიცხე მაქვს ბოლო ერთი თვეა, სისუსტე და პულსი აჩქარებული მაქვს, მალე ვიღლები, სიმხურვალეს ვგრძნობ და ეს ანთება როა იმის ბრალიაო.ძაან შეშფოთებული ვარ რამე სხვა ხო არ მჭირს?
თუ მარტო გლანდების ბრალია ზეგ თოდუს კლინიკაში მივდივარ და ოპერაციას გავიკეთებ

Posted by: vano_t 12 Apr 2010, 08:31
E N J I
QUOTE
ლეიკოციტებია 10200, ,მგონი ნორმა 9000ია
როე 27
დანარჩენი ყველაფერი წესრიგშია
თერაპევტმა ნახა ყელი და გლანდების ბრალია ქრონიკული ტონზილიტი გაქვსო
სულ სიცხე მაქვს ბოლო ერთი თვეა, სისუსტე და პულსი აჩქარებული მაქვს, მალე ვიღლები, სიმხურვალეს ვგრძნობ და ეს ანთება როა იმის ბრალიაო.ძაან შეშფოთებული ვარ  რამე სხვა ხო არ მჭირს?
თუ მარტო გლანდების ბრალია ზეგ თოდუს კლინიკაში მივდივარ და ოპერაციას გავიკეთებ

რამხელა სიცხეები გაქვს? რამდენი წლის ხარ? PM-ში შეგიძლია მომწერო, თუ აქ გერიდება.

ლეიკოციტების მაგხელა მომატება არაფერს არ ნიშნავს-ეგ ნორმაა ფაქტიურად. შეუძლია ექიმს რამოდენიმეჯერ გადაამოწმოს ყოველ სამ თვეში ერთხელ. თუ ნორმის ზედა ზღვართან ახლოს ხარ ან ცოტათი აცილებული ხარ ნორმის ზედა ზღვარს, მაშინ ეგ არაფერია. თანაც, ნორმის დადგენის საქმე საქართველოში როგორ არის არ ვიცი. იმ საავადმყოფოში სადაც მე ვმუშაობ, ნორმის ზედა ზღვრად ითვლება 11 000.

Posted by: roamer 12 Apr 2010, 18:00
ერთი კითხვა მაქვს და იქნებ მიპასუხოს ვინმემ
თუ წინა საღამოს კარგად დავნაყრდი,მეორე დღეს სახე მაქვს შეშუპებული; დიდი რაოდენობის სითხის მიღების შემთხვევაში მასე არ მემართება. შეიძლება თირკმელის ბრალი იყოს?

Posted by: marto 12 Apr 2010, 21:48
კითხვა მაქვს თუ შეიძლება

გამუდმებით მტკივა სახსრები თუ კუნთები, დილითაც რომ ვიღვიძებ ისეთი შეგრძნება მაქვს თითქოსს წინა დღეს ძალიან ბევრი ვიარე და ყველა კუნთი დაღლილი მაქვს და მტკივა.
თერაპევტთან ვიყავი და ტებანტინი დამინიშნა, მითხრა ეგ ნევრატიული ამბავიაო. წესიერად ვერ გავიგე რა არის. ერთი თვე ვსვი ეს წამალი, მერე მივედი და კიდევ მტკივათქო რომ ვუთხარი კიდევ ერთი თვე გამიგრძელა წამლის მიღება. ანალზიები წესრიგშია, მარტო რკინა მქონდა ცოტა დაბალი და წამლებს ვღებულობ. რისი ბრალი შეიძლება იყოს ეს კუნთების ტკივილი? ნევროპათოლოგთან ხომ არ მივსულიყავი?

Posted by: Nasty 12 Apr 2010, 22:58
ხალხნო, Sulcus ხომ არ იცით რა არის? მე კი ვიცი დაახლოებით იოგების ამბებშია, მაგრამ მეტი ინფორმაცია მაინტერესებს და wikipedia-ში ძალიან მწირი ინფორმაცია იყო ამაზე.

მადლობა წინასწარ

Posted by: womanly 13 Apr 2010, 11:02
კითხვა მაქვს ბავშვს მინდა გავუკეთო ანალიზი ჭიებზე და სად აკეთებენ? აუცილებელია თუ არა მისი წაყვანა? მ,ადლობთ წინასწარ

Posted by: marto 13 Apr 2010, 16:23
სად არიან ექიმები, არ გვპასუხობენ? sad.gif(((

Posted by: E N J I 13 Apr 2010, 17:10
ვერაფერი ვერ გავიგე
დღეს ვიყავი თოდუას კლინიკაში და არაფერი არ გჭირსო,არც გლანდები გაქვს ამოსაჭრელიო,ამ ორ სიტყვაში 30 ლარი გადავიხადე და წამოვედი, ეხლა კი ისევ სიცხე მაქვს
sad.gif sad.gif

Posted by: roamer 13 Apr 2010, 17:19
ვერავინ მიპასუხებს? sad.gif ..............

Posted by: E N J I 13 Apr 2010, 17:28
QUOTE
სად არიან ექიმები, არ გვპასუხობენ?

ძინავთ აშკარად

Posted by: marto 13 Apr 2010, 17:49
სიცხე თუ გაქვს რაღაც ანთებითი პროცესია, ალბათ sad.gif
მიკვირს რომ გლანდები არაა

Posted by: BadbadGirl 13 Apr 2010, 21:56
ხალხებო, კანჭის (ორივე ფეხზე) ძვლები მტკივა.
დავრბოდი ჩვეულებრივ, დაზიანება არ მქონია, ტკივილიც აქამდე არ მქონია. მაგრამ ახლა მტკივა და ვერც დავრბივარ მეორე დრეა უკვე (ყოველ დღე დავრბივარ) სიარულის დროსაც დისკომფორტს მიქმნის.

რისი ბრალი შეიძლება იყოს? და რა ვქნა?

Posted by: vano_t 14 Apr 2010, 00:29
roamer
QUOTE
ერთი კითხვა მაქვს და იქნებ მიპასუხოს ვინმემ
თუ წინა საღამოს კარგად დავნაყრდი,მეორე დღეს სახე მაქვს შეშუპებული; დიდი რაოდენობის სითხის მიღების შემთხვევაში მასე არ მემართება. შეიძლება თირკმელის ბრალი იყოს?

თირკმლის გამო თუ არის შეშუპება, მაშინ ეს შეშუპება გრქონიკულია. თუ ასე უცებ მოვიდა, არა მგონია თირკმელი იყოს. შეშუპებას სხვა რამეებიც იწვევენ: უძილობა, დაღლილობა, ალერგიული რეაქციები, ღვიძლის დაავადებები, გულიც.

თირკმელი, ღვიძლი და გული ადვილად შეიძლება გამოირიცხოს მარტივი ლაბორატორიებით და ფიზიკური გამოკვლევით.

marto
QUOTE
კითხვა მაქვს თუ შეიძლება

გამუდმებით მტკივა სახსრები თუ კუნთები, დილითაც რომ ვიღვიძებ ისეთი შეგრძნება მაქვს თითქოსს წინა დღეს ძალიან ბევრი ვიარე და ყველა კუნთი დაღლილი მაქვს და მტკივა.
თერაპევტთან ვიყავი და ტებანტინი დამინიშნა, მითხრა ეგ ნევრატიული ამბავიაო. წესიერად ვერ გავიგე რა არის. ერთი თვე ვსვი ეს წამალი, მერე მივედი და კიდევ მტკივათქო რომ ვუთხარი კიდევ ერთი თვე გამიგრძელა წამლის მიღება. ანალზიები წესრიგშია, მარტო რკინა მქონდა ცოტა დაბალი და წამლებს ვღებულობ. რისი ბრალი შეიძლება იყოს ეს კუნთების ტკივილი? ნევროპათოლოგთან ხომ არ მივსულიყავი?
პირველ რიგში უნდა მიხვიდე ნორმალურ თერაპევტთან. სახსრის და კუნთების ტკივილებს ბევრი რაიმე იწვევს. ამის დიაგნოზს ჭირდება გარკვეული ალბორატორიები, გამოკითხვა და გასინჯვა (უნდა გამოირიცხოს რევმატოლოგიური პრობლემები და მიოზიტები)

Nasty
QUOTE
ხალხნო, Sulcus ხომ არ იცით რა არის? მე კი ვიცი დაახლოებით იოგების ამბებშია, მაგრამ მეტი ინფორმაცია მაინტერესებს და wikipedia-ში ძალიან მწირი ინფორმაცია იყო ამაზე.

მადლობა წინასწარ
Sulcus ლათინური სიტყვაა და ნიშნავს ღარს. ღარები ბევრ რაიმეზე შეიძლება იყოს ანატომიურად.

womanly
QUOTE
კითხვა მაქვს ბავშვს მინდა გავუკეთო ანალიზი ჭიებზე და სად აკეთებენ? აუცილებელია თუ არა მისი წაყვანა? მ,ადლობთ წინასწარ
მე არ ვიცი, ვინმე სხვას ეცოდინება და გეტყვის.

BadbadGirl
QUOTE
ხალხებო, კანჭის (ორივე ფეხზე) ძვლები მტკივა.
დავრბოდი ჩვეულებრივ, დაზიანება არ მქონია, ტკივილიც აქამდე არ მქონია. მაგრამ ახლა მტკივა და ვერც დავრბივარ მეორე დრეა უკვე (ყოველ დღე დავრბივარ) სიარულის დროსაც დისკომფორტს მიქმნის.

რისი ბრალი შეიძლება იყოს? და რა ვქნა?
დაისვენე და კარგად იქნები. ალბათ shin splint არის (ქართულად არ ვიცი რა ქვია). გადატვირთვის შემდეგ იცის. თუ ტკივილმა არ გაგიარა 1-2 კვირაში, მაშინ ექიმი ნახე, რომ ფარული სტრესის მოტეხილობა გამორიცხოს (stress fracture, occult fracture).

Posted by: Solveig 14 Apr 2010, 00:53
womanly
QUOTE
კითხვა მაქვს ბავშვს მინდა გავუკეთო ანალიზი ჭიებზე და სად აკეთებენ? აუცილებელია თუ არა მისი წაყვანა? მ,ადლობთ წინასწარ

წესით, მაგას ადრე უბნის პოლიკლინიკებში აკეთებდნენ.

თუ სიმპტომები ჰქონდა ბავშVს და პოლიკლინიკა ვერაფერს უხერხებდა, მერე აპრაზიტოლოგიის ინსტიტუტში გზავნიდნენ, მაგრამ ახლა მგონი აღარც არსებობს ეგ ინსისტიტუტი.

გააჩნია, რაზე გინდა ბავშვის გამოკვლევა...და რატომ გაქვს ეჭვი, რომ ჭიით ინვაზია აქვს? იქნებ ჯერ ექიმთან წაგეყვანა და იმას გადაეწყვიტა-რისი ბრალი შეიძლება იყოს?

რაც შეეხება თან წაყვანას: თუ ენტერობიოზზე (მახვილა ჭიაზე) გინდა გამოკვლევა, ბავშვი აუცილებლად თან უნდა წაიყვანო, რადგან მაგის ანალიზს მწებავი ლენტით უღებენ..თუ სხვა ჭიებზეა ეჭვი (მაგალითად, ასკარიდოზი), მაშინ არაა აუცილებელი წაყვანა.

Posted by: womanly 14 Apr 2010, 10:41
Solveig
ხო ადრე გავუკეთე ოგონდ თან მყავდა წაყვანილი ეხლა დიდი ბიჭია 8 წლის და არ წამოვალოოო ასე მგონია რომ აქვს ჭიები ღამით უსველდება ბალიში და დილით გული ერევა ხოლმე ან შეგრძნება აქვს. ერევა იმ შემთხვევში თუ ვარჯიში აქვს ფეხბურთზე ვარჯიშობს. ნეტა ვერამოქსი დაუნიშნეს მაშინ და ეხლაც რომ დავალევინო მეთვითონ? თუ არ შეიძლება?

Posted by: Nasty 14 Apr 2010, 12:01
vano_t
QUOTE
NastyQUOTE
ხალხნო, Sulcus ხომ არ იცით რა არის? მე კი ვიცი დაახლოებით იოგების ამბებშია, მაგრამ მეტი ინფორმაცია მაინტერესებს და wikipedia-ში ძალიან მწირი ინფორმაცია იყო ამაზე.

მადლობა წინასწარ

Sulcus ლათინური სიტყვაა და ნიშნავს ღარს. ღარები ბევრ რაიმეზე შეიძლება იყოს ანატომიურად.


ამ შემთხვევაში საუბარია იოგების ღარზე. ხმა მაქვს დაკარგული რამოდენიმე კვირაა და მაგ ღარზე არსებული კისტა ახშობსო და საოპერაციოაო sad.gif

Posted by: E N J I 14 Apr 2010, 19:43
womanly
QUOTE
კითხვა მაქვს ბავშვს მინდა გავუკეთო ანალიზი ჭიებზე და სად აკეთებენ? აუცილებელია თუ არა მისი წაყვანა? მ,ადლობთ წინასწარ

ბაქტერიოციდულში აკეთებენ ყველანაირ ანალიზს

Posted by: mangustik 16 Apr 2010, 14:54
ტკივილი იღლიაში რისი ბრალი შეიძლება იყოს? საშინლად მტკივა უკვე 3 დღეა, მანამდე ერთი კვირა დროდადრო მტკიოდა მსუბუქად, ახლა არა აქვს მნიშვნელობა ვამოძრავებ თუ არა მაინც მტკივა, ან რის ექიმთან უნდა მივიდე?

Posted by: E N J I 16 Apr 2010, 15:40
mangustik
QUOTE
ტკივილი იღლიაში რისი ბრალი შეიძლება იყოს? საშინლად მტკივა უკვე 3 დღეა, მანამდე ერთი კვირა დროდადრო მტკიოდა მსუბუქად, ახლა არა აქვს მნიშვნელობა ვამოძრავებ თუ არა მაინც მტკივა, ან რის ექიმთან უნდა მივიდე?

აუცილებლად მიდი, ლიმფური ჯირკვალი ხო არ გაქ გადიდებული?

Posted by: mangustik 16 Apr 2010, 15:56
არა მგონი, ხელით ვერაფერს ვერ ვგრძნობ, გაციებული ვარ და შეიძლება მაგის ბრალი იყოს? თუმცა ტკივილები 3-4 დღით ადრე დამეწყო

Posted by: @ucnobinacnobi@ 16 Apr 2010, 20:11
კომპიუერს რომ ვუზივარ ცოტა ხანში თავის ტკივილი და გულისრევის შეგრძნება მეწყება. ადრე არ მქონა ეს სიმპტომები, ამ ბოლო ხანს დამცყო. რისი ბრალია ზუსტად ხომ ვერ მეტყვით და რით გამოვასწორო, თან გაციებულიც ვარ.
(შეიძლება ამი ბრალიც იყოს)

Posted by: vano_t 17 Apr 2010, 02:33
mangustik
QUOTE
ტკივილი იღლიაში რისი ბრალი შეიძლება იყოს? საშინლად მტკივა უკვე 3 დღეა, მანამდე ერთი კვირა დროდადრო მტკიოდა მსუბუქად, ახლა არა აქვს მნიშვნელობა ვამოძრავებ თუ არა მაინც მტკივა, ან რის ექიმთან უნდა მივიდე?

თერაპევთტან მიდი, კარგ თერაპევტთან. გასინჯივ და გამოკითხის გარეშე ძნელია რაიმეს თქმა. შეიძლება არც არაფერია.

@ucnobinacnobi@
QUOTE
კომპიუერს რომ ვუზივარ ცოტა ხანში თავის ტკივილი და გულისრევის შეგრძნება მეწყება. ადრე არ მქონა ეს სიმპტომები, ამ ბოლო ხანს დამცყო. რისი ბრალია ზუსტად ხომ ვერ მეტყვით და რით გამოვასწორო, თან გაციებულიც ვარ.
(შეიძლება ამი ბრალიც იყოს)
შეიძლება გაციების ბრალიც არის. დაანებე ცოტა ხანი კომპიტერს თავი, დაისვენე, სითხეები მიიღე (წყალი ან ნატურალური წვენები) და კარგად იქნები.

Posted by: E N J I 17 Apr 2010, 12:17
2 კვირაა სიცხე მაქვს ეს რა უბედურებაა
ამისთანა რა ვირუსია
მოვკვდი წამლებით და ნემსებით და შედეგი 0 mad.gif

Posted by: giogiorgi12 18 Apr 2010, 00:19
ამ ბოლო დროს ძილის წინ დამჩემდა პეხის გულების ფხანა გაუთავებლად მეფხანება და რამე ხომ არ შეიძლება იყოს ?

Posted by: vano_t 18 Apr 2010, 08:23
giogiorgi12
QUOTE
ამ ბოლო დროს ძილის წინ დამჩემდა პეხის გულების ფხანა გაუთავებლად მეფხანება და რამე ხომ არ შეიძლება იყოს ?

ფულს იშოვი smile.gif

ჯანმრთელობის ამბავში, თუ რაიმე გამონაყარი არ გაქვს, არ უნდა იყოს პრობლემა. თუ გამოანაყარი გაქვს, მაშინ ყველაზე ხშირი პრობლემა მაქ არის სოკოვანი დაავადება, ე.წ. "ათლეტის ტერფი" (athletes foot).

Posted by: giogiorgi12 18 Apr 2010, 10:46
QUOTE
ჯანმრთელობის ამბავში, თუ რაიმე გამონაყარი არ გაქვს, არ უნდა იყოს პრობლემა. თუ გამოანაყარი გაქვს, მაშინ ყველაზე ხშირი პრობლემა მაქ არის სოკოვანი დაავადება, ე.წ. "ათლეტის ტერფი" (athletes foot).

არანაირი გამონაყარი არ მაქ smile.gif
მადლობ პასუხისთვის

Posted by: ccmm 19 Apr 2010, 17:09
შეკითხვა მაქვს:

კუჭ-ნაწლავის (უფრო ნაწლავების) დაავადებებთან შეიძლება რამე კავშირში იყოს თავის ტკივილი?

წინასწარ მადლობა პასუხისთვის...

Posted by: vano_t 19 Apr 2010, 19:13
ccmm
QUOTE
კუჭ-ნაწლავის (უფრო ნაწლავების) დაავადებებთან შეიძლება რამე კავშირში იყოს თავის ტკივილი?

შეიძლება. მაგალითისათვის, მწვავე გასტროენტერიტისათვის არის დამახასიათებელი თავის ტკივილები. კრონის დაავადებასაც შეიძლება ახასიათებდეს თავის ტკივილები.

Posted by: Mako Subladze 20 Apr 2010, 15:12
მუხლი მტკივა.
ჯდომისას მიძლიერდება.
ვის მივმართო?

Posted by: Bridgeman 20 Apr 2010, 15:37
ლუდს, როგორც კი დავლევ, არა აქვს მნიშვნელობა ბევრს თუ ცოტას, აი ძაან ხშირად მინდება ფს ფს givi.gif
და რისი ბრალია ნეტა?

ხო სწორ ადგილას ვიკითხე?

Posted by: vano_t 20 Apr 2010, 19:52
Mako Subladze
QUOTE
მუხლი მტკივა.
ჯდომისას მიძლიერდება.
ვის მივმართო?

ორთოპედს.

Bridgeman
QUOTE
ლუდს, როგორც კი დავლევ, არა აქვს მნიშვნელობა ბევრს თუ ცოტას, აი ძაან ხშირად მინდება ფს ფს givi.gif
და რისი ბრალია ნეტა?
ფს ფს ლუდის მერე ნორმაა.

Posted by: Bridgeman 20 Apr 2010, 21:59
vano_t
QUOTE
ფს ფს ლუდის მერე ნორმაა. 

ერთხელ, ორჯერ, სამჯერ, ოთხჯერ,მარა 1000-ჯერ?
ძაან მალ-მალე მინდება ხოლმე sad.gif

Posted by: vano_t 20 Apr 2010, 23:14
Bridgeman
QUOTE
ერთხელ, ორჯერ, სამჯერ, ოთხჯერ,მარა 1000-ჯერ?
ოთხჯერ ისეც ნორმაა, ლუდის გარეშე. 1000-ჯერ ძაან გაზვიადებულია (ყოველ 1 წუთში და 40 წამში უნდა დარბოდე). 10-20 ჯერ შეიძლება გახვიდე დღის განმავლობაში ლუდის მერე. ყოველ 15-20 წუთში შეიძლება გასვლა ამ დროს. ლუდს აქვს შარდმდენი (საფისიფისე biggrin.gif) თვისებები, ეგ ბევრს ემართება. მარტო შენ არ ხარ ორიგინალური ამ საქმეში smile.gif

Posted by: bachachuna 21 Apr 2010, 11:53
გამარჯობათ, არ ვიცი ზუსტად ვის უნდა მივმართო, მაგრამ მაინტერესებს, ჭინჭარი იწვევს თუ არა მუცლის შებერილობას?

Posted by: Bridgeman 21 Apr 2010, 14:12
vano_t
QUOTE
ყოველ 15-20 წუთში შეიძლება გასვლა ამ დროს.

ხო აი ასე დაახლოებით ნახევარი საათის ინტერვალით..და თუ მარშრუტკაში ვარ და ვჯღანჯღარებ..ოოოიიი biggrin.gif

Posted by: ninoeqimi 21 Apr 2010, 18:19
გაინტერესებთ რა არის ღამით შარდის შეუკავებლობა ანუ ენურეზი? ნახეთ ლინკი
http://medgeo.net/2010/04/17/enuresis/
გაინტერესებთ რამდენად სახიფათოა ჩაბჟირება ანუ სუნთქვის შეჩერების შეტევები?
http://medgeo.net/2010/04/13/baby-8/

Posted by: vano_t 22 Apr 2010, 08:25
bachachuna
QUOTE
გამარჯობათ, არ ვიცი ზუსტად ვის უნდა მივმართო, მაგრამ მაინტერესებს, ჭინჭარი იწვევს თუ არა მუცლის შებერილობას?

კითხვა ვერ გავიგე. ნაჭინჭრზე გაჩნდა მუცლის შებერილობა თუ როგორ?

Posted by: shtori 23 Apr 2010, 23:27
ხომ არ იცით რას შეიძლებოდა გამოეწვია მხედველობის არევა. გარშემო ყველა გამოსახულება დაცურავს, ართმანეთსი ირევა. ფოკუსირებას ძლივს ვახდენ. თაბრუსხვევა არ მაქვს. თვალის ბუდეები მტკივა თან ცოტა.

თვალთ მაკლდება???

Posted by: vano_t 24 Apr 2010, 03:37
shtori
QUOTE
ხომ არ იცით რას შეიძლებოდა გამოეწვია მხედველობის არევა. გარშემო ყველა გამოსახულება დაცურავს, ართმანეთსი ირევა. ფოკუსირებას ძლივს ვახდენ. თაბრუსხვევა არ მაქვს.  თვალის ბუდეები მტკივა თან ცოტა.

თვალთ მაკლდება???

ყველაზე კარგი იქნება ოფთალმოლოგთან მიხვიდე და თვალი დაათვალიერონ. თვალის დათვალიერების გარეშე ძნელია რაიმეს თქმა, თანაც ოფთალმოლოგს გაცილებით უკეთ ესმის ეგ საქმე.

Posted by: bachachuna 24 Apr 2010, 13:20
vano_t
არა, ჭინჭარმა იცის თუ არა მუცლის შებერილობა, ან რამ შეიძლება გამოიწვიოს, მუცლის შებერილობა და გაზების დაგროვება, გაკეთრბული აქვს 2 წლის წინ ნაწლავების გაუვალობის ოპერაცია და ამ ბოლო პერიოდში დაეწყო საშინელი მუცლის ტკივილები და გაზების დაგროვება

Posted by: iza 24 Apr 2010, 13:58
სამი დღეა ყელში საშინლად მეწვის ლიმფური კვანძები. ამ 3 დღის წინათ ძალიან ვინერვიულე რაღაცაზე და მეორე დღიდან ესე ვარ. განსაკუთრებით მეწვის ღეჭვის და ნერწყვის გადაყლაპვის დროს. ამიტომ ვცდილობ მხოლოდ იოგურტი ან სითხეები მივიღო, მაგრამ ძალიან დავსუსტდი უკვე, თანაც საკმაოდ შრომატევადი სამსახური მაქვს და დღის ბოლოს უკვე ფეხზე ვერ ვდგები

ასევე ზუსტად ამ წვასთან ერთად ვიგრძენი გულის ქვემოთ, სადღაც ნეკნებთან საშინელი სიმძიმე. თითქოს რაღაც მაწევს, მინდა მოვიცილო და ვერ ვიცილებ, განსაკუთრებით მტკივა თუ მარცხენა მხარეს ვწვები და იძულებული ვარ სხვა მხარეს დავწვე რომ აღარ მეტკინოს.

ამ დრომდე აბსოლუტურად ყველა გამოკვლევა ჩატარებული მაქვს და აბსოლუტურად ჯანმრთელი ვიყავი. შესაძლოა ეს ლიმფური კვანძები მხოლოდ სტრესის გამო მეწვოდეს??? ხელს რომ ვკიდებ თითქოს გადიდებულიცაა

ექიმთან მისვლა მეშინია, არ მინდა რაიმე საშინელება მითხრას. არ ვიცი რა გავაკეთო

Posted by: niniaa 24 Apr 2010, 14:08
iza
სიცხეები გაქვს?სიბრძნის კბილი ამოსულია?
სისხლის ანალიზის გაკეთება აუცილებელია.

Posted by: shtori 24 Apr 2010, 14:08
vano_t
QUOTE
ყველაზე კარგი იქნება ოფთალმოლოგთან მიხვიდე და თვალი დაათვალიერონ. თვალის დათვალიერების გარეშე ძნელია რაიმეს თქმა, თანაც ოფთალმოლოგს გაცილებით უკეთ ესმის ეგ საქმე.

კიდევ თუ განმეორდა მივალ.
გმადლობ
)

Posted by: iza 24 Apr 2010, 14:30
niniaa


QUOTE
სიცხეები გაქვს?სიბრძნის კბილი ამოსულია?



სიცხე არ გამიზომია, მაგრამ ვგრძნობ რომ მომენტებში ცხელი ვარ. დავიწყებ გაზომვას. კი, სიბრძნის კბილი ამოსულია

Posted by: marto 24 Apr 2010, 16:33
ექიმებო იქნებ მითხრათ, რისი ბრალია მუხლების , საერთოდ სახსრები და კუნთების ტკივილი, როცა ც რეაქტიული ნორმაშია.


მიპასუხეთ გთხოვთ.

Posted by: vano_t 24 Apr 2010, 21:53
bachachuna
QUOTE
არა, ჭინჭარმა იცის თუ არა მუცლის შებერილობა, ან რამ შეიძლება გამოიწვიოს, მუცლის შებერილობა და გაზების დაგროვება, გაკეთრბული აქვს 2 წლის წინ ნაწლავების გაუვალობის ოპერაცია და ამ ბოლო პერიოდში დაეწყო საშინელი მუცლის ტკივილები და გაზების დაგროვება

მუცლის შებერილობა ძალიან ბევრმა რამემ იცის. მე ვერ გავიგე ჭინჭარი რა შუაშია. ჭინჭარი ჭამა შებერილმა თუ ჭინჭარმა დასუსხა? არასწორი კვება, ზედმეტი საკვების მიღება, წამლები, ინფექციები-ყველაფერმა იცის მუცლის შებერილობა. გარდა ამისა, შეიძლება ოპერაციის გვიანი გართულებაც იყოს. მოკლედ, ექიმის ნახვაა საჭირო, თუ მუცლის შებერილობა ხანგრძლივია და შემაწუხებელია.

marto
QUOTE
ექიმებო იქნებ მითხრათ, რისი ბრალია მუხლების , საერთოდ სახსრები და კუნთების ტკივილი, როცა ც რეაქტიული ნორმაშია.


მიპასუხეთ გთხოვთ.
ბევრი რამის: ვირუსული ინფექციები, გადაღლა, ბევრი მუშაობა, ანთებითი დაავადებები.

iza
QUOTE
სამი დღეა ყელში საშინლად მეწვის ლიმფური კვანძები. ამ 3 დღის წინათ ძალიან ვინერვიულე რაღაცაზე და მეორე დღიდან ესე ვარ. განსაკუთრებით მეწვის ღეჭვის და ნერწყვის გადაყლაპვის დროს. ამიტომ ვცდილობ მხოლოდ იოგურტი ან სითხეები მივიღო, მაგრამ ძალიან დავსუსტდი უკვე, თანაც საკმაოდ შრომატევადი სამსახური მაქვს და დღის ბოლოს უკვე ფეხზე ვერ ვდგები

ასევე ზუსტად ამ წვასთან ერთად ვიგრძენი გულის ქვემოთ, სადღაც ნეკნებთან საშინელი სიმძიმე. თითქოს რაღაც მაწევს, მინდა მოვიცილო და ვერ ვიცილებ, განსაკუთრებით მტკივა თუ მარცხენა მხარეს ვწვები და იძულებული ვარ სხვა მხარეს დავწვე რომ აღარ მეტკინოს.

ამ დრომდე აბსოლუტურად ყველა გამოკვლევა ჩატარებული მაქვს და აბსოლუტურად ჯანმრთელი ვიყავი. შესაძლოა ეს ლიმფური კვანძები მხოლოდ სტრესის გამო მეწვოდეს??? ხელს რომ ვკიდებ თითქოს გადიდებულიცაა

ექიმთან მისვლა მეშინია, არ მინდა რაიმე საშინელება მითხრას. არ ვიცი რა გავაკეთო
ექიმთან მისვლა თუ გეშინია, მაშინ უნდა მოითმინო. ყელის ტკივილი არის ფარინგიტი. ყველაზე ხშირად გაციების (ზემო სასნთქი ინფექციების ბრალია). ყველაფერზე ასეთი ნერვიულობა არ შეიძლება.

Posted by: DCM 25 Apr 2010, 10:25
ფიმოზი მაქ და ხომ არ იცით ოპერაცია დაახლოებით რა დაჯდება და სად მირჩევთ რო გავიკეთო. help.gif
დამიჯერეთ ძალიან ცუდია 18_ის რომ ხარ და ფიმოზი გაქ cry.gif

P.S.ისე ერთი თემა უროლოგთან შეკითხვებიც უნდა იყოს

Posted by: vano_t 25 Apr 2010, 22:58
DCM
QUOTE
ფიმოზი მაქ და ხომ არ იცით ოპერაცია დაახლოებით რა დაჯდება და სად მირჩევთ რო გავიკეთო. help.gif
დამიჯერეთ ძალიან ცუდია 18_ის რომ ხარ და ფიმოზი გაქ cry.gif

P.S.ისე ერთი თემა უროლოგთან შეკითხვებიც უნდა იყოს

სად კეთდება მაგაზე ოპერაცია ვერ გეტყვი. ალბათ შემოვა ვინმე და ეცოდინება, გაგარკვევს.

Posted by: roamer 28 Apr 2010, 12:24
ერთი შეკითხვა მაქვს: რამოდენიმე დღეა ჭამის შემდეგ მაქვს კუჭში სიმძიმის შეგრძნება; როგორ მოვიქცე?

Posted by: vaxushtii 28 Apr 2010, 22:00
ძალიან გთხოვთ კარგი ნერვოპათოლოგი მიმასწავლოთ?
აუცილებლად გამოცდილი სპეციალისტი მჭირდება, თუნდაც მასთან ვიზიტები ძვირი იყოს

Posted by: lala 28 Apr 2010, 22:12
კარგო ოტო ლარინგოლოგი მიმასწავლეთ რა

Posted by: E N J I 28 Apr 2010, 22:26
lala
QUOTE
კარგო ოტო ლარინგოლოგი მიმასწავლეთ რა

თოდუას კლინიკაში მიდი მაია მაისაშვილთან
ჩემი ექიმია და ძალიან მომწონს smile.gif)

Posted by: lgogokhia 29 Apr 2010, 01:09
კოლუმბიაში მივდივარ მაისის ბოლოს და CDC იძახის რომ yellow fever-ის ვაქცინის გაკეთება არ გაკეთება თავად გადაწყვიტეო.. ჰოდა ჩემი დაზღვევა არ ფარავს ამ ვაქცინას.. appointment დამანიშნინეს დღეს Travel Clinic-ში და ვიღაც ექთანმა (RN lol) უნდა გადამიწყვიტოს, რომელთანაც ლაპარაკის აზრს საერთოდ ვერ ვხედავ.. აბა ინტერნისტებო/ინფექციონისტებო გავიკეთო ყვითელი ცხელების ვაქცინა თუ კოღოს repellent-ები საკმარისია? მე ის მადარდებს რომ ზოგიერთი ქვეყანა ითხოვს ყვითელი ცხელების ვაქცინაციის სერთიფიკატს თუ იყავი კოლუმბიაში ბოლო 3 თვე და ვერ ვპოულობ საფრანგეთი, ავსტრია და ბრაზილია თუ თხოულობს მაგას. CDC კი წერს არაო, მაგრამ ბრაზილიის საკონსულოს ვებსაიტზე ეგ მოთხოვნა არის (რომელიც არ დამჭირდება 3 თვეში), დანარჩენს კი ვერ ვპოულობ (რომელიც მჭირდება უეჭველი), ანუ იმის თქმა მინდა რომ CDC-ს და საკონსულოს მოთხოვნები არ ემთხვევა და რა გარანტია მაქვს რომ არ მომთხოვენ იქ მერე. ცალკე არ მინდა ამ ვაქცინის გაკეთება თუ არ მჭირდება.. ასევე CDC-ს ცხრილის და ჩემი ვაქცინაციის ჩანაწერების მიხედვით არ ვარ up-to-date varicella-ზე და MMR-ზე. და ასევე HepA-ს მირჩევს CDC. მოკლედ, თქვენი პირადი აზრი მაინტერესებს ამ საკითხზე, რა გავიკეთო თუ საერთოდ არაფერი გავიკეთო და რუტინულ ვაქცინებს მერე სხვა დროსაც ვიზამ.. vano_t მათ შორის შენი თუ არ დაგეზარება..

Posted by: Romina 29 Apr 2010, 18:18
მოდერს ბოდიშს ვუხდი აქ რომ ვპოსტავ ამ შეკითხვას, მაგრამ არ ვიცი სად დავწერო. ვინმემ ხომ არ იცით თვითმფრინავში შპრიცს თუ შემატანინებენ აუცილებელი საჭიროების გამო? წინასწარ უნდა მივმართო ვინმეს? თუ ვინმეს გაქვთ გამოცდილება და იცით როგორ უნდა მოვიქცე PM-ში მომწერეთ რა, გთხოვთ.

Posted by: Romina 30 Apr 2010, 21:18
lgogokhia

http://www.mdtravelhealth.com/destinations/samerica/colombia.php

აქ ჩამოთვლილია რა ითვლება აუცილებლად და რა არა. რაც შეეხება ყვითელ ცხელებას, აქ მითითებულია რომ თუ ქალაქიდან არ გადიხარ, ნაკლებ სავარაუდოა დაგემართოს. მაგრამ ჩემი პირადი რჩევაა რომ გაიკეთოთ ვაქცინაცია. გადაწყვეტით იმის თქმა რომ ვირუსის გადამტანი კოღოს ჭაჭანება არ იქნება ქლაქში არ შეიძლება, კოღოებისგან აბსოლუტური თავდაცვა კი შეუძლებელი. მე გავიკეთებდი. ჰემორაგიული ცხელებაა ბოლოს და ბოლოს.

Posted by: wevri 30 Apr 2010, 23:16
ბოლო დროს დღეგამოშვებით მტკივა თავი, თვალები და
ამა ერთვის გულისრევა. მხედველობა თითქოს მიორდება ხოლმე (უმნიშვნელოდ). ზოგჯერ კი წონასწორობასაც ვკარგავ.
ამასთან, ბოლო ორი წლის მანძილზე სამჯერ მქონდა კუჭის არეში ისეთი შეტევა რომ სუნთქვა მეკვროდა. პლიუს
ციება და ტემპერატურის მომატება.

რამდენიმე ექიმს მივმართე. გადაღებული მაქვს ეხო და რენტგენიც. მაგრამ ზუსტ დიაგნოზს ვერ მისვამენ.
მირჩიეთ ვის უნდა მივაკიტხო

პ.ს. ზოგადად დღის განმავლობაში მიწევს ძალიან გადაღლა.

Posted by: Natuka NGN 30 Apr 2010, 23:30
wevri
სისხლის საერთოდ ანალიზი რას ამბობს? ჰემოგლობინი როგორი გაქვს? გვერდით თემაში დაწერე, რომ ბავშვი გყავს ძუძუზე. რ=იქნებ გამოფიტული ხარ?

Posted by: wevri 30 Apr 2010, 23:40
ყურადღებისთვის გმადლობ.
როგორც ექიმმა მითხრა ჰემოგლობინი ნორმაში მქონდა.

მდგომარეობიდან გამოსასვლელად მხოლოდ ძილი მშველის მეტი არაფერი.
პ.ს. ზემოთ დამავიწყდა აღმენიშნა, რომ მხოლოდ მარცხენა თვალლი მტკივა ხოლმე.

Posted by: Natuka NGN 1 May 2010, 00:12
wevri
მგონია რომ გადაღლილი და გამოფიტული ხარ. თან პატარა ძუძუზე გყავს. ერთი კვირა უნდა გამოიზინო, ვიტამინები მიიღო წვენების სახით. არა მგონია რამე გჭირდეს. უბრალოდ ხელის შეწყპაბ გინდა, დამხმარე, რმ ცოტა ხნით ვინმემ აიღოს რაღAც ფუნქციები და შენ ცოტა დაისვენო.

Posted by: capitan marvel 3 May 2010, 15:17
გამარჯობატ ზუსტად არ ვიცი ეს სად უნდა ვიკითხო და ვერ მიმასწავლით რომელ თემაში დავსვა კითხვა ტუბერკულოზური მენინგიტის შესახებ მჭირდება სრული ინფორმაცია.

Posted by: marto 4 May 2010, 15:24
სულ მცივა და მითხრეს ეგ ნევრატიულიაო და ვინ უნდა ვნახო? ნევროპათოლოგი ანუ?

Posted by: anuka2009 4 May 2010, 17:28
ძალიან გთხოვტ დახმარებას. წითელ მგლურაზე რამე თუ იცით მომწერეთ . ან მომწერეთ საიტი სადაც ამ დაავადების შესახებ რაიმე ინფორმაციას წავიკითხავ.
ეს დიაგნოზი და ორსულობა ერთად არათავსებადია?

Posted by: Natuka NGN 4 May 2010, 19:59
anuka2009
დაგიწერე გინეკოლოგის თემაში. სისტემური დაგიდგინეს თუ წამლით გამოწვეული?

Posted by: iza 4 May 2010, 21:22
vano_t

QUOTE
ექიმთან მისვლა თუ გეშინია, მაშინ უნდა მოითმინო. ყელის ტკივილი არის ფარინგიტი. ყველაზე ხშირად გაციების (ზემო სასნთქი ინფექციების ბრალია). ყველაფერზე ასეთი ნერვიულობა არ შეიძლება.



უკვე გამიარა, გმადლობ 2kiss.gif


ახლა სხვა პრობლემა მაქვს:

დღეს მეჩქარებოდა და გადამიბრუნდა ფეხი. თავიდან ოდნავი ტკივილი ვიგრძენი, მაგრამ სახლში რომ მოვედი, იმის შემდეგ საშინლად მტკივა, როგორც კი ტერფს დავდგავ ძირს, მეწყება აუტანელი ტკივილები, გარედან არაფერი მეტყობა. ცივ ტილოებს ვიდებ, სიარული მიჭირს ძალიან. შესაძლებელია ძვალი მქონდეს გაბზარული ან რაიმე???

Posted by: shtori 6 May 2010, 04:13
ჭამის დროს ძილის მორევა რისი ბრალია?

Posted by: just smile19 6 May 2010, 20:35
shtori
QUOTE
ჭამის დროს ძილის მორევა რისი ბრალია?



კუჭი იტვირთება, სისხლი უფრო დიდი რაოდენობით მიდის კუჭისკენ, თავის ტვინს აკლდება და ამიტომ გეძინება ))


ხოდა მითხარით რა ვინმემ

ოპერაცია (გლანდების) ციკლამდე 5-6 დღით ადრე შეიძლება? თუ უფრო დიდი შუალედი უნდა იყოს? პროფილაქტიკისთვის რამე კოაგულანტი რომ გაუკეთონ არ შეიძლება?


Posted by: Topazi 9 May 2010, 13:13
გამარჯობა .......

კიდურების შეშუპება, დაბუჟება და თან ქვედა კიდურებზე ლაქები, რისი მიზეზით შეიძლება იყოს გამოწვეული ......
პირველი ორი ჩივილი ამ ბოლო დროინდელია, ხოლო ლაქები უკვე კაი ხანია მაქვს, ექიმებიც კონკრეტულს ვერაფერს მეუბნებიან, ვენების სუსტი სარქვლები მაქვს ეგ ვიცი, გამოვიკვლიე დაახლოებით ერთი წლის წინ .....

იქნებ მირჩიოთ ვის მივმართო .....

Posted by: _SisterofNight_ 9 May 2010, 20:25
2 კვირის წინ შევამჩნიე მარცხენა მხარეს კისერთან ბურთივით გადიდებული, არ მივაქციე ყურადღება და გუშინ ისევ მომიხვდა ხელი, როგორც აღმოჩნდა ჯირკვალია შესიებული, არაფერი არ მაწუხებს, არც ავად ვყოფილვარ, ერთადერთი მთელი ზამთარია სიბრძნის კბილი ამომდის და ვერ ამოვიდა, მაგრამ ამ მომენტში ჩამცხრალია, ანუ არ მაწუხებს, სისხლის საერთო გავიკეთე ექიმის რჩევით და ნუ ძაან ოდნავ მაქვს ცვლილებები, მითხრა ზინატი დალიეო, ხვალ მივდივარ კიდე ჩემი ოჯახის ექიმთან რო გადავამოწმო და იმას ვაჩვენო პასუხი. მეტი რა შეიძლება რო ვქნა? ან რა არის ეს ჯირკვალი, ცხოვრებაში არ გამსიებია და რა უნდა? ხო, ტკივილით არ მტკივა.

Posted by: კახა251 9 May 2010, 21:23
iza
გარედან რამე დაზიანება ეტყობა? ან ჰსემოტრიალებული ხომ არ არის ოდნავ ან რამე, თუ ეგ ყველაფერი 1 დგე კიდე გაგრძელდა ისე რომ არ დაიცხრა ოდნავ მაინც ცადი ტრავმატოლოგთან

Posted by: Unico 9 May 2010, 23:44
კითხვა მაქვს: ყურში პერეკისის ჩაწვეთება შეიძლება? : )

Posted by: laliko 10 May 2010, 12:42
მოგესალმებით შეკითხვა მაქვს. ჩემ მეგობარს გამუდმებით პერიოდულად აწუხებს წელის ტკივილები. კისრიდან დაწყებული წელის გავლით და ფეხისკენ უვლის ტკივილები. ეჭვი მაქვს რომ ნერვის ანთება უნდა იყოს.
მაინტერესებს რომელ ექიმს ეხება ეს საკითხი და თუ იცით ვინმე მაგის კაი სპეციალისტი...


Posted by: Girl From Villa Villekulla 12 May 2010, 00:18
ორი კითხვა მაქვს:
1. როდესაც ვდგები (სკამიდან, საწოლიდან) ფეხებს ძვლივს ვასწორებ ისე მიკავდება ძვლები, როგორც ხანშიშესულ ადმიანს, ასევე გამუდმებით მიტკაცუნდება ფეხის ძვლები ისე, თითქოს უნდა გადატყდეს ან გაიბზაროსო, რატომ? sad.gif 21-ის ვარ ჯერ.
2. ფეხების პრობლემა მაქვს ასევე, სისხლძარღვების შევიწროვება, ნუ ეგ მამაჩემსაც ქონდა, მაგრამ მე დღეში რამოდენიჯერმე მეჭიმება ძარღვები, ადრე წვივზე მქონდა, ახლა ტერფზე გადმოინაცვლა, ვერ ვასწორებ ფეხს კარგა ხანი და ძალიან მტკივა sad.gif

ექიმთან წასვლას, რიგში დგომას, მათი ცივი , "ყველაფრისმცოდნე" გამომეტყველების ყურებას და საბოლოოდ არაფრის მომცემი, ზედმეტი უაზრო სიტყვების რახარუხის მოსმენას, თქვენ მითხარით რა მჭირს, რა უნდა ჩემს ორგანიზმზს, ასე სწრაფად რომ მაბერებს sad.gif

Posted by: blblbl 12 May 2010, 00:35
თუ ადამიანს სისხლის საერთო ანალიზი სრულ ნორმაში აქვს,შეიძლება რომ რამე ცუდი სჭირდეს?ანალიზში ხომ გამოცნდებოდა?

Posted by: lgogokhia 12 May 2010, 00:45
blblbl
სისხლის ანალიზსი თუ გულისხმობ complete blood count-ს მაშინ შეიძლება და თან ძალიან ბევრი რამე smile.gif

Posted by: blblbl 12 May 2010, 00:50
lgogokhia
უი რა კარგია შენ რომ აქ ხარ 2kiss.gif როგორ ხარ?გკითხავ მასინ რატო მაინტერესებს smile.gif დედაჩემს ყელში აქვს ისეთი შეგრძნება თითქოს ბურთია გაჩხერილი და უშლის,უთხრეს ნევროზი და დაუნიშნეს წამლები,მაგრამ ოდნავ უშველა და ისევ დაეწყო.დღEს ამბობს რომ სასაზე უკან პატარა ბუშტუკივიტ მაქვსო და ყელშიც ეს არ მქონდესო.სისხლის საერთო გაიკეთა გუშინდ ა ყველაფერი ნორმაშია არანაირი ცვლილება,რა შეიძლება იყოს? sad.gif

Posted by: lgogokhia 12 May 2010, 01:07
blblbl 2kiss.gif
სისხლის საერთო ანალიზი მე მგონი არაფერ შუაში იყო ამ ჩივილთან, თუმცა კარგია რომ გააკეთა.. უცხო სხეულის შეგრძნების ყველაზე ბანალური მიზეზები შეიძლება იყოს გასტროეზოფაგეალური რეფლუქსი ან საყლაპავის სფინქტერის და სხვადასხვა ფსიქოლოგიური ანომალიები. ამიტომ, პირველ რიგში ალბათ ზედა საჭმლის მომნელებელი სისტემის ენდოსკოპიას (upper GI endoscopy) გავაკეთებდი და მერე კი ალბათ ფსიქიატრთან წავიდოდი.. გახსოვდეს, რომ ფსიქიატრები მხოლოდ შიზოფრენიის ექიმები არ არიან..

Posted by: blblbl 12 May 2010, 01:13
lgogokhia
მადლობა smile.gifმე ხვალ ყელ_ყურთან ვუპირებდი წაყვანას,მაშინ მანდ წავიყვან.თუ არადა მერე ფიქიატრთან ეს რა თქმა უნდა ვიცი
QUOTE
გახსოვდეს, რომ ფსიქიატრები მხოლოდ შიზოფრენიის ექიმები არ არიან..

მე და ეს არ მეცოდინება შენი აზრით? biggrin.gif 2kiss.gif

Posted by: lgogokhia 12 May 2010, 01:55
blblbl თერაპევტთან მიიყვანე ნორმალურთან და ის ჩახედავს ყელშიც, გააგზავნის ენდოსკოპიაზე და GERD-საც უმკურნალებს თუ ეგაა smile.gif მაგრამ არ ვიცი ვინ არის თერაპევტი მანდ, ეგ შენ გაარკვიე smile.gif

Posted by: EKIKULI 12 May 2010, 16:22
მოკლედ კეთილო ხალხო მიპასუხეთ რა, უკვე მეორე წელია დავიტანჯე, ყელში ისეთი შეგრძნება მაქვს ტიტქოს რაგაც მაქვს ბურტივიტ გაჩხერილი, ხან ყლაპვაც მიჭრის,
საჭმელს რომ ვღეჭავ მგონია რომ ვერ გადავყლაპავ და რაც შEიძლება დიდხანს ვღეჭავ,ყელყურის ექიმთან ვიყავი და არაფერი არ გაქვსო,ხო კიდევ ასე მგონია რომ ყელი შეშუპებული მაქვს შიგნიდან და ეს შEგრძნება სულ მაქვს, განსაკუთრები ღამე მეწყება, რომ დავწვები პირი მიშრება,(არც ჩიყვი მაქვს გამოკვლეული ვარ მაგაზეც) და ნუ მეწყება სპაზმი რომ ნერწყვს ვერ ვყლაპავ, ამ დროს სულ მიდევს გვერდით წყალი და მენთილიანი საღეჭი რეზინი, რომ დავღეჭავ ცოტა შვებას ვგრძნობ თუ ამ ტანჯვაში ჩამეძინა ხო კაი მერე აღარ მაღვიძბს ტუ არ ჩამეძინა მაშინ დამამშვიდებელი უნდა დავლიო ოგონდ ენის ქვეშ, ყველაზე კარგად ოპტიმალი მშველის და ჩამეძინება ხოლმე. ხო კიდევ კუჭნაწლავის პრობლემები მაქვს და ექიმი მეუბნება აქედან არის გამოწვეულიო, წამლებს ვსვავ მაგრა რაღაც შედეგს ვერ ვატყობ, ამ ყველაფერ სიკეტესთან ერთAდ ჰაერის უკმარისობაც მაქვს განსაკუთრებით ჭამის მერე, და თუ დიდხანს გამიგრძელდა მერე ბრონქებზეც გადამდის სპაზმი.
რომ ვწვები კისერი მაქვს საშინლად დაჭიმული რაგაც მომენტში ვცდილიბ მოვადუნო, მაგრამ მერე ისევ ისე მაქვს, მოკლედ ვერ გავიგე ვის უნდა მივმართო თერაპევ|თი კი მეუბნება რომ კუჭნწლავის პრობლემის ბრალია მაგრამ ორი წელია პრიოდულად ვმკურნალობ და შედეგი ხომ უნდა მქონდეს. ძალიან მინდა ბავშვის გაჩენა მაგრამ ამ პრობლემების გამო მეშინია, ასეთი რამე ფეხმძიმობის დროს რომ დამემართოს არადა სულ მაქვს და არ მეხსნება ეს სპაზმებიsad.gif

Posted by: Parasomnia 13 May 2010, 13:16
Unico
QUOTE
კითხვა მაქვს: ყურში პერეკისის ჩაწვეთება შეიძლება? : )

მხოლოდ 3%-იანის smile.gif

EKIKULI
QUOTE
მოკლედ კეთილო ხალხო მიპასუხეთ რა, უკვე მეორე წელია დავიტანჯე, ყელში ისეთი შეგრძნება მაქვს ტიტქოს რაგაც მაქვს ბურტივიტ გაჩხერილი, ხან ყლაპვაც მიჭრის, საჭმელს რომ ვღეჭავ მგონია რომ ვერ გადავყლაპავ და რაც შEიძლება დიდხანს ვღეჭავ,ყელყურის ექიმთან ვიყავი და არაფერი არ გაქვსო,ხო კიდევ ასე მგონია რომ ყელი შეშუპებული მაქვს შიგნიდან და ეს შEგრძნება სულ მაქვს, განსაკუთრები ღამე მეწყება, რომ დავწვები პირი მიშრება,(არც ჩიყვი მაქვს გამოკვლეული ვარ მაგაზეც) და ნუ მეწყება სპაზმი რომ ნერწყვს ვერ ვყლაპავ, ამ დროს სულ მიდევს გვერდით წყალი და მენთილიანი საღეჭი რეზინი, რომ დავღეჭავ ცოტა შვებას ვგრძნობ თუ ამ ტანჯვაში ჩამეძინა ხო კაი მერე აღარ მაღვიძბს ტუ არ ჩამეძინა მაშინ დამამშვიდებელი უნდა დავლიო ოგონდ ენის ქვეშ, ყველაზე კარგად ოპტიმალი მშველის და ჩამეძინება ხოლმე. ხო კიდევ კუჭნაწლავის პრობლემები მაქვს და ექიმი მეუბნება აქედან არის გამოწვეულიო, წამლებს ვსვავ მაგრა რაღაც შედეგს ვერ ვატყობ, ამ ყველაფერ სიკეტესთან ერთAდ ჰაერის უკმარისობაც მაქვს განსაკუთრებით ჭამის მერე, და თუ დიდხანს გამიგრძელდა მერე ბრონქებზეც გადამდის სპაზმი. რომ ვწვები კისერი მაქვს საშინლად დაჭიმული რაგაც მომენტში ვცდილიბ მოვადუნო, მაგრამ მერე ისევ ისე მაქვს, მოკლედ ვერ გავიგე ვის უნდა მივმართო თერაპევ|თი კი მეუბნება რომ კუჭნწლავის პრობლემის ბრალია მაგრამ ორი წელია პრიოდულად ვმკურნალობ და შედეგი ხომ უნდა მქონდეს. ძალიან მინდა ბავშვის გაჩენა მაგრამ ამ პრობლემების გამო მეშინია, ასეთი რამე ფეხმძიმობის დროს რომ დამემართოს არადა სულ მაქვს და არ მეხსნება ეს სპაზმებიsad.gif


ხანდახან საკვები ან წყალი ხომ არ გცდება?
გარდა ამ ჩივილებისა, ხელები და ფეხები ხომ არ გეყინება?
ან კანზე რაიმე ცვლილება ხომ არ გაქვს?

Posted by: EKIKULI 13 May 2010, 19:37
Parasomnia
QUOTE
ხანდახან საკვები ან წყალი ხომ არ გცდება?

იშვიათად კი
QUOTE
გარდა ამ ჩივილებისა, ხელები და ფეხები ხომ არ გეყინება

ხელები და ფეხები უმეტესად ცივი მაქვს.
QUOTE
ან კანზე რაიმე ცვლილება ხომ არ გაქვს?

არა კანზე არ მაქვს ცვლილებები

Posted by: Parasomnia 13 May 2010, 21:15
EKIKULI
QUOTE
არა კანზე არ მაქვს ცვლილებები


არც გიშუპდება ხანდახან?

სახსრები ან კუნთები ხომ არ გაწუხებს?

კუჭ-ნაწლავზე რა პრობლემა გაქვს?
რაც შეეხება საყლაპავს, რენტგენი არ გადაგიღია?

ამდენ შეკითხვას იმიტომ გისვამ, რომ დაახლოებით მივხვდე რა დაავადებაა smile.gif
თუ გინდა პმ-ში მიპასუხე smile.gif

Posted by: Unico 14 May 2010, 23:21
Parasomnia
დიდი მადლობა


***
ახალი კითხვა მაქვს და თემის გახსნა არ მინდა. ისა, ჟანგბადის ბალონი რომ ვიყიდო აფთიაქში რეცეპტის გარეშე, შეიძლება?
user.gif

Posted by: Thutha 15 May 2010, 19:33
გამარჯობა!

82 წლის ქალს აწუხებს საშინელი ხველა. ნახველი სეროზულია, ქაფიანი. მოსმენით- ბრონქიტიაო ექიმმა, დაუნიშნა მკურნალობა, მათ შორის ანტიჰისტამინურებიც-არ შველის. ხველა ეწყება სიცივეზე,ოდნავ ნიავზეც კი. რას ფიქრობთ?

Posted by: texasuri jleta benzoxerxit 15 May 2010, 21:40
QUOTE (Thutha @ 15 May 2010, 19:33 )
გამარჯობა!

82 წლის ქალს აწუხებს საშინელი ხველა. ნახველი სეროზულია, ქაფიანი. მოსმენით- ბრონქიტიაო ექიმმა, დაუნიშნა მკურნალობა, მათ შორის ანტიჰისტამინურებიც-არ შველის. ხველა ეწყება სიცივეზე,ოდნავ ნიავზეც კი. რას ფიქრობთ?

და გულის დეკომპენსირებულ უკმარისობაზე, შარდმდენებით განტვირთვაზე არ უფიქრია ექიმს? სასწრაფოდ კომპეტენტური თერაპევტის კონსულტაცია,გულ-მკერდის გაშუქება,ექოკარდიოსკოპია,ეკგ და ა.შ.

Posted by: _SisterofNight_ 17 May 2010, 10:51
2 კვირის წინ შევამჩნიე მარცხენა მხარეს კისერთან ბურთივით გადიდებული, არ მივაქციე ყურადღება და გუშინ ისევ მომიხვდა ხელი, როგორც აღმოჩნდა ჯირკვალია შესიებული, არაფერი არ მაწუხებს, არც ავად ვყოფილვარ, ერთადერთი მთელი ზამთარია სიბრძნის კბილი ამომდის და ვერ ამოვიდა, მაგრამ ამ მომენტში ჩამცხრალია, ანუ არ მაწუხებს, სისხლის საერთო გავიკეთე ექიმის რჩევით და ნუ ძაან ოდნავ მაქვს ცვლილებები, მითხრა ზინატი დალიეო, ხვალ მივდივარ კიდე ჩემი ოჯახის ექიმთან რო გადავამოწმო და იმას ვაჩვენო პასუხი. მეტი რა შეიძლება რო ვქნა? ან რა არის ეს ჯირკვალი, ცხოვრებაში არ გამსიებია და რა უნდა? ხო, ტკივილით არ მტკივა.

Posted by: Lacrimosa_ 21 May 2010, 13:54
მამაჩემი ხვრინავს, ყოველთვის ხვრინავდა, მაგრამ ახლა რაღაც სხვაგვარად ხვრინავს.
ხანდახან მგონია იხრჩობა თქო და მისაშველებლად გავრბივარ user.gif
30 წლიანი მწეველის სტაჟი აქვს
რამოდენიმე წლის წინ ფილტვში წყალი ჰქონდა და სრულიად განიკურნა

ეს ხვრინვა იმას ხომ არ ნიშნავს, რომ ფილტვში ისევ რაიმე პრობლემა აქვს, ან თუ უბრალოდ ხვრინვა "სჭირს" და მე დამეწყო ნერვოზი და მეჩვენება რაღაცეები, რა შეიძლება მივცე ხვრინვის საწინააღმდეგო?

texasuri jleta benzoxerxit
Parasomnia

Posted by: section1 22 May 2010, 02:38
ხალხო ვიცი ესსულ სხვა პრობლემაა მაგრამ მაინც გკითხავთ! გთხოვთ მაგრა რო დამეხხმაროთ! სადღაც 2 წლის წინ დავიწყე დიეტა და პრაქტიკულათ საჭმელზე უარი ვთქვი! მაგრა გავხდი,ზომაზე მეტად! მერე ისე დაემთხვა რო სისხლი დავკარგე და დამეწყო თმის ცვენა! თვითნებური დიეტი თუ სისხლის დაკარგვის გამო მოხდა ეს არ ვიცი! შეიძლება ორივესი ერთად! რა აღარ ვიხმარე "ვიში" თავის კაფსულებთან ერთათ! ოზონი, რავიცი რაც კი ძვირფასი შამპუნები იყო და უარესი მიქნა! მერე ვიტამინებიც ვსვი და ფიტოვალის სპეციალური თმის კაფსულები სადღაც 1 თვე მაგრამ დიდი შედეგი არ მომცა და კუჭზეც ვერ ავიტანე! ახლა უკვე საგანგაშო მდგომარეობა მაქ,თმები პრაქტიკულათ აღარ მაქ! 3 ჯერ გადავიპარსე და უფრო და უფრო უარესი მომივიდა... გთხოვთ ყველანაირი ხუმრობის გარეშე თუ ვინმემ რამე საშვალება ან კონკრეტულათ ვინმე იცით მირჩიეთ! ძაან გთხოვთ cry.gif cry.gif cry.gif

Posted by: MITIATO 22 May 2010, 15:02
ჩემს ახლობელს აქვს შემდეგი ჩივილები: მარჯვენა ხელში დაბუჟების გრძნობა,მარმარილოსებრი კანი, მგრძნობელობის დაქვეითება. თუ რამეს აკეთებს,ეგრევე ეწყება თითების ყრუ ტკივილი და თითებზე კანი უთეთრდება.ძირს თუ დაუშვა ხელი,გრძნობს როგორ ჩადის სისხლი და ეს ჩივილები უქრება.
რისი ბრალი შეიძლება იყოს და რომელ ექიმს მიმართოს?

Posted by: henry13 23 May 2010, 20:46
ვაჟაზე რო რესპუბლიკურია იმის ნომერი ხო არავინ იცით

Posted by: კახა251 23 May 2010, 23:11
გამარჯობათ, ვარ 17 წლის ბიჭი, მაქვს მომატებული ტემპერატურა 37 დან 37.4 მდე დილიდან გამემდე ეგრეა მე 6 თვე დაიცკო უკვე და გრძნობა მაქვს თითქოს ვხურვარ, ეხლავე გეტყვიტ რეები ჩავიტარე: 3 ჯერ სისხლის ანალიზი, ნომრასჰია! ფილტვების რენგენი ტომოგრაპია მანდაც სუფტაა, ვიაყვი ფთიზიატრთან გავიკეტე ნაცხის ანუ ნახველის ანალიზი ნორმასჰია, ვიაყვი ჩიყვის ექიმთან არაფერი გჯირსო, ვიაყვი ყელ ყურის ექიმთან არაფერი გჩირსო, ვიაყვი თერაპევტტან მაგანაც რაზე გაგიკეტო თერაპიოაო ვერაფერს ვხედავ ხელჩასაჭიდსო, რავი მიღციეტ რა რამე რა გთხოოვტ
* * *
section1
აუცილებლად გაიკეტე სისხლის ანალიზი მე მგონი ანემიასტან გაქ საქმე და ეგ ანალიზის პასუხი მიიტანე თერაპევთტან
* * *
ანეიმას იცვევს სისხლის დაკარგვა შენ შემთხვევაში მე მგონი ეგრეა და ისიც დაცერე ფრჩხილებ იხო არ გაგიხდა ფშვნევადი

Posted by: Parasomnia 24 May 2010, 12:57
henry13
QUOTE
ვაჟაზე რო რესპუბლიკურია იმის ნომერი ხო არავინ იცით

39 57 14, 39 55 23


Posted by: ane mekeshi 24 May 2010, 19:42
მოწამვლის შემდეგ აუტანელი თავის ტკივილი დამრჩა... რა დავლიო, რომ ტკივილი შემიჩეროს და კუჭი არ გამიღიზიანოს? (ყოველნაირ წამალზე და საჭმელზე საშინელი უკურეაქცია მაქვს)

Posted by: MALI 26 May 2010, 10:58
Parasomnia

პმ ში მოგწერე და ნახე აბა?!

Posted by: E N J I 26 May 2010, 16:40
kaxa251
QUOTE
გამარჯობათ, ვარ 17 წლის ბიჭი, მაქვს მომატებული ტემპერატურა 37 დან 37.4 მდე დილიდან გამემდე ეგრეა მე 6 თვე დაიცკო უკვე და გრძნობა მაქვს თითქოს ვხურვარ, ეხლავე გეტყვიტ რეები ჩავიტარე: 3 ჯერ სისხლის ანალიზი, ნომრასჰია! ფილტვების რენგენი ტომოგრაპია მანდაც სუფტაა, ვიაყვი ფთიზიატრთან გავიკეტე ნაცხის ანუ ნახველის ანალიზი ნორმასჰია, ვიაყვი ჩიყვის ექიმთან არაფერი გჯირსო, ვიაყვი ყელ ყურის ექიმთან არაფერი გჩირსო, ვიაყვი თერაპევტტან მაგანაც რაზე გაგიკეტო თერაპიოაო ვერაფერს ვხედავ ხელჩასაჭიდსო, რავი მიღციეტ რა რამე რა გთხოოვტ

მეც ეგრე ვარ სულ სიცხეს ვგრძნობ და 37 მაქ ტემპ. ყველანაირი გამოკვლევა ჩავიტარე და არაფერი არ გჭირსო. ინფექციის ბრალია მგონი sad.gif უკვე მივეჩვიე და აღარც ვიზომავ



თერაპევტს მინდა ვკითხო გულმკერდის რენდგენი სად შეიძლება გავიკეთო, სადაც ფილტვებს იღებენ იქ? რო ვსუნთქავ ხიხინის ხმა მესმის და ყელი ხშირად მტკივა ანუ გლანდები მაქვს

Posted by: Parasomnia 27 May 2010, 00:50
E N J I
QUOTE
თერაპევტს მინდა ვკითხო გულმკერდის რენდგენი სად შეიძლება გავიკეთო, სადაც ფილტვებს იღებენ იქ? რო ვსუნთქავ ხიხინის ხმა მესმის და ყელი ხშირად მტკივა ანუ გლანდები მაქვს


ჯერ თერაპევტმა მოგისმინოს და მერე გაიკეთე რენტგენოგრაფია, იქნებ არაფერი არ არის, ტყუილად რატომ უნდა დასხივდე?

Posted by: texasuri jleta benzoxerxit 27 May 2010, 04:40
Lacrimosa_ სასურველია Fილტვების რენტგენოგრაფია, გარეგანი სუნთქვის შესწავლა (სპირომეტრია), ჭარბი წონა ხომ არ აქვს მამას?
section1 ენდოკრინოლოგს მიმართე,შესაძლებელია ხანგრძლივმა არარაციონალურმა დიეტამ მეორადი ენდოკრინოლოგიური ძვრები გამოიწვია.

MITIATO
სისხლძარღვოვან პათოლოგიას გავს,მიმართოს ანგიოლოგს და ნევროპათოლოგსაც

Posted by: Lacrimosa_ 27 May 2010, 10:29
texasuri jleta benzoxerxit
QUOTE
ჭარბი წონა ხომ არ აქვს მამას?

კი აქვს smile.gif, შეიძლება წონის ბრალი იყოს?

Posted by: E N J I 27 May 2010, 21:18

Parasomnia
QUOTE
ჯერ თერაპევტმა მოგისმინოს და მერე გაიკეთე რენტგენოგრაფია, იქნებ არაფერი არ არის, ტყუილად რატომ უნდა დასხივდე?

მომისმინა და არაფერი არ გაქვსო მაგრა რო ვსუნთქავ ხმა მესმის რაღაც, ადრე არ მქონდა ეგრე
ამიტო მინდა გადავიღო, სად აკეთებენ შეგიძლია მითხრა?

Posted by: Parasomnia 27 May 2010, 21:38
E N J I
QUOTE
მომისმინა და არაფერი არ გაქვსო მაგრა რო ვსუნთქავ ხმა მესმის რაღაც, ადრე არ მქონდა ეგრე ამიტო მინდა გადავიღო, სად აკეთებენ შეგიძლია მითხრა?

ყველგან სადაც რენტგენის აპარატი აქვთ smile.gif

Posted by: E N J I 27 May 2010, 22:41
Parasomnia
QUOTE
ყველგან სადაც რენტგენის აპარატი აქვთ

მადლობაა
ანუ გამოჩნდება ხო თუ რამე მჭირს? ოპერასთან არის რაღაც კლინიკა და მივალ

Posted by: Parasomnia 28 May 2010, 15:55
E N J I
QUOTE
ანუ გამოჩნდება ხო თუ რამე მჭირს?

გამოჩნდება.

Posted by: JERICHO 29 May 2010, 22:00
ყურში წყალი შემივიდა და რა ვქნა ? სახლში არ მინდა რომ ვთქვა

Posted by: aranormaluri 30 May 2010, 01:04
ხერხემლის გამრუდებასთან დაკავშირებით კითხვა მაქვს და სად შეიძლება რომ ვიკითხო?

Posted by: Lacrimosa_ 30 May 2010, 18:08
texasuri jleta benzoxerxit
QUOTE
გამოგზავნილია: 21 May 2010, 13:54  #19724702

მამაჩემი ხვრინავს, ყოველთვის ხვრინავდა, მაგრამ ახლა რაღაც სხვაგვარად ხვრინავს.
ხანდახან მგონია იხრჩობა თქო და მისაშველებლად გავრბივარ 
30 წლიანი მწეველის სტაჟი აქვს
რამოდენიმე წლის წინ ფილტვში წყალი ჰქონდა და სრულიად განიკურნა

ეს ხვრინვა იმას ხომ არ ნიშნავს, რომ ფილტვში ისევ რაიმე პრობლემა აქვს, ან თუ უბრალოდ ხვრინვა "სჭირს" და მე დამეწყო ნერვოზი და მეჩვენება რაღაცეები, რა შეიძლება მივცე ხვრინვის საწინააღმდეგო?

QUOTE
სასურველია Fილტვების რენტგენოგრაფია, გარეგანი სუნთქვის შესწავლა (სპირომეტრია), ჭარბი წონა ხომ არ აქვს მამას?

ინფარქტი მოუვიდა

Posted by: texasuri jleta benzoxerxit 30 May 2010, 19:35
Lacrimosa_
რამდენი წლის არის? დიაბეტიც ხომ არ აქვს? რამდენიმე წლის წინ სითხე ფილტვებში რა მიზეზით იყო,ხომ არ გახსოვს?

Posted by: just smile19 30 May 2010, 21:23
JERICHO
QUOTE
ყურში წყალი შემივიდა და რა ვქნა ? სახლში არ მინდა რომ ვთქვა



ბამბით ამოისუფთავე

ნუ გეშინია არაფერი მოგივა : D

Posted by: Lacrimosa_ 31 May 2010, 00:31
texasuri jleta benzoxerxit
56 წლისაა, დიაბეტი არა აქვს. ძალიან ჯანმრთელი კაცია
ფილტვებში სითხე ვერ გავიგეთ რატომ ჰქონდა, თუმცა მკურნალობის მერე ძალიან მალე დაშრა. იმდენად მალე, რომ თვითონ ექიმებს გაუკვირდათ.
ანალიზები ძალიან კარგი აქვსო ექიმმა, თვითონაც ნორმალურად გრძნობს თავს, ტკივილები ოდნავ აქვს და სიმძიმე აქვს გულზე, მადა არა აქვს საერთოდ. გუშინ დილის 2-4 საათზე გახდა ცუდად და მას მერე რეანიმაციაში ვართ.
ფილტვების პრობლემაზე რომ ვესაუბრეთ ექიმს, მერე ანტიბიოტიკი დაუნიშნა.
მწეველის ხიხინები აქვსო მხოლოდო, დაგვამშვიდა

Posted by: nino_sh 31 May 2010, 16:04
გამარჯობათ, იქნებ გამარკვიოთ,
ჩემს მეუღლეს რამდენიმე თვის წინ შევამჩნიე ხალის ფერის ლაქა კისერზე--თავთან ახლოს. ამას ყურადღება არ მივაქციეთ, ვიფიქრეთ რომ შეიძლება ახალი ხალი ყოფილიყო. შემდეგ გაცივდა, დაემართა მწვავე ბრონქიტი, ჰაიმორიტი. თერაპევტმა გამოუწერა მკურნალობის კურსი, ჩააბარებინა სისხლის საერთო ანალიზიც. ანალიზებმა დაასკვნა: ნორმოქრომული ანემია. ბრონქიტის მკურნალობის (სინუპრეტი, ერესპალი, აუგმენტინი)შემდეგ დაუნიშნა ანემიის მკურნალობა: ფეროვიტ ფორტე, მაგიტრონი.
გუშინ შევამჩნიე რომ სხვაგანაც გაუჩნდა თავზე ყავისფერი ლაქა. ხომ ვერ მეტყვით რისი ბრალი შეიძლება იყოს ეს, და ვის მივმართოთ, რომელ სპეციალისტს--დერმატოლოგს???


Posted by: BadbadGirl 31 May 2010, 23:36
კანის ექიმი ან თერაპევტი შემოდის აქა? კონსულტაცია მჭირდება ძალიან.

წინასწარ მადლობა


პ.ს. მაინტერესებს ჩემს შემთხვევაში ვის უნდა მივმართო და რაც მთავარია რა უნდა მოვითხოვო ექიმისაგან!

Posted by: Lacrimosa_ 1 Jun 2010, 08:08
texasuri jleta benzoxerxit
56 წლისაა, დიაბეტი არა აქვს. ძალიან ჯანმრთელი კაცია
ფილტვებში სითხე ვერ გავიგეთ რატომ ჰქონდა, თუმცა მკურნალობის მერე ძალიან მალე დაშრა. იმდენად მალე, რომ თვითონ ექიმებს გაუკვირდათ.
ანალიზები ძალიან კარგი აქვსო ექიმმა, თვითონაც ნორმალურად გრძნობს თავს, ტკივილები ოდნავ აქვს და სიმძიმე აქვს გულზე, მადა არა აქვს საერთოდ. შაბათს დილის 2-4 საათზე გახდა ცუდად და მას მერე რეანიმაციაში ვართ. ინფარქტი გვითხრეს, გულის წინა კედლის, მძიმე.
ფილტვების პრობლემაზე რომ ვესაუბრეთ ექიმს, მერე ანტიბიოტიკი დაუნიშნა.
მწეველის ხიხინები აქვსო მხოლოდო, დაგვამშვიდა

გუშინ ინტენსიურში გადაგვიყვანეს, მანამდე სიცხე არავის გაუსინჯავს, გუშინ ექთანმა თერმომეტრი როცა მოგვაწოდა, გავუსინჯეთ და 38.2 ჰქონდა, მორიგე ექიმს ვუთხარით და აღმოჩნდა, რომ იმ ფილტვის ანთება აქვს, რომელშიც სითხე ჰქონდა ადრე. რატომ არ გვიმხელდნენ, ვერ გავიგეთ. ანალგინს და ცეფამედს უკეთებენ ფილტვებისათვის. 2 საათის შემდეგ 37.8-ზე ჩამოვიდა სიცხე და 4 საათში 37-მდე. ამ დილასაც 37.1 ჰქონდა.

გადასხმა უდგას, ნიტრო აწერია მიკრულ ფურცელზე, ძალიან ნელა ჩამოდის წვეთები, 8 წამში.

რაც ყველაზე მეტად მიშლის ნერვებს: გუშინ დილას, როცა გადმოგვიყვანეს ინტენსიურში, წნევა ჰქონდა 120, შუადღეს 130, საღამოს 140, ამ დილას 145-150. შემდეგ მე წამოვედი და დედიკომ 1 საათის მერე დამირეკა 110/70-ზე აქვსო, ეტყობა ექთანმა ვერ გასინჯა პირველადო. წნევის გაზომვა არ ვიცით არცერთმა და შეიძლება ფილტვების ანთებასავით გვიმალავენ.

ძალიან მინდა რენტგენი გადავუღოთ და არ არსებობს მოწყობილობა, რომლითაც დაწოლილ ადამიანს რენტგენს გადაუღებენ? განძრევა არ შეიძლება. ძალიან მეშინია, ყველაფრის თავი და თავი ფილტვები არ იყოს.

Posted by: The DoctoR 2 Jun 2010, 17:58
ასეთი რაღაც მაინტერესებს:
როდესაც კუჭის მომატებული მჟავიანობა აქვს ადამიანს, რომელი მინერალური წყალი არის უკეთესი - "ნაბეღლავი", "ლიკანი" თუ "ბორჯომი" ?
ვიცი, რომ ნაბეღლავი უმატებს მჟავიანობას და ლიკანი ამცირებს თუ პირიქით.
და ეგ მაინტერესებს... იქნებ მიპასუხოთ

Posted by: Lizikuna3 5 Jun 2010, 13:04
ძალიან გთხოვთ იქნებ ვინმემ მიპასუხოთ. sad.gif
თუ აქ ვინმე დერმატოლოგია მას მივწერ მაგრამ დავათვალიერე და ვერაფერი ვიპოვე.

მოკლედ...
2 წლის წინ ზაფხულში მქონდა ხელებზე, კერძოდ თითებზე საოფლე ჯირკვლების გაღიზიანება და ეს ჩემი საცოდავი თითები 3ჯერ უფრო გასიებული მქონდა. ყველას ეშინოდა biggrin.gif
ვიყავი რამდნეიმე დერმატოლოგთან და გამომიწერე ათასი წამალი, მათ შორის დამამშვიდებლები... ნევროზიანი ხაარო, არამეთქი და მაინც 3 თუ 4 დამამშვიდებელი გამომიწერა. მაგრამ არ დამილევია რადგან მეორე დღეს საზღვარ გარეთ გავედი და უცხოელ ექიმთან მივედი. იქ ერთი მაზი გამომიწერეს და მშვენივრად მიშველა.
ხო სანამ ამ დამამშვიდებელს გამომიწერდნენ პირველმა ექიმმა სალიცილის მჟავა და რაღაც გამომიწერა. ერთმანეთში აურიე და ხელებზე წაისვიო. მეც მასე ვაკეთებდი და უარესი მემართებოდა. კიდევ მარგანეციანი ცხელი აბაზანები დამინიშნა, ისეთი ცხელი იყოს როგორსაც აიტანო. და მეც ვიხარშავდი ხელებს! biggrin.gif
ხოდა ეხლაც პატარა ბუშტუკები მაქვს ხელებზე. ჯერჯერობით სერიოზული არაფერი მაგრამ ძალიან მეშინია რომ ისევ ისე მექნება. იქნებ რამე მირჩიოთ. არ მინდა ისევ გასიებული ხელებიტ სიარული. ყველა მერიდებოდა. ეშინოდათ რამე არ გადაგვედოსო, არადა გადამდები არ იყო. sad.gif

Posted by: gvarjila18 8 Jun 2010, 11:52
QUOTE (Lizikuna3 @ 5 Jun 2010, 13:04 )
ძალიან გთხოვთ იქნებ ვინმემ მიპასუხოთ. sad.gif
თუ აქ ვინმე დერმატოლოგია მას მივწერ მაგრამ დავათვალიერე და ვერაფერი ვიპოვე.

მოკლედ...
2 წლის წინ ზაფხულში მქონდა ხელებზე, კერძოდ თითებზე საოფლე ჯირკვლების გაღიზიანება და ეს ჩემი საცოდავი თითები 3ჯერ უფრო გასიებული მქონდა. ყველას ეშინოდა biggrin.gif
ვიყავი რამდნეიმე დერმატოლოგთან და გამომიწერე ათასი წამალი, მათ შორის დამამშვიდებლები... ნევროზიანი ხაარო, არამეთქი და მაინც 3 თუ 4 დამამშვიდებელი გამომიწერა. მაგრამ არ დამილევია რადგან მეორე დღეს საზღვარ გარეთ გავედი და უცხოელ ექიმთან მივედი. იქ ერთი მაზი გამომიწერეს და მშვენივრად მიშველა.
ხო სანამ ამ დამამშვიდებელს გამომიწერდნენ პირველმა ექიმმა სალიცილის მჟავა და რაღაც გამომიწერა. ერთმანეთში აურიე და ხელებზე წაისვიო. მეც მასე ვაკეთებდი და უარესი მემართებოდა. კიდევ მარგანეციანი ცხელი აბაზანები დამინიშნა, ისეთი ცხელი იყოს როგორსაც აიტანო. და მეც ვიხარშავდი ხელებს! biggrin.gif
ხოდა ეხლაც პატარა ბუშტუკები მაქვს ხელებზე. ჯერჯერობით სერიოზული არაფერი მაგრამ ძალიან მეშინია რომ ისევ ისე მექნება. იქნებ რამე მირჩიოთ. არ მინდა ისევ გასიებული ხელებიტ სიარული. ყველა მერიდებოდა. ეშინოდათ რამე არ გადაგვედოსო, არადა გადამდები არ იყო. sad.gif

დერმატოლოგი არა ვარ მაგრამ ასეთი ჩივილებით ძნელია რამე გაიგო შეიძლება ლაბორატორია დაგჭირდეს, გირჩევნია დერმატოლოგს მიაკითხო, კონსტიტუციის ქუჩაზე რომ დერმატოვენეროლოგიური დისპანსერია მანდ მიდი ექიმ ალექსანდრე კაციტაძესთან ერთ-ერთი საუკეთესო დერმატოვენეროლოგია საქართველოში
* * *
QUOTE (_SisterofNight_ @ 17 May 2010, 10:51 )
2 კვირის წინ შევამჩნიე მარცხენა მხარეს კისერთან ბურთივით გადიდებული, არ მივაქციე ყურადღება და გუშინ ისევ მომიხვდა ხელი, როგორც აღმოჩნდა ჯირკვალია შესიებული, არაფერი არ მაწუხებს, არც ავად ვყოფილვარ, ერთადერთი მთელი ზამთარია სიბრძნის კბილი ამომდის და ვერ ამოვიდა, მაგრამ ამ მომენტში ჩამცხრალია, ანუ არ მაწუხებს, სისხლის საერთო გავიკეთე ექიმის რჩევით და ნუ ძაან ოდნავ მაქვს ცვლილებები, მითხრა ზინატი დალიეო, ხვალ მივდივარ კიდე ჩემი ოჯახის ექიმთან რო გადავამოწმო და იმას ვაჩვენო პასუხი. მეტი რა შეიძლება რო ვქნა? ან რა არის ეს ჯირკვალი, ცხოვრებაში არ გამსიებია და რა უნდა? ხო, ტკივილით არ მტკივა.

იმ მხარეს კარიესული კბილი თუ გაქვს მაგის ბრალიც შეიძლება იყოს,სავარაუდოდ კბილის ბრალია, კბილის ამოსვლის დროს ღრძილის ანთებაა ადგილობრივად რასაც ორგანიზმა სათანადოდ უპასუხა რისი დასტურიც არის ლიმფური ჯირკვალის გადიდება,სისხლის ანალიზზე ალბათ ე.დ.ს-ი და ლეიკოციტი იქნება უმნიშვნელოდ მომატებული რაც იგივე ღრძილის ანთებამ შეიძლება მოგცეს სწორედ მაგიტომ დაგინიშნეს ანტიბიოტიკი. კარგ სტომატოლოგს მიაკითხე და გაარკვევს კბილის ბრალი არის თუ არა
* * *
QUOTE (E N J I @ 26 May 2010, 16:40 )
kaxa251
QUOTE
გამარჯობათ, ვარ 17 წლის ბიჭი, მაქვს მომატებული ტემპერატურა 37 დან 37.4 მდე დილიდან გამემდე ეგრეა მე 6 თვე დაიცკო უკვე და გრძნობა მაქვს თითქოს ვხურვარ, ეხლავე გეტყვიტ რეები ჩავიტარე: 3 ჯერ სისხლის ანალიზი, ნომრასჰია! ფილტვების რენგენი ტომოგრაპია მანდაც სუფტაა, ვიაყვი ფთიზიატრთან გავიკეტე ნაცხის ანუ ნახველის ანალიზი ნორმასჰია, ვიაყვი ჩიყვის ექიმთან არაფერი გჯირსო, ვიაყვი ყელ ყურის ექიმთან არაფერი გჩირსო, ვიაყვი თერაპევტტან მაგანაც რაზე გაგიკეტო თერაპიოაო ვერაფერს ვხედავ ხელჩასაჭიდსო, რავი მიღციეტ რა რამე რა გთხოოვტ

მეც ეგრე ვარ სულ სიცხეს ვგრძნობ და 37 მაქ ტემპ. ყველანაირი გამოკვლევა ჩავიტარე და არაფერი არ გჭირსო. ინფექციის ბრალია მგონი sad.gif უკვე მივეჩვიე და აღარც ვიზომავ



თერაპევტს მინდა ვკითხო გულმკერდის რენდგენი სად შეიძლება გავიკეთო, სადაც ფილტვებს იღებენ იქ? რო ვსუნთქავ ხიხინის ხმა მესმის და ყელი ხშირად მტკივა ანუ გლანდები მაქვს

ნაღვლის ბუშტში ხომ არ არის რამე? ისე ენტეროვირუსმაც იცის შეკრულობა ან ფაღარათი ხომ არ გაქვს?
* * *
QUOTE (gvarjila18 @ 8 Jun 2010, 11:52 )
QUOTE (Lizikuna3 @ 5 Jun 2010, 13:04 )
ძალიან გთხოვთ იქნებ ვინმემ მიპასუხოთ. sad.gif
თუ აქ ვინმე დერმატოლოგია მას მივწერ მაგრამ დავათვალიერე და ვერაფერი ვიპოვე.

მოკლედ...
2 წლის წინ ზაფხულში მქონდა ხელებზე, კერძოდ თითებზე საოფლე ჯირკვლების გაღიზიანება და ეს ჩემი საცოდავი თითები 3ჯერ უფრო გასიებული მქონდა. ყველას ეშინოდა biggrin.gif
ვიყავი რამდნეიმე დერმატოლოგთან და გამომიწერე ათასი წამალი, მათ შორის დამამშვიდებლები... ნევროზიანი ხაარო, არამეთქი და მაინც 3 თუ 4 დამამშვიდებელი გამომიწერა.  მაგრამ არ დამილევია რადგან მეორე დღეს საზღვარ გარეთ გავედი და უცხოელ ექიმთან მივედი. იქ ერთი მაზი გამომიწერეს და მშვენივრად მიშველა.
ხო სანამ ამ დამამშვიდებელს გამომიწერდნენ პირველმა ექიმმა  სალიცილის მჟავა და რაღაც გამომიწერა. ერთმანეთში აურიე და ხელებზე წაისვიო. მეც მასე ვაკეთებდი და უარესი მემართებოდა. კიდევ მარგანეციანი ცხელი აბაზანები დამინიშნა, ისეთი ცხელი იყოს როგორსაც აიტანო. და მეც ვიხარშავდი ხელებს! biggrin.gif
ხოდა ეხლაც პატარა ბუშტუკები მაქვს ხელებზე. ჯერჯერობით სერიოზული არაფერი მაგრამ ძალიან მეშინია რომ ისევ ისე მექნება. იქნებ რამე მირჩიოთ. არ მინდა ისევ გასიებული ხელებიტ სიარული. ყველა მერიდებოდა. ეშინოდათ რამე არ გადაგვედოსო, არადა გადამდები არ იყო. sad.gif

დერმატოლოგი არა ვარ მაგრამ ასეთი ჩივილებით ძნელია რამე გაიგო შეიძლება ლაბორატორია დაგჭირდეს, გირჩევნია დერმატოლოგს მიაკითხო, კონსტიტუციის ქუჩაზე რომ დერმატოვენეროლოგიური დისპანსერია მანდ მიდი ექიმ ალექსანდრე კაციტაძესთან ერთ-ერთი საუკეთესო დერმატოვენეროლოგია საქართველოში
* * *
QUOTE (_SisterofNight_ @ 17 May 2010, 10:51 )
2 კვირის წინ შევამჩნიე მარცხენა მხარეს კისერთან ბურთივით გადიდებული, არ მივაქციე ყურადღება და გუშინ ისევ მომიხვდა ხელი, როგორც აღმოჩნდა ჯირკვალია შესიებული, არაფერი არ მაწუხებს, არც ავად ვყოფილვარ, ერთადერთი მთელი ზამთარია სიბრძნის კბილი ამომდის და ვერ ამოვიდა, მაგრამ ამ მომენტში ჩამცხრალია, ანუ არ მაწუხებს, სისხლის საერთო გავიკეთე ექიმის რჩევით და ნუ ძაან ოდნავ მაქვს ცვლილებები, მითხრა ზინატი დალიეო, ხვალ მივდივარ კიდე ჩემი ოჯახის ექიმთან რო გადავამოწმო და იმას ვაჩვენო პასუხი. მეტი რა შეიძლება რო ვქნა? ან რა არის ეს ჯირკვალი, ცხოვრებაში არ გამსიებია და რა უნდა? ხო, ტკივილით არ მტკივა.

იმ მხარეს კარიესული კბილი თუ გაქვს მაგის ბრალიც შეიძლება იყოს,სავარაუდოდ კბილის ბრალია, კბილის ამოსვლის დროს ღრძილის ანთებაა ადგილობრივად რასაც ორგანიზმა სათანადოდ უპასუხა რისი დასტურიც არის ლიმფური ჯირკვალის გადიდება,სისხლის ანალიზზე ალბათ ე.დ.ს-ი და ლეიკოციტი იქნება უმნიშვნელოდ მომატებული რაც იგივე ღრძილის ანთებამ შეიძლება მოგცეს სწორედ მაგიტომ დაგინიშნეს ანტიბიოტიკი. კარგ სტომატოლოგს მიაკითხე და გაარკვევს კბილის ბრალი არის თუ არა
* * *
QUOTE (E N J I @ 26 May 2010, 16:40 )
kaxa251
QUOTE
გამარჯობათ, ვარ 17 წლის ბიჭი, მაქვს მომატებული ტემპერატურა 37 დან 37.4 მდე დილიდან გამემდე ეგრეა მე 6 თვე დაიცკო უკვე და გრძნობა მაქვს თითქოს ვხურვარ, ეხლავე გეტყვიტ რეები ჩავიტარე: 3 ჯერ სისხლის ანალიზი, ნომრასჰია! ფილტვების რენგენი ტომოგრაპია მანდაც სუფტაა, ვიაყვი ფთიზიატრთან გავიკეტე ნაცხის ანუ ნახველის ანალიზი ნორმასჰია, ვიაყვი ჩიყვის ექიმთან არაფერი გჯირსო, ვიაყვი ყელ ყურის ექიმთან არაფერი გჩირსო, ვიაყვი თერაპევტტან მაგანაც რაზე გაგიკეტო თერაპიოაო ვერაფერს ვხედავ ხელჩასაჭიდსო, რავი მიღციეტ რა რამე რა გთხოოვტ

მეც ეგრე ვარ სულ სიცხეს ვგრძნობ და 37 მაქ ტემპ. ყველანაირი გამოკვლევა ჩავიტარე და არაფერი არ გჭირსო. ინფექციის ბრალია მგონი sad.gif უკვე მივეჩვიე და აღარც ვიზომავ



თერაპევტს მინდა ვკითხო გულმკერდის რენდგენი სად შეიძლება გავიკეთო, სადაც ფილტვებს იღებენ იქ? რო ვსუნთქავ ხიხინის ხმა მესმის და ყელი ხშირად მტკივა ანუ გლანდები მაქვს

ნაღვლის ბუშტში ხომ არ არის რამე? ისე ენტეროვირუსმაც იცის შეკრულობა ან ფაღარათი ხომ არ გაქვს?

ან შეკრულობა ხომ არ გაწუხებს?...

Posted by: Lizikuna3 8 Jun 2010, 19:48
gvarjila18
კაციტაძე ვიცი და სიმართლე რომ გითხრა არ მინდა მასთან მისვლა!
მანდაც ვიყავი ოღონდ სხვასთან ვიღაც ქალი იყო. აღარ მახსოვს მისი სახელი და იმდენი დამამშვიდებლები გამომიწერა გეგონება დასაბმელი გიჟი ვიყავი!
ახლა ვიღაც ბიჭზე მითხრეს, ვარკეთილში მუშაობსო. სახელი არ ვიცითო და იქნებ გაგიგიათ რამე! ვიცი ცოტა სულელური კითხვაა მაგრამ .....



ჰო კიდევ ეს წინასთან კავშირში არაა!
ხელები მიკანკალებს ხოლმე cry.gif რატომ ნეტა?

Posted by: SamaraT 9 Jun 2010, 19:23
ართრიტის (ფსორიაზული) მკურნალობის რა მეთოდები არსებობს?
ტკივილის გაყუჩება და ფასტუმ გელი არაა გამოსავალი, თვითონ ცოტა მაინც მკურნალობა რითი შეიძლება.. ? ? ? ?

Posted by: კახა251 10 Jun 2010, 13:30
gvarjila18
არა შეკრულობა არ მაწუხებს ამგრამ 2 -3 დღეში ერთხელ გავდივარ კუჭში, ისე მუცლის ექოც გავიკეტე ორგანოებზე და ყველაფერი ნორმის ფარგლებში იყო, ეხლა ვაპირებ სისხლის გამოკვლევას ვირუსებზე და ალერგიაზე... და დავწერ პასუხებს მადლობთ დახმარებისთვის ისე.

Posted by: meggi 10 Jun 2010, 21:12
გამარჯობათ, ვარ 29 კვირის ორსული , კოაგულოგრამის პასუხით ფიბრინოგენის კონცენტრაცია მაკვს 450, და ბეფიბრინოგენი "+", ეს ტელეფონიტ რომ ვუტხარი გაგუას დამინიშნა კარდიომაგნეზი (ტელ-ში ასე გავიგე, მაგრამ რომ ვიკითხე კარდიომაგნილიო, სწორია??) დღეში 1 , დილაობით და კურანტილი დილა საღამოს, 20 დღე, რაც მე ამ წამლების ანოტაციები წავიკითხე, დალევა დიდად არ მხიბლავს, მით უფრო რომ არც ფეხების დასიებას, არც დაბუჟებას არ ვუჩივი, თუმცა ერთი-ორჯერ მქონდა ფეხის გაკვანძის შემთხვევა უხერხული მოძრაობის დროს, რას მეტყოდით და რას მირჩევდით? წინასწარ დიდი მადლობაააა

Posted by: gvarjila18 11 Jun 2010, 10:41
QUOTE (meggi @ 10 Jun 2010, 21:12 )
გამარჯობათ, ვარ 29 კვირის ორსული , კოაგულოგრამის პასუხით ფიბრინოგენის კონცენტრაცია მაკვს 450, და ბეფიბრინოგენი "+", ეს ტელეფონიტ რომ ვუტხარი გაგუას დამინიშნა კარდიომაგნეზი (ტელ-ში ასე გავიგე, მაგრამ რომ ვიკითხე კარდიომაგნილიო, სწორია??) დღეში 1 , დილაობით და კურანტილი დილა საღამოს, 20 დღე, რაც მე ამ წამლების ანოტაციები წავიკითხე, დალევა დიდად არ მხიბლავს, მით უფრო რომ არც ფეხების დასიებას, არც დაბუჟებას არ ვუჩივი, თუმცა ერთი-ორჯერ მქონდა ფეხის გაკვანძის შემთხვევა უხერხული მოძრაობის დროს, რას მეტყოდით და რას მირჩევდით? წინასწარ დიდი მადლობაააა

კარდიომაგნილზე ვერაფერს გეტყვი,კურანტილი კიდევ ძალიან ხშირად ინიშნება გინეკილოგიაში სისხლის მიკროცირკულაციას აუმჯობესებს და პლაცენტასაც კარგად მიეწოდება სისხლი , უკუჩვენება ყველა წამალს აქვს, ჩემი რჩევა იქნება მკურნალ ექიმს დაუჯერო

Posted by: texasuri jleta benzoxerxit 11 Jun 2010, 16:56
Lizikuna3
ნინო ცისკარიშვილთან მიდი. დამაშვიდებლები თუ გამოგიწერა,ესე იგი გჭირდებოდა,ეს არ ნიშნავს,რომ "გიჟი" ხარ. დერმატოლოგიურ პათოლოგიათა დიდ ნაწილში ფსიქო-ნერვილ ფაქტორებს გადამწყვეტი როლი ენიჭება.

SamaraT
ფსორიაზული ართრიტის შემთხვევაში სტეროიდებით და/ან არასტეროიდებით მკურნალობაა ნაჩვენები,სიმძიმეს გააჩნია.
საერთოდ ამ ბოლო დროს ჩამოყალიბდა ახალი მიმდინარეობა-ფსიქოდერმატოლოგია. სადაც დერმატოლოგიური დაავადებები განიხილება როგორც ფსიქიკური აშლილობების ნიღაბი,ან მათ მიერ პროვოცირებული. ან პირიქით,დერმატოლოგიური პათოლოგია იწვევს ფსიქიკურ პრობლემებს (რა თქმა უნდა ნევროზული და არა ფსიქოზური რეგისტრის). ამიტომ დერმატოლოგთან მკურნალობის პარალელურად აუცილებელია ფსიქიატრთან ან ექიმ-ფსიქოთერაპევტთან კონსულტაცია, რის შედეგადაც უფრო მყარია მიღწეული რემისია. boli.gif yes.gif

Posted by: Lizikuna3 11 Jun 2010, 17:36
texasuri jleta benzoxerxit
ნინო ცისკარიშვილი სად მუშაობს? მე ზუსტად არ მახსოვს მაგრამ შეიძლება მასთანაც ვიყავი.
რაც შეეხება დამამშვიდებლებს თუ ერთი სახისას გამომიწერდნენ არაფერია ნამდვილად დავლევდი, მაგრამ მიუღებელია ჩემთვის 3 სახის ბალახ-ბულახის დალევა, კიდევ ცხელი წყლის აბაზანები, საინექციო წამლები და კიდევ ტაბლეტები ათასი სახის! თან თითოეული დღეში სამჯერ! გამოდის მთელი დღეები ძილში უნდა გამეტარებინა.

Posted by: elloellonatia 12 Jun 2010, 00:11
დღეს გავიკეთე სისხლის საერთო ანალიზი,შენიშვნაში ჩამიწერეს ანიზოციტოზი,ჩხირბირთვიანი მაქვს- 12,ბაზოფილი-2



ძალიან შემაშინეს შეიძლება ეს ტუბერკულოზის ნიშნები იყოს?ხელები მიოფლიანდება სასტიკად,ხელის გულები ამ ბოლო დროს მაქვს საშინლად ოფლიანი...


;( მირჩიეთ რამე რა ვქნა,tsh და t4 გავიკეთე,ვიფიქრე იქნებ ფარისებრიდან არის ტქო და გამოირიცხა,რა შეიძლება იყოს იქნებ დამეხმაროთ ;(

Posted by: texasuri jleta benzoxerxit 12 Jun 2010, 01:43
elloellonatia
12 ჩხირით და 2 ბაზოფილით დაეჭვდნენ ტუბერკულოზზე? და ჩივილებიდან ხელისგულების გაოფლიანებით? wow.gif
Lizikuna3
ამიტომ ჯობია დერმატოლოგმა მიხედოს კანს და ფსიქიატრმა და ექიმ-ფსიქოთერაპევტმა- ფსიქიკას boli.gif

ნინო ცისკარიშვილი იქვე მუშაობს სადაც კაციტაძე

Posted by: elloellonatia 12 Jun 2010, 01:59
texasuri jleta benzoxerxit


დაეჭვდნენ რა,მითხრეს შანსი არისოო...და რა შეიძლება იყოს ზოგადად,რისი ნიშანი? ;(

Posted by: utilizatori 12 Jun 2010, 08:30
elloellonatia
სისხლის ანალიზება არასპეციფიურია
ანუ ბევრი ანთებითი პროცესის დროს შეიძლება იყოს სისხლში ეგეთი ცვლილებები
ხელები რომ გიოფლიანდება ეგ გავს ტუბერკულოზის ნიშნებს თუმცა აუცილებელი არ არის
გამოკვლევაა საჭირო
გულმკერდის რენტგენი და მანტუს ტესტი ალბათ

Posted by: lizofobi 12 Jun 2010, 12:41
2 წელია სიცხე მაქვს, რა დავაშავე cry.gif

Posted by: elloellonatia 12 Jun 2010, 14:32
QUOTE (utilizatori @ 12 Jun 2010, 08:30 )
elloellonatia
სისხლის ანალიზება არასპეციფიურია
ანუ ბევრი ანთებითი პროცესის დროს შეიძლება იყოს სისხლში ეგეთი ცვლილებები
ხელები რომ გიოფლიანდება ეგ გავს ტუბერკულოზის ნიშნებს თუმცა აუცილებელი არ არის
გამოკვლევაა საჭირო
გულმკერდის რენტგენი და მანტუს ტესტი ალბათ

კიდევ რა შეიძლება იყოს ჩხირბირთვიანის ასე კატასტროფულად მატების მიზეზიი?

Posted by: konkretula 13 Jun 2010, 01:59
აუ ექიმი ვინაა? რაღაც რჩევა მჭირდება პატარა :შ

Posted by: კახა251 13 Jun 2010, 12:34
elloellonatia პირველ რიგში რაც უნდა გიაკეტო ეს არის მანტუ და ნაცხის (ნახველის) ანალზი, შემდგომ ფთიზიატრი შეაფასებს შენს მდგომარეობას და იმის მიხედვიტ გირჩევს რენგენს და ტომოგრაპიას, .... ხუდადოვზეა ტუბ დისპასნესირს გვერზევე ახალი შენობაა და იქ გადაიგე თუ რამეა ტომო.... რავიცი ხელის გულის ოფლიანობიტ თუ შენ ტუბზე დაეჭვდნენ საღოლ მაგათ, ტუბის პირველადი ნიშნებია: ტემეპრატურის მატება 37 დან 38 მდე, ხველა (შესაძლოა სისხლიანიც), საერთო სისუსტე, გამე ოფლიანობა! , მაინც სიფრთხილეს თავი არ სტკივა და ცადი ფთიზიატთან... smile.gif

Posted by: konkretula 15 Jun 2010, 01:08
ჭრილობასთან დაკავშირებით ვერავინ მომცემთ რჩევას? :შ

Posted by: gvarjila18 15 Jun 2010, 02:17
QUOTE (konkretula @ 15 Jun 2010, 01:08 )
ჭრილობასთან დაკავშირებით ვერავინ მომცემთ რჩევას? :...............

რა რჩევა გჭირდება?...დადე შეკითხვა თუ გინდა პმ მკითხე

Posted by: კახა251 16 Jun 2010, 13:27
გამარჯობათ. გამე ძილი მაქვს დარღვეული ანუ გამის 3 საათამდე მიწევს ხოლმე ფეხზე ყოფნა ან კომპიუტერთან ჯდომა, ვარ 17 წლის ბიჭი, და შეიძლება თუ არა რომ ამ გამის ძილის დარღვევამ გამოიწვიოს ტემპერატურის მატება დგის სააათებში 37 დან 37.4 მდე გამე მირეგულირდება და 37 ზე ქვევითაა 36.6 36.7 36.5, ჩატარებყლი მაქვს ყველა საჭირო გამოკვლევა ტუბიდან დაწყვებული სისხლის ანალიზებით და მთელი ამბებით და ყელ ყურ ცხვირით დამთავრებული. ან კიდე თუა შესაძლებელი ალერგიის ფონზე მქონდეს ასეთი ტემპერატურა. დილით როცა ვდგები რამენიმე წუთში მიტყდება აუტანელი ცემინება 10 - 15 ჯერ მაცემინებს და ტანზე ტაოს მაყრის, აგრეთვე ცხვირიდან მაქვს სულ გამონადენი Gტხოვთ უპასხოთ არ დამოტოვოთ. sad.gif

Posted by: gvarjila18 17 Jun 2010, 17:01
KAXA 251
ალერგიის ფონზე თავისუფლად შეიძლება ასეთი სუბფებრილური ტემპერატურა გქონდეს, აუცილებლად მიაკითხე ალერგოლოგს, მე გირჩევდი დავით თელიასთან მიხვიდე რესპუბლიკურ საავადმყოფოში თბილისში თუ ხარ რა თქმა უნდა თუ არადა სადაც ხარ იქ ხომ იქნება ვინმე გამოცდილი ალერგოლოგი

Posted by: SamaraT 17 Jun 2010, 20:35
ართრიტის, სახსრების ანთების სამკურნალოდ, ან რამე ნორმალური საშუალება, გართულებების გარეშე რომელია?

Posted by: კახა251 18 Jun 2010, 01:20
gvarjila18
სისხლის გადასხმის ინსტიტუში ვიაყვი და გავიკეტე სისხლის საერთო ანალიზი, მერე შეუტანე პროფესორი ქალი მანანა ღირდელაძე თუ არ ვცდები, ანალიზებში არაფერი არააო საეჭვოვო, და გამისნჯა ხელით. მარცხენა იღლიაში გამისინჯა პატარა კვანძი რავი თვითონ მითხრა ფეტვის სისქეაო. გამომიშვა ალერგოლოგთან, შარდის ანალიზზე და ყელ ყურთან და ისევ 2 თვეში დამიბარა. რა ინქება ჩორტ ივო ზნაეტ biggrin.gif (ალერგოლოგტან და ყელ ყურთან იმიტომ რომ დილით ან დგის ნებისმიერ მონკვეთში 10 -15 ჯერ ვაცემინებ ზედიზედ და ცხვირიდან ამქვს საშინელი გამონადენი, ხოლო ყელ ყურთან იმიტომ რომ ანალიზებშიც იყო ნაჩვენები რო ლორწოვანის გაღიზიანებააო სადგაცასო ამავე მიზეზით შარდზეც) ასე რომ პასუხებს დავცერ


მეც მაგას ვაპირებ ალერგოლოგთან და დალშე კიდე შარდზე და ყელ ყურზე
* * *
კვანძი ჯერ ჯერობით საშიში არააო და არც გამოსაკვლევი არააო და 2 თვეში ალბათ რო ჩავალ გადამიწყვეტავს

Posted by: gvarjila18 18 Jun 2010, 03:02
მანანა ღირდალაძე მაგარი ჰემატოლოგია მე მასწავლიდა ხო ფეტვის სისქე ?

Posted by: კახა251 18 Jun 2010, 10:53
gvarjila18
ხო რა ძაან პატარააო და რავი არ ღირსო ჯერ გასინჯვაო მაგაზეო 2 თვეში გამსინჯავს ისევ და გამიშვებს თუ საჭიროა გამოკვლევებზე

Posted by: luk@ 19 Jun 2010, 11:58
გამარჯობათ. 3 დღე იქნება რაც ხველა დამეწყო. ძლიერი ხველა არა მაგრამ მაინც. დღეს დილით კი პირის ბანვისას დამახველა და ნერწყვთან ერთდ სისხლიც ერია ძლივს შესამჩნევად... სანამ ეს მოხდებოდა ხველება ცივი წყლის დალევას ვაბრალებდი... რის გამო შეიძლებოდა მომხდარიყო? თავს საკმაოდ კარგად ვგრძნობ. არც სიცხე მაქვს და მადაც ნორმალური მაქვს... აქამდე მსგავსი არაფერი ყოფილა. რას მირჩევთ და რას შეეძლო გამოეწვია? რამდენად საშიშია?.... მადლობთ წინასწარ!

Posted by: კახა251 19 Jun 2010, 17:43
luk@
სისხლიანი ნახველი მრავალი დაზიანების შემტხვევაში შეიძლება ყოფილიყო. შეიძლება ელემენტარური ყელიც გქონდეს გაკაწრული, მაგრამ მაინც ცადი ფთიზიატრტან და გაესინჯე!

Posted by: luk@ 19 Jun 2010, 20:46
kaxa251
QUOTE
სისხლიანი ნახველი მრავალი დაზიანების შემტხვევაში შეიძლება ყოფილიყო. შეიძლება ელემენტარური ყელიც გქონდეს გაკაწრული, მაგრამ მაინც ცადი ფთიზიატრტან და გაესინჯე!


მადლობა! smile.gif

Posted by: konkretula 22 Jun 2010, 00:30
ვარ 163 სმ და 42 კგ biggrin.gif

ცოტა მომატება მინდა მაგრამ ვერაფრით ვერ ვიმატებ იქნება მირჩიოთ რამე, ან უბრალოდ ექიმი ვინც შეძლებს მირჩიოს რამე : D

მოკლედ გულის მანკი მქონდა და 6 თვის წინ გავიკეთე ოპერაცია.. ოპერაციის მერე 3-4 კილო მოვიმატე 1 თვეში მაგრამ მერე აღარა.. ყველანაირი ანალიზი მაქვს ჩატარებული, არც სისცლის, არც კუჭის და რავი რაც არის არაფრის ბრალი არაა რო ვერ ვიმატებ sad.gif

Posted by: J-Time 22 Jun 2010, 00:49
ხალხნო, ეხლა ვნახე eek.gif ,
რომ ყურთან, ყურის ოდნავ დაბლა, ასე 1-2 სანტიმეტრში, კისერსა და "სახის ძვალს" შორის მაქვს მრგვალი რაღაც (არ მოძრაობს!)
ვიზუალურად არ ეტყობა, ხელის დადებით ვნახე შემთხვევით - რა შეიძლება იყოს, აქ არის რამე ჯირკვალი?
ავნერვიულდი biggrin.gif

Posted by: კახა251 22 Jun 2010, 12:48
J-Time
ტკივილები არ გაქ? რა ზომისაა ჯირკვალი კანი ხომ არაა გაუხეშებული ან კიდე ფერი ხომ არ აქვს შეცვლილი?

Posted by: J-Time 22 Jun 2010, 14:21
kaxa251

არა, ტკივილები არ მაქვს, გუშინ ვნახე სრულიად შემთხვევით- ხელი მომიხვდა.
ზომა- პატარაა, რავიცი, თხილის ხელა ან უფრო პატარა.
არც კანია გაუხეშებული და არც ფერია შეცვლილი.
აქ, ამ ადგილას რამე ჯირკვალი არის, დავუშვათ, გაცივების გამო რომ იყოს გაზრდილი?
მარა, ხომ უნდა მოძრაობდეს, თუ არა?

Posted by: კახა251 22 Jun 2010, 14:52
J-Time
თავისუფლად შეიძლება რომ ყურის ანთებითაც იყოს გამოწვეული, მაგრამ რადგანაც ტკივილები არ გაქ ამას გამოვრიცხავ. ასევე შეიძლება ლიპომა იყოს ანუ ცხიმგროვა რომელიც გამაგრების შემთხვევაში ამოსაკვეთია, შეიძლება მნად გქონდა კიდევაც ადრე გაუამგრებელი ისე რომ არც კი მოგხვედრია ხელი და ახლა გამაგრდა და მისახედია უკვე, ლიპომა ანუ ცხიმრგროვა კეთილთვისებიანი სიმსივნეა რომელსაც არ აქვს მიდრეკილება გადაგვარების მიმართ. მაგრამ რახამც გამაგრებულია ყურადღება უნდა მიექცეს, ყელ ყურ ცხვირის ექიმთან შეიარე 1 ვიზიტი და გეტყვის რაცაა.

Posted by: J-Time 22 Jun 2010, 21:37
kaxa251

ჰო, მივალ რა თქმა უნდა, თან ადრე ლიმფურ ჯირკვლებზე მქონდა პრობლემები. დიდი მადლობა.

Posted by: CHLOE" 22 Jun 2010, 23:50
გამარჯობათ იქნებ დამეხმაროთ, გუშინ ტელევიზორის ყურებისას ძლიერად გავიცინე, ტვინზე ოდავ რაღაც ვიგრძენი და 10-15 წამი გამეთიშა გონება, თვალები მეხილა თურმე , მელაპარაკებოდნენ და საერთოდ არ მახსოვს, თითქოს ნარკოზში ვიყავი ბუნდოვნად მესმოდა ხმები, მერე ჩვეულებრივ გამოვფხიზლდი, მეინატერესებს საშიში ხო არ არის ეს 10 წამიანი გათიშვა ან რას უნდა გამოეწვია

Posted by: კახა251 23 Jun 2010, 00:17
CHLOE"
ნერვოპათოლოგთან მიდი მასე ძნელია რამის თქმა
* * *
ვარ 17 წლის ბიჭი, მე7 თვეა მაქვს სიცხე Dრის განმავლობაში მიწევს 11- 12 საათიდან გამის 10- 11 საათამდე ხან როგორ მაქვს ტემპერატურა ხან როგორ ხან არის დღეები რო 37 მაქვს ხან 37,2 ხან 37,3 და ხან კიდევ 37,4 (ბოლო ძალიან იშვიათად) ეს დაიწყო ურალო გაციებით ზამთარში დეკემბერში გავცივდი და ყელიც მტკიოდა და ჩვეულებრივად გაციების ფონზე მომცა 38,2 თუ რაგაცა ტემპერატურამ მერე 1- 2 დღეში წამლები დავლიე და ყვლეაფერმა გამიარა მაგრამ დამრჩა ესეთი სიცხეები. ხოდა ეხლა დავცერ რა გამოკვლევები ჩავიტარე

სისხლის ანალიზი 4 ჯერ ბოლოს წინა კვირაში გავიკეტე თბილისში სისხლის გადასხმის ინსტიტუტში პავლოვზე, ყველაფერი ნორმაშია დაგიწერთ აგერ
ბოლოს შედეგს ყველა ნორმაში იყო და ესეც არაა მგონი ცუდი და დავწერ მაინც
ჰემოგლობინი: გ/ლ135,0
ერთ 81
ერითროციტები: 4.18
ფერ.მაჩვენებელი: 0.98
რეტიკულოციტები %: 12%
ლეიკოციტები: 4,45

თრომბოციტები% : 55%
229,9
ე.დ.ს: 5
ჩხირბირთვიანი: 1
სეგმენტაბირთვიანი: 49
ბაზოფილი: 1
მონიციტი: 14 აი ეს მაქვს მომატებული 4-11 არის ნორმა
ლიმფოციტი: 33
ხო ეხალ გავაგძელოთ გამოკვლევების სია
ვიყავი ფთიზიატრთან რუსთავში თავდაპირველად, გადავიღე რენგენი და გამიშვა თბილისში ტომოზე მიუტანე და არაფერი არ არისო ასევე გავიკეთე ნაცხის (ნახველის) ანალიზი არაფერი არააო
გავიკეთე მუცლის ღრუს ექოსკოპია იქაც ყველაფერი ნორმის ფარგლებშია
ვიაყვი ჩიყვის ექიმთან გამსინჯა და არაფერი არ გეტყობაო, ვიყავი ყელ ყურ ცხვირის ექიმთან იმან მიხმინადრა რაღაც ანთება გაქო ლორწოვანისო და 2 ჯერ გამატარა მკურნალობის კურსი მაგრამ არ მიშველა, გავიკეთე 2 ჯერ შარდის ანალიზი ნორმაშია მანდაც, ბოლოს სისხლის გადასხმის ინსტიტუტში ანალიზზე რო ვიაყვი მანანა ღირდალაზესთან ხელით გამსინჯა და მარცხენა იღლიაში ფეტვის მარცვალივით პატარა კვანძი გაქო მაგრამ ჯერ არაა გამოსაკვლევიო არც საშიშიაო, 2 ტვეში დამიბარა მანამდე კი ყელ ყურ ცხვირთან ალერგოლოგთან და შარდის ანალიზზე გამისვა, (შარდის ანალიზი გადავიგე არაფერი იყო, ალერგოლოგთანაც ვიყავი მაგრამ იმან სიცხეები არ ციისო და ალერგია მანინც მაქვს და წამლები დამინიშნა) ყელ ყურ ცხვირის ექიმთან ვაპირებ ცასვლას ამ დღეებში და მანამდე მაინტერესებს თქვენი აზრი ამ მდგოარეობაზე შეაფასეთ და იმსჯელეთ! თქვენი აზრი ჩემტვის ძალიან მნიშვნელოვანია! შეიძლება მაგ კვანძს ასეთი სიცხეები გამოეწვია? ან რაიმე შაშიში შეიძლება იყოს? მადლობა წინასწარ!

Posted by: CHLOE" 23 Jun 2010, 14:49
kaxa251

ვინც იცის იმას ვკითხეsmile.gif)

არავინ არ მიპასუხებს? sad.gif(((((

Posted by: Alpinistka123 24 Jun 2010, 15:44

ექიმი ტუ ხარტ ვინმე ამჯამად შემომეხმიანედ რა კიტხვა მაქვს

Posted by: კახა251 24 Jun 2010, 15:56
ან ცჰემს კითხვას არავინ არ უპასუხებს? sad.gif იმხელა რაგაც ვცერე მაგლა sad.gif

Posted by: shtori 24 Jun 2010, 22:13
ვერცხლის წყლით მოწამვლას რა სიმპტომები აქვს?

Posted by: HestaPo 26 Jun 2010, 14:14
წლინახევრის ბავშვს საერთოდ არ აქვს სმენა (ყრუა), უჭირს სიარული, დიდხანს ვერ დგება თუნდაც ერთ ადგილზე დამოუკიდებლად, შესაბამისად მაინტერესებს არის თუ არა რაიმე კავშირი სმენის არქონასა და ორიენტაციისა და სიმძიმის ცენტრის დაკარგვას შორის.

Posted by: კახა251 26 Jun 2010, 14:44
HestaPo
რამდენადაც ვიცი მაგ დროს ფარისებრი ჯირკვლის გამოკვლევაა საჭირო, ვნახოთ აბა ვინ რას დაწერს smile.gif

Posted by: LUKA-BRAZI 26 Jun 2010, 15:36
აქ პასუხის ლოდინს, აჯობებს უბნის ექიმს მიმართოთ.

ფორუმს კი ყავს ექიმი იუზერები, მაგრამ დროის უქონლობის გამო იშვიათად შემოდიან.... მათ ლოდინში შეიძლება დიდი დრო გავიდეს ან ვინმემ არასწორი რამე გირჩიოთ და დაზარალდეთ.... -> http://forum.ge/?f=43&showtopic=34075354&st=0

გისურვებთ ჯანმრთელობას და სულიერ სიმშვიდეს smile.gif

Posted by: HestaPo 28 Jun 2010, 21:12
kaxa251

LUKA-BRAZI

გმადლობთ,
ბავშვი ექიმთან დაგვყავს, იმპლანტანტისთვის ვამზადებთ. რატომღაც მგონია, რომ ეს ორი (სმენა და ორიენტაცია) ერთმანეთთან კავშირშია და სმენის აღდგენის შემდეგ იმედი მაქვს რომ გამართულად შეძლებს გავლას. სხვამხრივ ბავშვს პრობლემა არ აქვს, ყველას გავასინჯეთ. მაგრამ ამ კითხვაზე პასუხს არ იძლევიან. ალბათ სულელური კითხვაა, არადა სადღაც (ალბათ დიქავერზე smile.gif )მსმენია ამის შესახებ.

Posted by: LUKA-BRAZI 28 Jun 2010, 23:01
HestaPo
სმენის ორგანო და წონასწორობის ორგანო, ორივე გვერდიგვერდ მდებარეობს. ერთი მოთავსებულია ლოკოკინაში, მეორე ნახევარმთვარისებურ რკალებში.... შეიძლება ბავშვს რაიმე თანდაყოლილი სტრუქტურული ცვლილება აქვს. შესაბამისად, იმის გამო რომ ორივე ორგანო ერთმანეთთან ახლოს არის, სმენის და წონასწორობის პრობლემაც მაგიტომ აქვს......

Posted by: HestaPo 29 Jun 2010, 16:42
LUKA-BRAZI

გმადლობ, ბოლოსდაბოლოს ვიპოვე ექიმი ვინც დამეთანხმა და კარგადაც განმიმარტა პრობლემის არსი. ანუ სმენაც არის წონასწორობის შენარჩუნების ერთერთი ელემენტია.

Posted by: fsiqea 30 Jun 2010, 19:09
ყელის ერთ-ერთ მხარეზე ჯირკვლების (ლიმფური) შესიება რას უნდა გამოეწვია? (ტკივილის გარეშე) (ზუსტად ჯირკვალი არაა გასიებული თუმცა მისი მიმდებარე ტერიტორია რა) რავიცი რა..
* * *
kaxa251
QUOTE
J-Time
თავისუფლად შეიძლება რომ ყურის ანთებითაც იყოს გამოწვეული, მაგრამ რადგანაც ტკივილები არ გაქ ამას გამოვრიცხავ. ასევე შეიძლება ლიპომა იყოს ანუ ცხიმგროვა რომელიც გამაგრების შემთხვევაში ამოსაკვეთია, შეიძლება მნად გქონდა კიდევაც ადრე გაუამგრებელი ისე რომ არც კი მოგხვედრია ხელი და ახლა გამაგრდა და მისახედია უკვე, ლიპომა ანუ ცხიმრგროვა კეთილთვისებიანი სიმსივნეა რომელსაც არ აქვს მიდრეკილება გადაგვარების მიმართ. მაგრამ რახამც გამაგრებულია ყურადღება უნდა მიექცეს, ყელ ყურ ცხვირის ექიმთან შეიარე 1 ვიზიტი და გეტყვის რაცაა.


უი ეხლა გადავხედე და დაახლოებით ასეთზე მაქვს მეც ლაპარაკი..

Posted by: კახა251 30 Jun 2010, 19:13
fsiqea
სხვა სიმპტომები გაქ რამე?

Posted by: fsiqea 30 Jun 2010, 19:21
kaxa251
QUOTE
fsiqea
სხვა სიმპტომები გაქ რამე?

სხვა? მაგალითად? მგონი არა..

Posted by: კახა251 30 Jun 2010, 19:23
რავი ან მომატებული ტემპერატურა, ან ხველა, ან სისუსტე მადის დაკარგვა წონასჰი კლება?

Posted by: fsiqea 30 Jun 2010, 19:26
kaxa251
QUOTE
რავი ან მომატებული ტემპერატურა, ან ხველა, ან სისუსტე მადის დაკარგვა წონასჰი კლება?

ცოტა ყელი მტკივასავით, ნერწყვის ყლაპვა მიჭირს, სისუსტე სულ მჭირს - ზარმაცი ვარ tongue.gif მადაზე და წონაზე პრობლემა არ მაქვს.. აი ტემპერატურა რავიცი, ყოველთვის ცვალებადი მაქვს ხოლმე.. ხან ძაან მცხელა ხან ძაან მცივა biggrin.gif აუ დღეს მთელი დღეა შეხვეული მაქვს, პირიქით ცუდად ხომ არ იმოქმედებს? ძაან შეშინებული ვარ sad.gif

Posted by: კახა251 30 Jun 2010, 19:31
QUOTE
ცოტა ყელი მტკივასავით, ნერწყვის ყლაპვა მიჭირს, სისუსტე სულ მჭირს - ზარმაცი ვარმადაზე და წონაზე პრობლემა არ მაქვს.. აი ტემპერატურა რავიცი, ყოველთვის ცვალებადი მაქვს ხოლმე.. ხან ძაან მცხელა ხან ძაან მცივააუ დღეს მთელი დღეა შეხვეული მაქვს, პირიქით ცუდად ხომ არ იმოქმედებს? ძაან შეშინებული ვარ

სჰეხვევა არაა საჯირო ტიტი არ მიიჯირო და რამე ტორე ისე არაა საჭირო შეხვევა, ასევე კარგი იქნება ფთიზიატრთან თუ მიხვალ მასეთი ლიმფური კვანძების გადიდება ტუბმაც იცის, (არ შეგეშინდეს უბრალოდ სიფრთხილეს თავი არ სტკივა)
ხო კიდევ მისმინე გამაგებულია თუ დახტუნავს თითის დადებისას გაისინჯე (ოღონდ ძაან არ დააჭირო) და ზუსტად რა ადგილასაა ეგეც დაწერე აბა

Posted by: fsiqea 30 Jun 2010, 19:50
kaxa251
QUOTE
სჰეხვევა არაა საჯირო ტიტი არ მიიჯირო და რამე ტორე ისე არაა საჭირო შეხვევა, ასევე კარგი იქნება ფთიზიატრთან თუ მიხვალ მასეთი ლიმფური კვანძების გადიდება ტუბმაც იცის, (არ შეგეშინდეს უბრალოდ სიფრთხილეს თავი არ სტკივა)
ხო კიდევ მისმინე გამაგებულია თუ დახტუნავს თითის დადებისას გაისინჯე (ოღონდ ძაან არ დააჭირო) და ზუსტად რა ადგილასაა ეგეც დაწერე აბა

რაააა, აუ..დახტუნავს გამაგრებული არაა..ზუსტად კისრის ბოლოში ლავიწის ძვლის (ჩაღრმავებული ადგილის) ზემოთ.. ისე ერთი ცოტა მოზრდილია დანარჩენი პატარ-პატარები..
ისე ადრეც მქონდა და როცა გავესინჯე ასე მითხრეს როცა რაიმე ანთება მიდის ორგანიზმში მაშინ ჩნდებაო (მაგალითად თუ გრიპი გქონდა ეს ბოლო დღეებიო, ან თუნდაც თუ კბილი გაწუხებდაო) სხვა დროს ასე გასიებით არ შეუწუხებივარ sad.gif

Posted by: კახა251 30 Jun 2010, 19:53
QUOTE
რაააა, აუ..დახტუნავს გამაგრებული არაა..ზუსტად კისრის ბოლოში ლავიწის ძვლის (ჩაღრმავებული ადგილის) ზემოთ.. ისე ერთი ცოტა მოზრდილია დანარჩენი პატარ-პატარები..
ისე ადრეც მქონდა და როცა გავესინჯე ასე მითხრეს როცა რაიმე ანთება მიდის ორგანიზმში მაშინ ჩნდებაო (მაგალითად თუ გრიპი გქონდა ეს ბოლო დღეებიო, ან თუნდაც თუ კბილი გაწუხებდაო) სხვა დროს ასე გასიებით არ შეუწუხებივარ

ოკ, და რამდენია ბევრია???
ხო გითხარი არ შეგეშინდეს მეთქი ეს ვარაუდია

Posted by: fsiqea 30 Jun 2010, 20:03
kaxa251
QUOTE
ოკ, და რამდენია ბევრია???
ხო გითხარი არ შეგეშინდეს მეთქი ეს ვარაუდია

იტოგში მაშინ რომ გამსინჯეს სურათიც მისაჩუქრეს და მქონდა ასე 4-5 პაწიები და ერთი დიდი..

Posted by: კახა251 30 Jun 2010, 20:21
და რა მხრივ გადაგიღეს რენგენი? ხო შეიძლება გაციებისაცაა, მე ყელზე აქეთ იქიდან მაქვს 2 ცალი მოხტუნავე ლიპომა ქვია მგონი!

Posted by: fsiqea 30 Jun 2010, 20:27
kaxa251
QUOTE
და რა მხრივ გადაგიღეს რენგენი? ხო შეიძლება გაციებისაცაა, მე ყელზე აქეთ იქიდან მაქვს 2 ცალი მოხტუნავე ლიპომა ქვია მგონი!

რედგენი არა, ექოსკოპია.. ჩიყვზე გავესინჯე და ჩიყვის ნაცვლად ესენი აღმომიჩინეს biggrin.gif ლიმფური ჯირკვლებიაო, მგონი იგივე უნდა იყოს რავიცი.. მაშინ კი დამამშვიდეს არაფერიაო და ეს შესიება.. და შენ არ გაესინჯე?

Posted by: კახა251 30 Jun 2010, 20:36
fsiqea
მე კი მაგრამ არაფერი არააო, უუფ მე ჩემ ისტორიას აღარც დავწერ წერია წინა გვერდებში 1 - 2 გვერდიტ უკან . . . ხოო ლიმფური ჯირკვლები თუა გადიდებული სისხლის საერთო ანალიზი უნდა გაკეთდეს ასევე.

Posted by: fsiqea 30 Jun 2010, 20:41
kaxa251
QUOTE
fsiqea
მე კი მაგრამ არაფერი არააო, უუფ მე ჩემ ისტორიას აღარც დავწერ წერია წინა გვერდებში 1 - 2 გვერდიტ უკან . . . ხოო ლიმფური ჯირკვლები თუა გადიდებული სისხლის საერთო ანალიზი უნდა გაკეთდეს ასევე.

ჩიყვზე, რომ გავესინჯე ეგეც თავისთავად გამიკეთეს და ძალიან კარგი პასუხები მქონდა, თანაც ისეთი რომ გაუკვირდათ კიდევაც ასე ყველაფერი როგორ ნორმაში გაქვსო..მერე ჩიყვის ექიმმა აქვე ხარ და ბარემ თერაპევტსაც გაესინჯეო და იმანაც ყველაფერი წესრიგში გაქვსო!!! თუმცა მაინც ექიმებს მე მკითხავებს ვეძახი.. biggrin.gif ერთი ჩიყვის ექიმთან მიხვიდე და იმან ცხრაასგან გაგიშვას რა უბედურებაა, მაგრამ ხომ იცი ექიმებს მაინც ყველა უჯერებს!!! ვნახოთ იქნებ დამიწყნარდეს თუ არა და მერე მივხედავ.. მაინც დიდი მადლობა მოსმენისათვის. wink.gif

Posted by: კახა251 30 Jun 2010, 21:28
fsiqea
არაფრის გაიხარე, დაწერე რაც იქნება მერე wink.gif ჯანმრთელობას გისურვებ

Posted by: Solveig 30 Jun 2010, 23:36
fsiqea
QUOTE
ჩიყვზე, რომ გავესინჯე ეგეც თავისთავად გამიკეთეს და ძალიან კარგი პასუხები მქონდა, თანაც ისეთი რომ გაუკვირდათ კიდევაც ასე ყველაფერი როგორ ნორმაში გაქვსო..მერე ჩიყვის ექიმმა აქვე ხარ და ბარემ თერაპევტსაც გაესინჯეო და იმანაც ყველაფერი წესრიგში გაქვსო!!! თუმცა მაინც ექიმებს მე მკითხავებს ვეძახი..  ერთი ჩიყვის ექიმთან მიხვიდე და იმან ცხრაასგან გაგიშვას რა უბედურებაა, მაგრამ ხომ იცი ექიმებს მაინც ყველა უჯერებს!!! ვნახოთ იქნებ დამიწყნარდეს თუ არა და მერე მივხედავ..


QUOTE
დახტუნავს გამაგრებული არაა..ზუსტად კისრის ბოლოში ლავიწის ძვლის (ჩაღრმავებული ადგილის) ზემოთ.. ისე ერთი ცოტა მოზრდილია დანარჩენი პატარ-პატარები..
ისე ადრეც მქონდა და როცა გავესინჯე ასე მითხრეს როცა რაიმე ანთება მიდის ორგანიზმში მაშინ ჩნდებაო (მაგალითად თუ გრიპი გქონდა ეს ბოლო დღეებიო, ან თუნდაც თუ კბილი გაწუხებდაო) სხვა დროს ასე გასიებით არ შეუწუხებივარ


სავსებით შესაძლებელია, რომ ლავიწის ლიმფური კვანძების გადიდება არანაირ კავშირში არ იყოს ე. წ. ჩიყვთან (ისიც საკითხავია, რა იგულისხმება მაგაში, ჩვენში ხომ ყველაფერს ჩიყვს ეძახიან..), ასე რომ, ძალიან კარგად მოქცეულა ენდოკრინოლოგი, თუ თერაპევტთან გაგიშვა.

კბილთან ნაკლებ კავშირშია ლავიწის კვანძები, იმიტომ, რომ მაგ არეს არ წრეტს..კბილების შემთხვევაში პირველ რიგში ყბის ლიმფური კვანძები დიდდება ხოლმე, რადგან ყველაზე ახლოს ეგაა ინფექციის კერასთან.

ისე, მე შენს ადგილზე არ მივატოვებდი მაგას უყურადღებოდ..

Posted by: fsiqea 1 Jul 2010, 00:14
Solveig
QUOTE
სავსებით შესაძლებელია, რომ ლავიწის ლიმფური კვანძების გადიდება არანაირ კავშირში არ იყოს ე. წ. ჩიყვთან (ისიც საკითხავია, რა იგულისხმება მაგაში, ჩვენში ხომ ყველაფერს ჩიყვს ეძახიან..), ასე რომ, ძალიან კარგად მოქცეულა ენდოკრინოლოგი, თუ თერაპევტთან გაგიშვა.

კბილთან ნაკლებ კავშირშია ლავიწის კვანძები, იმიტომ, რომ მაგ არეს არ წრეტს..კბილების შემთხვევაში პირველ რიგში ყბის ლიმფური კვანძები დიდდება ხოლმე, რადგან ყველაზე ახლოს ეგაა ინფექციის კერასთან.


არა მაგაზე ვლაპარაკობდით და რო უთხარი გავესინჯე თქო, რაზეო და მაგაზე უპასუხე, რომ მე ჩიყვის გასასინჯად მივედი და ჯირკვლების ამბავში აზრზეც არ ვიყავი..
ხო კბილები მეც გამოვცრიცხე იმიტომ რომ აშკარად არ მაქვს პრობლემები..
ისე შენ როგორ ფიქრობ რისი ბრალი უნდა იყოს??
QUOTE
ისე, მე შენს ადგილზე არ მივატოვებდი მაგას უყურადღებოდ..

თუ არ გამიარა ეს ორი დღე მივაკითხავ smile.gif მადლობა..

Posted by: კახა251 1 Jul 2010, 00:28
fsiqea
რამდენი ხანია ეგ ჯირკვლები გაქ?

Posted by: fsiqea 1 Jul 2010, 01:32
kaxa251
QUOTE
fsiqea
რამდენი ხანია ეგ ჯირკვლები გაქ?

აი მაგას ნაღდად ვერ ვიტყვი წარმოდგენა არ მაქვს..შეიძლება რამოდენიმე თვე, შეიძლება წელიწადიც rolleyes.gif

Posted by: Solveig 1 Jul 2010, 01:45
fsiqea
პირველ რიგში, რამე ფარული ინფექცია იქნება გამოსარიცხი. სხვა არეშიც ხომ არ გაქვს გადიდებული, მაგალითად, იღლიაში, ყბის ქვეშ? რამე ინფექციური დაავადება ხომ არ გადაგიტანია ახლო წარსულში?
ზემოთ დაწერე-ყელი მტკივაო?

ანალიზები დაგჭირდება და შეიძლება, სხვა გამოკვლევებიც..მით უმეტეს, თუ დიდი ხანია, რაც გაქვს.

Posted by: @Caesar@ 1 Jul 2010, 02:24
გამარჯობათ, პირვალეად მოვხვდი აქ smile.gif მაინტერსებს ერთი დეტალი რაც შელიება ცოტა ხურუშიანად მოგეჩვენოთ მარა აქ გაზრდილი ადამიანისგან რაღაც პონტში ლოგიკურად, მაგარი პრაფანი ვარ ანესთეზიის საკითხში და ამიტომ ვკითხულობ,მაქვს ცხვირი საოპერაციო სასწრაფოდ, ნაფტიზინზე ვარ ექვსი წელია და არა შეჩვევის გამო.... ძგიდია გასასწორელები და სავარაუდოთ 90% ნიჟარებიც ამოსაჭრელი, ხო და სასურველია რო გონზე ვიყო და კონტაქტი მქონდეს ქირურგთან, ანუ საერთოს არ ვიკეთებ და არ ვიძინებ... ეხლა კითხვა( არ გამთათხოთ უმეცრობისთვის და პრიმიტიზმისთვის):სხვა რამოდენიმე იძულებით(სასწრაფო) ოპერაციაზე ვყოფილვარ არა ფხიზელი, და ცხვირზეც რო საკმარისად გრადუსიანი მივიდე დაშავდება რამე?

Posted by: Lollipop 1 Jul 2010, 20:38
აუ რაღაც მაინტერესებს და აქეთ გამომიშვეს. biggrin.gif მოკლედ მკერდის ირგვლივ რაღაც ლაქები მაქვს ხოლმე, ადრე მითხრეს მზემ იცისო და ცოტახნის წინ მეგობარს ვანახე და ეგ ღვიძლის ბრალიაო. ეს ლაქები ხან მქრთალია ხან იბურცება ხოლმე, იშვიათად რა. ხან ყავისფერია ხან ვარდისფერი. ბევრი არაა სულ 5-6 ცალი მაგრამ რავიცი მაინც ვიკითხავ ყოველი შემთხვევისთვის. ღვიძლზე შეიძლება რამე მჭირდეს?

Posted by: კახა251 2 Jul 2010, 01:38
@Caesar@
QUOTE
გამარჯობათ, პირვალეად მოვხვდი აქ  მაინტერსებს ერთი დეტალი რაც შელიება ცოტა ხურუშიანად მოგეჩვენოთ მარა აქ გაზრდილი ადამიანისგან რაღაც პონტში ლოგიკურად, მაგარი პრაფანი ვარ ანესთეზიის საკითხში და ამიტომ ვკითხულობ,მაქვს ცხვირი საოპერაციო სასწრაფოდ, ნაფტიზინზე ვარ ექვსი წელია და არა შეჩვევის გამო.... ძგიდია გასასწორელები და სავარაუდოთ 90% ნიჟარებიც ამოსაჭრელი, ხო და სასურველია რო გონზე ვიყო და კონტაქტი მქონდეს ქირურგთან, ანუ საერთოს არ ვიკეთებ და არ ვიძინებ... ეხლა კითხვა( არ გამთათხოთ უმეცრობისთვის და პრიმიტიზმისთვის):სხვა რამოდენიმე იძულებით(სასწრაფო) ოპერაციაზე ვყოფილვარ არა ფხიზელი, და ცხვირზეც რო საკმარისად გრადუსიანი მივიდე დაშავდება რამე

არანაირად არაა რეკომენდირებული ნებისმიერი სახის ნარკოზზე არაფხიზელ მდგომარეობაში გაკეთდეს ნებისმიერი ქირურგიული ჩარევა
Lollipop
Lollipop
QUOTE
აუ რაღაც მაინტერესებს და აქეთ გამომიშვეს.  მოკლედ მკერდის ირგვლივ რაღაც ლაქები მაქვს ხოლმე, ადრე მითხრეს მზემ იცისო და ცოტახნის წინ მეგობარს ვანახე და ეგ ღვიძლის ბრალიაო. ეს ლაქები ხან მქრთალია ხან იბურცება ხოლმე, იშვიათად რა. ხან ყავისფერია ხან ვარდისფერი. ბევრი არაა სულ 5-6 ცალი მაგრამ რავიცი მაინც ვიკითხავ ყოველი შემთხვევისთვის. ღვიძლზე შეიძლება რამე მჭირდეს?

დერმატოლოგთან მიდი!

Posted by: ALE-777 4 Jul 2010, 23:18
გახშირებული,ბოდიში და ბოყინი რისი ბრალი შეიძლება იყოს?

Posted by: Solveig 5 Jul 2010, 08:36
ALE-777
კუჭი/საყლაპავი მილი.
გასტროენტეროლოგს უნდა გაესინჯო.

Posted by: ALE-777 5 Jul 2010, 09:11
Solveig

ნერვული შეტევები მქონდა და ეგ რამე შუაშია ხო არაა?

კუჭზე გავიკეთე გამოკვლევა,მაგრამ ყველაფერი რიგზეა.

Posted by: კახა251 5 Jul 2010, 09:18
QUOTE
ნერვული შეტევები მქონდა და ეგ რამე შუაშია ხო არაა?

იცის მაგანაც.

Posted by: lilucia 5 Jul 2010, 10:57
ერთი კითხვა მაქვს, ეს ბოლო დღეებია, რაც ძალიან დაცხა,, დილაობით ყურები მაქვს გაგუდულისავით, ოღონდ ისე არა თითქოს საცობიაო, აი თავის მიმდებარე ადგილებში რაღაცნაირად გაბრუებული მაქვს sad.gif რისი ბრალია?

Posted by: კახა251 5 Jul 2010, 12:19
lilucia
ექიმს მიაკითხე და ამოასუფთავებინე ყურები გაჭედილი გექნება

Posted by: lilucia 5 Jul 2010, 12:44
ამ გაჭედვამ თაბრუს ხვევაც იცის? სხვანაირად მაქვს, გაჭედილივით არ მაქვს

Posted by: კახა251 5 Jul 2010, 16:55
QUOTE
ამ გაჭედვამ თაბრუს ხვევაც იცის?

არა
ეგ უვკე აღარ ვიცი :X

Posted by: EKIKULI 5 Jul 2010, 17:10
ვერ მეტყვით კეტკუტის ძაფმა რამდენ ხანში უნდა გაიწოვოს?

Posted by: Lacrimosa_ 6 Jul 2010, 21:33
kaxa251
ალერგიის საწინააღმდეგო რა შეიძლება დალიო, როცა თვითონ წამლებზეც ალერგია გაქვს?
ანუ რამე ბუნებრივი პროდუქტი?

Posted by: კახა251 7 Jul 2010, 18:15
Lacrimosa_
ნაღდად არ ვიცი მაგდენი მე პირადად მაქვს ალერგია და ალერტეკს ვსვავ

Posted by: nakanaka 8 Jul 2010, 13:53
შეგიძლიათ მითხრათ რას ნიშნავს როცა ჭამის დროს გამცივნებს?(ამ სიცხეშეშუიც კი).ანუ როცა საჭმელს ვჭამ მცივა.თან უკვე დიდი ხანია მაქვს დაბალი სიცხე 37 მდე ხანდახან 37.2 გლანდებს ვაბრალებდი ოპერაცია გავიკეთე და მის მერე 4 თვე გავიდა მარა სიცხეები მაინც მაქვს და რისი ბრალი სეიდლება იყოს?

Posted by: khosita20 9 Jul 2010, 00:44
გამარჯობათ

მაინტერესებს ლამბლია ჭიების არსებობამ თუ შეიძლება გამოიწვიოს კუჭში გასვლის დროს მცირედი სისხლდენა???

Posted by: კახა251 9 Jul 2010, 01:08
QUOTE
მაინტერესებს ლამბლია ჭიების არსებობამ თუ შეიძლება გამოიწვიოს კუჭში გასვლის დროს მცირედი სისხლდენა???

არა რამდენადაც ვიცი სისხლდენა არ უნდა გამოიწვიოს, ექიმი არ ვარ მაგრამ გამიგია რომ სისხლდენა არ იცის ჭიამ
* * *
QUOTE
დიდი ხანია მაქვს დაბალი სიცხე 37 მდე ხანდახან 37.2 გლანდებს ვაბრალებდი ოპერაცია გავიკეთე და მის მერე 4 თვე გავიდა მარა სიცხეები მაინც მაქვს და რისი ბრალი სეიდლება იყოს?

და ამ სიცხესთან ერთად რაიმე სიმპთომები გაქ? ან სისუსტე ან მადის დაქვეითება ან წონის მვეთრი კლება, ან ხველება, რპმელიმე თუ გაქ ამ ჩამონათვალიდან მიაკითხე თერაპევტს, (შემდგომ ის გადაწყვეტს ვისთან გაგიშვას) ხოლო ჭამის დროს ცივება არ ვიცი რისი შედეგი შეიელბა იყოს, ეგ სიცხეები კი ყურადღება მისაქცევია!

Posted by: nakanaka 9 Jul 2010, 18:52

QUOTE
და ამ სიცხესთან ერთად რაიმე სიმპთომები გაქ? ან სისუსტე ან მადის დაქვეითება ან წონის მვეთრი კლება, ან ხველება, რპმელიმე თუ გაქ ამ ჩამონათვალიდან მიაკითხე თერაპევტს, (შემდგომ ის გადაწყვეტს ვისთან გაგიშვას) ხოლო ჭამის დროს ცივება არ ვიცი რისი შედეგი შეიელბა იყოს, ეგ სიცხეები კი ყურადღება მისაქცევია!

წონაში დავიკელი 52 ვიყავი და 46 ვარ ეხლა.(ამ წონაზე გავცერდი ტიტქმის 2 წელია)მადას არ ვუჩივი.თერაპევტთან ვიყავი ,ფილტვებზე რენდგენი გადავიღე.თერაპევტმა ყელ-ყურტთან გამაგზავნა.გლანდები ამოვიჭერი მარა არაფერი არ შეიცვალა სიცხეები ისევ მაქ.

Posted by: კახა251 9 Jul 2010, 19:14
nakanaka
ხოო ესეიგი ტუბი არაა, მეც მაქ მასეთ ისიცხეები პმ მომწერე და შევადაროთ ვის როგორ და რანაირად და იქ აგიხსნი რეებზე გამოვეკვლიე biggrin.gif

Posted by: m-009 10 Jul 2010, 03:17
ეს 3 დღეა მარცხენა პეხის კუნთი მეჭიმება (ძაან მტკივნეულია) და რისი ბრალი შეიძლება იყოს?

Posted by: Ze Moreira 10 Jul 2010, 11:01
მეც სიცხეები მაქვს თან დიდი ხანია უკვე
ფილტვები გადავიღე, მუცლის ღრუ, ჩიყვი, სისხლი ანალიზი მაგრამ არაფერი ჩანს
საღამოობით მიწევს 37.3-37.5
რისი ბრალი შეიძლება იყოს? შეიძლება ნერვების?

Posted by: კახა251 10 Jul 2010, 14:25
QUOTE
ეს 3 დღეა მარცხენა პეხის კუნთი მეჭიმება (ძაან მტკივნეულია) და რისი ბრალი შეიძლება იყოს?

გააცნია რომელ კუნთზეა საუბარი! მუხლის უკან დაბლითა კუნთზეა ლაპარაკი?
* * *
Ze Moreira
QUOTE
მეც სიცხეები მაქვს თან დიდი ხანია უკვე
ფილტვები გადავიღე, მუცლის ღრუ, ჩიყვი, სისხლი ანალიზი მაგრამ არაფერი ჩანს
საღამოობით მიწევს 37.3-37.5
რისი ბრალი შეიძლება იყოს? შეიძლება ნერვების?
რამე სიმპტომები გაქ?

Posted by: m-009 10 Jul 2010, 16:17
QUOTE (kaxa251 @ 10 Jul 2010, 14:25 )
QUOTE
ეს 3 დღეა მარცხენა პეხის კუნთი მეჭიმება (ძაან მტკივნეულია) და რისი ბრალი შეიძლება იყოს?

გააცნია რომელ კუნთზეა საუბარი! მუხლის უკან დაბლითა კუნთზეა ლაპარაკი?
* * *

დიახ. მუხლის ქვედა კუნთი .(გადაღლილობას ვერ დავაბრალებ არ ვმო/ზრაობ იმდენს რომ დამეჭიმოს,თან ყოფილი სპორცმენი ვარ და ადრე უფრო ვიტვირთვებოდი )

Posted by: Ze Moreira 10 Jul 2010, 16:18
kaxa251
არა
მაგრამ სიცხე დღის მეორე ნახევარში სულ მაქვს არ იკლებს
წელიწადზე მეტია

Posted by: კახა251 10 Jul 2010, 16:25
QUOTE
დიახ. მუხლის ქვედა კუნთი .(გადაღლილობას ვერ დავაბრალებ არ ვმო/ზრაობ იმდენს რომ დამეჭიმოს,თან ყოფილი სპორცმენი ვარ და ადრე უფრო ვიტვირთვებოდი )

ხოდა კუნთი როდესაც იწყებს მოდუნებას მაშინ იწყება ტკივილები ზუსტად, ცოტა ხანში მაქსიმუმ ერთ კვირაში თუ არ გაგიარა მაშინ არ ვიცი რა ხდება.

Ze Moreira
QUOTE
kaxa251
არა
მაგრამ სიცხე დღის მეორე ნახევარში სულ მაქვს არ იკლებს
წელიწადზე მეტია

ღამეც გაქვს? საათობრივად დაწერე აბა

Posted by: Ze Moreira 10 Jul 2010, 16:48
kaxa251
ღამე არ ვიცი გასაზომად არ ავმდგარვარ შუაღამეს
დილით არ მაქვს, ეხლა უკვე 37.3

Posted by: between 3 and 32 10 Jul 2010, 16:51
QUOTE
საღამოობით მიწევს 37.3-37.5

http://forum.ge/?f=43&showtopic=34064843&st=0
იქნებ გამოგადგეს

Posted by: user_katie 11 Jul 2010, 22:25
აუუ გთხოოოოვთ მალე მიპასუხეთ ვინმეეემ რააა, იღლიაში ჯირკვალი გამომივიდა ოდნაავ მტკივა და რა შეიძლება იყოოს?

Posted by: ქინქლა 12 Jul 2010, 13:04
არ ვიცი რამდენად სწორად ვსვავ ამ თემაში კითხვას მაგრამ უფრო მიახლოებული ვერაფერი ვიპოვე. თუ არის რამე მსგავსი იქნებ გადამიმასამართოთ მერე.

მოკლედ 8, 5 თვის ორსული ვარ, ორი თვეა მარჯვენა იღლიაში შევნიშნე რომ მაქვს სიმაგრე, არის მცირე ზომის, ერთი სანტიმეტრის სიგრძის, კანს ზემოთაც ჩანს ვიზუალურად, უდნავ წამოწეულია იმ ადგილას ხორცი და მცირე ზომის ბორცვს ქმნის. ხელის დადების დროს არ მტკივა. ხსენი მაქვს მაგრამ მცირე რაოდენობით, თუ ხელს მოვუჭერ მაშინ გამოდის ხოლმე ხან გამჭვირვალე სითხე ხან თეთრი და ბლანტი.

მოკლედ დავიბენი, რა არის ეს სიმაგრე? შესაძლოა ეს ორსულობასთან იყოს კავშირში თუ მამოლოგი მჭირდება?

Posted by: gvarjila18 14 Jul 2010, 21:12
QUOTE (Lollipop @ 1 Jul 2010, 20:38 )
აუ რაღაც მაინტერესებს და აქეთ გამომიშვეს.  biggrin.gif მოკლედ მკერდის ირგვლივ რაღაც ლაქები მაქვს ხოლმე, ადრე მითხრეს მზემ იცისო და ცოტახნის წინ მეგობარს ვანახე და ეგ ღვიძლის ბრალიაო. ეს ლაქები ხან მქრთალია ხან იბურცება ხოლმე, იშვიათად რა. ხან ყავისფერია ხან ვარდისფერი. ბევრი არაა სულ 5-6 ცალი მაგრამ რავიცი მაინც ვიკითხავ ყოველი შემთხვევისთვის. ღვიძლზე შეიძლება რამე მჭირდეს?

დერმატოლოგთან მიდი ღვიძლი რა შუაშია მე ვერ ვხდები....კონსტიტუციის ქუჩაზე ალექსანდრე კაციტაძესთან მიდი....იქამდე '"კლოტრიმაზოლ"-ის მალამო წაისვი დღეში სამჯერ ერთი 4-5 დღე თუ უკეთესობა თუ იქნა 10 დღემდე ადი...გამიწყრებიან აქაური ექიმები yes.gif ეგრე წამალს როგორ ნისნავო მაგრამ მერწმუნე 4-5 დღე არაფერს გიზამს...სოკომ იცის ეგეთი ლაქები თუ გიშველა დაწერე და დანიშნულების მეორე სერიას გეტყვი....
* * *
QUOTE (user_katie @ 11 Jul 2010, 22:25 )
აუუ გთხოოოოვთ მალე მიპასუხეთ ვინმეეემ რააა, იღლიაში ჯირკვალი გამომივიდა ოდნაავ მტკივა და რა შეიძლება იყოოს?

იღლიის ჰიგიენურმა პროცედურებმა იცის ეგ yes.gif სავარაუდოდ ინფექცია არის შეჭრილი და ლიმფური ჯირკვალი გადიდდა. ქირურგს მიაკითხე (აქედან ანტიბიოტიკების დანიშვნა არ იქნება სწორი )სიცხე ხომ არ გაქვს? კონსერვატიულმა მკურნალობამაც შეიძლება შედეგი გამოიღოს...რაც მალე მით უკეთესი....სკალპელს გადაურჩები gigi.gif gigi.gif
* * *
QUOTE (fsiqea @ 30 Jun 2010, 19:26 )
kaxa251
QUOTE
რავი ან მომატებული ტემპერატურა, ან ხველა, ან სისუსტე მადის დაკარგვა წონასჰი კლება?

ცოტა ყელი მტკივასავით, ნერწყვის ყლაპვა მიჭირს, სისუსტე სულ მჭირს - ზარმაცი ვარ tongue.gif მადაზე და წონაზე პრობლემა არ მაქვს.. აი ტემპერატურა რავიცი, ყოველთვის ცვალებადი მაქვს ხოლმე.. ხან ძაან მცხელა ხან ძაან მცივა biggrin.gif აუ დღეს მთელი დღეა შეხვეული მაქვს, პირიქით ცუდად ხომ არ იმოქმედებს? ძაან შეშინებული ვარ sad.gif

კარიესი ხომ არ გაქვს ან სიბრძნის კბილი ხომ არ ამოგდის?

Posted by: კახა251 14 Jul 2010, 22:23
QUOTE
QUOTE
რავი ან მომატებული ტემპერატურა, ან ხველა, ან სისუსტე მადის დაკარგვა წონასჰი კლება?

ცოტა ყელი მტკივასავით, ნერწყვის ყლაპვა მიჭირს, სისუსტე სულ მჭირს - ზარმაცი ვარ  მადაზე და წონაზე პრობლემა არ მაქვს.. აი ტემპერატურა რავიცი, ყოველთვის ცვალებადი მაქვს ხოლმე.. ხან ძაან მცხელა ხან ძაან მცივა  აუ დღეს მთელი დღეა შეხვეული მაქვს, პირიქით ცუდად ხომ არ იმოქმედებს? ძაან შეშინებული ვარ 

კარიესი ხომ არ გაქვს ან სიბრძნის კბილი ხომ არ ამოგდის?

კაუცრად კარიესი რა შუაშია კაც ყელი ტკივა, კბილის ან ღრძილის რაიმე ნაწილი კი არა tongue.gif , რას ქვაი ცვალებადი ტემპერატურა, იზომე დღეში ორჯერ დღეში 3 დღე და გაარკვიე როგორია ცვალებადობა და დაწერე
* * *
QUOTE
არ ვიცი რამდენად სწორად ვსვავ ამ თემაში კითხვას მაგრამ უფრო მიახლოებული ვერაფერი ვიპოვე. თუ არის რამე მსგავსი იქნებ გადამიმასამართოთ მერე.

მოკლედ 8, 5 თვის ორსული ვარ, ორი თვეა მარჯვენა იღლიაში შევნიშნე რომ მაქვს სიმაგრე, არის მცირე ზომის, ერთი სანტიმეტრის სიგრძის, კანს ზემოთაც ჩანს ვიზუალურად, უდნავ წამოწეულია იმ ადგილას ხორცი და მცირე ზომის ბორცვს ქმნის. ხელის დადების დროს არ მტკივა. ხსენი მაქვს მაგრამ მცირე რაოდენობით, თუ ხელს მოვუჭერ მაშინ გამოდის ხოლმე ხან გამჭვირვალე სითხე ხან თეთრი და ბლანტი.

მოკლედ დავიბენი, რა არის ეს სიმაგრე? შესაძლოა ეს ორსულობასთან იყოს კავშირში თუ მამოლოგი მჭირდება?

ოოოო არ შეგეშინდეს მაგრამ ჯერ სასწრაფოდ წადი შენს გინეკოლოგთან და კითხე ეგ მაგით შეიძლება თუ არა გამოწვეულიყო, მერე წადი დერმატოლოGტან და გაესინჯე, ეგ იმაზეა დამოკიდებული შენი კინეკოლოგი თუ ჩათვლის საჭიროდ (მაგრამ მე მაინც გირჩევ მისვლას) თუ მაგათ ვერაფერი ვერ გაგიგეს თერაპევტთან მიდი! ყურადღება მისაქცევია ეგეთი სიმაგრე

Posted by: mag_ 14 Jul 2010, 22:28
QUOTE
ოფისის ტრამვატოლოგია

QUOTE
ოფისის მცირე ქირურგიული დაავადებები

biggrin.gif
ეს სკამს ფეხი წამოკრა ტრავმა და მაუსისგან თითზე ართრიტია თუ რას ქვია ოფისის? biggrin.gif

მე რა მაინტერესებს ეხლა. გადავხედე პირველ გვერდს ამ "ჯანმრთელობაში" და არაა თემა სათაურით - "როგორ ვებრძოლოთ კორუფციას ქართულ მედიცინაში", უფრო სწორად კრიმინალს, რადგან ხშირად ფინანსური მოგების სურვილი გადადის ისეთი პრეპარატების "შეტენვაში" პაციენტისთვის, რომელიც სერიოზულ ზიანს აყენებს მას, ან სულაც ადამიანს ტყვილა ჭრიან ... ძაან ბევრი მსგავსი ფაქტია, მე ეხლა ავტორობას ვერ ვიკისრებ ძაან იშვიათი სტუმარი ვარ ფორუმზე, თუმცა ამ საკითხის მწვავედ დაყენება ჩემის აზრით აუცილებელიააა!!!
ეხლა მაინდამაინც ამ თემაში რატო დავწერე. თერაპევტებს მინდა ვკითხო, ეს სოკოს რაღაც ტაბლეტები რომ არის, ქართული წარმოების, მართლა ასეთი უვნებელია ჯანმრთელობისათვის თუ "გვაბოლებენ" და ექიმები უსაღებენ ქართველ ფარმაცევტებს ძვირადღირებულ წამლებს ? ან რა უნდა უყო იმ ექიმს, რომელიც პაციენტს ეგრევე უწერს 10 აბს კონკრეტულ სოკოს წამალს, ისე რომ არც კი ეკითხება თუ აწუხებს თირკმელი ან ღვიძლი? ხო შეიძლება ერთმა აბმაც კი დედა უტიროს მის ღვიძლს. ეგრეა. ექიმი არ ვარ, მაგრამ ეხლა აქ თუ წავიდა ასეთი ექიმების დაცვა, ესეიგი აქ მხოლოდ ასეთი ექიმები პოსტავენ wink.gif

Posted by: კახა251 14 Jul 2010, 22:31
user_katie
ეს იღლიაში ჯირკვლები რა მომრავლდა რა არის, მაგეთ რამეებს, ან დერმატოლოგი სინჯავს როგორც ვიცი ან ონნკოლოგი.

Posted by: khosita20 14 Jul 2010, 22:31
მეგობარს აქვს ლიმფური ჯირკვალი გადიდებული, დაახლოებით 11\4 -ზე მარჯვენა ყურის უკან.... ულტრასონოგრაფია გაიკეთა და აღმოჩნდა ეგ... და რა მაინტერესებს იცი, თუ ეს არის ლიმფური ჯირკვლის ავთვისებიანი სიმსივნე (ლიმფოგრანულომატოზი) ულტრასონოგრაფიის დროს არ გამოჩნდებოდა???
და კიდევ ამ ტიპის სიმსივნე განკურნებადია თუ არა?

Posted by: კახა251 14 Jul 2010, 22:37
QUOTE
ეს სკამს ფეხი წამოკრა ტრავმა და მაუსისგან თითზე ართრიტია თუ რას ქვია ოფისის?

მე რა მაინტერესებს ეხლა. გადავხედე პირველ გვერდს ამ "ჯანმრთელობაში" და არაა თემა სათაურით - "როგორ ვებრძოლოთ კორუფციას ქართულ მედიცინაში", უფრო სწორად კრიმინალს, რადგან ხშირად ფინანსური მოგების სურვილი გადადის ისეთი პრეპარატების "შეტენვაში" პაციენტისთვის, რომელიც სერიოზულ ზიანს აყენებს მას, ან სულაც ადამიანს ტყვილა ჭრიან ... ძაან ბევრი მსგავსი ფაქტია, მე ეხლა ავტორობას ვერ ვიკისრებ ძაან იშვიათი სტუმარი ვარ ფორუმზე, თუმცა ამ საკითხის მწვავედ დაყენება ჩემის აზრით აუცილებელიააა!!!
ეხლა მაინდამაინც ამ თემაში რატო დავწერე. თერაპევტებს მინდა ვკითხო, ეს სოკოს რაღაც ტაბლეტები რომ არის, ქართული წარმოების, მართლა ასეთი უვნებელია ჯანმრთელობისათვის თუ "გვაბოლებენ" და ექიმები უსაღებენ ქართველ ფარმაცევტებს ძვირადღირებულ წამლებს ? ან რა უნდა უყო იმ ექიმს, რომელიც პაციენტს ეგრევე უწერს 10 აბს კონკრეტულ სოკოს წამალს, ისე რომ არც კი ეკითხება თუ აწუხებს თირკმელი ან ღვიძლი? ხო შეიძლება ერთმა აბმაც კი დედა უტიროს მის ღვიძლს. ეგრეა. ექიმი არ ვარ, მაგრამ ეხლა აქ თუ წავიდა ასეთი ექიმების დაცვა, ესეიგი აქ მხოლოდ ასეთი ექიმები პოსტავენ


არა ექიმი ნაღდად არ ვარ მაგრამ ნამდვილად მართალს იძახი იმ მხრივ რომ ადამიაანს არც ეკითხებიან რა აწუხებს ისე უნიშნავენ წამალს, უბრალოდ ცდაა რა, ჩვენ კი მაგ შემთხვევაში საცდელი კურდღლები გამოვდივართ! ოღონდ რეალურად საშიში ტესტების პაციენტები ვართ !
* * *
QUOTE
მეგობარს აქვს ლიმფური ჯირკვალი გადიდებული, დაახლოებით 11\4 -ზე მარჯვენა ყურის უკან.... ულტრასონოგრაფია გაიკეთა და აღმოჩნდა ეგ... და რა მაინტერესებს იცი, თუ ეს არის ლიმფური ჯირკვლის ავთვისებიანი სიმსივნე (ლიმფოგრანულომატოზი) ულტრასონოგრაფიის დროს არ გამოჩნდებოდა???
და კიდევ ამ ტიპის სიმსივნე განკურნებადია თუ არა?

ლიმფური ჯირკვალი მრავალი მიზეზით შეიძლება იყოს გადიდებული ეს არაა აუცილებელი ავთვისებიანი რამე იყოს, უბრალოდ ბევრი მიზეზი აქვს მათი აქ ჩამოთვლა შორს წაგვიყვანს, და რადგანაც ულტრასონოგრაფია გაიკეთა და ეგ აღმოჩნდა ონკოლოგტანაც იქნებოდით ხო? და რა გითხრათ ? თუ არ დააკონკრეტა რომ ეს არის ავთვისებიანი ლიმფურიგრანულომა, ესეიგი მიზეზი ჯერ ჯერობით უცნობია და ალბათ გაარკვევენ, !

Posted by: Kaifistka 16 Jul 2010, 16:05
ნაღვლის ბუშტი მტკივა საშინლად sad.gif((( ექიმტან ვიყავი, ეხო გადამიღეს ცოტა გადიდებულია, ბილიარული ნალექია ძალიან ცოტაო და ქრონიკული ქოლეცისტიტო დასკვნა. დამინიშნა წამლები ოდესტონი დღეში სამჯერ ჭამის წინ და ჭამის მერე ანასპანი და არ მშველის ხალხო მტკივა ძალიან, გავალ ახლა კედლებზეე sad.gif((((((((((((((((((((((

Posted by: frontushka 17 Jul 2010, 13:52
ამ ბოლო დროს ხშირად მაქვს დაბალი წნევა, მაგალითად გუშინ 70/45, დღეს 80/60, რა შეიძლება იწვევდეს ასე დაბალ წნევას? საერთოდ 110/70 მაქვს ხოლმე

Posted by: iza 17 Jul 2010, 13:58
user_katie

QUOTE
აუუ გთხოოოოვთ მალე მიპასუხეთ ვინმეეემ რააა, იღლიაში ჯირკვალი გამომივიდა ოდნაავ მტკივა და რა შეიძლება იყოოს? 


რა ქენი, იყავი ექიმთან?? მეც მქონდა ადრე და რექსონას ბრალი აღმოჩნდა, იმის შემდეგ მეზიზღება ანტიპერსპირანტები. კატეგორიულად გამაფრთხილა ჰემატოლოგმა არ მიეკაროვო და აღარც ვეკარები.

გამიქრა მერე დროთა განმავლობაში, მაგრამ წუხელ ისევ აღმოვაჩინე და ახლა ისევ ვნერვიულობ. იმ ექიმმა კი მითხრა, ძალიან ნაზი კანი გაქვს და ზაფხულში იცის ჯირკვლებმა გაღიზიანებაო, მაგრამ ეხლა სანამ მივალ და ავიღებ ანალიზებს, სული ამომხდება ვიცი ნერვიულობისგან...


Posted by: @ucnobinacnobi@ 19 Jul 2010, 18:13
მოკლედ აგარ შემიძლია, ზამთარია თუ ზაფხული გაციებული ვარ, სურდო მაქ. რისი ბრალია და როგორ გავუმკლავდე იქნებ მიპასუხოთ?

Posted by: კახა251 19 Jul 2010, 20:12
@ucnobinacnobi@
და არ გიფიქრია რო შეიძლება ალერგია გჭირდეს?

Posted by: @ucnobinacnobi@ 19 Jul 2010, 22:20

QUOTE
@ucnobinacnobi@
და არ გიფიქრია რო შეიძლება ალერგია გჭირდეს?

ზამთარშიც? თან ძლიერი ცხვირწყალი მაქ

* * *
სურდო მაქ მჟავა შემირცხვენიქ

Posted by: კახა251 20 Jul 2010, 10:07
@ucnobinacnobi@
მერე რა ალერგიამ იცის ეგეთ ირაღაცეები ან კიდე ცხვირის ლორწოვანის ანთებამ ყელ ყურს და ალერგოლოგს მიმართე

Posted by: ggz 20 Jul 2010, 17:16
შარდის საერთო ანალიზი:

ხვ.წონა ---1019 (1015-1025)
PH-- 5 (4,8-7,4)
ცილა --- ნეგ.
გლუკოზა --- ნეგ.
კეტონები --- ნეგ.
ურობილინოგენი-- ნეგ.
ბილირუბინი --- ნეგ.
ნიტრიტები --- ნეგ.
ლეიკოციტები --- 44-46 (0-4)
ერიტროციტები--- 0-4 (ნეგ)

ოქსალატები საშუალო რაოდენობით.
ბრტყელი ეპითელი 15-19 მხედვ/არეში.
ლორწო საშულო რაოდენობით.
ბაქტერიები (+) აღმოჩნდა, (+) აღმოჩნდა სოკო


რომელ სპეციალისტს მივაკითხო?

Posted by: inesa1936 23 Jul 2010, 12:09
ამიხსენით რა განსხვავებაა - კვებით ინტოქსიკაციას და ალიმენტურ ინტოქსიკაციას შორის?

Posted by: just smile19 23 Jul 2010, 12:14
inesa1936
QUOTE
კვებით ინტოქსიკაციას და ალიმენტურ ინტოქსიკაციას შორის?



ერთიდაიგივე არაა? spy.gif

Posted by: lel_tbeli 24 Jul 2010, 20:18
ხალხო ვინმემ მიპასუხოს გეხვეწებით

ადამიანს რომელსაც არმოუცინეს სიმსივნე თირკმელზე, ოპერაციის მერე მისი გადარჩენის შანსები როგორია??????????

Posted by: კახა251 25 Jul 2010, 09:15
lel_tbeli
გააჩნია რა სტადიის სიმსივნეა, ოპერაციას რადგანაც უკეთებენ მე მგონი დასაწყის სტადიებში უნდა იყოს ხო? ოგორც ვიცი დასაწყის სტადიებში სიმსივნეს სრულყოფილად აქრობენ, ქირურგსაც გააჩნია, სპეციალისტი უნდა იყოს და არა დამწყები.

Posted by: Tikk 29 Jul 2010, 19:28
კარგი პულმუნოლოგი მიმასწავლეთ რა, ძალიან მჭირდება.

Posted by: ANGELofDEATH 30 Jul 2010, 22:30
iza
გამარჯობა, რა პრობლემაც გაწუხებ იღლიასთან დაკავშირებით მეც იგივე მაქვს,,, თუ შეგიძლია მითხრა რომელ ექიმთან იყავი და რა ანალიზები აიღე. მე მითხრეს მამოლოგთან უნდა მიხვიდეო და რავიცი, მაგის კომპეტენციაში შედის ვითომ იღლია? 2 წელია ეგრე მაწუხებს. პატარა ადგილი მიმაგრდება და მებურცება და ხანდახან ძალიან მტკივა... ნეტა ლიმფური ჯირყვალი იყოს და სხვა არაფერი sad.gif დავიტანჯე

ანტიპერსპერანტის ნაცვლად რას ხმარობ? sad.gif არ შემიძლია ანტიპერსპერანტის გარეშე ამ სიცხეში

Posted by: shtori 31 Jul 2010, 00:06
შესაბამისი თემა ვერ ვნახე,ა მიტომ გადავწყვიტე აქ მეკითხა

ხშირად მომდის რომ წამის წინ რა გავაკეთე არ მახსოვს, მაგალითად გუშინ ჩანთიდან ამოვიღე ფული, უნდა გადამეხადა, შევკარი ჩანთა და ხელში არაფერი მეჭირა. არადა ამოვიღე. ვერ გავიგე სად წავიღე. ნუ მოკლედ სახლში მოვედი და ვნახე, რაღაც სხვა ჩანთაც მქონდა თან და იმ ჩანთაში ჩამიდია. როგორ, როდის?

კითხვისას თუ ისეთი სიტყვები შემხვდა რომლის ასოებიც ხვა სიტყვებშიც მეორდება, იმ სხვა სიტყვად ვკითხულობ.
ციფრებზეც, ეწერა რაღაც სიფრები, სხვა ეწერა სხვას ვხედავდი. არადა იმდენჯერ დავხედე, წავიკითხე კიდეც, გადავწერე. მეორე დღეს აღმოვაჩინე თურე შეცდომით დამინახავს. რომ წერია ვთქვათ 2, როგორ უნდა მოგეჩვენოს ის 0-ად?

წითელი და მწვანე ფერი. ვხედავ მწვანეს - ვამობ წიტელს და პირიქით. ოღონდ ამ შემთხვევაში ვიცი რომ წიტელი წიტელია და რომ შეცდომით ვამბობ ოღონდ, რომ ვიტყვი მერე მახსენდება ეგ.

ნუ ესე ადრე მჭირდა, მერე აღარ და ახლა ისევ ავირიე.

დავიჯერო რომ ძალიან დაჩლუნგებული ტვინი მაქვს?

Posted by: DCM 8 Aug 2010, 11:50
ხომ არ იცით რას შეიძლება გამოეწვია ქუსლების აქერცვლა და სისხლდენა

Posted by: კახა251 8 Aug 2010, 12:25
ცუდ ფეხსაცმელს, ან კიდევ გაუხეშებულ კანს, სიარული მოგიწია? ბევრმა სიარულმაც იცის, შეიძლება კანიც გაქ გამომშრალი ქუსლებზე, დამარბილებელი ისვი და სეხედე თუ კიდე განაგრძო დერმატოლოგს მიმართე. smile.gif)

Posted by: roamer 12 Aug 2010, 12:02
კითხვა მაქვს. რამდენიმე წელია არ მიჭამია ხორცი , ხომ არ არის ეს საშიში ორგანიზმისთვის ან კანისთვის და რომელი პროდუქტები შეიძლება მივიღო რომ ვიტამინები არ დამაკლდეს?

Posted by: forumeli007 12 Aug 2010, 15:04
არ ვიცოდი სად დამეწერა და აქ დავწერე.. თუ ვინმემ რამე იცის ამის შესახებ, გთხოვთ მიპასუხოთ, ამრთლა დავიტანჯე უკვე... მაინტერესებს რა დაავადება იწვევს ღებინების შეგრძნებას? ღებინების შეგრძნებას და არა ღებინებას... სულ ერთია ეს შეგრძნება ჭამის შემდეგაც მაქვს და მშიერზეც.. გადამივლის ხოლმე ხანდახან, მაგრამ ძირითადად მაქვს... არანაირ წამალს არ ვიღებ, რომ მისი ბრალი იყოს, მანამდე მქონდა და ნაღვლის ბუშტს აბრალებდნენ, ამოვიჭერი, ერთი პერიოდი მართლა გადამიარა, მაგრამ უკვე მეორე კვირაა შემაწუხა და შემაწუხა რასაც ქვია sad.gifsad.gifsad.gif
ძალიან გთხოვთ თუ რამე იცით ამის შესახებ იქნებ მიპასუხოთ??

Posted by: rock in rose 12 Aug 2010, 15:09
forumeli007

კუჭის წყლული ხომ არ გაქვს? გამოკვლეული გაქვს?

Posted by: forumeli007 12 Aug 2010, 16:41
არამგონია კუჭის წყლული რომ მქონდეს... გამოკვლეული კი არ მაქვს ნამდვილად, მაგრამ არ მაწუხებს... მწარესაც ვჭამ და მჟავესაც და არაფერზე არ მაძლევს რეაქციას... მარტო ეს ოხერი ღებინების შეგრძნება sad.gif

დიდი მადლობა გამოხმაურებისათვის, კიდევ რაზე მივიტანო ეჭვი?

Posted by: Lizikuna3 14 Aug 2010, 12:58
ტრიპტაზინზე რამე ინფორმაცია ხომ არ გაქვთ ვინმეს? ძალიან ძალიან მაინტერესებს! მადლობთ წინასწარ.

Posted by: Natuka NGN 14 Aug 2010, 13:35
roamer
რძის პროდუქტები.
forumeli007
ნარვლის ბუშტი ამოჭრილი გაქვს? ჩემმა მეზობელმა გაიკეთა და მერე იარესებიდ აეწყო ვიდრე ადრე. რარAც კუჭის პრობლემა წამოვიდა. თან შენ მწარესაც ჭამ და მჯავესაც თურმე და ახლა ნაღველა რომ აღარაა ღვიძლს და კუჭს აქვს მისი ფუნქცია გადანაწილებული. მგონი მაინც არასწორი კვების შედეგია და თორმეტგოჯაც შესამოწმებელი გაქვს. არ გაშინებ, უბალოდ ყურადღება უნდა მიაქციო.

Posted by: Lacrimosa_ 15 Aug 2010, 20:09
გამარჯობა
დედიკოს უკვე ერთი კვირაა ორივე ხელის ზურგზე და ფეხებზე აქვს დაწინწკლული, დახორკლილივით. თვითონ ალერგიას აბრალებს. მაგრამ ექიმთან არ მიდის ცოცხალი თავით, არადა მეც ვერ ვახერხებ, რომ გავყვე. შაბათ-კვირას კი არაფერი არ მუშაობს.
აშკარად არა აქვს ენერგია, ზოგადად საოცრად ენერგიული და ძლიერი ქალია, ახლა კი წევს და ძინავს.

Posted by: Natuka NGN 15 Aug 2010, 22:25
Lacrimosa_
როგორ არ მუშაობს? უნდ აიყოს რამე... და სიცხე ხომ არ აქვს? უნდა მიიყვანო ექიმთან, რა იცი რაა... იქნებ მოიწამლა რამეთი... აიძულე...

Posted by: Lacrimosa_ 15 Aug 2010, 22:48
Natuka NGN
სიცხე არა აქვს, თვითონ მშვენივრად გრძნობს თავს, მაგრამ მე ხომ ვატყობ, რომ ისეთი არაა, როგორიც ჩვეულებრივად.
რადგან ალერგიაზე მიიტანა ეჭვი, არც სარეცხს ვაკარებ და არც ჭურჭელს, მაგრამ არ გადაუვიდა.
ზოგადად როგორ არ არიან მორიგე ექიმები, მაგრამ მე ვინც მირჩიეს და ვიცი, რომ ჭკვიანია, მასთან მინდა მივიდეს, და ის შაბათ-კვირას არაა, კვირის მანძილზე კი ორივენი ვმუშაობთ და ვერ მივიყვანე.
მოწამვლა არა მგონია
ზღვიდან ჩამოსვლის მერე გამოაყარა და იქნება რაიმე კავშირში?

Posted by: Natuka NGN 15 Aug 2010, 23:01
Lacrimosa_
ლოკალიზაციასაც გააჩნია, ანუ ფეხზე ქვემოთაა, ტერფზე თუ სად. ასე რა შემიძლია გითხრა. შეიძლება რაიმე ინფექციაც იყოს, თან თუ ზღვაზე იყო. ორივე ხელსა და ფეხზე პარალელურად აქვს სიწითლე?

Posted by: კახა251 16 Aug 2010, 10:39
Lizikuna3
როგორც ვიცი ფიქოტროპული წამალია.

Posted by: Lizikuna3 16 Aug 2010, 11:38
kaxa251
კი ნამდვილად მასეა. აქ იმიტომ დავწერე რომ ფსიქიატრიაზე მგონი არ არის თემა გახსნილი.
ისე უკვე ვიპოვე საკმარისი ინფორმაცია.
მადლობა.

Posted by: iza 17 Aug 2010, 18:08
ANGELofDEATH

QUOTE
გამარჯობა, რა პრობლემაც გაწუხებ იღლიასთან დაკავშირებით მეც იგივე მაქვს,,, თუ შეგიძლია მითხრა რომელ ექიმთან იყავი და რა ანალიზები აიღე. მე მითხრეს მამოლოგთან უნდა მიხვიდეო და რავიცი, მაგის კომპეტენციაში შედის ვითომ იღლია? 2 წელია ეგრე მაწუხებს. პატარა ადგილი მიმაგრდება და მებურცება და ხანდახან ძალიან მტკივა... ნეტა ლიმფური ჯირყვალი იყოს და სხვა არაფერი sad.gif დავიტანჯე

ანტიპერსპერანტის ნაცვლად რას ხმარობ? sad.gif არ შემიძლია ანტიპერსპერანტის გარეშე ამ სიცხეში


მამოლოგი რა შუაშია??? მე ვიყავი ჰემატოლოგთან სისხლის გადასხმის ინსტიტუტში. დღესაც ვიყავი, ექიმი - მიშა გადაბაძე. არაფერი არ არისო მითხრა, ეტყობა ოფლი არ გამოდის გარეთ წესიერადო.
რაც შეეხება ანტიპერსპირანტებს, საერთოდ არ ვხმარობ, რადგან წლების წინ ამიკრძალა მიშამ მათი გამოყენება. ზუსტად რექსონამ გამიღიზიანა პირველად ჯირკვალი იღლიაში. შენ თვითონ გასინჯე ნაზად თითის ბალიშებით - თუ მოძრავია, არანაირი პრობლემა არ არის. სისხლის საერთო ანალიზში გამოჩნდება თუკი რაიმე მორფოლოგიური ხასიათის ცვლილებაა სისხლში, მაგრამ არამგონია
2kiss.gif

თუ ძალიან გაწუხებს ოფლიანობა, უბრალო შავი სარეცხის საპნით დაიბანე იღლიები. ეს მირჩია ექიმმა. დიდი რაოდენობით სოდას შეიცავს სარეცხი საპონი. ისე კი თუ მალე მიხვალ ჰემატოლოგთან და გაესინჯები, კარგი იქნება.

მაპატიე ამდენხანს რომ ვერ გიპასუხე, ახლა შემოვედი ამ თემაში. დაწერე რას იზავ 2kiss.gif

Posted by: marilis_ 17 Aug 2010, 18:12
არ ვიცი, ამ განყოფილებაში უნდა დამესვა, თუ სხვაგან ეს კითხვა:
გუშინ და რამოდენიმე დღის წინ კიდევ, კლდეებზე მომიწია ფოფხვამ და ხელები მაქვს სულ დასერილი + მექავება საშინლად.
სპირტით ვიწმენდ წარამარა, მაგრამ არ მშველის, მექავება მაინც და რა ვუყო, რამე წამალს ხომ არ მირჩევდით? cry.gif

Posted by: loom 18 Aug 2010, 13:31
გამარჯობათ. რამოდენიმე კვირაა რაც ძალიან მტკივა წელი, აი ქვემოთ რომაა, ხერხემალი რომ მთავრდება, поясница. უფრო მეტად მაშინ მტკივდება როდესაც ვზივარ. 10-15 ცუთზე მეტს ვერ ვზივარ ვდგები გავივლი გამოვივლი და შედარებით მივლის. რისი ბრალი შეიძლება რომ იყოს? და რატომ მაინცდამაინც მჯდომარეს მტკივა?

Posted by: კახა251 18 Aug 2010, 15:57
marilis_
სპირტით მოწმენდა არ გიშველის, თუ მაგ ჭრილობებში ჭუჭყი გაქ გირჩევ აფთიაქში ხსნარი იყიდო აგარ მახოსვს რა ქვია ჭუჭყს ამოიგაცილებს wink.gif wink.gif wink.gif მერე ნელ ნელა შეგიხორცდება ,სპირტით მასე ჭრილობის მოზელა არ შეილება.

Posted by: marilis_ 18 Aug 2010, 16:24
kaxa251
QUOTE
სპირტით მოწმენდა არ გიშველის, თუ მაგ ჭრილობებში ჭუჭყი გაქ გირჩევ აფთიაქში ხსნარი იყიდო აგარ მახოსვს რა ქვია ჭუჭყს ამოიგაცილებს wink.gif wink.gif wink.gif მერე ნელ ნელა შეგიხორცდება ,სპირტით მასე ჭრილობის მოზელა არ შეილება.

მადლობა, მომიტანეს უკვე ხსნარი 2kiss.gif

Posted by: roamer 19 Aug 2010, 10:34
კიდევ ერთი კითხვა მაქვს. სახეზე პერიოდულად მაქვს გამონაყარი, რომელიც რჩება ლაქების სახით და გვიან გადადის. მითხრეს შეიძლება შინაგანი ორგანოების გაღიზიანებით იყოს გამოწვეული. ამისთვის უკვე რამოდენიმე დღეა გამოვრიცხე ცომეული, ტკბილეული, შემწვარი საკვები... შეიძლება ამან შედეგი გამოიღოს თუ ნაღვლის ბუშტია გაღიზიანებული?

Posted by: Temii 20 Aug 2010, 12:46
ვინმე კარგი თერაპეტი არ გეგულებათ სამე რომ მირჩიოთ...ვისაც ნამდვილად ავქს სამედიცინო განათლება ....

Posted by: ane mekeshi 21 Aug 2010, 17:11
არ ვიცი, აქ უნდა ვიკითხო თუ არა, მაგრამ თუ ვინმემ იცით, მიპასუხეთ, რა: რამდენად საშიშია კანში ჩარჩენილი კაქტუსის ეკლები? რამდენიმე წვრილი ეკალი ვერაფრით ამოვიღე sad.gif

Posted by: კახა251 22 Aug 2010, 10:25
ane mekeshi
შეიძლება დაგიჩირქდეს და ინფექცია წავიდეს ანუ. : ) ამოიღე როგორმე, დამიჯერე მოგიწევს მაინც : ). თუ შენ ვერ იღებ ადგილობრივ საავადმყოფოში ნებისმიერი ქირურგი ამოგიღებს, : )

Posted by: lady t 23 Aug 2010, 17:43
მეც არ ვიცი აქ უნდა ვიკითხო თუ არა,მაგრამ მაინტერესებს რა სირთულე შეიძლება მოყვეს ალკოჰოლის მიღებას,როცა წამალი (ანტიბიოტიკი) აქვს ადამიანს მიღებული დაახლოებით 8 საათით ადრე?

Posted by: Natuka NGN 23 Aug 2010, 18:09
lady t
არის პრეპარატები, რომელსაც პირდაპირი უკუჩვენება აქვს ალკოჰოლი, ზოგიერთი ანტიბიოტიკებსაც აქვს ეს მითითება. საერთოდ არაა სასურველი როგორც ვიცი წამლების მირებისას ალკოჰოლური თრობა. სირთულის რა გითხრა, შეიძლება შეამციროს წამლის მოქმედება.

Posted by: lady t 23 Aug 2010, 18:28
Natuka NGN
გმადლობთ.სტომატოლოგმა დამინიშნა ეს ანტიბიოტიკი,არადა უკვე 7 წელია ამ დღეს ვიკრიბებით მეგობრები და ცოტას გადავკრავთ ხოლმე....ნებისმიერ შემთხვევაში ძალიან დიდი მადლობა პასუხისთვის.

Posted by: libra_girl 24 Aug 2010, 01:22
ხალხო გასტროენტეროლოგი მირჩიეთ ვინმე ნორმალური..ვეძებ ოღონდ კარგსsmile.gif

Posted by: Gora Mborgali 24 Aug 2010, 14:11
მგონი ნაღველი მაქვს გაღიზიანებული და რას მირჩევთ? მადლობა წინასწარ... ან რა არ უნდა ვჭამო და ა.შ.

Posted by: Natuka NGN 24 Aug 2010, 14:32
Gora Mborgali
არის თემა ნაღვლის ბუშტზე და გადახედე. დაძებნე განყოფილების გზამკვლევის თემაში.

Posted by: pleita 24 Aug 2010, 23:58
ხალხო მალე მითხარით რა, საშინლად მტკივა კუჭი და გული მერევა, მთელი დღეა ვსუსნაობ და რეები ავურიე ერთმანეთში აღარ ვიცი, წეღან ცოტა ამოვიღე,მაგრამ არ მომეშვა და თითის ჩატენვები აღარ შემიძლია , რა გავაკეთო მითხარით რა help.gif

Posted by: კახა251 25 Aug 2010, 08:35
სითხე მიიღე რაც შეიძლება ბევრი არაფერი არ ჭამო სანამ გულისრევის შეგრძნება არ გაგივლის შემდეგ კი 1 დღიანი მსუბუქი დიეტა დაიცავი, დილით ჩაი, ყველი და პური ჭამე ხოლო შემდეგ მსუბუქი სადილი გრეჩიხა, ვახშამზეც ჩაი დალიე მეორე დღეს თუ გინდა 800 ცალი ხინკალი ჭამე კარგად იქნები უკვე biggrin.gif

Posted by: geosubtitles 27 Aug 2010, 18:06
არ ვიცი სწორედ მივმართავ თუ არა თემას, მარა ერტი შეკითხვა მაქვს:

გუშინ ღამე მოვყევი წვიმაში, ერთი ნახევარი საათი დაახლოებით წვიმაში ვიყავი, მაგრად შემცივდა, განსაკუთრებით გვერდებზე და ფეხებზე.
სახლში მოსვლისთანავე ცხელი დუში გადავივლე და ჩავწექი.
ღამე ცუდად მეძინა, სულ ვტრიალებდი და მეღვიძებოდა, მგონი სიცხეც მქონდა და ძველბში ვგრძნობდი უცნაურად (თითქოს ძვლებში მიღიტინებდა).
დღეს ვარ მაგრად დასუსტებული და სიცხე მაქ 37 და 5. ამ წუთას გავიზომე.

გამივლის თავისით თუ რამე პროფილაქტიკას მივმართო?


Posted by: კახა251 28 Aug 2010, 11:50
ფილტვების არეში ტკივილი ხომ არ გაქვს? ძვლების "ღიტინი" სერიოზული გაციების ნიშანია როგორც ვიცი, გახველებს? თუ გახველებს ან ფილტვების არეში გრძნობ ტკივილს მაშინ მიდი ექიმთან და მოასმენინე თუ არადა ჩაწექი ცხელი ჩაი და ოფლი მოიდინე, აფთიაქში გაციების საწინააღმდეგო წამალი იყიდე და სვი სითხეები ბევრი, ჭამე საჭმელი კარგად და არ იქვე თუ გინდა უმჯობესია ფეხზე იდგე ოღონდ არ შეგცივდეს და რამე, აბა შენ იცი ჯანმრთელობას გისურვებ! : )

Posted by: natia_m77 29 Aug 2010, 22:28
ინფექციონისტი ხართ რომელიმე? რაღაც კითხვა მაქვს

Posted by: Megi-meghan 4 Sep 2010, 12:12
var 55 wlis da didi xania temperatura makvs 35.2-35.8. tu amivida temperatura 37-mde,tavs ukve cudad vgrdznob.risi brali unda iyos es?sashishia?

Posted by: ninca 5 Sep 2010, 19:18
გამარჯობააა
მაინტერესებს, რამდენად შეიძლება ყოველ დღე მოხარშული კვერცხის ჭამა, 3 ცალის მაინც. დიეტაზე მინდა გადასვლა და იქ შედის...

Posted by: Nensi_1988 7 Sep 2010, 02:53
ბოლო 2 დღეა შარდვა მაქვს გახშირებული ეტყობა ბევრი სითხისა და საზაამთროს გამო მაგრამ ამას დაემატა ის რომ ყოველი მოშარვის დროს ყრუდ მტკივა ცალი თირკმელი და შარდის ბუშტის დაცლის მერე ეს ტკივილი ქრება. რასთან იქნება დაკავშირებული და რა გავაკეთო? საშიშია?

Posted by: rock in rose 8 Sep 2010, 11:00
ninca
QUOTE
მაინტერესებს, რამდენად შეიძლება ყოველ დღე მოხარშული კვერცხის ჭამა, 3 ცალის მაინც. დიეტაზე მინდა გადასვლა და იქ შედის...


ყოველდღე არ შეიძლება ნამდვილად no.gif ალერგიული რეაქციები შეიძლება გამოიწვიოს


Posted by: _SisterofNight_ 9 Sep 2010, 17:08
ფეხების ტკივილის გამო გავიკეთე პროტრომბინის ანალიზი, პასუხი წამოვიღე დღეს, მაგრამ სამწუხაროდ ექიმი აღარ დამხვდა და სანამ მივალ უფრო სწორად სანამ ჩავწერები იქნებ მიპასუხოთ, ჩემი მოკლე ჭკუით მომატებული მაქვს პროტრომბინი.

პროთრომბინის ინდექსში შედეგი მაქვს 103.3 და ნორმაში წერია 70-100

საერთაშორისო ნორმალიზებული ფარდობა 0.98-ია შედეგი და ნორმაა 1.0-1.21

დანარჩენი ნორმის ფარგლებშია. რამე უნდა დავლიო?

Posted by: shtori 12 Sep 2010, 22:43
ყელთან რომ არის ჩაღრმავება (სახელი არ ვიცი, სურათს დავდებ) მანდ 2 თვე იქნება პერიოდულად მტკივდება ხოლმე, გარედან, ზოგჯერ ხელის დაჭერითაც მტკივა. ჩაქცევა ან შესიება არ ჩანს. უბრალოდ მტკივა თან ძალიან, რამდენიმე წუთის განმავლობაში, გადამივლის და ისევ... რამდენიმე დღის შემდეგ ისევ მახსენებს თავს.
რისი ბრალი იქნება?



კი თუ არა?..

Posted by: Disk 15 Sep 2010, 01:34
გამარჯობათ,

მაქვს ყურის პრობლემა , როდესაც მესმის ხმამაღალი ხმაური (მუსიკა, ხმამაღალი ლაპარაკი) მარცხენა ყურში მეწყება ჩხრიალი.ექიმთან ვიყავი და ვერ მიხვდა რაშია საქმე , მე მეგონა საცობი მქონდა , მაგრამ ჩამხედა და მითხრა საცობი არ გაქვსო.

აღარ ვიცი რა ვქნა

Posted by: wurbela 18 Sep 2010, 19:42
კისლატის შესუნთქვამ შეიძლება იყოს კუჭის აშლლის მიზეზი, წყალიზე გადის ადამიანი, მოწამვლას აქვს ასეთი სიმპტომები?

Posted by: be happy 20 Sep 2010, 00:08
არის აქ ვინმე?.... ერთი კვირაა სიცხეები მაქვს, ექიმთან ვიყავი არც ფილტვებზე,არც სხვაგან რავი არაფერიაო. ვირუსი გექნებაო. სიცხე არ მიწევს 37,2 მაქვს ბოლო დღეებია. რა მჭირს ნეტა help.gif

Posted by: კახა251 20 Sep 2010, 10:01
be happy
საერთოდ მთელი დღე და ღამე გაქ? თუ როგორი განრიგით გაქ სიცხეები (პ.ს მე 10 თვეა დღის განმავლობაში მას მიწევს სიცხე გამოკვლევებმა ვერაფერი აჩვენა საეჭვო) მაინც სისხლის ანალიზზს თუ აიღებ კაის იზავ : ))

Posted by: be happy 20 Sep 2010, 11:50
kaxa251
QUOTE
საერთოდ მთელი დღე და ღამე გაქ? თუ როგორი განრიგით გაქ სიცხეები (პ.ს მე 10 თვეა დღის განმავლობაში მას მიწევს სიცხე გამოკვლევებმა ვერაფერი აჩვენა საეჭვო) მაინც სისხლის ანალიზზს თუ აიღებ კაის იზავ : ))

შუადღიდან უკვე მიწევს ზემოთ, იწყება 37, 37,2, და მაქსიმუმ ღამე 37,5. ეხლა მივდივარ და გავიკეთებ ანალიზს
user.gif

Posted by: კახა251 20 Sep 2010, 13:38
be happy
ერთი კვირა კიდე არაფერია ჯერ ჯერობით გამოსაკვლევი მაგრამ ალბათ თუ გაგრძელდა ეს სიცხეები ფილტვებზეც მოგიწევს მისვლა!

Posted by: be happy 20 Sep 2010, 19:12
kaxa251
QUOTE
ერთი კვირა კიდე არაფერია ჯერ ჯერობით გამოსაკვლევი მაგრამ ალბათ თუ გაგრძელდა ეს სიცხეები ფილტვებზეც მოგიწევს მისვლა!

ფილტვებზეც დღეს გადავიღე და ისეთი არაფერი მჭირს, უბრალოდ ,,ჰილუსები,, თუ რაღაც (ვერ გავიგე)უსტრუქტუროაო და აუგმენტინი დალიეო. და ჰემოგლობინი მაქვს დაბალი((( ხვალ კიდე შარდის ანალიზიო. იმედია რამე ინფექციური არ მჭირს
user.gif

Posted by: Nanuka444 22 Sep 2010, 17:23
be happy
QUOTE
ერთი კვირაა სიცხეები მაქვს, ექიმთან ვიყავი არც ფილტვებზე,არც სხვაგან რავი არაფერიაო. ვირუსი გექნებაო. სიცხე არ მიწევს 37,2 მაქვს ბოლო დღეებია. რა მჭირს ნეტა

არ ვარ ექიმი, უბრალოდ მსგავსი სიტუაცია მქონდა და გადავწყვიტე მეპასუხა.
თვეში გარკვეული დღეები სიცხე მქონდა 37-37.5 შუალედში განუწყვეტლივ, დილით უფრო ნაკლები და დღის ბოლოსკენ იწევდა მაღლა. არაფერი მტკიოდა, არც გაციებული ვიყავი.
დიდი ხნის ანალიზების და გასინჯვების შემდეგ მიზეზი აღმოჩნა ძალიან ბანალური: ოვულაციის შემდეგ ფაზაში, როცა გამოიყოფა პროგესტერონი, ტემპერატურა იწევს მაღლა, მენზის მოსვლის წინა დღეს უბრუნდება ნორმას და ოვულაციამდე ეგრეა.

* * *
ეხლა ჩემი შეკითხვა, ძალიან გთხოვთ იქნებ კომპეტენტურმა ადამიანმა/ექიმმა მიპასუხოს:
ორი დღე მქონდა სიცხე, I დღე 37, მეორე დღეს 37-38.5, დღეს უკვე აღარ მაქვს, მაგრამ მაქვს ოფლიანობა, მიჭირს სუნთქვა (ვხმარობ ცხვირის წვეთებს) და ამავდროულად ყრუდ მტკივა გულმკერდის არეში და ბეჭები. მაინტერესებს, ფილტვების ანთების ნიშნები ხომ არ არის???

პ.ს. ვარ 28 კვირის ორსული და შესაბამისად რენტგენს ვერ გადავიღებ.

Posted by: be happy 22 Sep 2010, 21:11
Nanuka444
QUOTE
არ ვარ ექიმი, უბრალოდ მსგავსი სიტუაცია მქონდა და გადავწყვიტე მეპასუხა. თვეში გარკვეული დღეები სიცხე მქონდა 37-37.5 შუალედში განუწყვეტლივ, დილით უფრო ნაკლები და დღის ბოლოსკენ იწევდა მაღლა. არაფერი მტკიოდა, არც გაციებული ვიყავი. დიდი ხნის ანალიზების და გასინჯვების შემდეგ მიზეზი აღმოჩნა ძალიან ბანალური: ოვულაციის შემდეგ ფაზაში, როცა გამოიყოფა პროგესტერონი, ტემპერატურა იწევს მაღლა, მენზის მოსვლის წინა დღეს უბრუნდება ნორმას და ოვულაციამდე ეგრეა.

არც მე მტკივა არაფერი. უბრალოდ პატარა ანთება მქონია და წამლებს ვსვამ. გამივლის ალბათ მეც და შენც. ექიმთან ჯობია მისვლა
2kiss.gif

Posted by: maka995 24 Sep 2010, 11:58
QUOTE
QUOTE
ერთი კვირაა სიცხეები მაქვს, ექიმთან ვიყავი არც ფილტვებზე,არც სხვაგან რავი არაფერიაო. ვირუსი გექნებაო. სიცხე არ მიწევს 37,2 მაქვს ბოლო დღეებია. რა მჭირს ნეტა


არ ვარ ექიმი, უბრალოდ მსგავსი სიტუაცია მქონდა და გადავწყვიტე მეპასუხა.
თვეში გარკვეული დღეები სიცხე მქონდა 37-37.5 შუალედში განუწყვეტლივ, დილით უფრო ნაკლები და დღის ბოლოსკენ იწევდა მაღლა. არაფერი მტკიოდა, არც გაციებული ვიყავი.
დიდი ხნის ანალიზების და გასინჯვების შემდეგ მიზეზი აღმოჩნა ძალიან ბანალური: ოვულაციის შემდეგ ფაზაში, როცა გამოიყოფა პროგესტერონი, ტემპერატურა იწევს მაღლა, მენზის მოსვლის წინა დღეს უბრუნდება ნორმას და ოვულაციამდე ეგრეა.

მამაკაცებმა რა უნდა ქნან ნეტავ ასეთ შემთხვევაში? 5 დღეა ჩემს მეუღლეს 37,2 აქვს სიცხე, საღამოობით უფრო, დღისით არ აქვს.

Posted by: blokator 26 Sep 2010, 12:41
QUOTE
საერთოდ მთელი დღე და ღამე გაქ? თუ როგორი განრიგით გაქ სიცხეები (პ.ს მე 10 თვეა დღის განმავლობაში მას მიწევს სიცხე გამოკვლევებმა ვერაფერი აჩვენა საეჭვო) მაინც სისხლის ანალიზზს თუ აიღებ კაის იზავ : ))


შუადღიდან უკვე მიწევს ზემოთ, იწყება 37, 37,2, და მაქსიმუმ ღამე 37,5. ეხლა მივდივარ და გავიკეთებ ანალიზს

ვაახ ეს სიცხეები,ყველას აწუხებს და არვინ ჩანს ჰორიზონტზე პასუხის გამცემი... მე უკვე დამტანჯა,მომბეზრდა თერმომეტრთან ერთად დაძინება გაღვიძება.და ეგეც რომ არ იყოს ფსიქიკა შემერყა,ასე მგონია ყველაფერი მჭირს ტუბიან შიდსიან ებშტეინ ბარიანად და ა.შ....

Posted by: be happy 27 Sep 2010, 12:48
blokator

QUOTE
ვაახ ეს სიცხეები,ყველას აწუხებს და არვინ ჩანს ჰორიზონტზე პასუხის გამცემი... მე უკვე დამტანჯა,მომბეზრდა თერმომეტრთან ერთად დაძინება გაღვიძება.და ეგეც რომ არ იყოს ფსიქიკა შემერყა,ასე მგონია ყველაფერი მჭირს ტუბიან შიდსიან ებშტეინ ბარიანად და ა.შ....

რამდენი ხანია სიცხეები გაქვს? თერმომეტრი ჩემი საუკეთესო მეგობარი გახდა biggrin.gif

Posted by: JERICHO 27 Sep 2010, 22:45
ფილტვების ანთების დროს რომ ანალიზებია, ჰემოგლობინის და ა.შ.
იმ ანალიზების გაკეთებისას გამოჩნდება თუ არა ის რომ რამოდენიმე დღის წინ მოწიე ? (ხვდებით ალბათ რაც)

Posted by: blokator 27 Sep 2010, 23:07
QUOTE (be happy @ 27 Sep 2010, 12:48 )
blokator

QUOTE
ვაახ ეს სიცხეები,ყველას აწუხებს და არვინ ჩანს ჰორიზონტზე პასუხის გამცემი... მე უკვე დამტანჯა,მომბეზრდა თერმომეტრთან ერთად დაძინება გაღვიძება.და ეგეც რომ არ იყოს ფსიქიკა შემერყა,ასე მგონია ყველაფერი მჭირს ტუბიან შიდსიან ებშტეინ ბარიანად და ა.შ....

რამდენი ხანია სიცხეები გაქვს? თერმომეტრი ჩემი საუკეთესო მეგობარი გახდა biggrin.gif

უკვე 2 თვეზე მეტია,გუშინ მომცა დიდი სიცხე 39,4 მქონდა მთელი ღამე მაგრამ ეგ სხვა რამის გამო.დაბალი სიცხეები უკვე 2თვეზე მეტია მაქვს

Posted by: კახა251 28 Sep 2010, 12:01
JERICHO
ანუ შენ გულისხმობ რომ სისხლის ანალიზის დროს გამოცნდება თუ არა >>>ის?<<< biggrin.gif არ ვიცი მგონი მაგას შარდის ანალიზით უფრო ადგენენ სისხლის ანალიზზე არ გამიგია
'
* * *
blokator
შენ მაგ ვითარებაში 2 თვეა ხარ და მე მასე 7 თვე ვიყავი სანამ ძირითადი რაღაცეები არ გამოვიკვლიე biggrin.gif ეხლა ცოტა დავწყნარდი მაგრამ იანც ვარ მისასვლელი 1-2 ჯერ ექიმთან სისხლის ანალიზი მ -4 განმოერებით სისხლის გადასხმის ინსტუტში biggrin.gif

Posted by: Ia-forumze 28 Sep 2010, 12:36
ვიმშობიარე 3 სექტემბერს

მე-12 დღეს დამეწყო სიცხეები, 38.2-მდე ადიოდა,
ჯერ რძისგან გვეგონა, მერე მასაჟისტმა გამორიცხა
ეს აქედან არ მოდისო.

წავედი გავიკეთე ეხო, საშვილოსნოში ყველაფერი წესრიგშია.

ჩავაბარე შარდის ანალიზი და გამოჩნდა ლეიკოციტები "დიდი რაოდენობით"
და ნიტრიტები
ასევე ნალექი, რის გამოც ვერ მოხერხდა ლეიკოციტების ზუსტი რაოდენობის განსაზღვრაო.

მითხრეს პიოლონეფრიტიაო (არ ვიცი თუ სწორად ვწერ)
დამინიშნეს ანტიბიოტიკი აპრიდი (ბავშვს ძუძუს ვაწოვებ და ეს ყველაზე უსაფრთხოაო)
იმ დღეს მხოლოდ ერთხელ მომცა სიცხე 37.9 და 10 წუთში თავისით დამიწია, მეორე-მესამე
დღეს აღარ მქონია და მომიხსნეს. ანუ გამომივიდა 3 დღე დალევა
ახლა ჩავაბარე განმეორებით ანალიზი, ნიტრიტები და ნალექი აღარ არის, მაგრამ
ლეიკოციტებზე წერია 10-25 leyc/u 1 +
ხვალ მივდივარ თირკმელებზე ეხოს გადასაღებად
მაგრამ მანამდე მაინტერესებს ლეიკოციტების ეს რაოდენობა ძალიან ბევრია?

იქნებ 2 სიტვით შეგეფასებინათ ჩემი მდგოამრეობა.

პს. სიცხეები არ მაქვს, ან რაიმე სხვა სიმპტომი

წინასწარ გიხდით მადლობას

Posted by: Solveig 29 Sep 2010, 01:46
Ia-forumze
http://www.anytestkits.com/utk-leukocytes-in-urine.htm

პიელონეფრიტი არის თირკმლის ანთების ერთ-ერთი ფორმა, შენს შემთხვევაში როგორც ჩანს, ბაქტერიით გამოწვეული (რაკი ანიტიბიოტიკმა იმოქმედა) და ლეიკოციტების მომატება შარდში არაა გასაკვირი.
ლეიკოციტები ასე უცებ არ დაიკლებს. მხოლოდ 1 დღეა, რაც სიცხე აღარ გაქვს, ხომ? ნელ-ნელა ჩადგება ნორმაში ეგეც.
თუმცა, ექიმთან მისვლა რა თქმა უნდა, აუცილებელია.

Posted by: freeandhappy 29 Sep 2010, 10:09
არ ვიცი სწორად ვსვავ თუ არა შეკითხვას. კითხვა მაქვს ასეთი, პირის ღრუდან რა შემთხვევაში არის სისხლდება, ხველებისას ან ღებინების დროს თუკი აქვს ადგილი მას. რა შეიძლება იყოს მიზეზი?

Posted by: LichGuarD 29 Sep 2010, 11:35
გამარჯობათ ექიმებო არაექიმებო და ავადმყოფებო smile.gif)) რაგაც კიტხვა მაკვს biggrin.gif მოკლედ მესამე კვირაა სიცხე მაქვს 37 მაქს 37,5 ხოდა რისი ბრალი შეიძლება იყოს საქართველოში არ ვარ და მაინც ცოტა არ იყოს და მეშინია biggrin.gif სავადმყოპოში ვიყავი და გამკეთეს სისხლის ანალიზი X რეი მოკლედ ფილტვები კარგ მდგომარეობაში მაკვს (რაც ძალიან მიკვირს დგესჰი 2 კოლოპ სიგარეტს ვეწევი) ასევე ანთებიტი პროცესიც არ არის biggrin.gif ექიმმა მიტხრა ერტი კვირაც მოითმინეო და თუ არ გაგიარა ისევ მოდიო :\ აშკარად არ გამიარა sad.gif ხო კიდევ რაგაც ახალი სიმფტომები მაქვს დისკომფორტი მაკვს ყელის პონტში ანუ ოდნავ ნერცყვის გადყლაპვა მიჭირს..... იკნებ ვინმემ დამამშვიდოს და მიტხრას რომ სამშობლოს ცოცხალი დავუბრუნდები biggrin.gif

Posted by: JERICHO 30 Sep 2010, 20:06
LichGuarD

მე ექიმი არ ვარ, მაგრამ ეგრე მქონდა ზუსტად, თავიდან ფილტვების ანთება ეგონათ და ბრონხიტი ყოფილა

Posted by: JERICHO 2 Oct 2010, 16:36
რა მაინტერესებს ბრონხიტის დროს მოწევა როგორ მოქმედებს ?

Posted by: texasuri jleta benzoxerxit 3 Oct 2010, 22:08
QUOTE (JERICHO @ 2 Oct 2010, 16:36 )
რა მაინტერესებს ბრონხიტის დროს მოწევა როგორ მოქმედებს ?

გადასარევად. მალამოსავით მოქმედებს gigi.gif boli.gif

Posted by: JERICHO 5 Oct 2010, 23:27
texasuri jleta benzoxerxit
მაგას რაც არ უნდა გასაკვირი იყოს მეც მივხვდი biggrin.gif
მაინტერესებს განსაკუთრებულად რთულად ხომ არ მოქმედებს ბრონხიტის შემთხვევაში

Posted by: Archil Mikelashvili 6 Oct 2010, 14:59
გამარჯობა,

ბოლო 3-4 დღეა შევამჩნიე ლაქები ენაზე. მითხრეს, რომ ეს შესაძლოა ნაღვლის ბუშტის გაღიზიანებით იყოს გამოწვეული, რომელიც მართლა მაწუხებს. მაინტერესებს, ხომ არ შეიძლება კუჭზე ან ღვიძლზეც იყოს პრობლემა? რა პროფილის ექიმს უნდა მივაკითხო ასეთ შემთხვევაში?

Posted by: sophia semionovna 6 Oct 2010, 19:05
მარჯვენა ხელის თითზე სისხლჩაქცევა მაქვს., ადრეც მქონდა მარცხენაზე., მხოლოდ თითებზე მემეართება თან მტკივა ის ადგილი . რა ხდება ჩემი თითების თავს? biggrin.gif

Posted by: Killuminater 8 Oct 2010, 17:18
ესეიგი, დიხანიის მომატება მინდა დაახლოებით, ვიცი რაც უნდა ვქნა! და ადრე და ეხლაც სირბილის შემდეგ ან სირბილისას მეწყება საშIნელი გულის ტკივილი მგონი გულის ტკივილი არის პრინციპშI და რას მირჩევთ ? რა ვქნა ? რისი ბრალია ? თუ რამე მჭირს :ს

Posted by: apocalips 10 Oct 2010, 19:48
:
❒ Numbness, burning, or tingling
❒ Soreness, aching, or tenderness
❒ Pain, throbbing, or swelling
❒ Tightness or stiffness
❒ Weakness or coldness



რა დაავადებებზეა საუბარი ვის შეუძლია მითხრას

Posted by: CrazyForFeeling 10 Oct 2010, 20:53
სიგარეტის მოწევა გლდანდების გაღიზიანებასთან რამე კავშირშია?


წლების წინ ვეწეოდი კაი ხანს, ახლაახნს დავიწყე კიდე და ვგრძნო გლანდები მაქვს ძალიან გაღიზიანებული, ყელელიც მტკივა.

არის რამე კავშირში?

Posted by: Lacrimosa_ 16 Oct 2010, 23:32
უცნაური კითხვა მაქვს.
გლანდების პრობლემა მქონდა ხოლმე ბავშვობაში (ჩირქოვანი წერტილები). შემდეგ ყელის ტკივილები მქონდა ხოლმე შემოდგომობით (საშინლად ვერ ვიტან ყელიან სვიტრებს და არასოდეს მაცვია), ექიმი მაძლევდა ფალიმინტს და ტამტუმ ვერდეს.
განსაკუთრებით მარცხენა მხარეს მტკივდებოდა ხოლმე ყელი.
ახლა ვამჩნევ, რომ ყელზე მარცხენა მხარეს ხშირად გამომდის ხოლმე "პრიჩიკივით" წერტილი, ხშირად არ არის ჩირქოვანი, მაგრამ წითელია და ძალიან მტკივნეული.
შეიძლება ეს იყოს გამოწვეული ყელის პრობლემით?

Posted by: just smile19 17 Oct 2010, 00:19
Lacrimosa_


გლანდები ამოჭრილი გაქვს?

Posted by: Lacrimosa_ 17 Oct 2010, 01:38
just smile19
არა, ჩათვალეს, რომ არ მქონდა იმდენად პრობლემური, რომ ამოეჭრათ

Posted by: just smile19 17 Oct 2010, 12:16
QUOTE
ახლა ვამჩნევ, რომ ყელზე მარცხენა მხარეს ხშირად გამომდის ხოლმე "პრიჩიკივით" წერტილი, ხშირად არ არის ჩირქოვანი, მაგრამ წითელია და ძალიან მტკივნეული



ხოდა ეგ ჩირქოვანი წერტილები გლანდებზე გამოდის და
მკურნალობა უნდა
შეიძლება თუ ქრონიკულია უკვე პროცესი ამოსაჭრელიც გახდეს : )
თუ ხშIრად გიჩირქდება შეიძლება მკურნალობას აზრი აღარ ქონდეს
ლარინგოლოგთან მიდი

Posted by: Lacrimosa_ 17 Oct 2010, 17:02
just smile19
მეზარება მკურნალობა და პირდაპირ რომ ამოვიჭრა, არ შეიძლება? რა პრობლემა შეიძლება გამოიწვიოს გლანდების ამოჭრამ?

Posted by: just smile19 17 Oct 2010, 17:10
Lacrimosa_


თუ ამოსაჭრელი არაა რატო უნდა ამოიჭრა, იმუნიტეტი დაგიქვეითდება
ხშირად თუ გაქვს ანგინა ალბათ უფრო ამოსაჭრელი იქნება




პ.ს. მეც მეზარებოდა მკურნალობა, თან სტრეპტიციტის დაღეჭვას ვერ ვიტანდი და ამოვიჭერი biggrin.gif
ხოდა იმის მერე იმუნიტეტი აღარ მაქვს გადასარევად


Posted by: Overlord667 18 Oct 2010, 00:01
გამარჯობა! კითხვა მაქვს: მაქვს დისბაქტერიოზი,პოსტდიზინტერიოზული კოლიტი ქრონიკული ჩატარებული მაქვს გამოკვლევები ხომ ვერ მიმასწავლიდით როგორ ვიკვებო და ასევე რომელი მცენარეული ნაყენებით ვიმკურნალო,ვსვამ ულტრაბიოტიკს, , ლინეკს.

Posted by: Lacrimosa_ 19 Oct 2010, 14:56
ისევ გამარჯობათ და ისევ უცნაური კითხვა მაქვს.
ხანდახან, სულ ოდნავი აღელვების (როგორც სამწუხარო, ისე სასიამოვნო ფაქტზე) დროსაც კი გული წითლად მეფორეჯება.
ამ საძაგელი თვისების გამო ხშირად უარის თქმა მიწევს გულამოღებულ ზედებზე.
რას ნიშნავს ეს და რა შემიძლია გავაკეთო, რომ აღარ ავჭრელდე
"მერედა შენც არ იღელვო" და "საქმე გამოგელია" პოსტებს არ ვართ.

Posted by: Qwerty-1 23 Oct 2010, 01:49
მდაა
ღამე არ გავა რო ხელის გაბუჟებამ არ გამაღვიძოს
დავაკვირდი და მივხვდი რო ღამე თავი მიდევს ზუსტად არტერიაზე, მაგრამ სხვაგვარად ვერ ვიძინებ, პროსტა რამე საშიში ხო არ არის 2 წელია ყოველდღE მეღვიძება
ეს ისე გამახსენდა რა მარცხენა ხელი მიბუჟდება ეხლა რატომღაც და მაგ პონტში user.gif

Posted by: rock in rose 23 Oct 2010, 12:55
Lacrimosa_
QUOTE
ისევ გამარჯობათ და ისევ უცნაური კითხვა მაქვს.
ხანდახან, სულ ოდნავი აღელვების (როგორც სამწუხარო, ისე სასიამოვნო ფაქტზე) დროსაც კი გული წითლად მეფორეჯება.
ამ საძაგელი თვისების გამო ხშირად უარის თქმა მიწევს გულამოღებულ ზედებზე.
რას ნიშნავს ეს და რა შემიძლია გავაკეთო, რომ აღარ ავჭრელდე
"მერედა შენც არ იღელვო" და "საქმე გამოგელია" პოსტებს არ ვართ.


აღელვების დროს მხოლოდ გულის მიდამოში გიწითლდებათ? სხვა არაფერი?


Qwerty-1

იქნებ სხვანაირად ძილს მეჩვიო user.gif

QUOTE
პროსტა რამე საშიში ხო არ არის 2 წელია ყოველდღE მეღვიძება


ყოველღამე გაღვიძება ბუნებრივია არ არის კარგი

Posted by: Qwerty-1 23 Oct 2010, 13:15
QUOTE
იქნებ სხვანაირად ძილს მეჩვიო

ჯერ ძილს უნდა მივეჩვიო user.gif
დღეში 3 საათი mo.gif

Posted by: rock in rose 23 Oct 2010, 13:23
Qwerty-1
QUOTE
ჯერ ძილს უნდა მივეჩვიო
დღეში 3 საათი


24 საათის განმავლობაში მხოლოდ 3 საათი გძინავს? spy.gif

Posted by: radiohead_lover 23 Oct 2010, 16:34
იქნებ ვინმემ მიპასუხოს... 1კვირის წინ საშინლად მტკიოდა მუცელი და გულისრევის შეგრძნება მქონდა მარა სულ ტყუიალდ.. 2დღე სიცხე მქონდა 37... მერე... ძალიან სუსტად ვარ კარგად მძინავს , ვჭამ ჩვეულებრივად მარა 1სათი ლექციაზეც ვერ ვჩერდები ცუდად ვარ უენერგიოდ, თავი მტკივდება და მოკლედ სუსტად ვარ, 2 დღეა თავი მტკივა არაფერი არ მშველის... კბილების კრაჭუნი ღამე დ რავიცი ყველაფერი მე მჭირს მგონი biggrin.gif

სისხლის ანალიზი რო გავიკეთო გავიგებ რა მჭირს?

პ.ს. 19 წლის ვარ თუ რამე მნიშვნელობა აქვს yes.gif

Posted by: Qwerty-1 23 Oct 2010, 17:19
QUOTE
24 საათის განმავლობაში მხოლოდ 3 საათი გძინავს?

ბოლო 3 წელია ჰო
ნუ
ზოგჯერ
ღამე 5 ზე ვიძინებ 3 საათში სკოლისთვის ვდგები, მოსულს ზოგჯერ 2 დან 5 მდე მძინავს, მაგრამ ეხლა აბიტურიენტი ვარ და მაგას ვერ ვასწრებ
საშუალოდ 4 საათი ღამეში mo.gif

Posted by: rock in rose 23 Oct 2010, 18:06
radiohead_lover

კკბილების კრაჭუნი ჭიებმა იცის, სისხლის საერთო ანალიზი გაიკეთე და თუ ეოზინოფილები გაქვს მომატებული მაშინ ჭიაზე უნდა იმკურნალო ექიმთან
სისხლის საერთო ანალიზით ჰემოგლობინსაც გაიგებ, მემგონი დაბალი უნდა გქონდეს

Qwerty-1
აბიტურიენტი ხარ ახლა და ამ მდგომარეობაში შენთვის 4 საათი ძილი ძალიან ცოტაა, ჯერ შენით სცადე ამ პრობლემის მოგვარება, ადრე დაწექი და ძილის წინ თბილი რძე დალიე თაფლით.. ან თუ რძე არ გიყვარს და თაფლზეც ალერგია გაქვს, 25 წვეთი ვალერიანის წვეთები გააზავე წყალში და ეგ დალიე. მშვიდ მუსიკას მოუსმინე და კარგად ჩაბნელებულ, განიავებულ ოთახში დაიძინე, თუ ამანაც ვერ გაჭრა, მაშინ აუცილებლად მიდი ექიმთან.. ის დაგინიშნავს შესაბამის წამალს

Posted by: radiohead_lover 23 Oct 2010, 18:45
rock in rose

QUOTE
კკბილების კრაჭუნი ჭიებმა იცის, სისხლის საერთო ანალიზი გაიკეთე და თუ ეოზინოფილები გაქვს მომატებული მაშინ ჭიაზე უნდა იმკურნალო ექიმთან
სისხლის საერთო ანალიზით ჰემოგლობინსაც გაიგებ, მემგონი დაბალი უნდა გქონდეს


დიდი მადლობა, ორშაბათს გავიკეთებ ანალიზს smile.gif 2kiss.gif

Posted by: Lacrimosa_ 23 Oct 2010, 21:52
rock in rose
გულმკერდის მიდამოებში ყელის გარდა სულ წითელი ტალები მაქვს ხოლმე. სახეზე არაფერი მემჩნევა

Posted by: Qwerty-1 24 Oct 2010, 01:06
rock in rose
QUOTE
აბიტურიენტი ხარ ახლა და ამ მდგომარეობაში შენთვის 4 საათი ძილი ძალიან ცოტაა, ჯერ შენით სცადე ამ პრობლემის მოგვარება, ადრე დაწექი და ძილის წინ თბილი რძე დალიე თაფლით.. ან თუ რძე არ გიყვარს და თაფლზეც ალერგია გაქვს, 25 წვეთი ვალერიანის წვეთები გააზავე წყალში და ეგ დალიე. მშვიდ მუსიკას მოუსმინე და კარგად ჩაბნელებულ, განიავებულ ოთახში დაიძინე, თუ ამანაც ვერ გაჭრა, მაშინ აუცილებლად მიდი ექიმთან.. ის დაგინიშნავს შესაბამის წამალს


2kiss.gif გაიხარე

მაგრამ რძეს ვერ ვიტან
ვერც დამამშვიდებელს
3 წელია ყველაფერი გამოცდილი მაქვს, მაგრამ როცა თვითონ მისწორდება ღამე სიფხიზლე, მაშინ რთულადაა საქმე

Posted by: loveless 25 Oct 2010, 17:07
ამ დღეებში წავალ ექიმთან... (რაიონში ვარ და ვერ ვახერხებ)
მაგრამ იქნებ მანამდე ვინმე დამეხმაროს, ცოტა შეშინებული ვარ და მეტად ხომ არ უნდა ძშევშინდე კიდევ.
ახალი გათხოვილი ვარ. 5 თვეა
გავცივდი , შარდვისას საშინელი ტკივილი მქონდა, ერთელ მალე გამიარა, მეორედაც დამემართა ერთი კვირის განმავლობაში.
როგორც კი გავთბი და თავს გავუფრთხილდი ტკივილმა გამიარა, მაგრამ უსიამოვნო შეგრძნება დარჩა... არის ხშირი შარდვა.
მუცელი გაბერილი მაქვს.. მთლიანად. საკევრცხეებთან შარდის ბუშტთანაც, და ზემოთაც. და დაჭიმული მაქვს. მუცელი ორსულივით მაქვს, უკვე ერთი თვეა რაც გაზრდა შევატყვე. არადა მენზი მქონდა . და ტესტი არ გამიკეთებია არც ერთხელ.
sad.gif
გული მისკდება სანამ ქალაქში ჩავალ.
და კიდევ ტკივილი მაქვს აქტის დროს..მუცელშიც..სადრაც შარდის ბუშტთან..და
ზოგჯერ უკანა ტანშიც sad.gif

Posted by: 1984 25 Oct 2010, 20:36
სად ვიკითხო არ ვიცი

ჭიპლარი ამოვიბანე დუშის დროს და დამისივდა, გამიწითლდა, ერთი კვირაა თითქმის, თან ტკივილებიც მაქვს იმავე არეში, სიარულის დროს და რომ ვწევარ წამოდგომის დროს მტკივა. მიშველეთ რამე, ზელიონკა წავისვი რამდენიმეჯერ

Posted by: ninca 26 Oct 2010, 23:09
საღამო მშვიდობისა

მაინტერესებს სისხლის ანალიზში წერია პლაზმური უჯრედი - 2. როგორც მივხვდი არ უნდა იყოს და არის. რას ნიშნავს ხომ ვერ მეტყვით?

Posted by: konkretula 27 Oct 2010, 14:52
ესეიგი რა მაინტერესებს. ორი დღის წინ სურდო მქონდა და ეგრევე ექიმთან გამაქანა დედაჩემთან არ გაგირთულდესო, არც სიცხე მქონია არც არაფერი. ნუ ექიმმა გამსინჯა და დამისკვნა რო ყელზე მაქ პრობლემებიი. მართლა მაქვს ერთი პატარა თეტრი წერტილი და ექიმმა დაჩირქებულიაო და ამპიცილინი გამომიწერა და კიდე რაღაც სავლები და საწუწნი ტაბლეტი. ქუჩაში გასვლა ამიკრძალა, მხოლოდ თბილი სითხეები სვიო და ასე შემდეგ. მეორე დღეა ვსვავ მაგრამ ისევ ისეთ მდგომარეობაში მაქვს ყელი. სურდო აღარ მაქვს, სიცხე არც მქონია და ჩვეულებრივად ვგრძნობ თავს.


მაინტერესებს მართლა ასეთი სერიოზულია თუ ეს ექიმი ატრაგიკულებს ყველაფერს??? biggrin.gif

Posted by: radiohead_lover 27 Oct 2010, 19:22
მოკლედ საერთო ანალიზი გავიკეთე და გადასარევი პასუხებიაო, ყველაფერი წესრიგშიაო smile.gif

აბა რას დავაბრალო ? biggrin.gif ალბათ წნევა მაქვს დაბალი



ორსულობა ანალიზებში გამოჩნდებოდა ხო?


მედეაში ვიყავი ძლივს მივაგენი საშინელი ადგილია, სხვა სიტუაციას ველოდი... 12მდე უნდა მოხვიდეო, წავედი მერე სისხლის გადასხმის ინსტიტუტში და იქ გავიკეთე

Posted by: rock in rose 27 Oct 2010, 20:14
konkretula
QUOTE
მაინტერესებს მართლა ასეთი სერიოზულია თუ ეს ექიმი ატრაგიკულებს ყველაფერს???


ექიმი ძალიან ამუქებს biggrin.gif ქუჩაში სიარულის აკრძალვა მეტისმეტია

თან ყელის მხრივაც არაა საგანგაშო მდგომარეობა


Posted by: konkretula 27 Oct 2010, 21:44
rock in rose
რაღაც ანტიბიოტიკების დალევა და იმის გამოვლება მიშველის??? biggrin.gif
თან თავიდა მაჯახა ნემსები დაგჭირდებაო და ძლივს გადავაფიქრებინე :| არდა არაფერი მჭირს, სიცხეც კი არ მაქვს biggrin.gif

მაგრამ თან ისაა, რომ მაინც არანაირი შედეგი არ აქვს ამ წამლებს, ისევ სიეა როგორც იყო :?

Posted by: rock in rose 28 Oct 2010, 19:23
konkretula

ახლა როგორ ხარ? smile.gif

QUOTE
თან თავიდა მაჯახა ნემსები დაგჭირდებაო და ძლივს გადავაფიქრებინე


მეტისმეტია biggrin.gif

QUOTE
მაგრამ თან ისაა, რომ მაინც არანაირი შედეგი არ აქვს ამ წამლებს, ისევ სიეა როგორც იყო :?


2 დღე მკურნალობა ცოტაა.. რამდენიმე დღეში კარგად იქნები

ნუ შეშინდები, სერიოზული ნამდვილად არაფერია smile.gif

Posted by: lovelorn 1 Nov 2010, 17:34
ექიმმა დამიწერა მუცლის ღრუს ექოსკოპია. და ექოსკოპისტმა მომეჩვენა რომ მხოლოდ ნაგველა და ელენთა მინახა.
სხვა არაფერი შედის მუცლის ღრუში?????
ბრმა ნაწლავი, ნაწლავები?????

Posted by: ia_Baggins 1 Nov 2010, 18:26
ყბის სახსარი მტკივა ძაააალიან ყურზეც გადადის რა ვქნა სტომატოლოგთან მივიდე თუ ყელ-ყურ-ცხვირთან?

კბილის ბრალი შეიძლება იყოს?

Posted by: Summer_Rain 2 Nov 2010, 12:52
სალამიsmile.gif

მაინტერესებს ალოეს, ბ1 და ბ6-ს ერთ ინექციად გაკეთება ხო შეიძლება.?
ვიცი რომ რომელიღაც ვიტამინებს ერთად არ იკეთებენ ამიტო გამიჩნდა ეს კითხვა.

Posted by: loveless 3 Nov 2010, 12:02
როგორც ჩანს აქ არავინ პასუხობს..ტყუილად ვკითხულობთ რამეს...

Posted by: BloodElf 3 Nov 2010, 20:24
მე მაპატიეთ თუ შეუფერებელ თემაში ვპოსტავ მაგრამ არ ვიცი რომელ ექიმს უნდა ვკითხო smile.gif) მაინტერესებს სისტემური (წითელი თუ არ ვცდები) მგლურა მემკვიდრეობით გადადის თუ არა? მაგალითად შეიძლება თუ არა ბებიისგან მემკვიდრეობით გადავიდეს შვილიშვილზე?

Posted by: lila2009 4 Nov 2010, 10:17
გთხოვთ დამეხმაროთ.. სხვა ანალიზებიც ჩასაბარებელი მაქვს, და ექიმთან შემდეგ მივალ, მანამდე იქნებ მითხრათ


ეს პასუხები საშიში ხომ არაა? ან რასთაანაა დაკავშირებული ეს ცვლილებები...


ჰემოგლობინი 8.7 (52) , ნორმა <12-16>
ერითროციტები 3.4 , ნორმა <4.1-5.1>
ფერადობის მაჩვენებელი 0.76 , < 0.85-1.05>
ჰემატოკრიტი 28 , <36-45%>
ჰემოგლობინის საშ.შემცველობა ერითროციტებში 31.4 , <31.7-36>



Posted by: კახა251 4 Nov 2010, 11:45
lila2009
და ექიმს არ ანახე? ექიმი არ ვარ მაგრამ აშკარად დაწეული გაქ ყველა მაჩვენებელი, თმის ცცვენა, ფრჩხილების მტვრევა ხომ არ გაქვს? აუცილებლად მიდი კარგ ექიმთან, უმჯობესია სისხლის ექიმს აჩვენო... რავი მე არც ექიმი ვარ და არც მინდა რომ შეგაშინო მაგრამ ანემიას გავს და ეჩვენე მაინც...

Posted by: BloodElf 4 Nov 2010, 13:26
ia_Baggins
რამდენი წლის ხარ? მეც მტკიოდა ყბა ძალიან პირს ვერ ვაღებდი ისე.. "სიბრძნის" კბილი ამომდიოდა და ის იწვევდა თურმე..

Posted by: ia_Baggins 4 Nov 2010, 15:00
BloodElf
21... ხო ყველამ მითხრა სიბრძნის კბილი ამოგდისო sad.gif

ეგრე ვიყავი 3 დღე ვერ ვაღებდი პირს ნორმალურად...

ბოლო 2 დღეა შემიმცირდა ტკივილი, მარა კბილი არ ჩანს ჯეეერ mo.gif

Posted by: BloodElf 4 Nov 2010, 19:39
ia_Baggins
ლამის ერთი წელი გავიდა და ნორმალურად არ ამომსვლია მეც.. მე გირჩევ სურატი გადაიღე კბილზე.. გამოჩნდება თუ ამოდის და სწორად ამოდის თუ არა.. სტომატოლოგიურ კლინიკაში მიდი და გადაგიღებენ..

Posted by: ia_Baggins 4 Nov 2010, 19:59
BloodElf
ხო მომიწევს მისვლა, გაიხარე smile.gif

Posted by: Solveig 6 Nov 2010, 00:55
lila2009
QUOTE
ს პასუხები საშიში ხომ არაა? ან რასთაანაა დაკავშირებული ეს ცვლილებები...


ჰემოგლობინი 8.7 (52) , ნორმა <12-16>
ერითროციტები 3.4 , ნორმა <4.1-5.1>
ფერადობის მაჩვენებელი 0.76 , < 0.85-1.05>
ჰემატოკრიტი 28 , <36-45%>
ჰემოგლობინის საშ.შემცველობა ერითროციტებში 31.4 , <31.7-36>

ანემია გაქვს.
თუმცა, სხვა გამოკვლევებიც საჭიროა დიაგნოზისათვის.
შრატის რკინა, ფერიტინი, ტრანსფერინი, ერითროციტების მორფოლოგია...

ექიმთან მიდი აუცილებლად. მკურნალობა გესაჭიროება.
რაიმე ტრავმა ხომ არ გადაგიტანია, რაც სისხლი დაკარგვასთან იყო დაკავშირებული? ან სხვა მიზეზების გამო (კუჭ-ნაწლავიდან ფარული სისხლდენა, ჭარბი მენსტრუაცია და ა.შ.) ხომ არ კარგავ სისხლს?
როგორ იკვებები?

Posted by: nalbo 6 Nov 2010, 08:59
imovane რისი წამალია ხომ ვერ მეტყვით, გვერდითი მოვლენებიც თუ იცით დაწერეთ რა.



p.S. თუ არასწორ ადგილზე ვსვამ კითხვას მაპატიეთ

Posted by: BloodElf 6 Nov 2010, 12:23
QUOTE (BloodElf @ 3 Nov 2010, 20:24 )
მე მაპატიეთ თუ შეუფერებელ თემაში ვპოსტავ მაგრამ არ ვიცი რომელ ექიმს უნდა ვკითხო smile.gif) მაინტერესებს სისტემური (წითელი თუ არ ვცდები) მგლურა მემკვიდრეობით გადადის თუ არა? მაგალითად შეიძლება თუ არა ბებიისგან მემკვიდრეობით გადავიდეს შვილიშვილზე?

მიპასუხეთ ვინმემ რაა.. 2kiss.gif

Posted by: ravi_shen 6 Nov 2010, 12:33
ა ვიცი ეს კითხვა სად უნდა დამესვა მაგრამ აშკარად არც ენდოკრინოლოგის კითხვა არ არის და ალერგოლოგის კითხვარი ვერ ვნახე ამიტომ აქვე ვიკითხავ..

წყლის დალევის შEმდეგ მეწყება გულის რევა და რისი ბრალი შEიძLება იყოს?
სადღAც 3 კვირაა წყალი არ დამილევია :შ

პ.ს. ცოფი გამორიცხეთ givi.gif

Posted by: კახა251 6 Nov 2010, 12:50
ravi_shen
გაუწყლოვანებას გავს... ექიმს ეჩვენე თორე ასე ვერავინ დაგეხმარება...
QUOTE
სადღAც 3 კვირაა წყალი არ დამილევია

:o_O smiliki: ღადაობ?

Posted by: ravi_shen 6 Nov 2010, 13:04
kaxa251
გაუწყლოვანება რას ქვია :შ

QUOTE
:o_O smiliki: ღადაობ?

არა
ერთი წვეტი წყალი არ დამილევია დედაჩEმი დამემოწმება

ნაბეღლავი :შ
ისე ალერგია მავქს და ზოგადათ ძაან ცუდი მდგომარეობა მავქს ელთიროქსინის გამო :ს
ჰოდა ახლა წყალზე გადავედი
ადრე მარწყვს და მსგავს წითელ რაღAცეებს ვერ ვჭამდი :შ

Posted by: ArxeoptericS 6 Nov 2010, 15:53
ნაბახუსევზე, ნერვიულობის დროს და ზოგ დღეებში როდესაც კარგად თავს ვერ ვგრძნობ ამ ადგილში მიბუჟდება ძვალი, გეგონება შიგნიდან რაღაც ღრღნისო რისი ბრალი შეიძლება იყოს?

http://pix.ge/

და კიდევ დავდივარ წელში მოხრილი მინდა რომ წელში გავსწორდე სად მირჩევთ მისვლას? ჩემი თავი მეზიზღება ვიდეობს რომ ვუყურებ ხოლმე sad.gif

Posted by: Solveig 6 Nov 2010, 18:20
BloodElf
QUOTE
სისტემური (წითელი თუ არ ვცდები) მგლურა მემკვიდრეობით გადადის თუ არა? მაგალითად შეიძლება თუ არა ბებიისგან მემკვიდრეობით გადავიდეს შვილიშვილზე?

პირდაპირი დამემკვიდრება არ ახასიათებს, ანუ აუცილებლად არ განუვითარდება (როგორც, ვთქვათ, ჰემოფილიაა ან კიდევ სისხლის შედედების სხვა დეფექტები, ჰემოგლობინოპათიები და ა. შ)..

დაავადება ასოცირებულია ზოგიერთი სახის ლეიკოციტური ანტიგენის ნაკრებთან (ანუ, ვარაუდობენ, რომ ამა თუ იმ ფენოტიპის ადამიანებს უფრო ადვილად ემართებათ), რადგან ეს ლეიკოციტური ანტიგენები მემკვიდრეობითია, შესაბამისად, დაავადების განვითარებაში მემკვიდრეობასაც აქვს მნიშვნელობა.

ვაჟებში ალბათობა უფრო ნაკლებია. სავარაუდოდ, ანდროგენების გამო.

Posted by: malimalia 7 Nov 2010, 21:57
ვარ 2 კვირის ნამშობიარევი ფიზიოლოგიურად
მაქვს დუგლასში სითხე
მწვავე ენტეროკოლიტი
გასტრიტი
ინფუზის ძვლის დაშორება დიდი და ტკივილი ბოქვენის ძვლის


ვსვავ ალმაგელს,ენზიმინს,ულტრაბიოტიკს,ბენევრონ BF,კალცი D3,ესკუროტინს

რამდენად სწორადაა დანიშნული მკურნალობა?
სხვა თერაპევტების აზრიც მაინტერესებს ძაან

Posted by: Killuminater 9 Nov 2010, 20:56
ხალხნო სურდო მაქვს და მიშველეთ რა რითი ვუშველო ?

ჩასაწვეთებლები და ეგეთი კვერცხობებ იარ მშველის : D

მომდის და მომდის

აღარ შემიძლია

რა წამალი მივიღო დღეში რამდენჯერ ?

ან რა ვჭამო რა დავლიო ?

დღეს დილით ლიმნიანი ჩაი მივირთვი და მერე ლიმონი და შაქარი ერთად

მიშველეთ


აღარ შემიძლია


Posted by: donvaso 9 Nov 2010, 21:05
Killuminater
რაც შეიძლება ბევრი სითხე მიიღე, თუ სიცხე არ გაქვს არ ღირს არაფრის დალევა

Posted by: Killuminater 9 Nov 2010, 21:19
donvaso

ნატურალური წვენებიც ?

პ.ს გაიხარე

სიცხე არ მაქვს მაგრამ სახეზე ვხურვარ : D





Posted by: კახა251 9 Nov 2010, 23:57
Killuminater
შენ დღეში ვარ.... მე ყელიც მტკივა... და სიცხეც მაქ 37.2... ლიმნიანი ჩაი დავლიე 4 ჯერ დღეს biggrin.gif:D არადა მაზეგ დაბადების დღე მაქ და არ მინდა ასე შევხვდე biggrin.gif

Posted by: Killuminater 10 Nov 2010, 18:16
kaxa251

საშინელებაა ეხლაც წვეთავს : D


მე დღეში ერთხელ ვსვავ ჩაის ისე ცოტხანში გავიკეტებ : ) არ დავიზარებ : |

Posted by: m-009 11 Nov 2010, 02:58
ოთხი დღეა მარცხენა ხელი მიბუჟდება, ძირითადად თითები ,რისი ბრალი უნდა იყოს სავარაუდოდ?

Posted by: კახა251 11 Nov 2010, 14:19
Killuminater
რავა ხარ?! მე გამიარა ეხლა პროსტა ცოტა სურდო შემომრჩა თორე ისე კარგად ვარ არამიშავს მალე ვარჯიშაც გავაგრძელებ... biggrin.gif

Posted by: Killuminater 11 Nov 2010, 16:35
kaxa251

რავი აბა არამიშავს სურდო ისევ მაქვს მაგრამ დღეს რაღაც შემიმსუბუქთდა

ცოტა მახველებს მაგრამ ვსვამ წამლებს და ვცდილობ რომ მალე გამოვკეთდე

ისე საშIნლად გამომიშრა ტუჩები : |


პ.ს შენ გამოჯანმრთელებულხარ : D და კარგია : )



Posted by: კახა251 12 Nov 2010, 01:23
Killuminater
აბა შენ იცი მალე გამოკეთდი

Posted by: tanamshromeli 12 Nov 2010, 22:58
QUOTE (ArxeoptericS @ 6 Nov 2010, 15:53 )
ნაბახუსევზე, ნერვიულობის დროს და ზოგ დღეებში როდესაც კარგად თავს ვერ ვგრძნობ ამ ადგილში მიბუჟდება ძვალი, გეგონება შიგნიდან რაღაც ღრღნისო რისი ბრალი შეიძლება იყოს?

http://pix.ge/

და კიდევ დავდივარ წელში მოხრილი მინდა რომ წელში გავსწორდე სად მირჩევთ მისვლას? ჩემი თავი მეზიზღება ვიდეობს რომ ვუყურებ ხოლმე sad.gif

მეც ზუსტად მასე ვარ ოღონდ სულ მტკივა ამ ბოლო დროს,,

Posted by: LOLITA_24 12 Nov 2010, 23:51
მერც მაქვს ერთი კითხვა smile.gif
ფერიტინის შემცველობა სისხლში ქალისთვის 60 ნორმალურია თუ დაბალია?

Posted by: _SisterofNight_ 18 Nov 2010, 17:28
რამდენიმე თვის წინ ანუ ზაფხულში მქონდა მარჯვენა ჯირკვალი გასიებული კისერზე, სისხლის ანალიზი წესრიგში მქონდა და ანტიბიოტიკის შედეგად გამიქრა, ეხლა ხელი მოვიკიდე და ისევ იმავე ადგილას მაქვს ოდნავ გადიდებულივით, ისე ძაან არა როგორც მქონდა, რა ვქნა? უფრო სწორად რა უნდა მივიღო ან რა გავაკეთო? ისე ყელი მტკიოდა ცოტახნის წინ და შეიძლება მაგის ბრალი იყოს?

Posted by: leopardibuba 19 Nov 2010, 14:11
26 წლის კაცი ვარ, მოშარდვის შემდეგ ურეთრას (პროსტატასთან ახლოს) რომ არ მივაჭირო ხელი შარდი ბოლომდე არ გამოდის და მერე თავისით იღვრება. შეგიძლიათ მითხრათ რა დაავადებაა და როგორ ვიმკურნალო.

Posted by: stereo_tipsha 19 Nov 2010, 17:18
მესამე დღეა მაქვს დაბალი სიცხე, 37, ისიც საღამოობით,.თან მახველებს,,,, ყელის ტკივილიც მქონდა საშინელი,,,,ახლა გამიარა....ბრონქიტი მაქვთ ვითომ?? ყელში რაღაც კაწვრის შეგრძნება მაქვს,,,,ეს სიცხე საშიშია?? რაღაც სავლებებს ვივლებ და მედიფონს ვწუწნი

Posted by: დიაბლო 20 Nov 2010, 15:16
ArxeoptericS
ალბათ წინა დღეს სკამის საზურგეს გირტყამდნენ ზურგში ან დაეცი. ან მეშვიდე სართულიდან გადმოვარდი ან ვარლოკის სიკნესი გადევს... ათასი ვარიანტია. არამგონია რამე დაავადება იყოს. ნაბახუსევიზმი
* * *
leopardibuba
უსაფრთხოების კოდს ხომარგთხოვს? თუარადა არაა საშიში ჯერ smile.gif

Posted by: კახა251 20 Nov 2010, 15:38
stereo_tipsha
QUOTE
მედიფონს ვწუწნი

მე არ მშველის...
QUOTE
ეს სიცხე საშიშია??

გააჩნია რა შემთხვევაში...
QUOTE
მესამე დღეა მაქვს დაბალი სიცხე, 37, ისიც საღამოობით,.თან მახველებს,,,,

გაციებული ხარ სავარაუდოდ...
QUOTE
ყელის ტკივილიც მქონდა საშინელი

ეს რომ არ დაგეწერა გეტყოდი ფთიზიატრს გაესინჯე მეთქი მაგრამ არ არის საფუძველი ჯერ ჯერობით : )

Posted by: stereo_tipsha 20 Nov 2010, 17:04
kaxa251
მადლობთ,.დავმშვიდდდიsmile.gif))

Posted by: Marmel 22 Nov 2010, 10:17
არ ვიცი სად დავსვა ეს კითხვა smile.gif 23 წლის ასაკში არის იმის შანსი, რომ ხერხემალი გაასწორონ? რამდენიმე მალაა გადახრილი. მასაჟებს აქვს აზრი თუ უკვე ძალიანაა დაგვიანებული?

Posted by: humaneika 23 Nov 2010, 17:31
ვერ მივხვდი რომელ განყოფილებაში უნდა დამეწერა და ბოლოს აქ დავწერ : ერთი კვირაა თავბრუს ხვევები მაქვს, სისხლის ანალიზი ავიგე და ყველაფერი წესრიგშია და სხვარისი ბრალი შეიძლება იყოს თავბრუსხვევა?

Posted by: Doxturius 27 Nov 2010, 17:51
humaneika
წნევას აკონტროლებ, რამდენი გაქვს?

Posted by: maromar 28 Nov 2010, 17:12
კითხვა უროლოგთან ვერ ვიპოვე და აქ დავწერ:

მამაჩემს ძალიან აწუხებს ხშირი შარდვა - განსაკუთრებით ღამით, ღამეში 6-7-ჯერ მაინც გადის ხოლმე მოსაშარდად. წვა შარდვის დროს არ აქვს, არც სხვა რამე არ აწუხებს საერთოდ, უბრალოდ ძალიან ხშირად გადის მოსაშარდად და ეს უკვე ალბათ სადღაც წელიწადზე მეტია გრძელდება.. ასაკით 61 წლისაა.. თქვენი აზრით რა შეიძლება იყოს მიზეზი? დიაბეტზე ხშირად ვფიქრობ, იმიტომ, რომ ბებიაჩემსაც ანუ დედამისსაც დიაბეტი ქონდა, თან სითხეებსაც ბევრს სვამს, მაგრამ არც ადრე სვამდა ნაკლებად სითხეებს.. პროსტატიტიც არ მგონია რავიცი, ამბობს წვა ან რამე სხვა დისკომფორტი არ მაქვსო და.. სხვა მიზეზები კიდევ რა შეიძლება იყოს? და რომელ უროლოგთან მირჩევთ რომ მივიდეს? წინასწარ მადლობა..

Posted by: AquariuMANIAK 28 Nov 2010, 20:19
არ ვიცი აქ უნდა დამეწერა თუ არა 1 სიტყვით

საჭმლის ჭამაზე მტკივდება კუჭი, ზოგჯერ თვალებთან მაქვს ჩაშავებული ზოგჯერ პირიდან მაწუხებს არასასიამოვნო სუნი როგორც არ უნდა კბილები ვიხეხო biggrin.gif

დამეხმარეთ ვის მივმართო? ან თუ იცით რამე ისეთი მეთოდი ან თუ საჭიროა დიეტა და ეგეთი რაღაცეები იქნებ გამიზიაროთ?


გმადლობთ წინასწარ

Posted by: player124 29 Nov 2010, 21:45
ყელში მაქ რაღაც უსიამოვნო შეგრძნება, თითქოს მებერებასავით, მთლად ტკივილსაც ვერ დავარქმევ მარა აშკარად ვგრძნობ რო რაღაც მაქ რა... რისი ბრალია?

Posted by: player124 2 Dec 2010, 22:58
მიპასუხეთ ვინმემ რააააააააააააა

Posted by: lala99 4 Dec 2010, 14:31
2 დღის წინ სისხლის ანალიზი ჩავაბარე ვენიდან... მე3 დღეა და მხოლოდ დღეს შევამჩნიე ნანემსარზე სიშავე..ირგვლივ მკრთალად დამილურჯდა და იქვე გვერდით ნაჩხვლეტზეც პატარა ჩაქცევაც მაქვს ძალიან შავად... sad.gif რა არის ეეეეეეს? ძალიან შემეშინდა ამდილით რომ ვნახეეეეეეეეეე

Posted by: Kaifistka 4 Dec 2010, 17:31
QUOTE
კანის დაავადებები

ორი შეკითხვა მაქვს
1. მენსტრუაციის წინა დღეებში მაყრის ან სახეზე, ანაც ზურგისა და მხრების მიდამოებში. ცოტას მაგრამ მაინ. რას შეიძლება დავაბრალო?! ან რა ვქნა რომ აღარ დამაყაროს ხოლმე. მერე გადამდის თავისით.
2. რთულად მიხორცდება კანი. ადრე ასე არ ვიყავი. ვსამდი თუთიასაც და არ მაწყუხებს შაქარი. ადრე ძალიან მალე მიხორცდებოდა და ახლა არა, ხშირად ნაწიბურებიც კი მრჩება ერთ უბრალო ნაკაწრზე. რა შეიძლება იყოს ამის მიზეზი?! ასევე როგორ ვუმკურნალო.


----------------------------------------
ხო და არვიცი ეს ამ თემაში შედის თუ არა, მაგრამ კიდევ ერთი შეკითხვა.
დაახლოებით ორი წლის წინ აღმომაჩნდა ბრუცელიოზი და ლაიმას ბორელიოზი. ბევრი ვიმკურნალე. გერმანიაშიც კი წავედი მაგის სამკუნალოდ. იქ ექიმმა მითხრა, რომ ბრუცელიოზი არ ინკურნება ხოლო ლაიამსგან განკურნება შესაძლებელია, მაგრამ დაახლოებით ათი წელი მაინც რჩება სისხლში.
ახლა ისევ გერმანაში სხვა ქალაქშია ავიღე ანალიზები და აჩვენა რომ არცერთი არ მჭირს. ისე არის რომ კვალიც კი არ არის რომ ოდესმე ნამყოფიც კი თუ იყოო.
სიფრთხილის მიზნით მიწერილია ანალიზებზე, რომ საჭიროა ხელახლა ჩატარება კიდევ ერთხელ გარკვეული პერიოდის დროის გასვლის შემდეგ.

ჩემი შეკითხვა-შესაძლებელია ბრუცელიოზისგან და ლამიასგან განვკურნებულიყავი?!
ბრუცელიოზი განკურენბადია?!
და ბორელიაც შესაძლოა ასე მალე განკურნებულიო?!



lala99
უი ეგ არაფერია. რომ აიღებენ ანალიზს ხელი უნდა დაგეჭირა კარგად მერე და მოგეხარა. ეტყობა არ დააჭირე კარგიად ხელი. ეგრე იცის ხოლმე. გაგივლის ძაან მალე 2kiss.gif

lovelorn
QUOTE

ექიმმა დამიწერა მუცლის ღრუს ექოსკოპია. და ექოსკოპისტმა მომეჩვენა რომ მხოლოდ ნაგველა და ელენთა მინახა.
სხვა არაფერი შედის მუცლის ღრუში?????
ბრმა ნაწლავი, ნაწლავები?????

ეხოსკოპაიზე ნაწლავები არ ჩანს. ნაწლავები რომ გამოჩნდეს რაღაცა სითხე სვამენ დ ამოკლედ ის სხვა პროცედურაა.

Posted by: Killuminater 8 Dec 2010, 21:58
ესეიგი რა ხდება ჩემ თავს

მარცხენა ყურის ბიბილოში შIგნით არის რაღაც პატარა ბურთულასავით და რას მირჩევთ ?

Posted by: sofunia 10 Dec 2010, 05:54
იქნებ მიჩიოთ რაიმე ხერხი როგორ გავაკეთო ბიცილინ5, უმალვე ქვავდება არსებობს რაიმე სპეციფიკა

Posted by: May21 11 Dec 2010, 11:10
კითხვა მაქვს ასეთი...8 წლის ბავშვს უტყდება სლოკინი და ვარანაირი ხერხით ვერ ვუჩერებთ.......საბან თAვისით არ გადასდის , ამასობაში გული ეღლება ძAლიან....მირჩიეთ ვის მივმართო....


პ.ს. ბოლოს ფეხით ვკიდებდით.... არ უწყდებოდა , მაგრამ ინტენსივობა შემცირდა. ანუ უფრო იშვიათAდ ემართება.....

წინასწარ გმადლობთ პასუხისათვის......

Posted by: DCM 11 Dec 2010, 18:40
რაშია საქმე smile.gif
მოკლედ ძალიან ბევრს ვჭამ yawn.gif
მაგრამ მაინც არ ვსუქდები
18წლის ვარ, ზუსტად არ ვიცი დიდი ხანია არ გავზომილვარ და სადღაც 180სმ უნდა ვიყო
წონაში კი სულ რაღაც 57კგ wow.gif

რისი ბრალი შეიძლება იყოს?

Posted by: gio_20 14 Dec 2010, 17:13
გამარჯობათ!მაინტერესებს უძილობას თუ შეუძლია გამოიწვიოს მოთენთილობა,უხალისობა და მთვრალივით სიარული?2 თვეა რაც ღამ-ღამობით 3-4 საათს მძინავს და დილით ვარ სულ მოთენთილი მთვრალივით ვარ

Posted by: aranormaluri 23 Dec 2010, 02:01
ერთი კითხვა მაქვს: ტანი მტკივა ფილტვების ადგილას, რომ ვახველებ განსაკუთრებით მაშინ. ასევე მტკივა ყელი ოღონდ შიგნიდან არა, თითქოს ხორხი, თვითონ ძვალი გარედან. სიცხე გავიზომე და 35, 5 მიჩვენა ორივე თერმომეტრმა.
რა ჯანდაბა შეიძლება იყოს?


Posted by: Username16 23 Dec 2010, 11:37
სეკსის მერე სახეზე მაკრის, არა და 24-ს ვარ, ცოლის მოყვანას ვაპირებ და ტეხავს მაგრად.
ბევრი რამ ვზადე, არ მსჰველის არაფერი. იქნებ მირჩიოთ რამე..
ისე, მითხრეს სისხლი სააბარეო, მაგის ბრალი სეიძლება იყოსო,, მიშველის ნეტა?
ან და ვის იცნობთ მაგარ ექიმს, PM-ში მომცერეთ ტელეფონი, წინასწარ დიდი მადლობა

Posted by: lala99 23 Dec 2010, 13:58
Kaifistka
კი გადავიდა სილურჯე 2kiss.gif

Posted by: Achmed the dead terrorist 23 Dec 2010, 18:47
გთხოვთ რამე მირჩიეთ რა...

რამოდენიმე დღის წინ ვიყავი თხილამურებზე სასრიალოდ, დიდი ხნის ფიზიკური პასიური პერიოდის მერე, ანუ ეგრევე ძალინ დიდი დატვირთვა მივიღე, წელი მტკივა(კუნთი) მის მერე,კატასტროფული ტკივილი არაა მაგრამ ძალიან მაწუხებს თითქმის ვერაფერს ვეღარ ვაკეთებ...
ძილის შემდეგ დილით აღარ მტკივა თითქმის მაგრამ როგორც კი სულ ოდნავ დავიტვირთავ თავს ეგრევე საშინლდ მტკვდება...სამი ოთხი დღეა ასე ვარ და ტკივილი არ გამდის.
ექიმთან ვერ მივდივარ გარკვეული მიზეზიბის გამო და რით ვუშველო სახლის [პირობებში ?

მადლობა წინასწარ.

Posted by: Sonishka 28 Dec 2010, 04:18
მოგესალმებით...

იქნებ დამეხმაროთ, ორი შეკითხვა მაქვს მედიკებთან:
1. მაინტერესებს, რას ნიშნავს რესპირატორული დისტრეს სინდრომი. ეს დიაგნოზი დაუსვეს მიოკარდიუმის ინფარქტის მქონე პაციენტს.
2. ამასთან, მიოკარდიუმის ინფარქტის ფონზე გამოხატული სუნთქვითი უკმარისობის, ბრონქოსპაზმისა და რესპირატორული დისტრეს სინდრომის მქონე პაციენტი აუცილებლად საჭიროებს თუ არა ხელოვნური სუნთქვის მართვის აპარატზე შეერთებას? პაციენტი ინტენსიური თერაპიის ბლოკში გადის მკურნალობას.

წინასწარ გმადლობთ.

Warning:
კითხვა დასვით კარდიოლოგის თემაში

Posted by: Luciferion 28 Dec 2010, 11:34
რა მაინტერესებს გერკულესი ხო არ ასუქებს? spy.gif

Posted by: Natuka NGN 3 Jan 2011, 00:48
თემა ძალიან გაიზარდა, ვხსნი ახლა თემას და გთხოვ თვექნი კითხვები იქ გაიმეოროთ.

* * *
aranormaluri
მიდი ექიმთან. ჯერ ერთი სხეულის ტემპერატურა დაბალია საკმაოდ. ზუსტად არ ვიცი, მაგრამ ექიმთან უნდა მიხვიდე და ფილტვები მოასმენინო.

Powered by Invision Power Board (http://www.invisionboard.com)
© Invision Power Services (http://www.invisionpower.com)